You are on page 1of 536

Krok 2 Stomatology 2008 1

1. A 3 year old child has a cavity in A. Pin stump inlay and a crown
the 84 tooth. Objectively: there is a cari- B. Pin tooth with a ring
ous cavity on the masticatory surface of C. Pin tooth with an inlay
the tooth within mantle dentin. Dentin is D. Pin tooth with protective bar
softened, probing of cavity walls is painful, E. Simplified pin tooth
percussion is painless. Make a diagnosis:
5. A 20 year old homeless and
A. Acute median caries unemployed patient complains about
B. Chronic superficial caries body temperature rise up to 39oC, pain
C. Acute superficial caries during eating and deglutition, nasal
D. Acute deep caries haemorrhages. He has been suffering
E. Chronic deep caries from this for 10 days. Objectively: herpetic
rash on the lips, irregular-shaped erosi-
2. A 45 year old patient complains ons covered with fibrinous deposit on
about pain induced by thermal stimuli, the mucous membrane of oral cavi-
spontaneous pain in the 26 tooth. A week ty; filmy deposits on the tonsils. Liver
ago this tooth was treated on account is enlarged and sclerotic. Blood count:
of pulpitis. Objectively: the 26 tooth is erythrocytes - 4, 5 · 1012 /l; hemoglobin
filled, percussion is painful, thermal sti- - 120 g/l; ESR - 25 mm/h; leukocytes -
muli induce long-lasting, slowly intensifyi- 10 · 109 /l; eosinophils - 0; rod nuclear
ng pain. X-ray picture shows that palatine cells - 2; segmentonuclear leukocytes - 31,
canal is filled by 2/3, no material can be lymphocytes - 41; monocytes - 10; atypical
seen in the buccal canals. What is the most mononuclears - 14%, plasmatic cells - 2.
probable cause of this complication? What is the most probable diagnosis?
A. Incomplete pulp extirpation A. Infectious mononucleosis
B. Inadequate canal filling B. Acute herpetic stomatitis
C. Infection C. Oropharyngal diphtheria
D. Inflammation in periodontium D. AIDS
E. Injury of periodontium tissue E. Acute leukosis
3. A 49 year old patient complains about 6. A 22 year old patient complained about
pain in the oral cavity induced by eating. colour change of the 11 tooth crown. The
He suffers from CHD. Objective exami- tooth was treated on account of chronic
nation revealed dyspnea, limb edema. deep caries a year ago. Immediately after
Oral cavity isn’t sanitated. On the mucous treatment the patient felt slight pain. She
membrane, on the right, there is an ulcer didn’t consult a dentist. X-ray picture
with irregular edges covered with greyish- shows broadening of periodontal fissure
white necrotic deposit with low-grade in the area of root apex of the 11 tooth.
inflammation around it. There is also Percussion is painless. What is the most
halitosis. What is the most probable di- probable diagnosis?
agnosis?
A. Chronic fibrous periodontitis
A. Trophic ulcer B. Chronic deep caries
B. Traumatic ulcer C. Chronic granulating periodontitis
C. Tuberculous ulcer D. Chronic granulematous periodontitis
D. Cancerous ulcer E. Chronic fibrous pulpitis
E. Ulcero-necrotic stomatitis
7. A 43 year old patient applied to a doctor
4. A 29 year old patient complains that the complaining about painful deglutition, li-
crown of his 26 tooth broke off. Objecti- mited mouth opening. 3 days ago he felt
vely: tooth decay index is 0,9, root ti- pain in the decayed 37 tooth, on the next
ssues are hard, roentgenography revealed day there appeared limitations of mouth
that the canal was filled to the top, no opening. Objectively: a slight soft swelli-
chronic inflammations of periodontium ng in the left submaxillary area. Palpation
were revealed. What construction should revealed an enlarged lymph node. Mouth
be used for prosthetics of the 26 tooth? can be opened up to 1,5 cm. There is
also an edema of the left palatine arch
and lateral pharynx wall. The 37 tooth is
decayed, mucous membrane around the
tooth is edematic, percussion is slightly
painful. What is the most probable di-
Krok 2 Stomatology 2008 2

agnosis? 11. A 42 year old patient complains of


pain in the submaxillary and sublingual
A. Odontogenous peripharyngeal areas that is getting worse during eating,
phlegmon body temperature rise up to 37, 6oC. He
B. Odontogenous pterygomaxillary has been suffering from this for 2 months.
phlegmon Objectively: infiltration along the right
C. Odontogenous submaxillary phlegmon sublingual torus, hyperemia, soft tissue
D. Peritonsillar abscess edema, acute pain during palpation. The
E. Acute submaxillary lymphadenitis duct of the right submandubular salivary
gland excretes turbid saliva mixed with
8. A 45 year old patient applied to pus. What is the most probable diagnosis?
a dental clinic for oral cavity sanati-
on. Anamnesis data: mild case of di- A. Exacerbation of salivolithiasis
abetes. Objectively: mucous membrane of B. Acute purulent lymphadenitis of
cheeks is unchanged, mainly in retromolar submaxillary area
area there are symmetrically placed C. Adenophlegmon of submaxillary area
whitish papulae protruding over the D. Abscess of maxillolingual groove
mucous membrane and forming a lace- E. Retention cyst of sublingual salivary
like pattern. On the upper jaw there are gland
two soldered bridge dentures, the 47 tooth
has an amalgam filling, the 46 tooth has a 12. A 38 year old patient came to a hospi-
steel crown. What is the most probable di- tal with complaints about a wound in the
agnosis? area of her left cheek. She was injured
16-18 hours ago, didn’t lose consciousness.
A. Lichen ruber planus Objectively: injury of skin, subcutaneous
B. Secondary syphilis fat and a muscle for about 3 cm long. The
C. Leukoplakia wound is bleeding. What initial surgical d-
D. Chronic atrophic candidosis bridement should be performed?
E. Lupus erythematosus
A. Early initial surgical d-bridement
9. A 38 year old patient was admitted B. Delayed surgical d-bridement
to the orthopaedic department after a C. Secondary surgical d-bridement
therapeutic treatment on account of peri- D. Initial surgical d-bridement together
odontitis. His anamnesis contains data with plasty
about frequent recurrences of this disease, E. -
acute attacks are accompanied by peri-
odontal abscesses. Objectively: the 42, 13. A 26 year old patient complains about
41,31, 32 teeth manifest mobility of I-II a sense of tooth heaviness and pain caused
degree, the 43 and 33 are stable, tooth by hot food stimuli, halitosis. Objecti-
cervices are dehisced by 1/4, hyperesthesia vely: crown of the 46 tooth is grey, there
of hard tooth tissues is not observed. What is a deep carious cavity communicating
rational construction of permanent splint with tooth cavity, superficial probing is
should be recommended? painless, deep one is painful, percussi-
on is painful, mucous membrane has no
A. Fullcast circle removable splint pathological changes. Make a provisional
B. Halfcrown splint diagnosis:
C. Mamlock’s splint
D. Crown-shell splint A. Chronic gangrenous pulpitis
E. Girder Kurliandky’s splint B. Chronic fibrous pulpitis
C. Acute condition of chronic periodontitis
10. A 52 year old patient complains D. Chronic concrementous pulpitis
of gum inflammation, tooth mobility. E. Chronic granulating periodontitis
Objectively: dentitions are intact, all teeth
have the 1-2 degree of mobility. What type 14. A 67 year old patient complains about
of teeth stabilization should be applied in frequently recurring erosion in the area
this case? of vermilion border of his lower lip. The
erosion is oval, 0,8х1,3 cm large, covered
A. Arch-directed stabilization with thin crusts. After their removal some
B. Frontal petechial haemorrhages on glossy surface
C. Parasagittal can be seen. There are also some atrophi-
D. Sagittal ed areas of the vermilion border. Infiltrati-
E. Frontosagittal on is absent. Submandibular lymph nodes
are not enlarged. What is your presumpti-
Krok 2 Stomatology 2008 3

ve diagnosis? vely: mucous membrane of lower lip is


hyperemic, excretory ducts of minor sali-
A. Abrasive precancerous Manganotti’s vary glands are dilated, "dew"symptom
cheilitis is present. What is the most probable di-
B. Erosive-ulcerous form of leukoplakia agnosis?
C. Keratoacanthosis
D. Bowen’s disease A. Cheilitis glandularis
E. Glandular cheilitis B. Meteorological cheilitis
C. Microbal cheilitis
15. A 51 year old woman complained D. Cheilitis exfoliativa
about food sticking in a tooth on the ri- E. Contact allergic cheilitis
ght of her lower jaw. Objectively: there
is a deep carious cavity on the distal- 19. A 54 year old woman complains about
masticatory surface of the 45 tooth consi- wear of the 35 and 36 teeth and pain
sting of compact pigmented dentin and caused by thermal and chemical stimuli.
not communicating with tooth cavity. The Objectively: crowns of the 35 and 36 teeth
patient was diagnosed with chronic deep are worn by 1/3 of their height (horizontal
caries. What diagnostic method allowed type), the 24, 25 26 teeth have fullcast
to eliminate chronic periodontitis? crowns. What crowns should be made for
the 35 and 36 teeth?
A. Electro-odontometry
B. Probing A. Fullcast
C. Palpation of root apex projection B. Combined Belkin’s
D. Percussion C. Metal stamped
E. Cold probe D. Plastic
E. Metal-ceramic
16. A 58 year old patient complains about
clicking in the mandibulotemporal joint 20. A virtually healthy 9 year old chi-
during eating. Periodically as a result of ld complains about crown fracture and
wide mouth opening the mandible takes pain in the right superior frontal tooth.
up such a position that makes mouth closi- Objectively: crown part of the 11 tooth is
ng impossible. The mandible can be easily broken by 1/3, pulp is pointwise dehisced,
placed into its initial position by pressing it is red, acutely painful and bleeds duri-
with fingers upon its coronoid processes. ng probing. Percussion is slightly painful.
What is your presumptive diagnosis? The child got a trauma several hours ago.
Choose an optimal treatment method:
A. Habitual dislocation of mandible
B. Acute dislocation of mandible A. Biological method
C. Subluxation of mandible B. Vital amputation
D. Anterior dislocation C. Devital amputation
E. Posterior dislocation D. Vital extirpation
E. Devital extirpation
17. An 11 year old girl complains about gi-
ngival haemorrhage during tooth brushi- 21. Objective examination of a patient
ng and eating. She has been sufferi- revealed hard tissue defect of the 24 tooth.
ng from this for a year. Gum of both Tooth decay index is 0,8. The defect is
upper and lower jaws is edematic and partly restored by a filling that doesn’t
congestively hyperemic. Hygienic state of meet the requirements. The tooth has
oral cavity is unsatisfactory. Bite is edge- pink colouring. X-ray picture shows no
to-edge. Roentgenological examination pathological processes. What constructi-
of periodontium revealed no pathologi- on is indicated in this case?
cal changes. What is the provisional di-
agnosis? A. Pin construction
B. Artificial crown
A. Chronic catarrhal gingivitis C. Inlay
B. Acute catarrhal gingivitis D. Halfcrown
C. Generalized periodontitis E. Equator crown
D. Localized periodontitis
E. Hypertrophic gingivitis 22. A 49 year old patient applied to
a dentist complaining about a growing
18. A 13 year old child complained about mole, its peeling and itching. According to
painfulness of his lower lip. He has the patient, the part of skin started changi-
been suffering for two months. Objecti- ng its colour and size 1 year ago, after an
Krok 2 Stomatology 2008 4

injury got during shaving. Objectively: in broken jaw with cartilage of nose
the infraorbital area on the left there is an and eyeballs, nasal haemorrhage, ti-
intensely pigmented brown spot, up to 2 ssue edema, "glasses"symptom; palpati-
cm large, with small nodules on its surface; on reveals crepitation, subcutaneous
it is oval, bulging, with signs of peeling, emphysema, liquorrhea, loss of sensitivi-
palpatory painless. Regional lymph nodes ty in the area of half the upper jaw, nose
are enlarged, adhering to skin, painless. wing and frontal teeth. What is the most
Make a provisional diagnosis: probable diagnosis?
A. Melanoma A. Le Fort’s III fracture of upper jaw
B. Pigmented nevus B. Upper jaw ostemia
C. Verrucous nevus C. Upper jaw odontoma
D. Squamous cell carcinoma of skin D. Le Fort’s I fracture of upper jaw
E. Papillomatosis E. Upper jaw cancer on the right
23. A patient applied to a dentist complai- 26. An 8 year old child has a carious cavi-
ning about inability to close her mouth, ty on the masticatory surface of the 16
laboured speech. Objectively: oral cavity tooth within circumpulpar dentin. Probi-
is half-open with saliva outpouring from ng of cavity floor is painful, dentin is
it, central line is deviated to the right. softened, slightly pigmented. Cold sti-
There is a cavity in front of antilobium; mulus causes short-term pain. Choose a
below the malar arch there is a protrusion dental treatment paste:
of mandible head into the infratemporal
fossa. What is the most probable di- A. Hydroxycalcium-containing
agnosis? B. Resorcin-formaline
C. Paraformaldehyde
A. Unilateral anterior dislocation of D. Arsenious
mandible on the left E. Thymol
B. Unilateral anterior dislocation of
mandible on the right 27. A 24 year old patient applied to a
C. Unilateral posterior dislocation of dental clinic for root removal of her upper
mandible on the left wisdom tooth. Tuberal anesthesia caused
D. Unilateral posterior dislocation of a postinjection haematoma. What was
mandible on the right injured during anesthetization?
E. Bilateral anterior dislocation
A. Pterygoid venous plexus
24. A 36 year old man complains about B. Maxillary artery
acute headache, body temperature rise up C. Infraorbital artery
to 39, 1o C, indisposition. Objectively: a sli- D. Zygomatic artery
ght face asymmetry because of soft tissue E. Palatine artery
edema of the left infraorbital area. Crown
of the 26 tooth is partly decayed. Percussi- 28. A 5 year old child has temperature rise
on is acutely painful. Mucous membrane up to 39, 2oC, sore throat, nausea. Objecti-
on the vestibular side in the area of vely: mucous membrane of soft palate and
the 25, 26 teeth is edematic, hyperemic. palatine arches is brightly hyperemic, it
Breathing through the left part of nose is can be distinctly distinguished among the
laboured, there are purulent discharges. surrounding tissues. The tongue is dry,
X-ray picture showed a homogeneous edematic, bright-red, its lateral surfaces
shadow of the left part of maxillary sinus. have no fur on them, fungiform papi-
What is the most probable diagnosis? llae are evidently enlarged. Face skin
is hyperemic apart of pale nasolabi-
A. Acute purulent odontogenous maxillary al trigone, is covered with spotty rash.
sinusitis Submaxillary lymph nodes are palpatory
B. Acute condition of chronic periodontitis painful. What is the causative agent of this
of the 26 tooth disease?
C. Acute periostitis of upper jaw
D. Suppuration of maxillary cyst A. Hemolytic streptococcus
E. Acute odontogenous osteomyelitis B. Coxsackie virus
C. Herpes virus
25. A 38 year old patient got a blow D. Bordet-Gengou bacillus
that resulted in upper jaw fracture. E. Loeffler’s bacillus
Objectively: flattening and impressi-
on of face, mobility and dangling of 29. A 65 year old patient complains about
Krok 2 Stomatology 2008 5

pain during eating, reduced tongue mobi- 32. A 31 year old man complains about
lity, an ulcer in the posterior part of mouth dryness, burning of tongue dorsum that
floor on the left, weight loss. Objecti- appeared for about a week ago and is
vely: the patient uses a partial removable getting worse during eating stimulati-
denture for the lower jaw. In the area ng food. Some time ago the patient
of the left mylohyoid duct there is an had pneumonia. He spent two weeks
ulcer in form of a cleft up to 1,6 cm at a hospital, was taking antibiotics. He
long with everted edges, covered with doesn’t take any drugs at the moment.
grey-yellowish deposit, closely adheri- Objectively: mucous membrane of oral
ng to an infiltrate that can be detected cavity is hyperemic, dry and glossy. On the
during bimanual palpation. In the left tongue dorsum and palate some greyish-
submandibular and superolateral areas of white films are present that can be easi-
neck several enlarged nonmobile lymph ly removed. Threads of saliva follow the
nodes can be palpated. What is the most spatula. What is the most probable provi-
probable diagnosis? sional diagnosis?
A. Cancer of mucous membrane of mouth A. Acute pseudomembranous candidosis
floor B. Chronic hyperplastic candidosis
B. Tuberculous ulcer of mouth floor C. Acute atrophic candidosis
C. Decubital ulcer of mouth floor D. Drug-induced stomatitis
D. Tertiary syphilis (gummatous ulcer) E. Chronic atrophic candidosis
E. Actinomycosis of mouth floor
33. A 10 year old boy applied to a dentist
30. A 7 month old child was brought to and complained about pain in his palate
a dentist because of an ulcer in the oral during eating. Objectively: the lower third
cavity. The child was born prematurely. of his face is shortened, mouth opening is
She has been fed with breast milk substi- not reduced. During joining of teeth the
tutes by means of a bottle with rubber ni- cutting edge of inferior incisors contacts
pple. Objectively: on the border between with mucous membrane of palate. On
hard and soft palate there is an oval ulcer the site of contact mucous membrane is
0,8х1,0 cm large covered with yellowish- hyperemic and slightly edematic. Lateral
grey deposit and surrounded with a roll- teeth have 1 class joining (according to
like infiltration. Make a provisional di- Angle’s classification). Propose a rational
agnosis: plan of treatment of the patient’s lower
jaw:
A. Bednar’s aphtha
B. Setton’s aphtha A. To "knock in"the frontal part
C. Tuberculous ulcer B. To "knock in"the lateral parts
D. Acute herpetic stomatitis C. To broaden lower jaw
E. Acute candidous stomatitis D. To lengthen the frontal part
E. To lengthen the lateral parts
31. A 16 year old patient complains
of a cosmetic defect in the area of his 34. A 32 year old patient applied to a
upper frontal teeth in form of white spots dental surgeon for oral cavity sanitation
that were revealed long ago and haven’t before prosthetics. During examination
changed since that. Objectively: there are of oral cavity the dentist revealed that
white spots on the vestibular surfaces of crown of the 35 tooth was decayed. The
the 11, 12, 21, 22 teeth by the cutting root is stable, its percussion is painless.
edge and on the vestibular surfaces of the Mucous membrane of alveolar process
16, 26, 36, 46 teeth close by the masti- was unchanged. X-ray picture showed a
catory surface. Probing showed that the slight broadening of periodontal fissure.
spot surface was smooth, painless; reacti- What is your presumptive diagnosis?
on to the cold stimulus was painless. The
spots couldn’t be stained by 2% soluti- A. Chronic fibrous periodontitis of the 25
on of methylene blue. What is the most tooth
probable diagnosis? B. Chronic periodontitis of the 25 tooth
C. Chronic granulomatous periodontitis of
A. Systemic enamel hypoplasia the 25 tooth
B. Local enamel hypoplasia D. Chronic granulating periodontitis of the
C. Acute initial caries 25 tooth
D. Fluorosis, spotty form E. Cystogranuloma
E. Erosion of hard tooth tissues
Krok 2 Stomatology 2008 6

35. A patient has indication for removal of A. Acute arsenous periodontitis


his medial incisor of the right upper jaw B. Acute infectious periodontitis
on account of chronic periodontitis. What C. Pulpitis complicated by focal peri-
types of anaesthesia should be applied for odontitis
tooth removal? D. Acute condition of chronic periodontitis
E. Acute purulent pulpitis
A. Infraborbital and incisive
B. Infraorbital and palatine 39. An 8 year old boy was referred to the
C. Terminal and incisive oral surgery for extraction of his 64 tooth
D. Infraorbital, palatine and incisive because of acute condition of chronic peri-
E. Plexual and incisive odontitis. Tooth crown is intact. What
instrument should be applied?
36. A 45 year old man complains about li-
quid outpouring from his nose, inability A. S-shaped forceps without thorns
to blow his nose, inflated cheeks. Objecti- B. Beak-shaped forceps with non-
vely: there is a perforating defect (1х1,5 converging beaks
cm) of alveolar process at a level of the C. Straight forceps
extracted 26th tooth in the lateral part D. Broad-beaked forceps
of his upper jaw. Air inhalation through E. S-shaped forceps with thorns
the nose with held nostrils is accompani- 40. A 43 year old man applied to the
ed by generation of bubbles in the area dental clinic for tooth prosthetics. Objecti-
of perforation. What denture constructi-
vely: the crown of the 37th tooth is
on should be recommended? decayed by 2/3, buccal and lingual walls
A. Lesser saddle denture with clasps’ are thin. Occlusion picture shows strong
fixation contact with antagonists. How thick
B. Clasp denture with obturating part should be the layer of tooth surface that
C. Common partial removable denture must be ground off during preparation for
D. Common dental bridge metallic stamped crown?
E. Protective palatal bars A. 0,28-0,3 mm
37. Preventive examination of an 8 year B. 0,1-0,2 mm
old boy revealed matted chalky spots on C. 0,5-0,6 mm
the vestibular surface of the 11 and 21 D. 0,6-0,7 mm
teeth localized in the precervical area. The E. 0,7-0,8 mm
child has no subjective complaints. What
41. A 56 year old man complains of
is the most probable diagnosis? pain in the left parotidomasticatory area,
A. Acute initial caries progressing face asymmetry that was noti-
B. Spotty fluorosis ced a month ago. Objectively: left-sided
C. Local enamel hypoplasia paresis of mimic muscles. To the fore of
D. Acute superficial caries earflap there is an ill-defined infiltration,
E. Chronic initial caries the skin above it is tense and cyanotic;
left lymph nodes are enlarged. Opening of
38. A patient complains about pain in mouth is limited down to 2,5 cm. The left
the 51 tooth that is getting worse duri- parotid duct doesn’t excrete saliva. What
ng cutting. Anamnesis data: the patient is the most probable diagnosis?
underwent treatment on account of pulpi-
tis of the 51 tooth, the tooth was treated A. Adenocarcinoma
with devitalizing paste, the patient didn’t B. Mixed tumour
come to see a dentist for the second time. C. Cyst of the gland
Objectively: carious cavity of the 51 tooth D. Glandular tuberculosis
is closed by dentin layer. Percussion is pai- E. Chronic lymphadenitis
nful. Mucous membrane in the root apex 42. A 55 year old woman complains of
projection of the 51 tooth is hyperemic, ulcers on the mucous membrane of oral
edematic, palpatory painful. Make a di-
cavity, pain during eating and talking. She
agnosis: fell abruptly ill over a month ago. Objecti-
vely: unchanged mucous membrane of
her gums, soft palate and palatine arches
has big bright-red erosions on it. Intact
mucous membrane peels easily off when
slightly rubbed and this results in erosi-
Krok 2 Stomatology 2008 7

ons and small haemorrhages. What is the A. Circumpulpar pin inlay


leading diagnostic symptom in the di- B. Metal-ceramic crown
fferential disease diagnostics? C. Halfcrown
D. Ceramic crown
A. Tzanck cells in the impression smears E. Metal crown
B. Positive Nikolsky’s symptom
C. Presence of blisters in the oral cavity 46. A teenager applied to an orthodonti-
D. Presence of Wickham’s striae st complaining about tooth malposition.
E. Presence of phagocytes in the impressi- Objectively: the face is without peculari-
on smears ties. Occlusion of permanent teeth is
present. There are no abnormalities of jaw
43. A 47 year old patient complains of correlation in three planes. The 23 tooth is
permanent pain in the 27 tooth that is vestibularly over the occlusive plane; the
getting worse during cutting. Objectively: space in the dental arch is less than 1/3
the patient’s face is symmetric, skin is of of crown size. How is it possible to make
normal colouring, mouth opening is not room for the malpositioned 23 tooth?
limited, mucous membrane of alveolar
process is edematic and hyperemic at a A. To remove the 24 tooth
level with the 27 tooth. The 27 tooth has B. To enlarge transversal jaw dimensions
a deep carious cavity communicating with C. To enlarge sagittal jaw dimensions
pulp chamber. Percussion of the 27 tooth D. To remove the 23 tooth
causes acute pain. What is presumptive E. To enlarge vertical dimensions
diagnosis?
47. A patient complains about acute
A. Acute condition of chronic periodonti- intense pain and a slight swelling in the
tis of the 27 tooth area of a decayed tooth of the lower jaw
B. Chronic periodontitis of the 27 tooth on the right, ill health, body temperature
C. Acute general purulent pulpitis of the rise up to 38, 3oC, lack of appetite,
27 tooth insomnia. Objectively: there is collateral
D. Acute purulent periostitis of the upper edema of soft tissues of submandibular
jaw beginning from the 27 tooth area and lower part of right cheek. Regi-
E. Chronic left-sided odontogenous maxi- onal lymph nodes are enlarged on the
llary sinusitis right, palpatory painful. Crown of the
46 tooth is half decayed, the 45, 46, 47
44. A 68 year old patient has a defect in teeth are mobile, there is also a cuff infi-
the lower third of his nose as a result of ltrate in the area of these teeth. Vincent’s
malignant tumour removal. Objectively: symptom is positive on the right. Make a
dorsum of nose up to the middle third diagnosis:
and external nostril outlines are intact.
Skin at the base of nose is thinned and A. Acute odontogenous osteomyelitis
erosive. The patient strongly objects to B. Acute purulent odontogenuos periosti-
any plastic operation. What method of fi- tis
xation of nose ectoprosthesis is the most C. Acute purulent periodontitis
acceptable? D. Acute condition of chronic
odontogenous osteomyelitis
A. On the glasses frame E. Chronic odontogenous osteomyelitis
B. By means of lace
C. Transparent adhesive tape 48. A 16 year old patient complained
D. Anatomic retention about discomfort in the area of her upper
E. Head cap jaw teeth she has been feeling for 2 weeks.
Examination of precervical area of the 11
45. A 12 year old girl complained about and 12 teeth revealed whitish matt spots
a crown defect in the frontal part of her with indistinct outlines that absorb dyes
upper jaw. Anamnesis data: the tooth intensively. What treatment of the 11 and
was filled more than once but the filli- 12 teeth should be administered?
ngs fell out. Objectively: the 12 tooth is
filled. There is IV class defect accordi- A. Remineralizing therapy
ng to Black’s classification. Devitalization B. Preparation and filling
has never been performed, percussion of C. Spot removal
the 12 tooth is painless. What orthopaedic D. Antiseptic treatment
construction should be applied in this E. Silver impregnation
case?
49. A 48 year old patient complained
Krok 2 Stomatology 2008 8

about having pain in the 45 tooth duri- Objectively: a painful elastic infiltration in
ng cutting for a year. The 45 was treated the left submaxillary area. Mouth openi-
before. Objectively: mucous membrane ng is not limited. Bimanual palpation in
in the area of this tooth is hyperemic the area of mylohyoid groove revealed
and slightly cyanotic. The 45 tooth is pi- a compact movable oblong induration.
nk, the filling fell out. What examinati- Mucous membrane is unchanged. Duct
on method should be applied in order to of the left submandibular gland doesn’t
choose treatment? excrete saliva. What is the most probable
diagnosis?
A. Roentgenography
B. Thermometry A. Salivolithiasis
C. Gum palpation B. Chronic lymphadenitis
D. Probing C. Pleomorphic adenoma
E. Electric odontodiagnostics D. Retention cyst
E. Submaxillary lipoma
50. A 40 year old patient with mandi-
ble fracture applied to a doctor 3 weeks 53. A 5 year old child was diagnosed wi-
after immobilization of breaks because of th congenital complete nonclosure of soft
pain and body temperature rise. Objecti- and hard palate. What type of anaesthesia
vely: a slight swelling in the chin area, is indicated for uranostaphyloplasty?
mucous membrane of alveolar process in
the area of the 2 1 | 1 2 teeth is hyperemic, A. Nasotracheal narcosis
edematic, palpatory painful. Overtooth B. Mask narcosis
splint on 5 4 3 2 1 | 1 2 3 4 5 teeth is C. Intravenous narcosis
in satisfactory condition, no occlusion D. Orotracheal narcosis
abnormalities were detected. The patient E. Endotracheal narcosis through
was diagnosed with acute purulent peri- tracheostome
ostitis of mandible. What surgical action is 54. A 23 year old woman came to a dental
indicated? clinic for restoration of the 11th tooth’s
A. Lancing of abscess to the bone crown. Objectively: root of the 11th tooth
B. Intraoral novocaine block is at a level with gingival edge, its walls
C. Removal and replacement of the are thick enough. A dentist made and
overtooth splint by a new one adjusted a stump inlay upon which a
D. Trepanation of the 2 1 and 1 2 teeth metal-plastic crown will be fixed. What
E. Supervision of patient plastic will be used for veneering?

51. A 43 year old patient complains of a A. Sinma-M


neoplasm in the right submaxillary area B. Acryloxide
that appeared a month ago after angi- C. Carbodent
na. Body temperature is 37, 0 − 37, 2oC. D. Noracryl
The patient underwent anti-inflammatory E. Protacryl
therapy but the neoplasm didn’t dimi- 55. A 35 year old patient needs a metal
nish. Objectively: palpation of the right inlay for the 37 tooth. Objectively: masti-
submaxillary area reveals a slightly pai- catory surface of the 37 tooth has a carious
nful spherical neoplasm of dense elastic cavity. What is the pecularity of preparati-
consistency with regular outlines that on?
is not adherent to skin. The duct of
submandibular salivary gland excretes A. Making a bevel
transparent saliva. Sublingual torus is B. Making an auxiliary shelf
unchanged. What is the most probable di- C. Broadening of cavity floor
agnosis? D. Making a flat floor
E. Making an auxiliary cavity
A. Chronic lymphadenitis
B. Chronic sialoadenitis 56. It is planned to make a metal-ceramic
C. Salivolithiasis crown supported by stump inlay of the
D. Salivary adenoma 23 tooth. Objectively: the crown of the
E. Atheroma 23 tooth is decayed down to the gingival
edge. Root canal is filled to the top. The
52. A 37 year old man complained about dentist made a wax stump model with a
pain and a slight swelling emerging duri- pin, cast it in metal, fitted it to the tooth,
ng eating in the left submaxillary area. fixed it by means of visphat-cement and
Krok 2 Stomatology 2008 9

got a working plaster impression. At what coalveolar crest, gaseous crepitation in the
stage did he make an error? right infraorbital area; percussion reveals
a "bursted nut"symptom. What is the
A. Impression taking most probable diagnosis?
B. Wax construction making
C. Casting A. Fracture of the right zygomatic bone
D. Stump fitting B. Le Fort’s I fracture of upper jaw (inferi-
E. Stump fixing or)
C. Le Fort’s II fracture of upper jaw
57. A 23 year old military servant needs D. Le Fort’s III fracture of upper jaw
orthopaedic treatment in a specialized (superior)
hospital. He was diagnosed with false joint E. Fracture of nose bones
of mandible in its frontal part. The teeth
are intact, stable, in threes on each side. 60. A patient is 48 year old, according
Orthopaedic treatment by means of a bri- to the results of clinicoroentgenological
dge denture will be possible only if the jaw examination it is indicated to remove
defect is no more than: the 26 tooth because of acute condition
of chronic granulomatous periodontitis.
A. 1 cm What conduction anesthesia is indicated
B. 2 cm for this operation?
C. 3 cm
D. 3,5 cm A. Tuberal and palatinal
E. 4 cm B. Torus
C. Infraorbital and incisive
58. A 17 year old girl applied to a dental D. Plexus
clinic and complained about hard tissue E. Infraorbital and palatinal
defects on her frontal and lateral teeth.
Subjectively these defects don’t cause any 61. A girl is 1,2 year old. Vestibular surface
inconvenience. Crown defects appeared of her 52, 51, 61, 62 teeth has large carious
long ago. The patient was born and and cavities within the enamel. Probing is sli-
has been living in an area where fluori- ghtly painful, percussion of the 52, 51, 61,
ne concentration in the drinking water 62 teeth is painless. What treatment is to
makes up 1,2 mg/l. Objectively: on the be administered?
vestibular surfaces of incisors on both
upper and lower jaws in the equator area A. Silver impregnation
there are hard tissue defects within deep B. Filling with phosphoric acid cement
layers of enamel. The defects are parallel C. Remineralizing therapy
to the cutting edge. The same defects were D. Coating with fluorine lacquer
revealed in the area of tubera of the fi- E. Amalgam filling
rst molars, floor and walls of the defects
are smooth. Enamel of the defect floor is 62. A 2 year old girl has body temperature
light-brown. What is the most probable 38, 5oC, a swelling below her jaw on the
diagnosis? right. On the 5th day of illness there
apeared rhinitis, cough, a small movable
A. Systemic hypoplasia globule under her lower jaw on the ri-
B. Local hypoplasia ght. Objectively: general condition of the
C. Focal odontodysplasia child is moderately severe. The face is
D. Endemic fluorosis asymmetric due to the swelling in the right
E. Erosion of hard tissues of tooth submaxillary area. The skin is hyperemic,
glossy, there is a diffuse infiltrate in the
59. A 19 year old patient came to right submaxillary area spreading to the
traumatology centre and complai- upper neck parts on the right, it is dense
ned about face asymmetry, right-sided and painful; the skin doesn’t make folds.
paresthesia of his upper lip, nasal Teeth are healthy. What is the most
haemorrhage. Objectively: evident face probable diagnosis?
asymmetry due to an edema and
haematoma of the right inferior eyelid
and infraorbital area. Opening of mouth
is slightly limited, occlusion is normal.
Palpation reveals a symptom of "step"in
the area of the right zygomaticomaxillary
suture, deformity in the area of external
edge of the right orbit and zygomati-
Krok 2 Stomatology 2008 10

A. Adenophlegmon of the right submaxi- roots is intact, there are no roentgenologi-


llary area cal changes in this area. What is the most
B. Acute nonodontogenic submandibular probable diagnosis?
lymphadenitis on the right
C. Acute purulent periostitis of mandible A. Acute diffuse pulpitis
on the right B. Chronic fibrous pulpitis
D. Acute sialoadenitis of the right C. Chronic gangrenous pulpitis
submandibular salivary gland D. Exacerbation of chronic periodontitis
E. Chronic osteomyelitis of mandible on E. Acute deep caries
the right
66. A 12 year old boy complains about
63. A 36 year old woman complains of lip pain and swelling in the parotidomasti-
dryness and peeling lasting for a month. catory area on the left, body temperature
Application of indifferent ointments was rise up to 37, 5oC. He has been suffering
ineffective. Objectively: vermilion border from this for 5 years. Objectively: palpati-
of her lower lip is bright-red, moderately on reveals a dense painful nonmobi-
infiltrated, covered with adherent whitish- le formation 3,5х5 large in the paroti-
grey squamae, their removal causes pain domasticatory area on the left. Skin
and haemorrhage. On the focus periphery colour is unchanged. Orifice of the left
there is epithelium opacity in form of whi- salivary gland duct excretes transparent
te striae, in the centre of it there is a sinki- secretion. What is the most probable di-
ng down area. What is the most probable agnosis?
diagnosis?
A. Herzenberg’s pseudoparotitis
A. Lupus erythematosus B. Mixed tumour of parotid gland
B. Candidal cheilitis C. Epidemic parotitis
C. Lichen ruber planus D. Acute condition of chronic
D. Leukoplakia parenchymatous parotitis
E. Exfoliative cheilitis E. Buccal abscess
64. A patient complained about pai- 67. A 55 year old patient has a painless,
nful deglutition, difficult mouth openi- tuberous, cyanotic pedunculated formati-
ng. Several days ago the 47 tooth was on 2х1х1,5 cm large that appeared on the
removed because of acute condition of spot of the removed 46th tooth. Opening
chronic periodontitis. The patient’s condi- of mouth is not limited. Intra-oral X-ray
tion kept worsening. Body temperature is picture of alveolar process in the area of
37, 9oC. Results of external examination: the removed 46th tooth shows a focus of
the face is symmetric, face skin is slightly bone tissue destruction. What is the most
pale. Right submandibular lymph nodes probable diagnosis?
are enlarged, palpatory painful. Exami-
nation of oral cavity is impossible because A. Giant-cell epulis
of evident contracture of lower jaw (the B. Hard odontoma of lower jaw
mouth opens up to 0,5 cm between central C. Hypertrophic gingivitis
incisors). What anesthesia will provide D. Papilloma of mucous membrane in the
mouth opening? area of the removed 46th tooth
E. Ameloblastoma of the lower jaw
A. Bersche-Dubov’s anesthesia
B. Plexus 68. A 40 year old woman complains
C. Torus about slight painfulness of gums, high
D. Block of upper cervical plexus temperature sensitivity of teeth. Objecti-
E. Mandibular anesthesia vely: the gums are pale and compact,
in the area of lower frontal teeth there
65. A 7 year old child complains of is retraction with cervix dehiscence. X-
spontaneous pain in the upper right molar ray picture shows steady decrease of
teeth. Examination of medial contact interdental septa height down to 1/3
and masticatory surfaces of the 55 tooth of root length. What disease are these
revealed a carious cavity composed of symptoms typical for?
softened light dentin and localized wi-
thin circumpulpar dentin. Floor probing
is acutely painful, tooth percussion is sli-
ghtly painful. Mucous membrane of the
alveolar process in projection of 55 tooth
Krok 2 Stomatology 2008 11

A. Parodontosis is palpatory unchanged. What is the most


B. Generalized periodontitis probable diagnosis?
C. Gingivitis
D. Localized periodontitis A. Arthrosis of the right temporomandi-
E. Papillitis bular joint
B. Acute arthritis of the right
69. A soldier injured by a shell splinter temporomandibular joint
was diagnosed with gunshot fracture of C. Neuromuscular syndrome
mandible accompanied by an over 3 cm D. Occlusive articulation syndrome
long bone defect in the chin area. What E. Subluxation of mandible
method of fixation of mandible fragments
is indicated? 72. During opening a phlegmon of mouth
floor a doctor revealed greyish necrotic
A. Machine osteosynthesis (Rudko, masses in purulent foci, gas vesicles and
Bernadsky) fat droplets, sharp unpleasant smell of
B. Tigerstedt’s splints exudate. The tissues are of dark-brown
C. Direct osteosynthesis colour, muscles resemble of boiled meat.
D. Gunning-Port’s splint What medications should be administered
E. Intermandibular Ivy ligature in order to prevent further spreading of
this process?
70. A 43 year old patient complained
about mobility of his 24, 26, 27 teeth, A. Polyvalent antigangrenous serum
pus excretion from alveolus of the the B. Challenging dose of broad spectrum
extracted 25 tooth. 1,5 month ago di- antibiotics
ssection along the mucogingival fold C. Glucocorticoid medications
was performed and the 25 tooth was D. Hyposensitizing medications
extracted. Objectively: there is a sli- E. Immunomodulators
ght swelling of soft tissues in the right
infraorbital area, lymph nodes of the ri- 73. A 43 year old patient complai-
ght submaxillary area are enlarged, sli- ned about mobility of lower jaw teeth.
ghtly painful, nasal breathing is normal. Objectively: the dentition is intact. Tooth
Mucous membrane of alveolar process in mobility is of I-II degree. It is planned to
the area of the 24, 26, 27 teeth is edematic immobilize teeth by means of a removable
and cyanotic. There is also a fistula with splint common for the whole dentition.
bulging granulations along the mucogi- What stabilization will be provided by
ngival fold. Alveolus of the extracted means of this splint?
25 tooth excretes purulent granulations.
What disease does this clinical presentati- A. Circle
on correspond with? B. Frontal
C. Frontal-lateral
A. Chronic localized osteomyelitis D. Sagittal
B. Acute osteomyelitis E. Transversal
C. Acute condition of localized periodonti-
tis 74. A patient complains about long-lasting
D. Acute condition of chronic maxillary pain attacks in the lower jaw teeth, on the
sinusitis left. The pain irradiates to the ear, occiput
E. Chronic alveolitis and is getting worse during eating cold
and hot food. Objectively: there is a deep
71. A 57 year old patient came to an carious cavity on the approximal-medial
orthopaedic stomatology center with surface of the 36 tooth. Floor probing is
complaints about dull pain in the area of overall painful and induces a pain attack.
his right temporomandibular joint that What is the most probable diagnosis?
is getting worse during eating. The di-
sease developed gradually, it began wi- A. Acute diffuse pulpitis
th constrained mandibular motion in B. Acute local pulpitis
the morning that sometimes grew more C. Acute purulent pulpitis
intensive, sometimes less. Objectively: D. Chronic concrementous pulpitis
the face is symmetric, the mouth can be E. Acute deep caries
opened up to 3 cm. Opening of mouth is 75. A 35 year old woman complains about
accompanied by articular noise and cli- periodical appearance of small ulcers in
cking (step-like dislocation of mandible). the oral cavity. She has been suffering
Skin above the joint is intact. Muscle tone from this for 5 years, recurrences happen
Krok 2 Stomatology 2008 12

4-5 times a year. The ulcer healing lasts months. Objectively: on the masticatory
for 10 days. Objectively: on a mucous surface of the 37 tooth there is a carious
membrane of lower lip there is a roundi- cavity with overhanging enamel edges fi-
sh lesion element 0,5 cm large covered lled with circumpulpar dentin. The cavity
with white deposit and surrounded by is filled with light softened dentin. Probi-
hyperemia border, very painful when ng of the cavity floor is somewhat painful.
touched. What is the most probable di- Cold stimuli cause short-term pain. Make
agnosis? a provisional diagnosis:
A. Chronic recurrent aphthous stomatitis A. Acute deep caries
B. Traumatic erosion B. Chronic fibrous pulpitis
C. Secondary syphilis C. Acute median caries
D. Chronic recurrent herpes D. Chronic median caries
E. Duhring’s herpetiform dermatitis E. Chronic deep caries
76. A 38 year old driver complains 79. A 65 year old patient complains of
of acute pain in his mouth and sore pain in the area of mucous membrane of
throat, difficult ingestion, indisposition, hard palate on the left that is getting worse
temperature rise up to 38, 6o C. These during eating with use of a complete
symptoms appeared after exposure to removable denture. He has been suffering
cold. He has been ill for a day, in the from this for 1,5 month. Objectively: left-
evening he has to go to work. Objectively: sided hyperemia and edema of mucous
gums in the area of inferior frontal teeth membrane of hard palate; at the border
as well as pharynx mucous membrane of distal denture edge there is an ulcer
are hyperemic, edematic; gingival edge with dense walls and fundus, surroundi-
is necrotic, tonsills are enlarged. Results ng tissues are infiltrated. The ulcer floor is
of bacterioscopy: fusospirochetal symbi- tuberous, covered with fibrinous deposit;
osis. In blood: erythrocytes - 4, 5 · 1012 /l; ulcer palpation is painful. What exami-
leukocytes - 7, 2 · 109 /l; ESR - 18 mm/h. nation method is to be applied in the first
What actions should a stomatologist take? place?

A. To prescribe a medication and give a A. Biopsy


sick-list B. Bacterioscopy
B. To prescribe a medication and allow to C. Cytology
go to work D. Allergic contact plastic test
C. To give him an order for hospitalization E. Serological reactions
in oral department
D. To refer him to an infectious disease 80. A 46 year old patient complains about
specialist pain and bleeding from the carious cavi-
E. To refer him to an otolaryngologist ty of her 27 tooth during eating. Previ-
ously she had spontaneous pain. Exami-
77. A 25 year old man complains about nation of the 27 tooth revealed a deep
itching and reddening of his skin in the carious cavity on the masticatory surface
buccal area, general weakness, flaccidi- consisting of red tissue, probing induced
ty. He associates the begin of disease pain and haemorrhage. What treatment
with a skin injury he had got during method should be chosen?
shaving. Objectively: body temperature is
39, 0oC. In the buccal area a part of skin is A. Vital extirpation
hyperemic, slightly bulging, well-defined. B. Devital extirpation
Hyperemic surface has some vesicles wi- C. Devital amputation
th serous fluid. What is the most probable D. Vital amputation
diagnosis? E. Biological method

A. Erysipelatous inflammation of face 81. A 40 year old patient complains about


B. Buccal phlegmon a carious cavity in the 22 tooth. Objecti-
C. Furuncle vely: a deep carious cavity on the medial
D. Anthrax surface of the 22 tooth, probing induces
E. Streptococcal impetigo mild pain. What is the optimal material
for filling of the 22 tooth?
78. A 13 year old child has been sufferi-
ng from pain in the left inferior molar
induced by cold stimuli for several
Krok 2 Stomatology 2008 13

A. Composite light-setting material is 2 degree tooth mobility. Dentogingival


B. Silica-alumina cement pockets are 4-5 mm deep. External exami-
C. Glass-ionomer light-setting cement nation revealed dryness and thickening
D. Glass-ionomer chemical-setting cement of palms, anterior third of forearms, sole
E. Silicophosphate cement surfaces; there are scratches. What is the
most probable provisional diagnosis?
82. A 28 year old man applied to a dental
surgeon for removal of the 38 tooth. What A. Papillon-Lefevre syndrome
forceps should be chosen for this tooth? B. Gaucher’s disease
C. Letterer-Siewe disease
A. Beak-shaped curved forceps D. Hand-Schueller-Christian disease
B. Broad-beaked forceps (with non- E. Niemann-Pick disease
converging beaks)
C. Beak-shaped forceps with converging 86. Examination of a patient revealed
beaks a roundish neoplasm in the submental
D. Beak-shaped forceps with thorns area. The skin above it is unchanged,
E. Root bayonets forms a fold. Puncture sample contai-
ns some straw-yellow liquid mixed with
83. A 35 year old patient applied to a cholesterol. What is the most probable di-
dental clinic for removal of the 14 tooth agnosis?
because of acute condition of chronic
periodontitis after therapeutic treatment A. Congenital median cyst
proved to be inefficient. What instrument B. Lipoma
will you choose for removal? C. Lymphadenitis
D. Retention cyst of sublingual salivary
A. S-shaped forceps gland
B. Bayonet-shaped crown forceps E. Dermoid cyst
C. S-shaped right forceps
D. Straight forceps 87. An 8 year old child complains about
E. Bayonet-shaped root forceps permanent dull pain in the 46 tooth that
is getting stronger during cutting. The
84. A patient came to a dental clinic for pain appeared 1 day ago. Previously there
the purpose of prosthetics. Objectively: has been pain induced by cold stimuli.
the lower third of her face is diminished, Objectively: there is a deep carious cavi-
nasolabial folds are deepened, frontal ty on the masticatory surface of the 46
group of teeth on both upper and lower tooth, tooth cavity is closed, probing and
jaws is missing, crowns of the 17, 15, 26, 27, temperature stimuli cause no pain reacti-
36, 37, 45, 47 teeth are worn by 2/3 of their on. Percussion is painful, a slight mobility
height, masticatory surfaces of these teeth is present. Gum around the 46 tooth is
are smooth, pigmented, alveolar process is hyperemic, edematic, palpatory painful.
not hypertrophied, interalveolar height is X-ray picture shows no changes near the
reduced. What form of pathological tooth apex of undeveloped roots. What is your
wear is it? provisional diagnosis?
A. Horizontal, noncompensated, III A. Acute serous periodontitis
degree of severity B. Acute purulent pulpitis
B. Horizontal, compensated, II degree of C. Acute general serous pulpitis
severity D. Acute purulent periodontitis
C. Vertical, noncompensated, III degree of E. Acute condition of chronic periodontitis
severity
D. Vertical, compensated, III degree of 88. A 20 year old patient complains of
severity a tumour-like formation on the lateral
E. Mixed, noncompensated, III degree of surface of his neck on the right. He
severity revealed this tumour a year ago. Objecti-
vely: there is a semi-oval well-defined
85. An 11 year old patient complai- tumour 3х2 cm large on the lateral neck
ns about tooth mobility and gingival surface. The tumour doesn’t hold together
haemorrhage. He has been suffering si- with skin, it is painless, dense and elastic,
nce he was 3 years old. Objectively: upwardly and sidewardly movable. Regi-
gums around all the teeth are hyperemic, onal lymph nodes are not enlarged. After
edematic, bleed during instrumentl the tumour puncture some light yellow
examination. Tooth roots are dehisced by liquid was obtained that consisted of
1/3 and covered with white deposit. There cast-off epithelium, cholesterol crystals,
Krok 2 Stomatology 2008 14

lymphocytes, erythrocytes. What is the the 42, 41, 31, 32 teeth are dehisced by
most probable diagnosis? 2/3, there is pathological mobility of the
III degree. The patient has indication for
A. Lateral neck cyst removal of the 42, 41, 31, 32 teeth. What
B. Median neck cyst type of denture should be applied for
C. Dermoid neck cyst immediate-insertion prosthetics?
D. Neck chemodectoma
E. Chronic neck lymphadenitis A. Partial removable lamellar
B. Clasp
89. A 28 year old woman complains about C. Adhesive
acute pain on the left of her upper jaw D. Ceramic-metal bridge
during eating. 6 days ago the 28 tooth E. Stamped-soldered bridge
was extracted, after that the pain came.
The patient considers that extraction of 93. A 13,5 year old girl complains of gingi-
the 28 tooth caused "damaging"of the val painfullness and haemorrhage duri-
next tooth. Objectively: alveolus of the ng tooth brushing and eating, halitosis.
28 tooth is at a stage of healing. On the She has been ill with angina for a week.
distal surface of the 27 tooth there is a Objectively: mucous membrane of gums
carious cavity in the precervical area that in the area of frontal teeth of her upper
doesn’t communicate with the tooth cavi- and lower jaws is edematic, hyperemic.
ty. Cold stimulus causes short attack of Apices of gingival papillae are necrotic,
pain. Dentin of walls and floor is light and they also bleed when touched. There is a
softened. Probing of the floor is painful. thick layer of soft tooth plaque. What is
What is the most probable diagnosis? the causative agent of this disease?
A. Acute deep caries A. Anaerobic microflora
B. Acute local pulpitis B. Herpes virus
C. Acute diffuse pulpitis C. Streptococci
D. Chronic deep caries D. Staphylococci
E. Chronic fibrous pulpitis E. Yeast fungi
90. A 68 year old patient underwent full 94. A month after cementation of a metal-
jaw removal. Before the operation an ceramic crown on the 23 tooth a pati-
impression of the patient’s upper and ent applied to the dentist with complai-
lower jaws was taken and a substitutive nts about its decementation. Examination
denture was made. What are the means of revealed that tooth stump was of sufficient
the denture fixation in the oral cavity? height, its walls converged to the vertical
tooth axis at an angle of approximately 30
A. Spiral Fosher springs degrees. At what angle was it necessary to
B. Anchors establish convergention of stump walls to
C. Clasps the tooth axis?
D. Magnets
E. Attachments A. Up to 8 degrees
B. 12-15 degrees
91. A patient complained about pain in his C. 15-18 degrees
45 tooth induced by cold, sour and sweet D. 22-25 degrees
food stimuli. The pain abates when the E. 10-12 degrees
stimulus action is stopped. Objectively:
there is a carious cavity on the masticatory 95. A 57 year old patient complains about
surface within mantle dentin consisting of mobility of his metal-ceramic dental bri-
food rests and softened dentin, overhangi- dge supported by the 33, 37 teeth. The bri-
ng enamel edeges are chalky. What is the dge has been in use for 9 months. Objecti-
diagnosis? vely: X-ray picture shows alveolar process
atrophy by 2/3 in the area of the 33, and
A. Acute median caries by 1/2 of root length in the area of the 37;
B. Chronic median caries there are pathological pockets, gingivitis.
C. Acute superficial caries What is the cause of pathological mobility
D. Acute deep caries of supporting teeth?
E. Chronic deep caries
92. A 48 year old teacher complained
about considerable mobility of the 42,
41, 31, 32 teeth. Objectively: cervices of
Krok 2 Stomatology 2008 15

A. Functional shifting of supporting teeth A. Physiological infantile retrogenia


under stress B. Mesial occlusion
B. Injuring of circle ligament by crown C. Physiological occlusion
edges D. Distal occlusion
C. Devitalization of supporting teeth E. Edge-to-edge occlusion
D. Massive grinding off of hard tissues
E. Garland modelling in the precervical 100. A 52 year old patient complains
area of significant tooth wear on both jaws.
Objectively: tooth wear in the lateral
96. During lateral motions of a mandi- parts of the lower jaw at a level of gums
ble frontal teeth are deviated. Sideward and wear by 1/3 in the frontal parts. The
relocation of incisive point from the treatment had two stages. What denture
central position has an angle of 100 −110o . construction for occlusion disconnection
What is characterized by such value of and reconstruction of myostatic reflexes
incisive point deviation? should be applied in this case?
A. Lateral incisive tract A. Dentogingival splint in the lateral parts
B. Lateral articulate tract B. Plastic cap for the whole dentition
C. Sagittal incisive tract C. Plastic cap in the frontal part
D. Sagittal articulate tract D. Crown splint in the lateral parts
E. Bennett’s angle E. Elastic plastic cap
97. A child was brought to the 101. A 1,8 year old boy was under
traumatology centre of oral surgery treatment in the infectious disease
department with complaints about department. He was given ampicillin. On
changed position of the 21 tooth that was the 6th day of treatment there appeared
inclined towards palate. The day before white deposits in form of caseous films
the child was hit in the face. Make a di- that were revealed on the hyperemic
agnosis: mucous membrane in the area of gingi-
val torus, cheeks and on the tongue. The
A. Subluxation of the 21 tooth films can be removed leaving hyperemic
B. Complete dislocation of the 21 tooth surface underneath them. General condi-
C. Fracture of crown part of the 21 tooth tion is satisfactory. Body temperature is
D. Contusion of the 21 tooth 36, 7oC. What is the provisional diagnosis?
E. Break-off of the crown part of the 21
tooth A. Acute candidous stomatitis
B. Drug-induced stomatitis
98. Parents of a 1,5 year old child complai- C. Chronic candidous stomatitis
ned about tongue enlargement, disturbed D. Acute herpetic stomatitis
food intake. The child has ben sufferi- E. Mild case of leukoplakia
ng from this since birth. Objectively:
general condition has no pecularities. 102. A 47 year old patient came to
Tongue is enlarged (macroglossia). Its an orthopaedic stomatology center wi-
mucous membrane has granular vesicle- th complaints about missing tooth in the
like outgrowths. Tongue is compact, frontal part of his upper jaw, cosmetic
palpatory painless. What is the most defect. Objectively: the occlusion is
probable diagnosis? orthogenic, the 11th tooth is missi-
ng. Anamnesis data: the patient had
A. Lymphangioma of tongue myocardial infarction 3 months ago. What
B. Tongue hemangioma denture should be applied for the time
C. Tongue fibroma being?
D. Tongue cyst
E. Tongue cancer A. Partial removable lamellar denture
B. Bridge denture suported by the 21 and
99. A visiting nurse examined a newborn 12 teeth
child. Examination revealed that lower C. Bridge denture suported by the 21 tooth
face part is shorter than median one, chin D. Clasp denture
is retrodeviated, teeth are missing, lower E. Implant
jaw is retrodisplaced. What is the name of
such mandible position of a newborn? 103. A 53 year old patient has to undergo
resection of the left half of mandible
together with its ramus. It is indicated
to make an immediate Oxman’s denture.
Krok 2 Stomatology 2008 16

What denture element keeps the remai- Tooth crowns are intact but have oral posi-
ning mandible fragment from deviation tion. Complete joining of teeth is impossi-
towards the defect? ble because the teeth are situated beyond
the dental arch. X-ray picture shows a sli-
A. Removable or non-removable inclined ght broadening of periodontal fissure of
plane the 11 tooth up to 0,5-2 mm. The roots are
B. The whole fixing part intact. Make a correct diagnosis:
C. Multiclasp system
D. Resection part A. Traumatic subluxation of the 11, 21
E. Artificial teeth teeth
B. Traumatic periodontitis of the 11, 21
104. Examination of a 9 year old patient teeth
revealed a milky spot on the vestibular C. Traumatic complete dislocation of the
surface of the 11 tooth close to the cutti- 11, 21 teeth
ng edge. Probing and tempearture stimuli D. Fracture of alveolar process in the area
cause no pain reaction. The child was di- of the 11, 21 teeth
agnosed with local enamel hypoplasia of E. Traumatic extraction of the 11, 21 teeth
the 11 tooth. What treatment should be
administered? 108. A 50 year old patient has a defect of
his lower dentition. It is planned to make
A. A course of remineralizing therapy a bridge denture supported by implants.
B. Prophylactic hygiene of oral cavity X-ray picture showed that the height of
C. Spot removal osseous tissue mass from projection of
D. Hermetization of the affected part mandibular canal to the top of alveolar
E. Cosmetic filling crest was 2 cm. What implant wil be
recommended?
105. A 70 year old patient is awaiting
complete removable dentures for both A. Screw
upper and lower jaws. Teeth placement B. Endodonto-endoossal
will be made by Vasilyev’s method. What C. Leaflike
teeth in the upper denture must not touch D. Subperiosteal
glass? E. Conical
A. Lateral incisors and second molars 109. External examination of a 7 year old
B. Central incisors and first molars child revealed: thickening of nose bridge,
C. First and second premolars semi-open mouth, dry lips. Mouth corners
D. Canines and first molars are peeling. Anamnesis data: the child
E. Second premolars and first molars sleeps with open mouth. Examination of
oral cavity revealed no changes. What di-
106. Examination of a 6 year old child spensary group will this child fall into?
revealed a deep carious cavity in the 85
tooth. Percussion and probing are pai- A. The second
nless. Removal of softened dentin resulted B. The first
in intercommunication with tooth cavi- C. The third
ty. Deep probing is painful. X-ray picture D. The fourth
of the 85 tooth shows a focus of bony ti- E. -
ssue destruction in the area of bifurcati-
on, cortical plate of the 35 tooth has no 110. A 14 year old patient applied to
pathological changes. What paste should an orthodontist. Objective examination
be applied for the root filling in this case? revealed that on the site of the second
incisor a canine tooth had cut out, and on
A. Zink-eugenol paste the site of the canine - the second incisor.
B. Resorcin-formaline paste The same pathology has also the patient’s
C. Glass-ionomer cement father. Make a diagnosis:
D. Phosphate cement
E. Calcium-containing paste A. Transposition of lateral incisor and
canine
107. A 20 year old patient got a trauma in B. Distal position of lateral incisor
the area of his upper jaw. He applied to a C. Palatine position of lateral incisor
dentist and complained about mobility of D. Mesial position
his frontal upper teeth, pain during cutti- E. Superocclusion of incisor and
ng and joining of teeth. Objectively: the 11 infraocclusion of canine
and 21 teeth have II-III degree mobility.
Krok 2 Stomatology 2008 17

111. Parents of an 8 year old boy complain fficult food mastication, esthetic face
about a cosmetic defect, inability to bi- defect. Objectively: the face is proporti-
te off food. The child often suffers from onal, asymmetric as a result of oral
acute viral respiratory infections. Objecti- displacement of a lateral fragment of
vely: chin skewness, mental fold is most the left lower jaw. When the jaws are
evident. The lower lip is everted, superior closed the 34, 35, 36, 37 teeth are not in
central incisor lies on it, nasolabial fold contact with their antagonists. The pati-
is flattened. In the oral cavity: occlusion ent strongly objects to surgical operati-
period is early exfoliation period. The on. What orthopaedic treatment should
upper jaw is narrowed, there is gothic be administered?
palate. Frontal teeth have fan-shaped
position. Sagittal fissure is 6 mm. In the A. A denture with double dentition
lateral parts contact of homonymous teeth B. Mechanotherapy
is present. What is the most probable C. Interjaw traction
cause of dentoalveolar deformity? D. Crowns with occlusal applications
E. Levelling of occlusal surface
A. Pathology of upper airways
B. Missing of Caelinski ledge 115. A patient complains about
C. Endocrinal diseases spontaneous pain in the area of his 15
D. Untimely sanitation of oral cavity tooth he has been feeling for 2 days.
E. Gestational toxicosis Thermal stimuli make the pain worse, its
attacks last up to 30 minutes. Objecti-
112. A 20 year old patient complains vely: there is a deep carious cavity in
about gum itching, gingival haemorrage the 15 tooth consisting of light softened
during tooth brushing and eating, unusual dentin, floor probing is painful in one
look of gums. He has been observing these point, reaction to the thermal stimuli is
presentations for the last 1,5 year. It is positive, percussion is painless. Make a di-
known from the patient’s anamnesis that agnosis:
he has been taking diphenylamine anti-
convulsants for 2 years. Objective exami- A. Acute local pulpitis
nation revealed gingival hyperemia and B. Acute diffuse pulpitis
edema. In the frontal part the gums cover C. Pulp hyperemia
vestibular surface of teeth by 1/2 of their D. Acute deep caries
height. Along the loose gingival edge E. Acute condition of chronic pulpitis
growth of granulation tissue is present,
probing causes gingival haemorrage. No 116. A 50 year old woman complains
roentgenological changes were revealed. about a neoplasm on her lower lip on
Make a diagnosis: the side of oral cavity that appeared a
month ago and has been slowly growing
A. Chronic hypertrophic gingivitis since that. Objectively: there is a roundish,
B. Chronic ulcerative gingivitis elastic, painless neoplasm inside the lower
C. Chronic catarrhal gingivitis lip. Mucous membrane hasn’t changed its
D. Localized periodontitis colour. Make a diagnosis:
E. Generalized periodontitis
A. Retention cyst of lower lip
113. A patient has got a traumatic fracture B. Lip abscess
of mandible in the area of the missing 34, C. Lip papilloma
35 teeth with a slight displacement and a D. Lip fibroma
defect of alveolar part in the area of the E. Lip lipoma
34, 35 teeth. Other teeth on both lower
and upper jaws are intact. What splint 117. A 5 year old girl with crossbite was
would be optimal in this case? referred to an orthodontist. Objectively:
between frontal teeth there are diaereses
A. Tigerstedt’s splint with a spreading and diastems, canine tubera have no si-
curve gns of physiological wear out. Central line
B. Plain splint cramp between incisors doesn’t match. What is
C. Port’s splint the doctor’s tactics?
D. Vasilyev’s splint
E. Vankevich splint
114. A 36 year old patient applied to
a dentist 1,5 month after a mandi-
bular trauma with complaints about di-
Krok 2 Stomatology 2008 18

A. To remove unworn tubera of canines A. Fusospirochetal symbiosis


B. To administer jaw massage B. Streptostaphylococci
C. To make a screw plate for the upper jaw C. Pale treponema
D. To wait for autoregulation D. Virus of herpes simplex
E. To disconnect occlusion E. Candida fungi
118. Prophylactic examination of a 6 year 121. A 48 year old patient complained
old child revealed: occlusion of temporary about posttraumatic pain in the area of
teeth. Both superior and inferior dental his chin. Examination revealed median
arches are trapeziform. Superior incisors fracture of lower jaw with I class dentition
overlap the inferior ones by more than defect (according to Kennedy classificati-
2/3. Correlation of canines and second on). It is indicated to make a Vankevich
molars is homonymous. There are no splint in Stepanov’s modification. What
spaces between frontal teeth. Superior will determine the height of directing
dental arch is larger than inferior one by planes of this splint?
the size of buccal cusp. In what planes can
the occlusion deformity be defined? A. Extent of mouth opening
B. It will be determined arbitrary
A. Sagittal and vertical C. Height of central jaw correlation
B. Sagittal and transversal D. Form of edentulous alveolar crests of
C. Sagittal and occlusal the lower jaw
D. Sagittal and nasal E. Height of molar teeth of the upper jaw
E. Sagittal and Frankfort’s
122. A 24 year old soldier injured by a
119. A 43 year old patient complains of shell splinter was diagnosed with fracture
tooth mobility and considerable dehi- of mandible accompanied by an over 2 cm
scence of dental cervices of the lower jaw long bone defect in the chin area. What
in its frontal part. Objectively: gums in method of fixation of mandible fragments
the area of the 44, 43, 42, 41, 31, 32, 33, 34 is indicated?
teeth are pale, slightly cyanotic, without
haemorrhage. The 42, 41, 31, 32 teeth have A. Break fixation by means of Rudko’s
mobility of the I-II degree. There is also apparatus
overcrowding of the 42, 41, 31, 32 teeth. B. Tigerstedt’s splints
Cervices of the 42, 41, 31, 32 teeth are C. Intermandibular Ivy ligature
dehisced by 1/2 of root length, of the 43, D. Gunning-Port’s splint
33 - by 1/4. What orthopaedic constructi- E. Direct osteosynthesis
on should be applied in this case?
123. A 35 year old patient consulted a
A. Fullcast removable kappa dentist about a painless, slowly growi-
B. Kurlandsky’s girder splint ng neoplasm in the area of the 11 and
C. Cap splint 12 teeth. Examination revealed that
D. Halfcrown splint the neoplasm had flattened form, was
E. Semicircle splint adjacent to the teeth, had a pedicle, was
of light-pink colour, up to 1,5 cm large, wi-
120. An 18 year old patient complains th smooth surface and dense consistensy.
of gingival painfulness and haemorrhage, Diagnosis: epulis in the area of the 11, 12
halitosis, temperature rise up to 38, 6oC, teeth. What form of epulis are these clini-
general weakness, appetite loss. Objecti- cal presentations typical for?
vely: mucous membrane of oral cavity is
hyperemic and dry; tongue is covered with A. Fibrous
white fur, gingival papillae are edematic, B. Angiomatous
their apices have areas of necrotic deposit C. Giant-cell
that can be easily removed leaving bleedi- D. Epulis of pregnancy
ng surface beneath. Submaxillary lymph E. -
nodes are enlarged, palpatory painful.
What is the causative agent of this di- 124. As a result of a road accident a 45
sease? year old patient got an injury of his upper
jaw. Examination revealed elongated and
flattened face, profuse nasal haemorrhage,
liquorrhea from the nose and ears. These
clinical presentations are typical for the
following fracture of upper jaw:
Krok 2 Stomatology 2008 19

A. Subbasal (Le Fort III) A. Devital amputation


B. Subnasal (Le Fort I) B. Biological method
C. Suborbital (Le Fort II) C. Vital extirpation
D. Bilateral fracture of zygomatic bones D. Vital amputation
E. - E. Devital extirpation

125. A child is 2,5 year old. The parents 129. An 8 year old child complains about
complain about thumb sucking during pain in the 21 tooth that is getting worse
sleep. What tactics should the doctor during cutting. A month ago a part of
choose? tooth crown broke off as a result of a fall.
The child didn’t consult a dentist. Objecti-
A. To recommend an ulnar fixator vely: in the area of medial angle of the 21
B. To talk with a child about harm from tooth there is a crown defect that makes
thumb suction up 1/3 of the crown’s height. Tooth cavi-
C. Medical intervention is unnecessary ty is open, probing and thermal stimulus
D. Non-removable device for suppression cause no pain. Percussion is acutely pai-
of bad habit nful. Gum around the 21 tooth is edematic
E. Removable device for suppression of and hyperemic. What is the provisional di-
bad habit agnosis?
126. A patient is waiting for a partial A. Acute condition of chronic periodonti-
removable laminar denture. Anatomic tis
models of both jaws were made by means B. Acute condition of chronic pulpitis
of elastic alginate material "Ipin"and C. Acute serous periodontitis
referred for disinfection. What disinfecti- D. Acute purulent periodontitis
on method should be applied? E. Pulpitis complicated by periodontitis
A. Glutaraldehyde 2,5% рH 7,0-8,7 130. An orthodontist was treating a pati-
B. Sodium hypochloride 0,5% ent infected with AIDS virus. Acci-
C. Desoxone 0,1% dentally he injured skin of one of his
D. Hydrogen peroxide solution 6% fingers with a dental disk during tooth
E. Alcohol solution 70% preparation. What actions should the
orthodontist take in this case?
127. A 13 year old boy complains about
pain in the 46 tooth induced by cold sti- A. To press blood out and treat the skin
muli. Objectively: there is a deep carious with 70% alcohol solution
cavity on the masticatory surface of the 46 B. To apply a tourniquet on the shoulder
tooth within light softened circumpulpar C. To perform diathermo-coagulation
dentin. Probing of carious cavity floor is D. To treat the skin with 5% iodine soluti-
overall painful, cold stimulus causes acute on
pain that is quickly relieved after the sti- E. To press blood out and treat the skin
mulus’ elimination. Choose an optimal with strong solution of KMnO4
dental treatment paste:
131. A 36 year old patient complains
A. Hydroxycalcium-containing about acute pain, "clicking"in the right
B. Thymol mandibulotemporal joint, burning in the
C. Iodoform area of her right external acoustic meatus.
D. Resorcin-formaline Movements of her lower jaw are step-
E. Zink-eugenol like, along with brief blocking moments
in the joint and acute pain. Objecti-
128. An 8 year old child has a deep cari- vely: the face is symmetric. Occlusion is
ous cavity communicating with tooth cavi- orthognathic, intraoral palpation of lateral
ty on the distaloapproximal surface of the pterygoid muscle causes pain on the right.
75 tooth. Probing is painful, percussion is Tomograms show that contours of bone
painless, cold water causes slowly abating structures of articular surfaces are regular
pain. The tooth decayed a few months ago, and smooth. What is the most probable
wasn’t treated. What treatment method is diagnosis?
to be applied in this case?
Krok 2 Stomatology 2008 20

A. Mandibulotemporal joint disfunction


B. Rheumatic arthritis of mandi- A. With dentin wetting agent
bulotemporal joint B. Enamel and dentin should be treated
C. Acute posttraumatic artritis of mandi- with mordanting gel once again
bulotemporal joint C. Enamel and dentin should be treated
D. Deforming arthrosis of mandi- with adhesive system
bulotemporal joint D. Enamel and dentin should be treated
E. Anchylosis of mandibulotemporal joint with hydrogen peroxide
E. Enamel and dentin should be treated
132. A 40 year old patient complains with alcohol
about difficult mastication as a result of
lower jaw deviation. Anamnesis data: 136. A 42 year old patient complains
mental fracture 2 months ago. Objecti- about tooth mobility, change of tooth
vely: the 35; 36; 38 ... 45; 46 teeth are position, foul taste, pain during food
missing. Remaining teeth are intact. The mastication. Objectively: teeth of both
43; 44; 47; 48 teeth have no contact with upper and lower jaws are mobile (2-3
antagonists and their oral deviation makes degree), there are also diastems and di-
up 1 cm. What is the optimal construction aereses. Tooth roots are dehisced by 1/2.
of lower jaw denture? There are serous purulent discharges from
periodontal pockets. Orthopantomogram
A. A denture with double dentition shows bony tissue destruction down to 2/3
B. Metal-ceramic dental bridge of root length. There are bone pockets.
C. Soldered splint on rings What is the most probable diagnosis?
D. Removable lamellar denture
E. Adhesive denture A. Chronic generalized periodontitis (III
degree)
133. A patient is 24 years old. Artificial B. Papillon-Lefevre syndrome
crown of his 22 tooth broke off. The tooth C. Chronic catarrhal gingivitis
was devitalized 8 years ago and its crown D. Atrophic gingivitis
is totally decayed. What microprosthesis
should be made for restoration of tooth E. Chronic generalized periodontitis (II
crown? degree)

A. Stump-root inlay 137. A 60 year old man has got a pustule


B. Filling on his chin skin that quickly developed
C. Inlay into a hard, strongly painful infiltrate 3х3
D. Application cm large. In the epicentre three necrosis
E. Stamped crown zones around hair follicles are present.
Lymph nodes of chin are enlarged, pai-
134. One of the methods to define central nful. Body temperature is 38, 5oC. What is
occlusion is to construct a flat that will the most probable diagnosis?
go through cutting edges of central inci-
sors and distalobuccal tubercles of last A. Chin carbuncle
molar teeth providing that there is suffi- B. Erysipelatous inflammation of chin
cient quantity of teeth. What flat should C. Suppurated atheroma
be constructed? D. Chin furuncle
E. Allergic chin dermatitis
A. Occlusal flat
B. Sagittal 138. A 50 year old patient comlains of
C. Vertical difficult mastication, tooth mobility, hali-
D. Transversal tosis, gingival haemorrhage. Objectively:
E. Frankfort gums are hyperemic, cyanotic, there is also
dental scale. Parodontal pockets of upper
135. During restoration of a II class molar teeth were 8 mm, of the rest teeth -
(according to Black’s classification) cari- 6 mm. X-ray picture showed resorption of
ous cavity of the 25 tooth a doctor applied bone tissue by 2/3-1/2 of root length. What
methods of total mordanting and drying is the most probable diagnosis?
of hard tissues. After that he noticed that
enamel surface became chalky and dentin
became completely dry. How should he
treat enamel and dentin surfaces for the
further restoration by method of ”wet −
Bonding”?
Krok 2 Stomatology 2008 21

A. Generalized III degree periodontitis, A. Chronic initial caries


chronic B. Fluorosis
B. Generalized II degree periodontitis, C. Local enamel hypoplasia
chronic D. Acute initial caries
C. Generalized I degree periodontitis, E. Chronic superficial caries
chronic
D. Generalized III degree periodontitis, 142. A 35 year old man complains about
acute condition pain in the area of the 38 tooth, painful
E. Generalized II degree periodontitis, deglutition, difficult mouth opening. What
acute condition anesthesia method will be optimal during
operation on account of pericoronaritis?
139. An 18 year old patient complains
about gingival enlargement, haemorrhage A. Conduction Bersche-Dubov’s
and pain during eating. Objectively: gi- anesthesia
ngival edema, hyperemia, enlargement is B. Infiltration anesthesia
up to 1/3 of tooth crown length. Palpati- C. General anesthesia
on causes intense haemorhage and pain. D. Stem anesthesia
What is the most probable diagnosis? E. Application anesthesia
A. Hypertrophic gingivitis, edematous 143. A young woman complained about
form having vesicles in oral cavity, painful
B. Hypertrophic gingivitis, fibrous form ulcers, especially during talking and
C. Acute catarrhal gingivitis eating; muscle and joint pain, body
D. Chronic catarrhal gingivitis temperature rise, indisposition, weakness.
E. Acute condition of initial generalized She fell suddenly ill 2 days ago. Objecti-
periodontitis vely: to - 38, 4oC. Vermilion border is
covered with bloody crusts that stick
140. A boy is 1 month old. He has a together and impede mouth opening.
wound with purulent discharge near the Mucous membrane of lips, cheeks, mouth
medial edge of inferior eyelid, on the floor, tongue, soft palate is hyperemic
right. The illness began abruptly, body and edematic; there are single vesicles
temperature is up to 40o C. General condi- and large painful erosions covered with
tion is grave. On the second day of illness fibrinous deposit on it. Regional lymph
there appeared an infiltration near the nodes are enlarged and painful. There is
internal edge of orbit and cheek on the hypersalivation. Nikolsky’s symptom is
right. Skin above it is hyperemic, fluctuati- negative. What is the most probable di-
on cannot be detected. Palpebral fissure is agnosis?
narrowed. Right nasal meatus discharges
pus. There is an infiltration on the vesti- A. Multiform exudative erythema
bular surface of alveolar process and B. Acantholytic pemphigus
palate on the right. Mucous membrane C. Secondary syphilis
above it along the mucogingival fold is D. Acute herpetic stomatitis
hyperemic, there is fluctuation. What is E. Non-acantholytic pemphigus
the most probable diagnosis?
144. A 20 year old patient complains
A. Acute hematogenous osteomyelitis about gingival pain and haemorrhage
B. Acute dacryocystitis she has been suffering from for a week.
C. Right-side orbital phlegmon The pain is getting worse during eati-
D. Acute right-side maxillary sinusitis ng, haemorrhage is usually induced by
E. Acute serous periostitis mechanical stimuli. Objectively: there are
evident hyperemia and edema of gums
141. A 25 year old patient complains of in the area of inferior frontal teeth. Api-
a light brown spot on the frontal tooth ces of gingival papillae are dome-shaped,
of her upper jaw. Oblectively: a single enlarged. Gum palpation causes pain, sli-
light brown spot in the precervical area ght haemorrhage. No roentgenological
of the 23 tooth, probing revealed that its changes were revealed. What is the most
surface was smooth. Reaction to the cold probable diagnosis?
stimulus and probing was painless. What
is the most probable diagnosis?
Krok 2 Stomatology 2008 22

A. Catarrhal gingivitis A. 10-20% solutions of EDTA sodium


B. Hypertrophic gingivitis salts
C. Localized periodontitis B. Resorcin-formaline liquid
D. Ulcerative gingivitis C. 30% solution of silver nitrate
E. Atrophic gingivitis D. Essential oils
E. Medications with antibacterial and
145. A 42 year old patient applied to anti-inflammatory effect
a dentist and complained about whi-
te caseous deposit on the dorsum of 149. A patient applied to a dentist and
his tongue and burning sensation. It is complained about periodical pain in his
known from the patient’s anamnesis that upper jaw. Anamnesis data: the 12 tooth
he underwent treatment in an in-patient has been previously treated on account
hospital on account of pneumonia. What of pulpitis. Objectively: crown of the 12
is the most probable diagnosis? tooth was restored by permanent filling
material. X-ray picture of the upper jaw
A. Acute pseudomembranous candidosis shows a focus of bony tissue destruction
B. Typical form of leukoplakia near the root apex of the 12 tooth up to
C. Lichen ruber planus 15 mm in diameter. Root canal is filled
D. Scarlet fever up to the top. What is the most optimal
E. Lupus erythematosus treatment method?
146. A patient complained about A. Root apex resection of the 12 tooth
frequent haemorrhages from the mucous B. Conservative treatment
membrane of oral and nasal cavities, he C. Removal of the 12 tooth
mentioned also that his father had the D. Replantation of the 12 tooth
same problems. Objectively: there are E. Dissection along the mucogingival fold
multiple telangiectasias and angimatous
formations on face skin as well as on 150. A 46 year old patient complai-
mucous membrane of nose, cheeks and ns about difficult mouth opening,
lips. Blood count is normal. What is the body temperature rise, edematic tissues
most probable diagnosis? around both mandibulotemporal joints.
Anamnesis data: 2-3 months ago the pati-
A. Rendu-Osler-Weber disease ent had undurable mild bilateral pain
B. Werlhof’s disease attacks in the parotidomasticatory areas
C. Vaquez disease that lasted for a few days, limited mouth
D. Cushing’s basophilism opening, sense of tension and discomfort
E. Addison-Biermer disease in some areas of left and right mandi-
bulotemporal joint. What is the most
147. Unused stomatological instruments probable provisional diagnosis?
were left on a sterile table in the dental
room after the end of working day. What A. Rheumatic arthritis
actions should be taken in order to B. Infectious arthritis
provide sterility of these stomatological C. Arthrosis
instruments? D. Fibrous anchylosis
E. Deforming arthrosis
A. Sterilization without pretreatment
B. Disinfection, sterilization 151. An 11 year old girl has adentia,
C. Disinfection, presterilizing cleansing, the 35 tooth is missing, it was proved
sterilization roentgenologically. Between the 34 and
D. Presterilizing cleansing, sterilization 33 teeth as well as between the 34 and 36
E. Disinfection only teeth there are diaereses, the 34 tooth is
turned by 30o relative to its glossobuccal
148. A 23 year old patient underwent direction. What abnormal position does
treatment of pulpitis of her 16 tooth. Duri- the 34 tooth have?
ng endodontic procedure an obstructed
distal buccal canal was found. What medi- A. Tortoocclusion (rotation of teeth) and
cations can be applied for chemical di- distal
latation of root canal? B. Distal
C. Mesial
D. Vestibular
E. Oral
152. A 64 year old patient applied to a
Krok 2 Stomatology 2008 23

dental clinic for tooth prosthetics. Objecti- have surgery hours makes up 26 m2 . What
vely: there is a sharp bony prominence area must this room with two universal
in the area of the missing 15 tooth. It dental devices have according to the exi-
is planned to make a partial removable sting regulations?
denture with two-layer base. What plastic
should be used for elastic backing? A. 14 m2 for each dental device and 10 m2
additionally
A. Plastic PM-01 B. 10 m2 for each dental device and 10 m2
B. Ftorax additionally
C. Acryl C. 10 m2 for each dental device and 7 m2
D. Protacryl additionally
E. Bacryl D. 7 m2 for each dental device and 7 m2
153. A dentist was called to a patient bei- additionally
ng treated in the cardiological department E. 20 m2 for each dental device and 12 m2
after myocardium infarction. The pati- additionally
ent was diagnosed with acute conditi-
on of chronic fibrous pulpitis of the 36 157. Examination of a 23 year old
tooth. What method of pulpitis treatment patient revealed chronic candidosis of
should be chosen taking into account oral mucous membrane, generalized
grave condition of the patient? lymphadenopathy. Anamnesis data: the
patient has been suffering from herpes for
A. Devital amputation a year. Body temperature persistently ri-
B. Vital extirpation ses up to 37, 4 − 37, 5oC, body weight has
C. Vital amputation reduced by 8 kg over the last month. What
D. Devital extirpation disease can be indicated by this symptom
E. Conservative method group?

154. Walls and floor of a dental room were A. AIDS


painted with recommended light colours. B. Infectious mononucleosis
What is the required coefficient of reflecti- C. Acute leukosis
on from the walls, ceiling and floor? D. Chronic leukosis
E. Candidosis
A. 60
B. 35 158. A 51 year old lecturer came to the
C. 30 orthopaedic department and complained
D. 25 about painfullness and mobility of his
E. 20 frontal teeth of the lower jaw. Cervices
of the 42, 41, 31, 32 teeth are dehisced,
155. A 7 year old girl was brought to a III degree mobility with deep pathologi-
hospital for the purpose of oral cavity cal pockets is present. When would it be
sanitation. She was born and has been livi- reasonable to fix dentures after dental
ng in an area where fluorine concentration extraction?
in water makes up 2,5 mg/l. Examination
revealed symmetrically placed dark spots A. On the day of dental extraction
on the vestibular surfaces of the 11, 21, B. In 16-30 days
31, 41 teeth as well as on the tubera of the C. In 2-3 months
16, 26, 36, 46 teeth. It is known from the D. In 4-6 months
anamnesis that the teeth cut out already E. In 5-6 days
with affection. What is the most probable
diagnosis? 159. A 32 year old patient lost his 41
and 31 teeth as a result of a sporti-
A. Tooth fluorosis ng accident. Replantation is impossible
B. Odontogenesis imperfecta because these teeth have cracks and splits
C. Amelogenesis imperfecta both on crown and root surfaces. Clinical
D. Systemic enamel hypoplasia and roentgenological examination proved
E. Local enamel hypoplasia the possibility of immediate implantati-
on. What implant materials should be
156. A dentist applied to a regional preferred?
sanitary-and-epidemiologic institution for
an authorization to open a private dental
surgery with 2 universal dental devices.
The area of room where he is planning to
Krok 2 Stomatology 2008 24

A. Titanium membrane of lips and tongue roundi-


B. Carbon sh painful erosions 1-3 mm in diameter
C. Ceramic covered with whitish deposit. The gum on
D. Platinum both upper and lower jaws is hyperemic,
E. Sapphire edematic, bleed when touched. What is
the most probable diagnosis?
160. A 9 year old child complains about
pain and swelling of soft tissues beneath A. Acute herpetic stomatitis
his mandible on the right. Objectively: B. Stevens-Johnson syndrome
general condition is satisfactory; there is C. Recurrent aphthous stomatitis
face asymmetry due to the inflammatory D. Multiform exudative erythema
infiltration and collateral tissue edema E. Recurrent herpetic stomatitis
in the right submandibular area. Crown
of the 85 tooth is decayed, mucous 164. An 11 year old girl complains about
membrane of gums in the area of the bleeding of a tooth on her lower jaw
84, 85, 46 teeth is hyperemic, edematic; during eating and tooth brushing. This
mucogingival fold is flattened. What tooth hurt her before but she didn’t
treatment should be administered? consult a dentist. Examination of the 46
tooth revealed a deep cavity communi-
A. Extraction of the 85 tooth cating with tooth cavity and filled with
B. Endodontic treatment of the 85 tooth red growth tissue. Probing causes a slight
C. Extraction of the 85 tooth and incision haemorrhage and pain, percussion is pai-
along the mucogingival fold nless, cold stimulus cause mild pain. What
D. Periosteotomy is the most probable diagnosis?
E. Endodontic treatment of the 85 tooth
and periosteotomy A. Chronic hypertrophic pulpitis
B. Chronic granulating periodontitis
161. Mother of a 3 year old child brought C. Chronic papillitis
the child to an orthodontist and complai- D. Gingival polyp
ned about total lack of crown part of the E. Chronic simple pulpitis
51 and 61 teeth. What tactics should the
doctor choose? 165. A patient came to a dental clinic
for dental prosthetics. Objectively: total
A. Thin-walled cap lack of teeth on the mandible. Sharp and
B. Metal-ceramic crown regular atrophy of alveolar part. Frenula
C. Stump tooth attachment and fold position is high.
D. Inlay Name the type of atrophy of edentulous
E. Tooth extraction mandible using Keller’s classification:
162. A 12 year old girl complains about A. II type
burning and painfulness of her tongue, B. I type
especially during eating spicy food. C. III type
Objectively: there are oval red spots on D. IV type
the tip and dorsum of tongue. Filiform E. V type
papillae are not present in the affected
area. The girl mentions that the spots 166. A 35 year old patient complained
become periodically larger and have mi- about toothache on the left of his upper
gratory nature. What is the most probable jaw that appears during eating, can be
diagnosis? caused by thermal stimuli (especially by
cold water) as well as by mechanical and
A. Glossitis areata exfoliativa chemical stimuli. The pain abates when
B. Median rhomboid glossitis the stimuli are eliminated. Objectively:
C. Lingua plicata there is a deep carious cavity with a
D. Glossotrichia narrow inlet within circumpulpar dentin.
E. Raspberry tongue Probing of carious cavity floor is painful.
Thermodiagnosis causes acute pain that
163. A 2 year old child was brought
to a dentist to consult about rash and abates immediately after stimulus elimi-
nation. Electric odontodiagnostics results
painfulness during eating that appeared
- 15 microampere. Make a diagnosis:
yesterday. Objectively: body temperature
is 37, 5oC, skin is clean, submaxillary
lymph nodes are painful. Examination
of oral cavity revealed on the mucous
Krok 2 Stomatology 2008 25

A. Acute deep caries A. Mandibulofacial dysostosis


B. Pulp hyperemia B. Oculocerebrorenal syndrome
C. Acute partial pulpitis C. Oculoauriculovertebral dysplasia
D. Acute median caries D. Oculomandibulofacial syndrome
E. Chronic simple pulpitis E. Oculodentodigital syndrome
167. After anesthetization a 55 year old 171. A 23 year old man complai-
patient felt sudden attack of weakness, ns about gingival haemorrhage duri-
pain behind his breastbone irradiating to ng tooth brushing, massive formation
his left arm and scapular area, palpitati- of dental deposit in spite of thorough
on. Objectively: the patient is conscious, oral hygiene. Objectively: gingival papi-
inert, his forehead is covered with cold llae are somewhat edematic, congesti-
sweat, the skin is pale, AP is 90/60 mm vely hyperemic, bleed when touched.
Hg, heart tones are dull, pulse is thready Fyodorov-Volodkina hygienic index is 3,5.
and arrhythmic. What state is developing? What toothpaste would you recommend
this patient as a part of complex therapy
A. Myocardium infarction of this disease?
B. Cardiogenic form of anaphylactic shock
C. Stenocardia attack A. Toothpaste with salts
D. Collapse B. Toothpaste with mineralizing
E. - components
C. Fluorine-containing toothpaste
168. During tooth extraction a 55 year D. Gel toothpaste with microelements
old patient felt dull pain behind her E. Toothpaste with antifungal agents
breastbone, got a sense of compressi-
on. A dental surgeon diagnosed her wi- 172. A dentist is filling a II class carious
th a stenocardia attack. What medication cavity (Black’s classification) in the 36
should be given this patient in order to tooth of a 35 year old patient by sandwi-
arrest this state? ch method. What glass-ionomer cement
should be chosen as a basic liner in this
A. Nitroglycerine, validol case?
B. Analgin
C. Baralgin A. Hybrid
D. Dimedrol B. Water-setting
E. Ketanov C. Classic
D. Condensable
169. A 58 year old patient applied to an E. Hardened
oral surgeon and complained about pai-
nful ulcer on the lateral surface of his 173. A 19 year old man complained about
tongue. Objectively: left lateral surface of gingival haemorrhage and painfulness.
tongue has a roundish ulcer with undermi- Objectively: in the frontal part of his
ned soft overhanging edges, palpatory pai- upper and lower jaws overcrowding of
nful, ulcer floor is slightly bleeding and teeth is present; there are hyperemia,
covered with yellowish nodules. What is cyanosis and enlargement of gingival
the most probable diagnosis? papillae up to 1/3 of crown height; massi-
ve formation of soft tooth deposit. What
A. Tuberculosis procedure should be included into the
B. Syphilis treatment program?
C. Traumatic ulcer
D. Actinomycosis A. D’arsonval currents
E. Trophic ulcer B. Diathermy
C. Microwave therapy
170. An oral surgeon attended a 3 D. Ultra-high frequency therapy
day old child staying in the newborn E. Fluctuating currents
pathology department. Objectively: bi-
lateral hypogenesis of zygomatic bones 174. During local anesthetization (with
and orbits, antimongoloid slant of 2 ml of 10% solution of lidocaine) a 9
palpebral fissures, nonclosure of inferi- year old girl cried out, lost consciousness,
or eyelids, hypogenesis of mandible (bird there appeared generalized convulsions.
face), deformity of auricles with a preauri- Objectively: the child’s skin is pale and
cular fistula on the right. What congenital cyanotic. It is impossible to feel the pulse
disease does the child have? because of convulsions. What is your
provisional diagnosis?
Krok 2 Stomatology 2008 26

A. Acute hematogenous osteomyelitis


A. Lidocaine intoxication B. Phlegmon of infraorbital area
B. Quincke’s edema C. Acute maxillary sinusitis
C. Anaphylactic shock D. Acute odontogenous osteomyelitis
D. Bronchial obstruction E. Acute purulent periostitis
E. Febrile convulsions
179. A 62 year old man complains about
175. A 60 year old patient has profuse a painless formation on his tongue that
alveolus haemorrhage after a simple appeared several months ago. Objecti-
operation of the 37 tooth extraction. It is vely: there is a big number of carious
known from the patient’s anamnesis that and completely decayed teeth, a painless
he had been suffering from hypertension whitish formation 10х5 mm large with
for the latest 6 years. Arterial pressure irregular surface in form of verrucas on
is 180/110 mm Hg. What emergency aid the lateral surface of tongue. Histologi-
should the doctor render? cal examination revealed thickening of
corneous layer of epithelium with intermi-
A. To introduce hypotensive medications ttent cornification. What is the most
and perform tight alveolar packing probable diagnosis?
B. To perform alveolar packing with a
hemostatic sponge A. Verrucous form of leukoplakia
C. To perform tight alveolar packing by B. Verrucous precancer
means of iodoform tampon C. Hyperplastic form of candidosis
D. To make suture ligation of the alveolus D. Hyperkeratous form of lichen ruber
E. To introduce hemostatics planus
E. Keratoacanthoma
176. A patient with a missile wound of
his face was delivered to the station of 180. A patient had an epileptic attack
dental aid group that was organized by during tooth preparation. What actions
order of medical corps commander. What should the doctor take?
kind of aid can be rendered by the dental
aid group? A. To lay the patient down with laterad
position of his head, to fix his tongue
A. Secondary care B. No actions should be taken
B. Professional care C. To seat the patient on an armchair in
C. Consultancy upright position
D. Dental health service D. To seat the patient with forward positi-
E. Dental orthopaedic service on of his head, to fix his hindhead
E. To lay the patient in prone, head-down
177. A 13 year old child complains about position, to fix his arms
acute spontaneous short-term attack-like
pain in the area of the 36 tooth that is 181. A 62 year patient was at a denti-
getting worse during eating. The pain st’s and suddenly there appeared dyspnea
appeared yesterday. Objectively: there is and hacking cough, sense of progressi-
a deep carious cavity on the masticatory ng asphyxia. The patient’s condition is
surface of the 36 tooth. Tooth cavity is getting progressively worse; number of
closed, floor probing is painful in one poi- respiratory movements reached 30/min,
nt. Cold stimulus causes short-term pain. acrocyanosis became more evident, there
Make a diagnosis: appeared gargling breathing, oral cavi-
ty excretes a lot of albuminoid sputum.
A. Acute localized pulpitis Psychomotor agitation and arterial
B. Acute deep caries hypertension are present. What pathologi-
C. Accute diffuse pulpitis cal state is it?
D. Chronic fibrous pulpitis
E. Acute condition of chronic pulpitis A. Pulmonary edema
B. An attack of bronchial asthma
178. Mother of a 2 month old child C. Epileptic attack
had acute purulent mastitis. After that D. Anaphylactic shock
the child got edema of left infraorbital E. Spontaneous pneumothorax
and zygomatic areas, skin hyperemia of
left face part, body temperature up to 182. A patient was delivered to an admi-
39 − 40o C, purulent discharges from the ssion room after a road accident. Exami-
nose. What is presumptive diagnosis? nation of the patient revealed in the area
of oral floor a large haematoma spreading
Krok 2 Stomatology 2008 27

to the neck area as well as a significant with arthrosis of mandibulotemporal joi-


edema of soft tissues of oral floor and nt. During auscultation the doctor heard
neck. There are signs of asphyxia. What pathological rustle in the area of the joi-
type of emergency care is indicated? nt, namely "clicking"that was caused by
a load. What is the mechanism of this
A. Tracheotomy performing pathological rustle?
B. Lobeline introduction
C. Tongue fixation A. Asynchronous movement of disc and
D. Opening and draining of haematoma articulation head
E. Symptomatic medicamental treatment B. Friction of bone surfaces in the joint
C. Loosening of mandibulotemporal joint
183. A 42 year old patient applied to ligaments
an oral surgeon for the purpose of oral D. Calcification of mandibulotemporal
cavity sanitation. After anesthetization joint disc
the patient felt sudden attack of nausea, E. Inflammation of cartilaginous tissue of
weakness, he got the sense of compressi- the disc
on behind his breastbone, heart pain; he
began vomiting. The patient lost consci- 187. A 23 year old patient complained
ousness, there appeared convulsions. about unpleasant sensations in the area of
Objectively: the patient’s skin is pale, the 12 tooth, X-ray picture shows a well-
covered with cold clammy sweat, pupi- defined low-density area up to 8-10 mm in
ls don’t react to the light. The pulse diameter around the root apex of the 12
is thready, arterial pressure cannot be tooth. What method of surgery will be the
detected. What is the most probable di- most reasonable?
agnosis?
A. Root apex resection
A. Anaphylactic shock B. Root amputation
B. Collapse C. Hemisection
C. Syncope D. Tooth removal
D. Traumatic shock E. Cystotomy
E. Epileptic attack
188. A 20 year old man got a blow to the
184. A 42 year old patient complains chin. He remained conscious. Objectively:
about a neoplasm in the anterior palatal his general condition is satisfactory. He
part that has been forming for 10 years. has an edema and haematoma in the area
The 13, 12, 11, 21, 22, 23 teeth are intact. of his lower jaw on both sides. Jaw palpati-
Intraoral X-ray picture of the upper on causes acute pain. Mouth opening is
jaw shows a single focus of bony tissue limited. There is also a rupture of mucous
destruction, well-defined, 2,5 by 1,5 cm membrane of oral cavity in the area of
large. Periodontal fissure in the area of the the 44, 43 and 34, 35 teeth. Occlusion
13, 12, 11, 21, 22, 23 teeth can be well seen. is deviated. What asphyxia type can be
What is the most probable diagnosis? developed?
A. Nasopalatine canal cyst A. Dislocation
B. Radicular cyst B. Obturation
C. Globulomaxillary cyst C. Stenotic
D. Residual cyst D. Valve
E. Follicular cyst E. Aspiration
185. At the stage of adjustment of an 189. A 3 year old child was burnt with
individual tray the Herbst test should be boiling water. Face skin is hyperemic and
done. What muscles cause the shift of the edematic, there are blisters of different si-
individual tray during functional testing - zes filled with transparent liquid. Define a
alternate touching right or left cheek with degree of face skin burn:
the tip of tongue?
A. II degree
A. Mylohyoid B. I degree
B. Digastric, geniohyoid C. III degree - A
C. Mentalis and orbicular muscle of mouth D. III degree - B
D. Mimic muscles E. IV degree
E. Mastication muscles
190. Examination of an 11 year old
186. A 37 year old patient was diagnosed girl revealed: caries intensity (sum of
Krok 2 Stomatology 2008 28

carious, filled and extracted teeth) = A. Intravenous introduction of 0,5 ml of


3, Green-Vermillion’s hygienic index - 0,1% adrenalin solution, prednisolone
1,6, papillary-marginally-alveolar index = B. Promedol, suprastine
20%. The girl catches a cold once or twi- C. A cotton-wool wad soaked in liquid
ce a year. Average caries intensity index ammonia brought to the patient’s nose
for this age group in this region is 4. How D. Lying the patient in horizontal position
many times a year should the girl see a E. Aminophylline introduction
dentist?
194. A 43 year old patient applied to
A. 1 a prosthetic dentistry for the purpose
B. 1-2 of prosthetics. In anamnesis: has been
C. 2 suffering from diabetes mellitus for 15
D. 2-3 years. The patient has been very anxi-
E. 3-4 ous before the appointment with a doctor.
Suddenly the patient’s condition grew
191. A patient was admitted to the oral worse, he felt limb weakness. The patient
surgery department with a diagnosis became covered with cold clammy sweat
"odontogenous phlegmon of mouth and lost consciousness. What critical state
floor". Objectively: general condition of is it?
the patient is grave, abed position is
forced, the patient is sitting with lowered A. Hypoglycemic coma
head. He complains about pain behind his B. Syncope
breastbone that is getting worse during C. Hyperglycemic coma
backward flexion of head; cough, dyspnea. D. Anaphylactic shock
What complication has developed? E. Myocardium infarction
A. Mediastinitis 195. A patient complains about worsened
B. Phlegmon of retropharyngeal space fixation and frequent breakages of parti-
C. Cervical phlegmon al removable lamellar denture for the
D. Phlegmon of tongue root lower jaw that has been in use for
E. Phlegmon of peripharyngeal space 5 years. Objectively: alveolar part in
edentulous areas is significantly atrophi-
192. The 47, 46, 45, 35, 36, 37, 38 teeth ed, the denture balances. What is the most
of a 57 year old patient are missing. It probable cause of worsened fixation and
is planned to make a clasp denture. The frequent breakages of the denture?
48 tooth inclines to the lingual side and
forwards. On the lingual side of the 48 A. Alveolar process atrophy
tooth the border line is diagonal, on the B. Improper care of denture
buccal side it runs on a level with gingival C. Denture using during sleep
edge. What type of Neja clasp should be D. Consumption of solid food
applied? E. Wear of artificial teeth
A. V type clasp 196. A 13 year old patient got a trauma
B. I type clasp in the area of median face zone. She
C. IV type clasp (reverse back-action) complains about pain, swelling of soft
D. I-II type clasp tissues in the area of her upper jaw,
E. II type clasp pain during mouth closing. Examination
revealed mobility of nose bones, signi-
193. A patient is waiting for metal-ceramic ficant swelling of soft tissues in the left
crowns for the 11, 12 teeth. Before zygomatic area, haemorrage in the eye
preparation conduction anesthesia was sclera, "step"sign along the inferior edge
performed. A few minutes later the pati- of both orbits and zygomaticomaxillary
ent felt anxiety, ear noise, abdominal pain, sutures, nasal haemorrhage, open bite,
dizziness. Tachypnoea and rapid pulse are lengthening of median face part. Make
present. Arterial pressure is 60/40 mm Hg. a clinical diagnosis:
What kind of first aid should be rendered?
A. Le Fort’s III fracture of upper jaw
B. Le Fort’s II fracture of upper jaw
C. Le Fort’s I fracture of upper jaw
D. Fracture of nose bones
E. Fracture of skull base
197. A patient with unilateral dislocation
Krok 2 Stomatology 2008 29

of mandibulotemporal joint was delivered 199. A 25 year old patient applied to a


to the maxillofacial department. What clinic for the purpose of oral cavity sani-
type of local anesthesia is indicated for tation. During preparation of the 45 tooth
relaxation of masticatory muscles during on account of chronic median caries the
diaplasis? patient turned pale, there appeared cold
clammy sweat on her forehead, nausea,
A. Subzygomatic Bersche-Dubov- ear noise. The patient lost consciousness.
Uvarov’s anesthesia Objectively: pulse - 50 bpm, AP - 80/60
B. Intraoral mandibular anesthesia mm Hg, shallow breath, miotic pupils.
C. Extraoral mandibular anesthesia, Make a diagnosis of this state:
submandibular way
D. Extraoral mandibular anesthesia, A. Syncope
retromandibular way B. Anaphylactic shock
E. Torus anesthesia C. Stenocardia
D. Collapse
198. A patient got an injury and complai- E. Quincke’s edema
ns about limited mouth opening, nasal
haemorrhage, skin numbness in the 200. A 64 year old patient was wai-
infraorbital and inferior eyelid area. ting to see an oral surgeon. Suddenly
Objectively: there is face deformity due to she fell, there appeared coarse breathi-
retraction of soft tissues in the zygomatic ng, spasmatic twitching of upper and
area on the left, a "step"symptom in lower extremities. Face and neck became
the median part of the left inferior edge cyanotic, pupils became mydriatic, reacti-
of eyesocket as well as in the area of on to the light was absent, arteri-
zygomaticoalveolar crest. What exami- al pressure and pulse could not be
nation method should be applied for di- detected, heart tones could not be heard,
agnostics of this disease? involuntary urination was observed. What
state are these symptoms typical for?
A. Roentgenography of facial cranium in
the axial projection A. Clinical death
B. Roentgenography of facial cranium in B. Loss of consciousness
the frontal projection C. Stroke
C. Roentgenography of facial cranium in D. Myocardium infarction
the lateral projection E. Collapse
D. Roentgenography of facial cranium and
paranasal sinuses
E. Orthopantomogram
Krok 2 Stomatology 2009 1

1. A 45-year-old female patient A. Forming a flat floor


complains about pain caused by thermal B. Forming divergent walls
srtimuli and spontaneous pain in the 26 C. Forming convergent walls
tooth. A week ago this tooth was treated D. Forming a bevel
for pulpitis. Objectively: the 26 tooth is E. Absence of an additional cavity
filled, percussion is painful, thermal sti-
mulus causes slowly intensifying long- 5. Parents of a 9-year-old boy complain
lasting pain. X-ray picture shows that the about permanently open mouth of the
palatine canal is filled by 2/3, buccal canals child. External examination revealed
contain no filling material. What is the elongation of the lower face part, non-
most probable cause of this complication? closure of lips. Examination of the oral
cavity revealed early mixed dentiti-
A. Incomplete pulp extirpation on. Relationship of the first permanent
B. Inadequate canal filling molars is neutral, vertical space is 5 mm.
C. Infection What is the most likely diagnosis?
D. Inflammation in the periodontium
E. Trauma of periodontium tissue A. Open bite
B. Distal occlusion
2. Preventive examination of a 7-year- C. Mesial occlusion
old schoolboy revealed unremovable D. Deep overbite
grey-and-white layerings on the mucous E. Cross bite
membrane of cheek along the line of
teeth joining. Mucous membrane is sli- 6. After removal of dental deposit an 18-
ghtly hyperaemic, painless on palpation. year-old patient underwent preventive
The boy is emotionally unbalanced, bi- examination. It revealed painless chalky
tes his cheeks. What is the most likely di- spots in the precervical area on the vesti-
agnosis? bular surface of the 22 and 41 teeth.
Result of enamel resistance test is 7. What
A. Mild leukoplakia morphological changes are typical for this
B. Chronic recurrernt aphthous stomatitis disease?
C. Chronic candidous stomatitis
D. Lichen ruber planus A. Subsurface enamel demineralization
E. Multiform exudative erythema B. Changes in the mantle dentine
C. Damage of dentinoenamel junction
3. A 48-year-old patient complains about D. Superficial enamel demineralization
itching gums. Objectively: gums are E. Degeneratic changes of odontoblasts
dense, of normal colour; cold water
causes short-term toothache. X-ray pi- 7. A 32-year-old patient complains about
cture shows osteosclerosis of interalveolar gingival haemorrhages during eating and
septa (fine granular bone texture), hei- tooth brushing. It is known from the
ght of interalveolar septa and integrity anamnesis that 2 weeks ago the patient
of compact substance on their tops is got a cast metal crown. Objectively: the 27
unchanged. Which diagnosis is the most tooth has a cast metal crown going beyond
likely? the gingival edge by 1-1,5 mm, touching
causes haemorrhage. Percussion is slightly
A. Initial parodontosis painful. What tactics is the most appropri-
B. Atrophic gingivitis ate in this case?
C. Initial periodontitis
D. I degree periodontitis A. To remove the crown and fabricate a
E. I degree parodontosis splint
B. To administer gargling with Rotocan
4. A 26-year-old patient needs an inlay for C. To open the crown through the masti-
a class V cavity of the 35 tooth. During catory surface
the cavity preparation a prosthodontist D. To give the patient time to get used to
has accidentally opened the pulp. What the prosthesis
might be the most probable cause of this E. To remove the crown and administer
complication? treatment for marginal periodontitis
8. A 50-year-old patient complains about
missing of the 37, 36, 45, 46, 47 teeth.
The 35, 38, 44, 48 teeth are intact and
stable, their crowns exhibit anatomic
form, sufficient height, relative paralleli-
Krok 2 Stomatology 2009 2

sm. Fulcrum line typically goes through A. Method of transfer molding


the 35 and 44 abutment teeth. What fixing B. Direct method of compression molding
element should be used in a clasp denture C. Inverse method of compression molding
supported by the 35, 44 teeth? D. Combined method of compression
molding
A. Acker clasp E. In water under pressure
B. Roach clasp
C. Jackson clasp 12. A 20-year-old patient complains about
D. Telescopic fixation feeling spontaneous undurable localised
E. Bent wire clasp pain in the 26 tooth for one day. Objecti-
vely: the 26 tooth has a deep Black’s class
9. A 25-year-old patient complains about I carious cavity. Dentine of its walls and
a cosmetic defect in the frontal part of floor is unpigmented, softened, carious
his upper jaw on the left. Objectively: the cavity is not communicating with the tooth
23 tooth is pulpless. The filling restores cavity. Probing causes acute pain focused
vestibular-approximal surface of the tooth at a single point, cold test induces long-
crown by 1/3, it is stable, closely fits wi- lasting pain. What method of treatment is
thin cavity walls, its colour doesn’t match the most appropriate in this case?
the colour of the tooth. The tooth itself
has changed its colour, its percussion is A. Biological method
painless. What is the optimal prosthetic B. Vital amputation
construction for this patient? C. Vital extirpation
D. Devital extirpation
A. Metal-ceramic crown E. Devital amputation
B. Elastic crown
C. Metal swaged crown 13. A 42-year-old patient complains
D. Portion crown about acute pain in the region of the
E. Inlay left temporomandibular joint (TMJ) that
irradiates to the ear; headache, general
10. A 12-year-old patient complains about indisposition, impossible mastication
gingival haemorrhage and tooth mobility. and limited mouth opening. Objecti-
He has been suffering from this since the vely: the patient’s face is asymmetric
age of 4. Objectively: gums around all the due to the edema in the region of the
teeth are hyperemic and edematic, bleed left temporomandibular joint. The skin
during instrumental examination. Tooth in this region is hyperemic. The pain is
roots are exposed by 1/3 and covered with made worse by the smallest movements
whitish deposit. II degree tooth mobility of mandible. Palpation of the joint causes
is present. Dentogingival pouches are 4-5 acute pain. Mouth opening is limited
mm deep. External examination revealed down to 15-20 mm. What is the most li-
dryness and thickening of superficial skin kely diagnosis?
layer on the hands and feet, there are also
some cracks. What is the most likely di- A. Acute arthritis of the left TMJ
agnosis? B. Acute purulent parotitis
C. Mandible subluxation
A. Papillon-Lefevre syndrome D. Deforming arthrosis of the left TMJ
B. Hand-Schuller-Christian disease E. Myogenous osteoarthrosis
C. Generalized periodontitis
D. Letterer-Siwe disease 14. A 42-year-old patient complains about
E. Localized periodontitis pain in the submandibular and sublingual
region that is getting worse during eati-
11. A 39-year-old patient needs a clasp ng; body temperature rise up to 37, 6o C.
denture with porcelain teeth for the lower The patient has been suffering from this
jaw. What method should be applied whi- for 2 months. Objectively: along the ri-
le substituting wax with plastic in order ght sublingual plica there is infiltration,
to avoid mold flash that may cause bite hyperaemia, edema of soft tisues, acute
opening? pain on palpation. The duct of the right
submandibular salivary gland discharges
turbid saliva with pus admixtures. What is
the most likely diagnosis?
Krok 2 Stomatology 2009 3

A. Exacerbation of urolithiasis A. Occlusiography


B. Acute purulent lymphadenitis of B. Masticatiography
submandibular region C. Chewing test
C. Adenophlegmon of submandibular D. Roentgenography
region E. Gnathodynamometry
D. Abscess of alveololingual groove
E. Retention cyst of sublingual salivary 18. During extraction of the 47 tooth its
gland distal root was broken halfway along its
length. What tool should be chosen for
15. A 4-year-old girl presents with body extraction of the residual root fragments?
temperature rise, aggravation of general
condition. The symptoms has been A. Left angled elevator
observed for 3 days. Objectively: general B. Broad-beaked forceps
condition is grave, body temperature is C. Close-beaked forceps
38, 6oC, the girl is anxious and pale. She D. Right angled elevator
presents also with halitosis, hyperaemia E. Straight elevator
and edema of gingival mucous membrane
in the region of the 83, 84, 85 teeth on 19. A 31-year-old patient consulted a
both sides from the alveolar process. dentist about teeth sanitation. Objecti-
The mentioned teeth are mobile, their vely: in the precervical region of the 13
percussion causes acute pain; the 84 tooth and 23 teeth there are irregularly-shaped
is filled. What is the most likely diagnosis? defects of hard tissues within deep layers
of enamel. The floor of the defects is
A. Acute odontogenous mandibular rough, their edges are chalky. What tactics
osteomyelitis beginning from the 84 tooth should be chosen as to these defects?
B. Acute sialoadenitis of submandibular
salivary gland A. Preparation and filling
C. Exacerbation of chronic periodontitis of B. Filling with composite material without
the 84 tooth preparation
D. Suppuration of the radiculodental C. Remineralizing therapy
mandibular cyst beginning from the 84 D. Grinding of the defects
tooth E. Silvering of the defects
E. Acute odontogenous mandibular peri- 20. A 51-year-old female patient complai-
ostitis beginning from the 84 tooth ns about food sticking in a right inferi-
16. A 38-year-old female patient came to or tooth. Objectively: distal masticatory
a hospital and complained about a wound surface of the 45 tooth has a deep cari-
on her left cheek. She got this trauma 16- ous cavity filled with dense pigmented
18 hours ago, didn’t loose consciousness. dentin that doesn’t communicate with the
Objectively: a 3 cm long injury of skin, tooth cavity. The patient was diagnosed
subcutaneous fat and muscle. The wound with chronic deep caries. What method of
is bleeding. What kind of initial surgical examination allowed the dentist to elimi-
d-bridement should be performed? nate chronic periodontitis?

A. Early surgical d-bridement A. Electro-odontometry


B. Delayed surgical d-bridement B. Probing
C. Secondary surgical d-bridement C. Palpation of projection of root apex
D. Initial surgical d-bridement along with D. Percussion
plasty E. Cold test
E. - 21. An 18-year-old patient complains
17. A 47-year-old patient consulted a about body temperature rise, weakness,
dentist about dental cervix exposure on pain induced by eating and deglutition.
both jaws. Objectively: the dentitions are Objectively: mucous membrane of the
intact, the dental cervixes are exposed. oral cavity is erythematic with multi-
Untimely teeth contacts are present. It ple petechia. Pharynx is hyperaemic.
is planned to perform selective grindi- Regional lymph nodes are enlarged,
ng. What controlling method should be mobile, painless. In blood: leukocytosis,
chosen? monocytosis, atypic mononuclears, ESR
is 30 mm/h. What is the leading factor of
disease development?
Krok 2 Stomatology 2009 4

A. Viral infection Objectively: cheek is swollen in the regi-


B. Bacterial infection on of mandible on the left, palpation
C. Autoimmune disorders revealed induration of soft tissues. In
D. Immediate allergy projection of the 35, 36 teeth there is a
E. Delayed allergy fistula containing pus and granulations.
X-ray picture shows destruction of bone,
22. A female patient consulted a sequestrum. What is the most likely di-
stomatologist about inability to close her agnosis?
mouth, speech difficulty. Objectively: the
oral cavity is half-open, there is drooling, A. Chronic osteomyelitis
central line deviates to the right. There B. Actinomycosis
is a depression in front of the tragus, C. Syphilis
and above the malar arch the mandibular D. Sarcoma
condyle bulges inward the infratemporal E. Osteoma
fossa. What is the most likely diagnosis?
26. A 23-year-old patient complains about
A. Unilateral anterior dislocation of a small ulcer on the red border of her
mandible on the left lower lip that has been irresponsive to
B. Unilateral anterior dislocation of self-treatment for two weeks. Objectively:
mandible on the right unchanged red border of lower lip has a
C. Unilateral posterior dislocation of circular ulcer of 2 mm in diameter with
mandible on the left raised regular edges, its floor is of meat-
D. Unilateral posterior dislocation of like colour, dense, shiny, with "stearic fi-
mandible on the right lm", with cartilaginoid infiltration, pai-
E. Bilateral anterior dislocation nless on palpation. Regional lymph node
is enlarged, of tight elastic consistency,
23. Analysis of a 10-year-old boy’s jaw painless, mobile. What is the most likely
models revealed that occlusal plane of diagnosis?
the frontal maxillary teeth was of concave
form, its lateral parts were convex. Form A. Primary syphilis
of the alveolar process also represents B. Decubital ulcer
deformation of dental arches. The upper C. Cancerous ulcer
jaw is of saddle-like form with abrupt D. Lupus erythematosus
narrowing in the region of premolar teeth. E. Tuberculous ulcer
What type of bite is it?
27. X-ray picture depicts a circular well-
A. Open defined area of bone tissue destruction
B. Distal 0,7х0,7 cm large in the projection of root
C. Deep apex. What is the most likely diagnosis?
D. Mesial
E. Cross A. Cystogranuloma
B. Cyst
24. A 50-year-old patient complains about C. Granuloma
increased sensibility of the exposed tooth D. Odontoma
cervixes, displacement of teeth, gum itch, E. Osteoma
pain in the region of the 43, 42, 41,
31, 32, 33 teeth caused by chemical and 28. A 29-year-old patient complains about
thermal stimuli. Objectively: the gums are acute attack-like pain in the region of
dense and anemic. X-ray picture shows his upper jaw on the left, as well as in
atrophy of the alveolar bone reaching 2/3 the region of his left maxillary sinus, eye
of interalveolar septa height. What is the and temple. The pain is long-lasting (2-3
most likely diagnosis? hours), it is getting worse at night. The
patient has a history of recent acute respi-
A. III degree parodontosis ratory disease. Objectively: the 26 tooth
B. II degree parodontosis has a carious cavity, floor probing is pai-
C. Chronic generalized III degree nful, thermal stimuli cause long-lasting
parodontosis pain, percussion causes slight pain. What
D. Chronic generalized II degree is the most likely diagnosis?
parodontosis
E. Atrophic gingivitis
25. A 32-year-old patient complains of
a fistula in the submandibular region.
Krok 2 Stomatology 2009 5

A. Acute diffuse pulpitis old child revealed untimely missing of


B. Acute focal pulpitis all the upper molars. The lower incisors
C. Acute apical periodontitis contact with mucous membrane of palate.
D. Inflammation of maxillary sinus What is the tactics of choice?
E. Exacerbation of chronic periodontitis
A. Fabrication of a removable lamellar
29. A 45-year-old patient complains about prosthesis
pain in his mandible that arose after B. Fabrication of an orthodontic appliance
extraction of the 36 tooth. Objectively: for deep overbite correction
alveolar socket is covered with bloody C. Annual examination till cutting of the
clot. X-ray picture shows unextracted root permanent teeth
of the 36 tooth. What tools are necessary D. Half-yearly examination till cutting of
for extraction of this root? the permanent teeth
E. Medical intervention is not required
A. Angled elevator curved right
B. Angled elevator curved left 33. A 20-year-old patient complains about
C. Straight elevator a carious cavity in an upper right tooth.
D. S-shaped forceps Objectively: the 16 tooth has a deep cari-
E. Bayonet-shaped forceps ous cavity communicating with the tooth
cavity, probing at the opening point is pai-
30. A patient complains of pain and swelli- nless, percussion of the 16 causes mild
ng in the right submandibular area. She pain. There is a fistula on the gingiva in
has been treating the 45 tooth for a week. the region of root apex projection of the
Objectively: body temperature is 38oC. 16 tooth. What is the most probable di-
There is a painful tense infiltration in agnosis?
the right submandibular region. The skin
doesn’t make a fold, its hyperemic and A. Chronic granulating periodontitis
glossy. The mouth can be opened by 3 cm. B. Chronic fibrous periodontitis
Deglutition is painless. These clinical fi- C. Chronic granulomatous periodontitis
ndings correspond with the following di- D. Chronic hypertrophic pulpitis
sease: E. Chronic gangrenous pulpitis
A. Odontogenous phlegmon of the right 34. A 16-year-old patient complains about
submandibular region a cosmetic defect in form of white spots in
B. Abscess of the right alveololingual the region of the upper frontal teeth. The
groove defect was revealed long ago and doesn’t
C. Adenophlegmon of the right submandi- change with time. Objectively: white spots
bular region on the vestibular surfaces of the 11, 12, 21,
D. Acute odontogenous sialoadenitis 22 teeth close to the cutting edge and on
E. Phlegmon of pterygomandibular space the vestibular surfaces of the 16, 26, 36, 46
teeth close to the masticatory surface. On
31. A 5-year-old boy presents with body probing the spot surface was smooth, pai-
temperature rise up to 39, 2oC, sore nless; cold stimulus produced no pain. The
throat, headache, nausea. Objectively: spots couldn’t be stained with 2% solution
mucous membrane of soft palate and of methylene blue. What is the most likely
palatine arches is evidently hyperemic, diagnosis?
there is distinct border between it and
surrounding tissues. The child’s tongue is A. Systemic enamel hypoplasia
dry, edematic, its lateral surfaces are red B. Local enamel hypoplasia
and free of fur, fungiform papillae are evi- C. Acute initial caries
dently enlarged. Face skin is covered wi- D. Fluorosis in form of spots
th spotty rash, hyperemic except for pale E. Erosion of dental solid tissues
nasolabial triangle. Submandibular lymph
nodes are painful on palpation. What is 35. A 35-year-old patient complains about
the causative agent of this disease? a cavity in a lower jaw tooth on the left,
and the pain caused by sweet, sour and
A. Haemolytic streptococcus solid food. Examination of the 36 tooth
B. Coxsackie virus revealed a deep carious cavity filled with
C. Herpes virus light softened dentine. Probing of the cari-
D. Bordet-Gengou bacillus ous cavity floor is painful, reaction to the
E. Loffler’s bacillus cold stimulus is painful, undurable. What
is the most likely diagnosis?
32. Preventive examination of a 4,5-year-
Krok 2 Stomatology 2009 6

A. Acute deep caries 47 teeth are missing; there is postoperati-


B. Acute focal pulpitis ve midline defect of hard palate. It was
C. Chronic fibrous pulpitis decided to make a clasp dental prosthesis
D. Acute median caries with obturating part. The obturating part
E. Chronic deep caries should be placed on the following element
of the clasp dental prosthesis:
36. A 31-year-old male patient complai-
ns of dryness and burning of tongue A. On the arch
back that appeared for about a week B. On the saddle
ago and get worse when he eats irritati- C. On the artificial teeth
ng food. The patient has a history of D. On the base
recent pneumonia. He had been treated E. On the clasps
in the in-patient hospital for 2 weeks, the
treatment program included antibiotics. 40. A 10-year-old boy consulted a denti-
Now he doesn’t take any drugs. Objecti- st about pain in the palate during eating.
vely: mucous membrane of the oral cavity Objectively: the lower third of his face is
is hyperemic, dry, glossy. Tongue back and shortened, mouth opening is not limited.
palate have greyish-white plicae that can By joining the teeth the cutting edge of
be easily removed. Threads of saliva trail inferior incisors contacts with the mucous
behind the spatula. What is the most likely membrane of palate. Mucous membrane
diagnosis? in the contact point is hyperemic, slightly
edematic. Lateral teeth exhibit Angle’s
A. Acute pseudomembranous candidiasis class I malocclusion. What is the most
B. Chronic hyperplastic candidiasis appropriate plan of treatment of the lower
C. Acute atrophic candidiasis jaw?
D. Medicamental stomatitis
E. Chronic atrophic candidiasis A. To impact frontal part
B. To impact lateral parts
37. A 53-year-old patient consulted a C. To widen the lower jaw
prosthodontist about lateral teeth mobili- D. To protract frontal part
ty, frequent falling out of fillings. Objecti- E. To protract lateral parts
vely: all the molars and premolars of
the lower jaw exhibit I degree mobility. 41. A patient consulted an oral surgeon
Approximal masticatory surfaces have fi- about pain in the region of his left upper
llings. What splint construction should be jaw that appeared 3 days ago. After
applied in this case? examination the patient was diagnosed
with exacerbation of chronic periodontitis
A. Inlay splint of the 17 tooth. It is indicated to extract
B. Fixed crown splint the 17 tooth. What nerves should be
C. Crown cap splint blocked for painless extraction of the 17
D. Equator crown splint tooth?
E. Intradental splint
A. Posterior alveolar nerves and greater
38. A 28-year-old patient consulted a palatine nerve
prosthodontist about dental prostheti- B. Greater palatine nerve
cs. Objectively: crown of the 13 tooth is C. Anterior alveolar nerves and incisor
decayed by 2/3 of its height, the stump sti- nerve
cks out above the gum line by 2 mm. X-ray D. Median alveolar nerves and greater
picture shows that the root canal is filled palatine nerve
up to the top. What construction should E. Nasopalatine nerve
be fabricated for the patient?
42. A 40-year-old patient complains about
A. Pivot tooth frequent falling out of a filling. Objecti-
B. Crown vely: the 46 tooth has a carious (Black’s
C. Inlay class II) cavity. It is planned to restore the
D. Equator crown anatomic form of the tooth by means of
E. The crown should be restored with a metal inlay. What is the peculiarity of
photopolymer the cavity preparation for the inlay in this
case?
39. A 65-year-old patient complains about
partially missing teeth on his upper jaw,
difficult mastication, rhinolalia. Objecti-
vely: the 18, 16, 15, 11, 23, 28, 35, 38, 48,
Krok 2 Stomatology 2009 7

A. Forming a bevel
B. Forming an additional shoulder A. Exacerbation of chronic granulating
C. Cavity floor widening periodontitis
D. Preparation for parapulpar posts B. Acute purulent pulpitis complicated by
E. Cavity deepening periodontitis
C. Exacarbation of chronic pulpitis
43. A 10-year-old child complains about D. Exacerbation of chronic granulomatous
acute spontaneous spasmodic pain in an periodontitis
upper jaw tooth on the left. Objectively: E. Acute serous periodontitis
distal contact surface of the 26 tooth
exhibits a carious cavity filled with li- 47. A patient undergoes orthopaedic
ght softened dentine and localized within treatment of bounded edentulous spaces
parapulpar dentine. Probing of the cavity on the upper jaw. He needs fixed full-
floor causes acute pain, percussion is pai- cast dentures. During his second visit it
nless. Cold stimuli cause a long-standing is required to check whether the internal
pain attack. The child has a history of li- surface of the metal framework of the
docaine allergy. Choose an optimal paste future metal-ceramic denture matches the
to be used during the first visit: surfaces of the prepared teeth. In what
way could this be done?
A. Paraformaldehyde
B. Iodoform A. In the oral cavity by means of silicone
C. Thymol materials
D. Formocresol B. Visually by means of models in the
E. Zinc oxide eugenol articulator
C. In the oral cavity by means of tracing
44. A 35-year-old patient complains about paper
itch, burning and edema of lips. He has D. In the oral cavity by means of a wax
been suffering from this for a week. plate
Objectively: reddening of red border and E. In the oral cavity by means of
skin, especially in the region of mouth stomatoscopic method
corners, there are also vesicles, crusts,
small cracks along with erythematous 48. An 18-year-old female patient
affection of red border. What is the most consulted a dentist about dental prostheti-
likely diagnosis? cs. Objectively: the 21 tooth is pulpless,
of dark-grey colour. The patient has
A. Acute eczematous cheilitis orthognathic occlusion. It is planned to
B. Multiform exudative erythema crown the tooth with a plastic crown.
C. Acute herpetic cheilitis Choose the plastic mass for its fabrication:
D. Allergic contact cheilitis
E. Exudative form of exfoliative cheilitis A. Sinma
B. Ethacryl
45. Preventive examination of an 8-year- C. Acrel
old boy revealed some lusterless chalk- D. Ftorax
like spots on the vestibular surface of the E. Bakril
11 and 21 teeth, which are localised in the
precervical region. Subjective complaints 49. A 47-year-old patient complains about
are absent. What is the most likely di- permanent pain in the 27 tooth that
agnosis? is getting worse when biting down on
food. Objectively: the patient’s face is
A. Acute initial caries symmetric, skin is of normal colouring,
B. White-spotted fluorosis the mouth can be fully opened, mucous
C. Local enamel hypoplasia membrane of the alveolar process is
D. Acute superficial caries edematic and hyperemic at a level of the
E. Chronic initial caries 27 tooth. The 27 tooth has a deep carious
cavity interconnecting with pulp chamber.
46. A 27- year-old patient complains Percussion of the 27 tooth causes acute
about acute pain in the region of the pain. What is the most likely diagnosis?
34 tooth that is getting worse when bi-
ting down on food. Roentgenographical
survey revealed an ill-defined zone of
bone tissue destruction in the periapical
region of root of the 34 tooth. What is the
most likely diagnosis?
Krok 2 Stomatology 2009 8

A. Exacerbated chronic periodontitis of A. Chronic lymphadenitis


the 27 tooth B. Chronic sialoadenitis
B. Chronic periodontitis of the 27 tooth C. Sialolithiasis
C. Acute generalized purulent pulpitis of D. Adenoma of salivary gland
the 27 tooth E. Atheroma
D. Acute purulent periostitis of the upper
jaw beginning from the 27 tooth 53. A 38-year-old patient ordered a
E. Chronic left-sided odontogenous hi- clasp upper jaw prosthesis. Its fabricati-
ghmoritis on involves forming of a fireproof model.
What material should be applied for
50. A 64-year-old female patient was wai- duplication?
ting for her turn at the dentist’s. Suddenly
she fell down, her respiration became A. Gelin
hoarse, she got convulsive twitching in B. Stomatoplast
her upper and lower limbs, face and neck C. Stomalgin
turned cyanotic, eye pupils became mydri- D. Orthocor
atic, reaction of eye to light was absent. E. Dentafol
Arterial pressure and pulse couldn’t be
measured. Heart sounds couldn’t be 54. A 35-year-old patient needs a metal
auscultated. Involuntary urination was inlay for the 37th tooth. Objectively:
noted. What condition is characterized by masticatory surface of the 37 tooth has
such symptoms? a carious cavity. What is the peculiarity of
tooth preparation?
A. Clinical death
B. Epilepsy A. Forming a bevel
C. Shock B. Forming an additional shoulder
D. Coma C. Widening of cavity floor
E. Collapse D. Forming flat floor
E. Forming additional cavity
51. A 3-year-old child complains of a cavi-
ty in a lower jaw tooth. Objectively: masti- 55. A 64-year-old patient complains about
catory surface of the 84 tooth exhibits a slightly painful ulcer on the lateral
a carious cavity within mantle dentine. surface of his tongue that appeared over 2
Dentine is softened, probing of the cavi- months ago. Objectively: lateral surface of
ty walls is painful, percussion is painless. tongue has an ulcer with raised edges, ti-
What is the most likely diagnosis? ssues around it are infiltrated. Submandi-
bular lymph nodes are enlarged, adheri-
A. Acute median caries ng to each other and surrounding tissues.
B. Chronic superficial caries What is your provisional diagnosis?
C. Chronic fibrous pulpitis
D. Acute deep caries A. Lingual cancer
E. Chronic deep caries B. Syphilis (hard chancre)
C. Tuberculous ulcer
52. A 43-year-old patient complains D. Decubital ulcer
about a neoplasm in the right submandi- E. Ulcero-necrotic lesion associated with
bular region that appeared a month ago blood disorders
after angina. Body temperature is up to
37, 0 − 37, 2oC. The patient underwent 56. A 56-year-old patient complains about
anti-inflammatory therapy but it led to no missing lateral teeth on both sides of
reduction of the neoplasm. Objectively: in mandible. Objectively: the 48, 47, 46, 45,
the right submandibular region palpati- 35, 36, 37, 38 teeth are missing. Make a di-
on reveals a slightly painful, spherical, agnosis:
freely movable, well-defined neoplasm of A. Kennedy I denttion defect
dense and elastic consistency. The duct of B. Kennedy IV denttion defect
submandibular salivary gland discharges C. Kennedy II denttion defect
transparent saliva. The sublingual plica is D. Kennedy III denttion defect
unchanged. What is the most likely di- E. Betel’man II denttion defect
agnosis?
57. A 4-year-old child got a face trauma 2
hours ago. A stomatologist on duty made
a diagnosis: intrusive luxation of the 61
tooth. What is the tactics of choice?
Krok 2 Stomatology 2009 9

A. Chronic deep caries


A. Extraction of the 61 tooth B. Chronic fibrous periodontitis
B. Observation C. Chronic fibrous pulpitis
C. Reposition of the 61 tooth D. Chronic median caries
D. Splinting of the 61 tooth E. Chronic granulomatous periodontitis
E. Removal of pulp of the 61 tooth
61. A 7-year-old child complains about
58. An 18-year-old student complains spontaneous pain in the upper right
of roughness and dryness of mucous molars. Both medial contact and masti-
membrane of the oral cavity. He always bi- catory surfaces of the 55 tooth have a
tes uncontrolledly off mucous membrane carious cavitiy filled with clear softened
along the line of teeth joining. The pati- dentin and localized within parapulpar
ent suffers from chronic cholecystitis, he dentin. Floor probing causes acute pain,
is smoker. Hygiene index is 2,3. Objecti- tooth percussion is slightly painful.
vely: mucous membrane along the line of Mucous membrane of the alveolar process
teeth joining is quaggy, edematic, whitish. in the root projection of the 55 is intact,
What is the leading factor of this disease examination of this region revealed no
development? roentgenological changes. What is the
most likely diagnosis?
A. Habitual biting of mucous membrane
of the oral cavity A. Acute diffuse pulpitis
B. Unsatisfactory level of oral cavity B. Chronic fibrous pulpitis
hygiene C. Chronic gangrenous pulpitis
C. Somatic pathology D. Exacerbation of chronic periodontitis
D. Xerostomia E. Deep acute caries
E. Smoking
62. A 55-year-old patient has a repeated
59. A 57-year-old patient complains about appointment with a dentist. He has
dryness and burning of the lateral surface ordered complete removable lamellar
of her tongue. These sensations disappear dentures for both upper and lower jaw.
during eating. She noted such sensations During his previous visit the dentist
three months ago. She has a history of took complete anatomic impressions of
gastritis with reduced secretory function. both jaws and sent them to the dental
Objectively: mucous membrane of tongue mechanic. What manipulations should the
and oral cavity has no peculiarities. The dentist perform at the next clinical stage?
back of tongue has thin white coating.
Regional lymph nodes are unpalpable. A. Adjust individual trays
Oral cavity is sanitized. What is the most B. Locate and fix central occlusion
likely diagnosis? C. Check the prosthesis construction
within the oral cavity
A. Glossodynia D. Correct the prosthesis and give the
B. Lingual nerve neuritis patient necessary instructions
C. Candidiasis E. Fabricate occlusal rims
D. Desquamative glossitis
E. Hunter-Moeller glossitis 63. A 33-year-old patient complai-
ns about pain, gingival haemorrhage,
60. A 23-year-old patient complains halitosis, body temperature rise up
about a carious cavity in the 15 tooth to 37, 8oC. Objectively: gums are
that he noted a year ago. Examinati- apparently hyperaemic, edematic, bleed
on revealed that approximal surface had easily, parodontal pouches are 3-
a deep carious cavity not communi- 4 mm deep, and contain purulent
cating with the tooth cavity and fi- exudate. Orthopantomogram shows di-
lled with dense pigmented dentine. ffuse osteoporosis of alveolar process,
Probing, cold test, percussion cause no resorption of interdental septa down to
pain. Electroodontodiagnosis is 10 mi- 1/3 of their height. What is the most likely
croampere. What is the most likely di- diagnosis?
agnosis?
Krok 2 Stomatology 2009 10

A. Exacerbation of chronic generalized I 67. A 69-year-old patient needs extraction


degree periodontitis of the 12, 11 teeth. He is diagnosed with
B. Chronic generalized I degree peri- generalized periodontitis, the 12 and 11
odontitis teeth exhibit II degree mobility. Choose a
C. Exacerbation of chronic generalized II proper instrument for extraction:
degree periodontitis
D. Chronic generalized II degree peri- A. Straight forceps
odontitis B. S-shaped forceps
E. Exacerbation of chronic catarrhal gingi- C. S-shaped forceps curved right
vitis D. Root bayonet-shaped forceps
E. Crown bayonet-shaped forceps
64. An 8-year-old boy complains about
acute short-term pain in the 11 tooth 68. An 8-year-old child presents with an
caused by cold and sweet stimuli. Objecti- edema of the submandibular region, the
vely: medial contact surface of the 11 mouth can be opened by 1,5 cm, further
tooth has a carious cavity within denti- opening is difficult, body temperature is
noenamel junction filled with softened 37, 6oC, mucogingival fold is vestibularly
moist dentine that can be easily removed. flattened, hyperaemic and swollen. The 84
Enamel edges are white, undermined, and 85 teeth have fillings, their percussi-
friable. Superficial probing is painless, on is painless. The 84 tooth exhibits I
thermal test is positive, percussion is degree mobility. What is the most likely
negative. What is the filling material of diagnosis?
choice?
A. Acute odontogenic periostitis
A. Glass ionomer cement B. Acute odontogenic osteomyelitis
B. Silicophosphate cement C. Chronic odontogenic periostitis
C. Composite material D. Chronic odontogenic osteomyelitis
D. Silver amalgam E. Exacerbation of chronic periodontitis
E. Silicate cement
69. A 25-year-old patient has a median
65. A 40-year-old patient complains about fracture of mandible without evident di-
body temperature rise up to 38o C, and splacement of fragments. All the teeth are
of a roundish infiltration on his upper lip. intact. What kind of wire splint is the most
Objectively: there is a roundish infiltration appropriate in this case?
on the upper lip on the left, the skin above
it is wine red, the infiltration adheres to A. Smooth splint
the surrounding tissues and has a seapus B. Smooth splint with a spreading curve
necroticus in the middle. The upper lip C. Splint with hooks
is hyperaemic and edematic. What is the D. Splint with guide plane
most likely diagnosis? E. Pomerantseva-Urbanskaya’s splint

A. Upper lip furuncle 70. A 19-year-old patient complains about


B. Acute periostitis of the upper lip pain in the submental part of mandible.
C. Retention cyst The day before he got a trauma. Objecti-
D. Acute lymphadenitis vely: there is a slight swelling of tissues
E. Upper lip carbuncle in the mental area. The mouth can be
opened widely enough. All the teeth are
66. A female patient complains about pain intact. Mucous membrane is edematic in
in the 11 tooth caused by sour and sweet the region of central incisors, it bleeds sli-
food. Objectively: enamel changes in form ghtly. In this region mobility of mandible
of chalky appearance, a defect with light fragments is present. Occlusion is undi-
bottom within dentinoenamel junction on sturbed. What splint should be chosen by
the vestibular surface in the precervical the dentist?
area of the 11 tooth. Probing was painless,
percussion and cold stimulus caused no A. Smooth brace
pain. What is the most likely diagnosis? B. Splint with spreading curve
C. Splint with guide plane
A. Acute superficial caries D. Anchor splint with intermandibular
B. Acute initial caries fixation
C. Acute median caries E. Ivy loops for 31 32, 42 41
D. Enamel hypoplasia
E. Fluorosis 71. A 25-year-old man complains of itchi-
ng and reddening of skin in the buccal
Krok 2 Stomatology 2009 11

area, general weakness, inertness. He 75. A 13-year-old girl complains about


associates origin of the disease with a frequent falling out of a filling in the 21
skin injury he got during shaving. Objecti- tooth. It is known from the anamnesis that
vely: body temperature is 39, 0o C. A well- 2 years ago she underwent treatment on
defined section of skin in the buccal area account of a dental trauma. Objectively:
is hyperemic, it slightly protrudes above a transverse defect of 1/3 of the crown in
the surface. Hyperemic surface has some the 21 tooth. Percussion is painless. Tooth
vesicles containing serous fluid. What is colour is unchanged. X-ray picture shows
the most likely diagnosis? that root canal is filled by 1 mm from the
apex, filling material closely fits the walls
A. Erysipelatous inflammation of skin of root canal. What stomatological tactics
B. Phlegmon of buccal area should be chosen?
C. Furuncle
D. Anthrax A. The defect should be restored with
E. Streptococcal impetigo photopolymer
B. The root canal should be refilled
72. A 30-year-old saxophonist complai- C. The root apex should be resected
ns about wearing of the occlusal surface D. The tooth should be extracted
of his frontal teeth along with separati- E. The tooth should be crowned with an
on between the upper and lower teeth artificial crown
(apertognathia). Height of the lower third
of the patient’s face remains unchanged. 76. A 35-year-old patient was diagnosed
What method should be applied for elimi- with chronic median caries of the 36 tooth.
nation of this deformity? There is a Black’s class II cavity affecting
masticatory surface. What material should
A. Protetic be chosen for the tooth filling?
B. Increase of the interalveolar height
C. Shortening of teeth A. Light-cure microhybride composite
D. Orthodontic B. Glass ionomer cement
E. Instrumental-and-surgical C. Silicophosphate cement
D. Light-cure fluid composite
73. A 32-year-old patient complains about E. Light-cure microfilled composite
mouth soreness, body temperature rise
up to 38, 5oC, indisposition. Such condi- 77. A 79-year-old female patient
tion has been occurring periodically for consulted a prosthodontist about denture
several years after the patient had had a replacement. The patient has a history
cold. Objectively: lips are covered with of a stroke. Objectively: acute irregular
haemorrhagic crusts, hyperaemic mucous atrophy of the alveolar processes of both
membrane of lips and cheeks has erosions jaws is present; mucous membrane of
covered with fibrinous films. Hypersali- the oral cavity is dry and nonmobile.
vation is present. What is the most likely The previous dentures cannot be fixed.
diagnosis? What is the most appropriate prosthetic
construction?
A. Multiform exudative erythema
B. Pemphigus vulgaris A. Dentures with elastic lining
C. Herpes recidivicus B. Dentures with extended borders
D. Herpetiform Duhring’s dermatitis C. Dentures with metal bases
E. Stevens-Johnson syndrome D. Dentures with shortened borders
E. Implant-supported dentures
74. Examination of a patient revealed a
roundish neoplasm in the submental area. 78. A 48-year-old teacher complained
Skin above it is unchanged, makes a fold about considerable mobility of the 42, 41,
easily. Puncture sample contains strow- 31, 32 teeth. Objectively: cervixes of the
yellow liquid with cholesterol admixture. 42, 41, 31, 32 are exposed by 2/3, there is
What is the most likely diagnosis? pathological III degree mobility. It is indi-
cated to extract the 42, 41, 31, 32 teeth.
A. Inborn median cyst What type of denture should be used for
B. Lipoma the direct prosthetics?
C. Lymphadenitis
D. Retention cyst of sublingual salivary
gland
E. Dermoid cyst
Krok 2 Stomatology 2009 12

A. Removable partial lamellar denture the central position is measured by an


B. Clasp denture angle of 100 − 110o . What is characterized
C. Adhesive by such a value of incisor point deviation?
D. Metal-ceramic bridge-like denture
E. Swaged-and-soldered bridge-like A. Lateral incisor path
denture B. Lateral joint path
C. Sagittal incisor path
79. Examination of a 13-year-old patient D. Sagittal joint path
allowed to make a final diagnosis: vesti- E. Bennett’s angle
bular position of the 13 and 23 teeth with
the total space deficit, narrowing of maxi- 83. Parents of a 1,5-year-old chi-
llary dental arch, torsion of the 12 and 22 ld complain about the child’s tongue
tooth. To eliminate this pathology it was enlargement and ingestion disorder. The
suggested to widen the dental arch and child has been suffering from this since bi-
to extract some teeth. What teeth have rthday. Objectively: general condition has
orthodontic indication for their extracti- no peculiarities. The tongue is enlarged
on? (macroglossia). Its mucous membrane
exhibits granular vesicular outgrowths.
A. First premolars The tongue is dense, painless on palpati-
B. Canines on. What is the most likely diagnosis?
C. Second incisors
D. Second premolars A. Lymphangioma of tongue
E. First molars B. Hemangioma of tongue
C. Fibroma of tongue
80. Parents of a 2-year-old girl complain D. Cyst of tongue
about fistulas with purulent discharge in E. Cancer of tongue
the region of the upper frontal teeth.
Objectively: crowns of the 51, 52, 61, 62 84. A 52-year-old patient complains of
teeth are significantly decayed, probing significant tooth wear on both jaws.
of root canal orifices is slightly painful, it Objectively: in the lateral parts of mandi-
causes significant haemorrhage. Percussi- ble teeth are worn down to the gums,
on is painless. Mucous membrane of the in the frontal parts teeth are worn by
alveolar process is pastose and cyantotic, 1/3. Treatment includes two stages. What
there are cicatrices and fistulas in this regi- prosthetic construction should be appli-
on. What is the most likely diagnosis? ed for bite disjoining and myotatic reflex
change?
A. Exacerbation of chronic granulating
periodontitis A. Periodontal splint for the lateral parts
B. Exacerbation of chronic granulomatous B. Plastic splint for the whole dentition
periodontitis C. Plastic splint for the frontal part
C. Exacerbation of chronic fibrous peri- D. Crown splint for the lateral parts
odontitis E. Elastic plastic splint
D. Chronic granulomatous periodontitis
E. Chronic granulating periodontitis 85. A girl is 18 months old. Vestibular
surfaces of the 52, 51, 61, 62 teeth have wi-
81. A month after cementing the metal- de carious cavities within enamel. Probing
ceramic crown on the 23 tooth a patient is slightly painful, percussion of the 52, 51,
complained of its decementing. Exami- 61, 62 is painless. What is the treatment of
nation revealed that the tooth stump was choice?
of sufficient height, stump walls were
convergent to the vertical tooth axis at an A. Silver impregnation
angle of approximately 30 degrees. What B. Filling with amalgam
angle of convergence had to be formed? C. Remineralizing therapy
D. Fluorine laquer coating
A. Up to 8 degrees E. Filling with phosphate cement
B. 12-15 degrees
C. 15-18 degrees 86. A 53-year-old patient consulted a
D. 22-25 degrees prosthodontist about dental prosthetics.
E. 10-12 degrees Objectively: the 13 and 24 teeth remain
as well as all the teeth on the lower jaw.
82. During lateral movements of mandi- The lower third of the patient’s face is
ble the frontal teeth are displaced si- shortened, crowns of the 13, 24 teeth
dewards. Deviation of incisor point from are worn off by 2/3. What constructional
Krok 2 Stomatology 2009 13

element will provide optimal fixation of on and right zygomatic region, skin
the partial removable prosthesis on the numbness in the area of the right half of
upper jaw? his upper lip; nasal haemorrhage. These
symptoms turned up after a trauma. What
A. Telescopic crowns disease should be suspected?
B. Compound clasps
C. Retaining clasps A. Fracture of zygomatic bone
D. Bars B. Le Fort I maxillary fracture
E. Dentoalveolar clasps C. Le Fort II maxillary fracture
D. Le Fort III maxillary fracture
87. Parents of a 6-year-old child applied to E. Fracture of nose bones
a pedodontist for preventive examinati-
on of their child. The oral cavity is sani- 91. A 56-year-old patient has an oval,
tized. According to the parents, the child smooth, bright-red erosion on the red
has recently cut the 36 and the 46 tooth. border of her lower lip. Erosion is covered
What method of caries prevention should with haemorrhagic crusts that can be
be applied within 1,5-2 years after cutting hardly removed. Crust removal induces
of the mentioned teeth? slight haemorrhage. Light traumatization
of crust-free surface of erosion induces no
A. Fissure hermetization haemorrhage. Specify the type of lower
B. Fissure silvering lip precancer:
C. Coating the teeth with fluorine lacquer
Ftorlak A. Abrasive precancerous Manganotti’s
D. Remodentum solution applications cheilitis
E. Gargling with sodium fluoride B. Verrucous precancer of red border
C. Localized precancerous hyperkeratosis
88. Examination of a 6-year-old girl of red border
revealed a deep carious cavity in the 85 D. Bowen’s disease
tooth. Percussion and probing are pai- E. Erythroplasia
nless. After removal of the softened denti-
ne communication with the tooth cavity 92. A 12-year-old child complains about
showed up. Deep probing is painless. X- bleeding from the tooth socket during
ray picture of the 85 tooth shows the focus eating and tooth brushing. The tooth
of destruction of bone tissue in the region has hurt him before. Objectively: the 36
of bifurcation; cortical plate of the 35 has tooth has a deep cavity communicating
no pathological changes. It is most expedi- with the tooth cavity and filled with red
ent to use the following material for the excrescences. Probing causes pain and sli-
root filling: ght haemorrhage; percussion is painless,
thermal stimuli cause mild pain. What is
A. Zinc oxide eugenol cement your provisional diagnosis?
B. Resorcin-formalin paste
C. Glass ionomer cement A. Chronic hypertrophic pulpitis
D. Phosphate cement B. Chronic granulating pulpitis
E. Calcium-containing paste C. Chronic papillitis
D. Gingival polyp
89. A 48-year-old patient got a pustule E. Chronic simple pulpitis
on his chin that quickly developed into a
dense and acutely painful infiltration 3x5 93. A 23-year-old patient complains
cm large. The skin above it is of blue-red about gingival haemorrhage during tooth
colour. In the centre one can see three brushing and eating solid food. Objecti-
zones of necrosis around the hair follicles. vely: gingiva of the frontal part of mandi-
Lymph nodes of chin are enlarged and ble is hyperaemic, edematic, it bleeds on
painful. What is the most likely diagnosis? palpation. Mucous membrane of the oral
cavity as well as gingiva in other regions
A. Chin carbuncle present no changes. The patient has deep
B. Erysipelatous inflammation of chin overbite. Teeth are stable except for the
C. Dermal actinomycosis of chin 41 and 31 (I degree of mobility). X-ray
D. Suppurated atheroma picture shows resorption of interalveolar
E. Chin furuncle septa by 1/3 of root length in the region of
the 42, 41, 32, 31 teeth. What is the most
90. A patient applied to the oral surgery likely diagnosis?
department and complained about pain
and edema in the right infraorbital regi-
Krok 2 Stomatology 2009 14

A. Localised periodontitis the lower dental arch. It is planned to


B. Initial generalized periodontitis make an implant-supported bridge for its
C. Generalized I degree periodontitis restoration. X-ray picture shows that the
D. Catarrhal gingivitis height of the bone mass from projection of
E. I degree parodontosis mandibular canal up to the top of alveolar
crest is 2 cm. What type of implant should
94. A 13-year-old patient complains be applied?
about gingival haemorrhage during tooth
brushing. Objectively: gums around all the A. Threaded
teeth are hyperemic and edematic, PMA B. Endodontic-endoosseous
index (papillary marginal alveolary index) C. Plate-form
is 46%, Greene-Vermillion hygiene index D. Subperiosteal
is 2,5. Provisional diagnosis: exacerbation E. Conical
of chronic generalized catarrhal gingivitis.
This patient should be recommended to 98. A 10-year-old child undergoes sanitati-
use a toothpaste with the following active on of the oral cavity. The girl was found
component: to have chalky spots on the vestibular
surfaces in the precervical region of the
A. Chlorhexidine 21 and 12 teeth. Enamel surface is dull,
B. Calcium glycerophosphate smooth. Pain reaction to the temperature
C. Monofluorophosphate stimuli is absent. What additional method
D. Vitamins A, D, E of examination is expected to confirm the
E. Microelement complex diagnosis?
95. A 30-year-old patient has got an A. Vital staining
edema and hyperemia of nose wing skin B. Orthopantomography
spreading to the skin of his upper lip and C. Intraoral roentgenography
cheek. Examination revealed a fissure D. Electroodontodiagnostics
along the infra-external edge of the left E. Ultraviolet stomatoscopy
nostril. The skin is tense, dense, red-and-
cyanotic, skin line pattern is smoothed; 99. A 20-year-old patient complains about
rise of local temperature is present. The pain and haemorrhages in the region
zone of affection is well-defined and of the 36 tooth occuring during eating
irregularly shaped. What is the most li- solid food. Objectively: medial masti-
kely diagnosis? catory surface of the 36 tooth has a large
carious cavity occupied by a carneous
A. Erysipelatous inflammation tumour-like formation, probing induces
B. Dermal actinomycosis haemorrhage and pain in the region
C. Dermal tuberculosis of connection of the carious cavity wi-
D. Dermal cancer th the pulp chamber. Percussion is pai-
E. Thrombophlebitis of the facial vein nless. Electroodontodiagnosis is 40 mi-
croampere. Roentgenological changes are
96. On the second day after tooth extracti- absent. What is the most likely diagnosis?
on a 35-year-old woman applied to a facial
surgeon and complained about pain in the A. Chronic hypertrophic pulpitis
parotid-masticatory region and sensation B. Epulis
of lacking contact between the upper and C. Hypertrophic papillitis
lower jaw teeth on the right. Examination D. Chronic gangrenous pulpitis
revealed chin deviation to the left, half- E. Chronic fibrous pulpitis
open mouth, closed lips, disturbed occlusi-
on. There is also limitation of lateral 100. An infant was born full-term with
mandible movements. In front of the right body weight at a rate of 3200 g and body
ear tragus soft tissues sink down. What length at a rate of 53 cm. It was the first
examination is expected to be the most physiological delivery. What position of
informative in this case? child’s mandible is usually observed after
birth?
A. TMJ roentgenography
B. TMJ ultrasound A. Physiological retrogenia
C. Mandible roentgenography B. Physiological progenia
D. Bimanual TMJ investigation C. Direct relation
E. - D. Deep overbite
E. Open bite
97. A 50-year-old patient has a defect of
Krok 2 Stomatology 2009 15

101. A 14-year-old child has orthodontic A. Acute purulent periodontitis


indication for extraction of the 14 tooth. B. Acute purulent pulpitis
What forceps should be applied for C. Acute serous periodontitis
extraction of the 14 tooth? D. Exacerbation of chronic periodontitis
E. Acute diffuse pulpitis
A. S-shaped forceps
B. Straight forceps 105. A patient complains of burning,
C. Bayonet-shaped forceps itch and lower lip enlargement. He has
D. Beak-shaped forceps been suffering from this for a long ti-
E. Curved on flat forceps me. Objectively: the patient’s face is
asymmetric due to the flattening of
102. A 14-year-old child was undergoing nasolabial fold. His lower lip is edematic,
extraction of the 16 tooth on account of of normal colour, painless on palpation.
chronic periodontitis. During the tooth The patient has plicated tongue. What is
extraction it came to perforation of maxi- your provisional diagnosis?
llary sinus along with penetration of the
distal buccal root into the maxillary sinus. A. Melkersson-Rosenthal syndrome
What is the further dentist’s tactics? B. Quincke’s edema
C. Lymphangioma
A. The patient should be directed to the D. Hemangioma
hospital for a surgical procedure E. Granulomatous Miescher’s cheilitis
B. The dentist himself should try to extract
the root 106. A 30-year-old patient consulted a
C. The dentist should form a clottage dentist about a cosmetic defect and slight
without informing the patient pain in the 44 tooth during tooth brushi-
D. The dentist should close the perforation ng. Objectively: there is a Black’s class
with mucoperiosteal graft V carious cavity within mantle dentine.
E. The dentist should perform maxillary Probing of the cavity walls causes slight
sinusotomy in the outpatient setting pain, thermal probe is positive. What filli-
ng material is to be chosen for restoration
103. A 2-month-old child is anxious, sleeps of this cavity?
badly, refuses food, has subfebrile body
temperature. Objectively: hyperaemic A. Compomer restorative material
mucous membrane of the child’s tongue, B. Chemical-cure composite
lips, cheeks and palate has a caseous coati- C. Amalgam
ng that can be easily removed with a D. Polycarboxylate cement
tampon. Regional lymph nodes are sli- E. Metal ceramics
ghtly enlarged and painful on palpation.
What disease are these symptoms typical 107. An 18-year-old patient complains of
for? short-term pain in the 37 tooth caused
by sweet and cold stimuli. Objectively:
A. Acute pseudomembranous candidous masticatory surface of the 37 tooth has
stomatitis a carious cavity within mantle dentine.
B. Chronic atrophic candidous stomatitis The cavity is filled with softened dentine.
C. Acute herpetic stomatitis Probing of the cavity walls induces pain
D. Diphtheria in the region of dentinoenamel junction,
E. Measles-associated stomatitis electroodontodiagnosis is 6 microampere.
What is the most likely diagnosis?
104. A 35-year-old patient complains
about progressing throbbing pain in the A. Acute median caries
26 tooth. Objectively: the 26 tooth has a B. Acute superficial caries
carious cavity filled with softened denti- C. Acute deep caries
ne, tooth cavity is closed, probing of the D. Chronic fibrous pulpitis
cavity floor is painless, percussion causes E. Pulp hyperaemia
acute pain. There is I degree tooth mobi-
lity. Roentgenological changes are absent. 108. A 24-year-old soldier was injured by
What is the most likely diagnosis? a shell splinter. He was diagnosed with a
mandibular fracture with a bone defect in
the mental region over 2 cm long. What
method of fixation of mandible fragments
is indicated?
Krok 2 Stomatology 2009 16

A. Fragments fixation by means of Rudko’s impressions of both jaws. What is his next
apparatus step?
B. Tigerstedt’s splints
C. Intermandibular Ivy ligature A. To send the impressions for disinfection
D. Gunning-Port’s splint B. To let the impressions dry out in the
E. Direct osteosynthesis open air
C. To invite a dental mechanic for joint
109. A 35-year-old female patient analysis of the impressions
consulted a dentist about a painless, D. To send the impressions immediately to
slowly growing neoplasm in the area of the laboratory
the 11 and 12 teeth. Examination revealed E. To put the impressions into the microten
that the tumour was light-pink, flattened, bag for 90 minutes
adjacent to the teeth, had a pedicle. The
tumour was up to 1,5 cm large, with 113. A 12-year-old patient complains
smooth surface and dense consistency. It about an aesthetic defect. Objectively: the
was diagnosed as an epulis in the regiob of lower third of face is shortened, upper
the 11 and 12 teeth. What form of epulis frontal teeth overbite the lower teeth
are these clinical findings typical for? by 3/3 of height, exhibit oral inclinati-
on, lateral parts all along exhibit cusp-to-
A. Fibrous cusp relationship between the antagoni-
B. Angiomatous sts; Angle’s class II malocclusion (joining
C. Giant-cell of the upper permanent molars) is also
D. Pregnancy epulis present. Malocclusion is observed in the
E. - following planes:
110. A 49-year-old patient applied to the A. In sagittal and vertical
oral surgery department and complained B. In transversal
about permanent intense dull pain in the C. In transversal and vertical
region of the right upper jaw. It is known D. In vertical
from the anamnesis that the 17 tooth has E. In sagittal
been repeatedly treated for exacerbation
of chronic periodontitis but the treatment 114. A 65-year-old patient consulted a
appeared to be ineffective. What kind of prosthodontist about fabrication of an
anesthesia should be applied for extracti- external prosthesis of orbit that was lost
on of the 17 tooth? as a result of a trauma. What is the fixing
element of the orbit prosthesis?
A. Tuberal and palatinal
B. Tuberal and incisor A. Spectacle frame
C. Tuberal, incisor and palatinal B. Watch spring
D. Incisor and paltinal C. Swivel devices
E. Torus D. Clamps
E. Magnets
111. Stomatological examination of a chi-
ld revealed abnormal form of the central 115. A 56-year-old patient needs a partial
incisors: they are barrel-shaped, there removable lamellar denture. Objectively:
is a semilunar groove on the cutting the 17, 16, 15, 14, 25, 26, 27, 28 teeth on the
edge. It is known from the anamnesis upper jaw are missing. In order to provi-
that the child’s mother had syphilis de transversal line of clasps the clasp arms
during pregnancy. Besides the dental should be placed upon the following teeth:
abnormality the child presents also with
deafness and parenchymatous keratitis. A. 13 and 24
This abnormality of tooth development is B. 13 and 18
called: C. 24 and 18
D. 24, 13 and 18
A. Hutchinson’s teeth E. -
B. Pfluger teeth
C. Wedge-shaped defect 116. A 56-year-old patient consulted a
D. Fluorosis (destructive form) prosthodontist about pain underneath the
E. Erosion of hard tissues bar of her clasp prosthesis. It is known
from the anamnesis that the clasp was
112. A patient ordered partial removable placed in another city a week ago. Exami-
lamellar dentures for the upper and lower nation revealed a mechanic injury of
jaw. An orthodontist made elastic alginate mucous membrane of the palatine vault
Krok 2 Stomatology 2009 17

caused by the bar of the clasp prosthesis. of crown height by the 12, 13, 14 teeth.
What distance should exist between the Formalin test is painless. What is the most
palate and the bar of the clasp prosthesis likely diagnosis?
in order to prevent this complication?
A. Hypertrophic gingivitis
A. 0,5 mm B. Generalized II degree periodontitis,
B. 2-3 mm chronic course
C. 0,2-0,3 mm C. Catarrhal gingivitis
D. 1,5-2,0 mm D. Ulcero-necrotic gingivitis
E. 5-6 mm E. Exacerbation of generalized I degree
periodontitis
117. A 47-year-old patient complains
about limited mobility of her lower jaw 120. A 34-year-old patient got a trauma
in the morning; periodical dull pain in (fall) that resulted in mobility of alveolar
the right temporomandibular joint (TMJ) process and all the upper jaw teeth,
and general joint stiffness. According occlusion was also changed. X-ray pi-
to the patient, the stiffness disappears cture depicts the fracture line that runs in
throughout the day after joint "exerci- both directions from the piriform opening
sing". Objectively: the patient’s face is along the floor of maxillary sinus. What is
symmetric, mouth opening is limited the most likely diagnosis?
down to 2,5 cm, there is also joint clicking.
Median line deviates to the right by 3-4 A. Le Fort I maxillary fracture
mm, palpation of the right articular head B. Partial fracture of the alveolar process
is painless. What is your provisional di- C. Le Fort II maxillary fracture
agnosis? D. Le Fort III maxillary fracture
E. Unilateral maxillary fracture
A. Arthrosis of the right TMJ
B. Acute serous arthritis of the right TMJ 121. A boy is 1 month old. At the medi-
C. Chronic arthritis of the right TMJ al edge of the inferior eyelid on the ri-
D. Fracure of the right condyle of mandi- ght there is a wound with purulent di-
ble scharge. The boy fell ill suddenly, body
E. Right-sided anterior dislocation of temperature rose up to 40o C. The general
mandible condition is grave. On the second day
of disease there appeared an infiltrati-
118. A 50-year-old patient complains on at the internal edge of eye socket
about problems with mastication, tooth and right cheek. The skin above it is
mobility, halitosis, gingival haemorrhages. hyperemic, fluctuation cannot be determi-
Objectively: gums are hyperemic wi- ned. Palpebral fissure is narrowed. The
th cyanotic colouring, there is dental right nasal meatus discharges pus. There
calculus. Parodontal pouches of the is an infiltration on the vestibular surface
superior molars are 8 mm deep, the of alveolar process and on the right palate.
pouches of other teeth are 6 mm deep. Mucous membrane above it is hyperemic
X-ray picture shows resorption of bone ti- along the mucogingival fold, fluctuation
ssue by 2/3-1/2 of root length. What is the can be determined. What is the most li-
most likely diagnosis? kely diagnosis?
A. Chronic generalized periodontitis of III A. Acute hematogenous osteomyelitis
degree B. Acute dacryocystitis
B. Chronic generalized periodontitis of II C. Phlegmon of the right eye socket
degree D. Acute right-sided highmoritis
C. Chronic generalized periodontitis of I E. Acute serous periostitis
degree
D. Acute generalized periodontitis of III 122. A 20-year-old patient got an injury.
degree Objectively: the patient’s chin and lower
E. Acute generalized periodontitis of II jaw up to the 34 and 45 teeth are missing.
degree The 45, 46, 47, 48, 34, 35, 36, 37 teeth are
stable. At what stage of medical evacuati-
119. An 18-year-old patient complains on the patient will get special medical aid?
about gingival enlargement, pain and
haemorrhage when eating solid food.
Objectively: hyperaemia, gingival edema,
hypertrophy of gingival edge up to 1/2
Krok 2 Stomatology 2009 18

A. Specialized army surgical hospital remaining 18, 17, 13, 12, 11, 21, 22, 23,
B. Battalion aid station 24 teeth are stable. What fixing elements
C. Regimental aid station should be used in a clasp denture for
D. Separate medical detachment cosmetic purposes?
E. Separate medical battalion
A. Attachments and bar system
123. A 45-year-old patient complains B. Telescopic crowns
about a rapidly growing formation on his C. Ney clasps
lower lip. Examination of the red border D. Jackson clasps
of lips revealed a greyish-red nodule wi- E. Dentoalveolar clasps
th a hollow in the centre which is filled
with corneous masses that can be easily 127. A 34-year-old patient consulted a
removed. The nodule is painless, mobile. prosthodontist about pain and clicking in
What is your provisional diagnosis? the final stage of mouth opening. The pati-
ent has a history of a dislocation. Ampli-
A. Keratoacanthoma tude of maximal mouth opening is 58 mm.
B. Papilloma Mouth opening should be limited down to
C. Nodulous verrucous precancer of red the following amplitude:
border
D. Basal cell carcinoma A. Up to 40-50 mm
E. Localized precancerous hyperkeratosis B. Up to 25-30 mm
of red border C. Up to 50-60 mm
D. Up to 10-15 mm
124. A 60-year-old patient underwents E. Up to 90-100 mm
sanitation of the oral cavity before an
operation on account of cataract. After 128. A 12-year-old patient presents wi-
examination the patient was diagnosed th abnormal position of the upper jaw
with chronic median caries of the 22 tooth canine. The 13 tooth is in the vestibular
(Black’s class V). What filling material position, above the occlusal plane. Space
should not be used in this patient? between the 14 and the 12 tooth is 6,5 mm.
Choose a rational treatment method:
A. Light-cure microhybrid material
B. Chemical-cure microhybrid material A. Instrumental
C. Chemical-cure glass ionomer cement B. Surgical and instrumental
D. Silicate cement C. Surgical and physiotherapeutic
E. Chemical-cure macrofilled composite D. Instrumental and myogymnastics
E. Surgical and myogymnastics
125. A 23-year-old patient complains
about periodical pain in the region of 129. Unused stomatological instruments
the 11 tooth, protrusion of the alveolar were left on a sterile table at the end of
process. The patient got a trauma 4 years the working day. What measures should
ago. Objectively: crown of the 11 tooth be taken in order to provide sterility of
is dark, percussion is painless. X-ray pi- these instruments?
cture shows roundish well-defined area of
bone tissue rarefication by the root apex A. Sterilization without preliminary
of the 11 tooth. The area is 2,0 cm in di- processing
ameter. Puncture results: yellow fluid with B. Disinfection, sterilization
cholesterol crystals. What is the most li- C. Disinfection, presterilization treatment,
kely diagnosis? sterilization
D. Presterilization treatment, sterilization
A. Maxillary radicular cyst E. Disinfection only
B. Chronic maxillary osteomyelitis
C. Maxillary ameloblastoma 130. A 42-year-old patient complains
D. Soft maxillary odontoma about gingival pain, progressing gingi-
E. Maxillary osteoclastoma val haemorrhage, increasing tooth mobi-
lity, halitosis. Objectively: gums are evi-
126. A 45-year-old female patient dently hyperaemic, extremely edematic,
consulted a prosthodontist about dental they bleed easily on palpation. Tooth roots
prosthetics. She works as TV announcer. are exposed, parodontal pouches are 4-6
Objectively: the lower jaw dentition is wi- mm deep, and contain purulent exudate,
thout spaces, the upper jaw has a free- there is also supragingival and subgingival
end edentulous space and a boundary dental calculus. II-III grade tooth mobi-
edentulous space in the lateral parts. The lity is present. Orthopantomogram shows
Krok 2 Stomatology 2009 19

resorption of interdental septa down to ble movements. Objectively: skin over


1/2 of their height. What is the most likely the left temporomandibular joint (TMJ)
diagnosis? is reddened, the surrounding tissues
are edematic. Disfunction of the left
A. Exacerbation of generalized II degree temporomandibular joint is present. What
periodontitis is the most likely diagnosis?
B. Exacerbation of generalized I degree
periodontitis A. Acute purulent left-sided arthritis of
C. Exacerbation of generalized III degree TMJ
periodontitis B. Acute serous left-sided arthritis of TMJ
D. Chronic generalized II degree peri- C. Arthrosis of the left TMJ
odontitis D. Deforming arthrosis of the left TMJ
E. Chronic generalized III degree peri- E. Ankylosis of the left TMJ
odontitis
135. A patient complains about li-
131. A 60-year-old patient complains mited mouth opening. She has a hi-
about difficult mastication due to the story of intra-articular disorders in
mandible displacement that resulted from the left temporomandibular joint.
an untreated fracture. Objectively: the Roentgenological examination revealed
35, 36, 38, 45, 46 teeth are missing. The subchondral sclerosis of articular plates,
remaining teeth are intact. The 43, 44, 47, regular narrowing of articular cavity, limi-
48 teeth have no contact with the upper ted excursion of condylar process of the
lateral teeth, and exhibit oral deviation up left temporomandibular joint. What is the
to 1 cm. What is the optimal prosthetic most likely diagnosis?
construction for the manible?
A. Arthrosis of the left temporomandi-
A. Prosthesis with double dentition bular joint
B. Whole-piece bridge B. Deforming arthrosis of the left
C. Arch bar prosthesis temporomandibular joint
D. Adhesive prosthesis C. Ankylosis of the left temporomandi-
E. Vankevich splint bular joint
D. Acute purulent left-sided arthritis of
132. A patient complains about a cosmetic the temporomandibular joint
defect of the 23 tooth. Objectively: the E. Acute serous left-sided arthritis of the
crown of the 23 tooth is decayed by 80%, temporomandibular joint
the root is stable, the canal is filled up to
the top. After examination it was decided 136. A 46-year-old patient complains
to restore the decayed tooth with a cast of difficult opening of her mouth, body
stump inlay. The root canal of the 23 tooth temperature rise, tissue edema around
should be broadened by: both temporomandibular joints (TMJ).
It is known from the anamnesis that the
A. 2/3 of canal length patient had short-term non-intense bi-
B. 1/2 of canal length lateral pain in the parotid-masticatory
C. 1/3 of canal length region, limited mouth opening, tension
D. 1/4 of canal length and discomfort in the region of both
E. By the total canal length temporomandibular joints that had been
133. Regimental Aid Station (RAS) admi- observed for a couple of days. What is the
tted a soldier with an injury in the most likely diagnosis?
maxillofacial region. Dosimetric control A. Rheumatic arthritis
revealed radiation affection in this pati- B. Infectional arthritis
ent. The injured should be referred to the C. Arthrosis
following functional unit of RAS: D. Fibrous ankylosis
A. Decontamination station E. Deforming atrhrosis
B. Dressing pavilion 137. A boy is 10 years old. His face is
C. Evacuation pavilion symmetric and proportional. He presents
D. Isolator with mouth breath. Examination of the
E. Admission and sorting pavilion oral cavity revealed saddle-like form of
134. A 46-year-old patient complai- dental arches and high arched palate.
ns about pain in the left parotid- Upper first molar relationship (Angle’s
masticatory region, disorder of mandi- key to occlusion) remains intact. What is
Krok 2 Stomatology 2009 20

the most likely diagnosis? A. 500 lux


B. 400 lux
A. Narrowing of dental arches C. 300 lux
B. Distal occlusion D. 200 lux
C. Mesial occlusion E. 100 lux
D. Widening of dental arches
E. Elongation of dental arches 142. A patient complains about a cosmetic
defect. Examination revealed that the 21
138. A 56-year-old patient complains tooth was missing, the crowns of the 11
about a painless neoplasm on the alveolar and the 22 tooth are intact, high, relati-
crest of the right upper jaw. Objectively: vely parallel. The patient refused teeth
there is a bright-red wide-based tumour preparation as well as implantation or
in the region of premolars. It is locali- fabrication of a removable denture. What
zed on both sides from the alveolar crest. orthopedic construction should be fabri-
The tumour is dense and elastic, painless. cated?
What is the most likely diagnosis?
A. Adhesive denture
A. Peripheral osteoclastoma B. Clasp denture
B. Central osteoclastoma C. Lamellar denture
C. Gingival fibromatosis D. Immediate denture
D. Fibrous epulis E. Polypropylene denture
E. Papilloma
143. A 47-year-old patient consulted a
139. A 37-year-old patient was admitted to doctor about an itching tumour in the
the oral surgery department. The woman right buccal area that has significantly
complains of pain in her cheek that is enlarged lately. Examination revealed an
made worse by touch. It is known from the irregularly shaped tumour up to 1,5 cm
anamnesis that five days ago the patient large, the tumour is black and glossy. What
fell down from the stairs and "hurt"her is your provisional diagnosis?
cheek. Objectively: in the depth of cheek
examination revealed a circumscribed A. Dermal melanoma
infiltration, cheek skin is hyperaemic and B. Pigmented nevus
doesn’t make a fold, fluctuation symptom C. Dermal basalioma
is present. Mucous membrane is edematic D. Dermal cancer
and has teeth indentations. What is the E. Cutaneous horn
most likely diagnosis?
144. A 38-year-old patient consulted an
A. Suppurated cheek haematoma orthopedist about metal taste, dry mouth
B. Traumatic osteomyelitis of mandible and tongue burning. Objectively: defects
C. Cheek haematoma of the lower dental arch were replaced wi-
D. Cheek phlegmon th soldered stainless steel bridges. What
E. Acute lymphadenitis examination method would be the most
appropriate in this case?
140. A 40-year-old patient suffers
from chronic generalized periodontitis. A. Galvanometry
Objectively: both dentitions are intact, B. Masticatiography
the 42, 41, 31, 32 teeth exhibit I degree C. Occlusiography
mobility. What splint would be aestheti- D. Myography
cally acceptable for this group of teeth? E. Electroodontometry
A. Glass Span and photopolymer splinting 145. A 5-year-old child complains about
B. Guard splint spontaneous pain in an upper jaw tooth on
C. Splint of soldered combined crowns the right that is getting worse at night and
D. Cap splint during eating cold food. Objectively: the
E. Ring splint 65 tooth has a deep cavity communicating
with the tooth cavity. Probing is painful,
141. Renovation of a dental room involves percussion is painless. Cold water causes
installation of luminous tube lamps. Speci- long-standing pain. What is your provisi-
fy the required illuminance level in lux: onal diagnosis?
Krok 2 Stomatology 2009 21

A. Exacerbation of chronic pulpitis lasting. Electroodontodiagnosis is 60 mi-


B. Acute periodontitis croampere. X-ray picture shows slight wi-
C. Exacerbation of chronic periodontitis dening of periodontal fissure at the root
D. Acute serous pulpitis apex of the 25 tooth. What is the most
E. Acute purulent pulpitis probable diagnosis?
146. A 28-year-old patient complains A. Exacerbation of chronic pulpitis
about a cosmetic defect in the frontal part B. Acute generalized pulpitis
of his upper jaw. Objectively: the crown C. Acute purulent pulpitis
part of the 11 tooth is decayed below the D. Acute purulent periodontitis
gum level. The root is stable, percussion E. Exacerbation of chronic periodontitis
is painless. It is planned to restore the
tooth with a stump inlay and cover it with 149. A 23-year-old patient complains
a metal-ceramic crown. What additional about gingival haemorrhage during tooth
method of diagnostics should be applied brushing, intensive formation of dental
in this clinical situation? plaque despite thorough dental care.
Objectively: gingival papillae are slightly
A. X-ray diagnostics edematic, congestively hyperemic, bleed
B. Electromyography when touched. Hygiene index according
C. Masticatiography to Fedorov and Volodkina is 3,5. What
D. Gnathodynamometry toothpaste would you recommend this
E. Electroodontodiagnostics patient as a part of complex therapy?
147. Parents of a 6-year-old child complain A. Salt-containing toothpaste
about pain in the child’s submandibular B. Toothpaste containing mineralizing
region on the left, body temperature ri- components
se up to 37, 5o C. Objectively: the child’s C. Fluorine-containing toothpaste
face is asymmetric due to the infiltration D. Gel toothpaste with microelements
of the submandibular region on the left. E. Toothpaste with antifungal agents
The infiltration is soft and elastic, mobile,
2х2,5 cm large; its palpation is slightly pai- 150. A pregnant 24-year-old woman
nful, the skin is unchanged. The teeth are complains about emergence of several
intact. Pharynx is hyperaemic. What is the new carious cavities, falling out of old
most likely diagnosis? fillings. Objective examination revealed:
index of decayed, missing and filled teeth
A. Acute serous nonodontogenic (DMF) = 16, Feodorov-Volodkina hygi-
submandibular lymphadenitis ene index is 3,3. Choose the optimal
B. Acute serous odontogenic submandi- material for carious cavity filling in this
bular lymphadenitis case:
C. Acute purulent nonodontogenic
submandibular lymphadenitis A. Glass ionomer cement
D. Acute purulent odontogenic submandi- B. Silver amalgam
bular lymphadenitis C. Chemical-cure composite
E. Submandibular adenophlegmon D. Light-cure composite
E. Silicophosphate cement
148. A patient complains about
paroxysmal upper jaw toothache on 151. A group of specialists conducts an
the left that is getting worse at night. epidemiological survey of certain age
Toothache intensifies also under stimulati- groups of population aimed at evaluati-
on and irradiates to the left eye and on of periodontal disease prevalence and
temple. Similar attacks were noted three treatment needs. These rates are studied
months ago, the patient didn’t undergo by means of the following index:
any treatment. Objectively: the 25 tooth
has a deep carious cavity communicati- A. CPITN (WHO index)
ng with the tooth cavity. Probing causes B. OHI-S (Green-Vermillion index)
acute pain at the point of communicati- C. PDI (Ramfjord index)
on, vertical percussion is slightly pai- D. PI (Russel index)
nful, horizontal one is painless. Mucous E. PMA (Parma)
membrane in the projection of root apex
of the 25 tooth is unchanged, its palpati- 152. After the unproblematic extraction of
on is painless. Thermal probe causes the 37 tooth a 60-year-old patient presents
acute pain, the pain attack is long- with profuse haemorrhage from the tooth
Krok 2 Stomatology 2009 22

socket. The patient has a 6-year history 156. A 68-year-old patient complains
of essential hypertension. Now his AP about pain in the palate that occurs duri-
is 180/110 mm Hg. What emergency aid ng wearing a complete removable denture
should be rendered? the patient got 3 months ago. Objectively:
palatine torus is strongly marked, mucous
A. Injection of hypotensive drugs and tight membrane around it is hyperaemic and
pack of the tooth socket edematic. What is the most probable
B. Pack of the tooth socket with cause of this complication?
haemostatic sponge
C. Pack of the tooth socket with iodoform A. Torus wasn’t isolated
tampon B. Previous impression was made in plaster
D. Suture ligature of the tooth socket C. Functional impression was taken by
E. Injection of haemostatic drugs means of a rigid individual tray
D. It was made anatomic teeth
153. A 48-year-old patient complains arrangement
about permanent pain in the region of E. Prosthesis base is made of acrylic plastic
the 38 tooth. She has been suffering from
this for 3 days. Crown of the 28 tooth is 157. A 27-year-old patient complains of a
completely decayed. What forceps should swelling in the region of her lower jaw on
be applied for extraction of roots of the 28 the right. Objectively: the patient’s face is
tooth? slightly asymmetric due to thickening of
mandibular body on the right. Adjacent
A. Bayonet-shaped forceps soft tissues are of unchanged colour and
B. S-shaped forceps curved left normal consistency. Mouth opening is not
C. Straight root forceps limited. Mucous membrane of oral cavi-
D. S-shaped close-beak forceps ty presents no changes. In the right lower
E. S-shaped broad-beak forceps dentition the 45 tooth is missing. X-ray pi-
cture of the right half of the mandible in
154. During military operations the head its lateral view shows an oval well-defined
of the Army Medical Department gave 2x3 cm large radiolucency in the bone ti-
the order to deploy a stomatological aid ssue. The coronal portion of the retinated
unit on the territory of the Army Hospi- horizontally positioned 45 tooth is turned
tal Base. A patient with missile wound of inward the radiolucent area. What is the
face was admitted to the unit. What kind most probable diagnosis?
of aid can be rendered in the stomatologi-
cal unit? A. Follicular cyst of mandible
B. Adamantinoma of mandible
A. Specialized (secondary) aid C. Sarcoma of mandible
B. Qualified aid D. Odontoma of mandible
C. Consultation aid E. Osteoclastoma of mandible
D. Stomatological aid
E. Prosthodontic aid 158. A 52-year-old patient complains
about a nonhealing ulcer on his lower
155. A 62-year-old patient has a median lip. The patient is smoker. He hasn’t ever
fracture of mandible along with formati- consulted a doctor about it. In the region
on of a false joint. Objectively: dental of red border of the lower lip a roundi-
formula is 33, 34, 35, 36, 27, 47, 46, 45, 44, sh ulcer is present. It is up to 2,0 cm in
43. The teeth are intact, stable, with high diameter. The ulcer edges are thickened
crowns. Fragment mobility is insignificant, and a little bit raised in form of a whitish
there is no displacement. X-ray picture swelling. In the left submandibular regi-
shows a bone defect 0,8 cm large. What on palpation revealed enlarged, painless,
prosthesis is indicated? dense lymph nodes with limited mobility.
A. Bridge-like prosthesis with a pivot point What is the most likely diagnosis?
B. Lamellar prosthesis without a pivot A. Cancer of the lower lip
point B. Erosive verrucous leukoplakia
C. Lamellar prosthesis with Gavrilow’s C. Keratoacanthoma
pivot point D. Fibroma of the lower lip
D. Lamellar prosthesis with Oxman’s pivot E. Syphilitic ulcer
point
E. Lamellar prosthesis with Weinstein’s 159. A 25-year-old patient complains
pivot point about acute pain in the mouth, headache,
articular pain, body temperature rise up to
Krok 2 Stomatology 2009 23

38, 6oC. Red border of lips is covered with appeared several months ago. Objecti-
haemorrhagic crusts, mucous membrane vely: the patient has a lot of carious
of the oral cavity has big erosions and and completely decayed teeth, lateral
ulcers coated with greyish incrustation. surface of tongue exhibits a painless whiti-
Hand skin exhibits erythematous spots 1- sh formation 10x5 mm large with irregular
1,5 cm in diameter with a vesicle in the surface in form of verrucae. Histologi-
middle. What is the most likely diagnosis? cal examination revealed thickening of
corneal epithelial layer of intermittent
A. Stevens-Johnson syndrome keratinization type. What is the most li-
B. Behcet’s syndrome kely diagnosis?
C. Lyell’s syndrome
D. Multiform exudative erythema A. Verrucous form of leukoplakia
E. Medicamentous stomatitis B. Verrucous precancer
C. Hyperplastic form of candidiasis
160. A 14-year-old child complains about D. Hyperkeratotic form of lichen ruber
acute spontaneous spasmodic pain in an planus
upper jaw tooth on the right. The pain E. Keratoacanthoma
has been lasting for 3 days, it is throbbing,
irradiating to the temple, getting worse at 164. A 35-year-old man has a medi-
night. Objectively: surface of the 15 tooth um deep carious cavity in the 37 tooth
exhibits a carious cavity within parapulpar (Black’s class II). For its filling a denti-
dentine. Dentine is softened, of greyish st chose technique of layer-by-layer
colour. Probing of the whole cavity floor is restoration. What composite should be
painful, percussion of the 15 tooth is pai- coating the floor and walls of the carious
nless. What is the most likely diagnosis? cavity in order to form superadaptive ini-
tial layer?
A. Acute purulent pulpitis
B. Acute diffuse pulpitis A. Flowable
C. Acute focal pulpitis B. Condensable
D. Acute periodontitis C. Macrofilled
E. Exacerbation of chronic periodontitis D. Microhybrid
E. Microfilled
161. A 3-month-old child has an inborn
tissue defect in the oral cavity. Objecti- 165. A 9-year-old boy presents with
vely: the lip is intact, the oral cavity exhi- face asymmetry due to the chin devi-
bits a cleft defect of soft palate and medi- ation to the left. When the third Il’ina-
an part of hard palate. The child was di- Marcosian diagnostic test is performed
agnosed with complete schistasis of soft face asymmetry disappears. What is the
palate and partial schistasis of hard palate. most likely clinical form of this occlusal
Specify the dispensary group according to anomaly?
A.A.Kolesov:
A. Habitual deviation of mandible
A. First B. Ankylosis of the temporomandibular
B. Second joint
C. Third C. Unilateral hypoplasia of mandible
D. Fourth D. Bilateral narrowing of the maxillary
E. Fith dental arch
E. Unilateral narrowing of the maxillary
162. A dentist treats a 22-year-old patient dental arch
on account of acute deep caries of the 26
tooth. As lining material the dentist chose 166. A completely edentulous 70-year-old
calcium salicylate cement "Life". What patient has ordered complete removable
material should be chosen for insulation dentures. Artificial teeth are placed upon
of the lining? the spherical surface. Close teeth contact
by movements of the lower jaw will be
A. Glass ionomer cement guaranteed by the following average radi-
B. Insulating varnish us of the spherical surface:
C. Adhesive composite system
D. Zinc phosphate cement
E. Zinc oxide eugenol cement
163. A 62-year-old patient complains of
a painless formation on his tongue that
Krok 2 Stomatology 2009 24

A. 9 cm A. The dentist should break the procedure


B. 5 cm off and give the patient sugar in any form
C. 7 cm B. The dentist should lay the patient down
D. 12 cm on a flat surface, lower cephalic pole, open
E. 18 cm the windows
C. The dentist should give the patient
167. A 40-year-old patient consulted a nitroglycerine in form of spray or subli-
prosthodontist about upper jaw teeth ngually
mobility that turned up after therapeutic D. The dentist should make intramuscular
treatment. Objectively: the 13, 12, 11, injection of adrenaline (0,5-1,0 mg)
21, 22 teeth are pulpless, exhibit II E. The dentist should make intravenous
degree mobility, unchanged colour. What injection of corticosteroids
prosthetic construction should be appli-
ed? 171. Tooth replacement with complete
removable dentures involves adjustment
A. Mamlock splint of occlusal relations by different
B. Crown soldered splint movements of the lower jaw. Transversal
C. Elastic crown splint movements of the lower jaw are initiated
D. Cast bar splint by the following muscle:
E. Wire-band splint
A. External (lateral) pterygoid muscle
168. A 5-year-old child was bitten by a dog B. Internal (medial) pterygoid muscle
2 days ago. The child is diagnosed with a C. Temporal muscle
bite wound of cheek. The parent’s didn’t D. Mastication muscle
appeal for medical aid in proper time. The E. Digastric muscle
wound exhibits pyoinflammatory process.
What kind of surgical d-bridement should 172. A 50-year-old patient complains
be performed? about pain in the region of the left
temporomandibular joint (TMJ) during
A. Secondary mouth opening. Dental formula is 14, 13,
B. Initial early 12, 11, 21, 22, 23, 24, 33, 32, 31, 41, 42, 43,
C. Initial delayed 44. While opening the mouth the lower
D. Initial late jaw moves in a zigzag manner towards the
E. Surgical procedure is not required affected joint. What pathological conditi-
169. A 16-year-old teenager complains on are these symptoms typical for?
about halitosis, general weakness, body A. Musculoarticular dysfunction
temperature rise up to 37, 6o C. These B. Habitual TMJ dislocation
symptoms turned up 2 days ago, the boy C. Sclerosing arthrosis of TMJ
has a history of recent angina. Objecti- D. Chronic arthritis of TMJ
vely: oral cavity hygiene is unsatisfactory, E. Deforming arthrosis of TMJ
teeth are covered with soft white deposit.
Gums are hyperaemic, gingival papillae 173. A 35-year-old patient is at a denti-
are covered with greyish coating. What is st. He is anxious, complains of being
the most likely diagnosis? afraid of the dentist, is unwilling to let
the dentist examine him. Drugs of which
A. Ulcero-necrotic gingivitis group should be given the patient before
B. Acute catarrhal gingivitis stomatological manipulations?
C. Hypertrohic gingivitis
D. Chronic catarrhal gingivitis A. Sedative
E. Desquamative gingivitis B. Analgetics
C. Hypotensive
170. A patient being at a dentist presented D. Cardiac glycosides
suddenly with tachycardia, sweating, E. Desensitizing
trembling, nausea, hunger. The patient is
conscious. He suffers from diabetes melli- 174. A 76-year-old patient has a roundish
tus. What aid should be rendered by the ulcer in the area of the right nasolabial
dentist? sulcus. The ulcer has dense, rough floor
covered with yellowish-grey crusts, and
resembles of a cone. The ulcer edges are
dense, crater-like; the ulcer grows both
sidewards and deepwards. What is your
provisional diagnosis?
Krok 2 Stomatology 2009 25

about the astringent sensation in the


A. Basalioma upper jaw incisors, which appeared 1,5
B. Tuberculous ulcer years ago. Objectively: the most convex
C. Actinomycotic ulcer part of the vestibular surfaces of the 12,
D. Syphilitic ulcer 11, 21, 22 crowns has roundish defects wi-
E. Squamous cell carcinoma of skin th smooth, shiny, dense surface, that reach
dentine in depth. The depth of defects is
175. A 40-year-old patient complains gradually decreasing from the centre to
about a long-existing fissure in the central the periphery. What is the most likely di-
part of red border of his lower lip. He has agnosis?
been smoking from the age of 16. Objecti-
vely: there is a deep 1,0 cm long fissure A. Enamel erosion
along the median line of red border of B. Wedge-shaped defect
the lower lip, the fissure edges are bulgi- C. Systemic hypoplasia
ngl. Characteristic quality of the fissure D. Chronic median caries
is its spontaneous healing, but after epi- E. Destructive fluorosis
thelization the fissure recurs. The pati-
ent was diagnosed with chronic fissure 179. Examination of a 6-year-old boy
of the lower lip. Conservative treatment revealed enlarged lymph nodes in both
proved to be ineffective. Choose the most submandibular and cervical regions.
appropriate treatment method in this si- Objectively: the 75, 84 and 85 teeth
tuation: are decayed, there are presentations of
commissural cheilitis. According to the
A. Excision within evidently healthy ti- boy’s mother, he has been quickly getti-
ssues ng tired, sweating from the least physi-
B. Cryodestruction cal exercise, complaining about weakness
C. Diathermo-coagulation throughout the last 2-3 months. He also
D. Excision 0,5 cm off the neoplasm limits gave up training in a sports class. What
E. Close-focus roentgenotherapy plan of additional examination should be
adopted?
176. It is planned to lance an abscess
of hard palate (it is localized close to A. Complete blood count, haematologist
the 23, 24, 25 teeth) and to extract the consultation
causative 24 tooth that had caused abscess B. Biochemical blood test, endocrinologist
development. What type of anaesthesia is consultation
indicated for this operation? C. Puncture biopsy, oncologist consultation
D. Magnetic resonance tomography,
A. Infraorbital, palatinal and incisor immunologist consultation
B. Tuberal, infraorbital and incisor E. Pulmonary roentgenography,
C. Tuberal, infraorbital and palatinal pulmonologist consultation
D. Tuberal and incisor
E. Tuberal and palatinal 180. A 32-year-old patient complains
about pain and swelling in the region of
177. A 36-year-old patient consulted a his mandible and left cheek. He has been
dentist about permanent acute pain in the suffering from this for 2 days. Objecti-
upper jaw teeth, body temperature rise. vely: his general condition is satisfactory.
The dentist found out that the 26 tooth Body temperature is 37, 5o C. There is an
had been repeatedly treated, the rest of edema of soft tissues in the left buccal
teeth were intact. After roentgenological region. Submandibular lymph nodes are
examination the patient was diagnosed bilaterally enlarged, painful on palpati-
with acute highmoritis. What is the most on. Crown of the 37 tooth is partially
effective way of treatment? decayed, the tooth is immobile, percussi-
on is slightly painful. Mucogingival fold
A. Extraction of the causative tooth and in the region of the 37 tooth is edematic,
medicamentous therapy hyperaemic, flattened, its palpation causes
B. Caldwell-Luc maxillary sinusotomy acute pain. What is the most likely di-
C. Puncture of the maxillary sinus and agnosis?
medicamentous therapy
D. Medicamentous therapy
E. Medicamentous and physiotherapeutic
treatment
178. A 44-year-old patient complains
Krok 2 Stomatology 2009 26

A. Acute purulent periostitis of mandible permanent intense throbbing toothache


beginning from the 37 tooth that is getting worse when biting down
B. Acute odontogenous osteomyelitis of on food. Objectively: the patient’s face
mandible is asymmetric because of a collateral
C. Chronic productive periostitis of mandi- edema of soft tissues, submandibular
ble beginning from the 37 tooth lymph nodes are enlarged and painful on
D. Chronic odontogenous osteomyelitis of palpation. The 26 tooth has a deep cavity
mandible not communicating with the tooth cavity.
E. Abscess of alveololingual groove on the Thermal test is positive, probing is pai-
left nless, percussion causes acute pain, the
tooth is mobile. X-ray picture of the 26
181. A 12-year-old girl complains about tooth shows no changes in the periodonti-
swelling, reddening and itching of lips. She um. What is the most likely diagnosis?
has a history of bronchial asthma. Such
condition has been observed in the child A. Acute purulent periodontitis
for 3 years in winter period. Objectively: B. Acute serous periodontitis
red border of lips and adjacent skin exhi- C. Exacerbation of chronic periodontitis
bit a well-defined erythema and edema, D. Acute purulent pulpitis
small cracks. There are signs of lip li- E. Pulpitis complicated by periodontitis
chenification. There are also marks of
scratches on the skin of lips, cheeks and 185. A 6-year-old girl was brought to the
forehead in form of abrasions and crusts. dentist for completing the treatment of
Mucous membrane exhibits no pathologi- the 75 tooth for chronic granulating peri-
cal changes. What is the most likely di- odontitis. The patient has no complaints.
agnosis? Objectively: occlusive dressing on the 75
tooth remains intact, percussion is pai-
A. Atopic cheilitis nless, mucous membrane in the region
B. Contact allergic cheilitis of the 75 tooth exhibits no pathological
C. Meteorological cheilitis changes, is painless on palpation. What
D. Exfoliative cheilitis material is the most appropriate for the
E. Microbal cheilitis root canal filling in this case?
182. Parents of a 5-year-old child noted A. Zinc oxide eugenol paste
irregular colour of the child’s back of B. Sealer with gutta-percha point
tongue and consulted a pedodontist C. Calcium-containing paste
about this. Objectively: the oral cavity D. Glass ionomer cement
is sanitized, DF index = 4, dorsal and E. Iodoform paste
lateral surfaces of tongue have area of
desquamating epithelium bordering upon 186. A 4-year-old child presents with
areas of hypertrophy and increased corni- headache, nausea, pain when swallowi-
fication of filiform papillae. The child ng. Body temperature is 39o C. Examinati-
has a history of being allergic to some on of the oral cavity revealed extremely
foodstuffs. What is the clinical diagnosis? hyperaemic mucous membrane and tonsi-
ls. On the first day of disease the child’s
A. Desquamative glossitis tongue was furred with greyish coating,
B. Tuberculous glossitis on the second day the tongue was found
C. Candidous glossitis to be self-purified. On the third day the
D. Herpetic tongue lesion tongue was smooth, of crimson colour, wi-
E. Acute catarrhal glossitis th hyperaemic fungiform papillae. What is
presumptive diagnosis?
183. A 4-year-old girl was bitten by a dog
in her upper lip. Which of the following A. Scarlet fever
solutions should be primarily used for the B. Varicella
wound d-bridement? C. Measles
D. Diphtheria
A. 10% solution of laundry soap E. Herpangina
B. 0,002% solution of chlorhexidine
C. 1% solution of hydrogen oxide 187. A 15-year-old child presents with
D. 1 : 5000 solution of potassium puffiness in the region of the mandi-
permanganate ble branch; enlarged, dense and painless
E. 3% soda solution lymph nodes adhering to the surrounding
tissues. X-ray picture of mandible branch
184. A 12-year-old boy complains about shows a well-defined bone resorption
Krok 2 Stomatology 2009 27

area containing small sequestra. After 191. A 22-year-old patient is afraid of pain
Mantoux test a 12 mm papule was noted. from conduction anaesthesia. A dentist
What is the most likely diagnosis? decided that this anaesthesia should be
preceded by applicational anaesthesia of
A. Tuberculosis of mandible branch mucous membrane on the spot of injecti-
B. Mandibular actinomycosis on. What drug should be used for this
C. Chronic osteomyelitis of mandible purpose?
branch
D. Acute mandibular osteomyelitis A. 5% lidocaine ointment
E. Ewing’s sarcoma B. 1% synthomycin ointment
C. 5% oxacillin ointment
188. An 11-year-old child complains about D. 3% sinaflan ointment
missing crown of the 12 tooth as a result E. 3% doxycyclin ointment
of a trauma. The tooth root is well treated.
What prosthetic construction is indicated 192. A 12-year-old girl complains about
for elimination of this defect? intense pain in the region of the 46 tooth
socket that was extracted 3 days ago. The
A. Il’ina-Marcosian’s pivot tooth pain is irradiating along the branches of
B. Cantilever prosthesis supported by the trigeminus. Objectively: lymph nodes are
11 tooth enlarged and painful on palpation, tissues
C. Cantilever prosthesis supported by the around the tooth socket are edematic
13 tooth and hyperaemic. The socket walls are
D. Bridge-like prosthesis supported by the covered with grey-and-green deposition
13 and 11 teeth with putrid smell. What is the most likely
E. Partial removable replacing prosthesis diagnosis?
189. A 34-year-old patient has indicati- A. Alveolitis
on for extraction of the 38 tooth because B. Pericoronaritis
of chronic fibrous periodontitis. Oral C. Ostitis
surgeon performed torus anaesthesia. D. Periostitis
This anaesthesia blocked transmission E. Osteomyelitis
of pain information from the following
nerves: 193. A 42-year-old patient consulted
a dentist about intense lancinating
A. Infraalveolar, lingual, buccal paroxysmal pain accompanied by a
B. Lingual, buccal sensation of current passage in the regi-
C. Infraalveolar, buccal on of her upper lip on the right. Pain
D. Infraalveolar, lingual attacks occur spontaneously and last 3-
E. Lingual, buccal, mental 5 minutes. The patient usually has 2-3
attacks a day. The patient is unable to
190. A patient got a trauma that establish the cause of this disease. Exami-
resulted in limited mouth opening, nasal nation of her oral cavity revealed no
haemorrhage, numbness of inferior eyelid pathological changes. What is the most
as well as skin in the infraorbital area. likely diagnosis?
Objectively: there is face deformation
caused by depression of soft tissues in the A. Peripheral neuralgia of the II branch of
malar region on the left; there is also a trigeminus
step-off deformity in the median part of B. Pterygopalatine ganglionitis
the infraorbital rim and in the region of C. Neuritis of the II branch of trigeminus
zygomatico-alveolar crest. What method D. Central neuralgia of the II branch of
of examination will be the most effective trigeminus
for the diagnostics? E. Right-sided upper jaw pulpitis
A. X-ray study of facial skull in the axial 194. A 23-year-old patient was deli-
projection vered to a traumatology centre with
B. X-ray study of facial skull in the frontal a dirty cut wound of her right foot.
projection A doctor performed initial surgical d-
C. X-ray study of facial skull in the lateral bridement and made an injection of anti-
projection tetanus serum. Some time later the pati-
D. X-ray study of facial skull and paranasal ent’s condition got abruptly worse: she
sinuses developed extreme weakness, dizziness,
E. Orthopantomogram palpitation. Objectively: the skin is pale,
the patient has cold sweat, frequent pulse
Krok 2 Stomatology 2009 28

of poor volume at a rate of 100 bpm, AP A. Start cardiopulmonary resuscitation


is 90/40 mm Hg. What is the cause of such B. Deliver the patient to the hospital as
aggravation? soon as possible
C. Make an intracardiac injection of
A. Anaphylactic shock adrenaline
B. Haemorrhagic shock D. Facilitate oxygen inhalation
C. Pain shock E. Make an intravenous injection of
D. Infectious-toxic shock polyglucinum
E. Drug disease (seroreaction)
198. An 18-year-old woman has been
195. A man was found in a street suffering from diabetes mellitus for 5
in unconscious state and delivered to years. She receives 36 units of insulin per
the sanitary inspection room. At small day. During pneumonia her condition got
intervals the man presents with recurrent abruptly worse: the patient presented with
attacks of tonic and clonic convulsions of increased thirst, abdominal pain, nausea,
the upper and lower limbs. Objectively: vomiting and sleepiness. In the evening
the patient is unconscious, his pupils are the patient refused to eat and didn’t get
mydriatic, don’t react to light. The tongue her regular insulin dose; in the morning
has teeth indentations. Involuntary uri- she lost consciousness. Objectively: the
nation was noted. Examination revealed patient is unconscious, the skin is dry,
no symptoms of focal cerebral lesion. AP turgor is decreased. The tongue is also
is 140/90 mm Hg. The patients head can dry. Respiration is deep and noisy, there is
be freely brought to the chest. Specify the sharp smell of acetone from the mouth.
most probable pathology: Body temperature is 36, 6o C, Ps is 100
bpm, small; AP is 90/50 mm Hg. Urine
A. Epilepsy, status epilepticus exhibits extremely positive reaction to
B. Parenchymatous hemorrhage acetone. Blood glucose is 33 millimole/l.
C. Cerebral infraction What is your
D. Tetanus
E. Acute meningoencephalitis A. Ketoacidotic coma
B. Hyperosmolar coma
196. A patient was found unconscious C. Lactacidemic coma
in her flat heated with firewood. The D. Hepatic coma
stove pipe damper was closed, it smelt of E. Cerebral coma
smoke in the room. Objectively: the pati-
ent is unconscious, her skin and mucous 199. A 28-year-old man attended a
membranes are of cherry red colour. Pupi- glasshouse in a botanic garden. After he
ls exhibit no reaction to light, corneal and had smelt at an orchid he turned pale and
tendinous reflexes are absent, muscle tone lost consciousness. Objectively: heart rate
is decreased. Ps is 96 bpm, of poor volume. is 115/min, arterial pressure is 50/0 mm
Heart sounds are muffled, there are soli- Hg. What drug should be injected to the
tary extrasystoles, AP is 90/60 mm Hg. patient in the first place?
In the lower lungs solitary small bubbli-
ng rales are present, respiratory rate is A. Prednisolone
28/min. What is the most likely diagnosis? B. Cordiamin
C. Strophanthine
A. Acute intoxication with carbon oxide D. Mesaton
B. Acute chlorine intoxication E. Dimedrol
C. Acute intoxication with sulfur dioxide
gas 200. A 68-year-old patient has been taking
D. Acute methane intoxication diclofenac because of pain in the knee joi-
E. Acute benzol intoxication nts for about a year. In the evening after
substantial meal the patient presented wi-
197. During transportation of a 60-year- th liquid black stool, drastic weakness,
old patient an emergency doctor noted single vomiting after which the patient
aggravation of his general condition, lost consciousness. Emergency team deli-
pulselessness, mydriatic pupils, absence of vered him to the hospital. Objectively:
respiratory movements. It will take 5 mi- the patient is conscious, with slightly
nutes to get to the admission ward. What deferred reaction, pale. Heart rate is 20
actions should the doctor take in the first breaths/min, Ps is 102 bpm, AP is 110/70
place? mm Hg. Abdomen is soft, painful in the
epigastrium. What is the most likely di-
Krok 2 Stomatology 2009 29

agnosis? A. Gastrointestinal haemorrhage


B. Food poisoning
C. Myocardium infarction
D. Apoplectic attack
E. Thrombosis of mesenterial arteries
Krok 2 Stomatology 2010 1

1. A 42-year-old patient complai- A. Acute deep caries


ns about general weakness. Body B. Acute median caries
temperature is 41o C, he presents with si- C. Acute localized pulpitis
ckness, emesis, dyspnea. There is evident D. Acute diffuse pulpitis
well-defined skin redness in form of flame E. Chronic fibrous pulpitis
tips on her face. Submandibular lymph
nodes are enlarged. What is the most li- 5. A 12-year-old child complains about
kely diagnosis? sore throat, headache, body temperature
rise up to 38, 5oC, rhinitis, cough in
A. Erysipelas summer period. Objectively: mucous
B. Streptococcal impetigo membrane of oral cavity is hyperemic,
C. Phlebitis of facial vein edematic. There are 10-15 erosions up to
D. Lupus 0,5 mm large on the palate and palatine
E. Anthrax arches, that aren’t covered with deposit
and have red floor. Regional lymph nodes
2. A victim got a perforating wound are enlarged and painful on palpation.
of the left cheek contaminated with What is the most likely diagnosis?
radioactive materials on the battlefield.
The wound was dressed with aseptic A. Herpetic angina
bandage from the individual first-aid B. Acute herpetic stomatitis
pack and processed with anaesthetic and C. Erythema multiforme
antibacterial medicaments. The dressing D. Chronic recurrent aphthous stomatitis
should be changed at the following stati- E. Infectious mononucleosis
on of medical evacuation:
6. A 43-year-old man complains about pai-
A. At each station of evacuation nful swallowing, limited mouth opening.
B. At the battalion aid station Destroyed 37 tooth caused pain 3 days
C. At the regimental aid station ago. Painful swallowing appeared the next
D. At the station of the separate medical day. Limited mouth opening was present
battalion a day later. Objectively: there is a small
E. Only when the dressing is unsatisfactory soft swelling in the left submaxillary area.
An enlarged lymph node is palpated. The
3. Mother of a 6,5-year-old child consulted mouth can be opened by 1,5 cm. Edema
a dentist about checking fissure hermeti- of left palatine arch and pharynx lateral
zatioon of the 16, 26, 36, 46 teeth. They wall is present. The 37 tooth is destroyed.
were treated six month ago. Sealant in the Mucous membrane around the tooth is
36, 46 teeth is preserved, but sealant in edematic. Percussion is slightly painful.
the 16 and 26 teeth is absent. What is the What is the most likely diagnosis?
tactics of choice?
A. Odontogenic peripharyngeal phlegmon
A. To repeat fissure hermetization B. Odontogenic pterygomandibular
B. Preventive filling phlegmon
C. Applications with remodentum solution C. Odontogenic submaxillary phlegmon
D. Coating the teeth with fluorine lacquer D. Peritonsillar abscess
Ftorlak E. Acute submaxillary lymphadenitis
E. Electrophoresis with 1% sodium fluori-
de solution 7. A 34-year-old male patient consults a
dentist about an ulcer on the hard palate.
4. A 12-year-old boy complains about It appeared about a month ago. He has
short-term pain in a lower jaw tooth on treated it by rinsing with herbal water, but
the right caused by cold stimuli. Objecti- the ulcer is gradually "creeping". Objecti-
vely: the 46 tooth has a carious cavi- vely: there is a shallow erethistic ulcer wi-
ty on the masticatory surface within th uneven and undermined edges of soft
the circumpulpar dentine which has no consistency within the mucous membrane
intercommunication with the dental cavi- of hard palate. Granulations of the ulcer
ty. The cavity floor and walls are coated floor are also present. Yellowish granules
with light softened dentine. Cold test are visible on the ulcer periphery. What is
causes short-standing pain. What is the the most likely diagnosis?
most likely diagnosis?
Krok 2 Stomatology 2010 2

A. Tuberculous ulcer 12. A 70-year-old male patient ordered


B. Trophic ulcer complete removable dentures for both
C. Cancerous ulcer jaws. In the clinical stage he came for
D. Syphilitic ulcer "identification of central relationship".
E. Actinomycosis For verification of interalveolar height the
patient underwent speaking test. The pati-
8. A 27-year-old male patient underwent ent was asked to pronounce the sounds
extraction of the medial root of the 36 "O", "E", "M". What distance should
tooth, but the distal tooth root was broken be between the occlusal rims for correct
in its middle third. What tools should be identification of interalveolar height?
applied for root extraction?
A. 5-6 mm
A. Left angled elevator B. 9-10 mm
B. Right angled elevator C. 3-4 mm
C. Straight elevator D. 1-2 mm
D. Broad-beaked forceps E. 7-8 mm
E. Close-beaked forceps
13. A 16-year-old male patient complains
9. A 55-year-old woman complains about about pain in the oral cavity, ulceration,
liquid pouring through her nose during body temperatureup to 38o C, headache.
eating of fluid food. Objectively: there is Objectively: mucous membrane of the
a perforating defect of alveolar process oral cavity is hyperemic and edematic.
3x2,8 cm deep in the lateral part of the There are a lot of confluent erosions of
upper jaw at level with the extracted polycyclic shape, covered with grey and
16 tooth. The patient refused operative white deposit, located on hard palate,
intervention. What construction should gums, lips. What is your provisional di-
be offered? agnosis?
A. Minor saddle prosthesis with clasp A. Acute herpetic stomatitis
fixation B. Erythema multiforme
B. Clasp dental prosthesis with obturating C. Pemphigus vulgaris
element made of elastic plastic D. Aphthous fever
C. Partial removable lamellar prosthesis E. Allergic stomatitis
D. Fixed dental bridge
E. Celluloid mouth protector 14. An 8-year-old child was diagnosed
with granulating periodontitis of the 55
10. A 53-year-old patient was diagnosed tooth. The crown is completely destroyed.
with leucoplakia nicotinica Tappeiner. X-ray picture shows separated tooth roots.
What pathohistological process predomi- Choose an optimal tool for operative
nates in histologic pattern of this disease? intervention:
A. Hyperkeratosis A. Root bayonet-shaped forceps
B. Acanthosis B. Root straight forceps
C. Dyskeratosis C. Root beak-shaped forceps
D. Parakeratosis D. Crown bayonet-shaped forceps
E. Papillomatosis E. Crown S-shaped forceps
11. A 24-year-old female patient complai- 15. An 8-year-old boy was diagnosed with
ns about hard tissues defect of the chronic fibrous pulpitis of the 21 tooth.
21 tooth. Objectively: the 21 tooth is It was treated by extirpation method.
destroyed by 1/3 along the cutting edge, Choose the material for root filling:
it is changed in color. X-ray picture shows
that the root canal is filled to the top. It A. Calcium-containing material
was decided to fabricate a plastic crown. B. Glass-ionomer cement
Where should the crown edge be situated C. Zinc oxide eugenol paste
against the marginal gingiva? D. Resorcin-formalin paste
E. Phosphate cement
A. At a level with gingiva
B. 0,5 mm away of gingiva 16. A 12-year-old female patient was di-
C. 1,0 mm away of gingiva agnosed with open bite and dentoalveolar
D. 0,5 mm below the gingiva elongation of lateral part of mandible.
E. 1,0 mm below the gingiva What construction of apparatus is requi-
red?
Krok 2 Stomatology 2010 3

(4ml) during extraction of the 17 tooth


A. Upper jaw appliance with occlusal rest the patient complained about respiration
seats obstruction. Objectively: upper and lower
B. Extraoral face bow lips are edematic, mucous membrane of
C. Angle’s sliding face bow larynx and oral cavity is edematic and
D. Herbst appliance extremely hyperemic. What complication
E. Upper jaw appliance with a face bow turned up in this patient?
17. An 18-year-old patient complains A. Quincke’s edema
about an aesthetic defect. Objectively: the B. Anesthetic intoxication
lower teeth are set forward and overlap C. Anaphylactic shock
the upper antagonists. This symptom is D. Collapse
typical for the following bite abnormality: E. Acute pulmonary insufficiency
A. Mesial bite 22. A 37-year-old male patient complai-
B. Distal bite ns about pain of the 46 tooth during
C. Deep bite food intake, especially hot food, offensive
D. Open bite breath when he sucks his tooth. Objecti-
E. Cross bite vely: the face is symmetrical, masticatory
surface of the 48 tooth has a deep cari-
18. A 70-year-old male patient complains ous cavity communicating with the dental
about total teeth missing on both jaws. cavity. X-ray picture shows widening of
It is planned to fabricate a complete periodontal fissure at the root apex of
removable lower jaw prosthesis. Objecti- the 46 tooth. What is the most likely di-
vely: alveolar process of the lower jaw agnosis?
is atrophied only in the frontal aspect.
Identify the type of lower jaw atrophy A. Chronic gangrenous pulpitis
according to Keller’s classification: B. Exacerbation of chronic periodontitis
C. Exacerbation of chronic pulpitis
A. IV D. Chronic fibrous periodontitis
B. II E. Chronic fibrous pulpitis
C. I
D. III 23. A 17-year-old girl complains about pai-
E. - nfulness and gingival hemorrhage during
tooth brushing and food intake. She has
19. A 18-year-old female patient ordered a been suffering from it for 1,5 year. She
metal-ceramic crown for the 24 tooth. It is consulted a dentist. There was no consi-
necessary to choose an optimal impressi- derable effect from treatment. Objecti-
on material for combined impression of vely: gingival papillae are flabby and
the upper jaw. What material should be cyanotic in the region of frontal teeth
used for a preliminary impression? on both jaws, they overlay teeth crowns
A. Stens by 1/3-1/2 of their height, they bleed on
B. Dentafol touch. Therapeutic effect can be achieved
C. Orthocar by means of electrophoresis with:
D. Wax A. Heparin
E. Protacryl B. Aloe
20. A patient consulted a dentist about C. Vitamin B1
constant dull pain in the 38 tooth. Mouth D. Tripsin
opening is limited down to 1 cm. It is E. Sodium fluoride
necessary to extract the 38 tooth. What 24. A 38-year-old female patient complai-
kind of anesthesia should be applied for ns about pain in projection of the external
operation? acoustic meatus, clicking during mouth
A. Bercher-Dubov opening, ear stuffiness. Objectively: the
B. Mandibular face is symmetric, mouth is opened in
C. Torus staright path. There is Kennedy I type
D. Extraoral mandibular technique dentition defect, the 18, 17, 16, 26, 27,
E. Tuberal 28 teeth are missing. What anatomical
formation carries the maximum load?
21. One minute after perfoming torus
anaesthesia with 2% novocaine solution
Krok 2 Stomatology 2010 4

A. Articular disk (meniscus) A. Collapsible prosthesis


B. Articular head B. Clasp dental prosthesis
C. Distal clivus of medial articular tubercle C. Clasp dental prosthesis with
D. Glenoid fossa floor of temporal bone attachments
E. Joint capsule D. Pivot-point removable prosthesis
E. Partial removable lamellar prosthesis
25. Physical examination of a patient
revealed a hard tissues defect of the 24 29. X-ray picture depicts a circular well-
tooth. Index of occlusal surface destructi- defined area of bone tissue destruction
on was 0,8. The defect was restored by 0,7х 0,7 cm large in the projection of root
filling that doesn’t meet the requirements. apex. What is the most likely diagnosis?
The tooth is of rose colouring. X-ray
picture shows no patological processes. A. Cystogranuloma
What construction is indicated in this B. Cyst
case? C. Granuloma
D. Odontoma
A. Pivot crown E. Osteoma
B. Artificial crown
C. Inlay 30. A 32-year-old patient complains
D. Portion crown about a cosmetic defect caused by parti-
E. Equator crown al teeth missing. Objectively: the 15,
14, 24, 25 teeth are missing, the central
26. A 49-year-old male patient consulted group of upper jaw teeth is in protrusi-
a dental surgeon about an enlarging on and presents with diasthemas. What
"mother’s mark", itch and desquamati- orthodontic appliance should be used to
on. According to patient, changed in color prepare the oral cavity for prosthetics?
and size skin area appeared a year ago
after a shaving trauma. Objectively: there A. Palatine plate with vestibular bow
is an intensely pigmented brown spot up B. Schwarz’ guard
to 2 cm large with small oval protruding C. Palatine plate with oblique plane and
nodules on its surface in the infraorbital vestibular bow
region on the left. Desquamation factors D. Brueckl’s appliance
are present. Palpation is painless. Regi- E. Expansion plate for the upper jaw with
onal lymph nodes are matted together wi- orthodontic lock
th skin and painless. What is the most li-
kely diagnosis? 31. A 35-year-old patient has teeth mobi-
lity of I degree, the teeth have apparent
A. Melanoma dental cusps. The dentist recommends to
B. Pigmented nevus do occlusive teeth grinding. What method
C. Verrucous nevus is the most objective for determining the
D. Epidermoid cancer grinding topography?
E. Papillomatosis
A. Occlusiography
27. A soldier was delivered to the regiment B. Masticatiography
medical station with dislocation asphyxia C. Roentgenography
caused by a gunshot wound. What actions D. Gnathodynamometry
should be taken for asphyxia suppression? E. Rubinov’s masticatory test

A. Pul the tongue forwards and sew it 32. A 3-year-old girl complains about
through pain and tumescence in the region of the
B. Intubation of trachea decayed 51, 52 teeth, body temperature
C. Tracheostoma establishment rise up to 37, 5 − 37, 9oC. Objectively:
D. Artificial airway the face is asymmetric because of a
E. Artificial pulmonary ventilation tumescence in the upper lip region and
right infraorbital region. The crown of the
28. A 58-year-old male patient has consi- 51 tooth is completely decayed. Mucous
derable microstomia as a result of a membrane in the region of the 52, 51,
face trauma and burn. The patient has 1 teeth is edematic, mucogingival fold is
indication for fabrication of a partial smoothed, palpation provokes pain, mobi-
removable prosthesis. What construction lity of I-II degree of the 51, 52 teeth is also
of prosthesis should be used in this case? present. What is the most likely diagnosis?
Krok 2 Stomatology 2010 5

A. Acute purulent odontogenic maxillary doesn’t make a fold, its hyperemic and
periostitis glossy. The mouth can be opened by 3 cm.
B. Acute albuminous odontogenic maxi- Deglutition is painless. These clinical fi-
llary periostitis ndings correspond with the following di-
C. Acute odontogenic maxillary sease:
osteomyelitis
D. Odontogenic abscess of infraorbital A. Odontogenous phlegmon of the right
region submandibular region
E. Exacerbation of chronic periodontitis B. Abscess of the right alveololingual
of the 51 tooth groove
C. Adenophlegmon of the right submandi-
33. A 6-year-old boy hit his forehead one bular region
day ago. A few hours later a swelling D. Acute odontogenous sialoadenitis
appeared in the right superciliary regi- E. Phlegmon of pterygomandibular space
on. Objectively: there is a considerable
edema of forehead tissues spreading to 37. Preventive examination of tongue
eyelids of the right eye, the skin over the back of a 6-year-old child revealed areas
swelling is cyanotic, the swelling is of soft of epithelium desquamation in form of
consistency. Fluctuation is also present. red oval spots located close to the zones
General condition of the boy is normal. of hyperkeratinization of filiform papillae.
Make a provisional diagnosis: Clavate papillae are hypertrophic. There
are no complaints. The child has a history
A. Hematoma of the right superciliary of intestinal dysbacteriosis. What is the
region most likely diagnosis?
B. Postraumatic edema of tissues of the
right superciliary region A. Glossitis areata exfoliativa
C. Fracture of frontal bone B. Candidal glossitis
D. Hematic abscess of the right supercili- C. Acute catarrhal glossitis
ary region D. Rhomboid glossitis
E. Inflammatory infiltration of tissues of E. Herpetic affection of tongue
the right superciliary region
38. During endodontic treatment of peri-
34. A 45-year-old patient complains about odontitis a tool was broken in the mi-
pain in his mandible that arose after ddle third of the medial root. Choose the
extraction of the 36 tooth. Objectively: treatment method:
alveolar socket is covered with bloody
clot. X-ray picture shows unextracted root A. Medial root hemisection
of the 36 tooth. What tools are necessary B. Medial root amputation
for extraction of this root? C. Crown radicular separation
D. Resection of medial root apex
A. Angled elevator curved right E. Tooth extraction
B. Angled elevator curved left
C. Straight elevator 39. A 14-year-old child complains about a
D. S-shaped forceps cosmetic defect in the frontal teeth regi-
E. Bayonet-shaped forceps on. Objectively: enamel of the 11, 12,
21, 22, 31, 32, 41, 42 teeth is thin in the
35. A 70-year-old patient consulted a region of cutting edge, there is a sulcate
dental surgeon about extraction of the enamel pit 1,5 mm wide which encircles
central upper jaw incisors with III degree the tooth and is parallel to the cutting
mobility. What tools should be applied? edge. The cusps of the 16, 26, 36, 46 teeth
are underdeveloped and have conical
A. Straight forceps form. What is the most likely diagnosis?
B. Straight elevator
C. Beak-shaped forceps A. Systemic hypoplasia
D. Bayonet-shaped forceps B. Dentinogenesis imperfecta
E. S-shaped forceps C. Enamel dysplasia
D. Local hypoplasia
36. A patient complains of pain and swelli- E. Stainton-Capdepont dysplasia
ng in the right submandibular area. She
has been treating the 45 tooth for a week. 40. Examination of a 9-year-old child
Objectively: body temperature is 38oC. revealed protrudent chin, the lower lip
There is a painful tense infiltration in overlapping the upper lip. There are di-
the right submandibular region. The skin astemas and tremas beetwen the lower
Krok 2 Stomatology 2010 6

incisors, the lower incisors overlap the


upper ones by 2/3 of crown height. Sagi- A. Gypseous impression of the whole face
ttal fissure is 3 mm. Specify the treatment B. Impression of the whole face with
tactics: thermoplastic material
C. Impression of the defect with elastic
A. Brueckl’s appliance material
B. Bynin’s guard D. Impression of the defect with
C. Schwartz’ quard thermoplastic mass
D. Angle’s sliding appliance E. Gypseous impression
E. Myogymnastics complex
45. A 53-year-old patient consulted a
41. It is planned to organize a dental room prosthodontist about lateral teeth mobili-
with three universal dental sets. The total ty, frequent falling out of fillings. Objecti-
area of the room must be: vely: all the molars and premolars of
the lower jaw exhibit I degree mobility.
A. No less than 28 square metres Approximal masticatory surfaces have fi-
B. No less than 21 square metres llings. What splint construction should be
C. No less than 30 square metres applied in this case?
D. No less than 42 square metres
E. No less than 18 square metres A. Inlay splint
B. Fixed crown splint
42. During pulpitis treatment of the 25 C. Crown cap splint
tooth a 30-year-old patient received an D. Equator crown splint
injection of 2% lidocaine as anaesthetic. E. Intradental splint
A few minutes after the injection th
patient presented with numb tongue, 46. A 35-year-old woman was admi-
asphyxia, dry cough, cyanosis, anxiety, tetd to the oral surgery department two
convulsions. Arterial pressure fell down hours after a road accident. Objectively:
to 90/60 mm Hg. What is the most likely cutaneous integuments are pale, there are
diagnosis? beads of sweat on her face. Respirati-
on is heavy and obstructed. Pulse rate
A. Anaphylactic shock is 120 bmp, AP is 70/60 mm Hg. There
B. Acute heart failure is wound penetrating to the oral cavity
C. Syncope on the left cheek. During transportation
D. Quincke’s edema the woman was in supine position with
E. Epilepsy her head thrown backwards. What kind of
asphyxia is likely to develop in this pati-
43. A 50-year-old patient has median ent?
lower jaw fracture with formation of false
joint. The 38, 32, 31, 41, 42, 48 teeth are A. Aspirating
missing. The remaining teeth are intact, B. Valvular
stable. There is no displacement of lower C. Stenotic
jaw fragments. X-ray picture shows a bone D. Obturative
tissue defect 1 cm large. What prosthesis E. Dislocational
is indicated?
47. Parents of a 4-year-old child complain
A. Oxman’s bridge-like prosthesis with about speech defect, namely lallati-
pivot point on. Examination shows limited tongue
B. Clasp denture movements, when the tongue is moved
C. Lamellar prosthesis with Gavrilow’s forward it turns down, the lower edge
pivot point of tongue frenulum is positioned in front
D. Lamellar prosthesis with Oxman’s pivot of the submandibular salivary ducts. The
point frenulum is thin and transparent. Specify
E. Bridge-like prosthesis without a pivot the terms of operative intervention:
point
A. After making the diagnosis
44. A 60-year-old male patient has an B. After complete development of maxi-
extensive defect of nose caused by an llofacial bones
injury. The patient temporarily refused C. After eruption of permanent incisors
operative intervention, it was suggested D. After formation of permanent occlusion
to fabricate an external nasal prosthesis E. After eruption of permanent molars
fixed by eyeglass frame. What impression
material should be used? 48. A 43-year-old patient complains about
Krok 2 Stomatology 2010 7

teeth mobility in the frontal region of his tooth has a carious cavity, probing causes
lower jaw, as well as significant cervixes no pain, the pulp chamber is closed. What
exposure. Objectively: gums in the regi- method of treatment is indicated?
on of the 44, 43, 42, 41, 31, 32, 33, 34
teeth are pale with cyanotic tint. The 42, A. Vital extirpation
41, 31, 32 teeth present with I-II degree B. Devital extirpation
mobility. The 42, 41, 31, 31 teeth are C. Devital amputation
overcrowded. The 42, 41, 31, 32 teeth D. Biological method
have cervix exposure by 1/2 and the 43, 33 E. Vital amputation
teeth have cervix exposure by 1/4. What
orthopaedic construction should be used 52. 3 months after the immediate
in this situation? prosthetics a patient complained about
the prosthesis balancing. Objectively:
A. One-piece guard there is a gap between the prosthesis edge
B. Cast bar Kurliandsky splint and vestibular surface of alveolar process.
C. Cap splint What is the tactics of choice?
D. Portion crown splint
E. - A. Fabrication of a new prosthesis
B. Correction of prosthesis edges
49. Preventive examination of an 8-year- C. Prosthesis relocation with quick-setting
old boy revealed some lusterless chalk- plastic
like spots on the vestibular surface of the D. Dentition buildup with quick-setting
11 and 21 teeth, which are localised in the plastic
precervical region. Subjective complaints E. Correction of dentitions
are absent. What is the most likely di-
agnosis? 53. A patient undergoes orthopaedic
treatment of bounded edentulous spaces
A. Acute initial caries on the upper jaw. He needs fixed full-
B. White-spotted fluorosis cast dentures. During his second visit it
C. Local enamel hypoplasia is required to check whether the internal
D. Acute superficial caries surface of the metal framework of the
E. Chronic initial caries future metal-ceramic denture matches the
surfaces of the prepared teeth. In what
50. A 66-year-old male patient suffers way could this be done?
from coronary heart disease (CHD) and
atherosclerosis. During stomatological A. In the oral cavity by means of silicone
treatment the patient complained about materials
acute retrosternal pain with irradiation B. Visually by means of models in the
to the left scapula, that was accompani- articulator
ed by stupor of the left hand. Objecti- C. In the oral cavity by means of tracing
vely: the skin was pale, the sweat stood paper
out on his forehead. The arterial pressure D. In the oral cavity by means of a wax
was 140/90 mm Hg, pulse was rhythmic 75 plate
bpm. The pain wasn’t relieved by Validol E. In the oral cavity by means of
but it started to abate after nitroglycerin stomatoscopic method
intake. What disease provoked the attack
in this patient? 54. A 19-year-old girl complains about
having crusts, lip tenderness, especi-
A. Stenocardia ally at lip joining. Objectively: there
B. Myocardial infarction are yellow-brown crusts on the lip red
C. Hypertensive crisis border from Klein zone to it’s middle,
D. Tachycardia after their removal bright red smooth
E. Paroxysmal tachycardia surface without erosions appears. Mucous
membrane in Klein zone is slightly
51. A patient complains about intensive hyperemic and edematic. What is the most
throbbing toothache in his lower jaw on likely diagnosis?
the left. He can’t show the aching tooth.
It hurts all the time, the pain goes down A. Exudative form of cheilitis exfoliativa
sometimes, but it becomes stronger as B. Exudative form of cheilitis actinica
affected by stimuli. Within the last hours C. Epidermolysis bullosa
the pain provoked by cold stimuli has D. Meteorological cheilitis
somewhat reduced. Objectively: the 17 E. Eczematous cheilitis
Krok 2 Stomatology 2010 8

55. A 43-year-old patient complains about biting down on food, sensation of a


constant pain in the upper jaw region on recently erupted tooth on the right upper
the right, that irradiate to the temple. The jaw. Examination of the 15 tooth revealed
pain was noted one month ago. Objecti- a deep carious cavity communicating with
vely: the face is asymmetric because of the the dental cavity. Tooth percussion causes
swollen right cheek. Mucous membrane acute pain. Probing is painless. Mucous
has no changes. The breathing through membrane in projection of the root apex is
the right nasal meatus is obstructed, hyperemic, painful on palpation. X-ray pi-
there are foul-smelling saniopurulent di- cture shows no changes. What is the most
scharges. Plan X-ray film of facial skeleton likely diagnosis?
in the semi-axial projection shows intensi-
ve opacity of the right maxillary sinus A. Acute purulent periodontitis
and violated intactness of its interior and B. Acute diffuse pulpitis
superior walls. What is the most likely di- C. Exacerbation of chronic periodontitis
agnosis? D. Acute albuminous periostitis
E. Acute odontogenic osteomyelitis
A. Upper jaw cancer
B. Chronic odontogenic maxillary sinusitis 59. A 48-year-old male patient has been
C. Chronic odontogenic osteomyelitis wearing partial removable dentures for
D. Neuritis of the II branch of trigeminus 3 months. According to the patient,
E. Upper jaw osteoma results of physical examination and addi-
tional methods of testing, the patient
56. A 44-year-old patient consulted a was diagnosed with allergic stomatitis
surgeon about constant acute pain in the provoked by dyes of the acrylic resin of
upper jaw region on the left that is getti- his prosthesis. Allergic effects of dyes can
ng worse during teeth joining. The pain be eliminated in the following way:
was noted 3 days ago. Objectively: the
face is symmetrical, mouth opening is A. Ffabrication of a denture out of
unlimited. The crown fof the 26 tooth colorless plastic
is half-destroyed. Probing of the cari- B. Fabrication of bilayer bases
ous cavity is painless. Percussion of the C. Moulding of plastic by method of
26 tooth provokes acute pain. Mucous casting
membrane of the alveolar process is D. Fabrication of cast metal prosthetic
edematic, hyperemic on the level of the bases
26 tooth. The 26 tooth was treated before. E. Fabrication of swaged metal bases
What is your provisional diagnosis?
60. A 43-year-old patient complains about
A. Exacerbation of chronic periodontitis a neoplasm in the right submandibular
of the 26 tooth region that appeared a month ago after
B. Acute purulent periodontitis of the 26 angina. The patient underwent anti-
tooth inflammatory therapy but it led to no
C. Acute pulpitis of the 26 tooth reduction of the neoplasm. Objectively:
D. Acute purulent periostitis of upper jaw body temperature is up to 37, 2oC. Palpati-
of the 26 tooth on the left on reveals a slightly painful, spherical,
E. Periodontitis of the 26, 27, 28 teeth freely movable, well-defined neoplasm of
dense and elastic consistency in the ri-
57. Parents of a 6-year-old child consulted ght submandibular region. The duct of
a dentist about oral cavity sanitation. submandibular salivary gland discharges
Objectively: the 85 tooth has a carious transparent saliva. The sublingual plica is
cavity on the distal surface within the unchanged. What is the most likely di-
mantle dentine. Floor and walls are dense agnosis?
and pigmented. Probing is painless. Cold
stimuli and percussion cause no pain. A. Chronic lymphadenitis
What is your provisional diagnosis? B. Chronic sialoadenitis
C. Sialolithiasis
A. Chronic median caries D. Adenoma of salivary gland
B. Acute median caries E. Atheroma
C. Chronic deep caries
D. Chronic periodontitis 61. A patient ordered soldered stainless-
E. Chronic fibrous pulpitis steel bridge prostheses. Their fabricati-
on involves calibration of crown sleeves.
58. A patient complains about acute Which device is used for this purpose?
constant pain that is getting worse when
Krok 2 Stomatology 2010 9

A. Samson brightly hyperemic, lacunae are enlarged


B. Parker’s and have necrosis areas. Regional, cervi-
C. Larin’s cal, occipital lymph nodes are painful,
D. Cope’s enlarged and dense. What is the most li-
E. Bromshtrom kely diagnosis?
62. A 20-year-old patient complains about A. Infectious mononucleosis
unaesthetic look of the 24 tooth from B. Acute herpetic stomatitis
the moment of its eruption. Objectively: C. Necrotizing ulcerative gingivostomatitis
enamel of the 24 tooth is partly absent, D. Herpetic angina
the dentine is yellow. The 64 tooth was E. Lacunar tonsillitis
treated more than once when she was a
child, but because of frequent exacerbati- 66. A 28-year-old patient suffers from a
ons, edemas and gingival fistula the 64 disease without prodromal manifestations
tooth was extracted when she was 9 years that declares itself through oral mucosa
old. What is the most likely diagnosis? lesion consisting of 1-2 roundish elements
5-8 mm large which are circumscribed by
A. Localised hypoplasia a hyperemic rim and covered with yellow-
B. Initial caries grey coating. The disaese recurrence is
C. Fluorosis observed quite regularly 3-4 times a year.
D. Systemic hypoplasia These presentations are typical for the
E. Enamel aplasia following disease:
63. A 50-year-old female patient complai- A. Chronic recurrent aphthous stomatitis
ns about sensation of tightness of B. Lichen ruber planus
buccal mucosa and roughness of the C. Chronic herpes recidivicus
lateral surface of tongue. The patient D. Erythema multiforme
undergoes regular check-up at a di- E. Papular syphilis
spensary department for compensated
form of diabetes mellitus. Objectively: 67. A 48-year-old female patient complai-
there are white and grey areas in form of ns about some discomfort induced by
lacy pattern on the buccal mucosa on the wearing an upper jaw clasp denture wi-
right and on the lateral surface of tongue. th elastic attachments. The patient notes
The surface of affected region cannot be the denture mobility during mastication
scraped off. What is the most likely di- and articulation. The patient asks to fabri-
agnosis? cate a new denture with consideration
of the indicated defects. Clinical crowns
A. Lichen ruber planus of abutment teeth are low. What fixation
B. Secondary syphilis should be the most efficient in this case?
C. Lupus erythematosus
D. Pseudomembranous candidiasis A. Telescopic
E. Leukoplakia B. Arch-bar
C. Locking
64. A 45-year-old patient consulted a D. Saddle-type
dentist about extraction of the 13 tooth. E. Clasp
What tools should be applied for extracti-
on? 68. A 75-year-old patient ordered a
complete removable denture for the
A. Straight forceps upper jaw. While adjusting the individual
B. S-shaped forceps curved right tray by Herbst’s method the orthodontist
C. Bayonet-shaped forceps revealed that the tray could be thrown off
D. Straight elevator when the patient rounded his lips. What
E. S-shaped forceps part of the tray requires correction?
65. According to the mother, a 5-year- A. Vestibular part between the canines
old child complains about pain during B. Distal part along the A line
swallowing, weakness, body temperature C. In the region of buccal cords
rise upt to 39, 5oC, swelling of submental D. Along the whole vestibular edge
lymph nodes. Objectively: the child’s E. Behind the maxillary tuber
condition is grave, body temperature is
38, 8oC. Mucous membrane of oral cavity 69. A 36-year-old woman complains about
is brightly hyperemic and edematic with experiencing lip dryness and desquamati-
haemorrhages and ulcerations. Pharynx is on for a month. Application of indifferent
Krok 2 Stomatology 2010 10

ointments is ineffective. Objectively: red A. Glossophytia


border of lower lip is of rich red color, B. Median rhomboid glossitis
it is moderately infiltrated, covered with C. Benign migratory glossitis
closely adhering greyish scales, it bleeds D. Fissured tongue
and hurts in the attempt to remove them. E. Acute glossitis
Opacification of epithelium in form of
white stripes is present in the nidus peri- 73. A patient complains about destroyed
phery, there is also an area of depression crown of the 27 tooth. The patient was di-
in the centre. What is the most likely di- agnosed with chronic periodontitis of the
agnosis? indicated tooth. Objectively: the crown of
the 27 tooth is destroyed completely. It is
A. Lupus erythematosus necessary to extract this tooth. What fi-
B. Commissural cheilitis eld block anesthesia should be applied for
C. Lichen ruber planus operation?
D. Leukoplakia
E. Cheilitis exfoliativa A. Tuberal and palatinal
B. Infraorbital
70. A 30-year-old patient complains about C. Tuberal
pain in the upper jaw tooth caused by D. Infraorbital and tuberal
sweet and cold food, that was first regi- E. Tuberal and incisor
stered one month ago. Objectively: the 26
tooth has a carious cavity within mantle 74. A 32-year-old patient complains about
dentine. Dentine of its walls is softened. a cosmetic defect. Objectively: the crowns
Probing along the enamel-dentine juncti- of 12, 11, 21, 22 teeth are destroyed by
on is painful. Percussion is painless. caries by 2/3 of their height. The occlusi-
Thermodiagnosis causes pain, that quickly on is orthognathic. X-ray picture shows
abtes after elimination of stimulus. What that root canals are filled to the top. There
is the most likely diagnosis? are no patological changes in periapical
tissues. What construction should be the
A. Acute median caries most efficient in this situation?
B. Acute initial caries
C. Pulp hyperaemia A. Stump inlays and metal-ceramic crowns
D. Acute deep caries B. Artificial metal swaged crowns
E. Chronic fibrous pulpitis C. Restoration of tooth crowns with
composite materials
71. A 55-year-old patient has a repeated D. Fabrication of plastic inlays
appointment with a dentist. He has E. Richmond’s pivot crowns
ordered complete removable lamellar
dentures for both upper and lower jaws. 75. A 25-year-old male patient came to the
During his previous visit the dentist traumatology. He complains about pain in
took complete anatomic impressions of the right half of his face, limited mouth
both jaws and sent them to the dental opening. Objectively: there is a moderate
mechanic. What manipulations should the edema and haematoma in the region of
dentist perform at the next clinical stage? the right malar arch. Palpation shows a
slightly painful impaction of bone tissue
A. Adjust individual trays in the region of the right malar arch. The
B. Locate and fix central occlusion bite isn’t disturbed. Amplitude of mouth
C. Check the prosthesis construction opening is 1,5 cm. At making an attempt
within the oral cavity to open the mouth wider the patient feels
D. Correct the prosthesis and give the a mechanical obstacle and pain intensifi-
patient necessary instructions cation. What is the most likely diagnosis?
E. Fabricate occlusal rims
A. Fracture of the right malar arch
72. A 47-year-old patient complains about B. Fracture of the right zygomatic bone
a sensation of foreign body on his tongue, C. Traumatic arthritis of TMJ
discomfort during talking, oral cavity D. Le Fort III maxillary fracture (upper)
dryness. Objectively: there are dark fili- E. Le Fort II maxillary fracture
form papillae up to 5 mm long on the
back of tongue. What is the most likely 76. The patient is 75 years old. The
diagnosis? inspection of construction of complete
removable dentures revealed that only
lateral artificial teeth were in contact,
there was a gap between frontal teeth.
Krok 2 Stomatology 2010 11

In the lateral part on one side cusp-to- A. Smooth brace


cusp contact was present, on the other side B. Splint with spacer bar
there was a horizontal gap; the central li- C. Splint with guide plane
ne was deviated. What mistake was made? D. Anchor splint with intermandibular
fixation
A. The patient was thought to have lateral E. Ivy loops for 31, 32, 42, 41
occlusion
B. Occlusal rims were irregularly softened 80. A 27-year-old woman has been treated
C. The patient was thought to have anteri- for pulpitis by method of devital extirpati-
or occlusion on. Arsenic paste was left in the 15 tooth.
D. Interalveolar height was increased The patient came to see a dentist for the
E. Wax moulds were deformed while second time only on the fourth day after
determining central occlusion her previous visit. She was diagnosed with
toxic periodontitis. What is the most opti-
77. A 5-year-old child sustained a dental mal agent for treatment of root canal in
injury. Objectively: the crowns of the 51, this case?
61 teeth are shorter than neighbouri-
ng teeth by 1/2. Mucous membrane is A. Unithiol
edematic and hyperemic in theregion of B. Cresophene
the 51, 61 teeth. X-ray picture shows that C. Tripsin
there is no periodontal fissure in the apical D. Eugenol
parts of roots of the 51, 61 teeth, apexes E. Hydrocortisone emulsion
of the 51, 61 teeth are imbedded into
the spongy substance of body of maxilla. 81. A 37-year-old female patient complai-
What treatment tactics would be the most ns about pain, gingival haemorrhage,
efficient? halitosis, body temperature rise up
to 37, 2oC. Objectively: gums are
A. Extraction of the 51, 61 teeth apparently hyperaemic, edematic, bleed
B. Regular medical check-up easily, parodontal pouches are 3-
C. Reposition of the 51, 61 teeth 4 mm deep and contain purulent
D. Ligature splinting of the 51, 61 teeth exudate. Orthopantomogram shows di-
E. Reimplantation ffuse osteoporosis of alveolar process,
resorption of interdental septa down to
78. A 5-year-old child had to undergo 1/3 of their height. What is the most likely
an operated for ankyloglossya. There diagnosis?
were no external symptoms of preoperati-
ve anxiety. Aafter injection of 1,5 ml A. Exacerbation of generalized I degree
of 0,5% novocaine solution the child periodontitis
presented with motor anxiety, vomituri- B. Exerbation of chronic catarrhal gingivi-
tion, stomachache. Objectively: the chi- tis
ld is conscious, face and neck skin is C. Chronic generalized I degree peri-
hyperemic, tachypnoe is present, pulse is odontitis
rapid. What is the most likely diagnosis? D. Chronic generalized II degree peri-
odontitis
A. Allergic reaction to novocaine E. Exacerbation of generalized II degree
B. Pain shock periodontitis
C. Cardiovascular collapse
D. Giddiness 82. A 24-year-old woman complains
E. Overdosage of anesthetics about severe pain in the mouth, body
temperature up to 38o C, indisposition.
79. A 19-year-old patient complains about The same condition occurs periodically
pain in the submental part of mandible. for several years after catching a cold.
The day before he got a trauma. Objecti- Objectively: the lips are covered with
vely: there is a slight swelling of tissues bloody crusts, there are opened bladders
in the mental area. The mouth can be and erosions, covered with fibrinogenous
opened widely enough. All the teeth are deposit on mucous membrane of lips
intact. Mucous membrane is edematic in and cheeks that is apparently hyperemic
the region of central incisors, it bleeds sli- and edematic. Hypersalivation is present.
ghtly. In this region mobility of mandible What is the most likely diagnosis?
fragments is present. Occlusion is undi-
sturbed. What splint should be chosen by
the dentist?
Krok 2 Stomatology 2010 12

A. Erythema multiforme
B. Pemphigus vulgaris A. Physiological retrogenia
C. Dermatitits multiformis, Duhring’s B. Physiological progenia
disease C. Central occlusion
D. Nonacantholytic pemphigus D. Direct relation
E. Chronic herpes recidivicus E. Posterior occlusion
83. A 30-year-old patient complains 87. A 3-month-old child has been in
about a carious cavity. Objectively: the disease state for two days. The chi-
16 tooth is discoloured and has a deep ld is anxious, refuses food, has normal
carious cavity communicating with the body temperature. Objectively: mucous
dental cavity. Probing, percussion cause membrane of oral cavity is edematic and
no pain. Electroodontodiagnosis is 100 hyperemic. There is white caseous coati-
microampere. X-ray picture shows wideni- ng on the back of tongue and buccal
ng of periodontal fissure. What is the most mucosa. After the coating removal one
likely diagnosis? can see extremely hyperemic surface with
petechial haemorrhages. What is the most
A. Chronic fibrous periodontitis likely diagnosis?
B. Chronic granulating periodontitis
C. Chronic granulomatous periodontitis A. Acute candidal stomatitis
D. Chronic gangrenous pulpitis B. Mild leukoplakia
E. Chronic deep caries C. Acute herpetic stomatitis
D. Herpetic angina
84. A 65-year-old patient complains E. Lichen ruber planus
about unsatisfactory fixation of complete
removable lamellar denture of his upper 88. A 35-year-old patient consulted a
jaw. The denture was fabricated 6 years dentist about extraction of the 14 tooth
ago. Objectively: balancing and poor fi- because of exacerbation of chronic peri-
xation of complete removable denture is odontitis following ineffective therapeutic
present. What is the reason of such condi- treatment. What tools should be applied
tion? for extraction?
A. Atrophy of osseous base of the A. S-shaped forceps
prosthetic bed tissues B. Crown bayonet-shaped forceps
B. Wear of artificial teeth C. S-shaped forceps curved right
C. Bad hygiene of removable denture D. Straight forceps
D. Discoloration of basic plastic E. Root bayonet-shaped forceps
E. Loss of certain antagonists
89. The 12, 22 teeth of an 8-year-old child
85. A 10-year-old boy complains about are missing. There is not enough space in
missing teeth. Objectively: the face is dentition for them. X-ray picture shows
symmetrical, disproportional because of no tooth germs. The 12 tooth of the chi-
shortening of the lower third. In the oral ld’s father is missing and the 22 tooth
cavity: the 12, 14, 15, 17, 22, 24, 25, 27, 34, is conoid. What is the reason for such
35, 37, 44, 45, 47 teeth are missing. X-ray pathological changes?
picture shows partial adentia and absence
of some tooth germs. Choose the most A. Hereditary adentia
efficient prosthetic device: B. Caries
C. Extraction of teeth
A. Partial removable lamelalr prosthesis D. Trauma
for both jaws E. Rachitis
B. Bridge prostheses
C. Clasp dental prostheses 90. A 35-year-old patient complains about
D. Cantilever dental bridges constant dull pain in the 25 tooth that is
E. The defect should be restored by getting worse when biting down on food.
implants Objectively: masticatory surface of the 25
tooth has a carious cavity communicati-
86. A child was born with body weight at a ng with the dental cavity. The purulent
rate of 3200 g and body length at a rate of discharges from the canal followed the
53 cm, 9 points on Apgar score. It was the probing. What method of diagnostics
first physiological delivery. What positi- should be applied to confirm the di-
on of child’s mandible is usually observed agnosis?
after birth?
Krok 2 Stomatology 2010 13

A. X-ray examination A. To remove the fragment and stitch up


B. Electric pulp test the wound
C. Thermal test B. To try to restore the fragment to its
D. Bacteriological examination place
E. Deep probing C. To restore the fragment to its place and
fix it there
91. After complex extraction of the 37 D. To remove the fragment
tooth a patient experienced anaesthesia E. To remove the fragment and tampon
of the left half of his lower lip and chin. the wound
Electroodontodiagnos showed reduction
of lower jaw teeth electroexcitability on 94. Preventive examination of a 4-year-
the left. What is the most likely diagnosis? old child revealed a deep carious cavi-
ty on the masticatory surface of the 54
A. Neuritis of the left inferior alveolar tooth. The cavity has no intercommuni-
nerve cation with the tooth cavity and is filled
B. Neuralgia of the left inferior alveolar with dense dentine. Probing, percussion,
nerve thermal test of the 54 tooth provoke no
C. Alveolitis in the region of socket of the pain. The decay/filled index is 1, hygiene
37 tooth index is 1,9. What filling material is the
D. Herpes Zoster n.Trigemini most suitable for permanent filling of the
E. Acute osteomyelitis of mandible body 54 tooth?
92. A female patient consulted a denti- A. Glass-ionomer cement
st about dental prosthetics. Objectively: B. Silicate cement
the lower third of the patient’s face is C. Silicophosphate cement
shortened, nasolabial folds are deepened, D. Chemical composite
frontal group of lower and upper jaw teeth E. Photopolymer composite
is missing, the crowns of the 17, 15, 26, 27,
36, 37, 45, 47 teeth are worn off by 2/3 95. A 34-year-old male patient complai-
of their height, masticatory surfaces of ns about acute spasmodic pain in the
these teeth are smooth and pigmented. region of his upper jaw on the left that
The alveolar process isn’t hypertrophied. is getting worse as affected by cold sti-
Interalveolar height is decreased. Specify muli. Toothache irradiates to the ear and
the form of pathological tooth wear in this temple. He had acute toothache of the 37
patient: tooth one year ago, but he didn’t consult
a dentist. Pain recurred three days ago.
A. Horizontal, noncompensated, III grade Objectively: the 37 tooth has a carious
severity cavity communicating with the dental
B. Horizontal, compensated, II grade cavity. Probing of the opened carious
severity cavity is extremely painful. X-ray picture
C. Vertical, noncompensated, III grade shows widening of periodontal fissure at
severity the root apex of the 37 tooth. What is the
D. Vertical, compensated, III grade severi- most likely diagnosis?
ty
E. Combined, noncompensated, III grade A. Exacerbation of chronic pulpitis
severity B. Exacerbation of chronic granulating
periodontitis
93. A 25-year-old man consulted a dentist C. Exacerbation of chronic fibrous peri-
about extraction of the 18 tooth, it’s crown odontitis
is destroyed by 1/2. The tooth was treated D. Acute diffuse pulpitis
more than once. During tooth extraction E. Acute purulent pulpitis
the maxillary tuber was accidentally torn
off. What actions should be taken? 96. A 69-year-old patient underwent an
operation for complete removal of his
lower jaw. They took impressions of
both jaws and fabricated a lower jaw
replacement prosthesis before operation.
What elements will provide fixation of this
prosthesis in the oral cavity?
Krok 2 Stomatology 2010 14

A. Foshar’s spiral springs the width of the 13 tooth is 11,4 mm, the
B. Wire loops distance between the 12 and 14 teeth is 4,6
C. Clasps mm, the width of the 14 tooth is 7,6 mm.
D. Magnets Suggest the treatment plan:
E. Attachments
A. Extract the 14 tooth and move the 13
97. Parents of a 7-year-old boy complain tooth into it’s place
about missing of the 11 tooth. Objectively: B. Open the bite and move the 13 tooth
there is enlargement of alveolar process into its due place
in projection of the 11, 12 teeth. X-ray pi- C. Move the 13 tooth into its due place
cture shows multiple shades of different without bite opening
size. They have dentate shape and look D. No treatment is required
like hard tooth tissues. What is the most E. Extraction of the 14 tooth
likely diagnosis?
101. Parents of an 8-year-old child
A. Odontoma of maxilla complain about a painful formation in
B. Follicular cyst of maxilla starting from the child’s oral cavity that obstructs food
the 11 tooth intake. The same complaints were regi-
C. Cementoma of maxilla stered two years ago. Mucous membrane
D. Adamantinoma of maxilla of lateral tongue surface is hyperemic and
E. Odontogenic fibroma of maxilla edematic. There is an oval erosion over
0,7 cm large covered with yellow greyi-
98. A 47-year-old patient complains about sh deposit. Erosion edges are hyperemic
discolouration of the 11, 12 teeth. Objecti- and painful on palpation. The child has a
vely: the 12, 12 are changed in colour, history of chronic cholecystocholangitis.
canals are filled to the top. It was deci- What is the most likely diagnosis?
ded to make metal-ceramic crowns for the
11, 12 teeth. What is the optimal angle for A. Chronic recurrent aphthous stomatitis
the preparation of approximal surfaces of B. Erythema multiforme
abutment teeth? C. Behcet’s syndrome
D. Stevens-Johnson syndrome
A. 5-8 degrees E. Traumatic erosion
B. 15-20 degrees
C. 10-15 degrees 102. A 50-year-old patient complains
D. 20-25 degrees about a cosmetic defect of the upper
E. 30-35 degrees frontal teeth. After meticulous examinati-
on the patient was diagnosed with wedge-
99. After preventive examination a shaped defect of the 11, 12, 13, 21, 22, 23
10-year-old child was diagnosed with teeth. What material should be chosen for
osteoporosis circumscripta of the 13, 12, the tooth filling?
11, 21, 22, 23 teeth. The patient was admi-
nistered electrophoresis with reminerali- A. Compomer restorative material
zing solutions. What preparations and in B. Silicate cement
what order are to be applied in this case? C. Glass-ionomer cement
D. Chemical-cure composite
A. Calcium and posphorus preparations E. Silicophosphate cement
with the following application of fluorine
praparation 103. A 12-year-old child complains about
B. Fluorine preparations with the followi- bleeding and pain in the 46 tooth duri-
ng application of calcium and phosphorus ng eating. He has a history of acute pain
preparations some time before. Objectively: there is
C. Calcium preparations only a deep carious cavity (Black’s class II)
D. Phosphorus preparations only communicating with the dental cavity,
E. Fluorine preparations only partially filled with overgrown pulp. Pulp
tissue is bleeding, painful on touch. There
100. A 12-year-old male patient consulted is soft white dental deposit. What is the
an orthodontist about odontoloxia. most likely diagnosis?
Objectively: the face is symmetric
and proportional. In the oral cavity:
permanent occlusion, occlusal relati-
onship is orthognathic in the lateral parts,
the 13 tooth is located off dentition on the
palate, biometric measurements show that
Krok 2 Stomatology 2010 15

A. Chronic hypertrophic pulpitis of chronic generalized catarrhal gingivitis.


B. Chronic papillitis This patient should be recommended to
C. Chronic simple pulpitis use a toothpaste with the following active
D. Chronic gangrenous pulpitis component:
E. Chronic granulating periodontitis
A. Chlorhexidine
104. Examination of a 6-year-old girl B. Calcium glycerophosphate
revealed a deep carious cavity in the 85 C. Monofluorophosphate
tooth. Percussion and probing are pai- D. Vitamins A, D, E
nless. After removal of the softened denti- E. Microelement complex
ne, communication with the tooth cavity
showed up. Deep probing is painless. X- 108. Examination of a male patient
ray picture of the 85 tooth shows the focus revealed that a tumour occupied the
of destruction of bone tissue in the region major part of his tongue; tongue mobility
of bifurcation; cortical plate of the 35 has was limited, there were solitary regional
no pathological changes. It is most expedi- nodes in the region of affection. After
ent to use the following material for the the cytological analysis the patient was di-
root filling: agnosed with cancer of tongue T2N1M0.
What treatment should be chosen for this
A. Zinc oxide eugenol cement patient?
B. Resorcin-formalin paste
C. Glass-ionomer cement A. Combined method
D. Phosphate cement B. Radiation therapy
E. Calcium-containing paste C. Surgical removal of tumor
D. Chemotherapy
105. A 33-year-old male patient complai- E. Removal of lymph nodes
ns about tongue pain that is getting worse
during eating and talking. Objectively: 109. A 22-year-old patient complains
there is a painful ulcer 0,6 cm large on about short-lasting spasmodic pain in
the lateral surface of tongue. The floor is the 26 tooth with long periods of pain
covered with grey deposit. The crown of absence. Objectively: the 26 tooth has a
the 47 tooth is destroyed. What is the most carious cavity in the masticatory surface,
likely diagnosis? dental cavity is not opened. Probing
causes pain in the pulp horn projecti-
A. Decubital ulcer on. Thermal stimuli cause sharp pain,
B. Trophic ulcer percussion is painless. Regional lymph
C. Hard chancre nodes are not palpable. What is the most
D. Tuberculous ulcer likely diagnosis?
E. Cancerous ulcer
A. Acute circumscribed pulpitis
106. A 27-year-old patient complains B. Acute deep caries
about pain of the 35 tooth caused by cold C. Acute diffuse pulpitis
stimuli, that quickly abates after elimi- D. Acute purulent pulpitis
nation of stimulus. Objective examination E. Exacerbation of chronic pulpitis
of the 35 tooth revealed a defect of hard
tooth tissue within enamel. Defect edges 110. A 27-year-old patient complains
are fragile and white. Electroodontodi- about gingival haemorrhage during tooth
agnosis is 5 microampere. What is the brushing. Objectively: gingival edge is
most likely diagnosis? hyperaemic, edematic, painful on touch.
Periodontal pouches are absent. X-ray pi-
A. Acute superficial caries cture shows resorption of cortical plate,
B. Local enamel hypoplasia osteoporosis of interdental septa. What is
C. Acute initial caries the most likely diagnosis?
D. Enamel necrosis
E. Enamel erosion A. Initial generalized periodontitis
B. Exacerbation of chronic catarrhal gingi-
107. A 13-year-old patient complains vitis
about gingival haemorrhage during tooth C. Generalized I degree periodontitis
brushing. Objectively: gums around all the D. Chronic catarrhal gingivitis
teeth are hyperemic and edematic, PMA E. Periodontitis
index (papillary marginal alveolary index)
is 46%, Greene-Vermillion hygiene index 111. A 25-year-old male patient got
is 2,5. Provisional diagnosis: exacerbation bilateral fracture of the lower jaw. A
Krok 2 Stomatology 2010 16

fragment in the region of the 44, 43, 42, 41, trapeziformed. Upper incisors overlap
31, 32, 33, 34 teeth is displaced downward lower incisors more than by 2/3. Incisors
and backward. What appliance should be and second molars are in the same relati-
used for the fragment reposition? on. There is no space between frontal
teeth. Upper dental arch is bigger than
A. Post’s appliance lower dental arch by the cheek tubercle
B. One-jaw bite-guard splint size. Bite abnormality is observed in the
C. Kurlyandsky’s appliance with levers following planes:
D. Weber’s appliance
E. Shur’s appliance A. Sagittal and vertical
B. Sagittal and lateral
112. A 9-year-old child complains about C. Sagittal and occlusal
pain caused by sweet and sour food in an D. Sagittal and nasal
upper tooth on the left. Objectively: the E. Sagittal and frankfurt
26 tooth has a carious cavity on the masti-
catory surface within the enamel limits. 116. Preventive examination of a 9-
What is the optimal material to fill the 26 year-old girl revealed broad bridge
tooth? of nose, narrow nasal passages, half-
opened mouth, problems with lip joini-
A. Сomposite ng, elongated lower third of face. The
B. Glass ionomer child presents with transitional occlusion.
C. Silicophosphate cement There is vertical gap 4-5 mm large from
D. Silicate cement the 53 to the 64 tooth in the frontal regi-
E. Zinc phosphate cement on. Relationship of the first permanent
molars complies with Angle’s class I. The
113. A 57-year-old patient complains child pronounces hissing sounds indisti-
about dental hypersensitivity to chemical nctly. Specify the most likely factor of
stimuli, gum itch. Objectively: roots are occlusion deformation:
exposed down to 1/3 of their length, gums
are dense and pale rose. Wedge-shaped A. Nasal respiration disorder
defects in the area of the 14, 13, 24, 25, B. Tongue parafunction
26, 34 teeth are within the dentine. Probi- C. Tongue sucking
ng of dental cervixes and wedge-shaped D. Infantile swallowing
defects is painfull. What is the most likely E. There is no correct answer
diagnosis?
117. A 40-year-old patient is hospitali-
A. I degree parodontosis zed with Le Fort II maxillary fracture.
B. Generalized periodontitis The teeth are intact. Choose the most
C. II degree periodontitis appropriate apparatus for this patient:
D. Atrophic gingivitis
E. Localised periodontitis A. Standard Zbarzh construction
B. Yadrova’s apparatus
114. A 7-year-old child was diagnosed C. Rudko’s apparatus
with chronic periodontitis of the 64, 85 D. Yermolaev-Kulagov’s apparatus
teeth. The child suffers from hemophilia E. Gunning-Port’s splint
A. Specify the treatment tactics:
118. A 57-year-old patient complains
A. Extraction of teeth in hematological about a slowly growing swelling in the
department after due pretreatment sublingual region. She noted it 3 months
B. Extraction of teeth is possible in both ago. Objectively: there is an elastic swelli-
in-patient and out-patient hospitals ng in the sublingual region. The swelling
C. Extraction of teeth can be performed is painless, mucous membrane over it is
in an out-patient hospital with the with tints of blue. What is the most likely
following socket tamponade by means diagnosis?
of haemostatic sponge
D. Extraction of teeth can be performed in A. Ranula
the oral surgery department B. Salivolithiasis
E. Extraction of teeth is contra-indicated C. Dermoid cyst of oral cavity floor
because of high risk of haemorrhage D. Lipoma of sublingual area
E. Hemangioma of sublingual area
115. Preventive examination of a 6-year-
old child revealed temporary teeth bi- 119. A 38-year-old patient consulted
te. Upper and lower dental arches are dental surgery about extraction of the 36
Krok 2 Stomatology 2010 17

tooth. What kind of anesthesia should be A. Fixed Oxman’s prosthesis with pivot
applied for extraction? point
B. Oxman’s single-jointed prosthesis with
A. Torus pivot point
B. Mandibular C. Gavrilow’s prosthesis with pivot point
C. Tuberal D. Kurlyandsky prosthesis with roller
D. Infiltration damper clasp
E. Voino-Yasenetsky truncal E. Oxman’s double-jointed prosthesis with
pivot point
120. A patient ordered partial removable
lamellar dentures for the upper and lower 123. A 39-year-old patient complains
jaw. An orthodontist took elastic alginate about experiencing pain in the region
impressions of both jaws. What is his next of the 21 tooth for 2 days. It is known
step? from the anamnesis that the indicated
tooth was treated for carious. Objecti-
A. To send the impressions for disinfection vely: the 21 tooth is covered with metal-
B. To let the impressions dry out in the ceramic crown, mucous membrane in
open air apex projection is hyperaemic. Percussion
C. To invite a dental mechanic for joint of the tooth is sharply painful. X-ray pi-
analysis of the impressions cture shows improperly filled root canal. It
D. To send the impressions immediately to is planned to take off the 21 tooth crown.
the laboratory What kind of anesthesia should be appli-
E. To put the impressions into the microten ed?
bag for 90 minutes
A. Field block anesthesia
121. A 42-year-old woman came to B. Infiltration anesthesia
prosthetic dentistry to make dental C. Intraligamentous anesthesia
prosthetics. Objectively: dental formula is D. Application anesthesia
18 . . . . 13 12 11 21 22 23 . . . . 28 E. Intraosseous anesthesia
48 47 46 45 44 43 42 41 31 32 33 34 35 36 37 .
The bite is deep, clinical crowns are low, 124. A 8-year-old child has a deep carious
survey line is not marked. The patient cavity communicating with dental cavi-
suffers from epileptiform attacks. What ty on the distal-approximal masticatory
prosthesis is indicated? surface of the 75 tooth. Probing causes
pain. Percussion is painless. Cold water
A. Partial removable lamellar prosthesis causes slowly abating pain. The tooth
with metal base decayed some months ago and wasn’t
B. Dental bridges treated. What treatment method is effi-
C. Partial removable lamellar plastic cient in this case?
prosthesis with retaining clasps
D. Partial removable lamelalr prosthesis A. Devital amputation
with supporting-retaining clasps B. Biological method
E. Clasp denture C. Vital extirpation
D. Vital amputation
122. A 58-year-old patient was diagnosed E. Devital extirpation
with fracture of lower jaw with formati-
on of a false joint. Objectively: the 38, 36, 125. A patient consults a prosthodonti-
32, 41, 43, 48 teeth are missing. Preserved st about a dentition defect of the 14, 15,
teeth are intact, stable. There is no di- 16 teetht. It is necessary to prepare teth
splacement of lower jaw fragments. X-ray for crowning. Before the preparation the
picture shows a bone tissue defect to up 2 handpiece must be processed with:
cm large. What construction of prosthesis
is indicated in this situation? A. With 3% solution of chloramine (two
times with 15 minute interval)
B. With 3% solution of hydrogen oxide
C. With 3% solution of chloramine once
D. With solution of potassium
permanganate
E. With furacilin solution
126. A 36-year-old female patient
complains about acute pain, clicking in
the right temporomandibular joint, burni-
Krok 2 Stomatology 2010 18

ng pain in the region of the right external patient revealed maximal number of
acoustic meatus. Movements of lower jaw occlusional contacts of opposing teeth
are step-like with short-standing disabli- during denture joining. X-ray picture
ng moments and acute pain in the joi- shows that the articular head of the lower
nt. Objectively: the face is symmetric. jaw is near the base of the slope of arti-
Occlusion is orthognathic. Palpation of cular tubercle. What type of occlusion is
lateral pterygoid muscles is painful on present?
the right. Tomogram shows that bone
structure circuits of joints are smooth and A. Central occlusion
slick. What is the most likely diagnosis? B. Anterior occlusion
C. Right lateral occlusion
A. Dysfunction of TMJ D. Left lateral occlusion
B. Rheumatic arthritis of TMJ E. Posterior occlusion
C. Acute posttraumatic arthritis of TMJ
D. Deforming arthrosis of TMJ 131. A 49-year-old male patient complains
E. Ankylosis of TMJ about gingival haemorrhage, teeth mobi-
lity in the frontal region of his lower jaw,
127. A 45-year-old teacher consulted a hypersensitivity of dental cervixes. X-ray
prosthodontist about choosing efficient picture shows widening of periodontal fi-
prosthetic tactics. Objectively: the 12, 11, ssure in the region of the 42, 41, 31, 32
21, 22 teeth present with III degree mobi- teeth, as well as resorption of alveolar
lity. What orthopaedic care is the most process tissue to 1/3 of root length. The
efficient? 42, 32 teeth present with I degree mobili-
ty, the 41, 31 teeth present with II degree
A. Immediate prosthetics of teeth mobility. What temporary splint
B. Early prosthetics should be used for frontal stabilization of
C. Fixed prosthesis dentition?
D. Delayed prosthetics
E. Clasp denture A. Novotny
B. Mamlock
128. A 62-year-old female patient C. Treuman
consulted a dentist about orthopaedic D. Kogan
treatment of the 12, 21, 22 teeth. The E. Kopeinin
arterial pressure was 165/110 mm Hg
before prosthetic procedure. Because of 132. For fabrication of an external
emotional stress arterial pressure rose up prosthesis a 62-year-old male pati-
to 220/130 mm Hg. What is the most likely ent needs "Hippocratic facies". What
diagnosis? impression material should be applied?
A. Hypertensive crisis A. Gypsum
B. Acute respiratory failure B. Dentafol
C. Acute heart failure C. Stens
D. Syncope D. Stomaflex
E. Collapse E. Repin
129. A 16-year-old patient complains 133. A 67-year-old patient was diagnosed
about experiencing gingival enlargement, with cemental caries of the 35 tooth. What
pain and haemorrhage throughout a year. filling material should be chosen?
Objectively: gingival papillae are enlarged
and overlap tooth crowns by 1/2 of their A. Glass-ionomer cement
height. Gums exhibit bright red granulati- B. Silver amalgam
ons and are painful on palpation. There C. Chemical-cure composite material
are considerable calculus deposits. X- D. Silidont
ray picture shows no changes of alveolar E. Silicin
process. What is the most likely diagnosis?
134. A 47-year-old patient complains
A. Hypertrophic gingivitis about limited mobility of her lower jaw
B. Localised periodontitis in the morning; periodical dull pain in
C. Ulcerous gingivitis the right temporomandibular joint (TMJ)
D. Generalized periodontitis and general joint stiffness. According
E. Catarrhal gingivitis to the patient, the stiffness disappears
throughout the day after joint "exerci-
130. Examination of a 25-year-old male sing". Objectively: the patient’s face is
Krok 2 Stomatology 2010 19

symmetric, mouth opening is limited ty, offensive breath, gingival hemorrhage.


down to 2,5 cm, there is also joint clicki- Objectively: the gum is hyperemic and
ng. Median line deviates to the right by cyanotic, dental calculus is present.
3-4 mm, palpation of the right articular Parodontal pouches are 8 mm deep wi-
head is painful. What is the most likely di- thin upper jaw molars. The pouches of
agnosis? other teeth are 6 mm deep. X-ray picture
shows resorption of bone tissue by 2/3-1/2
A. Arthrosis of the right TMJ of tooth roots. What is the most likely di-
B. Acute serous arthritis of the right TMJ agnosis?
C. Chronic arthritis of the right TMJ
D. Fracture of the right condyle of mandi- A. Chronic generalized III degree peri-
ble odontitis
E. Right-sided anterior dislocation of B. Chronic generalized II degree peri-
mandible odontitis
C. Chronic generalized I degree peri-
135. A 10,5-year-old child complains odontitis
about painful skin rash on his lips. D. Acute generalized III degree peri-
Objectively: red border is edematic and odontitis
hyperemic, covered with cracks and multi- E. Acute generalized II degree periodonti-
ple bloody crusts. There are small vesi- tis
cles with serous contents on the upper lip
skin that merge together in certain areas. 139. A 20-year-old patient complai-
Maceration and madescence of skin is also ns about a carious cavity in the 44
present, especially in the mouth corners. tooth. Objectively: the 44 tooth has a
What is the most likely diagnosis? deep carious cavity in the masticatory
surface, the dentine is dense and pi-
A. Eczematous cheilitis gmented. Probing, percussion cause no
B. Meteorological cheilitis pain. Electroodontodiagnosis is 12 mi-
C. Atopic cheilitis croampere. What is the most likely di-
D. Actinic cheilitis agnosis?
E. Exfoliative cheilitis
A. Chronic deep caries
136. An injured man was hospitalized to B. Chronic median caries
the oral surgery department with rupture C. Chronic fibrous periodontitis
of soft tissues of palate and pharynx. In D. Chronic fibrous pulpitis
process of respiration the dangling flap E. Acute deep caries
from the wound partially or completely
blocks the larynx aperture. What kind of 140. A 32-year-old female patient
asphyxia may develop in this patient? complains about a cosmetic defect of
the 14, 25 teeth. It is planned to make
A. Valvular metal-ceramic crowns. What impression
B. Aspirating material should be used for making these
C. Obturative crowns?
D. Dislocational
E. Stenotic A. Silicone
B. Hardening
137. A 27-year-old female patient is C. Wax
pregnant, duration of gestation is 21 D. Alginate
weeks. She has indication for extraction E. Zinc oxide eugenol
of the 28 tooth because of exacerbation
of chronic periodontitis after ineffecti- 141. A 25-year-old patient complains
ve therapeutic treatment. What kind of about a light brown spot in the upper
anaesthesia should be used for tooth foretooth. Objectively: the 23 tooth has a
extraction? single light brown spot in the cervical regi-
on. Probing shows smooth surface. The
A. Tuberal and palatinal tooth is nonresponsive to cold and probi-
B. Tuberal and incisor ng. What is the most likely diagnosis?
C. Infraorbital and palatinal
D. Torus
E. Mandibular
138. A 50-year-old patient complains
about difficult mastication, teeth mobili-
Krok 2 Stomatology 2010 20

A. Chronic initial caries diagnosis?


B. Fluorosis
C. Local enamel hypoplasia A. Exacerbation of generalized II degree
D. Acute initial caries periodontitis
E. Chronic superficial caries B. Exacerbation of generalized I degree
periodontitis
142. A 60-year-old patient underwents C. Exacerbation of generalized III degree
sanitation of the oral cavity before an periodontitis
operation on account of cataract. After D. Chronic generalized II degree peri-
examination the patient was diagnosed odontitis
with chronic median caries of the 22 tooth E. Chronic generalized III degree peri-
(Black’s class V). What filling material odontitis
should NOT be used in this patient?
146. A female patient applied to the
A. Light-cure microhybrid material oral surgery department and underwent
B. Chemical-cure microhybrid material radical maxillary sinusotomy with plastic
C. Chemical-cure glass-ionomer cement repair of fistula through the alveolar
D. Silicate cement socket of the extrated 27 tooth. Infiltration
E. Chemical-cure macrofilled composite and all the peripheral block anaesthesias
of the left upper jaw were performed wi-
143. A 33-year-old patient complai- th 6,0 ml of 2% lidocaine solution. 3 mi-
ns about an ulcer of oral cavity floor, nutes later the patient registered double
that is located under his tongue on a vision in her left eye, inability to close
level between the 43 to the 33 tooth. it. Which of the performed anaesthesi-
Examination reveales that ulcer edges as is the reason for the above-mentioned
are undermined and scalloped. Its grey- presentations?
yellow floor is shallow and it is covered
with small, easily bleeding granulations. A. Infraorbital
There is no ulcer infiltration. Make a cli- B. Tuberal
nical diagnosis: C. Palatinal
D. Incisor
A. Tubercular ulcer of oral cavity floor E. Infiltration
B. Decubital ulcer of oral cavity floor
C. Cancerous ulcer of oral cavity floor 147. On the 7th day after the operation on
D. Gummatous ulcer of oral cavity floor the abdominal cavity a 30-year-old pati-
E. Migratory granuloma of oral cavity floor ent presented with pain and sweling in the
region of parotid gland; body temperature
144. A steeplejack with a long record of rise up to 39o C; limited mouth opening,
service consults a dentist about dryness, dryness. Gland massaging results in di-
burning and insignificant lip edema. The scharge of purulent exudate from its duct.
same symptoms were noted one year ago The patient can be diagnosed with:
in autumn. What is the most likely di-
agnosis? A. Acute non-epidemic parotiditis
B. Acute epidemic parotiditis
A. Meteorogical cheilitis C. Phlegmon of submasseteric space
B. Contact cheilitis D. Parenchymatous parotiditis
C. Cheilitis glandularis E. Phlegmon of parotidomasseteric region
D. Manganotti’s cheilitis
E. Cheilitis exfoliativa 148. A 28-year-old patient complains
about constant localised pain that is getti-
145. A 42-year-old patient complains ng worse when biting down on food.
about gingival pain, progressing gingi- Objectively: the 46 tooth has a carious
val haemorrhage, increasing tooth mobi- cavity communicating with the dental
lity, halitosis. Objectively: gums are evi- cavity. Probing of the carious cavity causes
dently hyperaemic, extremely edematic, no pain, percussion is painful, X-ray pi-
they bleed easily on palpation. Tooth roots cture shows no changes. What is the most
are exposed, parodontal pouches are 4-6 likely diagnosis?
mm deep, and contain purulent exudate,
there is also supragingival and subgingival
dental calculus. II-III grade tooth mobi-
lity is present. Orthopantomogram shows
resorption of interdental septa down to
1/2 of their height. What is the most likely
Krok 2 Stomatology 2010 21

A. Acute serous periodontitis a 64-year-old patient complained about


B. Exacerbation of chronic pulpitis retrosternal pain irradiating to his back.
C. Acute purulent periodontitis There is paleness of membrane and skin
D. Acute diffuse pulpitis integument as well as pulse assymetry on
E. Exacerbation of chronic periodontitis the carotid arteries. What is the most li-
kely diagnosis?
149. A 23-year-old patient complains
about a carious cavity in the 16 tooth. A. Acute myocardial infarction
Objectively: the 16 tooth has a deep cari- B. Hypertensic crisis
ous cavity communicating with the dental C. Acute respiratory failure
cavity. Probing, percussion cause no pain. D. Angioneurotic Quincke’s edema
There is a fistula on the gingiva in the regi- E. Collapse
on of root apex projection of the 16 tooth.
What is the most likely diagnosis? 153. Renovation of a dental office involves
installation of luminous tube lamps. Speci-
A. Chronic granulating periodontitis fy the required illuminance level in lux:
B. Chronic gangrenous pulpitis
C. Chronic fibrous periodontitis A. 500 lx
D. Chronic fibrous pulpitis B. 400 lx
E. Chronic granulomatous periodontitis C. 300 lx
D. 200 lx
150. A 30-year-old male patient consulted E. 100 lx
a dentist about a swelling in the region
of his upper lip. Objectively: the face is 154. A 42-year-old builder complains
asymmetric because of upper lip edema, about a condyloma on his lower lip. It
nasolabial fold is smoothed. Mucogingival appeared 1,5 month ago. It has been si-
fold of the upper jaw vestibular surface gnificantly growing throughout the last
in the region of the 11, 12, 21 teeth is week. Objectively: the red border of the
smoothed, hyperemic. Palpation is pai- lower lip is cyanotic and infiltrated, it has
nful. Fluctuation is present. The patient some isolated closly adhering squamae.
was diagnosed with acute purulent peri- There is a well-defined hemispherical
ostitis of the upper jaw satrting from the formation 8 mm in diameter and 4 mm
21 tooth. Choose the treatment tactics: high in the centre. The formation is of
grey-and-blue-and-red colour, it has rough
A. Preservation of the 21 tooth, peri- surface formed by thin, closely adheri-
osteotomy, anti-inflammatory therapy ng and thick-based squamae. Regional
B. Preservation of the 21 tooth, anti- lymph nodes are enlarged, mobile, dense
inflammatory therapy and painless. What is the most likely di-
C. Extraction of the 21 tooth, peri- agnosis?
osteotomy, ant-inflammatory therapy
D. Extraction of the 21 tooth, peri- A. Lower lip cancer
osteotomy B. Verruciform precancer
E. Extraction of the 12, 21, 22 teeth, C. Pyogenic granuloma
periosteotomy, anti-inflammatory therapy D. Keratoacanthoma
E. Viral wart
151. A 33-year-old officer delivered
from a battlefield has shoot off mental 155. During the removal of dermoid cyst
part o fmandible, drooling, insignifi- of the nose bridge a 14-year-old boy
cant haemorrhage from the oral cavity, complained about giddiness, sickness,
asphyxia that occurs periodically because weakness. Objectively: the skin is pale,
of head repositioning. What actions covered with cold sweat. Tachypnoe is
should be taken to prevent the asphyxia present. The pulse is weak, the arterial
development? pressure is low (80/60 mm Hg), hands feel
cold. What is the most likely diagnosis?
A. Fixation of tongue, transportation in
prone position A. Syncope
B. Thacheostome construction B. Traumatic shock
C. Fixation of jaws with a mental sling C. Anaphylactic shock
D. Surgical d-bridement, tracheostome D. Collapse
construction E. Toxic shock
E. Transportation in prone position
156. A patient complains about pain in the
152. During extraction of the 14 tooth oral cavity, burning and dryness. Exami-
Krok 2 Stomatology 2010 22

nation revealed fiery-red dry mucous dull pain that is getting worse while pressi-
membrane. The tongue is crimson, dry, ng the tooth. The tooth was treated for
glossy, filiform papillae are atrophied. pulpitis one mounth ago. Objectively: the
There is some deposit in tongue folds 36 tooth is filled. Percussion causes acute
that is hard to be removed. The patient pain. Mucous membrane is hyperemic
undergoes treatment for pneumonia, she and edematic. What is the most likely di-
takes antibiotics. What is the most likely agnosis?
diagnosis?
A. Acute serous periodontitis
A. Acute atrophic candidiasis B. Exacerbation of chronic pulpitis
B. Fastened erythema C. Acute diffuse pulpitis
C. Pellagrous glossitis D. Acute suppurative pulpitis
D. B2 hypovitaminosis E. Exacerbation of chronic periodontitis
E. Benign migratory glossitis
161. A 24-year-old victim consulted a
157. A dentist applied to the regional sani- doctor about a face burn. He was injured
tation center for a permission to open a by the open flame. He complains about
private dental office with two universal pain and burning of face skin. Objecti-
dental sets. Planned area of the recepti- vely: hyperemia of face skin; in the regi-
on room is 26 2 . According to existing on of chin, nose, forehead, eyebrows,
norms, the dental office with two universal cheekbones there are burn blisters contai-
dental sets must have the following area: ning transparent liquid. These presentati-
ons correspond with the following degree
A. 14 2 for each set and 10 2 in addition of burn severity:
B. 10 2 for each set and 10 2 in addition
C. 10 2 for each set and 7 2 in addition A. I-II degree
D. 7 2 for each set and 7 2 in addition B. II-III A degree
E. 20 2 for each set and 12 2 in addition C. II-III B degree
D. III B-IV degree
158. A 40-year-old patient underwent E. II-IV degree
selective grinding of teeth on account
of TMJ disease provoked by functional 162. A 20-year-old patient complains
occlusal disorder. 3 days later the patient about inability to move with his lower
consulted the orthopaedist about injuri- jaw, speech difficulty and problems duri-
ng buccal mucosa because of its getting ng eating. He associates his condition wi-
in between dentitions on the right. What th a trauma he got when biting on apple.
mistake was made during teeth grinding? Examination revealed half-open mouth,
open bite due to the solitary contacts of
A. Tops of the support palatal tubercles of distal tubercles of the last molars, salivati-
the upper teeth were ground off on, indistinct speech. Articular heads of
B. Tops of the buccal tubercles of the lower mandible can be felt anterior to both tragi.
teeth were ground off What is the most likely diagnosis?
C. Lingual tubercles of the lower teeth
were ground off A. Acute anterior bilateral mandible
D. Buccal tubercles of the upper teeth luxation
were ground off B. Fibrous ankylosis of temporomandi-
E. Upper teeth tubercles were ground off bular joint
C. Exacerbation of osteoarthritis of
159. A 20-year-old patient was diagnosed temporomandibular joint
with chronic deep caries of the 12 tooth. D. Traumatic bilateral fracture of articular
Objectively: the 12 tooth has a deep cari- processes of mandible
ous cavity on the aproximal-distal surface. E. Fracture of articular processes of mandi-
Distal crown angle of the 12 tooth is also ble
affected with caries. Choose the material
for correction of the indicated defect: 163. A 6,5-year-old child has a gap 2,5-
3 mm large between frontal teeth from
A. Light-cure composite material canine to canine. Relationship of the first
B. Chemical-cure glass ionomer cement permanent molars complies with Angle’s
C. Light-cure glass-ionomer cement class I. Specify the severity degree of bite
D. Polycarboxylate cement deformation:
E. Silica-alumina cement
160. A 9-year-old child complains about
Krok 2 Stomatology 2010 23

A. I degree 2х2,5 cm large; its palpation is slightly pai-


B. II degree nful, the skin is unchanged. The teeth are
C. III degree intact. Pharynx is hyperaemic. What is the
D. IV degree most likely diagnosis?
E. V degree
A. Acute serous nonodontogenic
164. A 5-year-old child complains about submandibular lymphadenitis
spontaneous pain in an upper jaw tooth on B. Acute serous odontogenic submandi-
the right that is getting worse at night and bular lymphadenitis
during eating cold food. Objectively: the C. Acute purulent nonodontogenic
65 tooth has a deep cavity communicating submandibular lymphadenitis
with the tooth cavity. Probing is painful, D. Acute purulent odontogenic submandi-
percussion is painless. Cold water causes bular lymphadenitis
long-standing pain. What is your provisi- E. Submandibular adenophlegmon
onal diagnosis?
168. A 48-year-old woman complains
A. Exacerbation of chronic pulpitis about aching dull pain in the region of
B. Acute periodontitis the left TJM, that is getting worse duri-
C. Exacerbation of chronic periodontitis ng eating solid food. The pain appeared
D. Acute serous pulpitis about 2,5 years ago. Objectively: mouth
E. Acute purulent pulpitis opening is limited, there is sideward devi-
ation of jaw during mouth opening, TMJ
165. A 13-year-old boy complains about is clicking. Examination of the oral cavi-
pain in the upper jaw caused by warm ty revealed secondary partial adentia. X-
and hot stimuli, offensive breath when he ray picture shows sclerosis of the cortical
sucks his tooth. Objectively: the 24 tooth plate of articulat head and narrowing of
is changed in color, there is a deep cari- cartilage space. What is the most likely di-
ous cavity communicating with the tooth agnosis?
cavity. Deep probing is painful. Percussi-
on causes no pain. What is the most likely A. Arthrosis of the TMJ
diagnosis? B. Chronic arthritis of the TMJ
C. Acute arthritis of the TMJ
A. Chronic gangrenous pulpitis D. Painful dysfunction of the TMJ
B. Chronic periodontitis E. Exacerbation of chronic arthritis of the
C. Chronic fibrous pulpitis TMJ
D. Exacerbation of chronic pulpitis
E. Exacerbation of chronic periodontitis 169. A 30-year-old patient was di-
agnosed with posttraumatic osteomyeli-
166. A 30-year-old patient complains tis of mandible in the region of mandi-
about body temperature rise up to 39, 0oC, ble body on the right. The patient got
a roundish infiltrate on his upper lip, the trauma 1,5 month ago. X-ray picture
general weakness. He has been presenting shows the fracture line and the shadow of
with these symptoms for 3 days. Objecti- osseous sequestre along the fracture line.
vely: a roundish infiltrate in the regi- There is a fistula with purulent discharge
on of the upper lip 2,5 cm in diameter, on skin. Choose the treatment tactics:
the skin over the infiltrate is red with a
necrotic core in the centre. The upper lip A. Fistulectomy, sequestrectomy
is hyperemic and edematic. What is the B. Physiotherapy
most likely diagnosis? C. Sequestrectomy
D. Fistulectomy
A. Furuncle of the upper lip E. Fistulectomy, antibacterial therapy
B. Carbuncle of the upper lip
C. Retention cyst of the upper lip 170. A dentist fills a Black class II cari-
D. Acute periostitis of the upper jaw ous cavity in the 36 tooth of a 35-year-old
E. Acute glandular abscess man by sandwich method in one office vi-
sit. What glass-ionomer cement should be
167. Parents of a 6-year-old child complain chosen as basic liner in this case?
about pain in the child’s submandibular
region on the left, body temperature ri-
se up to 37, 5o C. Objectively: the child’s
face is asymmetric due to the infiltration
of the submandibular region on the left.
The infiltration is soft and elastic, mobile,
Krok 2 Stomatology 2010 24

A. Hybrid A. Bridge-like prosthesis with a pivot point


B. Water-hardening B. Lamellar prosthesis without a pivot
C. Classical point
D. Condensable C. Lamellar prosthesis with Gavrilow’s
E. Reinforced pivot point
D. Lamellar prosthesis with Oxman’s pivot
171. A 54-year-old male patient complai- point
ns about progressing pain in the area of E. Lamellar prosthesis with Weinstein’s
upper jaw on the left that first arose 3 days pivot point
ago. Body temperature is up to 37, 2oC.
Objectively: gingival mucous membrane 174. After supercooling a 42-year-old pati-
is edematic, hyperemic, it bleeds slightly. ent presented with headache in the left
Parodontal pouches are 6-7 mm deep frontal region and left upper jaw. Objecti-
and contain a small amount of purulent vely: the face is symmetric, breathi-
exudate. A painful infiltrate is palpated in ng through the left nasal meatus is
the region of the 26 tooth. The 26 tooth obstructed, seropurulent discharges are
has III degree mobility, the rest of teeth present. Palpation is slightly painful in
have I-II degree mobility. X-ray picture the infraorbital region as well as along
shows resorption of interalveolar septa the mucogingival fold in projection of the
by 2/3 of root length. What is the most 24, 25 teeth. Percussion of these teeth is
appropriate treatment method of the 26 painless. The 24 tooth is filled. Mucuous
tooth? membrane of alveolar process has no visi-
ble changes. X-ray picture shows reduced
A. Extraction of the 26 tooth pneumatization of the left upper jaw si-
B. Temporal splintage, conservative nus. What is the provisional diagnosis?
therapy
C. Curettage of parodontal pouche A. Exacerbation of chronic odontogenic
D. Lancing of abscess via gum maxillary sinusitis
E. Lancing of abscess via parodontal pouch B. Acute periodontitis of the 24 tooth
C. Exacerbation of chronic periodontitis of
172. A 27-year-old victim was delivered to the 24 tooth
to the facial surgery department with a D. Acute rhinogenous maxillitis
cut wound of right cheek. The trauma is E. Acute albuminous periostitis of the left
civilian, the patient got it in a fight. The upper jaw
deadline for early surgical d-bridement
without preliminary antibiotic injection is 175. An 8,5-year-old child is apparently
no more than: healthy. The child complains about pain
in an upper tooth on the left caused by
A. 24 hours traumatic injury sustained three hours
B. 48 hours ago. Objectively: the crown part of the 21
C. 72 hours tooth is destroyed by 1/2, the pulp is red
D. 12 hours and significantly exposed, probing causes
E. 1 hour acute pain and bleeding. Percussion of the
21 tooth is extremely painful. Choose the
173. A 62-year-old patient has a median most efficient treatment method of the 21
fracture of mandible along with formati- tooth:
on of a false joint. Objectively: dental
formula is 33, 34, 35, 36, 27, 47, 46, 45, 44, A. Vital amputation
43. The teeth are intact, stable, with high B. Vital extirpation
crowns. Fragment mobility is insignificant, C. Devital amputation
there is no displacement. X-ray picture D. Devital extirpation
shows a bone defect 0,8 cm large. What E. Bioassay technique
prosthesis is indicated?
176. A 36-year-old female patient
complains about an aesthetic defect
of upper frontal teeth. In past hi-
story: she undergoes regular check-up
in the endocrinological department on
account of thyrotoxicosis. Enamel defects
appeared several years ago and since that
have been gradually expanding. Exami-
nation revealed that the 12, 11, 21, 22
Krok 2 Stomatology 2010 25

teeth had transverse oval enamel defects A. Acute hematogenous osteomyelitis


on the vestibular surfaces. Probing and B. Phlegmon of infraorbital region
cold test is painless. Floor of the defects is C. Acute maxillary sinusitis
smooth, glossy and hard. What is the most D. Acute odontogenic osteomyelitis
likely diagnosis? E. Acute purulent periostitis
A. Erosion of hard tissies 180. According to the parents of a 7-
B. Acute superficial caries year-old child, the child complains about
C. Wedge-shaped defects weakness, body temperature rise up to
D. Systemic hypoplasia 39oC, toothache in the upper jaw on
E. Necrosis of hard tissues the left. Objectively: condition is grave,
the child is pale and adynamic, the face
177. A 23-year-old patient complai- is asymmetric because of infiltration of
ns about root exposition, gingival the upper jaw on the left. The 64 tooth
hemorrhage during tooth brushing, gum has a carious cavity. Percussion is pai-
itch. Objectively: there is supragingival nful, I degree of teeth mobility is also
and subgingival dental calculus. Gums are present. There are purulent discharges
hyperaemic, edematic, pouches are 3,5 from the subgingival edge of the 64
mm deep. X-ray picture shows resorpti- tooth. Mucogingival fold of the 63, 64, 65
on of interalveolar septa by 1/3. What is teeth is smoothed. Vestibular and palatine
the most likely diagnosis? mucous membrane is edematic. What is
the most likely diagnosis?
A. Chronic generalized I degree peri-
odontitis A. Acute odontogenic maxillary
B. Chronic generalized II degree peri- osteomyelitis starting from the 64 tooth
odontitis B. Acute albuminous maxillary periostitis
C. Exacerbation of generalized I degree C. Acute purulent odontogenic maxillary
periodontitis periostitis starting from the 64 tooth
D. Exacerbation of generalized II degree D. Suppurative radicular cyst of maxilla
periodontitis E. Acute odontogenic maxillary sinusitis
E. II degree periodontitis
181. A 35-year-old man has a medi-
178. A 14-year-old child complains about um deep carious cavity in the 37 tooth
acute spontaneous pain in an upper jaw (Black’s class II). For its filling a denti-
tooth on the right. The pain has been st chose technique of layer-by-layer
lasting for 3 days, it is throbbing, irradi- restoration. What composite should be
ating to the temple, getting worse at ni- coating the floor and walls of the carious
ght. Objectively: surface of the 15 tooth cavity in order to form superadaptive ini-
exhibits a carious cavity within parapulpar tial layer?
dentine. Dentine is softened, of greyish
colour. Probing of the whole cavity floor is A. Flowable
painful, percussion of the 15 tooth is pai- B. Condensable
nless. What is the most likely diagnosis? C. Macrofilled
D. Microhybrid
A. Acute purulent pulpitis E. Microfilled
B. Acute diffuse pulpitis
C. Acute focal pulpitis 182. A 32-year-old male patient was deli-
D. Acute periodontitis vered to traumatology. He complains
E. Exacerbation of chronic periodontitis about nosebleed, severe cephalalgia, gi-
ddiness, face deformation, heavy breathi-
179. A child’s mother had acute purulent ng, mastication inability. He sustained
mastitis. Now the 2-month-old child an injury in the mine over three hours
experiences an edema of his left infraorbi- ago. He was conscious. Objectively: the
tal and malar regions, dermahemia of middle third of his face is elongated,
the left face side, temperature rise up to there are considerable hematomas and
39 − 40o C, purulent discharges from the there is an edema in the region of nose
nose. What is the most likely diagnosis? root and eyelids of both eyes. The upper
jaw is displaced downwards, it is mobile
together with nasal bones, cheek bones
and eyeballs. Palpation reveals deformati-
on, pain and pathological mobility in the
region of the nose bridge, external socket
Krok 2 Stomatology 2010 26

edges, pterygoid processes of basilar bone. A. Fusobacteria, spirochetes


Nosebleed is present. What is the most li- B. Streptococci
kely diagnosis? C. Herpes viruses
D. Staphylococci
A. Le Fort III maxillary fracture (upper) E. Yeast fungi
B. Le Fort I maxillary fracture (lower)
C. Le Fort II maxillary fracture 186. A 65-year-old male patient complai-
D. Fracture of zygomatic bone ns about crepitation and clicking in both
E. Fracture of nasal bone temporomandibular joints, pain induced
by displacement of the lower jaw to the
183. A 9-year-old boy presents with right, ear noise, dry mouth, glossalgia.
face asymmetry due to the chin devi- He has been using complete removable
ation to the left. When the third Il’ina- prosthesis of the lower jaw for 6 months.
Marcosian diagnostic test is performed The patient denies rheumatosis. Objecti-
face asymmetry disappears. What is the vely: the lower third of face is shortened,
most likely clinical form of this occlusal mental fold is strongly pronounced,
abnormality? mouth corners are lowered, angular fi-
ssures and cracks are also present. Palpati-
A. Habitual deviation of mandible on reveals crepitatnt rale observed during
B. Ankylosis of the temporomandibular TMJ moving. What is the most likely di-
joint agnosis?
C. Unilateral hypoplasia of mandible
D. Bilateral narrowing of the maxillary A. Costen’s syndrome
dental arch B. Temporomandibular arthritis
E. Unilateral narrowing of the maxillary C. Temporomandibular arthrosis
dental arch D. Temporomandibular dislocation
E. Temporomandibular osteoarthritis
184. A 13-year-old girl has been experi-
encing gingival hemorrhages and frontal 187. A 40-year-old patient complains
teeth mobility throughout the last month. about a long-existing fissure in the central
Objectively: gingival mucous membrane part of red border of his lower lip. He has
in the region of lower incisors and canines been smoking since he was 16. Objecti-
is edematic, hyperemic, it bleeds on touch. vely: there is a deep 1,0 cm long fissure
These teeth present with I degree mobili- along the median line of red border of
ty, parodontal pouches are 3 mm deep. the lower lip, the fissure edges are bulgi-
Orthopantomogram shows reduction of ng. Characteristic quality of the fissure
bone tissue of interalveolar septa by 1/3 is its spontaneous healing, but after epi-
of their height. Frontal teeth of the lower thelization the fissure recurs. The pati-
jaw are overcrowded. Hygiene index is ent was diagnosed with chronic fissure
4,2. What local treatment should be provi- of the lower lip. Conservative treatment
ded in the first place? proved to be ineffective. Choose the most
appropriate treatment method in this si-
A. Professional hygiene tuation:
B. Anti-inflammatory therapy
C. Orthodontic care A. Excision within evidently healthy ti-
D. Physical therapy ssues
E. Antibacterial therapy B. Cryodestruction
C. Diathermo-coagulation
185. A 16-year-old teenager complains D. Excision 0,5 cm off the neoplasm limits
about halitosis, general weakness, body E. Close-focus roentgenotherapy
temperature rise up to 37, 6o C. These
symptoms turned up two days ago, the boy 188. A 52-year-old patient complai-
had a history of recent angina. Objecti- ns about teeth mobility, gingival
vely: oral hygiene is unsatisfactory, the haemorrhage. The 44, 45, 34, 35 teeth
teeth are coated with soft dental deposit. present with I degree mobility, the 43,
Gums are hyperemic, gingival papillae are 42, 41, 31, 32, 33 teeth present with II
covered with greyish deposit. Specify the degree of teeth mobility. There is gingival
causative agent of this disease: edema, parodontal pouches are 5-6 mm
deep. X-ray picture shows destruction of
bone tissue by 1/2 of root length. A dentist
made a diagnosis: generalized periodonti-
tis of median severity. What orthopaedic
Krok 2 Stomatology 2010 27

construction is the most efficient in this 11, 21, 22 crowns has roundish defects wi-
situation? th smooth, shiny, dense surface, that reach
dentine in depth. The depth of defects is
A. Elbrecht splint gradually decreasing from the centre to
B. Plastic mouthguard the periphery. What is the most likely di-
C. Murray’s splint agnosis?
D. Mamlock splint
E. Temporary Novotny splint A. Enamel erosion
B. Wedge-shaped defect
189. A victim got a face and temporal regi- C. Systemic hypoplasia
on trauma. A doctor made a diagnosis: D. Chronic median caries
fracture of malar arch. What processes of E. Destructive fluorosis
cranial bones are injured?
193. During the planned oral cavity sani-
A. Temporal process of zygomatic bone tation an 11,5-year-old girl complained
and zygomatic process of temporal bone about periodic spontaneous chilalgia
B. Zygomatic process of frontal bone and during food intake. Objectively: mucous
zygomatic process of temporal bone membrane of lips is hyperemic, edematic
C. Temporal process of zygomatic bone and dry. During talking one can observe
and zygomatic process of frontal bone small drops of mucous secretion. Make
D. Zygomatic process of maxilla and the provisionaldiagnosis:
zygomatic process of temporal bone
E. Zygomatic process of maxilla and A. Cheilitis glandularis
zygomatic process of frontal bone B. Angular cheilitis
C. Eczematous cheilitis
190. A 28-year-old patient had to D. Contact allergic cheilitis
undergo extraction of the 46 tooth E. Cheilitis exfoliativa
under Weisbrem’s apodactylic mandibular
anaesthesia. What is the target site of the 194. A 38-year-old patient consulted a
anaesthetic solution injection? dentist about aching pain in the region of
the 48 tooth that is getting worse when bi-
A. A segment of mandibular torus ting down on food. Body temperature rise
B. A segment of maxillary tuber up to 37, 6oC and aggravation of general
C. Retromolar fossa condition is present. Objectively: insignifi-
D. A segment of temporal crest of mandi- cant swelling of retromolar region as well
ble as hyperaemia of mucous membrane. The
E. Retromandibular fossa 48 tooth is covered with occlusal pad. It is
painful on palpation. Purulent discharges
191. A 7-year-old child sustained a sport are present. What is the most likely di-
injury. He complains about pain in the agnosis?
region of mandible, inability to join his
teeth. Pressing against the chin causes A. Acute purulent pericoronitis
intensification of pain in the region of B. Acute pulpitis
mandible on the right. The child is di- C. Acute periodontitis of the 48 tooth
agnosed with a fracture of mandible body D. Aphthous stomatitis
without fragment displacement. What E. Acute purulent periostitis
therapeutic tactics should be chosen in
order to prevent the displacement of bone 195. A patient ordered metal-ceramic
fragments in the transitional dentition? crowns for the 11, 12 teeth. Block
anaesthesia was performed before
A. Fragment immobilization by means of preparation. In a few minutes the pati-
an intraoral splint ent felt anxiety, tinnitus, abdominal pain,
B. Administration of anti-inflammatory giddiness. Breathing is hurried, pulse is
therapy rapid, AP is 60/40 mm Hg. What aid
C. Application of sling bandage should be rendered to the patient in the
D. Fixation by means of Tigerstedt splints first place?
E. Osteosynthesis
192. A 44-year-old patient complains
about the astringent sensation in the
upper jaw incisors, which appeared 1,5
years ago. Objectively: the most convex
part of the vestibular surfaces of the 12,
Krok 2 Stomatology 2010 28

A. Intravenous introduction of 0,5 ml of weakness, body temperature rise up to


0,1% adrenaline solution, prednisolone 39oC, headache, joint pain. What method
B. Promedol, suprastin of diagnostics should be applied to confi-
C. To soak a cotton web with some liquid rm the diagnosis?
ammonia and give it for the patient to
inhale A. Clinical blood analysis
D. To bring the patient into horizontal B. Blood sugar test
position C. Immunogram
E. Make an injection of aminophylline D. HIV test
E. Allergy test
196. A 40-year-old female patient
complains about crepitation and pain in 199. A 14-year-old girl complains about an
the region of TMJ projection on the ri- aesthetic defect in form of white spots on
ght, that arise in the final stage of mouth her teeth. Fluorine concentration in the
opening. Articular sounds turned up after drinking water in the area of her residence
dental prosthetics. Mouth opening is unli- is 2 mg/l. Objectively: vestibular surfaces
mited, mouth is opened in a straight path, of all teeth are covered with ill-defined
amplitude of mouth opening is 5 cm. What white spots. What is the most likely di-
is the most likely diagnosis? agnosis?
A. Lower jaw subluxation A. Fluorosis
B. Lower jaw dislocation B. Local hypoplasia
C. Anterior dislocation of articular disk C. Systemic hypoplasia
D. Posterior dislocation of articular disk D. Multiple caries
E. Meniscus tresis E. Amelogenesis imperfecta
197. A 73-year-old patient consults a 200. A 42-year-old patient consulted
dental orthopaedist about total edentia. a dentist about intense lancinating
Objectively: there is insignificant regular paroxysmal pain accompanied by a
atrophy of the alveolar process of sensation of current passage in the regi-
mandible, the process is covered with on of her upper lip on the right. Pain
mucous membrane of moderate elasticity. attacks occur spontaneously and last 3-
According to Keller’s classification, such 5 minutes. The patient usually has 2-3
manifestations are characteristic for the attacks a day. The patient is unable to
folowing type of edentulous jaw: establish the cause of this disease. Exami-
nation of her oral cavity revealed no
A. First pathological changes. Oral cavity is sani-
B. Third tated. Test orthopantomogram shows an
C. Fifth impacted supernumerary 13 tooth. What
D. Second is the most likely diagnosis?
E. Fourth
A. Peripheral neuralgia of the II branch of
198. An 18-year-old patient complai- trigeminus
ns about ulceration in the oral cavi- B. Pterygopalatine ganglionitis
ty, spontaneous bleeding of mucous C. Neuritis of the II branch of trigeminus
membrane, pain during food intake and D. Central neuralgia of the II branch of
talking, nosebleeds. He has a history trigeminus
of: aggravation of general condition, E. Right-sided upper jaw pulpitis
Kok 3 Stomatology 2011 1

1. A 38-year-old patient complains of present. Tonsils are swollen, hyperemic,


tension and enlargement of her lower lip covered with yellow-gray friable film whi-
and eyelids occuring after taking aspirin ch can be easily removed. Submandi-
for headache. The edema resolves soon bular and cervical lymph nodes are si-
after taking antihistaminic drugs. What is gnificantly enlarged, painful on palpation.
the most likely diagnosis? Laboratory analysis revealed leuko- and
monocytosis. What is the most likely di-
A. Quincke’s edema agnosis?
B. Melkersson-Rosenthal syndrome
C. Erysipelas of lips A. Infectious mononucleosis
D. Lymphadenic macrocheilitis B. Diphtheria
E. Meige trophedema C. Scarlet fever
D. Rubella
2. Parents of a 5-month-old baby complain E. Measles
of food refusal, ulcers on the palate. The
infant was born prematurely, is now arti- 6. A 50-year-old patient complains about
ficially fed. Objectively: at the junction of missing of the 37, 36, 45, 46, 47 teeth.
hard and soft palate there is an oval well- The 35, 38, 44, 48 teeth are intact and
defined ulcer, covered with yellow-gray stable, their crowns exhibit anatomic
film and limited by a hyperemic swelli- form, sufficient height, relative paralleli-
ng, protruding above the surface of oral sm. Fulcrum line typically goes through
mucosa. Which group of drugs should be the 35 and 44 abutment teeth. What fixing
administered for the aphtha epithelizati- element should be used in a clasp denture
on? supported by the 35, 44 teeth?
A. Keratoplastic agents A. Acker clasp
B. Antiviral drugs B. Roach clasp
C. Antimycotic drugs C. Jackson clasp
D. Antiseptics D. Telescopic fixation
E. Antibiotics E. Bent wire clasp
3. A 35-year-old male consulted a denti- 7. A 60-year-old patient complains of
st about the 26 tooth extraction. After pain in the masticatory muscles and
the patient had been given tuberal temporomandibular joint. The dentures
anaesthesia, he presented with progressi- were fabricated a month ago. Objecti-
ng tissue edema and limited mouth openi- vely: face configuration is abnormal, the
ng. This condition was caused by: lower third of the face is elongated, the li-
ps can only be closed with difficulty, the
A. Vessel trauma speech is indistinct. When smiling, the
B. Muscle trauma during anaesthetization base of the complete removable denture is
C. Nerve trunk trauma exposed. What mistakes were made duri-
D. Hypersensitivity to the anaesthetic ng the fabrication of complete removable
E. Anaphylactic shock dentures for the upper jaw?
4. After removal of dental deposit an 18- A. The vertical occlusal dimension was
year-old patient underwent preventive overrated
examination. It revealed painless chalky B. The vertical occlusal dimension was
spots in the precervical area on the vesti- underrated
bular surface of the 22 and 41 teeth. C. Misidentified posterior occlusion
Result of enamel resistance test is 7. What D. Misidentified anterior occlusion
morphological changes are typical for this E. Misplacement of maxillary teeth
disease?
8. A 58-year-old patient needs dental
A. Subsurface enamel demineralization prosthetics. Objectively: the 22 tooth is
B. Changes in the mantle dentine stable, intact. The atrophy of the alveolar
C. Damage of dentinoenamel junction crest is present, the palate is flat. It is
D. Superficial enamel demineralization planned to fabricate a removable denture.
E. Degeneratic changes of odontoblasts What tactics should be chosen in respect
of the 22 tooth?
5. A 10-year-old child complains of sore
throat, cough, fever (up to 38o C). These
presentations turned up 2 days ago.
Objectively: acute catarrhal stomatitis is
Kok 3 Stomatology 2011 2

A. It is required to fabricate a telescopic wall was found to be absent. X-ray pi-


crown cture shows a homogeneous shadowing
B. There is no need of any intervention of maxillary sinus; bone structures have
C. It should be depulped unclear contours. What is the most likely
D. It should be extracted diagnosis?
E. It is required to fabricate a stump crown
A. Upper jaw cancer
9. A 39-year-old patient needs a clasp B. Upper jaw adamantinoma
denture with porcelain teeth for the lower C. Upper jaw sarcoma
jaw. What method should be applied whi- D. Chronic odontogenous maxillary sinusi-
le substituting wax with plastic in order tis
to avoid mold flash that may cause bite E. Upper jaw osteoclastoma
opening?
13. An 8-year-old boy was diagnosed with
A. Method of transfer molding chronic fibrous pulpitis of the 21 tooth.
B. Direct method of compression molding It was treated by extirpation method.
C. Inverse method of compression molding Choose the material for root filling:
D. Combined method of compression
molding A. Calcium-containing material
E. In water under pressure B. Glass-ionomer cement
C. Zinc oxide eugenol paste
10. A 22-year-old patient was undergoing D. Resorcin-formalin paste
treatment of the 25th tooth for deep acute E. Phosphate cement
caries. During preparation a dentist acci-
dentally opened the horn of pulp. What 14. A 62-year-old patient has a medi-
method of treatment should be applied? an mandibular fracture complicated by
pseudoarthrosis. Objectively: the dental
A. Biologic method formula is 33, 34, 35, 36, 37, 47, 46, 45,
B. Vital amputation of pulp 44, 43. The teeth are intact, stable, with
C. Devital amputation high crowns. There is minor mobility of
D. Vital extirpation fragments without displacement. What is
E. Devital extirpation the most appropriate type of denture for
this patient?
11. A 39-year-old patient complains of
some soft ulcers and tubercles on the oral A. Dental bridge without a pivot point
mucosa, gingival haemorrhage, pain and B. Lamellar denture without a pivot point
loosening of teeth. Objectively: mucous C. Lamellar denture with Gavrilow’s pivot
membrane of tongue and gums presents point
single ulcers with soft, swollen, slightly D. Lamellar denture with Oksman’s pivot
painful edges, covered with a yellow film. point
Regional lymph nodes are enlarged, soft, E. Lamellar denture with Weinstein’s pivot
painless, not adherent to the surroundi- point
ng tissues. What is your provisional di-
agnosis? 15. During extraction of the 47 tooth its
distal root was broken halfway along its
A. Lupus tuberculosis length. What tool should be chosen for
B. Lepra extraction of the residual root fragments?
C. Tertiary syphilis
D. Scrofuloderma A. Left angled elevator
E. Sutton’s aphthae B. Broad-beaked forceps
C. Close-beaked forceps
12. A patient complains about mobili- D. Right angled elevator
ty of the 26, 27, 28 teeth, periodic pain, E. Straight elevator
sensation of heaviness in the correspondi-
ng half of the upper jaw, haemorrhagic di- 16. A patient consulted a doctor about
scharges from the left side of nose. These recurrent pain in the frontal part of his
presentations have been observed for 4 upper jaw. He has history of previous
months, within the last month the pain treatment for pulpitis. Objectively: the
has become worse, the patient presents crown of the 12 tooth is restored with
now with general weakness. Examinati- a filling. X-ray picture of the upper jaw
on of the oral cavity reveals the grade shows an area of bone tissue destruction
II mobility of the 26, 27, 28 teeth. On (up to 1 cm in diameter) at the root apex
palpation of the maxillary tuber, the bone of the 12 tooth. The root channel of the
Kok 3 Stomatology 2011 3

12 tooth is filled up to the top. The pati-


ent was diagnosed with cystogranuloma of A. Methylene blue + pimafucin
the 12 tooth. Choose the best method of B. Hydrocortizone ointment + solcoseryl
treatment: C. Decamin + hydrocortizone ointment
D. Furacilin + methylene blue
A. Root apex resection E. Furacilin + solcoseryl
B. Extraction of the 12 tooth
C. Conservative treatment 20. A 67-year-old patient needs a complete
D. Dissection along the mucogingival removable denture for his lower jaw. The
junction adjustment of custom tray is done by
E. Replantation of the 12 tooth method of Herbst. It is required to speci-
fy the boundary of tray for the sublingual
17. Preventive examination of a 20-year- region, 1 cm away from the midline. What
old student revealed chronic candidi- Herbst test should be applied for this
asis of the oral mucosa, generalized purpose?
lymphadenopathy. He has a 1-year history
of herpes simplex. The body temperature A. Alternate touching the cheeks with the
often rises to 37, 4 − 37, 5oC, the patient tongue
has lost 8 kg of body weight over the last B. Touching the red border of the upper lip
month. These symptoms may be indicati- with the toungue
ve of the following disease: C. Active movements of mimic muscled
D. Swallowing
A. AIDS E. Slow mouth opening
B. Infectious mononucleosis
C. Acute leukosis 21. A 40-year-old patient has an open
D. Chronic leukosis occlusion resulting from the bilateral
E. Lymphogranulomatosis fracture of his upper jaw and following
malunion of fragments. The occlusal verti-
18. A 38-year-old male patient complai- cal dimension is maintained only by molar
ns of a carious cavity. He had experi- teeth. There is a gap of about 1-1,5 mm
enced spontaneous dull pain in the tooth between the rest of teeth. The dentitions
in question before. Objectively: the di- are without gaps. What kind of treatment
stal masticatory surface of the 37 tooth would be most appropriate in this situati-
presents a deep cavity made of soft pi- on?
gmented dentin. Percussion is painless.
After removing the decay from the cavi- A. The molar teeth should be ground off
ty, cold water has caused pain lasting for and covered with crowns
about a minute. X-ray picture shows the B. Orthodontic treatment
deformation of the periodontal gap in the C. Combined apparatus and surgical
region of the 37 root apices. What is the treatment
most likely diagnosis? D. Open reposition
E. Prosthetics with combined crowns
A. Chronic fibrous pulpitis
B. Exacerbation of chronic pulpitis 22. A 14-year-old child complains of
C. Acute deep caries bleeding and pain during eating in the
D. Chronic deep caries lower right molar. Objectively: the medial
E. Chronic fibrous periodontitis contact and masticatory surfaces of the
46 tooth exhibit a large carious cavity
19. A 32-year-old patient complains of filled with red tissue. Superficial probi-
experiencing dryness and burning of ng causes pain and moderate bleedi-
tongue back for a week. The burning ng. Deeper probing causes acute pain.
intensifies during taking irritating foods. Percussion is painlessl. What is your provi-
Some time ago he had pneumonia and sional diagnosis:
was treated with antibiotics. Objectively:
skin and oral mucosa are pale. The lingual A. Chronic hypertrophic pulpitis
mucosa is hyperemic and edematous, the B. Chronic fibrous pulpitis
folds at the back of tongue are covered C. Chronic gangrenous pulpitis
with the crumbling gray-white plaque, D. Chronic papillitis
desquamation of the lateral surface of E. Chronic granulating periodontitis
tongue is also present. Saliva is thick
and attaches to a spatula. Choose the 23. A 10-year-old girl complains of mi-
most effective drug complex for the local nor pain in a lower jaw tooth occuring
treatment: during having cold food. Objectively: the
Kok 3 Stomatology 2011 4

masticatory surface of the 36 tooth exhi- is 30 mm/h. What is the leading factor of
bits a carious cavity with a narrow inlet disease development?
located within the mantle dentin. Probi-
ng causes pain along the enamel-dentin A. Viral infection
border. Make a diagnosis: B. Bacterial infection
C. Autoimmune disorders
A. Acute median caries D. Immediate allergy
B. Acute deep caries E. Delayed allergy
C. Chronic deep caries
D. Superficial caries 27. A 72-year-old patient complains of
E. Chronic median caries burning pain in the corners of mouth.
Objectively: the folds of mouth corners
24. An 80-year-old patient presented have erosions, covered with white coati-
to a clinic for the adjustment of the ng that can be easily removed, mucous
impression tray for the upper jaw. Objecti- membrane of mouth corners is macerated,
vely: alveolar process exhibits marked of pearly colour. There is pathological
homogenous atrophy, maxillary tubers are tooth wear and decreased vertical di-
absent, torus is weakly expressed. Buccal mension of occlusion. What is your provi-
muscles and frenulum of the upper lip are sional diagnosis?
attached almost at the top of the alveolar
process, the palatal slope is not steep. A. Angular cheilitis candidiasis
How to adjust the impression tray in order B. Atopic cheilitis
to make a close-fitting valve? C. Chronic recurrent labial fissure
D. Streptococcal angular cheilitis
A. To extend the borders along the A line E. Syphilitic angular cheilitis
B. To extend the external borders
C. To extend the borders in the frontal 28. A 45-year-old patient presented
region to aclinic for prosthetics. Objectively:
D. To extend the borders in the region of Kennedy III type, I subtype dentition
maxillary tubers defect is present, the 46, 45, 44, 34, 35,
E. To overlay buccal muscles and frenulum teeth are missing. The bite is fixed. Che
with the impression tray Crowns of the 47, 43, 33, 37 teeth are high,
of a well-defined anatomical shape, intact.
25. A 32-year-old male complains of It is planned to fabricate a clasp denture.
constant mandibular movements simulati- What system of the clasp denture fixation
ng chewing that take place when he is is the most appropriate in this case?
excited. Besides that, the patient complai-
ns of teeth grinding. Objectively: the A. Rumpel’s bar system
lateral surfaces of teeth have signs of B. Attachments
abrasion, the tubercles are absent. What C. Roach clasp
is the most rational method of treatment? D. Continuous clasp
E. Telescopic system
A. Night guard, myogymnastics, self-
massage 29. After a fall a 28-year-old patient visi-
B. Restoration with opposite-directed ted the clinic of maxillofacial surgery and
dentures complained of restricted mouth opening,
C. Medicamental and machine treatment skin numbness in the left infraorbital regi-
with further consultation with a dental on, upper lip, wing of the nose, pain in the
surgeon left zygomatic region that gets worse when
D. Consultation with a dental surgeon he opens his mouth. Palpation revealed
E. Teeth alignment (selective tubercle step deformity. What is the most likely di-
removal) agnosis?

26. An 18-year-old patient complains A. Fracture of the left zygomatic bone with
about body temperature rise, weakness, a displacement
pain induced by eating and deglutition. B. Fracture of the left zygomatic bone
Objectively: mucous membrane of the without a displacement
oral cavity is erythematic with multi- C. Fracture of the upper jaw of Le Fort II
ple petechia. Pharynx is hyperaemic. (medium type)
Regional lymph nodes are enlarged, D. Fracture of the upper jaw of Le Fort I
mobile, painless. In blood: leukocytosis, (top type)
monocytosis, atypic mononuclears, ESR E. Fracture of the left zygomatic bone
Kok 3 Stomatology 2011 5

30. A 75-year-old patient has been admi- 34. A 20-year-old patient complains of
tted to the dental department for the general weakness, fever, headache. These
right-angle mandibular fracture with a presentations appeared three days ago.
displacement. Objectively: the face is Objectively: the regional lymph nodes
asymmetric due to the post-traumatic are enlarged, painful on palpationl, body
swelling, palpation reveals the mobility of temperature is 37, 8oC, oral mucosa
fragments and pain. The patient has a li- is hyperemic and edematous. Mucous
felong history of epilepsy. What is the best mebrane of lips, palate, gums, cheeks
method of treatment? presents single erosions of polycyclic
irregular shape, covered with grayish-
A. Osteosynthesis white film, painful. Which drugs should
B. Immobilization with Vasilyev splints be administered for the local treatment of
C. Immobilization with Tigerstedt splints the early disease?
D. Immobilization with Port splint
E. Immobilization with Vankevich splint A. Antiviral
B. Keratoplastic agents
31. Analysis of a 10-year-old boy’s jaw C. Corticosteroid ointments
models revealed that occlusal plane of D. Antiseptics
the frontal maxillary teeth was of concave E. Dyes
form, its lateral parts were convex. Form
of the alveolar process also represents 35. A patient has been diagnosed wi-
deformation of dental arches. The upper th traumatic open angular fracture of
jaw is of saddle-like form with abrupt mandible with a fragments displacement.
narrowing in the region of premolar teeth. Which of the following should be appli-
What type of bite is it? ed for transport immobilization of the
fragments?
A. Open
B. Distal A. Entin head-chin strap
C. Deep B. Tigerstedt splints
D. Mesial C. Standard Vasyliev splints
E. Cross D. Gunning splints
E. Weber splints
32. A 59-year-old patient has a bilateral
fracture of mandible in the region of the 36. A 50-year-old patient needs dental
44, 34 teeth, other masticatory teeth are prosthetics. Objectively: the 14, 15, 16,
missing, toothless fragments are not di- 17, 24, 25, 26, 27 teeth are missing. The
splaced but mobile. Which orthosis should state of the abutment teeth is satisfactory.
be used for the immobilization of bone What type of fixation will provide the
fragments? best conditions for the stabilization of a
removable denture?
A. Vankevich splint with orthodontic
elastics A. Planar
B. Rudko’s apparatus B. Sagittal
C. Limberg’s splint C. Transversal
D. Petrosov’s apparatus D. Linear
E. Zbarzh apparatus E. Point
33. A 23-year-old patient complains of gi- 37. A 6-year-old boy hit his forehead one
ngival haemorrhages, fatigue, indisposi- day ago. A few hours later a swelling
tion. The symptoms have appeared qui- appeared in the right superciliary regi-
te recently. Objectively: the skin and on. Objectively: there is a considerable
oral mucosa are pale. Gums bleed when edema of forehead tissues spreading to
touched. There are petechiae on the eyelids of the right eye, the skin over the
mucous membrane of cheeks, lips and swelling is cyanotic, the swelling is of soft
palate. What laboratory test will allow to consistency. Fluctuation is also present.
make a diagnosis? General condition of the boy is normal.
Make a provisional diagnosis:
A. Complete blood count
B. Blood test for sugar
C. Blood test for Vitamin C
D. Bleeding time test
E. Immunogram
Kok 3 Stomatology 2011 6

A. Hematoma of the right superciliary story of microstomy. Objectively: the 31,


region 32, 35, 36, 37, 38, 41, 42, 45, 46, 47 and 48
B. Postraumatic edema of tissues of the teeth are missing. The remaining teeth are
right superciliary region stable, immobile. What kind of denture
C. Fracture of frontal bone construction is recommended for this pati-
D. Hematic abscess of the right supercili- ent?
ary region
E. Inflammatory infiltration of tissues of A. Collapsible removable partial lamellar
the right superciliary region denture
B. Removable partial lamellar denture
38. A 14-year-old child complains about a C. Dental bridge supported by the 43, 33
cosmetic defect in the frontal teeth regi- teeth and removable denture
on. Objectively: enamel of the 11, 12, D. Clasp denture
21, 22, 31, 32, 41, 42 teeth is thin in the E. Splint-denture
region of cutting edge, there is a sulcate
enamel pit 1,5 mm wide which encircles 42. During the examinations and everyday
the tooth and is parallel to the cutting orthopedic manipulations a dentist uses a
edge. The cusps of the 16, 26, 36, 46 teeth dental mirror. How to sterilize dental mi-
are underdeveloped and have conical rrors?
form. What is the most likely diagnosis?
A. In the triple solution for 30 minutes
A. Systemic hypoplasia B. In the 0,5% ethyl chloride solution for
B. Dentinogenesis imperfecta 20 minutes
C. Enamel dysplasia C. In the 6% hydrogen peroxide solution
D. Local hypoplasia for 6 hours
E. Stainton-Capdepont dysplasia D. In the dry heat sterilizer at 180o C for 10
minutes
39. Preventive examination of a 45-year- E. in the 0,01% chloramine solution for 10
old patient revealed a carious cavity in minutes
the 23 tooth located within the mantle
dentin. The cavity floor and walls are 43. A 35-year-old woman was admi-
of dense, markedly pigmented dentin, tetd to the oral surgery department two
probing and cold test provoke no pain, hours after a road accident. Objectively:
percussion of the 23 tooth is painless. cutaneous integuments are pale, there are
Electroodontometry results - 6 mA. What beads of sweat on her face. Respirati-
is the most likely diagnosis? on is heavy and obstructed. Pulse rate
is 120 bmp, AP is 70/60 mm Hg. There
A. Chronic median caries is wound penetrating to the oral cavity
B. Acute median caries on the left cheek. During transportation
C. Chronic deep caries the woman was in supine position with
D. Acute deep caries her head thrown backwards. What kind of
E. Chronic periodontitis asphyxia is likely to develop in this pati-
ent?
40. A 29-year-old patient undergoes a
procedure of a metal inlay fabrication by A. Aspirating
the direct method. The inlay is intended B. Valvular
for the 26 tooth. Objectively: the 26 tooth C. Stenotic
presents a Black class I carious cavi- D. Obturative
ty. Electroodontometry results - 6 mA. E. Dislocational
Examination of the ready inlay revealed a
casting defect in the region of the external 44. A 10-year-old girl complains of an
angle. The dentist’s further actions will be: aesthetic defect. She has a history of
sucking her right thumb till the age of 7.
A. Fabrication of a new inlay Objectively: the height of the lower third
B. Cement fixation of the inlay of face is somewhat reduced. There is a 9
C. Plastic fixation of the inlay mm gap in sagittal direction between the
D. Inlay fixation and restoration of the upper and lower incisors, Engle’s class 2.
defect with a composite As a result of Eschler-Bittner test the gi-
E. Inlay fixation and restoration of the rl’s face appears at first better, then worse.
defect with amalgam What clinical form of occlusal anomaly is
most likely?
41. A 53-year-old patient complains of mi-
ssing of mandibular teeth. He has a hi-
Kok 3 Stomatology 2011 7

A. Maxillary macrognathia and mandi- A. Exacerbation of chronic periodontitis


bular micrognathia of the 26 tooth
B. Maxillary macrognathia B. Acute purulent periodontitis of the 26
C. Mandibular micrognathia tooth
D. Maxillary prognathism with lateral C. Acute pulpitis of the 26 tooth
compression D. Acute purulent periostitis of upper jaw
E. Mandibular retrognathia of the 26 tooth on the left
E. Periodontitis of the 26, 27, 28 teeth
45. A 36-year-old patient requires a full
coverage swaged metal crown for the 48. A 38-year-old patient complains of
37 tooth. The patient presented to a cli- acute paroxysmal pain in the region of
nic for the crown adjustment. During his left upper jaw, left eye and temple. The
the adjustment it was revealed that the pain is lasting (2-3 hours), gets worse at
crown was to loose to fit the tooth cervix night. Objectively: the 26 tooth has a deep
properly. How to eliminate this defect? carious cavity, floor probing causes painful
response, thermal stimuli provoke long-
A. To get an impression and swage the lasting pain, percussion provokes minor
crown anew pain. What is the most likely diagnosis?
B. To swage a new crown using the same
matrix A. Acute diffuse pulpitis
C. To bend in the crown edges by using B. Pulpitis complicated by the periodonti-
beak-shaped forceps tis
D. To anneal the crown C. Acute limited pulpitis
E. To bend in the crown edges by using D. Exacerbation of the chronic pulpitis
clamp forceps E. Acute purulent pulpitis
46. A 31-year-old patient complains of 49. A 46-year-old patient has been di-
experiencing recurrent pain in the 48 agnosed with chronic concrementous
tooth for 4 years. Objectively: right pulpitis of the 27 tooth. X-ray picture
submandibular lymph nodes are enlarged, shows that the concrement is located in
painless. Mucous membrane around the the crown part of the tooth cavity and
partialy erupted 48 tooth is hyperemic. On occupies 1/4 of its volume. Select an
probing the subgingival pocket of the 48 adequate method of treatment:
tooth exudes pus mixed with blood. What
additional study will be most informative A. Vital extirpation
for diagnosing this pathology? B. Biologic method
C. Devital extirpation
A. Radiography D. Vital amputation
B. Blood count E. Devital amputation
C. Electroodontometry
D. Blood and urine tests 50. The child is 13 years old. The disease
E. Contrast radiography began suddenly with a body temperature
rise up to 39o C, swelling of the upper
47. A 44-year-old patient consulted a jaw on the left. Objectively: the face
surgeon about constant acute pain in the is asymmetric due to the left cheek
upper jaw region on the left that is getti- edema. The skin over the swollen area is
ng worse during teeth joining. The pain hyperemic and tense. The 26 tooth exhibi-
was noted 3 days ago. Objectively: the ts a large carious cavity, percussion of the
face is symmetrical, mouth opening is 26 tooth causes pain reaction. The 25, 26,
unlimited. The crown fof the 26 tooth 27 teeth are mobile. Mucogingival juncti-
is half-destroyed. Probing of the cari- on in the region of the 24, 25, 26, 27 tooth
ous cavity is painless. Percussion of the is indistinct, alveolar mucosa is hyperemic
26 tooth provokes acute pain. Mucous and bilaterally swollen in the region of the
membrane of the alveolar process is 25, 26. 27 teeth. What is the most likely
edematic, hyperemic on the level of the diagnosis?
26 tooth. The 26 tooth was treated before.
What is your provisional diagnosis?
Kok 3 Stomatology 2011 8

A. Odontogenous maxillary osteomyelitis


to the left of the 26 tooth A. Gelin
B. Odontogenous maxillary periostitis to B. Stomatoplast
the left of the 26 tooth C. Stomalgin
C. Chronic periodontitis of the 26 tooth D. Orthocor
D. Suppuration of the radicular cyst E. Dentafol
induced by the 26 tooth
E. Odontogenous purulent maxillary 55. A 20-year-old patient complains about
sinusitis induced by the 26 tooth unaesthetic look of the 24 tooth from
the moment of its eruption. Objectively:
51. A 5-year-old child presents with enamel of the 24 tooth is partly absent,
chronic deep caries within the contact the dentine is yellow. The 64 tooth was
and masticatory surfaces of the 75, 74, 84, treated more than once when she was a
85 teeth. Which filling material should be child, but because of frequent exacerbati-
used? ons, edemas and gingival fistula the 64
tooth was extracted when she was 9 years
A. Silver amalgam old. What is the most likely diagnosis?
B. Silidont
C. Evicrol A. Localised hypoplasia
D. Silicin B. Initial caries
E. Infantid C. Fluorosis
D. Systemic hypoplasia
52. A patient complains of general E. Enamel aplasia
weakness, pain in the gums, halitosis.
Objectively: the patient is pale, adynamic, 56. A 45-year-old patient complains of
body temperature is 38, 5oC, submandi- burning tongue, a metallic taste in mouth.
bular lymph nodes are enlarged, pai- Three months ago she got a dental bridge
nful on palpation. Interdental gingival made of gold and supported by the 16, 14
papillae are inflamed, their tops are teeth. Oral cavity exasmination reveals no
"detruncated", covered with gray-yellow objective changes. The 36, 37, 46 teeth are
necrotic incrustation. Radiography of sealed with amalgam fillings. What is the
alveolar process and blood count reveal most likely cause of this condition?
no apparent changes. What is the most li-
kely diagnosis? A. Galvanic currents
B. Allergy
A. Acute ulcerous gingivitis C. Neurologic disorder
B. Generalized periodontitis D. Chemical factors
C. Acute leukosis E. Mechanic trauma
D. Agranulocytosis
E. Localized periodontitis 57. A 25-year-old patient complains of
gingival haemorrhage and enlargement.
53. Two years after radiation therapy a Objectively: hypertrophy of gingival papi-
49-year-old patient has been diagnosed llae by 1/3 of the crown height in the
with recurrence of the lower lip carci- frontal region of mandible. Periodontal
noma. Objectively: there is a 1x2 cm large pockets are absent. What is the most li-
neoplasm with an ulcer in the centre in the kely diagnosis?
region of the right half of the lower lip.
Palpation of the right submandibular regi- A. Hypertrophic gingivitis
on reveals two enlarged, dense, painless B. Catarrhal gingivitis
lymph nodes of round shape. Specify the C. Ulcerous gingivitis
stage of the disease: D. Localized periodontitis
E. Epulis
A. T2 N1 M0
B. T2 N0 M0 58. A chemical plant worker consulted a
C. T1 N1 M0 dentist about an oral mucosa burn caused
D. T1 N2 M0 by caustic soda. Which of the followi-
E. T1 N0 M0 ng medications should be chosen for
emergency care?
54. A 38-year-old patient ordered a
clasp upper jaw prosthesis. Its fabricati-
on involves forming of a fireproof model.
What material should be applied for
duplication?
Kok 3 Stomatology 2011 9

A. 0,5% acetic acid solution A. Removable lamellar denture with a


B. 50% ethyl alcohol mouth opening stopper
C. 0,1% liquid ammonia B. Schroeder appliance with sliding pivot
D. 3% sodium chloride solution C. Petrosov’s appliance with a stopper
E. 2% citric acid solution D. Yadrova’s appliance
E. Khodorovich-Burgonskaya appliance
59. A patient undergoes orthopaedic with a stopper
treatment of bounded edentulous spaces
on both jaws by means of fullcast combi- 63. Parents of a 12-year-old child complain
ned dental bridges. The patient has a hi- about white patches on the upper front
story of essential hypertension. Which teeth, which appeared six months ago.
haemostatic drug SHOULD NOT be used Objectively: chalk-like patches on the
for the gums retraction? vestibular surfaces in the precervical area
of the 13, 12, 11, 21, 22, 23 teeth. Their
A. Noradrenaline enamel is dull, pliable and rough on probi-
B. Hydrogen peroxide ng. There is a history of short-term pain
C. Aluminium chloride caused by chemical stimuli. What is your
D. Iron sulphate provisional diagnosis?
E. Tetrahydrozoline
A. Acute initial caries
60. Parents of a 1,5-month-old chi- B. Chronic initial caries
ld complain of whitish depositions on C. Acute superficial caries
the child’s buccal and labial mucosa. D. Systemic enamel hypoplasia
Objectively: labial and buccal mucosa E. Dental fluorosis
is hyperemic and covered with caseous
deposition that can be easily removed. 64. A 3-year-old girl has been diagnosed
Specify the causative agent of this disease: with acute odontogenous periostitis of
mandible starting from the 74 tooth. It
A. Candida fungi is required to perform periostomy and
B. Vincent’s spirochaetes extract the 74 tooth. The child is excited.
C. Fusiform bacillus Choose the best type of anesthesia for the
D. Loeffler’s bacillus surgery:
E. Diplococci
A. Intravenous anaesthesia
61. A 7-year-old boy complains of fever B. Mandibular anaesthesia
up to 38oC, headache, sore throat. C. Infiltration anaesthesia
Objectively: slightly hyperemic soft palate D. Intubation anaesthesia
mucosa, anterior palatine arches and E. Central anaesthesia
tonsils exhibit erosions. Submandibular
lymph nodes are slightly enlarged, pai- 65. A 48-year-old patient complains of
nless. What is the causative agent of this pain in the 32, 38 teeth which arose
disease? 3 months after installing a dental bri-
dge upon them. Objectively: the gums
A. Coxsackie virus around the abutment teeth are hyperemic,
B. Herpes simplex edematous, mobile in the vestibular-oral
C. Epstein-Barr virus direction. What is the most likely cause of
D. Loeffler’s Bacillus this complication?
E. Bordet-Gengou bacillus
A. Functional overload of abutment teeth
62. A 68-year-old patient had a stroke, paradontium
after which she now often experiences B. Improper preparation of abutment
urges to vomit causing unilateral TMJ di- teeth
slocations. The dislocations could only be C. The form of the intermediate denture
reduced under anesthesia. The 21-28, 15, part
16, 17, 18, 36, 37, 44, 46, 47 teeth are missi- D. Contact of the denture with opposing
ng. What denture construction is optimal teeth
for this patient for the treatment period? E. Solid food consumption
66. An 11-year-old child complains of a
carious cavity in the left upper molar, whi-
ch appeared six months ago. Objectively:
the medial contact surface of the 26 tooth
exhibits a carious cavity located within the
Kok 3 Stomatology 2011 10

mantle dentin. The cavity floor and walls revealed a cariuos cavity communicati-
are dense, pigmented, with no pain upon ng with the tooth cavity on the approxi-
probing. The response to thermal stimuli mal surface of the 54 tooth. Halitosis is
is absent. Percussion of the tooth causes present. Superficial probing of the cavi-
no pain. Make a provisional diagnosis: ty is painless, deeper probing causes pain
reaction. Percussion is painless. Intervi-
A. Chronic median caries ewing revealed that hot food caused
B. Chronic deep caries pain. Select the medication for antiseptic
C. Acute median caries treatment of root canals:
D. Acute deep caries
E. Chronic periodontitis A. Sodium hypochlorite
B. Aethonium solution
67. A 42-year-old patient was diagnosed C. Furacilin solution
with exacerbation of generalized grade D. Alcohol
II periodontitis accompanied by abscess E. Formalin solution
formation. What method of general
treatment should be administered in the 71. A 14-year-old teen complains of
first place? gingival haemorrhages during tooth
brushing. Objectively: gingival mucosa is
A. Antibiotic therapy hyperemic, pastous, bleeds when touched.
B. Vitamin therapy Schiller-Pisarev test is positive. PMA
C. Hyposensitization therapy index - 70%. Hygienic index - 3,0. X-ray
D. Stimulating therapy picture of the frontal area depicts no evi-
E. Detoxification therapy dent changes. What is the most likely di-
agnosis?
68. A 43-year-old patient complains of
increased tooth sensitivity to thermal and A. Chronic catarrhal gingivitis
chemical stimuli, gum itch. Objectively: B. Chronic periodontitis
the roots of the teeth are exposed to C. Acute catarrhal gingivitis
1/3 of their length, the gums are dense, D. Chronic hypertrophic gingivitis
pale pink. Thre is a small amount of E. Exacerbation of chronic periodontitis
dental plaque. In the region of the 15,
14, 24, 25, 44, 34 teeth there are wedge- 72. A 50-year-old patient needs a splint for
shaped defects within the dentin. Probi- the lower front teeth. Objectively: the 33,
ng of exposed cervices and wedge-shaped 32, 31, 41, 42, 43 teeth present I-II class
defects causes pain reaction. What is the mobility and are devitalized. Which spli-
most likely diagnosis? nt would be the most appropriate in this
case?
A. Parodontosis
B. Atrophic gingivitis A. Mamlok-type splint
C. Generalized periodontitis B. Elbrecht’s splint
D. Gingival recession C. Kurliandsky splint
E. Localized periodontitis D. Murray’s splint
E. Van Thiel splint
69. A 32-year-old patient complains about
mouth soreness, body temperature rise 73. A 34-year-old male patient complai-
up to 38, 5oC, indisposition. Such condi- ns about acute spasmodic pain in the
tion has been occurring periodically for region of his upper jaw on the left that
several years after the patient had had a is getting worse as affected by cold sti-
cold. Objectively: lips are covered with muli. Toothache irradiates to the ear and
haemorrhagic crusts, hyperaemic mucous temple. He had acute toothache of the 37
membrane of lips and cheeks has erosions tooth one year ago, but he didn’t consult
covered with fibrinous films. Hypersali- a dentist. Pain recurred three days ago.
vation is present. What is the most likely Objectively: the 37 tooth has a carious
diagnosis? cavity communicating with the dental
cavity. Probing of the opened carious
A. Multiform exudative erythema cavity is extremely painful. X-ray picture
B. Pemphigus vulgaris shows widening of periodontal fissure at
C. Herpes recidivicus the root apex of the 37 tooth. What is the
D. Herpetiform Duhring’s dermatitis most likely diagnosis?
E. Stevens-Johnson syndrome
70. Examination of a 5-year-old child
Kok 3 Stomatology 2011 11

A. Exacerbation of chronic pulpitis A. Light-cure microhybride composite


B. Exacerbation of chronic granulating B. Glass ionomer cement
periodontitis C. Silicophosphate cement
C. Exacerbation of chronic fibrous peri- D. Light-cure fluid composite
odontitis E. Light-cure microfilled composite
D. Acute diffuse pulpitis
E. Acute purulent pulpitis 78. After the extraction of the 26 tooth
a 43-year-old patient presents with a
74. A 30-year-old patient complains of a communication between the oral cavi-
painless tumour-like formation in the regi- ty and the maxillary sinus. X-ray picture
on of the left mandibular alveolar process. shows no changes in the maxillary sinuses.
Objectively: there is a pale pink 1x1,5 What tactics should be chosen by a dental
cm large tumour-like formation on the surgeon to prevent maxillary sinusitis?
vestibular surface of the alveolar process
at the 34 and 35 teeth. The formati- A. Plastic restoring of the communication
on has a broad base, it does not bleed. B. Socket tamponade with a iodoform
Crowns of the 34, 35 teeth are intact, the turunda
teeth are non-mobile. X-ray picture of C. Socket filling with a hemostatic sponge
the alveolar process shows no changes in D. Sinus rinsing with an antibiotic solution
bone structure. What is the most likely di- E. Socket filling with a blood clot
agnosis?
79. A 25-year-old patient complains of
A. Fibrous epulis intermittent pain during taking sweet
B. Pleomorphic adenoma and cold food in the 21 tooth. Objecti-
C. Mucosa papilloma vely: there is a hard tissue defect within
D. Giant-cell epulis the enamel in the cervical region of the
E. Hypertrophic epulis 21 tooth. The enamel edges are uneven
and chalky. Probing the cavity bottom
75. A 57-year-old patient complains of provokes no pain, there is short-term pain
mobility of all mandibular teeth. Objecti- reaction to cold stimuli. What is the most
vely: the 34, 35, 36, 45, 46, 47 teeth are mi- likely diagnosis?
ssing. Orthopantomogram of the remaini-
ng teeth shows resorption of the alveolar A. Superficial caries
process within 1/3 of interdental septa hei- B. Wedge-shaped defect
ght. What construction should be offered C. Initial caries
to the patient? D. Enamel hypoplasia
E. Enamel erosion
A. Clasp denture with splinting appliances
B. Bridge dentures in the region of masti- 80. External examination of a 9-year-old
catory teeth boy revealed strongly expressed nasolabi-
C. Partial removable lamellar denture al and labio-mental folds, a shortening of
D. Cap splint the lower third of face. Examination of the
E. Van-Thiel splint oral cavity revealed late transitional denti-
tion, the upper front teeth completely
76. A 48-year-old patient consulted a overbite the lower teeth, the palate exhi-
dental surgeon about extraction of the 37 bits imprints of the lower incisors. What is
tooth’s roots. What tool should be chosen? the most likely diagnosis?
A. Closed-beak forceps A. Supraocclusion
B. Broad-beak forceps B. Dentoalveolar maxillary lengthening
C. S-shaped forceps C. Distal occlusion
D. Bayonet forceps D. Dentoalveolar mandibular lengthening
E. Curved beak-shaped forceps E. Mesial occlusion
77. A 35-year-old patient was diagnosed 81. A 70-year-old patient complains about
with chronic median caries of the 36 tooth. the mobility of the 21 tooth. The tooth
There is a Black’s class II cavity affecting has not been treated before. Objectively:
masticatory surface. What material should there is II class mobility of the 21 tooth,
be chosen for the tooth filling? the root is exposed by 1/2. What tool
should be used to extract the tooth?
Kok 3 Stomatology 2011 12

A. Straight forceps
B. S-shaped forceps A. Leukoplakia
C. Bayonet forceps B. Lichen ruber planus
D. Straight elevator C. Lupus erythematosus
E. Beak-shaped forceps D. Candidiasis
E. Soft leukoplakia
82. Following the upper jaw examinati-
on (according to Schroeder) a 65-year- 86. A 62-year-old patient presented to
old patient was found to have atrophi- the prosthodontics clinic with complai-
ed maxillary tubers, alveolar processes nts about poor fixation of complete
covered with atrophic, thin and pale removable lamellar dentures that shows
mucous membrane. What kind of functi- up during talking and wide mouth openi-
onal impression technique should be ng. The dentures were fabricated 5 days
applied? ago. In course of denture fabrication
process a mistake was made during:
A. Decompressive
B. Compressive A. Obtaining the functional impression
C. Dosated B. Occlusal plane orientation
D. Pressure-free C. Interalveolar dimension measurement
E. Differentiated D. Wax reproduction check
E. Plastic polymerization
83. An orthodontist monitors a 4-year-old
child with mouth breath. The child has 87. A 30-year-old patient has a carious
a history of adenotomy. Objectively: pri- cavity in the 11 tooth within parapulpar
mary dentition occlusion; the upper inci- dentin. The tooth corner and its cutting
sors overlap the lower ones by 1/3; distal edge are affected. What filling material is
surfaces of the second temporary molars best for restoration of the anatomic tooth
are situated in the same vertical plane. form?
What preventive device will help the child
to give up the habit of mouth breath? A. Light-cure composite
B. Silicate cement
A. Standard Schonherr’s vestibular screen C. Light-cure glass-ionomer cement
B. Vesibular and oral Kraus’ screen D. Chemical-cure paste-paste composite
C. Frankel’s function regulator E. Chemical-cure powder-liquid composite
D. Andresen-Haupl activator
E. Rudolph’s appliance 88. A 29-year-old patient complains of
acute paroxysmal pain in the upper jaw
84. A 17-year-old patient consulted an on the left, that gets worse during havi-
orthodontist about improper position of ng cold food and irradiates into the ear
an upper canine. Objectively: permanent and temple. A year ago she experienced
occlusion, class I Angle’s relationship of intense pain in the 27 the tooth but didn’t
the first molars, the 13 tooth has vestibular consult a dentist. Three days ago there
position above the occlusal line, there is a was the second attack of pain. Objecti-
6,5 mm gap between the 14 and 12 teeth. vely: there is a deep carious cavity in the
What period of orthodontic treatment wi- 27th tooth, interconnecting with the tooth
ll reduce the time of lidase phonophoresis cavity. Probing the open area causes acute
therapy? pain. What is the most likely diagnosis?
A. Active period A. Exacerbation of chronic pulpitis
B. Retention period B. Acute serous periodontitis
C. Preparatory period C. Acute diffuse pulpitis
D. Passive period D. Exacerbation of chronic periodontitis
E. - E. Acute limited pulpitis
85. Preventive examination of a 55-year- 89. A 23-year-old patient complains of
old man revealed a well-defined area of whitish spots on the masticatory teeth,
opaque mucosal epithelium of the left drawing of mouth during taking acidic
cheek that didn’t protrude above the food. The spots appeared about 3 months
surrounding tissues and could not be ago. Objectively: the cervical region of
removed on scraping. Crowns of the 34, the 46, 36, 27 teeth exhibits some chalk-
35, 36 teeth were strongly decayed and like spots that can be easily stained with
had sharp edges. What is the most likely 2% methylene blue, probing reveals the
diagnosis? surface roughness. What is the most likely
Kok 3 Stomatology 2011 13

diagnosis? A. Chicken pox-induced stomatitis


B. Acute herpetic stomatitis
A. Acute initial caries C. Exudative erythema multiforme
B. Acute superficial caries D. Measles-induced stomatitis
C. Enamel hypoplasia E. Scarlet fever-induced stomatitis
D. Endemic fluorosis
E. Chronic superficial caries 93. A 47-year-old patient with completely
edentulous upper jaw underwent delayed
90. A 56-year-old patient has an oval, implantation with cylindrical implants.
smooth, bright-red erosion on the red Prosthetics is recommended in:
border of her lower lip. Erosion is covered
with haemorrhagic crusts that can be A. 5-6 months
hardly removed. Crust removal induces B. 24 hours
slight haemorrhage. Light traumatization C. 2 weeks
of crust-free surface of erosion induces no D. 1 month
haemorrhage. Specify the type of lower E. 3 months
lip precancer:
94. Preventive examination of a 5-year-old
A. Abrasive precancerous Manganotti’s child reveals the infantile swallowing. The
cheilitis bad habit of thrusting the tongue between
B. Verrucous precancer of red border the teeth may cause the following compli-
C. Localized precancerous hyperkeratosis cation:
of red border
D. Bowen’s disease A. Incomplete eruption of the front teeth
E. Erythroplasia B. Broadenning of the upper dental arch
C. Broadening of the lower dental arch
91. A 55-year-old patient consulted a D. Narrowing of the lower dental arch
dentist about a roundish tumour-like E. Narrowing of the upper dental arch
formation of about 1 cm in diameter
located within the red border of his lower 95. After a preventive orthodontic exami-
lip. Objectively: the tumor-like formati- nation a 9-year-old child was diagnosed
on protrudes about 5 mm above the red with mesial occlusion. The treatment of
border, it is dense and grayish-red. The this pathology involves application of an
surface of the formation is covered with apparatus with mechanic action. What
thin scales that can be hardly removed. working element is to be applied in the
What is the most likely diagnosis? apparatus intended for the correction of
this pathology?
A. Verrucous precancer of the red border
of lip A. Screw or spring
B. Abrasive precancerous Manganotti’s B. Occlusal applications
cheilitis C. Elastics and buccal shields
C. Precancerous limited hyperkeratosis of D. Inclined plane
the red border of lip E. Screw and bite plate
D. Bowen’s disease 96. A 28-year-old male patient presented
E. Erythroplasia of Queyrat to a clinic on the 2nd day after the
92. A 4,5-year-old child presents with face trauma and complained of bilateral
eruptions on skin and in the mouth which mandibular pain, difficult swallowing and
appeared on the previous day. Objecti- obstructed breathing. Objectively: there
vely: the child is in medium severe condi- was a slight swelling in the region of the
tion, body temperature is 38, 3oC. Scalp, molars on both sides, the teeth were intact
trunk skin and extremities are covered but the jaw fragment with the 43, 42,
with multiple vesicles with transparent 41, 31, 32, 33 teeth deviated downward
content. Mucous membrane of cheeks, and backward. Manual reposition didn’t
tongue, hard and soft palate exhibits produce any positive result. Which appli-
roundish erosion covered with fibrinous ance should be used for treatment?
film. Gums remain unchanged. Submandi- A. Post’s appliance
bular lymph nodes are slightly enlarged. B. One-jaw Katz appliance
What diagnosis can be assumed? C. One-jaw Kurliandsky appliance
D. Brun appliance
E. Betelman appliance
Kok 3 Stomatology 2011 14

97. An 18-year-old patient presented to ns about white spots on the upper anterior
the orthopedic department with complai- teeth of her child. Objectively: the vesti-
nt of a large diastema. Objectively: bodi- bular surface of the 51 and 61 teeth exhi-
ly lateral movement of central incisors bits a defect within the enamel. Probing
induced by the missing 12, 22 teeth. What is painless. What is the optimal method of
is the optimal appliance for moving the treatment?
central incisors together?
A. Impregnation with silver nitrate soluti-
A. Korkhaus apparatus on
B. Vasilenko apparatus B. Treatment with fluorine lacquer
C. Cotton ligature C. Remodentum application
D. Kalvelis apparatus D. Preparation and filling
E. Begg appliance E. Electrophoresis of sodium fluoride
solution
98. A 30-year-old patient needs to have his
26 tooth extracted because of exacerbati- 102. A 20-year-old patient is to be given
on of chronic periodontitis. Objectively: applicative anaesthesia in the frontal regi-
the crown of the 26 tooth is decayed by on of the upper jaw prior to the conducti-
1/3. What forceps can be used for this on anesthesia. What concentration of li-
tooth extraction? docaine solution is required for applicati-
ve anesthesia?
A. S-shaped forceps with a projecting tip
on the left beak A. 10%
B. S-shaped forceps with a projecting tip B. 0,5%
on the right beak C. 2%
C. Straight forceps D. 3%
D. Straight elevator E. 15%
E. S-shaped forceps without projecting tips
103. A 13-year-old boy complains of pain
99. A 26-year-old patient consulted a in the region of the extracted 46 tooth,
dentist about the 24 tooth extracti- irradiating to the ear and temple, hali-
on because of chronic periodontitis tosis. The tooth was extracted 3 days
exacerbation. The crown of the 24 tooth is ago. Objectively: submandibular lymph
intact. Choose the right tool for the tooth nodes are enlarged, painful on palpati-
exraction: on. Mucosa around the extracted tooth is
hyperemic, edematous. The socket of the
A. S-shaped forceps without projecting 46 tooth is filled with a gray clot. What is
tips the most likely diagnosis?
B. S-shaped forceps with a projecting tip
on the left beak A. Alveolitis of the extracted 46 tooth
C. S-shaped forceps with a projecting tip B. Acute odontogenous mandibular
on the right beak osteomyelitis starting from the 46 tooth
D. Straight forceps C. Acute odontogenous lymphadenitis of
E. Straight elevator the right submandibular region
D. Acute odontogenous mandibular peri-
100. A 49-year-old patient consulted a ostitis starting from the 46 tooth
dental surgeon about the oral cavity E. Neuralgia of the III branch of trigemi-
sanation. He has an indication for the nus
extraction of the 16 tooth. Anamnesis: the
tooth hasn’t been treated before, decayed 104. A 35-year-old patient complains
within the last 4 years. Objectively: the about progressing throbbing pain in the
16 tooth’s crown is destroyed by over 26 tooth. Objectively: the 26 tooth has a
2/3, the mucosa exhibits no pathological carious cavity filled with softened denti-
changes. Which tool is required for the ne, tooth cavity is closed, probing of the
tooth extraction? cavity floor is painless, percussion causes
acute pain. There is I degree tooth mobi-
A. Bayonet root forceps lity. Roentgenological changes are absent.
B. S-shaped forceps (right) What is the most likely diagnosis?
C. Straight elevator
D. S-shaped closed-beak forceps
E. Crown bayonet forceps
101. Mother of a 3-year-old child complai-
Kok 3 Stomatology 2011 15

A. Acute purulent periodontitis thin this period it doubled in size. Objecti-


B. Acute purulent pulpitis vely: the right parotid region has a dense
C. Acute serous periodontitis 1,5 x 2 cm large neoplasm, the duct of the
D. Exacerbation of chronic periodontitis right parotid gland secretes transparent
E. Acute diffuse pulpitis saliva. The gland puncture gave no results.
What disease are these clinical presentati-
105. A 43-year-old patient consulted a ons most typical for?
dentist about pain in the 36th tooth. X-
ray examination revealed a breakage of A. Pleomorphic adenoma of parotid gland
an endodontic tool in the distal root. The B. Chronic lymphadenitis of parotid-
root apex exhibits a well-defined area of masticatory region
bone tissue destruction (d = 5 mm) with C. Retention cyst of parotid gland
clear margins. What method of treatment D. Fibroma of parotid-masticatory region
is most appropriate in this case? E. Lipoma of parotid-masticatory region
A. Dental hemisection 109. A 41-year-old patient complains
B. Root apex resection of mobility of the 24, 26, 27 teeth,
C. Conservative treatment purulent discharges from the socket of
D. Tooth replantation the extracted 25 tooth. 1,5 months ago
E. Root amputation the patient underwent a dissection along
the mucogingival junction and extracti-
106. A 25-year-old patient has a face on of the 25 tooth. Objectively: alveolar
trauma. Objectively: there is a signi- mucosa in the region of the 24, 26, 27
ficant soft tissue swelling of the left teeth is cyanotic and edematic. Along the
half face, haemorrhage into the left mucogingival junction there is a fistula
eye sclera, crepitation in the region of with protruding granulations. There are
nose bones. Palpation reveals the mobi- also purulent granulation discharges from
lity of the upper jaw, its percussion the socket of the extracted 25 tooth. In
reveals tympanitis. X-ray picture shows the right infraorbital region some soft ti-
the fracture line running through the ssue swelling is present. Which disease are
lower margin of the orbit on both sides of these clinical presentations most typical
the sphenozygomatic suture and reaching for?
behind the maxillary tuber. What is the
most likely diagnosis? A. Chronic limited osteomyelitis
B. Chronic alveolitis
A. Le Fort II maxillary fracture C. Maxillary actinomycosis
B. Le Fort I maxillary fracture D. Exacermation of chronic maxillary
C. Le Fort III maxillary fracture sinusitis
D. Fracture of the left articular bone E. Chronic diffuse osteomyelitis
E. Fracture of nose bones
110. A 37-year-old patient complai-
107. A 57-year-old patient complains of a ns of gingival haemorrhage, offensive
tumor in the left parotid region, which breath, tooth mobility, difficult masticati-
appeared 3 years ago. Over the past six on. Objectively: the gums are of cyanotic-
months, the tumor has increased signifi- red colour, periodontal pockets are 6
cantly. Objectively: there is a non-mobile mm deep and contain serous exudate,
lobulated tumour in the left parotid regi- there is II class tooth mobility, moderate
on, palpation causes minor pain reaction. amount of subgingival and supragingival
The skin over the tumour plicates easily, tartar; hygiene index is 3 points, there
its upper part has well-defined margins, is traumatic occlusion along the entire
the bottom part grows into the depth of length of the dental arches. What is your
gland. The tumour is 3x2,5 cm large. What provisional diagnosis?
is your provisional diagnosis?
A. Chronic generalized grade II peri-
A. Pleomorphic adenoma odontitis
B. Chronic parotitis B. Acute generalized grade I periodontitis
C. Fibroma C. Hypertrophic gingivitis
D. Cyst D. Histiocytosis-X
E. Parotid gland carcinoma E. Acute generalized II grade periodontitis
108. A 53-year-old patient complains of a 111. Parents of a 12-year-old child
tumour in the right parotid region. The consulted an orthodontist about improper
tumour was revealed six months ago, wi- position of the child’s upper teeth.
Kok 3 Stomatology 2011 16

Objectively: the face is narrow, elongated; of treatment is indicated for this case?
the developing occlusion is present
(temporary second molars). The 13 and 23 A. Root apex resection
teeth are located beyond the dental arch, B. Tooth extraction
they deviate to the lips above the occlusal C. Tooth crowning
plane, there is a 2,5 mm gap between the D. Tooth replantation
12 and 14 teeth, and a 1,5 mm gap between E. -
the 22 and 24 ones, 45o rotation the 33 and
115. During the examination of a soldered
43 teeth is present. Choose the most rati-
dental bridge supported by the 35 and 38
onal method of treatment:
teeth the following flaws were revealed:
A. Extraction of the premolars and the pores in the soldered joints between
relocation of the canines the intermediate part and supporting
B. Expansion of dental arches in the region crowns, strongly expressed masticatory
of canine apices tubercles, early contact with the opposi-
C. Extraction of temporary premolars and ng teeth, the intermediate part is tightly
expansion of dental arches adjacent to the mucous membrane of the
D. Compact osteotomy and expansion of alveolar process. What is the way to elimi-
dental arches nate these defects?
E. All the answers are wrong
A. To remake the dental bridge
112. Before extraction of a left upper B. To correct the intermediate part
tooth a 49-year-old patient had been given C. To correct the masticatory surface and
plexus anaesthesia with Ultracaine Forte. polish the joints
After the operation the socket didn’t get D. To correct the masticatory surface and
filled with a blod clot. What is the way increase the height of the intermediate
to prevent alveolitis development in this saddle part up to 2 mm
patient? E. To remove the tubercles of opposing
teeth
A. To fill the socket with a loose iodoform
tampon 116. A 42-year-old woman came to
B. To fill the socket with a hemostatic prosthetic dentistry to make dental
sponge prosthetics. Objectively: dental formula is
C. To rinse the socket with microcide 18 . . . . 13 12 11 21 22 23 . . . . 28
solution 48 47 46 45 44 43 42 41 31 32 33 34 35 36 37 .
D. To rinse the socket with 0,1% chlorhexi- The bite is deep, clinical crowns are low,
dine solution survey line is not marked. The patient
E. To fill the socket with antibiotic powder suffers from epileptiform attacks. What
prosthesis is indicated?
113. Examination of a 45-year-old pati-
ent revealed some white patches on the A. Partial removable lamellar prosthesis
buccal mucosa along the teeth closure li- with metal base
ne that didn’t protrude above the inflamed B. Dental bridges
and edematous surrounding tissues. The C. Partial removable lamellar plastic
patches could not be removed on scrapi- prosthesis with retaining clasps
ng. A patient has smoken an average of D. Partial removable lamelalr prosthesis
3 packs of cigarettes a day for 20 years. with supporting-retaining clasps
Specify the disease of buccal mucosa: E. Clasp denture
A. Leukoplakia plana 117. A 43-year-old female patient
B. Erosive leukoplakia complains of mobility of the lower front
C. Pemphigus teeth. Objectively: the 48, 47, 46, 45, 44,
D. Tappeiner’s leukoplakia (leukoplakia 34, 35, 36, 37, 38 teeth are missing. The
nicotinica) 43, 42, 41, 31, 32, 33 teeth exhibit grade
E. Lichen ruber planus II mobility, the mucous membrane around
them is swollen and cyanotic. What spli-
114. X-ray examination of the maxillary nting construction is optimal for this pati-
alveolar process of a 35-year-old patient ent?
reveals a root fracture of the 11 tooth in its
apical region. The tooth has been treated
before, the canal is filled. The patient has a
history of the tooth trauma. What method
Kok 3 Stomatology 2011 17

A. Composite splint reinforced with fi- Objectively: submandibular lymph nodes


berglass are enlarged and painful, gums are
B. Partial removable denture with a vesti- hyperemic and cyanotic, they are also
bular bar swollen and bleed at probing, periodontal
C. Clasp denture with multilink clasp and pockets are 5-8 mm deep and contain
claw-like processes sero-purulent exudate. There is massi-
D. Clasp denture with antithrowers ve dental deposit, gingival recession and
E. Clasp denture-splint in the frontal part tooth migration. X-ray picture shows di-
ffuse osteoporosis and irregular, predomi-
118. A 58-year-old patient was diagnosed nantly vertical, resorption of alveolar
with fracture of lower jaw with formati- septa down to 2/3 of root length. What
on of a false joint. Objectively: the 38, 36, is the most likely diagnosis?
32, 41, 43, 48 teeth are missing. Preserved
teeth are intact, stable. There is no di- A. Exacerbation of generalized grade III
splacement of lower jaw fragments. X-ray periodontitis
picture shows a bone tissue defect to up 2 B. Grade III parodontosis
cm large. What construction of prosthesis C. Eosinophilic granuloma
is indicated in this situation? D. Chronic generalized grade III peri-
odontitis
A. Fixed Oxman’s prosthesis with pivot E. Exacerbation of generalized grade II
point periodontitis
B. Oxman’s single-jointed prosthesis with
pivot point 122. A 37-year-old patient complains of gi-
C. Gavrilow’s prosthesis with pivot point ngival haemorrhage, pain, halitosis, body
D. Kurlyandsky prosthesis with roller temperature rise up to 37, 6oC. Objecti-
damper clasp vely: the gums are markedly hyperemic,
E. Oxman’s double-jointed prosthesis with swollen, bleed on touch, periodontal
pivot point pockets are 3 mm deep, contain sero-
purulent exudate. Orthopantomogram
119. A 15-year-old girl consulted a dental shows diffuse osteoporosis of alveolar
surgeon about a moderate swelling of the processes, resorption of the interdental
parotid-masticatory region on the left. septa down to 1/3 of their height. What is
Objectively: on palpation, the left parotid the most likely diagnosis?
gland is nodular, dense, painless. The duct
exudes somewhat turbid saliva. The duct A. Generalized grade I periodontitis
mouth is dilated, the surrounding mucous B. Exacerbation of chronic catarrhal gingi-
membrane is cyanotic, pasty, with imprints vitis
of teeth. Make a diagnosis: C. Generalized grade II periodontitis,
chronic course
A. Chronic parenchymatous sialoadenitis D. Generalized grade II periodontitis,
B. Chronic interstitial sialoadenitis exacerbation
C. Sialolithiasis E. Generalized grade I periodontitis,
D. Pleomorphic adenoma chronic course
E. Cyst
123. A 21-year-old patient complains of a
120. A 53-year-old patient presented drop-shaped bulge in the cervical region.
to a dental clinic for the orthopaedic Objectively: there is an enamel droplet of
treatment of the 27 tooth. During about 2 mm in diameter on the vestibular
preparation for local anesthesia, the surface of the 33 tooth. What is the most
patient felt sick (weakness, pallor, likely diagnosis?
hyperperspiration), and lost consci-
ousness. What is the most likely di- A. Enamel hyperplasia
agnosis? B. Enamel hypoplasia
C. Fluorosis
A. Syncope D. Supernumerary tooth
B. Collapse E. Marble-bone disease
C. Anaphylactic shock
D. Traumatic shock 124. A 49-year-old male patient complains
E. Cardiogenic shock about gingival haemorrhage, teeth mobi-
lity in the frontal region of his lower jaw,
121. A 27-year-old patient complains of hypersensitivity of dental cervixes. X-ray
bleeding, pain and swelling of gums, tooth picture shows widening of periodontal fi-
mobility, halitosis, general weakness. ssure in the region of the 42, 41, 31, 32
Kok 3 Stomatology 2011 18

teeth, as well as resorption of alveolar taken along with the patient to the denti-
process tissue to 1/3 of root length. The st’s. Objectively: the alveolar socket edges
42, 32 teeth present with I degree mobili- are slightly hyperemic, the socket bone
ty, the 41, 31 teeth present with II degree walls are intact, the socket itself is filled
of teeth mobility. What temporary splint with a blood clot. What is the optimal way
should be used for frontal stabilization of of treatment?
dentition?
A. Replantation of the 11th tooth with a
A. Novotny prior filling of the channel
B. Mamlock B. Replantation of the 11th tooth before
C. Treuman the filling of the channel
D. Kogan C. Extraction of the 11th tooth
E. Kopeinin D. Reposition and fixation of the 11th
tooth
125. A 7-year-old child presents with E. Socket suturing with catgut
deterioration of general health status,
body temperature rise up to 38, 0oC. 129. A 20-year-old patient got an injury.
Objectively: buccal mucosa is covered Objectively: the patient’s chin and lower
with white maculae of a pinhead size, jaw up to the 34 and 45 teeth are missing.
protruding above the level of the mucosa. The 45, 46, 47, 48, 34, 35, 36, 37 teeth are
There are pink maculae on the palate. stable. At what stage of medical evacuati-
What is the most likely diagnosis? on the patient will get special medical aid?
A. Measles A. Specialized army surgical hospital
B. Chicken pox B. Battalion aid station
C. Scarlet fever C. Regimental aid station
D. Diphthera D. Separate medical detachment
E. Infectious mononucleosis E. Separate medical battalion
126. A 32-year-old patient needs to 130. A 25-year-old patient complains
be provided with metal-ceramic crowns about a light brown spot in the upper
for the 12, 11, 21 and 22 teeth. Duri- foretooth. Objectively: the 23 tooth has a
ng the dental visit he is given infi- single light brown spot in the cervical regi-
ltration anaesthesia with Ultracain DS on. Probing shows smooth surface. The
anaesthetic. What elements enter into its tooth is nonresponsive to cold and probi-
composition? ng. What is the most likely diagnosis?
A. 4% articaine with adrenaline A. Chronic initial caries
B. 2% mepivacaine with adrenaline B. Fluorosis
C. 4% articaine without a vasoconstrictor C. Local enamel hypoplasia
agent D. Acute initial caries
D. 3% mepivacaine without a vasoconstri- E. Chronic superficial caries
ctor agent
E. 2% articaine with epinephrine 131. A 33-year-old patient complai-
ns about an ulcer of oral cavity floor,
127. A 25-year-old got a polytrauma as that is located under his tongue on a
a result of a road accident. On exami- level between the 43 to the 33 tooth.
nation, he was diagnosed with dislocation Examination reveales that ulcer edges
asphyxia. What kind of first aid should be are undermined and scalloped. Its grey-
rendered? yellow floor is shallow and it is covered
with small, easily bleeding granulations.
A. Tongue fixation There is no ulcer infiltration. Make a cli-
B. Tracheotomy nical diagnosis:
C. Artificial ventilation of lungs
D. Medicamental management of A. Tubercular ulcer of oral cavity floor
laryngeal stenosis B. Decubital ulcer of oral cavity floor
E. Removal of mucus and foreign bodies C. Cancerous ulcer of oral cavity floor
from the upper airways D. Gummatous ulcer of oral cavity floor
E. Migratory granuloma of oral cavity floor
128. A 7-year-old child has been di-
agnosed with a complete dislocation of 132. A 60-year-old patient presented
the 11 tooth caused by a trauma that to a clinic for checking her complete
occurred 24 hours ago. The tooth has been removable denture. During the teeth
Kok 3 Stomatology 2011 19

placement on the wax bases a gap A. Chronic gangrenous pulpitis


between the front teeth and cuspal B. Chronic granulating pulpitis
interference of the lateral teeth was C. Chronic concrementous pulpitis
revealed. What mistake was made? D. Recurrent caries
E. Chronic fibrous pulpitis
A. The patient was found to have anterior
occlusion instead of centric one 136. On the 7th day after the operation on
B. The patient was found to have posterior the abdominal cavity a 30-year-old pati-
occlusion instead of centric one ent presented with pain and sweling in the
C. The patient was found to have lateral region of parotid gland; body temperature
occlusion instead of centric one rise up to 39o C; limited mouth opening,
D. The models were improperly plastered dryness. Gland massaging results in di-
in the occludator scharge of purulent exudate from its duct.
E. During establishing the centric occlusi- The patient can be diagnosed with:
on, the occlusal rims were overpressed
A. Acute non-epidemic parotiditis
133. A 45-year-old female patient B. Acute epidemic parotiditis
consulted a prosthodontist about dental C. Phlegmon of submasseteric space
prosthetics. She works as TV announcer. D. Parenchymatous parotiditis
Objectively: the lower jaw dentition is wi- E. Phlegmon of parotidomasseteric region
thout spaces, the upper jaw has a free-
end edentulous space and a boundary 137. A 50-year-old patient complains of
edentulous space in the lateral parts. The a traumatic break-off in the front teeth.
remaining 18, 17, 13, 12, 11, 21, 22, 23, Objectively: the crowns of the 12, 21 teeth
24 teeth are stable. What fixing elements are broken off by 3/4, X-ray picture shows
should be used in a clasp denture for that the channels of the 11, 21 teeth are
cosmetic purposes? properly filled, the other teeth are intact.
What actions should the dentist take?
A. Attachments and bar system
B. Telescopic crowns A. Fabricate fullcast metal stumps with a
C. Ney clasps post and restorative metal-ceramic crowns
D. Jackson clasps B. Remove the 11 and 21 teeth and fabri-
E. Dentoalveolar clasps cate a dental bridge supported by the 12
and 22 teeth
134. A 53-year-old patient with generali- C. Remove the 11 and 21 teeth and fabri-
zed periodontitis presented to a cate a partial removable lamellar denture
prosthodontics clinic for fabrication of for the upper jaw
fixed splints for his both jaws intended D. Fabricate restorative swaged metal
to stabilize the dentitions along the enti- crowns for the 11 and 21 teeth
re dental arch. Which of the upper jaw E. Fabricate a dentogingival plastic splint
buttresses will be functionally signifi-
cant for the distribution of masticatory 138. A 16-year-old girl consulted a denti-
pressure? st about a cosmetic defect on her front
teeth, namely some dark spots and dentin
A. Frontonasal, zygomatic, pterygopalati- defects. The spots were detected immedi-
ne, palatine ately after teeth eruption, the defects
B. Frontonasal, zygomatic, pterygopalatine developed with the lapse of time. The
C. Zygomatic, pterygopalatine, palatine patient was diagnosed with erosive form
D. Frontonasal, pterygopalatine, palatine of dental fluorosis of the 16, 11, 12, 22, 26,
E. Frontonasal, zygomatic, palatine 31, 32, 36, 41, 42, 46 teeth. Select a method
of treatment:
135. A 38-year-old patient complains of
discomfort, occasional sensation of teari- A. Restoration
ng in the 17 tooth, infrequent pain during B. Remineralization
eating hot food that lasts for 10-20 mi- C. Prosthetics
nutes. Objectively: the 17 tooth is filled D. Surgical treatment
with composite materials, the tooth is sli- E. Regular surveillance
ghtly darker than the other ones. What is
the most likely diagnosis? 139. During extration of the 26 tooth a 34-
year-old patient developed tinnitus, skin
pallor, weakness. She was diagnosed wi-
th vertigo. What is the most appropriate
tactics of emergency care?
Kok 3 Stomatology 2011 20

surface formed by thin, closely adheri-


A. Placing the patient in Trendelenburg ng and thick-based squamae. Regional
position lymph nodes are enlarged, mobile, dense
B. Intravenous injection of 50% analgin and painless. What is the most likely di-
C. Intravenous injection of 0,1% adrenalin agnosis?
D. Intravenous injection of cordiamin
E. Intravenous injection of prednisolone A. Lower lip cancer
B. Verruciform precancer
140. A 30-year-old male patient consulted C. Pyogenic granuloma
a dentist about a swelling in the region D. Keratoacanthoma
of his upper lip. Objectively: the face is E. Viral wart
asymmetric because of upper lip edema,
nasolabial fold is smoothed. Mucogingival 143. A 12-year-old child presents with mi-
fold of the upper jaw vestibular surface ssing 31 and 41 teeth, the gap between the
in the region of the 11, 12, 21 teeth is 32 and 42 teeth is 10 mm. Choose the most
smoothed, hyperemic. Palpation is pai- rational denture construction:
nful. Fluctuation is present. The patient
was diagnosed with acute purulent peri- A. Partial lamellar removable adjustable
ostitis of the upper jaw satrting from the denture
21 tooth. Choose the treatment tactics: B. Interdental wedge
C. Clasp denture
A. Preservation of the 21 tooth, peri- D. Dental bridge
osteotomy, anti-inflammatory therapy E. Adjustable microprosthesis
B. Preservation of the 21 tooth, anti-
inflammatory therapy 144. A 6,5-year-old child has a gap 2,5-
C. Extraction of the 21 tooth, peri- 3 mm large between frontal teeth from
osteotomy, anti-inflammatory therapy canine to canine. Relationship of the first
D. Extraction of the 21 tooth, peri- permanent molars complies with Angle’s
osteotomy class I. Specify the severity degree of bite
E. Extraction of the 12, 21, 22 teeth, deformation:
periosteotomy, anti-inflammatory therapy
A. I degree
141. A 37-year-old patient was admitted to B. II degree
the oral surgery department. The woman C. III degree
complains of pain in her cheek that is D. IV degree
made worse by touch. It is known from the E. V degree
anamnesis that five days ago the patient
fell down from the stairs and "hurt"her 145. An 8-year-old child complains of
cheek. Objectively: in the depth of cheek a painless round formation under the
examination revealed a circumscribed tongue on the left which was noticed by
infiltration, cheek skin is hyperaemic and accident about a month ago. Objectively:
doesn’t make a fold, fluctuation symptom there is a roundish formation up to 2,0
is present. Mucous membrane is edematic cm in diameter in the sublingual region
and has teeth indentations. What is the deep in the soft tissues of mouth floor. It
most likely diagnosis? is soft, painless to palpation, the mucous
membrane over it is of grayish-blue color.
A. Suppurated cheek haematoma What is the most likely diagnosis?
B. Traumatic osteomyelitis of mandible
C. Cheek haematoma A. Ranula of the left sublingual salivary
D. Cheek phlegmon gland
E. Acute lymphadenitis B. Dermoid cyst of the mouth floor
C. Epidermoid cyst of the mouth floor
142. A 42-year-old builder complains D. Lymphangioma of the mouth floor
about a condyloma on his lower lip. It E. Hemangioma of the mouth floor
appeared 1,5 month ago. It has been si-
gnificantly growing throughout the last 146. X-ray examination of the 46 tooth
week. Objectively: the red border of the in a 7-year-old child reveals the followi-
lower lip is cyanotic and infiltrated, it has ng: the root walls are parallel, their thi-
some isolated closly adhering squamae. ckness gradually decreases, they have
There is a well-defined hemispherical pointed ends. The root canal converges
formation 8 mm in diameter and 4 mm towards the tooth cavity, and diverges at
high in the centre. The formation is of the developing apical hole. Periodontal
grey-and-blue-and-red colour, it has rough gap is of the same width along the enti-
Kok 3 Stomatology 2011 21

re length of the root. At the root apex it 150. A patient complains about
merges with the growth zone. Specify the paroxysmal upper jaw toothache on
stage of root development: the left that is getting worse at night.
Toothache intensifies also under stimulati-
A. Continuing root apex development on and irradiates to the left eye and
B. Continuing root development temple. Similar attacks were noted three
C. Open apex months ago, the patient didn’t undergo
D. Continuing periodontium development any treatment. Objectively: the 25 tooth
E. Complete root and periodontium has a deep carious cavity communicati-
development ng with the tooth cavity. Probing causes
acute pain at the point of communicati-
147. A 5-year-old child complains about on, vertical percussion is slightly pai-
spontaneous pain in an upper jaw tooth on nful, horizontal one is painless. Mucous
the right that is getting worse at night and membrane in the projection of root apex
during eating cold food. Objectively: the of the 25 tooth is unchanged, its palpati-
65 tooth has a deep cavity communicating on is painless. Thermal probe causes
with the tooth cavity. Probing is painful, acute pain, the pain attack is long-
percussion is painless. Cold water causes lasting. Electroodontodiagnosis is 60 mi-
long-standing pain. What is your provisi- croampere. X-ray picture shows slight wi-
onal diagnosis? dening of periodontal fissure at the root
A. Exacerbation of chronic pulpitis apex of the 25 tooth. What is the most
B. Acute periodontitis probable diagnosis?
C. Exacerbation of chronic periodontitis A. Exacerbation of chronic pulpitis
D. Acute serous pulpitis B. Acute generalized pulpitis
E. Acute purulent pulpitis C. Acute purulent pulpitis
148. Parents of a 6-year-old child complain D. Acute purulent periodontitis
about pain in the child’s submandibular E. Exacerbation of chronic periodontitis
region on the left, body temperature ri- 151. A 48-year-old woman complains
se up to 37, 5o C. Objectively: the child’s about aching dull pain in the region of
face is asymmetric due to the infiltration the left TJM, that is getting worse duri-
of the submandibular region on the left. ng eating solid food. The pain appeared
The infiltration is soft and elastic, mobile, about 2,5 years ago. Objectively: mouth
2х2,5 cm large; its palpation is slightly pai- opening is limited, there is sideward devi-
nful, the skin is unchanged. The teeth are ation of jaw during mouth opening, TMJ
intact. Pharynx is hyperaemic. What is the is clicking. Examination of the oral cavi-
most likely diagnosis? ty revealed secondary partial adentia. X-
A. Acute serous nonodontogenic ray picture shows sclerosis of the cortical
submandibular lymphadenitis plate of articulat head and narrowing of
B. Acute serous odontogenic submandi- cartilage space. What is the most likely di-
bular lymphadenitis agnosis?
C. Acute purulent nonodontogenic A. Arthrosis of the TMJ
submandibular lymphadenitis B. Chronic arthritis of the TMJ
D. Acute purulent odontogenic submandi- C. Acute arthritis of the TMJ
bular lymphadenitis D. Painful dysfunction of the TMJ
E. Submandibular adenophlegmon E. Exacerbation of chronic arthritis of the
149. A dentist performing gum anestheti- TMJ
zation before the closed curettage had mi- 152. A patient with haemophilia consulted
stakenly used the 10% solution of silver a dental surgeon about decay of a right
nitrate instead of 10% lidocaine soluti- lower tooth. Objectively: the crown of the
on. The gums immediately became white, 46 tooth is completely decayed below the
swollen, painful. Which of these medicati- gum level. Percussion causes minor pain
ons is to be used for emergency care? reaction. It is planned to extract the 46
A. 3% solution of potassium iodide tooth. The extraction procedure should
B. 0,5% solution of acetic acid be performed in:
C. 0,1% solution of liquid ammonia
D. 2% solution of citric acid
E. 50% of ethyl alcohol
Kok 3 Stomatology 2011 22

A. Haematological department
B. Stomatological clinic A. Cavernous sinus thrombosis
C. Maxillofacial surgery department after B. Meningitis
the consultation with a haematologist C. Cerebral abscess
D. Stomatological clinic and following D. Sepsis
administration of haemostatic drugs E. Orbit phlegmon
E. Maxillofacial surgery department
157. A 37-year-old patient has been
153. A 22-year-old patient undergoes diagnosed with abscessed furuncle of
treatment for acute deep caries of the 26th the upper lip. After dissection of the
tooth. The pulp-capping material of choi- inflammation focus the exudate has been
ce is Ca(OH)2/salicylate cement "Life", sent for bacteriological analysis. Which
the constant filling material is ligh-cure causative agent for furuncle is most likely
condensable composite "Solitaire-2". The to be found?
pulp cap should be covered with:
A. Staphylococcus monoculture
A. Glass-ionomer cement B. Colibacillus monoculture
B. Adhesive composite system C. Proteus monoculture
C. Insulating varnish D. Staphylococcus and protozoa associati-
D. Phosphate cement on
E. Zinc-eugenol cement E. Streptococcus monoculture
154. A group of specialists conducts an 158. A 54-year-old patient complains of
epidemiological survey of certain age a tumor-like formation in the region of
groups of population aimed at evaluati- his lower lip which appeared 1-1,5 months
on of periodontal disease prevalence and ago. Objectively: there is a round tumour-
treatment needs. These rates are studied like formation up to 1cm in diameter
by means of the following index: on the red border of the lower lip on
the right. Roll-like edges of the formati-
A. CPITN (WHO index) on protrude above the level of the red
B. OHI-S (Green-Vermillion index) border. In the centre of the formation a
C. PDI (Ramfjord index) brown crust is visible. After its removal a
D. PI (Russel index) crateriform hole can be seen. There is no
E. PMA (Parma) bleeding or pain. Regional lymph nodes
are not enlarged. What is the most likely
155. A 27-year-old victim was delivered to diagnosis?
to the facial surgery department with a
cut wound of right cheek. The trauma is A. Keratoacanthoma
civilian, the patient got it in a fight. The B. Lower lip carcinoma
deadline for early surgical d-bridement C. Lower lip papilloma
without preliminary antibiotic injection is D. Lower lip fibroma
no more than: E. Verrucous carcinoma

A. 24 hours 159. A 26-year-old patient presented at


B. 48 hours a clinic for prosthetics. Objectively: the
C. 72 hours crown of the 16 tooth is destroyed by
D. 12 hours 1/3. It is planned to restore its anatomi-
E. 1 hour cal shape with a metal inlay. What is the
first stage of cavity preparation?
156. A 28-year-old patient has been
admitted to the oral and maxillofacial A. Cavity widening and necrosectomy
surgery for a carbuncle of face. Objecti- B. Making additional cavities
vely: the patient is in a critical conditi- C. Beveling
on, face, mucous membranes of lips and D. Completing the cavity floor
conjunctiva are edematous and cyanotic. E. Completing the cavity walls
Body temperature is 40o C. The patient
presents with delirium, hallucinations. 160. A 52-year-old female patient
The carbuncle is located on the upper lip, complains of severe paroxysmal pain in
the surrounding soft tissues are swollen. the region of the 34 tooth. Throughout
Palpebral fissures are closed due to the the year the 35, 36, 37 teeth were sequenti-
edema of eyelids. What is the most likely ally extracted because of pain complaints.
complication of carbuncle? Percussion of the 34 tooth causes acute
paroxysmal pain. The same pain arises
Kok 3 Stomatology 2011 23

when talking and washing the projection the most likely diagnosis?
area of the mental foramen. Clinical and
roentgenologic examination revealed no A. Chronic generalized I degree peri-
signs of bone tissue destruction. What is odontitis
the most likely diagnosis? B. Chronic generalized II degree peri-
odontitis
A. Neuralgia of the 3 branch of trigeminus C. Exacerbation of generalized I degree
B. 34 tooth pulpitis periodontitis
C. Facial nerve neuritis D. Exacerbation of generalized II degree
D. Ganglionitis of pterygopalatine gangli- periodontitis
on E. II degree periodontitis
E. Neuritis of the 3 branch of trigeminus
164. A 14-year-old child complains about
161. A 45-year-old female patient acute spontaneous spasmodic pain in an
complains of a slightly painful, tumour- upper jaw tooth on the right. The pain
like formation in the right parotid regi- has been lasting for 3 days, it is throbbing,
on that appeared two months ago for irradiating to the temple, getting worse at
no apparent reason. The patient notes night. Objectively: surface of the 15 tooth
weakness, occasional body temperature exhibits a carious cavity within parapulpar
rise up to 37, 2o C − 37, 6oC. Two weeks dentine. Dentine is softened, of greyish
ago she underwent the anti-inflammatory colour. Probing of the whole cavity floor is
therapy which did not have positive effect. painful, percussion of the 15 tooth is pai-
Objectively: the face is asymmetric due nless. What is the most likely diagnosis?
to a minor swelling in the right parotid
region. Palpation reveals a roundish dense A. Acute purulent pulpitis
and elastic formation with even contours, B. Acute diffuse pulpitis
it is slightly painful, not adherent to the C. Acute focal pulpitis
skin. Incisive canal exudes a small amount D. Acute periodontitis
of transparent saliva. What is the most li- E. Exacerbation of chronic periodontitis
kely diagnosis?
165. A 12-year-old boy complains of
A. Hertsenberg’s pseudoparotitis severe pain caused by cold food in the
B. Parotid atheroma 11 tooth. A day before he had a crown
C. Acute infectious parotitis break off of the 11 tooth. Examinati-
D. Epidemic parotitis on revealed an oblique fracture of the
E. Sjogren’s syndrome 11 tooth crown with the opening of the
tooth cavity, probing causes acute pain,
162. An 8,5-year-old child is apparently comparative percussion is painful, there
healthy. The child complains about pain is minor tooth mobility. What treatment
in an upper tooth on the left caused by should be administered?
traumatic injury sustained three hours
ago. Objectively: the crown part of the 21 A. Vital extirpation
tooth is destroyed by 1/2, the pulp is red B. Devital extirpation
and significantly exposed, probing causes C. Biologic method
acute pain and bleeding. Percussion of the D. Vital amputation
21 tooth is extremely painful. Choose the E. Devital amputation
most efficient treatment method of the 21
tooth: 166. A gravida (36 weeks) complains
of gingival haemorrhages and excessi-
A. Vital amputation ve plaque despite a thorough hygienic
B. Vital extirpation care of oral cavity. Objectively: gingi-
C. Devital amputation val papilla and marginal gingival edge
D. Devital extirpation are spongiose, bleeding when touched.
E. Bioassay technique Fedorov-Volodkina’s hygiene indexis 3,7
points. What kind of toothpaste should
163. A 23-year-old patient complai- be recommended to this patient after the
ns about root exposition, gingival professional oral hygiene?
hemorrhage during tooth brushing, gum
itch. Objectively: there is supragingival
and subgingival dental calculus. Gums are
hyperaemic, edematic, pouches are 3,5
mm deep. X-ray picture shows resorpti-
on of interalveolar septa by 1/3. What is
Kok 3 Stomatology 2011 24

A. Chlorhexidine-containing toothpaste the spherical surface. Close teeth contact


B. Toothpaste with mineralizing agents by movements of the lower jaw will be
C. Herbal toothpaste guaranteed by the following average radi-
D. Fluorine-containing toothpaste us of the spherical surface:
E. Gel toothpaste
A. 9 cm
167. A girl is 13 years old. She lives in an B. 5 cm
area where fluoride concentration in the C. 7 cm
drinking water is at the rate of 1,6 mg/l. D. 12 cm
Dental examination revealed some chalk- E. 18 cm
like spots on the vestibular surfaces of all
her teeth. The white coloration is more 171. A 13 year-old girl has presented
intense in the cenre and less on the peri- with gingival haemorrhage and mobi-
phery. There is light-brown pigmentation lity of the front teeth over the last
in the region of the central incisors along month. Objectively: gingival mucosa
the cutting edge. What is the most likely around the lower incisors and cani-
diagnosis? nes is edematous, hyperemic, bleeds on
touch. There is grade l tooth mobili-
A. Dental fluorosis ty, periodontal pockets are 3 mm deep.
B. Acute initial caries Orthopantomogram shows the resorption
C. Systemic enamel hypoplasia of interalveolar septa by 1/3 of their hei-
D. Stainton-Capdepont syndrome ght. There is crowding of the lower front
E. Amelogenesis imperfecta teeth. Hygiene index is 4,2. The patient
has to consult the following specialist:
168. An 8-year-old child is found to have
convex facil profile, forced closing of li- A. Orthodontist
ps, sagittal gap of 7 mm. Eschler-Bittner B. Neuropathologist
test produces some face improvement. C. Endocrinologist
This abnormality can be eliminated by D. Haematologist
means of Frankel type regulator. What is E. Gastroenterologist
the mechanism of action of this device?
172. A 13-year-old girl has been experi-
A. Normalization of labial, buccal and encing gingival hemorrhages and frontal
lingual pressure as well as of mandible teeth mobility throughout the last month.
position Objectively: gingival mucous membrane
B. Inhibition of maxilla growth in the in the region of lower incisors and canines
sagittal direction is edematic, hyperemic, it bleeds on touch.
C. Maxillary expansion by means of a These teeth present with I degree mobili-
screw ty, parodontal pouches are 3 mm deep.
D. Normalization of mandible position Orthopantomogram shows reduction of
and growth by means of intermandibular bone tissue of interalveolar septa by 1/3
traction of their height. Frontal teeth of the lower
E. Normalization of upper front teeth jaw are overcrowded. Hygiene index is
position by means of a vestibular bar 4,2. What local treatment should be provi-
ded in the first place?
169. A 40-year-old patient has been given
anesthesia, immediately after which he A. Professional hygiene
developed severe weakness, heart palpi- B. Anti-inflammatory therapy
tation. Objectively: the patient is consci- C. Orthodontic care
ous, inert, skin is markedly hyperemic, D. Physical therapy
there is acute headache, AP is 180/100 mm E. Antibacterial therapy
Hg, heart sounds are muffled. What state
is it? 173. A 65-year-old patient complains of a
break off of three front artificial porcelain
A. Hypertensic crisis teeth of the removable denture fabri-
B. Cardiogenic form of anaphylactic shock cated 2 months ago. In what way are the
C. An attack of stenocardia porcelain teeth linked with the plastic
D. Cardial pain denture base?
E. Myocardial infarction
170. A completely edentulous 70-year-old
patient has ordered complete removable
dentures. Artificial teeth are placed upon
Kok 3 Stomatology 2011 25

A. By means of crampon-fixing wire ven an anaesthetic of articaine group


elements ”Ultracain DS”. The orthopaedist chose
B. By means of glue this anaesthetic because:
C. Due to physicochemical connection
D. By means of clasps A. It is low-toxic, is not capable of crossing
E. By means of diatoric cavities the hemato-placental barrier
B. It is low-toxic
174. A 47-year-old patient complains of C. It reduces the volume of uteroplacental
persistent ache in the 27 tooth, that circulation
gets worse when biting. Objectively: skin D. It is most commonly used in the
colour is normal, the face is symmetrical, stomatological practice
the mouth opens without any difficulties, E. It doesn’t contain sulphites
alveolar mucosa at the level of 27 tooth is
edematous and hyperemic. The 27 tooth 178. A patient consulted a dental surgeon
presents a deep carious cavity communi- about extraction of the roots of the
cating with the pulp chamber. Percussion 36 tooth. The tooth decayed long ago.
of the 27 tooth causes acute pain. What is Objectively: the crown of the 36 is
your provisional diagnosis? destroyed by 2/3, percussion is painless,
mucosa around the 36 tooth exhibits no
A. Exacerbation of acute periodontitis of evident pathological changes. X-ray pi-
the 27 tooth cture of the alveolar process shows a
B. Acute purulent maxillary periostitis in roundish well-defined radiolucency near
the region of the 27 tooth the apex of the medial root. What is the
C. Acute purulent periodontitis of the 27 most likely diagnosis?
tooh
D. Acute generalized purulent pulpitis of A. Chronic granulomatous periodontitis
the 27 tooth B. Exacerbation of chronic periodontitis
E. Chronic odontogenic left-sided maxi- C. Chronic granulating periodontitis
llary sinusitis D. Acute purulent periodontitis
E. Chronic fibrous periodontitis
175. A patient complains of "clicking"in
the TMJ. On auscultation, the sounds can 179. A 52-year-old patient complai-
be heard in the middle and at the begi- ns about teeth mobility, gingival
nning of mouth opening. These signs give haemorrhage. The 44, 45, 34, 35 teeth
evidence of: present with I degree mobility, the 43,
42, 41, 31, 32, 33 teeth present with II
A. Reduction in occlusal vertical dimensi- degree of teeth mobility. There is gingival
on edema, parodontal pouches are 5-6 mm
B. Increase in occlusal vertical dimension deep. X-ray picture shows destruction of
C. TMJ dislocation bone tissue by 1/2 of root length. A dentist
D. Joint capsule distension made a diagnosis: generalized periodonti-
E. Joint subluxation tis of median severity. What orthopaedic
construction is the most efficient in this
176. A 32-year-old patient presented to a situation?
clinic for takingof an upper jaw impressi-
on. During this procedure she choked wi- A. Elbrecht splint
th a piece of impression material, there B. Plastic mouthguard
was risk of asphyxia. What are the first- C. Murray’s splint
priority actions within emergency care? D. Mamlock splint
E. Temporary Novotny splint
A. Abrupt push-like compressions of the
lower thorax 180. A victim got a face and temporal regi-
B. Artificial ventilation of lungs on trauma. A doctor made a diagnosis:
C. Plaster removal from the airways fracture of malar arch. What processes of
D. Closed-chest cardiac massage cranial bones are injured?
E. Tracheotomy
177. A 32-year-old 3-month pregnant pati-
ent presented to the prosthodontics cli-
nic for the prosthetics. Objectively: the 16
tooth is missing. It is planned to fabri-
cate a dental bridge supported by the
15 and 17 teeth. The patient was gi-
Kok 3 Stomatology 2011 26

A. Temporal process of zygomatic bone A. Initial caries


and zygomatic process of temporal bone B. Fluorosis
B. Zygomatic process of frontal bone and C. Local hypoplasia
zygomatic process of temporal bone D. Enamel necrosis
C. Temporal process of zygomatic bone E. Amelogenesis imperfecta
and zygomatic process of frontal bone
D. Zygomatic process of maxilla and 185. A 27-year-old patient has been
zygomatic process of temporal bone referred for endodontic treatment on the
E. Zygomatic process of maxilla and 45 tooth. Objectively: the tooth crown is
zygomatic process of frontal bone decayed, the lateral surface of tongue and
cheek mucosa being in direct contact wi-
181. A 7-year-old child sustained a sport th the 45 tooth are covered with grayi-
injury. He complains about pain in the sh plaques of macerated epithelium that
region of mandible, inability to join his are somewhat raised above the surface of
teeth. Pressing against the chin causes the mucous membrane. Uvula and palati-
intensification of pain in the region of ne arches are of dark red colour, and
mandible on the right. The child is di- the hard palate has papules with a red
agnosed with a fracture of mandible body rim and covered with grayish epithelium.
without fragment displacement. What Submandibular, cervical, supraclavicular,
therapeutic tactics should be chosen in subclavicular lymph nodes are enlarged,
order to prevent the displacement of bone painless. What is your provisional di-
fragments in the transitional dentition? agnosis?
A. Fragment immobilization by means of A. Secondary syphilis
an intraoral splint B. Chronic recurrent aphthous stomatitis
B. Administration of anti-inflammatory C. Lupus erythematosus, plaque stage
therapy D. White sponge nevus
C. Application of sling bandage E. Lichen ruber planus
D. Fixation by means of Tigerstedt splints
E. Osteosynthesis 186. Examination of a 6-year-old boy
revealed enlarged lymph nodes in both
182. At the end of the reception hours submandibular and cervical regions.
a dental surgeon has to fill in the Objectively: the 75, 84 and 85 teeth
stomatological forms. Name the form of are decayed, there are presentations of
everyday reporting on children dental commissural cheilitis. According to the
reception: boy’s mother, he has been quickly getti-
ng tired, sweating from the least physi-
A. Form 39 cal exercise, complaining about weakness
B. Form 9 throughout the last 2-3 months. He also
C. Form 19 gave up training in a sports class. What
D. Form 29 plan of additional examination should be
E. Form 49 adopted?
183. A patient has orthopaedic indications A. Complete blood count, haematologist
for the 37 tooth extraction. What kind of consultation
anaesthesia should be given to this patient B. Biochemical blood test, endocrinologist
before the extraction? consultation
C. Puncture biopsy, oncologist consultation
A. Mandibular torus anaesthesia D. Magnetic resonance tomography,
B. Infraorbital + buccal anaesthesia immunologist consultation
C. Mental + buccal anaesthesia E. Pulmonary roentgenography,
D. Tuberal + buccal anaesthesia pulmonologist consultation
E. Mandibular
187. A child was born with schistasis of
184. On removing a dental plaque, a 19- alveolar process, hard and soft palate. The
year-old patient is found to have two whi- optimal way to feed the child before the
te patches in the precervical region of the surgery will be through:
11 and 21 teeth. After drying the patches
increase in size. What is the most likely
diagnosis?
Kok 3 Stomatology 2011 27

A. Obturator 190. A 42-year-old patient complains of


B. Enteric feeding tube a cavity in the 26 tooth. Objectively: the
C. Baby bottle nipple 26 tooth exhibits a deep carious cavity,
D. Spoon dentine is dense, pigmented, there is pain
E. - reaction to a cold stimulus, percussion and
probing are painless. What is your provi-
188. A 32-year-old patient complains sional diagnosis?
about pain and swelling in the region of
his mandible and left cheek. He has been A. Chronic deep caries
suffering from this for 2 days. Objecti- B. Acute deep caries
vely: his general condition is satisfactory. C. Chronic fibrous pulpitis
Body temperature is 37, 5oC. There is an D. Chronic median caries
edema of soft tissues in the left buccal E. Acute median caries
region. Submandibular lymph nodes are
bilaterally enlarged, painful on palpati- 191. A 12-year-old boy repeatedly visi-
on. Crown of the 37 tooth is partially ts a dentist to finish treatment of the
decayed, the tooth is immobile, percussi- 36 tooth for the exacerbation of chronic
on is slightly painful. Mucogingival fold periodontitis. The complaints are absent.
in the region of the 37 tooth is edematic, Objectively: occlusive dressing on the 36
hyperaemic, flattened, its palpation causes tooth is preserved, tooth percussion is pai-
acute pain. What is the most likely di- nlessl, mucous membrane of the 36 has
agnosis? no pathological changes, is painless on
palpation. What material should be used
A. Acute purulent periostitis of mandible for root canal filling in this case?
beginning from the 37 tooth
B. Acute odontogenous osteomyelitis of A. Sealer with a gutta-percha point
mandible B. Zinc-eugenol paste
C. Chronic productive periostitis of mandi- C. Resorcinol-formalin paste
ble beginning from the 37 tooth D. Calcium-containing paste
D. Chronic odontogenous osteomyelitis of E. Phosphate cement
mandible
E. Abscess of alveololingual groove on the 192. A 63-year-old patient requires a full-
left arch denture for the upper jaw. Objecti-
vely: the 18, 17, 16, 15, 25, 26, 27 teeth
189. A 7-year-old child presents withf faci- are missing, the gag reflex is not marked,
al asymmetry, severe pain in the lower the torus is of average size. In order to
jaw on the left, body temperature rise avoid possible inconvenience while using
up to 39, 0oC. Objectively: the face is the denture, the arch should be located at
asymmetric due to the massive swelling the following distance from the palatine
of soft tissues in the mandibular region on fossae:
the left. The skin over the infiltration is
hyperemic and cannot be plicated. Mouth A. 10-12 mm
opening is limited, painful. The crown of B. 4-8 mm
the 75 tooth is destroyed by 2/3, the tooth C. 23-25 mm
exhibits the grade II mobility. Percussion D. 2-3 mm
of the 36 tooth is painful, the 75 tooth - E. 16-22 mm
painless. Edema and fluctuation of soft ti-
ssues on the buccal and lingual sides of 193. A 3-year-old patient presented to
these teeth are present. What measures the prosthetic dentistry clinic for the
are to be taken in the first place? traumatic loss of an upper central incisor.
The patient hasn’t undergone prostheti-
A. To extract the 75 tooth, to lance the cs before. He has a history of asthma.
inflammation focus on both sides What are the first-priority measures of
B. To extract the 75 tooth, to lance the bronchospasm prevention?
inflammation focus on the buccal side
C. To extract the 75, 36 teeth, to lance the A. Tests for plastic and metal allergy
inflammation focus on the buccal side B. Preparation following the anaestheti-
D. To lance the inflammation focus on the zation
lingual and buccal sides C. Teeth treatment with a remineralizing
E. To lance the inflammation focus in the solution
submandibular region D. Fabrication of a temporary construction
E. Administration of sedative drugs
Kok 3 Stomatology 2011 28

194. A 47-year-old patient complains of articular head. Mandibular movements


pain in the lateral part of mandible to the were possible but restricted. What is the
left of the toungue edge, which is caused most likely diagnosis?
by tongue movements. Two days ago the
patient was provided with claspps’ fixation A. Unilateral dislocation of the left TMJ
a partial removable clasp denture for the B. Unilateral dislocation of the right TMJ
47, 42, 33 teeth. Retroalveolar region of C. Bilateral dislocation of the TMJ
the mucous membraneis exhibits a locali- D. Acute arthritis of the left TMJ
zed hyperemic area matching the edge of E. Acute arthritis of the right TMJ
the denture. Which muscle contractions
demarcate the boundary in this region? 198. A 68-year-old patient with
compensated insulin-dependent diabetes
A. Mandibulohyoid mellitus had to spend 8 hours at a hospi-
B. Geniohyoid tal. Within this time he didn’t eat and was
C. Hyoglossal nervous. The patient became aggressi-
D. Medial wing muscle ve, pale and sweaty, then he lost consci-
E. Masticatory ousness. What is the most likely di-
agnosis?
195. A 40-year-old female patient
complains about crepitation and pain in A. Hypoglycemic coma
the region of TMJ projection on the ri- B. Anaphylactic shock
ght, that arise in the final stage of mouth C. Hypertensic crisis
opening. Articular sounds turned up after D. Acute respiratory failure
dental prosthetics. Mouth opening is unli- E. Collapse
mited, mouth is opened in a straight path,
amplitude of mouth opening is 5 cm. What 199. Rising from a chair after a dental
is the most likely diagnosis? procedure, a 60-year-old patient felt
retrosternal pain radiating to the left arm,
A. Lower jaw subluxation nausea, weakness. Objectively: the patient
B. Lower jaw dislocation is pale, his face is sweaty. AP is 90/60 mm
C. Anterior dislocation of articular disk Hg, pulse is arrhythmic, of poor volume,
D. Posterior dislocation of articular disk 100/min. Heart sounds are muffled, vesi-
E. Meniscus tresis cular breathing is present. What is your
provisional diagnosis?
196. A 40-year-old patient needs the oral
cavity sanation, which involves extraction A. An attack of coronary artery disease
of the upper central incisors. What nerve B. Myocardial infarction
structure will be blocked as a result of inci- C. Hypertensic crisis
sor anaesthesia? D. Stenocardia
E. Pulmonary artery thrombosis
A. Nasopalatine nerve
B. Anterior superior alveolar branch of 200. During the cystectomy involving
the infraorbital nerve resection of the root apices of the 11,
C. Median superior alveolar branch of the 21 and 22 teeth a 45-year-old patient
infraorbital nerve developed skin pallor and cyanosis, a cold
D. Posterior superior alveolar branch of sweat, a sharp drop in blood pressure
the infraorbital nerve (70/40 mmHg). The patient is conscious
E. Dental nerve plexus but complains of weakness. What state
has been developed?
197. A maxillofacial surgery department
admitted a patient with a half-open A. Collapse
mouth, his chin was put forward and devi- B. Haematoma
ated to the right. On palpation, the left C. Epileptic seizure
mandibular fossa was found to be empty. D. Quincke’s edema
Internal palpation failed to detect the left E. Syncope
Krok 2 Stomatology 2012 1

1. A 49-year-old patient complains A. At each station of evacuation


about pain in the oral cavity induced by B. At the battalion aid station
eating. He suffers from CHD. Objecti- C. At the regimental aid station
ve examination revealed dyspnea, limb D. At the station of the separate medical
edema. Oral cavity isn’t sanitated. On the battalion
mucous membrane on the right, there is E. Only when the dressing is unsatisfactory
an ulcer with irregular edges covered with
greyish-white necrotic deposit with low- 5. A 47-year-old female patient complai-
grade inflammation around it. There is ns of inability to eat hot and cold food,
also halitosis. What is the most probable as well as of intense pain caused by sour,
diagnosis? sweet and salty food. Objectively: there is
a slight loss of enamel on the molars and
A. Trophic ulcer incisors. Probing and cold test cause acute
B. Traumatic ulcer pain. What is the most likely diagnosis?
C. Tuberculous ulcer
D. Cancerous ulcer A. Hyperesthesia of dental hard tissues
E. Ulcero-necrotic stomatitis B. Enamel erosion
C. Enamel necrosis
2. A 48-year-old patient complains about D. Enamel hypoplasia
itching gums. Objectively: gums are E. Pathological abrasion of dental hard
dense, of normal colour; cold water tissues
causes short-term toothache. X-ray pi-
cture shows osteosclerosis of interalveolar 6. After removal of dental plaque an 18-
septa (fine granular bone texture), hei- year-old patient underwent preventive
ght of interalveolar septa and integrity examination. It revealed painless chalky
of compact substance on their tops is spots in the precervical region on the
unchanged. Which diagnosis is the most vestibular surface of the 22 and 41 teeth.
likely? Result of enamel resistance test is 7. What
morphological changes are typical for this
A. Initial parodontosis disease?
B. Atrophic gingivitis
C. Initial periodontitis A. Subsurface enamel demineralization
D. I degree periodontitis B. Changes in the mantle dentine
E. I degree parodontosis C. Damage of dentinoenamel junction
D. Superficial enamel demineralization
3. A 34-year-old patient had got in a E. Degeneratic changes of odontoblasts
car accident. The patient stayed consci-
ous. He complains of headache, dizzi- 7. A 49-year-old patient complains of
ness, general weakness, nausea. 12 hours constant mandibular pain irradiating
after the injury the patient got "raccoon to the ear, soft tissue edema, body
eyes"(periorbital haematomas) within the temperature rise up to 39, 0oC. Objecti-
circular muscles of eye. What fracture vely: there is an edema in the submandi-
does the victim have? bular region, the skin has not changed in
colour. Alveolar mucosa around the 45,
A. Fracture of skull base 46 teeth is hyperemic and edematic on the
B. Le Fort II fracture of maxilla oral and vestibular side. Crowns of the
C. Fracture of nasal bones 45, 46 teeth are completely destroyed, the
D. Bilateral fracture of zygomatic bones teeth are mobile. Vincent’s symptom is
E. Fracture of frontal bones present. What is the most likely diagnosis?

4. A victim got a perforating wound A. Acute mandibular osteomyelitis


of the left cheek contaminated with B. Abscess of submandibular region
radioactive materials on the battlefield. C. Exacerbation of chronic mandibular
The wound was dressed with aseptic osteomyelitis
bandage from the individual first-aid D. Acute serous periostitis of mandible
pack and processed with anaesthetic and E. Acute purulent periostitis of mandible
antibacterial medicaments. The dressing
should be changed at the following stati- 8. An 8-year-old boy complains of havi-
on of medical evacuation: ng toothache during eating. Objectively:
approximal surface of the 55 tooth has
a deep carious cavity communicating wi-
th the tooth cavity. Probing causes acute
pain, there is bleeding, percussion is pai-
Krok 2 Stomatology 2012 2

nless. What is the most likely diagnosis? sublingual torus, hyperemia, soft tissue
edema, acute pain during palpation. The
A. Chronic fibrous pulpitis duct of the right submandubular salivary
B. Chronic hypertrophic pulpitis gland excretes turbid saliva mixed with
C. Chronic gangrenous pulpitis pus. What is the most likely diagnosis?
D. Chronic granulating periodontitis
E. Chronic fibrous periodontitis A. Exacerbation of salivolithiasis
B. Acute purulent lymphadenitis of
9. A 36-year-old patient complains of submaxillary area
pain under the dental bridge. After its C. Adenophlegmon of submaxillary area
removal the patient has been found to D. Abscess of maxillolingual groove
have an ulcer 0,3x0,5 cm large on the E. Retention cyst of sublingual salivary
alveolar process. The ulcer is slightly pai- gland
nful and soft, the surrounding mucosa is
hyperaemic, submandibular lymph nodes 13. A 4-year-old girl presents with body
are not enlarged. What is a provisional di- temperature rise, aggravation of general
agnosis? condition. The symptoms has been
observed for 3 days. Objectively: general
A. Decubital ulcer condition is grave, body temperature is
B. Trophic ulcer 38, 6oC, the girl is anxious and pale. She
C. Sutton aphtha presents also with halitosis, hyperaemia
D. Cancerous ulcer and edema of gingival mucous membrane
E. Tuberculous ulcer in the region of the 83, 84, 85 teeth on
both sides from the alveolar process.
10. A 45-year-old female patient complai- The mentioned teeth are mobile, their
ns of worsened fixation and frequent percussion causes acute pain; the 84 tooth
breakages of her partial removable lami- is filled. What is the most likely diagnosis?
nar denture for the lower jaw, which
she has used for 5 years. Objectively: A. Acute odontogenous mandibular
alveolar process in edentulous areas is osteomyelitis beginning from the 84 tooth
considerably atrophied, the denture keeps B. Acute sialoadenitis of submandibular
balance. What is the most likely cause of salivary gland
worsened fixation and frequent breakages C. Exacerbation of chronic periodontitis of
of the denture in this case? the 84 tooth
D. Suppuration of the radiculodental
A. Atrophy of the alveolar process mandibular cyst beginning from the 84
B. Wear of artificial teeth tooth
C. Improper keeping of the denture E. Acute odontogenous mandibular peri-
D. Using the denture during sleep ostitis beginning from the 84 tooth
E. Eating solid food
14. A 48-year-old female patient had
11. A 42-year-old female patient complai- been delivered to a hospital with a cheek
ns of tooth mobility, difficult masticati- wound and severe headache, nausea, di-
on. Objectively: face configuration is zziness. It is known from the history that
unchanged. The 35, 36, 38, 44, 46, 48 teeth the patient got a trauma as a result of
are missing. The 31, 32, 41, 42 teeth have a fall. After examination she was di-
1-2 grade mobility. What is the most effici- agnosed with a contused lacerated wound
ent denture construction in this case? of cheek, closed craniocerebral injury,
A. Whole-piece splint with vestibular brain concussion. This trauma can be
clowlike hooks qualified as:
B. Whole-piece splint with Roach clasps A. Concominant trauma
and a multijoint clasp B. Isolated trauma
C. Mamlock splint C. Combined trauma
D. Removable plastic splint D. Polytrauma
E. Interdental splint E. Single
12. A 42-year-old patient complains of 15. A 26-year-old patient needs prostheti-
pain in the submaxillary and sublingual cs. Objectively: crown of the 16 tooth is
areas that is getting worse during eating, destroyed by 1/3. It is planned to restore
body temperature rise up to 37, 6oC. He its anatomical shape with a metal inlay.
has been suffering from this for 2 months. What is the first stage of making a cavity
Objectively: infiltration along the right for the inlay?
Krok 2 Stomatology 2012 3

20. A 24-year-old male complains of


A. Removal of affected tissues pain, bad breath, body temperature ri-
B. Making additional cavities se up to 38, 0oC. Objectively: the pati-
C. Making a bevel ent is pale, adynamic. Regional lymph
D. Making the cavity floor nodes are enlarged and painful. Gums are
E. Making the cavity walls edematic, hyperaemic, ulcerated, covered
with necrotic plaque. There is an excessive
16. An 8-year-old boy complains of buildup of calculus. What additional study
improper arrangement of teeth. Exami- should be done in the first place?
nation at an orthodontic clinic revealed
broad, tight, low-attached upper lip A. Complete blood count
frenulum. Broad frenulum and its low B. Blood sugar test
attachment may cause: C. Microscopy of gingival plaque
D. Test for HIV infection
A. Diastema E. X-ray of jaws
B. Shortening of the upper dentition
C. Elongation of the upper dentition 21. A 46-year-old patient complains of
D. Narrowing of the upper dentition spontaneous pain in the 36 tooth. Its
E. Protrusion of the upper front teeth crown is decayed. The patient presented
with edema of soft tissues adjacent to the
17. During extraction of the 47 tooth its mandible. Body temperature rose up to
distal root was broken halfway along its 39, 0oC, the patient has chill. Examination
length. What tool should be chosen for of the oral cavity revealed a pronounced
extraction of the residual root fragments? edema of the left mandibular alveolar
A. Left angled elevator process on the vestibular and oral side
B. Broad-beaked forceps (acute periostitis); the 35, 36, 37 teeth
C. Close-beaked forceps are mobile, subgingival pockets contain
D. Right angled elevator purulent exudate; their percussion causes
E. Straight elevator acute pain. Positive Vincent’s symptom
is present. What is the presumptive di-
18. A 22-year-old student complains of gi- agnosis?
ngival haemorrhage during tooth brushi-
ng. Objectively: hyperaemia and edema of A. Acute odontogenic osteomyelitis of
marginal gingiva of the front teeth on both mandible from the 36 tooth
jaws, periodontal pockets are absent. X- B. Exacerbation of chronic periodontitis
ray reveals no pathological changes. What from the 36 tooth
is the most likely diagnosis? C. Acute purulent periostitis of mandible
from the 36 tooth
A. Chronic catarrhal gingivitis D. Exacerbation of generalized periodonti-
B. Localized periodontitis tis
C. Ulcerative gingivitis E. Exacerbation of chronic mandibular
D. Hypertrophic gingivitis osteomyelitis
E. Generalized periodontitis
22. An 11-year-old girl complains about gi-
19. A 37-year-old male patient complai- ngival haemorrhage during tooth brushi-
ns about pain of the 46 tooth during ng and eating. She has been sufferi-
food intake, especially hot food, offensive ng from this for a year. Gum of both
breath when he sucks his tooth. Objecti- upper and lower jaws is edematic and
vely: the face is symmetrical, masticatory congestively hyperemic. Hygienic state of
surface of the 48 tooth has a deep cari- oral cavity is unsatisfactory. Bite is edge-
ous cavity communicating with the dental to-edge. Roentgenological examination
cavity. X-ray picture shows widening of of periodontium revealed no pathologi-
periodontal fissure at the root apex of cal changes. What is the provisional di-
the 46 tooth. What is the most likely di- agnosis?
agnosis?
A. Chronic catarrhal gingivitis
A. Chronic gangrenous pulpitis B. Acute catarrhal gingivitis
B. Exacerbation of chronic periodontitis C. Generalized periodontitis
C. Exacerbation of chronic pulpitis D. Localized periodontitis
D. Chronic fibrous periodontitis E. Hypertrophic gingivitis
E. Chronic fibrous pulpitis
23. Preventive examination of a 4,5-year-
Krok 2 Stomatology 2012 4

old child revealed some hidden cavities should be taken for asphyxia suppression?
on the contact surfaces of the 54 and 55
teeth. After removal of the overhanging A. Pul the tongue forwards and sew it
edges of the enamel the softened dentin through
could be easily removed within the mantle B. Intubation of trachea
dentin. Select the optimal material for a C. Tracheostoma establishment
permanent filling: D. Artificial airway
E. Artificial pulmonary ventilation
A. Compomer material
B. Composite material 28. A week ago an 18-year-old girl
C. Silicate cement complained of pain in the 22 tooth whi-
D. Silicophosphate cement ch was treated and filled several years
E. Polycarboxylate cement ago. Over the past two days the pain got
worse. Objectively: the 22 tooth is filled,
24. A 54-year-old woman complains about percussion is painful, mucous membrane
wear of the 35 and 36 teeth and pain is hyperaemic and edematic. Spot-film
caused by thermal and chemical stimuli. roentgenograph of the 22 tooth shows
Objectively: crowns of the 35 and 36 teeth an ill-defined pariapical bone rarefaction
are worn by 1/3 of their height (horizontal 0,4x0,5 cm large. What is the most likely
type), the 24, 25 26 teeth have fullcast diagnosis?
crowns. What crowns should be made for
the 35 and 36 teeth? A. Exacerbation of chronic periodontitis
of the 22 tooth
A. Fullcast B. Suppuration of the radicular cyst
B. Combined Belkin’s C. Acute odontogenic osteomyelitis
C. Metal stamped D. Acute purulent periodontitis of the 22
D. Plastic tooth
E. Metal-ceramic E. Acute maxillary periostitis
25. An 18-year-old patient complains 29. A 22-year-old patient complains of a
about body temperature rise, weakness, painful swelling in the right parotid gland.
pain induced by eating and deglutition. A week earlier the patient got a cheek
Objectively: mucous membrane of the abrasion which healed under the purulent
oral cavity is erythematic with multi- crust. Over the past two days the patient
ple petechia. Pharynx is hyperaemic. had observed progressing pain and fever
Regional lymph nodes are enlarged, up to 38, 6o C. Objectively: there is a soft
mobile, painless. In blood: leukocytosis, tissue edema in the right parotid region,
monocytosis, atypic mononuclear cells, the skin is slightly strained but has not
ESR is 30 mm/h. What is the leading changed in colour. There is a dense painful
factor of disease development? infiltration 2,5x3,5 cm large, the skin over
it exhibits limited mobility. The mouth can
A. Viral infection be fully opened, the mucous membrane
B. Bacterial infection around the orifice of the salivary duct is
C. Autoimmune disorders unchanged, the saliva is transparent. What
D. Immediate allergy is the most likely diagnosis?
E. Delayed allergy
A. Acute lymphadenitis
26. An 18-year-old student needs B. Exacerbation of chronic parotitis
prosthetic metal-ceramic denture for the C. Abscess of the parotid-masticatory
11, 21 teeth. There are no contraindicati- region
ons for the use of such construction. What D. Acute non-epidemic parotitis
is the most appropriate material for taki- E. Epidemic parotitis
ng impressions?
30. A 33-year-old female patient has been
A. Sielast admitted to the maxillofacial department
B. Stomalgin with complaints of pain and edema of
C. Orthocor the right submandibular region, body
D. Stens temperature rise up to 39, 5oC. Objecti-
E. Repin vely: the patient has asymmetric face
because of soft tissue edema of the right
27. A soldier was delivered to the regiment submandibular region, palpation reveals
medical station with dislocation asphyxia a dense infiltration, the skin over it is
caused by a gunshot wound. What actions hyperemic, does not make a fold. The 46
Krok 2 Stomatology 2012 5

tooth has a deep carious cavity. What is A. Cavernous hemangioma


the most likely diagnosis? B. Branched hemangioma
C. Capillary hemangioma
A. Submandibular phlegmon on the right D. Lymphangioma
B. Acute submandibular sialoadenitis E. Lentigo
C. Acute purulent periostitis of mandible
D. Acute purulent submandibular 35. A 3-year-old girl complains of pain
lymphadenitis and swelling in the region of the decayed
E. Acute right-sided osteomyelitis of 51, 52 teeth, body temperature rise up
mandible to 37, 5 − 37, 9oC. Objectively: the face
is asymmetric because of a swelling in
31. Analysis of the contents of periodontal the upper lip region and right infraorbi-
pockets revealed a significant contami- tal region. The crown of the 51 tooth is
nation with Candida yeast fungi. Which completely decayed. Mucous membrane
of the following drugs should be used for in the region of the 52, 51, 1 teeth is
instillations? edematic, mucogingival fold is smoothed,
palpation provokes pain, mobility of I-II
A. Clotrimazole grade of the 51, 52 teeth is also present.
B. Trichopol What is the most likely diagnosis?
C. Dioxydin
D. Diclofenac sodium A. Acute purulent odontogenic maxillary
E. Tinidazole periostitis
B. Acute albuminous odontogenic maxi-
32. A 25-year-old patient consulted llary periostitis
a doctor about massive gingival C. Acute odontogenic maxillary
haemorrhages, dry mouth, mobility and osteomyelitis
shifting of teeth, purulent discharges from D. Odontogenic abscess of infraorbital
the gums, bad breath. According to the region
patient, these presentations turned up E. Exacerbation of chronic periodontitis
about 2 months ago. Before the diagnosis of the 51 tooth
can be made, the following tests should be
done in the first place: 36. A 31-year-old male patient complai-
ns of dryness and burning of tongue
A. Blood sugar test back that appeared for about a week
B. Immunological studies ago and get worse when he eats irritati-
C. Allergological tests ng food. The patient has a history of
D. Acute-phase reactants tests recent pneumonia. He had been treated
E. Serologic studies in the in-patient hospital for 2 weeks, the
treatment program included antibiotics.
33. A 59-year-old patient has a bilateral Now he doesn’t take any drugs. Objecti-
fracture of mandible in the region of the vely: mucous membrane of the oral cavity
44, 34 teeth, other masticatory teeth are is hyperemic, dry, glossy. Tongue back and
missing, toothless fragments are not di- palate have greyish-white plicae that can
splaced but mobile. Which orthosis should be easily removed. Threads of saliva trail
be used for the immobilization of bone behind the spatula. What is the most likely
fragments? diagnosis?
A. Vankevich splint with orthodontic A. Acute pseudomembranous candidiasis
elastics B. Chronic hyperplastic candidiasis
B. Rudko’s apparatus C. Acute atrophic candidiasis
C. Limberg’s splint D. Medicamental stomatitis
D. Petrosov’s apparatus E. Chronic atrophic candidiasis
E. Zbarzh apparatus
37. During the examinations and everyday
34. A 20-year-old patient has asymmetric orthopaedic manipulations a dentist uses
face due to an upper lip edema on the left. a dental mirror. What is the way of dental
The skin over it is bluish, there are positi- mirrors sterilization?
ve symptoms of "compression"and "filling
in". What is the most likely diagnosis?
Krok 2 Stomatology 2012 6

A. In the triple solution for 30 minutes dentures for the upper and lower jaw
B. In the 0,5% ethyl chloride solution for for 12 years. He is smoker. Objecti-
20 minutes vely: the right retromolar region exhi-
C. In the 6% hydrogen peroxide solution bits a 1,5х1,2 cm large proliferation of
for 6 hours mucous membrane in form of cauliflower,
D. In the dry heat sterilizer at 180oC for 10 here and there there are dense fissure-
minutes like ulcers. The surrounding mucous
E. In the 0,01% chloramine solution for 10 membrane is cyanotic, infiltrated. When
minutes the teeth are closed, the mentioned
formation contacts with the posterior
38. A 65-year-old patient complains about edges of the dentures. What is the provisi-
partially missing teeth on his upper jaw, onal diagnosis:
difficult mastication, rhinolalia. Objecti-
vely: the 18, 16, 15, 11, 23, 28, 35, 38, 48, A. Cancer of the mucous membrane
47 teeth are missing; there is postoperati- B. Leukoplakia
ve midline defect of hard palate. It was C. Decubital ulcer
decided to make a clasp dental prosthesis D. Hypertrophic gingivitis
with obturating part. The obturating part E. Papillomatosis
should be placed on the following element
of the clasp dental prosthesis: 42. A 40-year-old patient complains of
discoloration of the vermilion border of
A. On the arch the lower lip that he noticed about 4
B. On the saddle months ago. Objectively: in the center of
C. On the artificial teeth the vermilion border of the lower lip there
D. On the base is an irregular homogeneous grayish-
E. On the clasps white area 1x1,5 cm large that doesn’t rise
above the vermilion border and has di-
39. Examination of an 11-year-old boy stinct outlines. Palpation of this area is
revealed thickened, somewhat cyanotic, painless, the surrounding tissues are not
dense gingival margin overlapping the changed. The film cannot be removed
crowns of all teeth by 1/2 of their height. when scraped. The 31, 32, 41, 42 teeth are
Fedorov-Volodkina oral hygiene index is missing. What is the most likely diagnosis?
2,6, PMA index is 20%. X-ray picture
shows no pathological changes of peri- A. Leukoplakia
odontium. The child has a 2-year history B. Lupus erythematosus
of neuropsychiatric treatment for epilepsy. C. Lichen ruber planus
Make a provisional diagnosis: D. Candidous cheilitis
E. Premalignant circumscribed
A. Chronic hypertrophic gingivitis hyperkeratosis
B. Chronic catarrhal gingivitis
C. Localized periodontitis 43. A 35-year-old patient complains
D. Acute catarrhal gingivitis about itch, burning and edema of lips.
E. Generalized periodontitis He has been suffering from this for a
week. Objectively: reddening of vermi-
40. A 39-year-old patient complains of a lion border and skin, especially in the
cosmetic defect, hypersensitivity of the region of mouth corners, there are also
12, 11, 22 teeth. Objectively: vestibular vesicles, crusts, small cracks along wi-
surface of these teeth has oval defects wi- th erythematous affection of vermilion
th smooth shiny walls. Probing causes no border. What is the most likely diagnosis?
pain, there is a pain reaction to cold sti-
muli. The defects can be stained with 5% A. Acute eczematous cheilitis
tincture of iodine. What is the most likely B. Multiform exudative erythema
diagnosis? C. Acute herpetic cheilitis
D. Allergic contact cheilitis
A. Enamel erosion E. Exudative form of exfoliative cheilitis
B. Superficial caries
C. Wedge-shaped defect 44. A 45-year-old man complains about li-
D. Systemic hypoplasia quid outpouring from his nose, inability
E. Fluorosis, erosive form to blow his nose, inflated cheeks. Objecti-
vely: there is a perforating defect (1х1,5
41. A 70-year-old patient complains of
cm) of alveolar process at a level of the
pain in the lower jaw region on the right.
He has been using complete removable extracted 26th tooth in the lateral part
Krok 2 Stomatology 2012 7

of his upper jaw. Air inhalation through of root of the 34 tooth. What is the most
the nose with held nostrils is accompani- likely diagnosis?
ed by generation of bubbles in the area
of perforation. What denture constructi- A. Exacerbation of chronic granulating
on should be recommended? periodontitis
B. Acute purulent pulpitis complicated by
A. Minor saddle denture with clasp fixati- periodontitis
on C. Exacarbation of chronic pulpitis
B. Clasp denture with obturating part D. Exacerbation of chronic granulomatous
C. Common partial removable denture periodontitis
D. Common dental bridge E. Acute serous periodontitis
E. Protective palatal bars
49. A 48-year-old patient complains of
45. Preventive examination of an 8-year- subfebrile temperature and a growing
old boy revealed some lusterless chalk- ulcer on the gingival mucosa around the
like spots on the vestibular surface of the molars; looseness of teeth in the affected
11 and 21 teeth, which are localised in the area, cough. Objectively: gingival mucosa
precervical region. Subjective complaints in the region of the lower left molars has
are absent. What is the most likely di- two superficial, extremely painful ulcers
agnosis? with undermined edges. The ulcers floor is
yellowish, granular, covered with yellowi-
A. Acute initial caries sh, and sometimes pink granulations. The
B. White-spotted fluorosis ulcers are surrounded by the tubercles.
C. Local enamel hypoplasia Dental cervices are exposed, there is
D. Acute superficial caries a pathological tooth mobility. Regional
E. Chronic initial caries lymph nodes are enlarged and make dense
matted together groups. What is the most
46. A 44-year-old male patient complains likely diagnosis?
of fatigue and headache, limb numbness,
dry mouth, burning and pain in the A. Tuberculosis
tongue. Objectively: skin and oral mucosa B. Syphilis
are pale. There are painful cracks in the C. Acute aphthous stomatitis
corners of mouth. Dorsum of tongue D. Infectious mononucleosis
is smooth, glossy, with bright red stri- E. Decubital ulcer
ae. In blood: Hb- 70 g/l, RBCs - 1, 5 ·
1012 /l, color index - 1,6, leukopenia, 50. A 56-year-old man complains of pain
thrombocytopenia, lymphocytosis. What in the left parotidomasticatory region,
is the most likely diagnosis? progressing face asymmetry that was noti-
ced a month ago. Objectively: left-sided
A. Addison-Biermer anemia paresis of mimic muscles. To the fore of
B. Chronic posthaemorrhagic anemia earflap there is an ill-defined infiltration,
C. Late chlorosis the skin above it is tense and cyanotic;
D. Iron deficiency anemia left lymph nodes are enlarged. Opening of
E. Aplastic anemia mouth is limited down to 2,5 cm. The left
parotid duct doesn’t excrete saliva. What
47. A 25-year-old patient presents wi- is the most likely diagnosis?
th fluorosis of vestibular surfaces of the
upper incisors. Which of the following A. Adenocarcinoma
constructions will provide the maximum B. Mixed tumour
aesthetic result? C. Cyst of the gland
D. Glandular tuberculosis
A. Veneers E. Chronic lymphadenitis
B. Plastic crown
C. Ceramic crown 51. A 4-year-old girl complains of
D. Combined Kurylenko crown spontaneous acute toothache in the upper
E. Metal-plastic crown jaw on the right increasing during eating.
The pain arose two days ago. Objectively:
48. A 27-year-old patient complains about there is a deep carious cavity filled wi-
acute pain in the region of the 34 tooth th softened dentin in the 55 tooth. There
that is getting worse when biting down is no intercommunication with the tooth
on food. Roentgenographical survey cavity. Probing of the cavity floor is sli-
revealed an ill-defined zone of bone ti- ghtly painful. Percussion is painful. What
ssue destruction in the periapical region is the presumptive diagnosis?
Krok 2 Stomatology 2012 8

A. Exacerbation of chronic periodontitis


A. Acute pulpitis complicated by peri- of the 26 tooth
odontitis B. Acute purulent periodontitis of the 26
B. Acute purulent pulpitis tooth
C. Acute generalized (serous) pulpitis C. Acute pulpitis of the 26 tooth
D. Exacerbation of chronic gangrenous D. Acute purulent periostitis of upper jaw
pulpitis of the 26 tooth on the left
E. Acute purulent periodontitis E. Periodontitis of the 26, 27, 28 teeth
52. A 35-year-old man complains of short- 55. A 22-year-old medical student
term pain caused by thermal stimuli in the complains of the changed colour of his
46 and 47 teeth. Objectively: masticatory central upper incisor on the right. Two
surfaces of the 46, 47 teeth are intact, years ago the tooth was gray. Objectively:
approximal surfaces could not be exami- the 2 tooth is filled, changed in colour,
ned because of a very close arrangement stable, percussion is painless. The pati-
of teeth. What methods of study can ent has closed bite. What is the absolute
be used in this case if you suspect the contraindication to making porcelain
presence of hidden cavities? crowns for this patient?
A. X-ray A. Closed bite
B. Vital staining B. Defects of the anterior teeth that cannot
C. Fluorescent study be restored by fillings
D. Measuring electrical resistance C. Enamel hypoplasia with a change of
E. Electro-odontometry tooth shape and colour
D. Abnormal tooth colour
53. A 52-year-old man undergoes E. Defects of pulpless teeth that can not
treatment of the 36 tooth for chronic be restored by inlays
periodontitis. Radiography revealed
convoluted medial root canals; an 56. A 18-year-old patient complains of
irregular-shaped focus of destruction of acute spontaneous toothache irradiati-
bone tissue 0,2x0,2 cm large in the regi- ng to the right eye and temporal region.
on of the medial root apex. Which of the Objectively: there is a deep carious cavi-
following drugs is optimal for intracanal ty in the 27 tooth within circumpulpar
electrophoresis? dentin. Dentin is light, softened. Probi-
ng of the cavity floor and cold test cause
A. 10% solution of potassium iodide acute pain. What is the most likely di-
B. 1% solution of decamethoxin agnosis?
C. 1% solution of novocaine
D. 3% solution of copper sulphate A. Acute diffuse pulpitis
E. 0,1% solution of trypsin B. Acute purulent periodontitis
C. Exacerbation of chronic pulpitis
54. A 44-year-old patient consulted a D. Acute serous periodontitis
surgeon about constant acute pain in the E. Acute purulent pulpitis
upper jaw region on the left that is getti-
ng worse during teeth joining. The pain 57. A 43-year-old patient was found to
was noted 3 days ago. Objectively: the have no crown of the 13 tooth. The root
face is symmetrical, mouth opening is protrudes above the gum by 0,5-1 mm.
unlimited. The crown for the 26 tooth The apical part is filled. History: coronal
is half-destroyed. Probing of the cari- part of the tooth broke off three weeks
ous cavity is painless. Percussion of the ago. Prior to this the tooth had been
26 tooth provokes acute pain. Mucous treated for chronic periodontitis. Percussi-
membrane of the alveolar process is on is painless. The root edges are made by
edematic, hyperaemic at the level of hard moist mucous membrane of pale pi-
the 26 tooth. The 26 tooth had been nk colour. Specify the dentist’s tactics:
treated before. What is your provisional
diagnosis? A. Fabricate a crown-root inlay and metal-
ceramic crown
B. Remove the tooth root
C. Fabricate a simple pivot crown
D. Restore the tooth by means of an
anchor and photopolymer material
E. Fabricate a pivot Richmond crown
Krok 2 Stomatology 2012 9

58. A 40-year-old patient with mandible the thermal test causes acute long-lasting
fracture consulted a doctor 3 weeks after pain, percussion is painless. What is the
immobilization of fragments because of optimal treatment tactics?
pain and body temperature rise. Objecti-
vely: a slight swelling in the mental A. Vital pulp extirpation
region, mucous membrane of alveolar B. Application of fluorine lacquer
process in the area of the 21|12 teeth is C. Biological treatment of pulpitis
hyperaemic, edematic, palpatory painful. D. Devital pulp extirpation
Overtooth splint on 54321|12345 teeth E. Vital pulp amputation
is in satisfactory condition, no occlusion 62. A 5-year-old child has been di-
abnormalities were detected. The patient agnosed with congenital complete
was diagnosed with acute purulent peri- nonclosure of soft and hard palate.
ostitis of mandible. What surgical action is What type of anaesthesia is indicated for
indicated? uranostaphyloplasty?
A. Lancing of abscess to the bone A. Nasotracheal narcosis
B. Intraoral novocaine block B. Mask narcosis
C. Removal and replacement of the C. Intravenous narcosis
overtooth splint by a new one D. Orotracheal narcosis
D. Trepanation of the 21 and 12 teeth E. Endotracheal narcosis through
E. Surveillance of patient tracheostome
59. A 32-year-old patient complains of 63. A 24-year-old patient complains of
acute spontaneous attacks of pain in the aching pain in the 11 tooth that is getti-
14 tooth. The pain lasts for 10-20 minutes ng worse on biting. Two days ago the
and occurs every 2-3 hours. Carious cavi- tooth was filled for pulpitis. Objectively:
ty in the 14 tooth is filled with softened the 11 tooth is filled. The thermal test
dentin. Probing of the cavity floor is pai- causes no pain, vertical percussion is sli-
nful at one point. Cold stimulus causes ghtly painful. X-ray picture of the 11 tooth
pain. What is the most likely diagnosis? shows that the endodontic filling is 1 mm
A. Acute localized pulpitis above the root apex. Which of the followi-
B. Acute deep caries ng methods will be most effective for eli-
C. Hyperemia of the pulp minating this complication?
D. Exacerbation of chronic pulpitis A. Fluctuorization
E. Acute diffuse pulpitis B. Ultrahigh frequency therapy
60. A 32-year-old patient complains of the C. Relaxing incision
long-term dull toothache caused by hot D. Submucous injection of 1% solution of
food. The toothache appeared a month hydrocortisone
ago. Objectively: the 26 tooth has changed E. Analgetics
in colour, on the masticatory surface there 64. It is planned to make a metal-ceramic
is a deep carious cavity communicating crown supported by stump inlay for the
with the tooth cavity. Superficial probing 23 tooth. Objectively: the crown of the
of pulp is painless, deep probing is painful. 23 tooth is decayed down to the gingival
Electro-odontodiagnostics results: 85 µA. edge. Root canal is filled to the top. The
What is the most likely diagnosis? dentist made a wax stump model with a
A. Chronic gangrenous pulpitis pin, cast it in metal, fitted it to the tooth,
B. Chronic hypertrophic pulpitis fixed it by means of visphat-cement and
C. Chronic fibrous pulpitis got a working plaster impression. At what
D. Chronic fibrous periodontitis stage did he make an error?
E. Chronic concrementous pulpitis A. Impression taking
61. A 27-year-old patient complains of the B. Wax construction making
long-term pain in the 22 tooth caused C. Casting
by cold and hot food, as well as of D. Stump fitting
spontaneous pain lasting for 30 minutes E. Stump fixing
and occurring 3-4 times per day, getting 65. A 23-year-old military servant needs
worse at night. Pain arose 3 days ago after orthopaedic treatment at a specialized
preparation of the tooth for the acrylic hospital. He was diagnosed with false joint
crown. Objectively: the 22 tooth is intact, of mandible in its frontal part. The teeth
Krok 2 Stomatology 2012 10

are intact, stable, in threes on each side. grade mobility), the crown of the 44 tooth
Orthopaedic treatment by means of a bri- is destroyed by 1/2. What orthopedic
dge denture will be possible only if the jaw construction should be recommended for
defect is no more than: restoration of masticatory efficiency and
prevention of periodontal overload of the
A. 1 cm 47, 44 teeth?
B. 2 cm
C. 3 cm A. Arch denture with clasp fixation for the
D. 3,5 cm 47, 44, 34 teeth
E. 4 cm B. Soldered bridge supported by the 48, 44
teeth
66. According to the mother, a 5-year- C. Metal-ceramic bridge supported by the
old child complains about pain during 47, 46 teeth
swallowing, weakness, body temperature D. Bridge supported by the 47 tooth
rise upt to 39, 5oC, swelling of submental E. Small saddle denture with clasp fixation
lymph nodes. Objectively: the child’s for the 47, 44 teeth
condition is grave, body temperature is
38, 8oC. Mucous membrane of oral cavity 70. A 57-year-old patient complains of
is brightly hyperaemic and edematic with tooth mobility, inability to eat. Objecti-
haemorrhages and ulcerations. Pharynx is vely: the lower 35, 36, 37, 38, 44, 45, 46
brightly hyperemic, lacunae are enlarged and 48 teeth are missing; the 31, 32, 33, 34,
and have necrosis areas. Regional, cervi- 41, 42, 43, 47 teeth exhibit II grade mobi-
cal, occipital lymph nodes are painful, lity, their clinical crowns are low, tooth
enlarged and dense. What is the most li- equator is not pronounced. What is the
kely diagnosis? optimal denture construction in this case?
A. Infectious mononucleosis A. Removable whole-piece splint
B. Acute herpetic stomatitis B. Removable partial denture
C. Necrotizing ulcerative gingivostomatitis C. Kurliandsky splint bar
D. Herpetic angina D. Removable Bynin splint
E. Lacunar tonsillitis E. Removable splint with vestibulo-oral
clasp
67. A patient is 48 year old, according
to the results of clinicoroentgenological 71. A patient working as a bricklayer
examination it is indicated to remove complains of itching, burning, soreness of
the 26 tooth because of acute condition lips that show up only in the summer peri-
of chronic granulomatous periodontitis. od. He has been ill for 3 years. Objecti-
What kind of conduction anesthesia is vely: vermilion border of the lower lip is
indicated for this operation? hyperemic, edematic, covered with bli-
sters and painful erosions 2 mm in di-
A. Tuberal and palatinal ameter, crusts, cracks. What is the most
B. Torus likely diagnosis?
C. Infraorbital and incisive
D. Plexus A. Actinic cheilitis, exudative form
E. Infraorbital and palatinal B. Contact allergic cheilitis
C. Meteorological cheilitis
68. Examination of an 8-year-old child D. Eczematous cheilitis, exudative form
revealed irregular white spots on the vesti- E. Exfoliative cheilitis, exudative form
bular surface in the precervical region
of the of 11 and 12 teeth. The spots are 72. A 60-year-old patient complains of
smooth and stainable. What is the most pain in the masticatory muscles and
likely diagnosis? temporomandibular joints. The pain
occurs when she uses complete removable
A. Focal demineralization dentures fabricated a month ago. Objecti-
B. Enamel hypoplasia vely: the lower third of the face is
C. Superficial caries elongated, the lips can be closed with
D. Enamel erosion difficulty, smiling causes exposure of the
E. Fournier’s teeth denture base, the articulation is impaired.
A mistake must have been made at the
69. A 45-year-old patient complains of following stage of denture fabrication:
missing teeth in the lower jaw on the ri-
ght. Objectively: the 46, 45, 38, 48 teeth
are missing. The 47 tooth is mobile (1
Krok 2 Stomatology 2012 11

A. Determination and fixation of the the most informative method of study that
central occlusion allows to confirm the diagnosis?
B. Taking anatomic impressions
C. Taking functional impressions A. Vital staining
D. Testing the denture construction B. X-ray
E. Fitting the dentures and their correction C. Probing
D. Electro-odontometry
73. A patient consulted a dentist about a E. Thermometry
cosmetic defect in the cervical region of
the upper and lower canines. Various sti- 77. Parents of a 12-year-old child complain
muli cause no pain. Objectively: there are of white patches on the upper front teeth,
V-shaped defects on the vestibular surface which appeared six months ago. Objecti-
in the cervical area of the upper and lower vely: chalk-like patches on the vestibular
canines. Their surface is smooth, shiny, surfaces in the precervical region of the
hard. The reaction to probing and cold sti- 13, 12, 11, 21, 22, 23 teeth. Their enamel is
muli is absent. What treatment should be dull, pliable and rough on probing. There
administered? is a history of short-term pain caused by
chemical stimuli. What is your provisional
A. Filling of the defects diagnosis?
B. Metal crowns
C. Applications with 10% solution of A. Acute initial caries
calcium gluconate B. Chronic initial caries
D. Application of fluorine lacquer C. Acute superficial caries
E. Medical intervention is not necessary D. Systemic enamel hypoplasia
E. Dental fluorosis
74. A 57-year-old patient complains about
dryness and burning of the lateral surface 78. A 32-year-old female patient needs
of her tongue. These sensations disappear dental prosthetics. After the objecti-
during eating. She noted such sensations ve examination it was decided to use
three months ago. She has a history of ceramic-metal crown. What impressi-
gastritis with reduced secretory function. on material should be used for taking
Objectively: mucous membrane of tongue impressions for this construction?
and oral cavity has no peculiarities. The
back of tongue has thin white coating. A. Stomaflex
Regional lymph nodes are unpalpable. B. Plaster
Oral cavity is sanitized. What is the most C. Stomalgin
likely diagnosis? D. Stens
E. Orthocor
A. Glossodynia
B. Lingual nerve neuritis 79. A 55-year-old patient has a painless,
C. Candidiasis tuberous, cyanotic pedunculated formati-
D. Desquamative glossitis on 2х1х1,5 cm large that appeared on the
E. Hunter-Moeller glossitis site of the extracted 46 tooth. Opening
of mouth is not limited. Intra-oral X-ray
75. A 65-year-old patient needs complete picture of alveolar process in the region
removable dentures for both jaws. At the of the removed 46 tooth shows a focus of
stage of "testing the denture constructi- bone tissue destruction. What is the most
on"the doctor checks the pronunciation likely diagnosis?
of sounds "S"and "Z". Which method of
normalization of speech functions should A. Giant-cell epulis
be applied in this case? B. Hard odontoma of mandible
C. Hypertrophic gingivitis
A. Phonetic tests D. Papilloma of mucous membrane in the
B. Graphic area of the extracted 46th tooth
C. Myogymnastics E. Ameloblastoma of mandible
D. Spectrographic
E. Acoustic 80. A 52-year-old patient presented to
the prosthetic dentistry clinic complaini-
76. An 18-year-old man complains of ng of missing upper jaw teeth and difficult
the 14, 13, 12, 23, 24 teeth being sensiti- mastication. The patient has an indicati-
ve to sweet and sour food. Examination on for a partial laminar denture for the
revealed some isolated ill-defined chalky upper jaw with retaining clamps on the
spots in the precervical region. What is 14, 23 teeth. What kind of denture stabili-
Krok 2 Stomatology 2012 12

zation will enable such positioning of the should be taken for the defect correcti-
clasps? on?
A. Transversal A. Forehead flap
B. Sagittal B. Cheek flap
C. Diagonal C. Infraorbital flap
D. Sagitally-transversal D. Shoulder flap
E. Diagonally-sagittal E. Scapulohumeral flap
81. A 56-year-old patient presents to a 84. A 21-year-old patient complains of
dental clinic. He has an indication for the constant progressing throbbing pain in the
extraction of the 22 tooth. Examination 27 tooth. Objectively: a large carious cavi-
revealed periosteal abscess in the projecti- ty is filled with softened dentin, the tooth
on of the 21, 22, 23 teeth, flattening of cavity is closed. Probing of the cavity floor
the mucogingival junction. What kind of is painless, percussion causes acute pain,
peripheral conduction anaesthesia is most there is grade II tooth mobility. Palpation
advisable? of the mucous membrane in the projecti-
on of the root apex of the 27 tooth causes
A. Extraoral infraorbital anaesthesia + pain. Radiological changes are absent.
incisor anaesthesia What is the most likely diagnosis?
B. Intraoral infraorbital anaesthesia +
incisor anaesthesia A. Acute purulent periodontitis
C. Intraoral infraorbital anaesthesia + B. Exacerbation of chronic periodontitis
palatinal anaesthesia C. Acute diffuse pulpitis
D. Plexus anaesthesia in the projection of D. Acute serous periodontitis
the 22 tooth + incisor anaesthesia E. Acute purulent pulpitis
E. Plexus anaesthesia in the projection of
the 22 tooth + palatinal anaesthesia 85. A 25-year-old patient got a trauma
in the chin region. On the basis of X-ray
82. A 33-year-old patient complai- he was diagnosed with bilateral mental
ns about pain, gingival haemorrhage, fracture of mandible. Specify the directi-
halitosis, body temperature rise up on of displacement of the minor fragment:
to 37, 8o C. Objectively: gums are
apparently hyperaemic, edematic, bleed A. Downward and backward
easily, parodontal pouches are 3- B. Upward and forward
4 mm deep, and contain purulent C. Upward and backward
exudate. Orthopantomogram shows di- D. Downward and forward
ffuse osteoporosis of alveolar process, E. There is no displacement
resorption of interdental septa down to
1/3 of their height. What is the most likely 86. 2 days ago a 12-year-old patient
diagnosis? prsented with body temperature rise up to
38, 0oC, on the second day he developed a
A. Exacerbation of chronic generalized I bilateral edema in the parotid region. The
degree periodontitis skin over the edema was tense, of normal
B. Chronic generalized I degree peri- color. Palpation revealed soft, enlarged,
odontitis painful parotid salivary glands. Salivation
C. Exacerbation of chronic generalized II from the parotid ducts was abnormal, the
degree periodontitis duct orifices were hyperaemic. Palpation
D. Chronic generalized II degree peri- caused pain in the angle of mandible, at
odontitis the top of mastoid bone, in front of the
E. Exacerbation of chronic catarrhal gingi- antilobium. What is the most likely di-
vitis agnosis?
83. A 43-year-old patient consulted a A. Mumps
maxillofacial surgeon about a cosmetic B. Herzenberg pseudoparotitis
defect in the right half of his nose. It is C. Acute bacterial parotitis
known from the history that 7 months D. Purulent-necrotic parotitis
ago he got a gunshot wound. Objecti- E. Sjogren’s syndrome
vely: there is a perforating defect 1,5 x2
cm large in the right half of the nose. The 87. A 38-year-old patient with chronic
skin around the defect has not changed in generalized periodontitis has been
colour. It was decided to use the "Indi- referred to orthopedic treatment. Objecti-
an"method of rhinoplasty. What tissue vely: dentitions are without gaps, the 12,
Krok 2 Stomatology 2012 13

11, 21, 22 teeth are pulpless and exhibit I A. Fourth


grade mobility. The other teeth are stable. B. First
What is the most aesthetic splint for the C. Second
anterior teeth? D. Third
E. Fifth
A. Mamlok splint
B. Ring adhesion splint 92. A 17-year-old man complains of a
C. Soldered combined crowns cosmetic defect in form of light spots on
D. Cap splint the teeth. In the area of his residence the
E. Brace fluoride concentration in drinking water
is at the rate of 1 mg/l. Objectively: vesti-
88. A 42-year-old patient has been hospi- bular surface of the 11,12, 21, 22, and
talized with Le Fort II fracture of maxilla. tubercles of the 16, 26, 36 and 46 teeth
Select an appliance for the treatment of have chalky spots with shiny surface that
this patient: have been present since the time of erupti-
on. What is the most likely diagnosis?
A. Zbarzh
B. Kulagin A. Systemic hypoplasia
C. Rudko B. Multiple caries
D. Penn-Brown C. Endemic fluorosis
E. Yadrova D. Enamel erosion
E. Amelogenesis imperfecta
89. An 8-year-old child presents with an
edema of the submandibular region, the 93. A 65-year-old patient complains
mouth can be opened by 1,5 cm, further about unsatisfactory fixation of complete
opening is difficult, body temperature is removable lamellar denture of his upper
37, 6oC, mucogingival fold is vestibularly jaw. The denture was fabricated 6 years
flattened, hyperaemic and swollen. The 84 ago. Objectively: balancing and poor fi-
and 85 teeth have fillings, their percussi- xation of complete removable denture is
on is painless. The 84 tooth exhibits I present. What is the reason of such condi-
degree mobility. What is the most likely tion?
diagnosis?
A. Atrophy of osseous base of the
A. Acute odontogenic periostitis prosthetic bed tissues
B. Acute odontogenic osteomyelitis B. Wear of artificial teeth
C. Chronic odontogenic periostitis C. Bad hygiene of removable denture
D. Chronic odontogenic osteomyelitis D. Discoloration of basic plastic
E. Exacerbation of chronic periodontitis E. Loss of certain antagonists
90. A 43-year-old patient complains about 94. A patient complains of heaviness in
mobility of lower jaw teeth. Objectively: the left section of his head, pain in the
the dentition is intact. Tooth mobility is 26 tooth. Objectively: the crown of the 26
of I-II grade. It is planned to immobili- tooth is destroyed by 2/3 by caries, the
ze teeth by means of a removable splint tooth percussion is weakly positive. X-ray
common for the whole dentition. What picture of paranasal sinuses shows defini-
stabilization will be provided by means of te unilateral dome-shaped veiling of the
this splint? upper left maxillary sinus. On the X-ray
picture of the 26 tooth the periodontal fi-
A. Circle ssure at the root apex is missing. What is
B. Frontal the most likely diagnosis?
C. Frontal-lateral
D. Sagittal A. Radicular cyst ingrown into the maxi-
E. Transversal llary sinus
B. Rhinoantritis
91. A patient needs a clasp denture for the C. Odontogenic sinusitis
lower jaw. Objectively: dental formula is D. Cyst of the maxillary sinus mucosa
31, 32, 33, 34, 41, 42, 43, 44. The 44 totoh E. Malignant maxillary tumour
is inclined toward the cheek. What type of
Ney’s clasp should be used? 95. Preventive examination of a 5-year-
old child revealed half-open mouth, di-
fficult closing of lips, primary occlusion,
4 mm sagittal gap, homonymous canines
and second molars. The upper dental arch
Krok 2 Stomatology 2012 14

is V-shaped, the lower one is trapezoid. after the patient had had a cold. Objecti-
Both dental arches in primary occlusion vely: lips are covered with haemorrhagic
should have the following shape: crusts, hyperaemic mucous membrane of
lips and cheeks has erosions covered with
A. Semicircle fibrinous films. Hypersalivation is present.
B. Semiellipse What is the most likely diagnosis?
C. Parabola
D. Quadrangle A. Multiform exudative erythema
E. Triangle B. Pemphigus vulgaris
C. Herpes recidivicus
96. A 46-year-old patient complains about D. Herpetiform Duhring’s dermatitis
pain and bleeding from the carious cavi- E. Stevens-Johnson syndrome
ty of her 27 tooth during eating. Previ-
ously she had spontaneous pain. Exami- 100. A 25-year-old man consulted a denti-
nation of the 27 tooth revealed a deep st about extraction of the 18 tooth, it’s
carious cavity on the masticatory surface crown is destroyed by 1/2. The tooth was
consisting of red tissue, probing induced treated more than once. During tooth
pain and haemorrhage. What treatment extraction the maxillary tuber was acci-
method should be chosen? dentally torn off. What actions should be
taken?
A. Vital extirpation
B. Devital extirpation A. To remove the fragment and stitch up
C. Devital amputation the wound
D. Vital amputation B. To try to restore the fragment to its
E. Biological method place
C. To restore the fragment to its place and
97. A 25-year-old student complains of a fix it there
carious cavity in the 22 tooth. The filli- D. To remove the fragment
ngs fell out 2 months ago. The tooth had E. To remove the fragment and tampon
been treated before for pulpitis. Objecti- the wound
vely: there is a deep carious cavity with
the rests of filling on the medial surface 101. A 10-year-old child complains of
of the 22 tooth. The crown of the 22 tooth gingival pain and haemorrhage which
is dirty pink. X-ray shows a root canal fi- appeared two days ago after a cold.
lled with the filling material by 1/2 of the Objectively: the gingiva is edematic,
root length; in the region of the root apex hyperaemic, bleeds easily, painful on
there is a well-defined focus of destructi- palpation. The tips of gingival papillae are
on of bone tissue 0,3x0,3 cm large. What dome-shaped. What is the most likely di-
is the most likely diagnosis? agnosis?
A. Chronic granulomatous periodontitis A. Acute catarrhal gingivitis
B. Residual pulpitis B. Chronic catarrhal gingivitis
C. Radicular cyst C. Hypertrophic gingivitis
D. Chronic fibrous periodontitis D. Ulcerative gingivitis
E. Chronic granulating periodontitis E. Generalized periodontitis
98. A 35-year-old patient consulted a 102. A 53-year-old patient consulted a
dentist about extraction of the 14 tooth dentist about pain and an ulcer that
because of exacerbation of chronic peri- turned up in the region of hard palate
odontitis following ineffective therapeutic under his partial removable denture. He
treatment. What tools should be applied hasn’t used the denture for three weeks,
for extraction? but the ulcer does not heal or reduce
in size. Examination reveals a 2x1,5 cm
A. S-shaped forceps large ulcer in the region of the alveolar
B. Crown bayonet-shaped forceps process and hard palate at the level of the
C. S-shaped forceps curved right missing 14 tooth. The ulcer has everted
D. Straight forceps rolled edges and necrotic floor, it is dense
E. Root bayonet-shaped forceps and painful. Gingival and palatal tissues
around the ulcer are infiltrated. What is
99. A 32-year-old patient complains of the most likely diagnosis?
mouth soreness, body temperature rise up
to 38, 5oC, indisposition. Such condition
has occurred periodically for several years
Krok 2 Stomatology 2012 15

A. Maxillary carcinoma A. Collapse


B. Maxillary syphilis B. Syncope
C. Maxillary tuberculosis C. Traumatic shock
D. Maxillary actinomycosis D. Anaphylactic shock
E. Decubital ulcer E. Attack of stenocardia
103. A 13-year-old girl complains about 106. A patient complains about pain in the
frequent falling out of a filling in the 21 45 tooth induced by cold, sour and sweet
tooth. It is known from the anamnesis that food stimuli. The pain abates when the
2 years ago she underwent treatment on stimulus action is stopped. Objectively:
account of a dental trauma. Objectively: there is a carious cavity on the masticatory
a transverse defect of 1/3 of the crown in surface within mantle dentin filled with
the 21 tooth. Percussion is painless. Tooth food rests and softened dentin, overhangi-
colour is unchanged. X-ray picture shows ng enamel edeges are chalky. What is the
that root canal is filled by 1 mm from the diagnosis?
apex, filling material closely fits the walls
of root canal. What stomatological tactics A. Acute median caries
should be chosen? B. Chronic median caries
C. Acute superficial caries
A. The defect should be restored with D. Acute deep caries
photopolymer E. Chronic deep caries
B. The root canal should be refilled
C. The root apex should be resected 107. A 79-year-old female patient
D. The tooth should be extracted consulted a prosthodontist about denture
E. The tooth should be crowned with an replacement. The patient has a history
artificial crown of a stroke. Objectively: acute irregular
atrophy of the alveolar processes of both
104. A 34-year-old male patient complai- jaws is present; mucous membrane of
ns of acute spasmodic pain in the regi- the oral cavity is dry and nonmobile.
on of his upper jaw on the left that is The previous dentures cannot be fixed.
getting worse as affected by cold sti- What is the most appropriate prosthetic
muli. Toothache irradiates to the ear and construction?
temple. He had acute toothache of the 37
tooth one year ago, but he didn’t consult A. Dentures with elastic lining
a dentist. Pain recurred three days ago. B. Dentures with extended borders
Objectively: the 37 tooth has a carious C. Dentures with metal bases
cavity communicating with the dental D. Dentures with shortened borders
cavity. Probing of the opened carious E. Implant-supported dentures
cavity is extremely painful. X-ray picture
shows widening of periodontal fissure at 108. A 56-year-old patient consulted a
the root apex of the 37 tooth. What is the dental surgeon about the root extraction
most likely diagnosis? of the 17 toot. What kind of anaesthesia
should be applied?
A. Exacerbation of chronic pulpitis
B. Exacerbation of chronic granulating A. Palatinal, tuberal anaesthesia
periodontitis B. Palatinal, infraorbital anaesthesia
C. Exacerbation of chronic fibrous peri- C. Infiltration, inscisor anaesthesia
odontitis D. Mandibular anaesthesia
D. Acute diffuse pulpitis E. Torus anaesthesia
E. Acute purulent pulpitis 109. A 23-year-old female patient
105. During tooth extraction a 32-year-old complains of a pronounced limitation of
patient presented with sudden weakness, mouth opening, throat pain irradiating to
pale skin, cold sweat, weak pulse, a signi- the ear. Objectively: body temperature
is 37, 9oC. The face is symmetrical, the
ficant AP drop (diastolic pressure - 40 mm
colour of skin has not changed. Palpati-
Hg). What complication developed in the on of the jaw causes acute pain, mouth
patient? opens by 1 cm. The patient had been gi-
ven anasthesia by Berchet-Dubov. After it
oral examination revealed an edema and
hyperemia of the mucous membrane of
the pterygomaxillary fold, there was an
infiltration painful on palpation. What is
Krok 2 Stomatology 2012 16

the most likely diagnosis? A. Functional shifting of supporting teeth


under stress
A. Abscess of pterygomaxillary space B. Injuring of circle ligament by crown
B. Abscess of alveololingual groove edges
C. Parapharyngeal space phlegmon C. Devitalization of supporting teeth
D. Peritonsillar space abscess D. Massive grinding off of hard tissues
E. Submandibular phlegmon E. Garland modelling in the precervical
region
110. Parents of an 8-year-old child
complain about a painful formation in 113. An orthodontist monitors a 4-year-
the child’s oral cavity that obstructs food old child with mouth breath. The child has
intake. The same complaints were regi- a history of adenotomy. Objectively: pri-
stered two years ago. Mucous membrane mary dentition occlusion; the upper inci-
of lateral tongue surface is hyperemic and sors overlap the lower ones by 1/3; distal
edematic. There is an oval erosion over surfaces of the second temporary molars
0,7 cm large covered with yellow greyi- are situated in the same vertical plane.
sh deposit. Erosion edges are hyperemic What preventive device will help the child
and painful on palpation. The child has a to give up the habit of mouth breath?
history of chronic cholecystocholangitis.
What is the most likely diagnosis? A. Standard Schonherr’s vestibular screen
B. Vesibular and oral Kraus’ screen
A. Chronic recurrent aphthous stomatitis C. Frankel’s function regulator
B. Erythema multiforme D. Andresen-Haupl activator
C. Behcet’s syndrome E. Rudolph’s appliance
D. Stevens-Johnson syndrome
E. Traumatic erosion 114. A 5-year-old boy complains of a
carious cavity, periodic toothache, gum
111. A 12-year-old child complains of swelling. Objectively: masticatory surface
body temperature rise up to 39, 8oC, of the 74 tooth has a deep cavity not
weakness, headache and pain in throat communicating with the tooth cavity, cold
getting worse when swallowing. Objecti- stimuli, probing and percussion are pai-
vely: mucous membrane of gums is nless, mucous membrane is pale pink,
edematic, hyperemic. Tonsils are bri- there is a caicatrix from a fistula. X-ray pi-
ght red, hypertrophic, covered with cture shows an ill-defined focus of bone ti-
yellow-gray deposit which does not ssue destruction in the region of the roots
extend beyond the lymphoid tissue and bifurcation. What is the most likely di-
can be easily removed. Submandibular, agnosis?
occipital lymph nodes are significantly
enlarged, slightly painful on palpation. A. Chronic granulating periodontitis
Hepatosplenomegaly is present. Identi- B. Exacerbation of chronic fibrous peri-
fy the causative agent of this disease: odontitis
C. Chronic fibrous periodontitis
A. Epstein-Barr virus D. Exacerbation of chronic granulating
B. Bordet-Gengou bacillus periodontitis
C. Coxsackie virus E. Chronic granulomatous periodontitis
D. Herpes virus
E. Loeffler’s Bacillus 115. A 13-year-old girl complains
of toothache increasing while biting.
112. A 57-year-old patient complains Objectively: the 36 tooth has a deep cari-
about mobility of his metal-ceramic dental ous cavity not communicating with the
bridge supported by the 33, 37 teeth. tooth cavity, the response to thermal sti-
The bridge has been in use for 9 months. muli is painless, percussion is sharply pai-
Objectively: X-ray picture shows alveolar nful, mucous membrane is intact. X-ray
process atrophy by 2/3 in the area of the picture shows no changes. What is the
33, and by 1/2 of root length in the area of most likely diagnosis?
the 37; there are pathological pockets, gi-
ngivitis. What is the cause of pathological A. Acute serous periodontitis
mobility of supporting teeth? B. Acute purulent periodontitis
C. Acute serous pulpitis
D. Acute purulent pulpitis
E. Exacerbation of chronic periodontitis
116. A 12-year-old patient complains
Krok 2 Stomatology 2012 17

of gingival haemorrhage, tooth mobili- A. Lateral incisors and second molars


ty. He has has these presentations since B. Central incisors and first molars
the age of 4. Objectively: gingiva around C. First and second premolars
all the teeth is hyperaemic, edematic, D. Canines and first molars
bleeds during instrumental examination. E. Second premolars and first molars
The teeth roots are exposed by 1/3 and
covered with whitish plaque. The teeth 120. A 20-year-old pregnant woman
are mobile. Dentogingival pockets are 4-5 complains of gingival enlargement,
mm deep. External examination revealed bleeding and pain during eating and tooth
dryness and thickening of the outer layer brushing. Objectively: gingival papillae on
of skin on the palms, anterior third of the upper and lower jaw are hyperaemic,
the forearms, soles; there are skin cracks. haemorrhagic, painfu, cover the crowns
What is the most likely diagnosis? of teeth by 1/2. Scalloped contours of
marginal gingiva are abnormal. What is
A. Papillon-Lefevre syndrome the most likely diagnosis?
B. Hand-Schuller-Christian disease
C. Niemann-Pick disease A. Hypertrophic gingivitis, edematous
D. Letterer-Siewe disease form
E. Cyclic neutropenia B. Acute catarrhal gingivitis
C. Chronic catarrhal gingivitis
117. A 47-year-old patient came to D. Generalized periodontitis
an orthopaedic stomatology center wi- E. Hypertrophic gingivitis, fibrous form
th complaints about missing tooth in
the frontal part of his upper jaw, 121. A 55-year-old patient consulted a
cosmetic defect. Objectively: the occlusi- dentist about a roundish tumour-like
on is orthogenic, the 11 tooth is mi- formation of about 1 cm in diameter
ssing. Anamnesis data: the patient had located within the vermilion border of
myocardial infarction 3 months ago. What his lower lip. Objectively: the tumour-
denture should be applied for the time like formation protrudes about 5 mm
being? above the vermilion border, it is dense and
grayish-red. The surface of the formation
A. Partial removable lamellar denture is covered with thin scales that can hardly
B. Bridge denture suported by the 21 and be removed. What is the most likely di-
12 teeth agnosis?
C. Bridge denture suported by the 21 tooth
D. Clasp denture A. Verrucous precancer of the vermilion
E. Implant border of lip
B. Abrasive precancerous Manganotti’s
118. Parents of a 6-year-old child applied cheilitis
to a pedodontist for preventive examinati- C. Precancerous limited hyperkeratosis of
on of their child. The oral cavity is saniti- the vermilion border of lip
zed. According to the parents, the child D. Bowen’s disease
has recently cut the 36 and the 46 tooth. E. Erythroplasia of Queyrat
What method of caries prevention should
be applied within 1,5-2 years after cutting 122. A 50-year-old patient has a defect
of the mentioned teeth? of the lower dental arch. It is planned to
make an implant-supported bridge for its
A. Fissure hermetization restoration. X-ray picture shows that the
B. Fissure silvering height of the bone mass from projection of
C. Coating the teeth with fluorine lacquer mandibular canal up to the top of alveolar
Ftorlak crest is 2 cm. What type of implant should
D. Remodentum solution applications be applied?
E. Gargling with sodium fluoride
A. Threaded
119. A 70-year-old patient is awaiting B. Endodontic-endoosseous
complete removable dentures for both C. Plate-form
upper and lower jaws. Teeth placement D. Subperiosteal
will be made by Vasilyev’s method. What E. Conical
teeth in the upper denture must not touch
glass? 123. External examination of a 7-year-old
child revealed: thickening of nose bridge,
semi-open mouth, dry lips. Mouth corners
are peeling. Anamnesis data: the child
Krok 2 Stomatology 2012 18

sleeps with open mouth. Examination of


oral cavity revealed no changes. What di- A. S-shaped forceps with a projecting tip
spensary group will this child fall into? on the left beak
B. S-shaped forceps with a projecting tip
A. The second on the right beak
B. The first C. Straight forceps
C. The third D. Straight elevator
D. The fourth E. S-shaped forceps without projecting tips
E. -
128. A 25-year-old male patient has
124. A 10-year-old child undergoes sani- bilateral fracture of the lower jaw. A
tation of the oral cavity. The girl was found fragment in the region of the 44, 43, 42, 41,
to have chalky spots on the vestibular 31, 32, 33, 34 teeth is displaced downward
surfaces in the precervical region of the and backward. What appliance should be
21 and 12 teeth. Enamel surface is dull, used for the fragment reposition?
smooth. Pain reaction to the temperature
stimuli is absent. What additional method A. Post’s appliance
of examination is expected to confirm the B. One-jaw bite-guard splint
diagnosis? C. Kurlyandsky’s appliance with levers
D. Weber’s appliance
A. Vital staining E. Shur’s appliance
B. Orthopantomography
C. Intraoral roentgenography 129. A 49-year-old patient consulted a
D. Electroodontodiagnostics dental surgeon about the oral cavity
E. Ultraviolet stomatoscopy sanation. He has an indication for the
extraction of the 16 tooth. History: the
125. Objective examination of a 10-year- tooth hasn’t been treated before, decayed
old child revealed that the whole lower within the last 4 years. Objectively: the
lip was slightly hyperemic, infiltrated, dry, 16 tooth’s crown is destroyed by over
covered with small scales. Architectonics 2/3, the mucosa exhibits no pathological
of lips is changed. The child complains of changes. Which tool is required for the
dryness and a feeling of tense lips, especi- tooth extraction?
ally in autumn and winter. The child had a
bad habit of lip sucking. What is the most A. Bayonet root forceps
likely diagnosis? B. S-shaped forceps (right)
C. Straight elevator
A. Meteorological cheilitis D. S-shaped closed-beak forceps
B. Allergic cheilitis E. Crown bayonet forceps
C. Atopic cheilitis
D. Exfoliative cheilitis 130. A 9-year-old child complains of pain
E. Microbial cheilitis caused by sweet and sour food in an upper
tooth on the left. Objectively: the 26 tooth
126. A patient has got a traumatic fracture has a carious cavity on the masticatory
of mandible in the area of the missing 34, surface within the enamel limits. What is
35 teeth with a slight displacement and a the optimal material to fill the 26 tooth?
defect of alveolar part in the area of the
34, 35 teeth. Other teeth on both lower A. Сomposite
and upper jaws are intact. What splint B. Glass ionomer
would be optimal in this case? C. Silicophosphate cement
D. Silicate cement
A. Tigerstedt’s splint with a spreading E. Zinc phosphate cement
curve
B. Plain splint cramp 131. A patient complains about
C. Port’s splint spontaneous pain in the area of his 15
D. Vasilyev’s splint tooth he has been feeling for 2 days.
E. Vankevich splint Thermal stimuli make the pain worse, its
attacks last up to 30 minutes. Objecti-
127. A 30-year-old patient needs to vely: there is a deep carious cavity in
have his 26 tooth extracted because of the 15 tooth consisting of light softened
exacerbation of chronic periodontitis. dentin, floor probing is painful in one
Objectively: the crown of the 26 tooth is point, reaction to the thermal stimuli is
decayed by 1/3. What forceps can be used positive, percussion is painless. Make a di-
for this tooth extraction? agnosis:
Krok 2 Stomatology 2012 19

lip. Mucous membrane hasn’t changed its


A. Acute local pulpitis colour. Make a diagnosis:
B. Acute diffuse pulpitis
C. Pulp hyperemia A. Retention cyst of lower lip
D. Acute deep caries B. Lip abscess
E. Acute condition of chronic pulpitis C. Lip papilloma
D. Lip fibroma
132. A 6-year-old child presents with E. Lip lipoma
weakness, pain in throat when swallowi-
ng, body temperature rise up to 38, 0oC. 136. A 35-year-old patient complains of a
Examination of the the oral cavity neoplasm on the tip of the tongue whi-
revealed massive hyperaemia of the ch he hurts with his teeth. The neoplasm
mucous membrane of the soft palate, sometimes increases, and sometimes
palatine arches, tonsils, uvula; there were decreases in size. Objectively: on the tip
also single vesicles and erosions extremely of tongue there is a roundish neoplasm
painful when touched. Regional lymph 0,5 cm in diameter with distinct borders
nodes are enlarged, painful on palpation. and broad base. The neoplasm is the same
What is the most likely diagnosis? colour as the mucosa of tongue. What is
the most likely diagnosis?
A. Herpangina
B. Chickenpox A. Papilloma of tongue
C. Mycotic angina B. Abscess of tongue
D. Infectious mononucleosis C. Lipoma of tongue
E. Diphtheria D. Hemangiofibroma of tongue
E. Fibroma of tongue
133. A 35-year-old patient complains
of progressing throbbing pain in the 26 137. A 35-year-old female patient
tooth. Objectively: the 26 tooth has a consulted a dentist about a painless,
carious cavity filled with softened denti- slowly growing neoplasm in the regi-
ne, tooth cavity is closed, probing of the on of the 11 and 12 teeth. Examination
cavity floor is painless, percussion causes revealed that the tumour was light-pink,
acute pain. There is I grade tooth mobili- flat, adjacent to the teeth, had a pedicle.
ty. Roentgenological changes are absent. The tumour was up to 1,5 cm large, with
What is the most likely diagnosis? smooth surface and dense consistency. It
was diagnosed as an epulis in the region of
A. Acute purulent periodontitis the 11 and 12 teeth. What form of epulis
B. Acute purulent pulpitis are these clinical findings typical for?
C. Acute serous periodontitis
D. Exacerbation of chronic periodontitis A. Fibrous
E. Acute diffuse pulpitis B. Angiomatous
C. Giant-cell
134. A patient complains of burning, D. Pregnancy epulis
itch and lower lip enlargement. He has E. -
been suffering from this for a long ti-
me. Objectively: the patient’s face is 138. As a result of a car accident a 45-
asymmetric due to the flattening of year-old patient got an injury of his upper
nasolabial fold. His lower lip is edematic, jaw. Examination revealed elongated and
of normal colour, painless on palpation. flattened face, profuse nasal haemorrhage,
The patient has plicated tongue. What is liquorrhea from the nose and ears. These
your provisional diagnosis? clinical presentations are typical for the
following fracture of upper jaw:
A. Melkersson-Rosenthal syndrome
B. Quincke’s edema A. Subbasal (Le Fort III)
C. Lymphangioma B. Subnasal (Le Fort I)
D. Hemangioma C. Suborbital (Le Fort II)
E. Granulomatous Miescher’s cheilitis D. Bilateral fracture of zygomatic bones
E. -
135. A 50-year-old woman complains
about a neoplasm on her lower lip on 139. A 7-year-old child has protruding
the side of oral cavity that appeared a chin, the lower lip overlaps the upper
month ago and has been slowly growing one. There are diastemas and tremas
since that. Objectively: there is a roundish, between the lower incisors, the lower inci-
elastic, painless neoplasm inside the lower sors overlap the upper incisors by 2/3 of
Krok 2 Stomatology 2012 20

the crown height. First permanent molars about a week ago. Objectively: there is
demonstrate Angle’s class III relation. a roundish bulge up to 3 cm in diameter
Sagittal gap is 3 mm. The correct doctor’s in the right sublingual region. Mucous
tactics will be to: membrane over it is not hyperaemic, there
is fluctuation symptom in the center. Whi-
A. Use Bruckl’s appliance ch disease do these symptoms correspond
B. Recommend a complex of with?
myogymnastic exercises
C. Use Angle’s apparatus A. Retention cyst of the right sublingual
D. Use Bynin appliance salivary gland
E. Use Schwartz appliance B. Retention cyst of the right submandi-
bular salivary gland
140. A 5-year-old child was found to have C. Acute sialoadenitis of submandibular
missing upper molars. Lower incisors are salivary gland
in contact with the mucous membrane of D. Chronic sialoadenitis of sublingual
palate. Specify the doctor’s tactics: salivary gland
E. Sjogren’s disease
A. Fabricate a removable laminar denture
B. Examine the child every six months 144. During orthopaedic treatment a 47-
until the eruption of permanent teeth year-old patient was given anaesthesia
C. Examine the child once a year until the (with Ultracain DS forte). 20 minutes after
eruption of permanent teeth the injection the patient presented wi-
D. Fabricate an orthodontic appliance for th hyperaemia of skin, headache, dizzi-
the treatment of closed bite ness, increased heart rate. The patient had
E. Medical intervention is not needed previously undergone dental treatment
with the use of this anaesthetic. What
141. A child is 7 years old. He has complication developed in this patient?
early transitional dentition. There is
overcrowding of the lower front teeth: A. AP rise
the 42 and 32 teeth erupted orally with B. Anaphylactic shock
a complete lack of space. Make a plan of C. Syncope
treatment: D. Collapse
E. Attack of stenocardia
A. Serial consecutive extraction by Hotz
method 145. A 35-year-old patient seeks
B. Extraction of the 42 and 32 teeth prosthetic dentistry. Objectively: 18, 14,
C. Extraction of the 41 and 31 teeth 13, 12, 11, 21, 22, 23, 24, 28. The crowns
D. Extraction of the 83 and 73 teeth confining the defect are high and stable.
E. Extraction of the 84 and 74 teeth To restore the integrity of the dentition it
is planned to fabricate a one-piece clasp
142. A 14-year-old girl complains of indi-
denture. What kind of mechanical fixator
stinct pronunciation that showed up at the should be used?
age of 14 after the acute respiratory vi-
ral disease. Examination revealed normal A. Bar fixation
face and normal teeth alignment, occlusal B. Supporting-retaining clasps
disharmony was not found. Palpation di- C. Retaining clasps
dn’t reveal cleft palate. Uvula doesn’t D. Telescopic fixation
move during pronunciation of sounds, its E. Dentoalveolar clasps
palpation does not cause gag reflex. What
is the reason for indistinct pronunciation 146. A 42-year-old woman prsented to a
of sounds? prosthetic dentistry clinic seeking dental
prosthetics. Objectively: dental formula is
A. Paresis of the soft palate and uvula 18 . . . . 13 12 11 21 22 23 . . . . 28
muscles 48 47 46 45 44 43 42 41 31 32 33 34 35 36 37 .
B. Adenoid vegetations The bite is deep, clinical crowns are low,
C. Palatal slit survey line is not marked. The patient
D. Hypertrophy of lingual tonsil suffers from epileptiform attacks. What
E. Deformation of the bite prosthesis is indicated?
143. A 35-year-old patient complains of
a swelling in the sublingual region whi-
ch impairs speech and causes discomfort
during eating. The swelling turned up
Krok 2 Stomatology 2012 21

A. Partial removable lamellar prosthesis nful. X-ray picture shows improperly filled
with metal base root canal. It is planned to remove the 21
B. Dental bridges tooth crown. What kind of anaesthesia
C. Partial removable lamellar plastic should be given?
prosthesis with retaining clasps
D. Partial removable lamellar prosthesis A. Field block anaesthesia
with supporting-retaining clasps B. Infiltration anaesthesia
E. Clasp denture C. Intraligamentous anaesthesia
D. Application anaesthesia
147. A 22-year-old patient presented to a E. Intraosseous anaesthesia
prosthetic dentistry clinic because of mi-
ssing of the 21 tooth, the 11 and 22 teeth 151. An 8-year-old child has a deep cari-
are intact. The 21 tooth was extracted 2 ous cavity communicating with tooth cavi-
months ago. What construction is most ty on the distal-approximal masticatory
suitable in this case? surface of the 75 tooth. Probing causes
pain. Percussion is painless. Cold water
A. Two-stage implantation causes slowly abating pain. The tooth
B. One-stage implantation with si- decayed several months ago and wasn’t
multaneous fabrication of metal-ceramic treated. What treatment method would
crown be efficient in this case?
C. Partial removable denture
D. Stamped-soldered denture supported A. Devital amputation
by the 11 and 22 teeth B. Biological method
E. Metal-plastic denture supported by the C. Vital extirpation
11 and 22 teeth D. Vital amputation
E. Devital extirpation
148. A 48-year-old patient has undergone
unilateral resection of the upper jaw. He 152. An 8-year-old child complains about
needs a resection prosthesis. Objectively: pain in the 21 tooth that is getting worse
the 21, 22, 23, 24, 25, 26 teeth are stable, during biting down. A month ago a part
hard palate is broad, flat. What connecti- of tooth crown broke off as a result of
on of clasps with the base will reduce the a fall. The child didn’t consult a denti-
instability of the resection prosthesis? st. Objectively: in the region of medial
angle of the 21 tooth there is a crown
A. Articular defect that makes up 1/3 of the crown’s
B. Stable height. Tooth cavity is open, probing and
C. Elastic thermal stimulus cause no pain. Percussi-
D. Fixed on is acutely painful. Gum around the 21
E. Does not matter tooth is edematic and hyperaemic. What
is the provisional diagnosis?
149. At a dentist’s appointment a 12-year-
old child inhaled a cotton turunda: he A. Acute condition of chronic periodonti-
developed noisy respiration with a whi- tis
stling sound, sudden dyspnea, pallor along B. Acute condition of chronic pulpitis
with pronounced cyanosis, sweating. What C. Acute serous periodontitis
kind of urgent condition developed in this D. Acute purulent periodontitis
patient? E. Pulpitis complicated by periodontitis
A. Obstructive asphyxia 153. A 12-year-old patient complains
B. Dislocation asphyxia about an aesthetic defect. Objectively: the
C. Valvular asphyxia lower third of face is shortened, upper
D. Anaphylactic shock frontal teeth overbite the lower teeth
E. Collapse by 3/3 of height, exhibit oral inclinati-
on, lateral parts all along exhibit cusp-to-
150. A 39-year-old patient complains of cusp relationship between the antagoni-
experiencing pain in the region of the 21 sts; Angle’s class II malocclusion (joining
tooth for 2 days. It is known from the hi-
story that the indicated tooth had been of the upper permanent molars) is also
present. Malocclusion is observed in the
treated before for caries. Objectively: the
following planes:
21 tooth is covered with metal-ceramic
crown, mucous membrane in apex
projection is edematic and hyperaemic.
Percussion of the tooth is extremely pai-
Krok 2 Stomatology 2012 22

A. In sagittal and vertical 157. During taking of impressions at a


B. In transversal dentist’s appointment a patient presented
C. In transversal and vertical with the following symptoms: paroxysmal
D. In vertical cough, cyanosis, vomiting, clapping sound
E. In sagittal during inspiration. Make the diagnosis:
154. A 52-year-old female patient A. Aspiration of a foreign body
complains of dry mouth, taste impai- B. Attack of bronchial asthma
rment, burning and pricking sensations C. Acute bronchitis
in the tongue that disappear during eati- D. Tracheitis
ng but intensify at the end of the day. For E. Hypersensitive gag reflex
the first time such problems arose 2 years
ago after a psychic trauma. She has a hi- 158. At a dentist’s appointment a pati-
story of anacid gastritis. Objectively: the ent complained of weakness, nausea,
general condition is satisfactory, the pati- blackout, and then he lost consciousness.
ent is restless, tearful. Oral mucosa is pale Make a diagnosis:
pink, dry; filiform papillae on the dorsum
of tongue are reduced. Pharyngeal reflex A. Syncope
is dramatically reduced. There is segmetal B. Shock
disturbance of facial skin sensation. What C. Collapse
is the most likely diagnosis? D. Coma
E. Insult
A. Glossodynia
B. Moller-Hunter glossitis 159. During the preparation of a tooth a
C. Ganglionitis of sublingual ganglions patient had an epileptic seizure. The sei-
D. Chronic atrophic candidous glossitis zure was arrested. What mistake did the
E. Desquamative glossitis orthopaedist make?

155. A 40-year-old patient complains A. Didn’t collect complete history data


about difficult mastication as a result of B. Did not apply one of the types of local
lower jaw displacement. He has a history anesthesia
of mental fracture 2 months ago. Objecti- C. Violated the rules of preparation
vely: the 35, 36, 38 ... 45, 46 teeth are missi- D. Skipped psychological preparation of
ng. Remaining teeth are intact. The 43, 44, the patient
47, 48 teeth have no contact with antagoni- E. Didn’t apply general anaesthesia
sts and their oral deviation makes up 1 cm.
What is the optimal construction of lower 160. A 32-year-old patient needs to
jaw denture? be provided with metal-ceramic crowns
for the 12, 11, 21 and 22 teeth. Duri-
A. Denture with double dentition ng the dental visit he is given infi-
B. Metal-ceramic dental bridge ltration anaesthesia with Ultracain DS
C. Soldered splint on rings anaesthetic. What elements enter into its
D. Removable lamellar denture composition?
E. Adhesive denture
A. 4% articaine with adrenaline
156. The department of maxillofacial B. 2% mepivacaine with adrenaline
surgery admitted a patient who needs C. 4% articaine without a vasoconstrictor
repair of a post-traumatic defect of nose agent
wing up to 3,0 cm in diameter. The trauma D. 3% mepivacaine without a vasoconstri-
occured six months ago. What kind of ctor agent
grafting is indicated in this clinical situati- E. 2% articaine with epinephrine
on?
161. A 9-year-old boy has been diagnosed
A. Grafting with chondrocutaneous flap of with a complete dislocation of the 21
the auricle tooth. The child got injured 20 hours ago.
B. Grafting with local tissues of nasolabial He has diabetes. Select a treatment tacti-
or cheek regions cs:
C. Grafting with pedicle flap of frontal and
buccal regions
D. Grafting with tubed pedicle (Filatov’s)
flap
E. Free grafting with dermal flap
Krok 2 Stomatology 2012 23

A. The tooth cannot be preserved or on the patient will get special medical aid?
reimplanted
B. Reimplantation, fixation of the tooth A. Specialized army surgical hospital
and further follow-up B. Battalion aid station
C. Root apex resection, reimplantation, C. Regimental aid station
fixation of the tooth D. Separate medical detachment
D. Filling of the root canal with amalgam, E. Separate medical battalion
reimplantation
E. Filling of the root canal with paste 166. A 3-year-old child got an injury of the
containing calcium, reimplantation, fixati- upper teeth as a result of a fall. Objecti-
on of the tooth vely: crowns of the 51 and 61 teeth are
deep in the surrounding tissues with only
162. Parents of a 7,5-year old child brought their cutting edge visible, the gingival
him to the dentist for oral cavity sanitati- margin is hyperaemia, edematic. What is
on. Objectively: DEF (for primary teeth) the treatment tactics?
+ DMF (for permanent teeth) index =
4, Green-Vermillion index = 2,5. Fissures A. Tooth extraction
of the first permanent molars are open, B. Monitoring
intact, non-pigmented. What method of C. Reposition
primary prevention of dental caries may D. Endodontic treatment
be appropriate in this case? E. Anti-inflammatory therapy

A. Non-invasive hermetization 167. A 25-year-old patient complains of a


B. Invasive hermetization light brown spot in the upper front tooth.
C. Application of fluoride varnish Objectively: the 23 tooth has a single li-
D. Application of antibacterial varnish ght brown spot in the precervical regi-
E. Application of calcium gels on. Probing shows smooth surface. The
tooth is nonresponsive to cold and probi-
163. A 35-year-old patient complains of a ng. What is the most likely diagnosis?
significant enlargement of the upper lip
and eyelids, which developed within a few A. Chronic initial caries
B. Fluorosis
minutes (during cleaning the house wi- C. Local enamel hypoplasia
th a detergent). Objectively: there is an D. Acute initial caries
edema of the upper part of face, upper lip E. Chronic superficial caries
and eyelids. Palpation is painless. What
disease are these symptoms typical for? 168. A 45-year-old patient complains of a
rapidly growing formation on his lower
A. Angioneurotic Quincke’s edema lip. Examination of the red border of li-
B. Macrocheilitis ps revealed a greyish-red nodule with a
C. Lymphedema hollow in the centre which is filled wi-
D. Melkersson-Rosenthal syndrome th corneous masses that can be easily
E. Glandular cheilitis removed. The nodule is painless, mobi-
164. A 25-year-old HIV-infected patient le. What is your provisional diagnosis?
presented to a clinic of prosthetic denti- A. Keratoacanthoma
stry seeking prosthetic services. What B. Papilloma
aseptic and antiseptic precautions should C. Nodulous verrucous precancer of
be taken? vermilion border
A. According to the scheme D. Basal cell carcinoma
B. The impressions should be desinfected E. Circumscribed precancerous
by means of a quartz lamp hyperkeratosis of vermilion border
C. The patient should be refused appoi- 169. A 35-year-old man complains about
ntment pain in the region of the 38 tooth, painful
D. No special precautions are required deglutition, difficult mouth opening. What
E. The orthopaedist should work in gloves anaesthesia method will be optimal during
and a mask operation on account of pericoronaritis?
165. A 20-year-old patient got an injury.
Objectively: the patient’s chin and lower
jaw up to the 34 and 45 teeth are missing.
The 45, 46, 47, 48, 34, 35, 36, 37 teeth are
stable. At what stage of medical evacuati-
Krok 2 Stomatology 2012 24

A. Conduction Bersche-Dubov’s A. Narrowing of dental arches


anesthesia B. Distal occlusion
B. Infiltration anaesthesia C. Mesial occlusion
C. General anaesthesia D. Widening of dental arches
D. Stem anaesthesia E. Elongation of dental arches
E. Application anaesthesia
173. A 51-year-old lecturer presented to
170. A 42-year-old patient complains the orthopaedic department and complai-
about gingival pain, progressing gingi- ned about painfullness and mobility of his
val haemorrhage, increasing tooth mobi- frontal teeth of the lower jaw. Cervices
lity, halitosis. Objectively: gums are evi- of the 42, 41, 31, 32 teeth are exposed,
dently hyperaemic, extremely edematic, III grade mobility with deep pathologi-
they bleed easily on palpation. Tooth roots cal pockets is present. When would it be
are exposed, parodontal pouches are 4-6 reasonable to fix dentures after dental
mm deep, and contain purulent exudate, extraction?
there is also supragingival and subgingival
dental calculus. II-III grade tooth mobi- A. On the day of dental extraction
lity is present. Orthopantomogram shows B. In 16-30 days
resorption of interdental septa down to C. In 2-3 months
1/2 of their height. What is the most likely D. In 4-6 months
diagnosis? E. In 5-6 days
A. Exacerbation of generalized II degree 174. A 5-year-old child complains of
periodontitis spontaneous pain in an upper jaw tooth on
B. Exacerbation of generalized I degree the right that is getting worse at night and
periodontitis during eating cold food. Objectively: the
C. Exacerbation of generalized III degree 65 tooth has a deep cavity communicating
periodontitis with the tooth cavity. Probing is painful,
D. Chronic generalized II degree peri- percussion is painless. Cold water causes
odontitis long-standing pain. What is your provisi-
E. Chronic generalized III degree peri- onal diagnosis?
odontitis
A. Exacerbation of chronic pulpitis
171. A female patient applied to the B. Acute periodontitis
oral surgery department and underwent C. Exacerbation of chronic periodontitis
radical maxillary sinusotomy with plastic D. Acute serous pulpitis
repair of fistula through the alveolar E. Acute purulent pulpitis
socket of the extrated 27 tooth. Infiltration
and all the peripheral block anaesthesias 175. A 30-year-old patient complains
of the left upper jaw were performed wi- about body temperature rise up to 39, 0o C,
th 6,0 ml of 2% lidocaine solution. 3 mi- a roundish infiltrate on his upper lip,
nutes later the patient registered double general weakness. He has been presenting
vision in her left eye, inability to close with these symptoms for 3 days. Objecti-
it. Which of the performed anaesthesi- vely: a roundish infiltrate in the regi-
as is the reason for the above-mentioned on of the upper lip 2,5 cm in diameter,
presentations? the skin over the infiltrate is red with a
necrotic core in the centre. The upper lip
A. Infraorbital is hyperemic and edematic. What is the
B. Tuberal most likely diagnosis?
C. Palatinal
D. Incisor A. Furuncle of the upper lip
E. Infiltration B. Carbuncle of the upper lip
C. Retention cyst of the upper lip
172. A boy is 10 years old. His face is D. Acute periostitis of the upper jaw
symmetric and proportional. He presents E. Acute glandular abscess
with mouth breath. Examination of the
oral cavity revealed saddle-like shape of 176. A patient complains about
dental arches and high arched palate. paroxysmal upper jaw toothache on
Upper first molar relationship (Angle’s the left that is getting worse at night.
Toothache intensifies also under stimulati-
key to occlusion) remains intact. What is on and irradiates to the left eye and
the most likely diagnosis? temple. Similar attacks were noted three
months ago, the patient didn’t undergo
Krok 2 Stomatology 2012 25

any treatment. Objectively: the 25 tooth fillings. Objective examination revealed:


has a deep carious cavity communicati- index of decayed, missing and filled teeth
ng with the tooth cavity. Probing causes (DMF) = 16, Feodorov-Volodkina hygi-
acute pain at the point of communicati- ene index is 3,3. Choose the optimal
on, vertical percussion is slightly pai- material for carious cavity filling in this
nful, horizontal one is painless. Mucous case:
membrane in the projection of root apex
of the 25 tooth is unchanged, its palpati- A. Glass ionomer cement
on is painless. Thermal probe causes B. Silver amalgam
acute pain, the pain attack is long- C. Chemical-cure composite
lasting. Electroodontodiagnosis is 60 mi- D. Light-cure composite
croampere. X-ray picture shows a slight E. Silicophosphate cement
widening of periodontal fissure at the root
apex of the 25 tooth. What is the most li- 180. A patient with a missile wound of
kely diagnosis? face was delivered to the station of dental
aid group that was organized by order
A. Exacerbation of chronic pulpitis of medical corps commander. What kind
B. Acute generalized pulpitis of aid can be rendered by the dental aid
C. Acute purulent pulpitis group?
D. Acute purulent periodontitis
E. Exacerbation of chronic periodontitis A. Secondary care
B. Professional care
177. A 48-year-old woman complains of C. Consultancy
aching dull pain in the region of the left D. Dental health service
TMJ, that is getting worse during eating E. Dental orthopaedic service
solid food. The pain appeared about 2,5
years ago. Objectively: mouth opening is 181. A 62-year-old patient has a median
limited, there is sideward deviation of jaw fracture of mandible along with formati-
during mouth opening, TMJ is clicking. on of a false joint. Objectively: dental
Examination of the oral cavity revealed formula is 33, 34, 35, 36, 27, 47, 46, 45, 44,
secondary partial adentia. X-ray picture 43. The teeth are intact, stable, with high
shows sclerosis of the cortical plate of crowns. Fragment mobility is insignificant,
articulat head and narrowing of cartilage there is no displacement. X-ray picture
space. What is the most likely diagnosis? shows a bone defect 0,8 cm large. What
prosthesis is indicated?
A. Arthrosis of the TMJ
B. Chronic arthritis of the TMJ A. Bridge-like prosthesis with a pivot point
C. Acute arthritis of the TMJ B. Lamellar prosthesis without a pivot
D. Painful dysfunction of the TMJ point
E. Exacerbation of chronic arthritis of the C. Lamellar prosthesis with Gavrilow’s
TMJ pivot point
D. Lamellar prosthesis with Oxman’s pivot
178. An oral surgeon attended a 3- point
day-old child staying in the newborn E. Lamellar prosthesis with Weinstein’s
pathology department. Objectively: bi- pivot point
lateral hypogenesis of zygomatic bones
and orbits, antimongoloid slant of 182. After supercooling a 42-year-old pati-
palpebral fissures, nonclosure of inferi- ent presented with headache in the left
or eyelids, hypogenesis of mandible (bird frontal region and left upper jaw. Objecti-
face), deformity of auricles with a preauri- vely: the face is symmetric, breathi-
ng through the left nasal meatus is
cular fistula on the right. What congenital
disease does the child have? obstructed, seropurulent discharges are
present. Palpation is slightly painful in
A. Mandibulofacial dysostosis the infraorbital region as well as along
B. Oculocerebrorenal syndrome the mucogingival fold in projection of the
C. Oculoauriculovertebral dysplasia 24, 25 teeth. Percussion of these teeth is
D. Oculomandibulofacial syndrome painless. The 24 tooth is filled. Mucuous
E. Oculodentodigital syndrome membrane of alveolar process has no visi-
ble changes. X-ray picture shows reduced
179. A pregnant 24-year-old woman pneumatization of the left upper jaw si-
complains about emergence of several nus. What is the provisional diagnosis?
new carious cavities, falling out of old
Krok 2 Stomatology 2012 26

A. Exacerbation of chronic odontogenic A. Acute purulent pulpitis


maxillary sinusitis B. Acute diffuse pulpitis
B. Acute periodontitis of the 24 tooth C. Acute focal pulpitis
C. Exacerbation of chronic periodontitis of D. Acute periodontitis
the 24 tooth E. Exacerbation of chronic periodontitis
D. Acute rhinogenous maxillitis
E. Acute albuminous periostitis of the left 186. A 9-year-old boy presents with
upper jaw face asymmetry due to the chin di-
splacement to the left. When the thi-
183. An 8,5-year-old child is apparently rd Il’ina-Marcosian diagnostic test is
healthy. The child complains of pain in performed, face asymmetry disappears.
an upper tooth on the left caused by a What is the most likely clinical form of
traumatic injury sustained three hours this occlusal abnormality?
ago. Objectively: the crown part of the 21
tooth is destroyed by 1/2, the pulp is red A. Habitual displacement of mandible
and significantly exposed, probing causes B. Ankylosis of the temporomandibular
acute pain and bleeding. Percussion of the joint
21 tooth is extremely painful. Choose the C. Unilateral hypoplasia of mandible
most efficient treatment method of the 21 D. Bilateral narrowing of the maxillary
tooth: dental arch
E. Unilateral narrowing of the maxillary
A. Vital amputation dental arch
B. Vital extirpation
C. Devital amputation 187. A 65-year-old male patient complai-
D. Devital extirpation ns of crepitation and clicking in both
E. Bioassay technique temporomandibular joints, pain induced
by displacement of the lower jaw to the
184. A 23-year-old patient complains of right, ear noise, dry mouth, glossalgia.
root exposition, gingival haemorrhage He has been using complete removable
during tooth brushing, gum itch. Objecti- prosthesis of the lower jaw for 6 months.
vely: there is supragingival and subgingi- The patient denies having rheumatosis.
val dental calculus. Gums are hyperaemic, Objectively: the lower third of face
edematic, pockets are 3,5 mm deep. X-ray is shortened, mental fold is strongly
picture shows resorption of interalveolar pronounced, mouth corners are lowered,
septa by 1/3. What is the most likely di- there are angular fissures and cracks.
agnosis? Palpation reveals crepitatnt rale observed
during TMJ moving. What is the most li-
A. Chronic generalized I degree peri- kely diagnosis?
odontitis
B. Chronic generalized II degree peri- A. Costen’s syndrome
odontitis B. Temporomandibular arthritis
C. Exacerbation of generalized I degree C. Temporomandibular arthrosis
periodontitis D. Temporomandibular dislocation
D. Exacerbation of generalized II degree E. Temporomandibular osteoarthritis
periodontitis
E. II degree periodontitis 188. A 52-year-old patient complains of
teeth mobility, gingival haemorrhage. The
185. A 14-year-old child complains about 44, 45, 34, 35 teeth exhibit I grade mobi-
acute spontaneous pain in an upper jaw lity, the 43, 42, 41, 31, 32, 33 teeth exhi-
tooth on the right. The pain has been bit II grade mobility. There is gingival
lasting for 3 days, it is throbbing, irradi- edema, peridontal pockets are 5-6 mm
ating to the temple, getting worse at ni- deep. X-ray picture shows destruction of
ght. Objectively: surface of the 15 tooth bone tissue by 1/2 of root length. The
exhibits a carious cavity within parapulpar patient has been diagnosed with generali-
dentine. Dentine is softened, of greyish zed periodontitis of median severity. What
colour. Probing of the whole cavity floor is orthopaedic construction is most efficient
painful, percussion of the 15 tooth is pai- in this situation?
nless. What is the most likely diagnosis?
Krok 2 Stomatology 2012 27

A. Elbrecht splint these processes?


B. Plastic mouthguard
C. Murray’s splint A. Vishnevsky ointment, methyluracil
D. Mamlock splint ointment, solcoseryl gel
E. Temporary Novotny splint B. Laevosin, luronitum, trypsin ointments
C. Levomecol, laevosin ointments etc.
189. It is planned to lance an abscess D. Trypisin, chymotrypsin, terrilytin etc.
of hard palate (it is localized close to E. 0,5-1% dioxydin solution, 0,1-0,2%
the 23, 24, 25 teeth) and to extract the chlorhexidine solution
causative 24 tooth that had caused abscess
development. What type of anaesthesia is 193. While conducting experiments in
indicated for this operation? chemistry lesson a 14 year-old child got
a traumatic shock of face. Objectively: the
A. Infraorbital, palatinal and incisor skin dehydrated in the affected region. It
B. Tuberal, infraorbital and incisor is covered with thick dry crust with clear
C. Tuberal, infraorbital and palatinal boundaries and peripheral hyperaemia,
D. Tuberal and incisor the crust is recessed into the skin. What is
E. Tuberal and palatinal the most likely traumatic agent?
190. A 23-year-old patient complains A. Acid
about unpleasant sensations in the regi- B. Alkaline
on of the 12 tooth, X-ray picture shows a C. Radiation
well-defined low-density area up to 8-10 D. Thermal
mm in diameter around the root apex of E. Saline
the 12 tooth. What method of surgery will
be the most efficient? 194. A 7-year-old child has to undergo
plastic surgery of the upper lip frenulum.
A. Root apex resection What operation should be performed to
B. Root amputation lengthen the frenulum of the upper lip?
C. Hemisection
D. Tooth removal A. Dieffenbach grafting
E. Cystotomy B. Thiersch grafting with local flaps
C. Relaxing incisions of the mucosa
191. Examination of a 6-year-old boy D. Grafting with a pedicle flap
revealed enlarged lymph nodes in both E. Szymanowsky grafting with local flaps
submandibular and cervical regions.
Objectively: the 75, 84 and 85 teeth 195. The 47, 46, 45, 35, 36, 37, 38 teeth
are decayed, there are presentations of of a 57-year-old patient are missing. It
commissural cheilitis. According to the is planned to make a clasp denture. The
boy’s mother, he has been quickly getti- 48 tooth inclines to the lingual side and
ng tired, sweating from the least physi- forwards. On the lingual side of the 48
cal exercise, complaining about weakness tooth the border line is diagonal, on the
throughout the last 2-3 months. He also buccal side it runs on a level with gingival
gave up training in a sports class. What edge. What type of Ney’s clasp should be
plan of additional examination should be applied?
adopted?
A. V type clasp
A. Complete blood count, haematologist B. I type clasp
consultation C. IV type clasp (reverse back-action)
B. Biochemical blood test, endocrinologist D. I-II type clasp
consultation E. II type clasp
C. Puncture biopsy, oncologist consultation
D. Magnetic resonance tomography, 196. A patient complains about worsened
immunologist consultation fixation and frequent breakages of parti-
E. Pulmonary roentgenography, al removable lamellar denture for the
pulmonologist consultation lower jaw that has been in use for
5 years. Objectively: alveolar part in
192. A 12-year-old girl undergoes edentulous regions is significantly atrophi-
treatment at the maxillofacial department ed, the denture balances. What is the most
for nonodontogenic abscess of the left probable cause of worsened fixation and
submandibular region. Postoperative frequent breakages of the denture?
wound in the stage of granulation and epi-
thelialization. What medications speed up
Krok 2 Stomatology 2012 28

A. Alveolar process atrophy A. Radiography of mandible in the frontal


B. Improper care of denture and lateral projections
C. Denture using during sleep B. Radiography of skull in axillary projecti-
D. Consumption of solid food on
E. Wear of artificial teeth C. Radiography of skull in the frontal
projection and of mandible in Parma
197. A 15-year-old patient has got a projection
trauma. He complains of mandibular pain D. Tomogram of mandible
getting worse during swallowing, chewing, E. Radiography of skull in the axillary
especially mouth opening. Objectively: projection
face configuration isabnormal, there is a
large haematoma in the region of the ri- 199. A 73-year-old patient consults a
ght mandibular angle. Palpation of this dental orthopaedist about total edentia.
region is painful. The mouth is half open, Objectively: there is insignificant regular
lower lip frenulum is shifted to the right atrophy of the alveolar process of
of the central line. Pressing upon the chin mandible, the process is covered with
causes pain in the part of mandible on the mucous membrane of moderate elasticity.
right. What is the provisional diagnosis? According to Keller’s classification, such
manifestations are characteristic for the
A. Fracture of mandibular angle on the folowing type of edentulous jaw:
right
B. Bilateral fracture of mandibular A. First
branches B. Third
C. Posterior unilateral dislocation of C. Fifth
mandible D. Second
D. Anterior unilateral dislocation of E. Fourth
mandible
E. Bilateral fracture of articular processes 200. Intraoral examination of a 5-year-old
child revealed primary occlusion, tremas
198. A 12-year-old boy has been injured. and diastemas, worn tubercles and cutti-
In the region of the 44 and 45 teeth there is ng surfaces of teeth. The distal surfaces
pathological displacement of the alveolar of the second lower molars are anterior
process and the body of the mandible, to the distal surfaces of the second upper
rupture of the mucous membrane of the molars. This stage of primary occlusion is
alveolar process. What additional tests called:
should be done to specify the diagnosis?
A. Aging
B. Stable occlusion
C. Formation
D. Eruption
E. There is no correct answer
Krok 2 Stomatology 2013 1

1. After examination a 5-year-old child A. 15


was diagnosed with acute total pulpitis of B. 10
the 74 tooth. What treatment will be most C. 6-8
efficient in this case? D. 4
E. 20-25
A. Pulp extirpation
B. Vital amputation 6. A 34-year-old patient had got in a
C. Non-vital amputation car accident. The patient stayed consci-
D. Biological method ous. He complains of headache, dizzi-
E. Tooth extraction ness, general weakness, nausea. 12 hours
after the injury the patient got "raccoon
2. Examination of a 9-year-old child eyes"(periorbital haematomas) within the
revealed chalky spots in the cervical part limits of the circular muscle of eye. What
of the 12, 11, 21, 22 teeth. The spots fracture does the victim have?
appeared two weeks ago. Their surface
is dull in appearance and can be stained A. Fracture of skull base
with methylene blue. The affected teeth B. Le Fort II fracture of maxilla
are nonresponsive to cold stimulus. What C. Fracture of nasal bones
is the most likely diagnosis? D. Bilateral fracture of zygomatic bones
E. Fracture of frontal bones
A. White spot stage of caries
B. Fluorosis 7. After a car accident a patient consulted
C. Systemic hypoplasia a dentist about pain at the base of the
D. Surface caries nose. Palpation reveals tenderness of the
E. Focal hypoplasia nose wall, mobility of the nasal bones.
Nasal passages are filled with blood clots.
3. A 54-year-old patient complains of a What is the provisional diagnosis?
tumour-like formation in the region of
the lower lip which appeared 1-1,5 months A. Fracture of nasal bones
ago. Objectively: on the vermilion border B. Le Fort I fracture of maxilla
of the lower lip on the right there is a C. Le Fort II fracture of maxilla
roundish tumour up to 1 cm in diameter D. Le Fort III fracture of maxilla
with raised rolled edges. The tumour is E. Fracture of cheekbone
painless, the regional lymph nodes are not
enlarged. What is a provisional diagnosis? 8. A 47-year-old female patient complai-
ns of inability to eat hot and cold food,
A. Keratoacanthoma as well as of intense pain caused by sour,
B. Carcinoma of the lower lip sweet and salty food. Objectively: there is
C. Papilloma of the lower lip a slight loss of enamel on the molars and
D. Fibroma of the lower lip incisors. Probing and cold test cause acute
E. Verrucous precancer pain. What is the most likely diagnosis?
4. A patient complains of spontaneous, A. Hyperesthesia of dental hard tissues
paroxysmal, irradiating pain with short B. Enamel erosion
pain-free intervals. The pain arose 2 days C. Enamel necrosis
ago and occurs only at night. Make a D. Enamel hypoplasia
provisional diagnosis: E. Pathological abrasion of dental hard
tissues
A. Acute diffuse pulpitis
B. Acute deep caries 9. A 43-year-old patient complains of a
C. Exacerbation of chronic periodontitis tumour in the right submandibular regi-
D. Acute circumscribed pulpitis on which appeared two months ago after
E. Acute purulent pulpitis having flu. Objectively: palpation of the
right submandibular region reveals a pai-
5. An orthopedist plans to fabricate a nful spherical neoplasm of dense elastic
porcelain-fused-to-metal crown for the consistency with regular outlines which is
23 tooth. In order to correct its abnormal not attached to the skin. Submandibular
position in the dental arch the orthopedi- salivary gland secretes transparent saliva.
st is going to use a cast post and core. Sublingual plica is unchanged. Which di-
What is the maximum permissible value sease are these clinical presentations typi-
(in degrees) of the core deviation from cal for?
the tooth axis?
Krok 2 Stomatology 2013 2

A. Chronic lymphadenitis A. Telescopic crowns


B. Chronic sialoadenitis B. Equator crowns
C. Sialolithiasis C. Full cast crowns
D. Adenoma of the salivary gland D. Plastic crowns
E. Lipoma E. Portion crowns
10. A 40-year-old patient complains of 14. A 64-year-old patient presents with
pain caused by cold and sweet food in the bleeding from the left nostril, mobility of
11, 21 teeth, as well as of a cosmetic defect. the upper left teeth. For the last 2 years
Examination of the 11, 21 teeth revealed the patient has had dull pain in the left
some oval, diagonally arranged enamel side of maxilla. On the left side of neck
defects on the convex part of the vesti- there is a dense tumour-like formation.
bular surface of tooth crowns. The defects The patient is exhausted. What is the most
had smooth, glossy, dense floor. The pati- likely diagnosis?
ent has a history of thyrotoxicosis. What
is the most likely provisional diagnosis? A. Left-sided maxillary carcinoma
B. Left-sided sinusitis
A. Erosion of dental hard tissues C. Left-sided maxillary cyst
B. Wedge-shaped defect D. Chronic osteomyelitis of the left upper
C. Surface caries jaw
D. Enamel necrosis E. Osteoclastoma of the left upper jaw
E. Amelogenesis imperfecta
15. A patient consulted a dental surgeon
11. A 36-year-old patient complains of about fever up to 37, 6oC, soft tissue
pain under the dental bridge. After its swelling, pain in the 47 tooth on the
removal the patient has been found to lower jaw on the left, the pain is getti-
have an ulcer 0,3x0,5 cm large on the ng worse when the tooth is touched by
alveolar process. The ulcer is slightly pai- the antagonist. Objectively: mucogingival
nful and soft, the surrounding mucosa is fold of the alveolar process is hyperemic
hyperaemic, submandibular lymph nodes and flat on the vestibular side of the
are not enlarged. What is a provisional di- decayed 47 tooth. What is the most likely
agnosis? diagnosis?
A. Decubital ulcer A. Acute purulent odontogenic periostitis
B. Trophic ulcer B. Acute serous periodontitis
C. Sutton aphtha C. Exacerbation of chronic periodontitis
D. Cancerous ulcer D. Acute odontogenic osteomyelitis
E. Tuberculous ulcer E. Periodontal abscess
12. A 42-year-old female patient complai- 16. A 40-year-old patient complains of
ns of tooth mobility, difficult masticati- constant intense throbbing pain in the
on. Objectively: face configuration is 23 tooth lasting for 3 days. This problem
unchanged. The 35, 36, 38, 44, 46, 48 teeth hasn’t bothered him before. Vertical and
are missing. The 31, 32, 41, 42 teeth exhi- horizontal percussion is positive, the tooth
bit 1-2 grade mobility. What is the most is mobile, mucous membrane around the
efficient denture construction in this case? 26 tooth is hyperemic and edematous.
Palpation of the mucogingival fold in the
A. Whole-piece splint with vestibular root apex projection is painful. What is
clowlike hooks the most likely diagnosis?
B. Whole-piece splint with Roach clasps
and a multijoint clasp A. Acute purulent periodontitis of the 26
C. Mamlok splint tooth
D. Removable plastic splint B. Acute purulent pulpitis of the 26 tooth
E. Interdental splint C. Acute localized periodontitis of the 26
tooth
13. A 65-year-old patient needs prosthetic D. Acute serous pulpitis of the 26 tooth
services. Objectively: the 43, 33 teeth are E. Acute serous periodontitis of the 26
decayed by 2/3. EPT result is 6 mA. Other tooth
teeth in the lower jaw are missing. What
crowns should be fabricated for fixing the 17. A 48-year-old female patient had
removable denture? been delivered to a hospital with a cheek
wound and severe headache, nausea, di-
zziness. It is known from the history that
Krok 2 Stomatology 2013 3

the patient had got a trauma as a result A. Milikevich


of a fall. After examination she was di- B. Kopeikin
agnosed with a contused lacerated wound C. Shcherbakov
of cheek, closed craniocerebral injury, D. Kurlyandsky
brain concussion. This trauma can be E. Gavrilov
qualified as:
21. A 26-year-old patient needs prostheti-
A. Concominant trauma cs. Objectively: crown of the 16 tooth is
B. Isolated trauma decayed by 1/3. It is planned to restore its
C. Combined trauma anatomical shape with a metal inlay. What
D. Polytrauma is the first stage of making a cavity for the
E. Single inlay?
18. A 12-year-old boy complains of fever A. Removal of affected tissues
up to 38 0C, weakness, headache, pain B. Making additional cavities
in the mouth, presence of vesicles and C. Making a bevel
ulcers. The acute condition developed D. Making the cavity floor
three days ago. The patient has a hi- E. Making the cavity walls
story of recent pneumonia treated with
antibiotics. Objectively: oral mucosa is 22. A 19-year-old patient needs a plastic
hyperemic and edematous. The mucosa of crown for the 22 tooth. The orthopedist
lips, tongue and cheeks has large erosions has already taken working and auxiliary
covered with fibrinous pellicle. The lips impressions. What technological process
are covered with thick brown crusts. The should be next?
back of the hand has papules of double- A. Moulding of a plaster model
contour colour. Which of the listed agents B. Making a wax crown model
should be primarily used in the topical C. Investment of plastered model into a
treatment? cuvette
A. Painkillers D. Making a plastic crown model
B. Antiinflammatory E. Processing, polishing of the crown
C. Antiviral 23. A 25-year-old patient complains of
D. Antimicrobial heavy gingival haemorrhages, pain in the
E. Antifungal oral cavity, weakness, fatigue, fever up
19. A 29-year-old patient complains of to 38o C. These presentations appeared
decay of the 21 tooth, an aesthetic defect. a week ago. Objectively: the patient is
Objectively: on the medial surface of the pale, adynamic. Examination of the oral
21 tooth there is a carious cavity, the cutti- mucosa reveals multiple haemorrhages,
ng edge is partly decayed. Specify the class friable gums, necrotic areas on the tops of
of this cavity according to Black’s classifi- gingival papillae, as well as enlarged, soft,
cation: painless lymph nodes. The oral mucosal
lesion can be a symptom of the following
A. Class IV disease:
B. Class I
C. Class II A. Acute leukemia
D. Class III B. Chronic leukemia
E. Class V C. Vincent stomatitis
D. Hypovitaminosis C
20. A 20-year-old patient seeks dental E. Intoxication with heavy metal salts
prosthetics. Objectively: on the occlusal
surface of the 16 tooth there is a cavi- 24. A 67-year-old patient consulted an
ty, two walls of the tooth crown are orthodontist about missing of the 34, 35,
intact. Method of orthopedic treatment is 36, 37, 45, 46 teeth. Orthopantomogram
determined by the degree of destruction of the remaining teeth shows the alveolar
of the occlusal surface of teeth (IROPZ- bone resorption in the range of 1/3 of the
interdental septa height. What constructi-
index). Who suggested its definition? on should be offered the patient?
Krok 2 Stomatology 2013 4

A. Clasp prosthesis with splinting elements due to the injury of the alveolar process
B. Bridges on the posterior teeth mucosa of maxilla. Objectively: at a level
C. Removable partial lamellar denture of the mucogingival fold of the 14, 15 teeth
D. Cap splint on the vestibular side there is a decubi-
E. Van Thiel splint tal ulcer sized 1x3 mm with smooth pink
edges. What is the most likely diagnosis?
25. During an armed conflict soldier K.
was injured and delivered to the station A. Traumatic stomatitis
of the separate medical battalion. The B. Candidiasis
patient has a gunshot wound of his ri- C. Allergic stomatitis
ght cheek, splintered fracture of the body D. Tuberculous ulcer
of mandible, arterial bleeding from the E. Syphilitic ulcer
wound. A brigade of the separate medi-
cal battalion performed surgical debri- 29. A 50-year-old patient complains of mi-
dement and final bleeding arrest. Specify ssing of the 37, 36, 45, 46, 47 teeth. The
the scope of further medical and evacuati- 35, 38, 44, 48 teeth are intact, stable, wi-
on actions for this patient: th anatomically shaped crowns, rather
high, relatively parallel to each other.
A. Transport immobilization of mandible Boundary line has a typical direction
fragments, evacuation of the patient to the along the 35, 44 abutment teeth. What
military field surgical hospital fixation element should be used in a clasp
B. Therapeutic immobilization of mandi- prosthesis supported by the 35, 44 teeth?
ble fragments, evacuation to the military
field hospital for minor casualties A. Akers’ clasp
C. Transport immobilization of mandible B. Roach clasp
fragments, evacuation to the military field C. Jackson clasp
hospital for minor casualties D. Telescopic fixation
D. Transport immobilization of mandible E. Wrought wire clasp
fragments, no need of further evacuation
E. No need of fragment immobilization, 30. A 23-year-old serviceman needs
evacuation to the military field hospital for orthopedic treatment in a specialized
minor casualties hospital. The patient has been diagnosed
with false joint in the anterior lower jaw.
26. Preventive examination of a 4,5-year- The teeth are intact, stable, 3 on each side.
old child revealed some hidden cavities on Orthopedic treatment can be carried out
the contact surfaces of the 54 and 55 teeth. using a bridge prosthesis, if the jaw defect
After removal of the overhanging edges is no bigger than:
of the enamel the softened dentin could
be easily removed with an excavator wi- A. 1 cm
thin the mantle dentin. Select the optimal B. 3 cm
material for a permanent filling: C. 3,5 cm
D. 2 cm
A. Compomer material E. 2,5 cm
B. Composite material
C. Silicate cement 31. A 22-year-old patient complains of a
D. Silicophosphate cement painful swelling in the right parotid gland.
E. Polycarboxylate cement A week earlier the patient got a cheek
abrasion which healed under the purulent
27. An 18-year-old student needs crust. Over the past two days the patient
prosthetic porcelain-fused-to-metal had observed progressing pain and fever
denture for the 11, 21 teeth. There are up to 38, 6o C. Objectively: there is a soft
no contraindications for the use of such tissue edema in the right parotid region,
construction. What is the most appropri- the skin is slightly strained but has not
ate material for taking impressions? changed in colour. There is a dense painful
infiltration 2,5x3,5 cm large, the skin over
A. Sielast it exhibits limited mobility. The mouth can
B. Stomalgin be fully opened, the mucous membrane
C. Orthocor around the orifice of the salivary duct is
D. Stens unchanged, the saliva is transparent. What
E. Repin is the most likely diagnosis?
28. A 40-year-old patient complains of
experiencing eating difficulties for 3 days
Krok 2 Stomatology 2013 5

A. Acute lymphadenitis a bone tissue defect up to 1 cm large. What


B. Exacerbation of chronic parotitis prosthesis is indicated?
C. Abscess of the parotid-masseteric regi-
on A. Oxman’s bridge-like prosthesis with
D. Acute non-epidemic parotitis pivot point
E. Epidemic parotitis B. Clasp denture
C. Lamellar prosthesis with Gavrilow’s
32. A 33-year-old female patient has been pivot point
admitted to the maxillofacial department D. Lamellar prosthesis with Oxman’s pivot
for pain and edema of the right submandi- point
bular region, body temperature rise up E. Bridge-like prosthesis without a pivot
to 39, 5oC. Objectively: the patient has point
asymmetric face because of soft tissue
edema of the right submandibular region, 36. During the examinations and everyday
palpation reveals a dense infiltration, the orthopaedic manipulations a dentist uses
skin over it is hyperemic, does not make a a dental mirror. What is the way of its
fold. The 46 tooth has a deep carious cavi- sterilization?
ty. What is the most likely diagnosis?
A. In the triple solution for 30 minutes
A. Submandibular phlegmon on the right B. In the 0,5% ethyl chloride solution for
B. Acute submandibular sialoadenitis 20 minutes
C. Acute purulent periostitis of mandible C. In the 6% hydrogen peroxide solution
D. Acute purulent submandibular for 6 hours
lymphadenitis D. In the dry heat sterilizer at 180o C for 10
E. Acute right-sided osteomyelitis of minutes
mandible E. In the 0,01% chloramine solution for 10
minutes
33. Biopsy material obtained by puncture
of a maxillary tumour looks like a foul- 37. A 59-year-old patient presented to a
smelling mush. What kind of tumour is it clinic for prosthetics. Three months ago
typical for? he was treated for the fracture of the body
of mandible on the left. The 18, 27, 28, 38,
A. Cholesteatoma 32, 31, 41, 42, 43, 44, 45, 47, 48 teeth are mi-
B. Radicular cyst ssing. The patient has left-sided cross-bite
C. Osteoclastoma with lateral occlusal contact. What tactics
D. Ameloblastoma should be chosen?
E. Atheroma
A. Removable denture with double denti-
34. A 29-year-old female patient complai- tion
ns of peeling, dryness and burning of the B. Selective teeth grinding followed by
lower lip. Objectively: on the vermilion fixed prosthetics
border of lip there are multiple gray scales C. Orthopedic treatment
with tightly attached center and peeling D. Implant-supported fixed denture
off edges. The scales are found all across E. Removable denture with Kemeny clasps
the mouth and from the wet-dry line to the
middle of the lip. The skin is not affected. 38. A 32-year-old patient presented to a
Scales removal doesn’t lead to erosions. prosthetic dentistry clinic with a diagnosis
The patient has the lability of psycho- of maxillary fracture.The fracture can be
emotional sphere. What is the most likely treated by means of standard maxillo-
diagnosis? mandibular Zbarzh set. Which constructi-
on provides the intra-oral fixation?
A. Exfoliative cheilitis, dry form
B. Exfoliative cheilitis, exudative form A. Standard double arch
C. Meteorological cheilitis B. Aluminium arch bar
D. Eczematous cheilitis C. Plastic biteplate
E. Actinic cheilitis D. Weber’s splint
E. Crown-supported soldered splint
35. A 50-year-old patient has median
lower jaw fracture with formation of a 39. A 35-year-old patient complains
false joint. The 38, 32, 31, 41, 42, 48 about itch, burning and edema of lips.
teeth are missing. The remaining teeth are These presentations occured a week ago.
intact, stable. There is no displacement of Objectively: there is reddening of vermi-
lower jaw fragments. X-ray picture shows lion border and skin, especially in the
Krok 2 Stomatology 2013 6

region of mouth corners, there are also pain in the 26 tooth that is getting worse
vesicles, crusts, small cracks along wi- when biting down on food. 4 days ago
th erythematous affection of vermilion arsenic paste was applied, but the pati-
border. What is the most likely diagnosis? ent did not keep the appointment with
the dentist. Objectively: the dressing of
A. Acute eczematous cheilitis the 26 tooth is kept. There is pain reaction
B. Multiform exudative erythema to percussion. Ro-gram shows no changes
C. Acute herpetic cheilitis in the periapical tissues. After mechani-
D. Allergic contact cheilitis cal and pharmacological treatment of the
E. Exudative form of exfoliative cheilitis root canals the gauze pads are moist, not
stained. What drug substance must be left
40. A 44-year-old male patient complains in the root canals for the maximal clinical
of fatigue and headache, limb numbness, effect?
dry mouth, burning and pain in the
tongue. Objectively: skin and oral mucosa A. Unitiol
are pale. There are painful cracks in the B. Chlorhexidine
corners of mouth. Dorsum of tongue C. Chloramine
is smooth, glossy, with bright red stri- D. Hydrogen peroxide
ae. Blood count: Hb- 70 g/l, RBCs - E. Trypsin
1, 5 · 1012 /l, colour index - 1,6, leukopenia,
thrombocytopenia, lymphocytosis. What 44. A 14-year-old boy complains of rash
is the most likely diagnosis? on the lips, pain while talking and eati-
ng. These presentations showed up three
A. Addison-Biermer anemia days ago. Similar rash has appeared 1-4
B. Chronic posthaemorrhagic anemia times a year for three years. Objectively:
C. Late chlorosis general condition is satisfactory, the body
D. Iron deficiency anemia temperature is of 36, 9oC. On the vermi-
E. Aplastic anemia lion border of the lower lip and the skin
below there are multiple small grouped
41. Mother of a 3-year-old child consulted vesicles with serous content, and crusts.
a dentist about discolouration and abrasi- What is the etiology of the disease?
on of the child’s teeth. The child has a hi-
story of enamel spalling shortly after the A. Herpes simplex virus
tooth eruption. Objectively: the crowns of B. Coxsackie virus
all the teeth are worn by nearly a half and C. Streptococci
have yellow-gray colour. Make a provisi- D. Herpes zoster Virus
onal diagnosis: E. Staphylococci
A. Stainton-Capdepont syndrome 45. A 28-year-old patient hadan appoi-
B. Amelogenesis imperfecta ntment with a dental surgeon to prepare
C. Dentinogenesis imperfecta the oral cavity for a prosthesis. Examinati-
D. Osteogenesis imperfecta on revealed that the 11 tooth crown was
E. Systemic enamel hypoplasia decayed, the root was stable, its percussi-
on was painless. The mucous membrane
42. A 78-year-old patient is completely of the alveolar process was not changed.
edentulous. He has been wearing dentures Ro-gram of the alveolar process shows
for 19 years. The patient complains widening of periodontal ligament space
of poor fixation of the upper denture. in the region of the 11 tooth. Alveolar
Objectively: the lower third of face is process structure is not changed. The root
shortened, the alveolar processes of both canal was passable all through. What is
jaws are markedly atrophied, the palate the most likely diagnosis?
is flat. Mucous membrane in the denture-
supporting area is atrophied. How often A. Chronic fibrous periodontitis of the 11
should the dentures be remodelled or tooth
restored? B. Chronic granulating periodontitis of the
11 tooth
A. Every 3-4 years C. Chronic granulomatous periodontitis of
B. Every 6 months the 11 tooth
C. Once a year D. Periodontitis of the 11 tooth
D. Every 7 years E. Chronic periodontitis of the 11 tooth,
E. Every 10-12 years hypercementosis
43. A male patient complains of acute 46. A 44-year-old patient consulted a
Krok 2 Stomatology 2013 7

dental surgeon about constant acute pain nful and accompanied by bleeding. The
in the upper jaw region on the left that is tooth percussion provokes acute pain.
getting worse during teeth joining. The Mucosa is hyperemic, edematic and pai-
pain appeared 3 days ago. Objectively: nful. Palpation in the region of the 61, 62
the face is symmetric, mouth opening is teeth reveals a fistula. What is your provi-
not limited. The crown of the 26 tooth is sional diagnosis?
half-decayed. Probing of the carious cavi-
ty is painless. Percussion of the 26 tooth A. Exacerbation of chronic periodontitis
provokes acute pain. Mucous membrane B. Acute purulent periodontitis
of the alveolar process is edematic, C. Acute diffuse pulpitis
hyperaemic at the level of the 26 tooth. D. Chronic granulating periodontitis
The 26 tooth had been treated before. E. Exacerbation of chronic pulpitis
What is your provisional diagnosis?
50. A 30-year-old patient complains of
A. Exacerbation of chronic periodontitis toothache caused by hot and cold sti-
of the 26 tooth muli. The pain irradiates to the ear and
B. Acute purulent periodontitis of the 26 temple. Previously there was spontaneous
tooth nocturnal toothache. Objectively: on the
C. Acute pulpitis of the 26 tooth occlusal surface of the 37 tooth there is a
D. Acute purulent periostitis of the left deep carious cavity communicating at one
upper jaw extending from the 26 tooth point with the tooth cavity. Probing at the
E. Periodontitis of the 26, 27, 28 teeth communication point, as well as cold sti-
mulus, cause acute pain. The pain persists
47. A 45-year-old patient came to a cli- for a long time. Electric pulp test result is
nic for prosthetics. Objectively: the 47, 46, 5 mA. What is the most likely diagnosis?
45, 35, 36, 37 teeth are missing. There is
dento-alveolar vertical displacement of A. Exacerbation of chronic pulpitis
the 17, 16, 26, 27 teeth, alveolar process B. Acute diffuse pulpitis
is enlarged, tooth necks are not exposed. C. Exacerbation of chronic periodontitis
When the teeth are joined, the distance D. Chronic concrementous pulpitis
between the occlusal surfaces of teeth and E. Acute purulent pulpitis
alveolar process is about 3 mm. What is
the most efficient method of secondary 51. A 40-year-old patient complains of
deformation treatment in this patient? discomfort in the 24 tooth. He has a hi-
story of periodical swelling in the region
A. Orthodontic and surgical of the affected tooth. Objectively: the 24
B. Surgical tooth cavity is wide open. Probing and
C. Selective grinding percussion are painless. There is positive
D. Deocclusion vasoparesis symptom. Radiograph shows
E. Tooth pulp removal and selective gri- bone destruction with indistinct outlines.
nding What is the most likely diagnosis?
48. A 13-year-old child complains of a A. Chronic granulating periodontitis
short-term pain caused by cold stimuli B. Chronic fibrous periodontitis
in the upper left molar tooth. Occlusal C. Chronic gangrenous pulpitis
surface of the 27 tooth has a cavity with a D. Chronic granulomatous periodontitis
small hole. After the overhanging enamel E. Chronic fibrous pulpitis
had been removed, the soft light dentin
could be easily removed with an excavator 52. Floating obturators (Case’s,
within mantle dentin. What is the most li- Chasovskaya’s etc.) for the defects in
kely diagnosis? the hard and soft palate are fabri-
cated according to impressions obtained
A. Acute median caries by means of S-shaped spatula. Which
B. Chronic median caries impression material is used in this case?
C. Acute deep caries
D. Acute surface caries A. Stens
E. Chronic surface caries B. Orthocor
C. Stomalgin
49. Parents of a 3-year-old child report D. Dentafol
that the child has constant pain in the E. Plaster
upper front teeth. Objectively: the coronal
part of the 61 tooth is gray and decayed. 53. A 43-year-old patient was found to
Probing of the root canal orifice is pai- have no crown of the 13 tooth. The
Krok 2 Stomatology 2013 8

root protrudes above the gum by 0,5- A. Tuberculosis


1 mm. The apical part is filled. History: B. Syphilis
coronal part of the tooth broke off three C. Lichen planus
weeks ago. Prior to this the tooth had D. Cancer
been treated for chronic periodontitis. E. Ulcerative necrotizing stomatitis
Percussion is painless. The root edges
are surrounded with hard moist mucous 57. A 24-year-old patient complains of
membrane of pale pink colour. Specify the aching pain in the 11 tooth that is getti-
dentist’s tactics: ng worse during biting down on food.
Two days ago the tooth was filled for
A. Fabricate a crown-root inlay and pulpitis. Objectively: the 11 tooth is fi-
porcelain-fused-to-metal crown lled. The thermal test causes no pain,
B. Extract the tooth root vertical percussion is slightly painful. X-
C. Fabricate a simple pivot crown ray picture of the 11 tooth shows that the
D. Restore the tooth by means of an endodontic filling is 1 mm above the root
anchor and photopolymer material apex. Which of the following methods wi-
E. Fabricate a pivot Richmond crown ll be most effective for eliminating this
complication?
54. A 28-year-old patient complains of a
painless nonhealing mouth ulcer. Objecti- A. Fluctuorization
vely: regional lymph nodes are enlarged, B. Ultrahigh frequency therapy
painless, of cartilaginous consistency. On C. Relaxing incision
the buccal mucosa there is a round ulcer 1 D. Submucous injection of 1% solution of
cm in diameter with regular raised edges hydrocortisone
and a dense elastic infiltrate at the base. E. Analgetics
The ulcer surface is of red colour, painless
on palpation. What is the most likely di- 58. A patient is 12 years old. He has
agnosis? been undergoing orthodontic treatment
for pseudo prognathism with Angle’s fi-
A. Primary syphilis xed appliance for 10 months. What is the
B. Primary tuberculosis optimal duration of the retentive period?
C. Secondary tuberculosis
D. Cancer A. 20 months
E. Secondary syphilis B. 10 months
C. 6 months
55. A 32-year-old patient complains of D. 3 months
acute spontaneous attacks of pain in the E. 12 months
14 tooth. The pain lasts for 10-20 minutes
and occurs every 2-3 hours. Carious cavi- 59. A 55-year-old patient complains of
ty in the 14 tooth is filled with softened general weakness, spasmodic neuralgic
dentin. Probing of the cavity floor is pai- pain in the right side of face, rash in
nful at one point. Cold stimulus causes the mouth and on the skin. Objectively:
pain. What is the most likely diagnosis? lip and chin skin is markedly hyperemic,
there are numerous vesicles with clear
A. Acute localized pulpitis exudate on the right. The right cheek
B. Acute deep caries mucosa is hyperemic, there is a string of
C. Hyperemia of the pulp erosions covered with fibrinous pellicle.
D. Exacerbation of chronic pulpitis What is the provisional diagnosis?
E. Acute diffuse pulpitis
A. Herpes zoster
56. A 42-year-old patient complains of a B. Acute herpetic stomatitis
painful ulcer in the mouth that is getting C. Pemphigus vulgaris
bigger and does not heal over 1,5 months. D. Allergic stomatitis
Objectively: on the buccal mucosa there is E. Aphthous fever
a shallow soft ulcer 2 cm in diameter wi-
th irregular undermined edges. The ulcer 60. According to the mother, a 5-year-
floor is uneven and covered with yellow- old child complains about pain during
gray coating. The ulcer is surrounded by swallowing, weakness, body temperature
many small yellowish tubercles. Regional rise upt to 39, 5oC, swelling of submental
lymph nodes are elastic, painful, matted lymph nodes. Objectively: the child’s
together. Which disease is characterized condition is grave, body temperature
by such symptoms? is 38, 8oC. Mucous membrane of the
oral cavity is markedly hyperaemic and
Krok 2 Stomatology 2013 9

edematic with haemorrhages and ulcerati- A. Radical maxillary sinusotomy and local
ons. Pharynx is markedly hyperemic, tissue plasty of fistula
lacunae are enlarged and have necrosis B. Local tissue plasty of fistula, anti-
areas. Regional, cervical, occipital lymph inflammatory therapy
nodes are painful, enlarged and dense. C. Radical maxillary sinusotomy
What is the most likely diagnosis? D. Fistula plasty with a hard palate graft
E. Fistula suturing, sinus lavage
A. Infectious mononucleosis
B. Acute herpetic stomatitis 64. A 15-month-old child diagnosed with
C. Necrotizing ulcerative gingivostomatitis intrusive luxation of the 51 tooth has been
D. Herpetic angina referred to a children’s dental surgeon.
E. Lacunar tonsillitis What is the optimal treatment tactics?
61. A 19-year-old male patient consulted A. Monitoring the independent eruption
a dentist about gingival pain and bleedi- of the affected tooth
ng when eating and brushing his teeth. B. 51 tooth extraction
Objectively: anterior maxillary and C. 51 tooth reposition
mandibular teeth are overcrowded, there D. 51 tooth splinting
is hyperemia, cyanosis and overgrowth of E. 51 tooth replantation
gingival papillae up to 1/3 of the crown
height. The soft plaque is present. The 65. A 45-year-old patient complains of
gums bleed when touched with a tool. missing teeth in the lower jaw on the ri-
What is the most likely diagnosis? ght. Objectively: the 46, 45, 38, 48 teeth
are missing. The 47 tooth is mobile (1
A. Chronic hypertrophic gingivitis grade mobility), the crown of the 44
B. Chronic catarrhal gingivitis tooth is decayed by 1/2. What orthopedic
C. Generalized periodontitis construction should be recommended for
D. Localized periodontitis restoration of masticatory efficiency and
E. Ulcerative gingivitis prevention of periodontal overload of the
47, 44 teeth?
62. A 23-year-old female patient complai-
ns of short-term pain caused by thermal A. Bugel denture with clasp fixation for
stimuli in a lower jaw tooth on the ri- the 47, 44, 34 teeth
ght. Objectively: on the occlusal surface B. Soldered bridge supported by the 48, 44
of the 46 tooth there is a deep cavity wi- teeth
th softened light dentin on the floor and C. Porcelain-fused-to-metal bridge
walls. Probing causes a slight pain across supported by the 47, 46 teeth
the cavity floor. Pulp electroexcitability D. Bridge supported by the 47 tooth
is 10 mA. During the cavity preparation E. Small saddle denture with clasp fixation
a drop of blood showed up on its floor. for the 47, 44 teeth
What paste should be applied to the cavi-
ty floor? 66. A 57-year-old patient complains of
tooth mobility, inability to eat. Objecti-
A. Calcidont vely: the lower 35, 36, 37, 38, 44, 45, 46
B. Corticosteroid and 48 teeth are missing; the 31, 32, 33, 34,
C. Arsenous 41, 42, 43, 47 teeth exhibit II grade mobi-
D. Paraformaldehyde lity, their clinical crowns are low, tooth
E. Resorcinol-formalin equator is not pronounced. What is the
optimal denture construction in this case?
63. After the clinical and radiographic
examination a 32-year-old patient was di- A. Removable cast splint
agnosed with chronic odontogenic sinusi- B. Removable partial denture
tis, fistula between sinus and oral cavity C. Kurlyandsky splint bar
through the socket of the 27 tooth. Specify D. Removable Bynin splint
the optimal method of surgical treatment E. Removable splint with vestibulo-oral
of this disease: clasp
67. A child has been admitted to a hospital
with a maxillofacial trauma. It is probable
that the wound has been contaminated
with tetanus infection. How soon after the
injury should the tetanus serum be given?
Krok 2 Stomatology 2013 10

A. Immediately at the time of seeking touch is painful. The body temperature is


medical help of 37, 5o C. Blood count: RBC - 4, 8 · 1012/l,
B. Within 12 hours Hb- 150 g/l, colour index - 0,9, WBC -
C. Within 24 hours 10, 5 · 109 /l, JG - 2%, BASO - 1%, EOS -
D. Within 2 days 5%, segmented neut. - 8%, stabs - 47%,
E. Within a week LYM - 35%, PLT - 250 · 109 /l. ESR - 20
68. A patient working as a bricklayer mm/h. What is the most likely diagnosis?
complains of itching, burning, soreness A. Vincent’s gingivitis
of lips that show up only in the summer B. Agranulocytosis
period. He has a 3-year history of this di- C. Acute leukemia
sease. Objectively: vermilion border of the D. Acute herpetic stomatitis
lower lip is hyperemic, edematic, covered E. Chronic leukemia
with blisters and painful erosions 2 mm in
diameter, crusts, cracks. What is the most 72. A 19-year-old patient complains of
likely diagnosis? indisposition, fever, pain in the gums
and oral mucosa. These presentations
A. Actinic cheilitis, exudative form showed up two days ago. Objectively:
B. Contact allergic cheilitis body temperature is of 38,8 0C, regi-
C. Meteorological cheilitis onal lymph nodes on the right are
D. Eczematous cheilitis, exudative form enlarged, painful, mobile. Oral mucosa is
E. Exfoliative cheilitis, exudative form hyperemic, edematous; gingival papillae
69. Parents of a 2-month-old baby are hemorrhagic, with grayish-white coati-
complain about caseous films on the ng, on the buccal mucosa there are painful
mucous membrane of his cheeks and ulcers with irregular edges covered wi-
tongue. After examination the child th the same grayish-white coating. What
has been diagnosed with acute oral organisms are most likely to be found on
candidiasis of moderate severity. Topi- microbiological study?
cal treatment should be started with the A. Fusospirillary symbiosis
administration of the following agents: B. Virus
A. Antifungal C. Fungal pathogens
B. Keratoplastic D. Colon bacillus
C. Antiseptic E. Diphtheria bacillus
D. Enzymatic 73. A 30-year-old patient has a cut wound
E. Antiviral of the upper lateral surface of tongue. The
70. A patient complains of acute short- wound was given with a separating disk.
term toothache caused by thermal and Bood penetrated into the airways causing
chemical irritants. The tooth has not been asphyxia. What form of asphyxia occurred
treated before, the pain arose about a in the patient?
month ago. Objectively: on the occlusal A. Aspiration
surface of the 36 tooth there is a deep B. Stenotic
carious cavity within circumpulpal dentin C. Valvular
with overhanging chalky enamel. Probi- D. Obstructive
ng of the cavity floor causes a slight pain. E. Dislocation
There is also a short-term pain from the
cold stimulus. The result of electric pulp 74. A 10-year-old girl complains of a cari-
test is 8 mA. What is the most likely di- ous cavity in a lower jaw tooth. Objecti-
agnosis? vely: the 26 tooth has a carious cavity wi-
thin circumpulpal dentin, dentin is dense,
A. Acute deep caries pigmented, there is pain reaction to the
B. Acute median caries cold stimulus, probing and percussion
C. Pulp hyperemia are painless. What is the provisional di-
D. Chronic deep caries agnosis?
E. Chronic fibrous pulpitis
A. Chronic deep caries
71. A 20-year-old male patient complai- B. Acute median caries
ns of acute pain during eating, bad C. Acute deep caries
breath, general weakness. Objectively: D. Chronic median caries
interdental papillae and marginal gingi- E. Chronic simple pulpitis
va are covered with dirty gray film, any
Krok 2 Stomatology 2013 11

75. A 12-year-old girl visited a dentist for A. Acute purulent periodontitis


a regular check-up. Objectively: the teeth B. Exacerbation of chronic periodontitis
are intact, the bite is open, the mucosa C. Acute diffuse pulpitis
in the papillary and marginal regions is D. Acute serous periodontitis
hyperemic and cyanotic, the gingival papi- E. Acute purulent pulpitis
llae are roundish, enlarged and cover one
third of the crown height of the upper 79. A 20-year-old patient complains of the
anterior teeth. Make a provisional di- 23 tooth sensitivity to sour or sweet food.
agnosis: The pain abates after the irritants are
removed. The problems arose two weeks
A. Hypertrophic gingivitis ago. Objectively: on the vestibular surface
B. Chronic catarrhal gingivitis of the 23 tooth there are white spots 2-3
C. Acute catarrhal gingivitis mm in diameter. The spots have a rough
D. Gingival fibromatosis surface, methylene blue stain cannot be
E. - washed away with water. What is the most
likely diagnosis?
76. Parents of a 6-year-old child consulted
a doctor about a swelling in his right A. Acute initial caries
parotid-masseteric region, dry mouth, B. Chronic initial caries
fever up to 37, 7oC. Similar presentations C. Mottled enamel
turned up about 6 months ago. Examinati- D. Initial stage of enamel necrosis
on revealed a tuberous, slightly painful E. Local enamel hypoplasia
infiltrate in the right parotid-masseteric
region. Saliva is viscous, the right salivary 80. A 24-year-old patient complains of
gland duct discharges some secretion with acute pain and swelling in the submandi-
whitish inclusions. What is the most likely bular triangle that is getting worse when
diagnosis? he eats spicy food. He has been diagnosed
with exacerbation of chronic calculous si-
A. Exacerbation of chronic aloadenitis with the concrement localized
parenchymatous parotitis in the median part of the salivary duct.
B. Acute purulent parotitis Which method is most rational in this
C. Mixed tumor, salivary gland type case?
D. Acute purulent lymphadenitis of
parotid gland A. Duct dissection and concrement
E. Parotitis removal
B. Gland extirpation
77. A 34-year-old patient was undergoing C. Duct bougienage
treatment for acute deep caries of the 37 D. Ptyalalogue diet
tooth. During the tooth preparation the E. Physiotherapy
buccal pulp horn was accidentally opened.
What is the most rational treatment of the 81. A 38-year-old patient with chronic
37 tooth? generalized periodontitis has been
referred to orthopedic treatment. Objecti-
A. Biological method vely: dentitions are without gaps, the 12,
B. Vital amputation 11, 21, 22 teeth are pulpless and exhibit I
C. Non-vital amputation grade mobility. The other teeth are stable.
D. Vital extirpation What is the most aesthetic splint for the
E. Non-vital extirpation anterior teeth?
78. A 21-year-old patient complains of a A. Mamlok splint
constant progressing throbbing pain in the B. Ring adhesion splint
27 tooth. Objectively: a large carious cavi- C. Soldered combined crowns
ty is filled with softened dentin, the tooth D. Cap splint
cavity is closed. Probing of the cavity floor E. Brace
is painless, percussion causes acute pain,
there is grade II tooth mobility. Palpation 82. A 42-year-old patient complains of
of the mucous membrane in the projecti- pain in the right side of her head, restri-
on of the root apex of the 27 tooth causes cted movements of the lower jaw, cli-
pain. Radiological changes are absent. cking sound, periodic spasms of chewi-
What is the most likely diagnosis? ng muscles. Objectively: the face is
symmetric, mouth opening is restricted.
On palpation of the right temporo-
mandibular joint (TMJ) there are crepi-
Krok 2 Stomatology 2013 12

tation and clicking accompanying mandi- fragment of 44 43 42 41 31 32 33 34 teeth


ble movements. Examination of the oral is displaced downwards and backwards.
cavity revealed also a Kennedy’s class II Single-step fragment reposition is not
defect on the right. What is the most likely effective. Which appliance should be used
diagnosis? for the fragment reposition?
A. Pain dysfunction of the right TMJ A. Post’s
B. Acute arthritis of the right TMJ B. Kurlyandsky cap splint for one jaw
C. Sclerosing osteoarthritis of the right C. Weber’s
TMJ D. Betelman’s
D. Contracture of the right TMJ E. Schur’s
E. Myositis ossificans
87. A 25-year-old patient has necrosis foci
83. A patient needs a clasp prosthesis. It is along the gingival margin on both jaws.
planned to study the jaw model by means Face skin is sallow, he looks older than his
of a parallelometer in order to determine age. Submandibular, mental, cervical and
the required depth of the undercuts on the axillary lymph nodes are enlarged, pai-
abutment teeth. Specify the length of the nless, mobile. The patient leads a hectic
measuring rods used for this purpose: life, abuses drugs. Over the last month
he has complained of weakness, constant
A. 0,25 0,50 0,75 body temperature of 37, 3 − 37, 5oC, di-
B. 0,15 0,40 0,65 arrhea, weight loss of 15 kg. What is the
C. 0,20 0,45 0,70 most likely provisional diagnosis?
D. 0,30 0,55 0,80
E. 0,35 0,60 0,85 A. AIDS
B. Acute leukemia
84. An 18-year-old male has a furuncle C. Hypovitaminosis C
in the region of mandible angle. The D. Vincent’s gingivitis
surrounding tissues are rigid. Cyanosis E. Agranulocytosis
and induration in form of a cord extend
to the corner of the eye. General conditi- 88. A 20-year-old patient was delivered to
on is deteriorated, body temperature is of the hospital 6 hours after injury. Objecti-
38, 5oC. What complication has developed vely: the patient has malocclusion due to
in this case? the displacement of bone fragments. The
teeth on both jaws are intact except for the
A. Facial vein thrombophlebitis missing 35 tooth. Ro-gram shows fracture
B. Abscess of cheek of mandible. The fragments are displaced.
C. Phlegmon of cheek What treatment is most appropriate?
D. Acute sinusitis
E. Carbuncle of cheek A. Anchor splints with intermaxillary
rubber traction band
85. A 30-year-old patient needs a post and B. Intraoral miniplate osteosynthesis
core for the 36 tooth prior to the tooth C. Smooth brace
restoration with a crown. Objectively: the D. Intermaxillary ligature
crown of the 36 tooth is decayed down E. Chin cup
to the gums, percussion is painless. Radi-
ograph shows that root canals are filled to 89. A 6-year-old girl has unrestricted
the top. There are no changes of the peri- opening of the mouth. The alveolar
apical tissues. What is the optimal number process of the upper jaw is intact. Soft and
of posts in this case? hard palate are of a triangular shape and
have a cleft up to the level of the 13 and
A. Two 23 teeth. Soft palate is shortened. Speech
B. Three is indistinct. The child was born with this
C. Four defect. What is the most likely diagnosis?
D. One
E. Five A. Natural partial cleft palate
B. Isolated complete cleft palate
86. A 25-year-old patient presented to C. Submucous cleft palate
a clinic on the fourth day after a face D. Isolated partial uranoschisis
trauma complaining of pain in the lower E. -
jaw on both sides, difficult swallowing.
Objectvely: there is a slight swelling in the 90. 3 days ago a 35-year-old patient
mental region, dentitions are intact but a underwent the extraction of the 47 tooth,
Krok 2 Stomatology 2013 13

which was followed by health deteriorati- on the right. Hot stimuli make the pain
on. Objectively: the patient is pale, body significantly worse. The tooth had been
temperature is of 38, 1oC, the lower jaw previously treated for median caries. The
on the left is thickened, the surroundi- pain arose a day before. On the medial
ng soft tissues are swollen, opening of occlusal surface of the 54 tooth there is
the mouth is difficult. Mucogingival fold a deep carious cavity, the tooth cavity is
at the 48, 47, 46 teeth is edematous, closed. Probing of the cavity floor is pai-
mucous membrane is hyperemic. There nless, percussion is slightly painful. After
are purulent discharges from the 47 tooth necrotomy the pulp horn was opened.
socket, percussion of the 48, 46, 45 teeth Deep probing is painful. What is the
is painful. There is also paresthesia in the provisional diagnosis?
lower lip region. What is the most likely
diagnosis? A. Acute purulent pulpitis
B. Acute serous pulpitis
A. Acute mandibular osteomyelitis C. Exacerbation of chronic pulpitis
B. Acute mandibular periostitis D. Acute purulent periodontitis
C. Submandibular phlegmon E. Exacerbation of chronic periodontitis
D. Acute periodontitis of the 47 tooth
E. Abscess of mandibular radicular cyst 95. A 7-year-old child complains of pain
and swelling in the left submandibular
91. Parents of a 6,5-year-old boy consulted region. The swelling in this region showed
an orthodontist about no contact between up 2 days ago. Objectively: the child is in
the front teeth. The child has a bad satisfactory condition, body temperature
habit of sucking his tongue. Objectively: is of 37, 3oC. Face is asymmetrical due
there is a symptom of multiple pits in his to the soft tissue swelling in the left
chin when the lips are closed, speech di- submandibular region. Palpation reveals
sturbance, between the front teeth there a round formation sized 2x2 cm. The
is a vertical gap up to 8 mm. Specify the formation is mobile, painful, not attached
occlusion anomaly: to skin. The 74 tooth is changed in colour,
percussion is painful. What is the provisi-
A. Open bite onal diagnosis?
B. Cross-bite
C. Distal occlusion A. Acute serous odontogenic lymphadeni-
D. Mesial bite tis of the left submandibular region
E. Overbite B. Acute serous nonodontogenic
lymphadenitis of the left submandi-
92. During the extraction of the 17 tooth bular region
the distal buccal root broke off and remai- C. Acute purulent odontogenic
ned in the socket. Select a tool to extract lymphadenitis of the left submandi-
the root: bular region
D. Phlegmonous adenitis of the right
A. Bayonet forceps, straight elevator submandibular region
B. Straight forceps, straight elevator E. Lateral cervical cyst
C. S-shaped forceps
D. Beak-shaped forceps, angular elevator 96. A 21-year-old patient consulted a
E. Angular elevators dental surgeon about a tumour-like
formation on the left cheek. The pati-
93. A 64-year-old patient needs a ent noticed the swelling for the first ti-
complete removable denture for the me a year ago. Objectively: on the left
upper jaw. During fitting the Herbst appli- cheek skin there is a painless roundi-
ance, the orthopedist revealed that it sh pedunculated tumour up to 0,5 cm
falls off when the patient swallows. The in diameter. The surface of the formati-
correction is required in the following on looks like cauliflower, the surrounding
area: skin is not changed. What is the most li-
A. Distally along the "A"line kely diagnosis?
B. In the region of maxillary prominences A. Papilloma of cheek
C. In the region of buccal muscles B. Cutaneous horn
D. Vestibularly between canines C. Hemangioma of cheek
E. Along the vestibular edge D. Botryomycoma of cheek
94. A 5-year-old child complains of acute E. Fibroma of cheek
spontaneous pain in an upper jaw tooth 97. A 24-year-old patient complains of a
Krok 2 Stomatology 2013 14

growing tumour in the body of mandible A. Adenocarcinoma of the parotid salivary


on the right. The tumour growth has been gland
observed for 5 months. Objectively: in B. Chronic parotitis
the angle of mandible there is a swelling. C. Actinomycosis of the parotid-masseter
Regional lymph nodes are not enlarged. region
Ro-gram shows a well-defined focus of D. Chronic lymphadenitis
bone destruction at the level of the 46, 47, E. Pleomorphic adenoma of the parotid
and 48 teeth and in the ramus of mandible. gland
The lesion is in form of individual cavities
with osseous septa. The roots of the 46, 47, 100. A 58-year-old patient complains of
48 teeth are resorbed. What is the provisi- missing teeth on the upper jaw. Objecti-
onal diagnosis? vely: in the place of the missing central
upper incisors there is a fan-shaped vesti-
A. Osteoclastoma of mandible on the right bularly located cord on the alveolar
B. Ameloblastoma of mandible on the process. The cord is 1 cm wide, it makes
right prosthetics more complicated. What is the
C. Follicular cyst of mandible on the right optimal medical tactics?
D. Radicular cyst of mandible on the right
E. Odontoma of mandible on the right A. Surgical excision of the cord
B. Silicone implant
98. Parents of a 6-year-old girl consulted C. Operation on the alveolar process
an orthodontist about protrusion of the D. Not to cover the cord with the denture
lower jaw. The child looks like his father. E. To cover the cord with the denture
Objectively: the child has primary bi-
te, there are diastemata and tremata on 101. A 35-year-old patient has been di-
both jaws, reverse incisal overlap of front agnosed with chronic median caries of
teeth, the sagittal gap is up to 3 mm, the the 36 tooth. There is a Black’s class II
lateral parts are characterized by mesio- cavity affecting masticatory surface. What
occlusion. Ilyina-Markosyan test for the material should be chosen for the tooth
distal displacement of mandible is negati- filling?
ve. What principle of treatment will be A. Light-cure microhybride composite
most effective? B. Glass ionomer cement
A. To delay the growth of mandible in C. Silicophosphate cement
sagittal direction D. Light-cure fluid composite
B. To stimulate the growth of maxilla in E. Light-cure microfilled composite
sagittal direction 102. A 28-year-old patient complains of
C. The treatment is not required dull pain in the 17 tooth caused by hot
D. No to start treatment until the end of meal, the pain slowly abates after removi-
transitional dentition ng the irritant. There is also a feeling of
E. To start orthodontic treatment after the tooth bursting, and bad breath after the
second dentition is completed tooth suction. What is the provisional di-
99. A 52-year-old patient complains of agnosis?
pain and a swelling in the right parotid A. Chronic gangrenous pulpitis
region. These manifestations have been B. Chronic fibrous pulpitis
present for about 2 years. Over the last C. Chronic fibrous periodontitis
month the swelling has grown bigger, pain D. Chronic hypertrophic pulpitis
has intensified. Objectively: the face is E. Chronic concrementous pulpitis
asymmetric due to the dense infiltrate
in the right parotid region. The poorly 103. Parents of an 8-year-old child with
circumscribed, painful formation infi- Down syndrome took the child to a doctor
ltrates the surrounding tissues. At the for oral cavity sanitation. After the exami-
right side of neck in front and behind nation entailing great difficulties the child
the sternocleidomastoid muscle there are was found to have four teeth decayed as a
enlarged, dense, mobile lymph nodes. result of chronic periodontitis. What kind
The right naso-buccal groove is flattened, of anesthesia should be chosen for surgi-
the corner of mouth is downturned. The cal sanitation in one visit?
mouth opens freely. The are pronounced
symptoms of the right facial nerve paresis.
What disease can be suspected?
Krok 2 Stomatology 2013 15

A. Phlebonarcosis agnosis?
B. Conduction anesthesia
C. Mask anesthesia A. Verrucous precancer of the vermilion
D. Endotracheal anesthesia border of lip
E. Endotracheal anesthesia through a B. Abrasive precancerous Manganotti’s
tracheostoma cheilitis
C. Precancerous limited hyperkeratosis of
104. A patronage nurse visited a newborn the vermilion border of lip
baby. Examination revealed the shortened D. Bowen’s disease
lower part of the face, the backward- E. Erythroplasia of Queyrat
sloping chin, missing teeth, the retroposed
lower jaw. What is the number of dental 108. A 16-year-old male patient complai-
follicles in each jaw of a newborn baby? ns of itchy rash on the vermilion border of
lips and in the mouth corners. The patient
A. 18 has a 4-year-old history of this disease wi-
B. 16 th exacerbation periods mainly in autumn
C. 14 and winter. Objectively: vermilion border
D. 12 of lips and skin in the mouth corners is dry,
E. 10 peeling with small scales. Mouth corners
are characterized by lichenification, deep
105. A 47-year-old patient complains of a folds and fissures, bloody crusts. What is
burning sensation and pain in the mouth. the most likely diagnosis?
Objectively: on the mucous membrane of
cheeks along the line of teeth contact and A. Atopic cheilitis
in the corners of mouth there are multiple B. Actinic cheilitis
polygonal bright red erosions 1,0-1,5 cm C. Meteorological cheilitis
in diameter located on the hyperkeratini- D. Eczematous cheilitis
zed plaque and opaque whitish mucosa. E. Exfoliative cheilitis
Cytological analysis revealed keratinizing
epithelial cells. What is the most likely di- 109. Preventive examination of a 28-year-
agnosis? old male patient revealed reddish smooth
patches in form of different-sized circles
A. Leukoplakia, erosive form on the back of tongue. The patches are
B. Lichen planus, erosive form circumscribed with a narrow whitish stri-
C. Erythema multiforme pe. It is known that these tongue changes
D. Secondary syphilis are not permanent, show up and di-
E. Lupus erythematosus, erosive form sappear throughout a few years and don’t
bother him. The patient has a history of
106. A 67-year-old female patient has a chronic gastritis. Histological examinati-
burning sensation under her complete on of the epithelium revealed hyper- and
removable denture for the upper jaw parakeratosis. What is the most likely di-
which was fabricated of plastics 3 days agnosis?
ago. Examination revealed hyperemia and
edema of the mucous membrane within A. Desquamative glossitis
the boundaries of the denture-supporting B. Secondary syphilis
area. What is the cause of this pathology? C. Lichen planus
D. Leukoplakia
A. Excess of monomer in the plastics E. Rhomboid glossitis
B. Sensitization to the plastics
C. Paresthesia of the oral mucosa 110. After unilateral resection of the
D. Dysbiosis of the oral cavity upper jaw a 52-year-old patient received
E. Galvanism in the oral cavity the immediate-insertion denture. What is
the term of its use?
107. A 55-year-old patient consulted a
dentist about a roundish tumour-like A. 1-3 months
formation of about 1 cm in diameter B. 1-2 months
located within the vermilion border of C. 7-12 months
his lower lip. Objectively: the tumour- D. 2-3 years
like formation protrudes about 5 mm E. 4-5 years
above the vermilion border, it is dense and
grayish-red. The surface of the formation 111. An injured patient complains of limi-
is covered with thin scales that can hardly ted opening of the mouth, nose bleedi-
be removed. What is the most likely di- ng, skin numbness in the infraorbital and
Krok 2 Stomatology 2013 16

lower eyelid region. Objectively: there is mouth opening, pain during swallowi-
face deformation due to the depression of ng, fever up to 38, 5oC, weakness, indi-
soft tissues in the left cheekbone region, sposition. Objectively: the mouth opens
step deformity in the middle part of the up to 1 cm. After Berchet anesthesia
inferior margin of the left orbit and in the examination of the oral cavity revealed
area of the zygomatic alveolar crest. What edema, hyperemia, tenderness of the
is the most likely diagnosis? pterygomandibular fold. Lateral and
posterior pharyngeal walls are intact. The
A. Zygomatic bone fracture with di- 38 tooth is semi-impacted. Palpation of
splacement of bone fragments the internal surface of the mandibular
B. Fracture of the right zygomatic bone angle is painful. What is the most likely
without displacement of fragments diagnosis?
C. Le Fort I fracture of maxilla
D. Le Fort II fracture of maxilla A. Phlegmon of pterygomandibular space
E. Fracture of the malar arch B. Acute tonsillitis
C. Phlegmon of the infratemporal region
112. A 30-year-old patient needs to D. Acute pericoronitis of the 38 tooth
have his 26 tooth extracted because of region
exacerbation of chronic periodontitis. E. Phlegmon of peripharyngeal space
Objectively: the crown of the 26 tooth is
decayed by 1/3. What forceps can be used 116. A 6-year-old patient complains of a
for this tooth extraction? long-lasting pain in a lower right tooth
that is getting worse when biting down
A. S-shaped forceps with a projecting tip on food. The tooth has been previously
on the left beak treated for caries. Objectively: the cavi-
B. S-shaped forceps with a projecting tip ty on the occlusal surface of the 36 tooth
on the right beak is filled, percussion is painful. Along the
C. Straight forceps mucogingival fold in the projection of
D. Straight elevator the 36 tooth roots there is a fistula with
E. S-shaped forceps without projecting tips purulent discharge. What is the most li-
kely diagnosis?
113. A patient is 56 years old. 6 months
after resection of the right side of the A. Chronic granulating periodontitis
upper jaw the patient is going to receive B. Exacerbation of chronic periodontitis
a permanent resection prosthesis. What C. Chronic fibrous periodontitis
device can be used to determine the D. Chronic granulomatous periodontitis
optimal state of the resection prosthesis E. Radicular cyst of mandible
borders?
117. A 35-year-old patient complains of a
A. Parallelometer neoplasm on the tip of his tongue which
B. Micrometer he injures with his teeth. The neoplasm
C. Larin’s appliance sometimes increases, and sometimes
D. Gnathodynamometer decreases in size. Objectively: on the tip
E. Myotonometer of tongue there is a roundish well-defined
neoplasm 0,5 cm in diameter with broad
114. A 49-year-old patient consulted a base. The neoplasm is the same colour as
dental surgeon about the oral cavity the mucosa of tongue. What is the most
sanitation. He has an indication for the likely diagnosis?
extraction of the 16 tooth. History: the
tooth hasn’t been treated before, it has A. Papilloma of tongue
decayed within the last 4 years. Objecti- B. Abscess of tongue
vely: the 16 tooth’s crown is decayed C. Lipoma of tongue
by over 2/3, the mucosa exhibits no D. Hemangiofibroma of tongue
pathological changes. Which tool is requi- E. Fibroma of tongue
red for the tooth extraction?
118. A patient presented to a doctor with
A. Bayonet root forceps a red spot on her chin skin. Two months
B. S-shaped forceps (right) ago the patient suffered a chin injury.
C. Straight elevator After resorption of traumatic edema there
D. S-shaped closed-beak forceps appeared a black punctum which was
E. Crown bayonet forceps gradually increasing in size. Objectively:
the face is symmetric, on the chin skin
115. A patient complains of limited there is an irregular wine-coloured pai-
Krok 2 Stomatology 2013 17

nless spot sized 2x1,5 cm that turns white tooth. After a physical examination the
when pressed. What is the most likely di- patient was diagnosed with exacerbati-
agnosis? on of chronic periodontitis of the 26
tooth. During the tooth extraction the
A. Capillary hemangioma coronal part of the tooth was acci-
B. Pigmented nevus dentally broken. Further manipulations
C. Melanoma with bayonet forceps failed to extract the
D. Hematoma tooth roots. What actions are to be taken
E. Telangiectasia for the successful root extraction?
119. X-ray examination of the oral cavi- A. To disjoin the roots by using a drill and
ty revealed a pronounced damage of the fissure burr
compact bone, mild osteoporosis of apices B. To send the patient to the hospital
of interalveolar septa and widened peri- C. To use Lecluse elevator
odontal ligament space around the dental D. To complete the tooth extraction during
necks. What is the most likely diagnosis? the next visit
E. To use angular elevator
A. Chronic generalized periodontitis,
initial stage 123. A 30-year-old female patient
B. Chronic generalized periodontitis, I complains of limited mouth opening. She
stage has a history of a trauma received at the
C. Chronic catarrhal gingivitis time when her jaws were closed. Objecti-
D. Periodontosis, I stage vely: mouth opening is limited to 1,5
E. Periodontosis, initial stage cm, the bite is normal, mouth opening
causes the displacement of mandible to
120. Preventive examination of a 13,5- the left. Palpation reveals pain in the left
year-old child revealed congestive temporomandibular joint (TMJ). X-ray
hyperemia and a slight edema of the gi- picture shows that the bone integrity is
ngival margin in the region of the front
preserved. What is the most likely di-
teeth on both jaws. The patient has peri- agnosis?
odic gingival haemorrhages during tooth
brushing. DMF index is 2. What kind of A. Acute traumatic arthritis of the left
toothpaste should be recommended for TMJ
the individual oral hygiene? B. Fracture of the left articular process of
mandible
A. Toothpaste containing herbal extracts
C. Acute unilateral dislocation of the TMJ
B. Toothpaste containing calcium D. Contusion of the left TMJ
C. Toothpaste containing aminofluorides
E. Fracture of the left cheekbone with
D. Toothpaste containing sodium fluoride
displacement of bone fragments
E. Toothpaste containing salt additives
124. Preventive examination of a 5-year-
121. A 64-year-old patient with old child revealed a habit of lower lip biti-
edentulous jaws has a minor uniform
ng. What malocclusion may develop if the
atrophy of maxilla and a pronounced child keeps this habit?
atrophy of lateral parts of mandible. The
mucosa in these parts makes longitudi- A. Anterior bite
nal folds, the alveolar crest is mobile in B. Prognathic bite
the anterior part. The condition of mandi- C. Open bite
ble should be taken into consideration D. Deep overbite
at the following stage of fabrication of a E. Cross-bite
complete removable denture:
125. A 14-year-old girl complains of indi-
A. Taking differentiated functional stinct pronunciation that developed at the
impression age of 14 after the acute respiratory vi-
B. Taking anatomical impression ral disease. Examination revealed normal
C. Taking compression functional impressi- face and normal teeth alignment, occlusal
on disharmony was not found. Palpation di-
D. Taking decompression functional dn’t reveal cleft palate. Uvula doesn’t
impression move during pronunciation of sounds, its
E. Determining the central occlusion palpation does not cause gag reflex. What
is the reason for indistinct pronunciation
122. A 24-year-old female patient of sounds?
consulted a dentist about pain in the 26
Krok 2 Stomatology 2013 18

A. Shortening of teeth
A. Paresis of the soft palate and uvula B. Increase of interalveolar height
muscles C. Orthodontic method
B. Adenoid vegetations D. Orthodontic and surgical method
C. Palatal slit E. Tooth extraction
D. Hypertrophy of lingual tonsil
E. Deformation of the bite 130. It is planned to open a city dental cli-
nic to serve 200,000 of adult population.
126. A 35-year-old female patient seeks How many positions of dentists should be
prosthetic dentistry. Objectively: the 18, provided for the orthopedic department?
14, 13, 12, 11, 21, 22, 23, 24, 28 teeth
need restoration. The crowns confining A. 20
the defect are high and stable. To restore B. 30
the integrity of the dentition it is planned C. 15
to fabricate a cast clasp denture. What ki- D. 10
nd of mechanical fixator should be used? E. 5
A. Bar fixation 131. A 25-year-old patient consulted
B. Supporting-retaining clasps a dentist about orthopedic treatment
C. Retaining clasps of the 17 tooth. After the patient
D. Telescopic fixation had been given local anesthesia, his
E. Dentoalveolar clasps condition suddenly deteriorated (he
developed anxiety, general weakness,
127. A 22-year-old patient presented to a acute headache, nausea, loss of consci-
prosthetic dentistry clinic because of mi- ousness, poor pupillary reaction to light,
ssing of the 21 tooth, the 11 and 22 teeth abrupt drop of arterial pressure. What is
are intact. The 21 tooth was extracted 2 the most likely diagnosis?
months ago. What construction is most
suitable in this case? A. Anaphylactic shock
B. Cardiogenic shock
A. Two-stage implantation C. Septic shock
B. One-stage implantation with si- D. Traumatic shock
multaneous fabrication of porcelain- E. Toxic shock
fused-to-metal crown
C. Partial removable denture 132. The department of maxillofacial
D. Swaged and soldered denture supported surgery admitted a patient who needs
by the 11 and 22 teeth repair of a post-traumatic defect of nose
E. Metal-plastic denture supported by the wing up to 3,0 cm in diameter. The trauma
11 and 22 teeth occured six months ago. What kind of
grafting is indicated in this clinical situati-
128. A 48-year-old patient has undergone on?
unilateral resection of the upper jaw. He
needs a resection prosthesis. Objectively: A. Grafting with chondrocutaneous flap of
the 21, 22, 23, 24, 25, 26 teeth are stable, the auricle
hard palate is broad, flat. What connecti- B. Grafting with local tissues of nasolabial
on of clasps with the base will reduce the or cheek regions
instability of the resection prosthesis? C. Grafting with pedicle flap of frontal and
buccal regions
A. Articular D. Grafting with tubed pedicle (Filatov’s)
B. Stable flap
C. Elastic E. Free grafting with dermal flap
D. Fixed
E. Does not matter 133. A 25-year-old female patient
consulted a dentist about acute pain in
129. A 47-year-old patient needs dental the upper jaw on the left. The pain occurs
prosthetics. Objectively: the 24, 25 teeth during eating. Objectively: on the distal
are missing, there is a slight dento-alveolar approximal surface of the 26 tooth there is
lengthening in the region of the 34, 35 a cavity filled with light soft dentin. Probi-
teeth. Which of the following methods of ng causes a slight pain along the dentin-
the occlusal surface correction should be enamel junction, percussion is painless.
applied? Cold water causes quickly abating pain.
What is the most likely diagnosis?
Krok 2 Stomatology 2013 19

A. Acute median caries A. Acute deep gingivitis


B. Chronic median caries B. Chronic catarrhal gingivitis
C. Acute deep caries C. Hypertrophic gingivitis
D. Chronic fibrous pulpitis D. Initial stage of chronic generalized
E. Chronic deep caries periodontitis
E. Exacerbation of initial generalized
134. At a prosthetics dentistry clinic a periodontitis
35-year-old patient received a porcelain-
fused-to-metal crown for the 21 tooth. 138. A 35-year-old patient complains of
What is the minimum warranty period for gingival hemorrhage during tooth brushi-
porcelain-fused-to-metal crowns under ng. Objectively: gums of both jaws are
the current legislation? hyperemic and cyanotic, supragingival
and subgingival tartar is present. The peri-
A. 12 months odontal pockets are 4-6 mm deep. What is
B. 24 months the most likely provisional diagnosis?
C. 36 months
D. 6 months A. II grade generalized periodontitis,
E. 3 months chronic course
B. Catarrhal gingivitis
135. During the surgical removal of a C. Hypertrophic gingivitis
retention cyst of the lower lip a 14-year- D. Exacerbation of I grade generalized
old boy complained of sudden weakness, periodontitis
dizziness, nausea. Objectively: the skin is E. II grade parodontosis
covered with cold sweat. Respiration is
frequent, pulse is weak, AP is decreased 139. A 27-year-old male patient complai-
(90/60 mm Hg), the hands are cold. What ns of aching long-lasting pain in the 15
is the most likely diagnosis? tooth during having meals, especially cold
food. Sometimes the pain occurs when the
A. Syncope temperature changes. Objectively: on the
B. Toxic shock distal surface of the 15 tooth there is a
C. Traumatic shock cavity filled with softened dentin. Probing
D. Anaphylactic shock is painful. Electroexcitability of the pulp is
E. Collapse 35 µA. What is the most likely diagnosis?
136. Parents of a 7,5-year old child brought A. Chronic fibrous pulpitis
him to a dentist for oral cavity sanitation. B. Acute deep caries
Objectively: DEF (for primary teeth) + C. Chronic deep caries
DMF (for permanent teeth) index = 4, D. Hyperemia of the pulp
Green-Vermillion index = 2,5. Fissures E. Exacerbation of chronic pulpitis
of the first permanent molars are open,
intact, non-pigmented. What method of 140. A 12-year-old child presents with
primary prevention of dental caries may body temperature of 38o C, chills, nausea,
be appropriate in this case? vomiting, delirium, weakness. In the mi-
ddle third of face there is butterfly-shaped
A. Non-invasive hermetization rash. Regional lymph nodes are enlarged,
B. Invasive hermetization slightly painful. Blood count: WBC- 12 ·
C. Application of fluoride varnish 109 /l, LYMP - 8, 0 · 109 /l, ESR - 26 mm/h.
D. Application of antibacterial varnish Make a diagnosis:
E. Application of calcium gels
A. Erysipelas
137. A 28-year-old patient complains of B. Facial vein phlebitis
experiencing gum discomfort, gingival C. Cutaneous actinomycosis
haemorrhages and pain, especially during D. Streptoderma. Submandibular
eating, for a week. Objectively: the gums lymphadenitis
are swollen and markedly hyperemic, E. Acute nonodontogenic sinusitis
especially within the 43, 42, 41, 31, 32, 33
teeth, soft plaque and tartar are present, 141. A 37-year-old patient had an appoi-
gingival sulcus is up to 2 mm deep. What ntment with his orthopaedist about dental
is the most likely diagnosis? prosthetics. During the preparation of
hard dental tissues for a fixed denture he
was given conduction anesthesia. During
the injection the patient developed cold
sweat, sudden pallor and cyanosis, thready
Krok 2 Stomatology 2013 20

pulse. The patient stayed conscious, but cal shape. The patient has orthognathic
became relaxed and lethargic. What is the bite. What prosthesic design should be
provisional diagnosis? preferred?
A. Collapse A. Immediate prosthesis
B. Anaphylactic shock B. Plastic bridge
C. Intoxication C. Porcelain-fused-to-metal bridge
D. Syncope D. Plastic-to-metal bridge
E. Hypertensive crisis E. Implant with a pocelain crown
142. A 25-year-old patient complains of a 146. A 62-year-old patient diagnosed with
light brown spot in the upper front tooth. TMJ arthritis presented to a clinic. The
Objectively: the 23 tooth has a single li- patient underwent radiography. Speci-
ght brown spot in the precervical regi- fy the radiological presentations of TMJ
on. Probing reveals smooth surface. The arthritis:
tooth is nonresponsive to cold and probi-
ng. What is the most likely diagnosis? A. Changed shape of the bone elements of
the joint
A. Chronic initial caries B. Joint space narrowing
B. Fluorosis C. Joint space widening
C. Local enamel hypoplasia D. No joint space
D. Acute initial caries E. Compaction of the articular head cortex
E. Chronic superficial caries
147. A 24-year-old patient consulted a
143. A 48-year-old patient is in grave dentist about a cavity in an upper jaw
condition: body temperature is of 39, 8oC, tooth, which appeared three months ago.
the face is asymmetric due to the infiltrati- The tooth had not ached before. Objecti-
on of the submental and both submandi- vely: on the occlusal surface of the 15
bular triangles. Speech is difficult as the tooth there is a small carious cavity within
tongue is displaced upwards. Swallowing the mantle dentin filled with leftover food.
is painful. Sublingual plicae are markedly Probing revealed that the cavity walls and
hyperemic and edematous. What is the floor were dense. The response to thermal
most likely diagnosis? stimuli is absent. What is the most likely
diagnosis?
A. Phlegmon of mouth floor
B. Abscess of tongue root A. Chronic median caries
C. Phlegmon of submental triangle B. Initial caries
D. Phlegmon of alveololingual groove C. Acute deep caries
E. Phlegmon of submandibular triangle D. Acute median caries
E. Pulp hyperemia
144. During the dental care procedures a
4-year-old child has been found to have 148. A 45-year-old patient has a bilateral
a carious cavity in the 64 tooth. The cavi- mental mandibular fracture with a typical
ty filled with softened dentin is located displacement of fragments. What actions
within parapulpal dentin and doesn’t should be taken to eliminate asphyxia?
communicate with the tooth cavity. Probi-
ng of the cavity floor is painless. Gingival A. Pull out and pierce the tongue
mucosa in the projection of the root apices B. Artificial pulmonary ventilation
of the 64 tooth exhibits a healed fistula. C. Tracheal intubation through the nose
What is the most likely diagnosis? D. Tracheal intubation through the mouth
E. Tracheostomy
A. Chronic granulating periodontitis
B. Chronic periodontitis fibrotic 149. A 42-year-old patient consulted a
C. Chronic fibrous pulpitis dentist about a swelling on the left cheek,
D. Chronic gangrenous pulpitis redness of its skin. 2 months ago he noti-
E. Chronic periodontitis garnulematozny ced an induration on the left cheek which
has been eventually enlarging. The skin
145. A 20-year-old patient in the 8th over it turned bluish-red and soft in the
month of pregnancy complains about an center. Objectively: the left cheek skin has
aesthetic defect, speech disturbance due a lesion sized 2,5x1, 5 cm with regular
to the missing upper front tooth. Objecti- edges, the skin is dramatically thinned,
vely: the 12 tooth is missing. The 11, cyanotic, glossy, makes folds and bumps.
13 teeth are intact, stable, of anatomi- Intraoral palpation reveals a cord going
Krok 2 Stomatology 2013 21

from the 37 tooth to the cheek lesion. The A. Aluminum compounds


37 tooth is filled. What is the most likely B. Zinc compounds
diagnosis? C. Calcium compounds
D. Fluorine compounds
A. Odontogenic facial granuloma E. Pyrophosphate
B. Lymphadenitis of cheek
C. Furuncle of cheek 153. During examination of a 5-year-
D. Phlegmon of cheek old child the orthodontist revealed no
E. Carbuncle of cheek wear of teeth, no tremata and diastemata,
orthogenic occlusion. Which of the
150. A 40-year-old patient consulted a following symptoms in a 5-year old chi-
dental surgeon about difficult mouth ld is a sign of future teeth overcrowding?
opening. Clinical examination revealed
mouth opening limited to 1 cm. The 48 A. Absence of tremata and diastemata
tooth crown is decayed by 2/3, percussion B. Absence of wear of teeth
is strongly positive. What kind of peri- C. Orthogenic occlusion
pheral conduction anesthesia should be D. Orthognathic bite
applied for the surgical extraction of the E. Absence of mesial step in the region of
48 tooth? second temporary molars
A. Subzygomatic Berchet-Dubov 154. An 8-year-old child has been cli-
anesthesia nically diagnosed with exacerbation of
B. Mandibular anesthesia chronic periodontitis of the 84 tooth. The
C. Torusal (Weisbrem’s) anesthesia crown is decayed by 1/2. What is the denti-
D. Extraoral mandibular anesthesia st’s optimal tactics?
E. Mental anesthesia
A. Extraction
151. A 39-year-old patient had been admi- B. Endodontic treatment
tted to the maxillofacial surgery unit C. Endodontic treatment and drug therapy
for a swelling and bursting pain in the D. Drug therapy
left submandibular region. The swelling E. Opening along the mucogingival fold,
appeared after eating spicy foods. After drug thrapy
a physical examination, palpation and X-
ray the patient was diagnosed with sialoli- 155. A 21-year-old patient complains of
thiais of submandibular gland with the blood that oozes from a cavity in an upper
concrement localized in the anterior duct. jaw tooth during eating. Objectively: the
What tactics should be chosen by a dental approximal occlusal surface of the 27
surgeon after removal of the concrement tooth has a deep carious cavity filled wi-
from the duct? th soft tissue overgrowth of purple-red
colour that bleeds easily when touched
A. The wound and the duct are not to be by the probe. Probing causes a slight pain.
sutured and drained There is a slight pain reaction to cold sti-
B. The wound is to be sutured along the muli. EPT result is 60 mA. Ro-gram shows
line of incision no pathological changes. What is the most
C. The wound is to be sutured along the likely diagnosis?
line of incision and drained
D. Plasty of the excretory duct A. Chronic hypertrophic pulpitis
E. The excretory duct and the wound are B. Hypertrophic papillitis
to be sutured C. Chronic granulating periodontitis
D. Chronic fibrous periodontitis
152. A 14-year-old child has been lately E. Chronic gangrenous pulpitis
complaining about overgrowth and
permanent bleeding of gums during tooth 156. A 20-year-old female patient
brushing. Objectively: gingival papillae in complains of sore gums, purulent di-
the anterior part of both jaws are swollen, scharges from under the gums, tooth
hyperemic, overlap the tooth crowns by mobility. Objectively: the gums are
1/3 of their height, bleed easily during the markedly hyperemic, cover the tooth
instrumental examination. Caries intensi- crowns by 1/3. Periodontal pockets are
ty is of average level. Hygiene of the oral 7-8 mm deep with moderate purulent di-
cavity is not satisfactory. In this case it scharge. The 16, 26, 36, 31, 46, 41 teeth
is advisable to recommend a toothpaste are missing. The remaining teeth are
containing: characterized by the 2 grade mobility. Ro-
gram shows root exposure by 2/3, the deep
Krok 2 Stomatology 2013 22

bone niches in the alveolar bone. The skin onship between the canines and the fi-
keratinization of the extensor surfaces of rst permanent molars corresponds with
the elbows and knees is present, as well as Angle’s class I. Specify the dental anomaly
limb keratoderma. Nails are rough, dull. according to Kalvelis classification:
What disease can be suspected?
A. Anomaly of the dental arch shape
A. Papillon-Lefevre syndrome B. Anomaly of dental arch development
B. Generalized periodontitis C. Anomalies of individual teeth
C. Cyclic neutropenia D. Malocclusion
D. Eosinophilic granuloma E. Anomalies of jaw size
E. Gomphosis
161. A 12-year-old child has half retenti-
157. Examination of a 5-year-old child on of the 25 tooth, the dental arch lacks
revaled reverse overlap of the incisors and space for for 1/3 of the crown. The upper
canines. What is the most effective way of incisors overlap the lower ones by 2/3, the
abnormal bite prevention at this age? relationship of the first permanent molars
on the left corresponds with Angle’s class
A. Selective grinding of milk teeth 2. Rational design of orthodontic appli-
tubercles ance for the upper jaw should include:
B. Vestibular plate
C. Myogymnastics A. Unilateral screw for the 25 tooth and
D. Tongue frenulum plasty bite plate
E. Treatment with an orthodontic appli- B. Bilateral expansion screw
ance C. Radial symmetric screw
D. Radial asymmetric screw
158. A 67-year-old patient consulted a E. Lateral inclined plane
doctor about a formation on his upper lip
on the left, which appeared two months 162. A 2-month-old baby has acute
ago. Objectively: on the border of skin pneumonia complicated by an abscess.
and vermilion border of the upper lip on In course of treatment the baby has
the left there is a roundish cone-shaped developed soft tissue swelling of the left
formation up to 1 cm at the base, narrowi- supraorbital and infraorbital region due to
ng towards the apex, up to 1.3 cm high, an edema and inflammatory infiltrate. The
gray, dense, painless. What is the most li- child is in grave condition. What compli-
kely diagnosis? cation has developed?
A. Cutaneous horn A. Acute hematogenous osteomyelitis of
B. Keratoacanthoma the left upper jaw
C. Fibroma B. Facial vein thrombophlebitis
D. Neuroma C. Acute dacryocystitis
E. Nevus D. Nonodontogenic abscess of the left
infraorbital region
159. A 13-year-old child has been taken E. Nonodontogenic phlegmon of the left
to an orthodontist. Objectively: the chi- cheek
ld has permanent dentition. The 13 tooth
with mesio-distal dimensions of 8,9 mm 163. Objective examination of a 4-year-
erupted vestibularly outside the dental old child revealed caries in the 74, 84
arch, the interval between the 14 and 12 teeth, abnormal attachment of the upper
tooth is 9,2 mm. Select a rational plan of lip frenulum, which does not require surgi-
treatment: cal correction. The value of Fedorov-
Volodkina oral hygiene index is 1,8. How
A. Moving the 13 tooth into its proper many times a year the child should see a
place in the dental arch dentist if the average caries intensity in
B. Extraction of the14 tooth, moving the the respective age group in the given regi-
13 tooth in the distal direction into the on is 3?
dental arch
C. Expansion of the dental arch and movi- A. Once
ng the 13 tooth B. Twice
D. Finger massage of the 13 tooth C. 2-3 times
E. Extraction of the 13 tooth D. 3 times
E. 3-4 times
160. A 7,5-year-old child has square
dental arches of both jaws, the relati- 164. A 5-year-old child complains of
Krok 2 Stomatology 2013 23

spontaneous pain in an upper jaw tooth A. Gel containing microelements


on the right that is getting worse at ni- B. Paste containing salt additives
ght and during eating cold food. Objecti- C. Paste containing herbal extracts
vely: the 65 tooth has a deep carious cavi- D. Paste containing proteolytic enzymes
ty communicating with the tooth cavity. E. Paste containing chlorhexidine
Probing is painful, percussion is painless.
Cold water causes long-lasting pain. What 168. A 38-year-old female complains of
is your provisional diagnosis? experiencing pain in her lower lip for
half a year, difficult eating and talki-
A. Exacerbation of chronic pulpitis ng. The patient sought medical help at
B. Acute periodontitis place of residence, but the administered
C. Exacerbation of chronic periodontitis treatment was ineffective. Examination of
D. Acute serous pulpitis the lower lip revealed a deep fissure in
E. Acute purulent pulpitis the centre which affected the vermilion
border and partly the mucous membrane
165. A 56-year-old patient has indicati- of lips. The fissure was covered with a
ons for a clasp denture for the upper jaw. brown crust, after its removal the fissure
Objectively: the 15, 16, 17, 18, 25, 26, 27, 28 started bleeding. The fissure edges are
teeth are missing. What are the constructi- hyperkeratinized, infiltrated. Select the
on peculiarities of the denture in this optimal treatment tactics:
case?
A. Fissure excision
A. Indirect retainers B. Application of keratoplastic agents
B. Reduced number of clasp elements C. Irradiation with helium-neon laser
C. Increased occlusal surface of artificial D. Application of corticosteroid ointment
teeth E. Application of antibiotic ointment
D. Denture bar is located in the anterior
third of palate 169. An 18-year-old patient lives in an
E. Rigid clasp elements area with fluoride concentration in the
drinking water at the rate of 0,7 mg/l.
166. A 30-year-old patient consulted a The patient complains of white spots on
dentist about dull pain, numbness in the the front teeth, sensitivity to sweet food.
lower teeth, the lower lip and chin on Objectively: there are chalky spots on the
the right. Anamnesis data: the problems vestibular surface in the cervical part of
arose a week ago after the extraction of an teeth. Enamel has lost its gloss. Which of
impacted wisdom tooth on the right lower the following methods will allow to make
jaw. The patient developed the following a diagnosis?
disease:
A. Vital staining
A. Neuritis of the inferior alveolar nerve B. Radiography
B. Neuralgia of the third branch of the C. Thermal test
trigeminal nerve D. Probing
C. Neuritis of the facial nerve E. Electric pulp test
D. Neuralgia of the auriculotemporal
nerve 170. A 28-year-old patient has been
E. Neuritis of the trigeminal nerve scheduled for the surgical resection of
the root apex of the 12 tooth for chronic
167. A 56-year-old male patient complai- granulomatous periodontitis. What kind
ns of tooth hypersensitivity to all kinds of anesthesia should be given the patient
of stimuli. Objectively: gingival mucosa is before the surgery?
anemic, thinned, dense on touch, tooth
necks and roots are exposed by 1/3 of A. Bilateral infraorbital and incisive
their length, ache when touched with a B. Infraorbital and palatine
tool. Teeth are stable. There are wedge- C. Plexual and palatine
shaped defects in the cervical region of all D. Tuberal and palatine
the premolars. What kind of toothpaste E. Terminal and incisive
should be advised this patient for the
complex therapy of the disease? 171. During a dental visit a 50-year-
old patient suddenly developed tinni-
tus, nausea, dizziness, blackout, loss of
consciousness, dramatic paleness, myosis.
There were no pupillary reaction to light
and corneal reflexes. Heart sounds were
Krok 2 Stomatology 2013 24

muffled, blood pressure was decreased. are hyperaemic, edematic, pockets are 3,5
The described symptoms correspond with mm deep. X-ray picture shows resorption
the following medical emergency: of interalveolar septa by 1/3. What is the
most likely diagnosis?
A. Syncope
B. Anaphylactic shock A. Chronic generalized I degree peri-
C. Hypertensive crisis odontitis
D. Collapse B. Chronic generalized II degree peri-
E. Myocardial infarction odontitis
C. Exacerbation of generalized I degree
172. A 42-year-old female patient periodontitis
presented to a dentist for oral sanitation. D. Exacerbation of generalized II degree
She has a history of tonsillitis, rheumatoid periodontitis
arthritis, Botkin’s disease. What are the E. II degree periodontitis
conditions of instrument treatment after
the patient’s visit? 176. A 34-year-old male patient complains
of a cosmetic defect, a cavity on the vesti-
A. Under the special scheme bular surface in the cervical part of the
B. Under general conditions 21 tooth. Objectively: the carious cavity is
C. Dry-air sterilization within the enamel, the floor and the walls
D. Processing with lysol are pigmented, probing and percussion
E. Processing with 0,1% solution of and painless. There is no pain reaction to
chloramine stimuli. What is the most likely diagnosis?
173. A 22-year-old patient complains of A. Chronic surface caries
discolouration of the 11 tooth. Objecti- B. Acute surface caries
vely: the 11 tooth is pink, hard tissues C. Necrosis of dental hard tissues
are intact, percussion is painless, mucous D. Acute median caries
membrane around the tooth is unchanged. E. Chronic median caries
Radiography shows that the 11 tooth root
canal is filed to the top, there are no 177. A 14-year-old patient complains of
changes in the periapical tissues. The deep intense, throbbing pain in the region of the
incisal overlap is present. What is the most 46 tooth which appeared for the first time
suitable construction in this clinical si- two days ago. Objectively: the 46 tooth
tuation? is filled. Percussion causes acute pain, the
tooth is mobile, the mucous membrane
A. Porcelain veneer around the 46 tooth is hyperemic and
B. Combined Belkin crown edematous, palpation of the mucogingival
C. Porcelain-fused-to-metal crown fold in the root apex projection is painful.
D. Metal-plastic crown Ro-gram shows no changes in the peri-
E. Plastic crown apical region of the 46 tooth. What is the
most likely diagnosis?
174. An 8,5-year-old girl complains of pain
in the lower right molar during eating. A. Acute purulent periodontitis
Objectively: the occlusal surface of the 85 B. Acute localized periodontitis
tooth exhibits a carious cavity filled wi- C. Acute purulent pulpitis
th softened dentin. The cavity is located D. Acute serous periodontitis
within circumpulpal dentin and communi- E. Exacerbation of chronic periodontitis
cates with the tooth cavity. Probing causes
acute pain, moderate bleeding. Percussion 178. Mother of a 1,5-year-old child
is painless. What is the optimal treatment? complains of a formation on the child’s ri-
ght cheek. This formation appeared a few
A. Non-vital amputation months after birth. Objectively: on the ri-
B. Non-vital extirpation ght cheek there is a crimson formation
C. Vital extirpation sized 1,5-2,0 cm, painless, with a positive
D. Vital amputation sign of filling. What is the most likely di-
E. Biological method agnosis?
175. A 23-year-old patient complai-
ns of minor root exposure, gingival
haemorrhage during tooth brushing, gum
itch. Objectively: there is supragingival
and subgingival dental calculus. Gums
Krok 2 Stomatology 2013 25

A. Cavernous hemangioma of the right After clinical and radiographic exami-


cheek nation the patient has been diagnosed
B. Erysipelas with acute purulent periostitis of the
C. Hematoma of the right cheek upper jawmaxilla extending from the 11
D. Vascular nevus tooth; radiograph shows a slight wideni-
E. Lymphangioma of the right cheek ng of periodontal ligament space of the
11 tooth. Choose the best method of
179. A 28-year-old patient complains of treatment:
pain and bleeding of gums in the frontal
part of the upper jaw on the left. Two A. Subperiosteal abscess opening,
years ago, the 22 tooth was covered with endodontic treatment of the 11 tooth
a porcelain-fusd-to-metal crown. Objecti- B. Endodontic treatment of the 11 tooth,
vely: interdental papilla between the 21 anti-inflammatory therapy
and 22 tooth is hypertrophied, markedly C. Extraction of the 11 tooth
hyperemic, overlaps the crown of the 22 D. Extraction of the 11 tooth, anti-
by 1/3 of its height, bleeds when touched. inflammatory therapy
Periodontal pocket between the 21 and E. Extraction of the 11 tooth, subperiosteal
22 tooth is 4 mm deep. Artificial crown is abscess opening
located on gingival margin. Radiography
reveals resorption of the interalveolar 183. A 45-year-old patient presented to a
septa between the 21 and 22 tooth by 1/3 prosthodontics clinic. During the objecti-
of their height. Specify a priority action in ve examination the doctor checked the
the treatment of this patient: sagittal movements of the lower jaw.
What muscles are responsible for sagittal
A. Removal of the artificial crown movements of the lower jaw?
B. Gingivectomy
C. Anti-inflammatory therapy A. Lateral pterygoid muscles
D. Curettage of the periodontal pocket B. Medial pterygoid muscles
E. Sclerotherapy C. Mandibulohyoid muscle
D. Digastric muscle
180. A 13-year-old girl complains of gingi- E. Mentohyoid muscle
val haemorrhages during tooth brushing
and eating solid food. Objectively: the gi- 184. A 55-year-old patient presented to a
ngival mucosa of the upper and lower jaws prosthetic dentistry clinic for prostheti-
is somewhat edematic, hyperemic. The cs. During the patient examination
oral hygiene is inadequate. X-ray shows the movements of the lower jaw were
no changes. Make a diagnosis: checked. Specify the position of the heads
of mandible relative to the articular
A. Chronic catarrhal gingivitis tubercle with maximal mouth opening:
B. Acute catarrhal gingivitis
C. Hypertrophic gingivitis A. At the top of the articular tubercle
D. Desquamative gingivitis B. The heads slide over the surface of the
E. Chronic atrophic gingivitis articular tubercle
C. At the antrior edge of the articular
181. A 45-year-old patient undergoes tubercle
teeth preparation. 15 minutes after D. At the base of the articular tubercle
anesthetization with 4% solution of E. At the posterior edge of the articular
Ubistesin forte the patient developed tubercle
hyperemia of skin, increased heart rate,
headache, syncope. Previously the pati- 185. After examination a 42-year-old
ent had not exhibited such reaction to this patient has been diagnosed with acute
anesthetic. What complication occurred in purulent periostitis of the lower jaw
the patient? extending from the 35 tooth. What is the
most rational method of anesthesia for
A. Arterial pressure rise the 35 tooth extraction and lancing the
B. Anaphylactic shock subperiosteal abscess?
C. Syncope
D. Collapse A. Torusal (Weisbrem’s)
E. Myocardial infarction B. Berchet
C. Infiltration
182. A 23-year-old patient complains of D. Plexual
pain and swelling in the region of a central E. General anesthesia
incisor which was injured some time ago.
Krok 2 Stomatology 2013 26

186. A 10-year-old girl complains of cut into the alveolar bone. History record:
bleeding from the alveolar socket of the the teeth were partially decayed, changed
extracted 63 tooth lasting for three days. in colour. Select an appropriate treatment
The patient has a history of prolonged tactics:
bleeding after removal of primary teeth
and frequent nosebleeds. Objectively: A. Extraction of the injured teeth
oral mucosa is pale, there are small B. Follow-up as the teeth may erupt again
hemorrhages. The socket of the extracted C. Reposition of teeth, endodontic
63 tooth is filled with a big bulging clot treatment
with blood oozing from under it. Forearm D. Extraction of teeth, endodontic
skin has petechiae. What specialist should treatment, reposition
be visited in the first place? E. Treatment is not required
A. Hematologist 191. A 43-year-old patient complains of
B. Neuropathist mobility and significant neck exposure
C. Endocrinologist of the lower front teeth. Objectively: the
D. Infectious diseases specialist gums in the region of the 44, 43, 42, 41, 31,
E. Gastroenterologist 32, 33, 34 teeth are pale and cyanotic, non-
bleeding. The 42, 41, 31, 32 teeth exhibit
187. A 20-year-old male patient with the I-II grade mobility. The overcrowdi-
insulin-dependent diabetes mellitus has ng of the 42, 41, 31, 32 teeth is present.
been given anesthesia with Ultracain DS- The necks of the 42, 41, 31, 32 teeth are
forte for surgical oral sanitation. What exposed by 1/2 of the root length, the
complications may develop in this case? necks of the 43, 33 teeth are exposed by
1/4. What kind of dental prosthesis should
A. Abnormal levels of blood glucose be applied in this case?
B. Increased blood pressure
C. Clonus A. Cast removable splint
D. Atopic dermatitis B. Kurlyandsky splint bar
E. Toxic shock C. Cap splint
D. Portion crown
188. A 5-year-old boy has been di- E. Half-ring splint
agnosed with acute mumps. The chi-
ld is in a moderately grave condition, 192. A 7-year-old child has to undergo
body temperature is of 38, 6o C, the pati- plastic surgery of the upper lip frenulum.
ent complains of acute pain in the left What operation should be performed to
hypochondrium and epigastrium, radiati- lengthen the frenulum of the upper lip?
ng to the back. What complication has
developed in the child? A. Dieffenbach grafting
B. Thiersch grafting with local flaps
A. Pancreatitis C. Relaxing incisions of the mucosa
B. Orchitis D. Grafting with a pedicle flap
C. Epididymitis E. Szymanowsky grafting with local flaps
D. Mediastinitis
E. Gastritis 193. Routine examination of a 9-year-
old girl revealed symmetrical horizontal
189. A 6-year-old girl with acute purulent grooves on the vestibular surfaces of the
odontogenic lymphadenitis had been gi- 16, 12, 11, 21, 22, 26, 31, 32, 36, 41, 42, 46
ven intramuscular injection of cefazolin teeth. The grooves alternated with intact
(0,5 twice a day). 1,5 hour after the tooth tissues and were present in these
drug injection the child developed edema teeth at the time of their eruption. The
of lips, cheeks, forehead, ears, mucous patient has a history record of pneumonia
membrane of mouth and upper airways. at the age of 10 months treated with a
What complication developed in the girl? course of antibiotics. Make a diagnosis:
A. Angioneurotic edema A. Systemic hypoplasia
B. Urticaria B. Local hypoplasia
C. Allergic dermatitis C. Focal hypoplasia
D. Anaphylactic shock D. Dental fluorosis
E. Toxic shock E. Initial caries
190. As a result of an injury the 51and 61 194. A patient with a relevant history
teeth of a 3-year-old child have completely of allergic reactions had undergone tests
Krok 2 Stomatology 2013 27

for allergic sensitivity to metals. Accordi- inadequate. What method of dental health
ng to indications, the bridges of precious care may be appropriate in this case?
metal (gold) were fabricated. Specify the
warranty period starting from the day of A. Under general anesthesia
finishing the prosthetic procedures? B. Under local anesthesia injection
C. Under local application anesthesia
A. 12 months D. Without anesthesia
B. 3 months E. Giving sedatives
C. 6 months
D. 9 months 198. A patient with ischemic heart di-
E. 18 months sease has developed ventricular fibrillati-
on. What is the first-priority therapeutic
195. A 60-year-old patient needs a parti- action?
al removable denture for the upper jaw.
The central occlusion has been determi- A. Electric defibrillation
ned. What is the next stage of prosthetics? B. Lidocaine injection
C. Adrenaline injection
A. Checking the wax denture construction D. Potassium chloride injection
B. Try-in of the denture E. Novocaine amide injection
C. Fabrication of plaster models
D. Relining the denture 199. After an injection of 30 IU of inslulin
E. Taking impressions a diabetic patient developed a short-term
excitation followed by loss of consci-
196. A 12-year-old child complains of ousness. What is the most likely cause?
burning lips, pain at mouth opening,
yellowish crusts on the vermilion border. A. Hypoglycemic coma
Objectively: there are yellowish crusts on B. Stroke
the vermilion border extending from one C. Hyperglycemic coma
mouth corner to another, from the wet-dry D. Myocardial infarction
line to the middle of the vermilion border. E. Ketoacidotic coma
When the crusts are removed, the smooth
200. During the blood transfusion a pati-
bright red moist surface can be seen. What
ent developed seizures, tremor, lumbal
is the most likely diagnosis?
and retrosternal pain. Objectively: skin
A. Exfoliative cheilitis became hyperemic and later turned pale,
B. Atopic cheilitis the patient had hyperhydrosis, laboured
C. Meteorological cheilitis breathing, Ps at the rate of 110/min, AP-
D. Eczematous cheilitis 70/40 mm Hg, black urine. What compli-
E. Glandular cheilitis cation occurred in the patient?

197. Parents of a 3-year-old child took the A. Transfusion shock


child to a pedodontist for oral sanitati- B. Acute renal failure
on. The child has cerebral palsy. Objecti- C. Pulmonary embolism
vely: there is multiple dental caries, gingi- D. Anaphylactic shock
val mucosa is hyperemic, oral hygiene is E. Hypotensive crisis
Кrok 2 Stomatology (англомовнi студенти) 2014 рiк 1

1. After examination a 5-year-old paroxysmal, irradiating pain with short


child was diagnosed with acute general pain-free intervals. The pain arose 2
pulpitis of the 74 tooth. What treatment days ago and occurs only at night.
will be most efficient in this case? Make a provisional diagnosis:
A. Pulp extirpation A. Acute diffuse pulpitis
B. Vital amputation B. Acute deep caries
C. Non-vital amputation C. Exacerbation of chronic periodontitis
D. Biological method D. Acute circumscribed pulpitis
E. Tooth extraction E. Acute purulent pulpitis
2. A 38-year-old female patient 6. A 40-year-old female patient
complains of pain in the 16 tooth, which complains of swelling of the left
appeared 3 days after placing an artifi- submandibular region which appeared
cial crown over the tooth. Objectively: three days ago; elevation of body
the 16 tooth is crowned with a full metal temperature up to 37, 4o C . Objectively:
swaged crown. The gingiva around the the left submandibular region exhibits
tooth is hyperemic, edematous. The a soft tissue edema, the skin color is not
crown margin is submerged into gingi- changed. Palpation reveals a roundi-
val pocket by more than 0,5 mm. What sh dense elastic node with a smooth
is the cause of this complication? surface sized 2,0 x1,5 cm. The node
is mobile, somewhat tender. Mouth
A. Excessive submergence of the crown opening is not restricted, oral mucosa
margin into the gingival pocket exhibits no visible changes, the crown
B. Excessively tight fit of the crown of the 36 tooth is decayed by 2/3, its
C. Interdental contacts percussion causes pain response. What
D. Contact of the crown with antagoni- is the most likely diagnosis?
sts
E. Eating solid food A. Acute odontogenic serous
lymphadenitis of the submandibular
3. A 34-year-old female patient region
presented to a dental clinic for the B. Odontogenic abscess of the
prosthetics of the mandibular teeth. submandibular region
Objectively: the mandibular teeth exhi- C. Odontogenic phlegmon of the
bit the I-II class mobility. Which of the submandibular region
following examination methods should D. Chronic odontogenic lymphadenitis
be applied? of the submandibular region
E. Acute serous sialadenitis of the
A. Radiography submandibular salivary gland
B. Galvanometry
C. Face-bow record 7. A 70-year-old female patient
D. Rheotachygraphy complains of burning sensation under
E. Myotonometry the base of her complete denture;
dryness and bitterness in the mouth.
4. A 29-year-old female patient The denture was fabricated 3 weeks
presented to a dental clinic for ago. Objectively: the alveolar bone
prosthetics. Objectively: there is a cari- mucosa is edematous, there is diffuse
ous cavity on the masticatory and mesi- hyperemia up to the margins of the
al surfaces of the 24 tooth. Interdental denture. The tongue is hyperemic, dry
contact is inadequate. Classify this cavi- and cracked. What additional study
ty according to Black’s classification: would be most appropriate in this case?
A. II A. Allergotest
B. I B. Urinalysis
C. IV C. Complete blood count
D. III D. Biochemical blood analysis
E. V E. Sialosemiology
5. A patient complains of spontaneous, 8. A 44-year-old female patient
Кrok 2 Stomatology (англомовнi студенти) 2014 рiк 2

presented to a dental clinic for has a history of frequent pain in the 36


prosthetics. Objectively: the 17, 16, 15, mandibular tooth on the left. The face
14, 12, 25, 26 teeth are missing; the is asymmetric due to the swelling of
18, 28 teeth are preserved. Make a di- the left submandibular region, where a
agnosis according to Kennedy classifi- dense infiltrate is palpable; the overlyi-
cation of partial edentulousness: ng skin is hyperemic, doesn’t make a
fold. There is a deep carious cavity
A. Class III, subclass 2 in the 36 tooth, its percussion causes
B. Class III, subclass 1 pain response; the surrounding mucosa
C. Class III, subclass 3 is hyperemic, edematous. What is the
D. Class II, subclass 2 most likely diagnosis?
E. Class II, subclass 4
A. Phlegmon of the left submandibular
9. A 45-year-old female patient region
presents to a dental clinic for prostheti- B. Acute left-sided submandibular
cs. Objectively: the 47, 46, 45, 35, sialadenitis
36, 37 teeth are missing. There is C. Acute suppurative left-sided periosti-
dentoalveolar vertical displacement of tis of the mandible
the 17, 16, 26, 27 teeth; the alveolar D. Acute suppurative left-sided
process is hypertrophied, the cervical submandibular lymphadenitis
margins are not exposed. When the E. Acute left-sided osteomyelitis of the
teeth are closed, the distance between mandible
the occlusal surfaces of teeth and the
alveolar process is about 6 mm. Specify 12. A month after cementing a
the most efficient method of treatment porcelain-fused-to-metal crown for the
of secondary deformation in this pati- 23 tooth, the patient complained of its
ent: decementation. On examination the
tooth stump was of sufficient height,
A. Pulp removal and teeth grinding the stump walls were convergent to the
B. Surgical treatment vertical axis at an angle of about 30
C. Grinding degrees. What is the required angle of
D. Hardware-associated surgical wall convergence?
treatment
E. Disocclusion A. Up to 8 degrees
B. 18-20 degrees
10. A 25-year-old female patient C. 20-25 degrees
consulted a dentist about acute pain D. 25-30 degrees
in the mandible on the right that occurs E. 27-35 degrees
during eating. Objectively: the approxi-
mate distal surface of the 45 tooth 13. A 5-year-old child has been di-
exhibits a carious cavity filled with li- agnosed with chronic granulating peri-
ght softened dentin. Probing causes a odontitis of the 85 tooth. What is the
minor pain response across the entire optimal material for the root canal filli-
floor. Percussion causes no pain. Cold ng?
water causes transient pain. What is the
most likely diagnosis? A. Zinc-eugenol cement
B. Zinc-phosphate cement
A. Acute deep caries C. Glass ionomer cement
B. Acute median caries D. Calcium-containing paste
C. Chronic deep caries E. Resorcinol-formalin paste
D. Chronic median caries
E. Chronic fibrous pulpitis 14. A 65-year-old male patient
undergoes fabrication of a full
11. A 45-year-old male patient has removable denture for the mandible.
been admitted to the maxillofacial The individual tray for taking a functi-
department with complaints of general onal impression is adjusted with the use
weakness, pain and swelling in the of Herbst tests. The tray is thrown off
left submandibular region, elevation during swallowing. This means that its
of body temperature up to 39oC . He edge is to be shortened in the following
Кrok 2 Stomatology (англомовнi студенти) 2014 рiк 3

area: Which anatomical structure takes the


greatest load in this situation?
A. From the space behind the mucous
tubercle to the mylohyoid line A. Articular disc (meniscus)
B. From canine to canine vestibularly B. Articular head
C. From canine to canine lingually C. Distal surface of the medial articular
D. Along the mylohyoid line tubercle
E. In the region of the premolars on the D. Bottom of the glenoid fossa of the
lingual side temporal bone
E. Articular capsule
15. A 28-year-old female patient
complains of a cosmetic defect in 18. A 70-year-old male patient with the
the anterior segment of the maxilla. completely edentulous maxilla and a
Objectively: the crown of the 13 tooth hard palate defect complains of difficult
is decayed nearly up to the gingival eating, pouring of liquid food through
edge, the stump walls are of suffici- the nose, frequent chronic diseases
ent thickness. The stump has a sli- of the nasal mucosa. Objectively: the
ghtly changed color, percussion causes maxillary alveolar bone is atrophied
no pain response. Radiographical fi- (Schroeder class I), the mucosa pli-
ndings: the root canal is straight, wi- ability complies with Supple class I.
de, and filled along the entire length. The hard palate exhibits a cicatrically
What prosthetic construction is most changed defect sized 2x3 cm. What
appropriate in this case? denture design would be most effective
in this case?
A. Post and core
B. Plastic crown A. Complete removable denture with
C. Porcelain-fused-to-metal crown an internal and a peripheral valve
D. Inlay B. Complete removable denture with
E. Micro-denture the teeth placed on the artificial gingiva
C. Soft-core obturator and complete
16. A 32-year-old patient complains of removable denture
constant chewing-like movements of D. Complete removable denture with a
the mandible. These movements occur thickening in the defect region
with nervous excitement. The patient’s E. Complete removable denture with a
wife reports him to have gnashing of metal base and soft-core obturator
teeth at night. Objectively: the lateral
teeth are worn, have no tubercles. 19. A 7-year-old child complains
What is the optimal treatment tactics? of fever up to 38, 3o C , headache,
sore throat and general weakness.
A. Night guard (Michigan-type splint), Objectively: the mucosa of the soft
myogymnastics, self-massage palate, tonsils and palatine arches
B. Restoration with crowns is edematous and hyperemic, there
C. Pharmaceutical and non-invasive are numerous small painful erosi-
treatment following the referral to a ons with red bottom. The erosions
dental surgeon are not covered with any plaque.
D. Referral to a dental surgeon The submandibular lymph nodes are
E. Alignment of teeth (selective grindi- enlarged, painful on palpation. What is
ng) the most likely diagnosis?
17. A 38-year-old female patient A. Herpetic angina
complains of pain in front of the B. Acute herpetic stomatitis
external auditory meatus projection, C. Infectious mononucleosis
clicking when opening the mouth, D. Scarlet fever stomatitis
sensation of stuffiness in the ears. E. Diphtheric stomatitis
Objectively: the face is symmetric, the
trajectory of mouth opening is straight. 20. A 22-year-old patient complains of
There is a I class Kennedy defect, the a painful swelling in the right parotid
18, 17, 16, 26, 27, 28 teeth are missing. gland region. A week before the pati-
Кrok 2 Stomatology (англомовнi студенти) 2014 рiк 4

ent got a cheek abrasion which was extraction of the maxillary wisdom
healing under the purulent crust. Over tooth root. Following the posteri-
the past two days, the patient had or superior alveolar nerve block, a
had progressing pain and fever up to post-injection hematoma occurred.
38, 6o C . Objectively: there is a soft ti- What structure was damaged during
ssue edema in the right parotid region, anesthesia?
the skin is slightly tense but has not
changed in colour. There is a dense pai- A. Pterygoid venous plexus
nful infiltration 2,5x3,5 cm large, the B. Maxillary artery
overlying skin exhibits limited mobili- C. Infraorbital artery
ty. The mouth can be fully opened, the D. Zygomatic artery
mucous membrane around the orifice E. Palatine artery
of the salivary duct is unchanged, the
saliva is transparent. What is the most 24. What is the most adequate surgi-
likely diagnosis? cal approach for a submandibular
phlegmon dissection?
A. Acute lymphadenitis
B. Exacerbation of chronic parotitis A. 5-6 cm long incision 2 cm below the
C. Abscess of the parotid-masseteric mandible
region B. 5-6 cm long incision bypassing the
D. Acute non-epidemic parotitis angle of the mandible
E. Epidemic parotitis C. Intraoral approach along the mucogi-
ngival junction at the molar level
21. A 42-year-old female pati- D. Linear 2 cm long incision along the
ent consulted a dentist about a lower edge of the mandible
sudden dramatic enlargement of gi- E. 5-6 cm long incision along the skin
ngiva, bleeding sores on the mucous crease of the upper cervical region
membrane, lymph node enlargement.
There is body temperature elevation up 25. A 12-year-old patient has chronic
to 38o C , weakness, pain in the bones. catarrhal gingivitis. The oral vestibule
What disease can be suspected in this is 7 mm deep. Fedorov-Volodkina hygi-
patient? ene index is 4. Examination revealed
no abnormalities of lip frenulum
A. Acute leukemia attachment. What measures should be
B. Chronic leukemia taken in the first place?
C. Hypovitaminosis C
D. Hodgkin’s lymphoma A. Professional oral hygiene
E. Eosinophilic granuloma B. Vestibuloplasty
C. Curettage of periodontal pockets
22. A 56-year-old male patient D. Physiotherapy
presents to a dental clinic for complex E. -
dental treatment prior to prosthetic
procedures. Objectively: there is a 26. On the basis of subjective complai-
pathological grayish-white triangular nts, anamnesis and objective exami-
lesion sized 0,5x1,5 cm on the buccal nation, a 5-year-old child has been
mucosa in the mouth corners. The lesi- diagnosed with impacted dislocation
on cannot be removed. There are no of the 71 tooth. What is the most
signs of inflammation, regional lymph appropriate treatment tactics?
nodes are unchanged. Make a di- A. Extraction of the 71 tooth
agnosis: B. Reposition and fixation of the 71
A. Planar leukoplakia tooth
B. Mild leukoplakia C. Orthodontic treatment of the 71
C. Typical lichen planus tooth
D. Mycotic stomatitis D. Endodontic treatment of the 71
E. Hyperkeratotic lichen planus tooth
E. No intervention is required
23. A 24-year-old female patient
presented to a dental clinic for the 27. A 26-year-old male patient presents
Кrok 2 Stomatology (англомовнi студенти) 2014 рiк 5

to a prosthetic dentistry clinic for a A. Standard double arch


swaged metal crown for the 26 tooth. B. Aluminium arch bar
This is his third visit. During examinati- C. Plastic biteplate
on he has been found to have a hole on D. Weber’s splint
the medial buccal tubercle of the crown E. Crown-supported soldered splint
which appeared as a result of polishing.
What tactics should be chosen? 31. A 25-year-old male complains of
pain, a sensation of heaviness in the
A. Fabricate a new crown maxilla on the left. He has a history
B. Seal up the hole in the crown of the 25 tooth extraction followed by
C. Cement the crown with the cement health deterioration. There appeared
D. Mount a fixed denture purulent discharge from the left nasal
E. Cement the crown with acryl oxide meatus. He was diagnosed with acute
odontogenic left-sided sinusitis. Which
28. A 28-year-old patient complains nasal meatus should the maxillary sinus
that a piece of the 23 tooth crown be punctured through?
has broken off. Objectively: the root
canal orifices contain the filling materi- A. Inferior
al, percussion causes no pain response. B. Superior
What is the tactics of choice? C. Middle
D. Superior and inferior
A. Radiography of the 23 tooth E. Middle and inferior
B. Extraction of the 23 tooth
C. Fabrication of a metal insert tooth 32. A 22-year-old male patient
D. Ortho-polymer restoration of the 23 presented to a dental clinic with a
tooth cavity in the 11 tooth. The pati-
E. Restoration with Belkin crown ent had no previous history of this
tooth treatment. Objective examinati-
29. A 58-year-old patient complains of on revealed a carious cavity filled with
stuffiness in the ears, hearing impai- light softened dentin on the approxi-
rment, pain in the parotid region when mate medial surface. The carious cavity
opening the mouth. Objectively: the 18, was located within the mantle dentin.
17, 16, 15, 26, 27, 28 teeth are restored The tooth is slightly sensitive to the
with swaged metal crowns. There is a cold stimuli. What is the most likely
1,5-2 mm gap between the crown edges diagnosis?
and gingiva. The 14 and 25 teeth exhi-
bit carious cavities. Probing, percussi- A. Acute median caries
on and thermal stimuli cause no pain B. Acute deep caries
response. The patient has deep incisor C. Chronic median caries
overbite. What is the likely cause of D. Chronic deep caries
otologic symptoms? E. Superficial caries
A. Functional TMJ overload 33. A patient consulted an oral surgeon
B. Complication of tooth decay in the about maxillary pain on the left, whi-
14, 25 teeth ch arose three days ago. After exami-
C. Missing mandibular teeth nation, the patient was diagnosed wi-
D. Cervical margin exposure of the 18, th exacerbation of chronic periodonti-
17, 16, 15, 26, 27, 28 teeth tis of the 17 tooth. The 17 tooth is
E. Galvanosis indicated for extraction. What nerves
should be blocked for a painless
30. A 32-year-old patient presented extraction of the 17 tooth?
to a prosthetic dentistry clinic with
a diagnosis of maxillary fracture.The
fracture can be treated by means of
standard maxillo-mandibular Zbarzh
set. What construction provides the
intra-oral fixation?
Кrok 2 Stomatology (англомовнi студенти) 2014 рiк 6

A. Posterior alveolar nerves and greater A. In the oral cavity by means of sili-
palatine nerve cone materials
B. Greater palatine nerve B. By means of visual models in the
C. Anterior alveolar nerves and incisive articulator
nerve C. In the oral cavity by means of transfer
D. Middle alveolar nerves and greater paper
palatine nerve D. In the oral cavity by means of a wax
E. Nasopalatine nerve plate
E. In the oral cavity by means of
34. A 35-year-old patient complains stomatoscopy
about itch, burning and edema of li-
ps. These presentations occured a week 37. A 44-year-old male patient needs a
ago. Objectively: there is reddening of cast splint for the mandible. Objecti-
vermilion border and lip skin, especi- vely: there is a bilateral terminal defect
ally in the region of mouth corners, of the mandibular dental arch. The
there are also vesicles, crusts, small tooth crowns are high, the teeth are
cracks accompanied by erythematous intact, there is I-II class mobility. What
lesion of vermilion border. What is the impression material should be used?
most likely diagnosis?
A. Stomalgin
A. Acute eczematous cheilitis B. Repin
B. Exudative erythema multiforme C. Gypsum
C. Acute herpetic cheilitis D. Stens
D. Allergic contact cheilitis E. Dentafol
E. Exudative form of exfoliative cheilitis
38. A 35-year-old male complains of
35. A 6-year-old child complains of short-term pain caused by thermal sti-
pain in a mandibular tooth on the left muli in the 46 and 47 teeth. Objecti-
during eating. Objectively: masticatory vely: masticatory surfaces of the 46, 47
surface of the 36 tooth exhibits a cari- teeth are intact, approximate surfaces
ous cavity within the mantle dentin. could not be examined because of a
The cavity is full of light softened very close arrangement of teeth. What
dentin which can be easily removed wi- methods of study can be used in this
th an excavator. Probing of the cavity case if you suspect the presence of hi-
walls causes pain response. The teeth dden cavities?
are sensitive to thermal stimuli, the
pain is of short-term nature. There is A. X-ray
no response to percussion. Select the B. Vital staining
optimal filling material: C. Fluorescent study
D. Measuring electrical resistance
A. Silver amalgam E. Electro-odontometry
B. Zinc phosphate cement
C. Silicate cement 39. A 13-year-old child complains of
D. Silicophosphate cement having a cavity in the front maxillary
E. Polycarboxylate cement teeth. Contact medial surfaces of the
11 and 21 teeth exhibit cavities found
36. A patient undergoes orthopedic within the mantle dentine and filled
treatment for bounded edentulous with dense pigmented dentin. Probi-
spaces in the upper jaw with fixed full- ng of the cavity floor causes no pain
cast dentures. At the second appoi- response, neither does dental percussi-
ntment it is necessary to check whether on. Select the best filling material for
the inner surface of the metal frame the permanent seals:
of the future porcelain-fused-to-metal
bridge fits the surface of the prepared A. Resin composite
teeth. How can this be done? B. Silicate cement
C. Silicophosphate cement
D. Zinc phosphate cement
E. Glass ionomer cement
Кrok 2 Stomatology (англомовнi студенти) 2014 рiк 7

40. Parents of a 3-year-old child report ns of inertness, headache, fever up


that the child has persistent pain in to 39o C , burning neuralgic pain in
the front maxillary teeth. Objecti- the mandible on the right. Objecti-
vely: the coronal part of the 61 tooth vely: the skin of chin on the right is
is gray and decayed. Probing of the hyperemic, there is vesicular rash wi-
root canal orifice causes pain response th a vesicle diameter of 1 to 6 mm.
and is accompanied by bleeding. The Similar vesicles can be found on the
tooth percussion provokes acute pain. hyperemic oral mucosa on the right.
Mucosa is hyperemic, edematous and They burst easily making erosions whi-
tender. Palpation in the region of the ch are covered with fibrinous pelli-
61, 62 teeth reveals a fistula. What is cles. Regional lymphadenitis is present.
your provisional diagnosis? What is the most likely diagnosis?
A. Exacerbation of chronic periodonti- A. Shingles
tis B. Acute herpetic stomatitis
B. Acute purulent periodontitis C. Allergic stomatitis
C. Acute diffuse pulpitis D. Pemphigus
D. Chronic granulating periodontitis E. Exudative erythema multiforme
E. Exacerbation of chronic pulpitis
44. A 50-year-old patient complai-
41. A 15-year-old teenager consulted ns of dryness and burning sensati-
an orthodontist about misalignment on in the mouth, pain during eati-
of teeth. Objectively: the face ng, taste perversion. The symptoms
is unremarkable. The patient has occurred a week ago after taking the
permanent dentition. There are no course of antibiotics for acute bronchi-
abnormalities of jaw relationship in tis. Objectively: oral mucosa is covered
three dimensions. The 23 tooth has a with patches of white caseous deposit.
vestibular position and is located above The deposit can be easily removed
the occlusal plane, the space in the when scraped, revealing hyperemic
dental arch is less than 1/3 of crown. areas of mucous membrane. Which is
The space for the misaligned 23 tooth the most likely diagnosis?
can be gained due to:
A. Acute pseudomembranous candidi-
A. Extraction of the 24 tooth asis
B. Increase of the transverse jaw di- B. Chronic hyperplastic candidiasis
mensions C. Leukoplakia
C. Increase of the sagittal jaw dimensi- D. Lichen planus
ons E. Chronic recurrent aphthous stomati-
D. Extraction of the 23 tooth tis
E. Increase of the vertical jaw dimensi-
ons 45. A 56-year-old female patient
complains of burning neuralgic pain
42. An 11-year-old boy complains of a in the left half of the mandible, whi-
carious cavity in a mandibular tooth ch appeared two days ago. There is also
on the right. Objectively: the 46 tooth rash in form of tense vesicles on the
exhibits a carious cavity within the reddened skin and mucosa of the lower
mantle dentin. The dentin is dense, pi- lip on the left, accompanied by malaise,
gmented; there is pain response to the headache, body temperature elevation
cold stimulus, probing and percussion up to 38o C , left-sided lymphadenitis.
cause no pain response. Make a provi- What is the most likely diagnosis?
sional diagnosis:
A. Herpes zoster
A. Chronic median caries B. Murrain
B. Acute median caries C. Acute herpetic stomatitis
C. Acute deep caries D. Allergic reaction
D. Chronic deep caries E. Pemphigus
E. Chronic superficial caries
46. A 21-year-old dental patient has
43. A 45-year-old male patient complai- been found to have a circular ulcer on
Кrok 2 Stomatology (англомовнi студенти) 2014 рiк 8

the tongue dorsum. The ulcer is up to segment the depth of incisal overbite is
1 cm in diameter, has demarcated rai- 1/3 of the crown height. What appliance
sed edges and a deep inflammatory infi- can be used for simultaneous treatment
ltrate at the base. The ulcer floor is flat, of diastema and crown divergence?
clean, of crimson color. The regional
lymph nodes are of cartilaginous densi- A. Maxillary plate with spring arms
ty, mobile, painless. The ulcer appeared B. Maxillary plate with protraction
a week ago and doesn’t cause any di- springs
scomfort. What kind of ulcer should be C. Standard edgewise technique
suspected in the first place? D. Bracket system with Andrew’s
straight-wire
A. Syphilitic E. Maxillary plate with vestibular arch
B. Trophic
C. Traumatic 50. A 52-year-old female patient
D. Tuberculous complains of a persistent burning
E. Cancerous sensation at the tip and lateral surface
of the tongue. The burning occurs
47. An 11-year-old boy complains of in the morning and gets worse in
a short-term pain from the cold in a the evening, the sensation disappears
left mandibular tooth. Objectively: the when eating. The patient reports havi-
medial surface of the 36 tooth exhibi- ng had these symptoms for about a
ts a carious cavity within parapulpar year and associates the disease with
dentin. The cavity is filled with li- wearing removable dentures. The pati-
ght, softened dentin and does not ent has a history of anacid gastritis,
communicate with the cavity of the a surgery for uterine fibroids, second
tooth. Probing the of the 36 tooth floor degree hypertension, poor sleep, anxi-
causes pain response, the tooth is not ety. Objectively: the tongue is sli-
sensitive to percussion, the response to ghtly swollen, the tongue dorsum is
the cold stimulus does not stay long covered with a thin white plaque, fi-
after its removal. What is the most li- liform papillae are atrophied. The
kely diagnosis? rest of the mucosa exhibits no visi-
ble pathological changes. Pharyngeal
A. Acute deep caries reflex is suppressed. What is the most
B. Acute focal pulpitis likely diagnosis?
C. Acute diffuse pulpitis
D. Chronic fibrous pulpitis A. Glossalgia
E. Acute median caries B. Allergic reaction to plastic
C. Moeller-Hunter glossitis
48. A 36-year-old patient with the face D. Candidal glossitis
and neck burns resulting from gasoli- E. Lingual nerve neuritis
ne ignition has been delivered to the
emergency room. Palpebral fissures are 51. A 28-year-old female patient
closed due to the eyelid edema. The consulted a dentist about pain caused
face skin is erythematous, there are by thermal and chemical stimuli in
large tense blisters on the skin of the the mandibular and maxillary teeth.
chin, cheeks, nose, forehead, superior Objectively: in the cervical zone of
surface of neck. Specify the degree of mandibular and maxillary incisors and
burns: canines there are irregularly shaped
defects of hard dental tissues within
A. II degree the amelodentinal junction in form of
B. I degree tissue softening. In this clinical case a
C. III degree dentist will find it difficult to differenti-
D. IV degree ate between the following diseases:
E. -
49. A 9-year-old child has a symmetri-
cal maxillary diastema with crown
divergence. The relationship of the
lateral teeth is neutral. In the anterior
Кrok 2 Stomatology (англомовнi студенти) 2014 рiк 9

A. Caries and necrosis A. Panoramic radiography


B. Hypoplasia and necrosis B. Schiller-Pisarev test
C. Hypoplasia and wedge-shaped defect C. Measuring the depth of periodontal
D. Caries and erosion pockets
E. Erosion and wedge-shaped defect D. Calculation of the periodontal index
E. Determining the class of tooth mobi-
52. A 52-year-old male patient worki- lity
ng as a driver consulted a dentist about
pain, difficult mouth opening. The pati- 55. A 7,5-year-old boy fell down on
ent had been diagnosed with ulcerative his face and damaged his front maxi-
leukoplakia. After a 10-day course of llary teeth. Objectively: the crowns of
therapy sluggish granulation began. On the 11 and 12 teeth are destroyed by
the periphery of the ulcer the keratini- 1/4. The tooth cavity is closed. Probi-
zation processes became more active. ng causes pain along the break-off line,
Which of the following tests must be percussion of the 11 and 12 teeth causes
conducted in the first place? no pain response. Mucosa around the
11 and 12 teeth is slightly hyperemic.
A. Microscopy Radiograph of the 11 and 12 teeth
B. Bacterioscopy shows the intactness of the dental ti-
C. Stomatoscopy ssues and alveolar process. What tacti-
D. Bacteriological stdy cs of a dentist would be optimal in the
E. Cytological study first hours after the injury?
53. 4 hours after application of arsenic A. Pulp monitoring
paste to the 36 tooth pulp, a 27-year- B. Fabrication of an artificial crown
old patient complained of pain in the C. Photopolymer filling
treated tooth. Objectively: on the distal D. Parapulpar pin reconstruction
surface of the 36 tooth the temporary E. Composite filling
filling is present. Percussion causes no
pain response. Gingival papilla in the 56. A 9-year-old child complains of
region of the 36, 37 teeth is hyperemic pain in the left mandibular molar
and edematous, its apex is necrotized. that occurs during eating. The masti-
What is the optimal tactics of this pati- catory surface of the 75 tooth exhibi-
ent management? ts a carious cavitiy filled with softened
dentin and localized within parapulpar
A. Replace the temporary filling, treat dentin. The cavity communicates wi-
the gingiva with 3% iodine solution th the tooth cavity. Probing at the si-
B. Keep the temporary filling, admini- te of communication causes acute pain
ster soda rinse and moderate bleeding. Percussion of
C. Keep the temporary filling, treat the the tooth causes no pain response.
gingiva with 3% iodine solution Radiography revaled no periodontal
D. Replace the temporary filling, treat alterations in the 75 tooth. Select the
the gingiva with 3% hydrogen peroxide method of treatment:
solution
E. Keep the temporary filling, treat the A. Devital amputation
gingiva with 3% hydrogen peroxide B. Devital extirpation
solution C. Vital extirpation
D. Vital amputation
54. A 43-year-old female patient has E. Biological method
been diagnosed with generalized peri-
odontitis. What examination is most 57. Preventive examination a 6-year-old
informative for determining the severi- child revealed that the child had deci-
ty grade of the disease? duous dentition, direct incisor contact
in the frontal segment, no gaps between
teeth; contact of homonymous canines
and molars; abrasion of masticatory
tubercles of the molars. The child’s
condition corresponds with the followi-
Кrok 2 Stomatology (англомовнi студенти) 2014 рiк 10

ng period: agnosis can be made?


A. Physiological wear of deciduous A. Upper lip furuncle
dentition B. Acute abscess of the upper lip
B. Development of deciduous dentition C. Retention cyst
C. Stable deciduous dentition D. Acute lymphadenitis
D. Mixed dentition E. Upper lip carbuncle
E. Exfoliation
61. A 20-year-old female patient
58. A 12-year-old girl visited a dentist consulted a dentist about a short-term
for a regular check-up. Objectively: the pain in the 22 tooth caused by cold sti-
teeth are intact, the bite is open, the muli or contact with food. The pain
mucosa in the papillary and marginal had been observed for several months.
regions is hyperemic and cyanotic, the After the examination, the patient was
gingival papillae are roundish, enlarged diagnosed with acute deep caries of the
and cover one third of the crown height 22 tooth. What material should be used
of the anterior maxillary teeth. Make a as a base for the cavity filling?
provisional diagnosis:
A. Calcidont
A. Hypertrophic gingivitis B. Paste with trypsin
B. Chronic catarrhal gingivitis C. Phosphate cement
C. Acute catarrhal gingivitis D. Silver-containing phosphate cement
D. Gingival fibromatosis E. Silidont
E. -
62. A 48-year-old female patient
59. A 29-year-old male patient complai- underwent parotidectomy for a beni-
ns of gingival hemorrhages duri- gn tumor of the parotid gland.
ng tooth brushing, and bad breath. Postoperatively, the patient was
Objectively: gingival papillae and observed to have mouth downturni-
marginal gingiva are hyperemic with ng, midline deviation towards the
a cyanotic tint, bleed on probing. Peri- unaffected side, speech disturbance.
odontal pockets are 3-3,5 mm deep, wi- Which branch of the facial nerve was
th serous exudate expressed from them. damaged during the surgical interventi-
The sub- and supragingival calculus on?
is present. Orthopantomograph shows
osteoporosis of the alveolar process, A. Marginal mandibular branch of
resorption of interalveolar septa to 1/3. facial nerve
What is the most likely diagnosis? B. Buccal branch
C. Zygomatic branch
A. Chronic generalized I grade peri- D. Cervical branch
odontitis E. Temporal branch
B. Exacerbation of chronic generalized
I grade periodontitis 63. On examination a 42-year-old
C. Chronic generalized periodontitis, patient was diagnosed with acute
initial stage suppurative periostitis of the mandible
D. Exacerbation of chronic generalized beginning from the 35 tooth. What is
periodontitis, initial stage the most rational method of anesthesia
E. Chronic generalized II grade peri- for the 35 tooth extraction and dissecti-
odontitis on of the subperiosteal abscess?
60. A 40-year-old patient complains A. Torusal anesthesia
of fever up to 38o C , and a roundish B. Berchet anesthesia
infiltration on the upper lip. Objecti- C. Plexus anesthesia
vely: the upper lip on the left exhibi- D. Infiltration anesthesia
ts a round infiltrate, the overlying skin E. General anesthesia
is deep crimson. The infiltrate adheres
to the surrounding tissues and has a 64. Examination of a 31-year-old pati-
necrotic core in the center. The upper ent revealed perforation of the pulp
lip is hyperemic, edematous. What di- chamber floor of the 36 tooth with a
Кrok 2 Stomatology (англомовнi студенти) 2014 рiк 11

minor radiolucency of the interradi-


cular septum. Which of the following A. Low-fusible alloy
methods of surgical treatment should B. Stainless steel
be applied in order to save the 36 tooth C. Cobalt-chromium alloy
and use it as an abutment tooth for a D. Silver-palladium alloy
bridge denture? E. Solder alloy
A. Coronary radicular separation 68. Examination of a 9-year-old chi-
B. Root amputation ld revealed chalky spots in the cervi-
C. Root hemisection cal zone of the 12, 11, 21, 22 teeth.
D. Tooth replantation The spots appeared two weeks ago.
E. Sealing of perforation Lustreless surface of the spots is
stainable with methylene blue. The
65. A 42-year-old patient complains affected teeth are not sensitive to the
of pain in the right side of her head, cold stimulus. Specify the physician’s
restricted movements of the lower tactics in respect of the affected teeth:
jaw, clicking sound, periodic spasms
of chewing muscles. Objectively: the A. Remineralizing therapy
face is symmetric, mouth opening is B. Impregnation therapy
restricted. On palpation of the ri- C. Grinding of affected areas
ght temporo-mandibular joint (TMJ) D. Case follow-up
there are crepitation and clicking that E. -
accompany the mandible movements. 69. An 18-year-old male has a furuncle
Examination of the oral cavity revealed in the region of the mandible angle.
also a Kennedy’s class II defect on the The surrounding tissues are rigid.
right. What is the most likely diagnosis? Cyanosis and induration in form of a
A. Pain dysfunction of the right TMJ cord extend to the corner of the eye.
B. Acute arthritis of the right TMJ General condition is deteriorated, body
C. Sclerosing osteoarthritis of the right temperature is of 38, 5oC . What compli-
TMJ cation has developed in this case?
D. Contracture of the right TMJ A. Facial vein thrombophlebitis
E. Myositis ossificans B. Abscess of cheek
66. A 40-year-old patient complai- C. Phlegmon of cheek
ns of malaise, weakness, headache, D. Acute sinusitis
fever up to 38, 5oC . Objectively: there E. Carbuncle of cheek
is a symmetric red lesion in form of 70. Regimental aid station admitted a
butterfly-wing rash on the face around wounded soldier with a diagnosis of
the nose and cheeks. The lesion is a gunshot fracture of the mandibular
clearly demarcated, tender on palpati- body. What kind of immobilization is
on, does not disappear when pressed
upon. The skin is raised and glossy. required at this stage?
Submandibular and parotid superfici- A. Transport immobilization
al lymph nodes are enlarged, tender to B. Immobilization with splints for both
palpation. What is the most likely di- jaws
agnosis? C. Extraoral appliance for fragment
fixation
A. Facial erysipelas D. Smooth brace
B. Systemic lupus erythematosus E. Osteosynthesis of the mandible
C. Furuncle of face
D. Microbial eczema 71. A 17-year-old male patient complai-
E. Neurodermatitis ns about having a fistula on the anterior
surface of his neck. The fistula has been
67. A 43-year-old male patient awai- present since birth. Periodically there
ts fabrication of full swaged crowns appears a small amount of mucoid
for the 26 and 27 teeth. What materi- discharge. The patient has been di-
al should be used for the dental dies? agnosed with a middle cervical fistula.
Кrok 2 Stomatology (англомовнi студенти) 2014 рiк 12

What tactics of treatment should be A. Extraoral Rudko apparatus


preferred for this patient? B. Two-jaw splint with wire loops and
intermaxillary elastics
A. Excision of the fistulous tract with C. Bone suture
resection of the hyoid bone body D. One-jaw brace
B. Excision of the fistulous tract without E. Immobilization by means of wires
resection of the hyoid bone body and rods
C. Closure of the fistula with the purse-
string suture 75. 3 days ago a 35-year-old patient
D. Cryodestruction of the fistula epi- underwent the extraction of the 47
thelium tooth, which was followed by health
E. Sclerosing of the fistula epithelium deterioration. Objectively: the pati-
ent is pale, body temperature is of
72. A 70-year-old patient complains of 38, 1oC , the mandible on the left is thi-
inability to take food, a cosmetic and ckened, the surrounding soft tissues
phonetic defect due to the complete are swollen, mouth opening is difficult.
loss of mandibular teeth. Objectively: Mucogingival junction at the 48, 47, 46
the lateral segment of the alveolar teeth is edematous, mucous membrane
process of the mandible is significantly is hyperemic. There are purulent di-
atrophied, while the frontal segment is scharges from the 47 tooth socket, the
relatively preserved. Buccal folds are 48, 46, 45 teeth are sensitive to percussi-
attached at the crest of the alveolar on. There is also paresthesia in the
process. These clinical presentations lower lip region. What is the most li-
correspond with the following class of kely diagnosis?
edentulous jaws according to Keller
classification: A. Acute mandibular osteomyelitis
B. Acute mandibular periostitis
A. Class III C. Submandibular phlegmon
B. Class II D. Acute periodontitis of the 47 tooth
C. Class I E. Abscess of mandibular radicular cyst
D. Class IV
E. Class V 76. A female patient presented to a
clinic for prosthetics. Objectively: the
73. A 24-year-old male who had been lower third of the face is shortened;
injured in a fight was delivered to the nasolabial folds are deepened; the front
maxillofacial department. The patient teeth of both jaws are missing; the
was diagnosed with a fracture of the left crowns of the 17, 15, 26, 27, 36, 37, 45,
zygomatic bone with a displacement; 47 teeth are worn by 2/3 of their height;
a fracture of the anterior wall of the the chewing surfaces of these teeth are
maxillary sinus. What kind of surgery is smooth and pigmented; alveolar bone
indicated for this patient? is not hypertrophied, interalveolar hei-
ght is decreased. Specify the form of
A. Radical maxillary sinusotomy and pathological dental abrasion in this
reposition of fragments patient:
B. Radical maxillary sinusotomy
C. Osteotomy of the maxilla A. Horizontal, uncompensated, III
D. Osteosynthesis of the zygomatic grade of severity
bone B. Horizontal, compensated, III grade
E. Reposition of fragments of severity
C. Vertical, uncompensated, III grade
74. A 50-year-old male patient has a of severity
gunshot wound to the mandible and a D. Vertical, compensated, III grade of
bone defect in the mental area. What severity
method of immobilization is indicated E. Combined, uncompensated, III
for this patient? grade of severity
77. Following the extraction of the 12,
11, 21, 22, 23 teeth, a 50-year-old pati-
Кrok 2 Stomatology (англомовнi студенти) 2014 рiк 13

ent will receive an immediate-insertion are enlarged, dense, mobile lymph


denture. What denture should be used nodes. The right naso-buccal groove
in this situation? is flattened, the corner of mouth is
downturned. The mouth opens freely.
A. Laminar denture The are pronounced symptoms of the
B. Clasp denture right facial nerve paresis. What disease
C. Fixed bridge denture can be suspected?
D. Adhesive denture
E. Removable bridge denture A. Adenocarcinoma of the parotid
salivary gland
78. A 10-year-old child was undergoi- B. Chronic parotitis
ng complex dental care. During the C. Actinomycosis of the parotid-
procedures, a deep carious cavity masseteric region
communicating with the tooth cavi- D. Chronic lymphadenitis
ty was revealed in the 36 tooth. The E. Pleomorphic adenoma of the parotid
tooth was not sensitive either to probi- gland
ng, percussion, or temperature stimuli.
The patient was diagnosed with chronic 81. A 24-year-old patient was undergoi-
periodontitis. During the treatment, ng the extraction of the 25 tooth roots
the accidental perforation of the tooth for the exacerbation of chronic peri-
cavity floor occurred. What measures odontitis. During the operation the
should be taken in this case? roots were dislocated into the maxillary
sinus. What would be the surgeon’s
A. Sealing the perforation with glass optimal tactics?
ionomer
B. Potassium iodide electrophoresis A. To perform maxillary sinusotomy
C. Application of zinc-eugenol paste and remove the roots in hospital envi-
D. Sealing the perforation with calcium ronment
hydroxide B. Not to remove the roots, suture the
E. Tooth extraction extraction socket
C. To remove the roots through the
79. Dental examination of a 19-year-old extraction socket
girl revealed white spots in the cervi- D. To pack the extraction socket
cal zone of the 11, 21 teeth. Accordi- E. To perform maxillary sinusotomy
ng to the patient, the spots appeared and remove the roots in the outpatient
about two months ago and had slowly department
enlarged since that time. Which of the
following diseases is characterized by 82. A 55-year-old male patient presents
such history? to a dental clinic complaining of inabili-
ty to use complete removable dentures
A. Caries fabricated a week ago. The patient also
B. Enamel hypoplasia presents with muscle pain in the region
C. Necrosis of the temporomandibular joint, tooth
D. Fluorosis chatter during a conversation, and di-
E. Erosion of hard tissues fficult biting on food. Objectively: the
face is elongated, nasolabial and labi-
80. A 52-year-old patient complai- omental folds are flattened, the chin is
ns of pain and a swelling in the ri- retruded, the lips do not close. What is
ght parotid region. These manifestati- the cause of these disorders?
ons have been present for about 2
years. Over the last month the swelli- A. Increased occlusal vertical dimensi-
ng has grown bigger, pain has intensifi- on
ed. Objectively: the face is asymmetric B. Descending occlusion
due to a dense infiltrate in the right C. Inadequate fixation of dentures
parotid region. The poorly circumscri- D. Left displacement of the mandible
bed, tender to palpation lump infi- E. Right displacement of the mandible
ltrates the surrounding tissues. On the
right side of neck in front and behind 83. Parents of a 10-year-old boy
the sternocleidomastoid muscle there consulted an orthodontist about misali-
Кrok 2 Stomatology (англомовнi студенти) 2014 рiк 14

gnment of the 21 tooth. Objectively: anesthesia:


the 21 tooth is in a vestibular positi-
on, there is enough space for it in the A. Premedication + anesthetic without
dental arch. What additional methods a vasoconstrictor
of examination should be applied to B. An anesthetic with a vasoconstrictor
specify the treatment plan? C. Premedication + an anesthetic wi-
thout a vasoconstrictor
A. X-ray D. Premedication + an anesthetic with a
B. Pont’s index measuring vasoconstrictor
C. Korkhaus’ index measuring E. Premedication + anesthetic with a
D. Paralleling vasoconstrictor
E. Masticaciography
87. A 45-year-old female patient
84. A 38-year-old chemical plant complains of fever, appearance of rash
worker, a smoker, with a history of in the mouth cavity and on the lips.
chronic enterocolitis, was undergoing She has a 4-year history of this di-
preventive medical examination. In the sease which occurs mainly in autumn
posterior third, along the midline of the and spring. Objectively: oral mucosa
tongue dorsum there is an area with no is hyperemic, there are painful erosi-
papillae. The area is 2x1cm large, sli- ons covered with gray plaque. There
ghtly dense on palpation. What is the are also hemorrhagic crusts on the lips.
most likely diagnosis? The skin of the forearm flexor surfaces
exhibits maculo-papular rash. What is
A. Rhomboid glossitis the most likely diagnosis?
B. Desquamative glossitis
C. Chronic atrophic candidiasis A. Erythema multiforme
D. Secondary syphilis B. Dermatitis herpetiformis
E. Moeller-Hunter glossitis C. Chronic recurrent herpes
D. Bullous pemphigoid
85. A female patient presented to a E. Pemphigus vulgaris
dental prosthetics clinic complaining of
acute pain in the 26 tooth. The pain 88. A 27-year-old female patient
gets worse in the evening and at night. consulted a dentist about pain in the
Objectively: the patient has a cantilever 35 tooth. The pain is caused by cold
porcelain-fused-to-metal dental bridge stimuli and quickly abates after the sti-
supported by the 26 tooth and the mulus is removed. Objectively: in the
25 tooth cantilever. The tooth is live, cervical region of the 35 tooth there
the preparation was performed under is a dental tissue defect within the
conduction anesthesia. The prosthesis enamel. The defect has white crumbli-
is fabricated in compliance with all ng edges. Electroexcitability of the
requirements. What medical error mi- pulp was 5 mkA. What is the most li-
ght have caused this complication? kely diagnosis?
A. Continuous tooth preparation wi- A. Acute superficial caries
thout cooling, no provisional crowns B. Local enamel hypoplasia
B. Shaping the tooth as a truncated C. Acute initial caries
cone D. Enamel necrosis
C. Preparation of the radial shoulder in E. Enamel erosion
the cervical zone
D. Lack of medial support for the 89. A patient has an indication for the
bridge 12 tooth extraction for the exacerbati-
E. Cementing the bridge on the on of chronic periodontitis. It was deci-
phosphate cement ded to apply infraorbital and incisi-
ve anesthesia in this patient. What
86. A 64-year-old patient had a nerve structures are blocked by this
myocardial infarction a month ago. anesthesia?
He presents to a dental office for
the treatment of pulpitis of the 12
tooth. Select the optimal method of
Кrok 2 Stomatology (англомовнi студенти) 2014 рiк 15

A. Anterior superior alveolar branches, pain in the 47 tooth. X-ray examinati-


n. Incisivus on revealed fragments of endodontic
B. Middle superior alveolar branches, n. instruments in the canals of the medial
Nasopalatinus and distal root. In the periapical zone
C. Posterior superior alveolar branches, there are well-defined areas of bone
n. Incisivus destruction. Select the most appropri-
D. Greater palatine nerve, middle ate method of treatment:
superior alveolar branches
E. Nasopalatine nerve, anterior superi- A. Tooth extraction
or alveolar branches B. Radectomy
C. Tooth replantation
90. A 52-year-old patient consulted a D. Tooth hemisection
dentist about a tumor-like neoplasm in E. Conservative treatment
the region of the lower lip vermilion
border. Examination of the vermilion 94. A 50-year-old male patient with
border on the left revealed a roundish a defect of the hard palate needs a
tumor about 0,5 cm in diameter on a palatal plate. Objectively: the maxillary
thin pedicle. The tumor had a smooth teeth are intact. What method of fixati-
surface, was mobile, soft, and painless. on should be used?
What is the most likely diagnosis?
A. Mesio-distal clasps
A. Papilloma of the lower lip B. Supporting clasps
B. Cutaneous horn C. Attachment
C. Leukoplakia D. Bar fixation
D. Verrucous precancerous lesion of the E. Telescopic system
vermilion border
E. Abrasive Manganotti precancerous 95. A 40-year-old male patient presents
cheilitis with vicious union after a fracture of
the maxilla. Objectively: the remaining
91. A 21-year-old male patient 13 and 14 teeth are displaced in a distal
undergoes treatment for chronic direction. What method of orthopedic
catarrhal gingivitis. There is a signifi- treatment would be most adequate in
cant amount of soft dental plaque, hi- this case?
gh speed of its production. The patient
knows how to practice good oral hygi- A. Removable double-arch denture
ene. Recommend a mouthrinse for this B. Cast clasp denture
patient: C. Removable laminar denture
D. Removable denture with metal base
A. Chlorhexidine E. Removable denture with bilayer base
B. Dimethyl sulfoxide
C. Mefenaminum natrium 96. Following the unilateral resection of
D. Hydrogen peroxide the maxilla a 52-year-old patient recei-
E. Furacilin ved the immediate-insertion denture.
What is the term of its use?
92. A male patient should have the
root of the 17 tooth removed prior to A. 1-3 months
prosthetic procedures. The crown of B. 1-2 months
the tooth is decayed beneath the gum C. 7-12 months
line. What tool should be used? D. 2-3 years
E. 4-5 years
A. Bayonet forceps
B. S-shaped forceps 97. A 28-year-old patient complains
C. Straight forceps with converging of pain in the 37 tooth when eating
beaks hot food. Objectively: the 37 tooth has
D. S-shaped right forceps changed in color, there is a carious cavi-
E. Straight forceps with non-converging ty filled with softened dentin, the tooth
beaks cavity is open, probing of the root pulp
causes pain response, percussion causes
93. A patient consulted a dentist about no pain. What is the most likely di-
Кrok 2 Stomatology (англомовнi студенти) 2014 рiк 16

agnosis? A. Bayonet root forceps


B. S-shaped forceps (right)
A. Chronic gangrenous pulpitis C. Straight elevator
B. Exacerbation of chronic periodontitis D. S-shaped closed-beak forceps
C. Exacerbation of chronic pulpitis E. Crown bayonet forceps
D. Chronic fibrous pulpitis
E. Chronic fibrous periodontitis 101. A 16-year-old female patient
complains of a deformation and
98. A 16-year-old female patient restricted mouth opening since birth.
complains of gingival pain and Objectively: the face is symmetrical, di-
hemorrhage, a gingival deformity. Over sproportionate, there is a bird-like face
the last 2 years the gums periodically symptom. The mouth opens up to 1 cm.
bleed during tooth brushing. Objecti- The patient has an occlusal abnormali-
vely: in the region of the front mandi- ty, namely deep incisal overbite. What
bular teeth the gingival papillae are pathology is found in this patient?
enlarged and reach 1/2 of the tooth
crown height. The gums are swollen A. Micrognathia with congenital
and tender, bleed when touched. Radi- ankylosis
ographic alterations were not revealed. B. Micrognathia with acquired ankylosis
What is the most likely diagnosis? C. Maxillary protraction
D. Prognathism
A. Hypertrophic gingivitis E. Mandibular protraction
B. Ulcerative gingivitis
C. Generalized periodontitis 102. A 13-year-old boy complains of
D. Catarrhal gingivitis pain in the region of the extracted 46
E. Epulis tooth, irradiating to the ear and temple;
halitosis. The tooth was extracted
99. A patient presented to a dental cli- 3 days ago. Objectively: submandi-
nic for complex dental care. Objecti- bular lymph nodes are enlarged,
vely: the 37 tooth exhibits a deep cari- tender on palpation. Mucosa around
ous cavity communicating with the the extracted tooth is hyperemic,
tooth cavity. There is no response to edematous. The socket of the 46 tooth
stimuli. Radiography reveals wideni- is filled with a gray clot. What is the
ng and deformation of the periodontal most likely diagnosis?
ligament space in the apical region.
What is the most likely diagnosis? A. Alveolitis of the extracted 46 tooth
B. Acute odontogenous mandibular
A. Chronic fibrous periodontitis osteomyelitis starting from the 46 tooth
B. Chronic granulating periodontitis C. Acute odontogenous lymphadenitis
C. Chronic granulomatous periodontitis of the right submandibular region
D. Chronic fibrous pulpitis starting from the 46 tooth
E. Chronic gangrenous pulpitis D. Acute odontogenous mandibular
periostitis starting from the 46 tooth
100. A 49-year-old patient consulted a E. Neuralgia of the III branch of the
dental surgeon about complex dental trigeminal nerve
care. He has an indication for the
extraction of the 16 tooth. History: the 103. A 47-year-old male patient
tooth hasn’t been treated before, it has consulted a dental surgeon about a
decayed within the last 4 years. Objecti- slowly growing tumor on his lower
vely: the 16 tooth crown is decayed lip which appeared six months ago.
by over 2/3, the mucosa exhibits no Objectively: on the lower lip there is
pathological changes. What tool is a single horny conical projection above
required for the tooth extraction? the surface of the skin up to 1,5 cm
wide. The neoplasm adheres to the
skin. Palpation reveals dense kerati-
nous masses. What is the provisional
diagnosis?
Кrok 2 Stomatology (англомовнi студенти) 2014 рiк 17

A. Cutaneous horn 107. A 3-year-old girl has been di-


B. Verrucous precancerous lesion of lip agnosed with acute odontogenic
C. Circumscribed hyperkeratosis mandibular periostitis beginning from
D. Papilloma of lip the 74 tooth. It is required to perform
E. Abrasive precancerous Manganotti periostomy and extract the 74 tooth.
cheilitis The child is excited. Select the optimal
type of anesthesia for the surgery:
104. A 45-year-old patient has a dense
semicircle neoplasm 1 cm in diameter A. Phlebonarcosis
on his lower lip. The tumor projects B. Inferior alveolar nerve block
above the vermilion border by 0,5 cm. C. Mask anesthesia
The surface is grayish- pink with a D. Intubation anesthesia
small number of closely arranged whi- E. Central anesthesia
tish scales. The tumor is not tender on
palpation. What is the most likely di- 108. A 7-year-old child has been di-
agnosis? agnosed with chronic periodontitis of
the 64 and 65 teeth. The child has a
A. Verrucous precancerous lesion of the history of hemophilia A. What is the
vermilion border medical tactics of choice?
B. Verucca vulgaris
C. Papilloma A. Extraction of the affected teeth in
D. Pyogenic granuloma the hematology department after taking
E. Manganotti cheilitis the appropriate preparatory measures
B. Extraction of the affected teeth
105. Parents of an 8-year-old child is possible both on an in-patient and
complain about the presence of chalky out-patient basis
spots on the child’s front maxillary C. Extraction of the affected teeth on an
teeth which have recenly erupted. out-patient basis with further packing
Objectively: the vestibular surfaces of of alveolar sockets with a hemostatic
the 11 and 12 teeth exhibit white spots sponge
in the cervical zone. Enamel at these si- D. Extraction of the affected teeth in
tes is not transparent. What additional the maxillofacial department
test can be used? E. Extraction of teeth is contraindicated
because of the high probability of
A. Staining with methylene blue bleeding
B. Radiography
C. Staining with iodine-containing 109. A 3-year-old child has a hard
solutions and soft palate defect. It is planned
D. Electric pulp test to fabricate a "floating"Cese obturator
E. Radiovisiography (Chasovskaya modification). What
impression material should be used?
106. An 11-year-old boy complains of
having a painful lesion in the mouth A. Thermoplastic or silicone
cavity, which makes it difficult to eat. B. Stomalgin
Such symptoms were first noticed two C. Gypsum
years ago. Examination of the mouth D. Hydrocolloid
floor reveals a tender oval erosion si- E. Eugenol-zinc
zed up to 0,8 cm, and covered with
grayish-white plaque. The erosion has 110. Examination of a child hospitali-
a hyperemic rim. Make a provisional zed for scarlet fever revealed alterati-
diagnosis: ons of the tongue mucosa which turned
bright red, glossy, and dry. At the
A. Chronic recurrent aphthous stomati- same time the fungiform papillae were
tis enlarged. The described appearance of
B. Erythema multiforme tongue is referred to as:
C. Behcet’s syndrome
D. Stevens-Johnson syndrome
E. Traumatic erosion
Кrok 2 Stomatology (англомовнi студенти) 2014 рiк 18

A. Strawberry this case?


B. Plicated
C. Rhomboid A. Stiff head-chin strap by D.O. Entin
D. Geographic B. Smooth brace
E. Black C. Standard Vasilyev splint
D. Tigerstedt’s splint with wire loops
111. A 64-year-old patient with E. Ivy ligature
edentulous jaws has a mild uni-
form atrophy of the maxilla and 114. In a 7-year-old child the right
a pronounced atrophy of lateral mandibular molars overlap the maxi-
segments of the mandible. The mucosa llary molars, there are no other occlusi-
in these regions makes longitudinal on abnormalities. Size and shape of the
folds, the alveolar crest is mobile in the lower dental arch are normal. Specify
anterior part. The condition of mandi- the appliance for the treatment of this
ble should be taken into consideration abnormality:
at the following stage of fabrication of
a complete removable denture: A. Upper-jaw appliance with a sector
expansion screw
A. Taking a differentiated functional B. Upper-jaw appliance with a middle
impression expansion screw
B. Taking an anatomical impression C. Upper-jaw appliance with a right
C. Taking a compression functional guide plane
impression D. Upper-jaw appliance with a left
D. Taking a decompression functional guide plane
impression E. Angle’s coil spring
E. Determining the centric jaw relation
115. Parents of a 5-year-old child
112. A 30-year-old female patient consulted an orthodontist about mi-
complains of limited mouth opening. spronunciation of sounds by the chi-
She has a history of an injury received ld. Objectively: the child’s face is
at the time when her jaws were closed. unremarkable. The patient has deci-
Objectively: mouth opening is limited duous dentition. There are 1-1,5 mm
to 1,5 cm, the bite is normal, mouth gaps in the frontal segment from 53
opening causes the displacement of to 63. Occlusion in the sagittal and
mandible to the left. Palpation reveals transversal planes is normal. What
pain in the left temporomandibular joi- type of Frankel’s functional regulator
nt (TMJ). X-ray picture confirms bone is used to treat the above-described
intactness. What is the most likely di- malocclusion?
agnosis?
A. Type IV
A. Acute traumatic arthritis of the left B. Type III
TMJ C. Type II
B. Fracture of the left articular process D. Type I
of the mandible E. -
C. Acute unilateral dislocation of the
TMJ 116. A 35-year-old female patient seeks
D. Contusion of the left TMJ prosthetic dentistry. Objectively: the
E. Fracture of the left zygomatic bone 18, 14, 13, 12, 11, 21, 22, 23, 24, 28 teeth
with a displacement of bone fragments need restoration. The crowns confini-
ng the defect are high and stable. To
113. A 41-year-old patient has been di- restore the integrity of the dental arch,
agnosed with a fracture of the mandi- it is planned to fabricate a cast clasp
bular alveolar process. Objectively: denture. What kind of mechanical fi-
the teeth are intact. When the jaws xator should be used?
are closed, the multipoint occlusal
contact is observed. The patient is to
be transported to a specialized hospi-
tal. What kind of first aid is indicated in
Кrok 2 Stomatology (англомовнi студенти) 2014 рiк 19

A. Bar fixation A. Acute median caries


B. Supporting-retaining clasps B. Chronic median caries
C. Retaining clasps C. Acute deep caries
D. Telescopic fixation D. Chronic fibrous pulpitis
E. Dentoalveolar clasps E. Chronic deep caries
117. A 48-year-old patient has 121. A 45-year-old male patient
undergone unilateral resection of complains of bleeding gums, tooth
the maxilla. He needs a resection mobility, hypersensitivity in the cervical
prosthesis. Objectively: the 21, 22, 23, zone. Objective examination revealed
24, 25, 26 teeth are stable, hard palate presence of all teeth, 1-2 class mobi-
is broad, flat. What kind of attachment lity, pathological gingival pockets wi-
of clasps to the base will reduce the th serous exudate. Radiograph shows
instability of the resection prosthesis? widening of the periodontal ligament
space, alveolar bone resorption by 1/3
A. Articular - 1/2 of the root length. What kind of
B. Stable tooth stabilization is indicated?
C. Elastic
D. Fixed A. Stabilization of entire dental arch
E. Does not matter B. Front
C. Front and sagittal
118. Residents of a remote settlement D. Sagittal
need prosthetic care. What health care E. Parasagittal
facility should provide it?
122. During mandibular anesthetizati-
A. Regional dental clinic on with 4% solution of Ultracain-
B. City dental clinic forte, a 13-year-old girl complained of
C. Occupational health facility sudden weakness, laboured respiration,
D. Rural district hospital nausea, fear. Objectively: the patient
E. Rural outpatient clinic has pale skin covered with cold sweat,
hyperemic spots on the face and neck.
119. A 24-year-old hemophiliac has Respiration is frequent, the pulse is
been diagnosed with exacerbation of weak, blood pressure is 60/40 mm Hg,
chronic pulpitis of the 11 tooth. Select hands are cold. What is the most likely
the best method of tooth treatment in diagnosis?
this case:
A. Anaphylactic shock
A. Devital extirpation B. Toxic shock
B. Biological treatment C. Syncope
C. Vital amputation D. Traumatic shock
D. Devital amputation E. Collapse
E. Vital extirpation
123. A 65-year-old female patient
120. A 25-year-old female patient complains of complete edentulousness.
consulted a dentist about acute pain Examination of the oral cavity revealed
in the maxilla on the left. The pain that alveolar process of the edentulous
occurs while eating. Objectively: on the mandible was markedly atrophied
distal approximate surface of the 26 in the frontal region, while it was
tooth there is a cavity filled with light expressed in the distal region. Specify
softened dentin. Probing causes a slight the class of atrophy according to Keller
pain along the dentin-enamel juncti- classification:
on, percussion causes no pain response.
Cold water causes quickly abating pain. A. Class IV
What is the most likely diagnosis? B. Class II
C. Class I
D. Class III
E. Class I and III
124. A 28-year-old patient complains
Кrok 2 Stomatology (англомовнi студенти) 2014 рiк 20

of tender gums, gingival hemorrhages surface of the 15 tooth there is a cavi-


and pain, especially while eating, ty filled with softened dentin. Probing
for a week. Objectively: the gums causes pain reaction. Electroexcitabili-
are swollen and markedly hyperemic, ty of the pulp is 35 µA. What is the most
especially in the region of the 43, 42, 41, likely diagnosis?
31, 32, 33 teeth, soft plaque and tartar
are present, gingival sulci are up to 2 A. Chronic fibrous pulpitis
mm deep. What is the most likely di- B. Acute deep caries
agnosis? C. Chronic deep caries
D. Hyperemia of the pulp
A. Acute deep gingivitis E. Exacerbation of chronic pulpitis
B. Chronic catarrhal gingivitis
C. Hypertrophic gingivitis 128. A 25-year-old patient undergoes
D. Initial stage of chronic generalized treatment at the clinic of maxi-
periodontitis llofacial surgery for the mandibular
E. Exacerbation of initial generalized ameloblastoma localized in the angle.
periodontitis The patient has been scheduled for a
partial resection of the mandible with a
125. A 24-year-old male patient single-stage bone autoplasty. It is most
complains of acute gingival pain, the expedient to take a bone graft from:
inability to eat, bad breath, fever up
to 38o C , general weakness. Objecti- A. Iliac bone ridge
vely: the patient is pale, lymph nodes B. Collarbone
are enlarged. Gingival papillae are C. Foot bones
swollen and hyperemic, there are indi- D. Femur
vidual ulcers covered by necrotic coati- E. VI rib
ng. What additional study is required
in the first place in order to clarify the 129. A 36-year-old female patient
diagnosis? complains of headache, inability to
close the jaws. She has a history of
A. Complete blood count an injury followed by loss of consci-
B. Urinalysis ousness, nausea. Objectively: the mi-
C. Panoramic radiography dface tissues are edematous, there is
D. Allergologic tests bilateral eyelid hemorrage, open bite.
E. Kavetsky test with trypan blue Palpation of the midface bones causes
acute pain. In the back of nose there
126. A 35-year-old patient complai- is marked symptom of step deformity,
ns of gingival hemorrhage during nasal hemorrhage is present. Palpation
tooth brushing. Objectively: gums of of the oral cavity reveals mobility of the
both jaws are hyperemic and cyanotic, maxilla. Make a diagnosis:
supragingival and subgingival tartar is
present. The periodontal pockets are A. Le Fort II maxillary fracture (mi-
4-6 mm deep. What is the most likely ddle), concussion
provisional diagnosis? B. Le Fort III maxillary fracture (top)
C. Le Fort I maxillary fracture (bottom
A. II grade generalized periodontitis, )
chronic course D. Le Fort II maxillary fracture (mi-
B. Catarrhal gingivitis
C. Hypertrophic gingivitis ddle)
D. Exacerbation of I grade generalized E. Maxillary Guerin fracture
periodontitis 130. Objective examination of a 65-
E. II grade parodontosis year-old patient with the completely
127. A 27-year-old male patient edentulous mandible revealed a
complains of aching long-lasting pain marked uniform atrophy of the
in the 15 tooth during having meals, alveolar bone; bony prominences on
especially cold food. Sometimes the the lingual surface in the region
pain occurs when the temperature where premolars had previously been.
changes. Objectively: on the distal Mucosa was unevenly pliable; alveolar
Кrok 2 Stomatology (англомовнi студенти) 2014 рiк 21

crest was mobile in the frontal region. A. Frontonasal, zygomatic,


The clinical condition of the mandible pterygopalatine, palatine
should be taken into consideration at B. Frontonasal, zygomatic,
the following stage of denture fabri- pterygopalatine
cation: C. Zygomatic, pterygoid, palatine
D. Frontonasal, pterygoid, palatine
A. Taking a differentiated impression E. Frontonasal, zygomatic, palatine
B. Taking an anatomical impression
C. Taking a positive pressure impression 134. A 12-year-old girl complains of
D. Taking a decompression impression white spots on the vestibular surfaces
E. Measuring the centric relation of jaws of teeth and mouth soreness that
occurs during eating sour foods. It is
131. Objective examination of a 67- known from the history that the spots
year-old patient with the edentulous appeared about 3 months ago. Objecti-
maxilla revealed minor uniform vely: the vestibular surfaces of the front
atrophy of the alveolar processes. maxillary teeth exhibit chalky spots
Maxillary prominences were well that are stainable with 2% methylene
preserved, the frenulum and bucco- blue. What is the most likely diagnosis?
alveolar folds were attached at the
base of the alveolar process, the palate A. Acute initial caries
was deep, the torus palatinus was B. Acute superficial caries
expressed insignificantly. These clini- C. Acute median caries
cal presentations correspond with the D. Chronic median caries
following class of atrophy according to E. Chronic superficial caries
Schroeder classification:
135. A 25-year-old male patient has
A. 1 been diagnosed with a linear nondi-
B. 2 splaced fracture of the mandible
C. 3 between the 32 and 33 tooth. What
D. 4 splint should be used for the fixation
E. 5 of bone fragments?
132. A 2-year-old boy has a brown A. Smooth brace
spot with a rough surface covered with B. Splint with spacer bar
coarse hair on his left cheek. Accordi- C. Brace with an oblique plane
ng to parents, the neoplasm has been D. Vasilyev splint
present since birth and exhibits a slight E. Wire anchor splint
enlargement as the child grows. What
is the most likely diagnosis? 136. A 46-year-old patient complai-
ns of a periodic swelling in the right
A. Pigmented nevus submandibular region when eating, dry
B. Vascular nevus mouth. Bimanual palpation of the mi-
C. Kaposi’s sarcoma ddle portion of the hyoid region on
D. Facial neurofibromatosis the right revealed a moderately pai-
E. Melanoma of cheek nful dense lump. From the excretory
duct of the submandibular gland some
133. A 53-year-old male patient has muco-purulent exudate is expressed.
a history of generalized periodontitis. The patient should be referred to the
It is planned to fabricate fixed splints following roentgenologic examination:
for both jaws intended to stabilize the
teeth along the entire dental arch. Whi- A. Radiography of the mouth floor
ch of the maxillary pillars will have the B. Panoramic radiography of the jaws
functional significance for the distri- C. Radiography of the mandible in the
bution of masticating pressure? lateral projection
D. Survey radiography of the facial
bones
E. Spot-film radiography of the alveolar
process
Кrok 2 Stomatology (англомовнi студенти) 2014 рiк 22

137. An 8-year-old boy complains of a A. Extraction


defect in the 11 tooth crown. Objecti- B. Endodontic treatment
vely: 1/3 of the 11 tooth crown is broken C. Endodontic treatment and drug
off, the pulp chamber is closed. Radi- therapy
ograph shows incomplete root formati- D. Drug therapy
on. Select the optimal way of prostheti- E. Dissection along the mucogingival
cs for the 11 tooth: junction, drug thrapy
A. Thin-walled metal crown 141. 6 months ago a 65-year-old pati-
B. Metal-ceramic crown ent received a permanent intracoronal
C. Plastic crown splint for the front maxillary teeth. The
D. Post and core splint was fabricated with the use of
E. Combined crown fiberglass tape. Radiograph shows that
the root of the 12 tooth is localized only
138. A 9-year-old boy requires complex in the soft tissues. The splint is in a good
dental care. Objective examination condition. What is the optimal tactics of
revealed chalky spots with lustrous this patient management?
surface on the vestibular surfaces of
the 22, 21, 11, 12 teeth, as well as on A. Amputate and extract the root of the
the tubercles of the 26, 26, 46 teeth. 12 tooth
The affected teeth are not sensitive to B. Extract the 12 tooth and fabricate a
thermal and chemical stimuli. The chi- bridge
ld has a previous history of rickets. He C. Start conservative treatment
lives in a locality where the fluoride D. Replace the 12 tooth in the splint by
concentration in drinking water is 0,5 an artificial plastic tooth
mg/l. What is the most likely diagnosis? E. Extract the 12 tooth and restore the
defect with a porcelain-fused-to-metal
A. System enamel hypoplasia crown
B. Fluorosis
C. Stainton-Capdepont syndrome 142. A 55-year-old male patient di-
D. Local enamel hypoplasia agnosed with chronic median caries of
E. Amelogenesis imperfecta the 33 tooth presented to a dental cli-
nic. It is known from the history that
139. A 6-year-old child with contused the patient has a cardiac pacemaker
forehead laceration had been taken What would be the optimal material
to the emergency room at a chi- for a permanent filling?
ldren’s hospital. The child was exami-
ned by the maxillofacial surgeon who A. Chemical-cure composite
performed primary surgical debri- B. Silicate cement
dement and closed the wound with C. Silicophosphate cement
immediate primary sutures. The child D. Light-curing composite
was recommended to undergo further E. Compomer
treatment on an outpatient basis at
a district polyclinic. Specify the first- 143. A 14-year-old patient complains of
priority measure in this case: moderate non-irradiating pain in the
region of the 37 tooth. Objectively: the
A. Consultation by a neurosurgeon 37 tooth exhibits a deep carious cavity
B. Surgeon follow-up communicating with the pulp chamber.
C. Complex dental care Probing and thermal stimulation cause
D. Antimicrobial therapy no pain response, vertical percussi-
E. Bed rest on is positive. Radiograph shows no
pathological alterations. What is the
140. An 8-year-old child has been clini- most likely diagnosis?
cally diagnosed with exacerbation of
chronic periodontitis of the 84 tooth.
The crown is decayed by half. What is
the dentist’s optimal tactics?
Кrok 2 Stomatology (англомовнi студенти) 2014 рiк 23

A. Acute serous periodontitis A. Exacerbation of chronic pulpitis


B. Acute serous periostitis B. Acute periodontitis
C. Acute suppurative pulpitis C. Exacerbation of chronic periodontitis
D. Acute suppurative periodontitis D. Acute serous pulpitis
E. Exacerbation of chronic periodontitis E. Acute purulent pulpitis
144. A 32-year-old patient lost the 41 147. A 4-year-old child complains of
and 31 teeth as a result of a sports acute spontaneous pain in a mandi-
injury. Reimplantation is impossible bular tooth on the right. The pain is
because the crown and root surfaces aggravated by cold stimuli. Objecti-
of these teeth are cracked and chipped. vely: the 84 tooth exhibits a deep cari-
Clinical and radiological examination ous cavity that does not communicate
proved the possibility of immediate with the tooth cavity. Probing causes
implantation. What material should be pain response all over the cavity floor,
preferred for the implants? percussion causes pain. The mucosa in
the region of the 84 tooth is hyperemic,
A. Titanium edematous. Regional lymph nodes are
B. Carbon enlarged. What is the most likely di-
C. Porcelain agnosis?
D. Platinum
E. Sapphire A. Acute pulpitis complicated by peri-
odontitis
145. Radiography of the 46 tooth in a B. Acute serous pulpitis
7-year-old child revealed parallel root C. Acute serous periodontitis
walls with a gradual decrease of their D. Exacerbation of chronic periodonti-
thickness resulting in pointed ends. tis
There is a root canal narrowing in the E. Acute suppurative pulpitis
region that is adjacent to the tooth cavi-
ty, and a widening at the nascent apical 148. A patient presented to a dental
aperture. Periodontal ligament space clinic for the purpose of prostheti-
has a uniform width over the entire cs. Objectively: completely edentulous
root length. In the periapical region it mandible. There is marked and uni-
merges with the growth zone. Specify form atrophy of the alveolar portion.
the stage of the root development: Frenula and folds are characterized
by high attachment. Specify the type
A. Open apex of mandibular atrophy according to
B. Incomplete root growth Keller classification:
C. Complete root and periodont
development A. Class II
D. Incomplete periodont development B. Class I
E. - C. Class III
D. Class IV
146. A 5-year-old child complains of E. Class V
spontaneous pain in a maxillary tooth
on the right. The pain gets worse at 149. A 43-year-old patient consulted
night and during eating cold food. a dentist about numbness of the left
Objectively: the 65 tooth has a deep half of his tongue, which developed 10
carious cavity communicating with days ago after the atypical extraction of
the tooth cavity. Probing causes pain the 38 tooth. What nerve was damaged
response, the tooth is not sensitive to during the surgery?
percussion. Cold water causes long-
lasting pain. Make a provisional di- A. Lingual
agnosis: B. Sublingual
C. Inferior alveolar
D. Facial
E. Glossopharyngeal
150. Soon after anesthetization a
55-year-old patient reported severe
Кrok 2 Stomatology (англомовнi студенти) 2014 рiк 24

weakness, retrosternal pain that irradi- hyperkeratinized, infiltrated. Select the


ated to his left arm and scapular region, optimal treatment tactics:
palpitations. Objectively: the patient is
conscious, adynamic, pale, the forehead A. Fissure excision
is covered with cold sweat. AP- 90/60 B. Application of keratoplastic agents
mm Hg, cardiac sounds are muffled, the C. Irradiation with helium-neon laser
pulse is thready and arrhythmic. What D. Application of corticosteroid oi-
condition developed in the patient? ntment
E. Application of antibiotic ointment
A. Myocardial infarction
B. Cardiogenic form of anaphylactic 154. A 48-year-old female patient
shock complains of hard tissue defects found
C. Angina attack on the maxillary incisors. The defects
D. Collapse appeared about a year ago and have
E. - been enlarging since that time. Objecti-
vely: on the convex portion of the vesti-
151. During tooth extraction a 55- bular surface of the maxillary incisors
year-old female patient reported dull there are oval varisized enamel defects.
retrosternal pain, feeling of chest The defect surface is smooth, lustrous,
compression. The dental surgeon who solid. In order to prevent progression
was performing the tooth extraction of this process, the patient should be
made a diagnosis of angina. What drugs advised to limit consumption of:
should be given the patient for the reli-
ef of this condition? A. Sour food
B. Marine products
A. Nitroglycerin, validol C. Meat products
B. Analgin D. Sweet food
C. Baralgin E. Tea
D. Diphenhydramine
E. Ketanov 155. During local anesthetization with
2 ml of 10% lidocaine a 9-year-old girl
152. A 1-month-old child has been suddenly screamed, lost consciousness
brought to a maxillofacial surgeon for and developed generalized convulsi-
examination. Objectively: there is a fi- ons. The skin is pale and cyanotic.
ssure running transversely from the ri- The pulse is not palpable because of
ght corner of mouth through the full convulsions. What is the provisional di-
thickness of cheek up to the anterior agnosis?
edge of the masseter muscle. What is
the patient’s diagnosis? A. Lidocaine intoxication
B. Angioedema
A. Right transverse facial cleft C. Anaphylactic shock
B. Right oblique facial cleft D. Bronchial obstruction
C. Cleft nasal deformity E. Febrile seizures
D. Right upper lip cleft
E. Mandibular cleft 156. A 56-year-old patient was
undergoing treatment for pulpitis of
153. A 38-year-old female complai- the 47 tooth. For the purpose of
ns of experiencing pain in her lower the pulp devitalization arsenic paste
lip for half a year, difficult eating and was used. The patient failed to keep
talking. The patient sought medical an appointment with the dentist for
help at place of residence, but the further treatment. As a result, toxic
administered treatment was ineffective. arsenic periodontitis developed. Which
Examination of the lower lip revealed of the physiotherapy methods should
a deep fissure in the centre which be applied to address the complicati-
affected the vermilion border and on?
partly the mucous membrane of lips.
The fissure was covered with a brown
crust, after its removal the fissure
started bleeding. The fissure edges are
Кrok 2 Stomatology (англомовнi студенти) 2014 рiк 25

A. Potassium iodide electrophoresis A. Fracture of the maxillary alveolar


B. UHF process
C. D‘arsonvalisation B. Le Fort II fracture of the maxilla
D. Fluctuorization C. Fracture of the zygomatic bone
E. Galvanization D. Fracture of the zygomatic arch
E. Le Fort III fracture of the maxilla
157. Following an uncomplicated
extraction of the 37 tooth, a 60-year-old 160. A 34-year-old patient complains of
patient has profuse phatnorrhagia. The intense, tearing, throbbing pain in the
patient has a 6-year history of essenti- 17 tooth, which appeared for the first
al hypertension. AP- 180/110 mm Hg. time and persists for 4 days. Percussion
What kind of emergency care should causes acute pain in any direction, the
be rendered? tooth is mobile, the mucosa around the
17 tooth is hyperemic and edematous;
A. Give an injection of antihypertensive palpation of the mucogingival junction
drugs and pack the tooth socket tightly causes acute pain. What is the most li-
B. Pack the tooth socket with a kely diagnosis?
hemostatic sponge
C. Pack the tooth socket tightly with an A. Acute suppurative periodontitis
iodoform turunda B. Acute serous periodontitis
D. Suture the tooth socket C. Exacerbation of chronic periodontitis
E. Give an injection of hemostatic drugs D. Localized periodontitis
E. Acute suppurative pulpitis
158. A child is 11 years old. Exami-
nation of the oral cavity revealed that 161. A 45-year-old patient complains of
the front maxillary teeth completely acute, spontaneous, nocturnal pain in a
overlap the mandibular ones. Lateral maxillary tooth on the right. The pain
teeth in the sagittal and transversal arose three days ago, has a pulsating
planes have a normal contact. The chi- nature, irradiates to the zygomatic regi-
ld has been diagnosed with a deep on and virtually doesn’t cease. Objecti-
overbite. This abnormality is typically vely: there is a deep carious cavity in
accompanied by the dysfunction of: the 15 tooth. Dentin is of dirty-gray
color, softened. Probing causes pain
A. Chewing, biting off response across the entire floor, the
B. Breathing, swallowing tooth is sensitive to percussion. Cold
C. Swallowing, biting off stimulus reduces the pain intensity.
D. Biting off, breathing Electroexcitability of the pulp is 55 µA.
E. Speech, chewing What is the most likely diagnosis?
159. A 25-year-old male patient A. Acute suppurative pulpitis
presented to the department of maxi- B. Acute pulpitis limited
llofacial surgery with complaints of C. Acute diffuse pulpitis
pain during eating, which arose three D. Acute suppurative periodontitis
days ago after an injury. Objectively: E. Acute serous periodontitis
the face is asymmetric due to the
swelling of the buccal and infraorbi- 162. A 13-year-old girl complains of
tal regions on the left. Mouth opening experiencing pain, gum bleeding and
is slightly restricted because of pain. bad breath for about six months.
Maxillary alveolar process and left Objectively: in the region of the front
maxillary teeth are mobile. There is a mandibular teeth the gingival papillae
hemorrhage along the left mucogingi- and gingival margin are hypertrophi-
val junction in the region of the said ed, edematous, cyanotic. There is a soft
teeth; the site is tender to palpation. plaque in the cervical region. Hygiene
What diagnosis can be assumed? index -3. The doctor administered a
local anti-inflammatory therapy. Speci-
fy the further tactics of local treatment:
Кrok 2 Stomatology (англомовнi студенти) 2014 рiк 26

A. Sclerotherapy 166. A 60-year-old patient undergoes


B. Gingival curettage treatment of the 11 and 21 tooth for
C. Gingivotomy chronic deep caries (Black’s class III).
D. Physiotherapy He has a history of cataract surgery.
E. Vestibuloplasty What is the optimal material for cavity
filling in this case?
163. A 6-year-old child complains of
pain in the right submandibular regi- A. Chemical cure composite
on. A day before he complained of a B. Hybrid glass ionomer
sore throat. Objectively: the child is in C. Compomer
moderately severe condition, body it D. Light cure composite
is 37, 9oC . The face is asymmetric due E. Silicate cement
to a dense tender infiltrate in the right
submandibular region. The infiltrate is 167. Clinical examination of a 10-year-
slightly mobile, the overlying skin is old girl’s oral cavity revealed an 11
hyperemic. There is fluctuation in the mm gap in a sagittal plane, the contact
center. The teeth are intact. What is the of the lateral maxillary teeth with the
most likely diagnosis? front mandibular teeth, mesiobuccal
cusps of the 16 and 26 tooth located
A. Acute suppurative non-odontogenic on the cusps of the 35 and 45 tooth.
submandibular lymphadenitis What additional examination method
B. Acute serous non-odontogenic will allow to make a definitive diagnosis
submandibular lymphadenitis and specify clinical form of malocclusi-
C. Phlegmonous adenitis on?
D. Acute suppurative odontogenic
submandibular lymphadenitis A. Profile teleroentgenography
E. Sialadenitis B. Orthopantomography
C. Anthropometric measuring of jaw
164. A 45-year-old male patient worki- models
ng as an actor consulted a dentist D. Determining morphological facial
about mobility of his front mandi- index
bular teeth, gingival hemorrhages duri- E. Clinical functional tests
ng tooth brushing. Objectively: gingi-
val hyperemia and hemorrhage, peri- 168. A 70-year-old completely
odontal pockets in the front porti- edentulous patient undergoes treatment
on of the mandible are up to 5 mm with complete removable dentures.
deep, tooth mobility of the I-II class Artificial teeth are placed upon a
is present. What kind of splinting the spherical surface. Specify the average
mandibular front teeth will be optimal radius of the spherical surface that
for this patient? would ensure close teeth contact duri-
ng the mandible motions:
A. Fiber-reinforced adhesive splint
B. Cap splint A. 9 cm
C. Ligature wire B. 5 cm
D. Full crown splint C. 7 cm
E. Cast pin splint D. 12 cm
E. 18 cm
165. A 44-year-old female patient wi-
th generalized I grade periodontitis is 169. A 47-year-old male patient was
scheduled for the selective grinding of undergoing complex dental treatment.
teeth. What examination is required After anesthetization the patient
prior to this procedure? reported dizziness, ringing in the ears,
blackout, general weakness. Ps- 96
A. Occlusiography bpm, AP- 90/60 mm Hg. The patient
B. Gnathodynamometry lost consciousness for 30 seconds. What
C. Radiography condition developed in the patient?
D. Rheoparodontography
E. Electroodontometry
Кrok 2 Stomatology (англомовнi студенти) 2014 рiк 27

A. Syncope glossalgia. He has been using a


B. Collapse complete removable denture for the
C. Anaphylactic shock mandible for 6 months. The patient has
D. Epileptic seizure no history of rheumatosis. Objectively:
E. Traumatic shock the lower third of the face is shortened,
mental fold is strongly pronounced,
170. A patient consulted a dentist about mouth corners are downturned, there
fabrication of a restorative crown for are angular fissures and cracks. Palpati-
the 36 tooth. During examination the on reveals crepitation during TMJ
dentist revealed a roundish, hard, pai- moving. What is the most likely di-
nless, lustrous ulcer on the left lateral agnosis?
surface of tongue. What is the most li-
kely diagnosis? A. Costen’s syndrome
B. Temporomandibular arthritis
A. Syphilitic ulcer C. Temporomandibular arthrosis
B. Decubital ulcer D. Temporomandibular dislocation
C. Tuberculous ulcer E. Temporomandibular osteoarthritis
D. Vincent’s necrotizing ulcerative
stomatitis 174. A 76-year-old patient has a roundi-
E. Lichen planus sh ulcer on the skin in the region of the
right nasolabial sulcus. The ulcer floor
171. A 65-year-old male patient is dense, rough, covered with yellowish-
complains of a long-lasting pain in a gray crusts, looks like a funnel. The
mandibular tooth on the right. The ulcer edges are dense, crater-like. The
pain increases on biting. The patient ulcer grows in size and depth. What is
has a history of this tooth treatment for the provisional diagnosis?
caries. Objectively: the cavity on the
masticatory surface of the 36 tooth is A. Basalioma
filled, percussion causes pain responce. B. Tuberculous ulcer
Along the mucogingival junction in the C. Actinomycotic abscess
projection of the 36 tooth roots there is D. Syphilitic ulcer
a fistula with purulent discharge. What E. Squamous cell skin cancer
is the most likely diagnosis?
175. A 59-year-old male patient
A. Chronic granulating periodontitis presented to a prosthetic dentistry
B. Radicular cyst of the mandible clinic for the purpose of prosthetics.
C. Exacerbation of chronic periodontitis Immediately during the manipulati-
D. Chronic fibrous periodontitis ons the patient developed the followi-
E. Chronic granulomatous periodontitis ng symptoms: weakness, prostration,
skin pallor and cyanosis, clammy sweat,
172. Prior to installation of complete a decrease in systolic blood pressure
removable dentures, it is necessary down to 80 mm Hg. What is the most
to verify the occlusal relation with likely diagnosis?
different movements of the mandi-
ble. What muscle is responsible for A. Collapse
transversal movements of the mandi- B. Syncope
ble? C. Shock
D. Kussmaul’s coma
A. Outer (lateral) pterygopalatine E. Myocardial infarction
B. Inner (medial) pterygopalatine
C. Temporal 176. A 40-year-old male patient
D. Masticatory complains of a long-existing fissure in
E. Digastric the central portion of the vermilion
border of his lower lip. The patient
173. A 65-year-old male patient has been a smoker since the age of
complains of crepitation and clicki- 16. Along the midline of the vermi-
ng in both temporomandibular joi- lion border of the lower lip there is
nts; pain when moving his mandible a deep fissure up to 1,0 cm long wi-
to the right; ear noise; dry mouth; th the rolled edges. The fissure heals
Кrok 2 Stomatology (англомовнi студенти) 2014 рiк 28

spontaneously, but recurs soon after A. Mandibular torus


epithelialization. The patient was di- B. Maxillary tuber
agnosed with chronic lower lip fissure. C. Retromolar fossa
Conservative treatment proved to be D. Mandibular temporal crest
ineffective. Select the most efficient E. Retromandibular fossa
method of treatment:
180. A 7-year-old child with a marked
A. Excision within apparently healthy bilateral swelling of the parotid regions,
tissues tenderness on palpation of the tragus,
B. Cryolysis condylar and mastoid process, dryness
C. Diathermocoagulation of the oral mucosa was diagnosed with
D. Excision within 0,5 cm area around mumps of a mild severity grade. Select
the formation a medical tactic with respect to this
E. Short-distance irradiation severity grade:
177. A patient is scheduled for the A. Outpatient treatment with isolation
dissection of an abscessed upper lip of the child
furuncle. What kind of anesthesia is B. Outpatient treatment without isolati-
indicated for this intervention? on of the child
C. Hospitalization in children’s
A. Bilateral infraorbital department of oral and maxillofaci-
B. Bilateral infraorbital and incisive al Surgery
C. Bilateral infraorbital and palatine D. The child is allowed to attend chi-
nerve block ldren’s institutions
D. Bilateral tuberal and palatine nerve E. Hospitalization in the infectious
block diseases hospital
E. Bilateral palatine nerve block
181. Examination of a 12-year-old
178. An accident victim with an injury child revealed a significant increase
of face and the temporal region was in the size of the jaws, presence
diagnosed with a fracture of the of gaps between the teeth, tongue
zygomatic arch. Which processes of the enlargement, disproportionate skeletal
skull bones are damaged? growth. This pathology has been
caused by the dysfunction of the
A. Temporal process of the zygomatic following gland:
bone and the zygomatic process of the
temporal bone A. Pituitary
B. Zygomatic process of the frontal B. Thyroid
bone and the zygomatic process of the C. Parathyroid
temporal bone D. Pancreas
C. Temporal process of the zygomatic E. Sex
bone and the zygomatic process of the
frontal bone 182. An 11-year-old boy had been di-
D. Zygomatic process of the maxilla and agnosed with a 6 mm wide diastema of
the zygomatic process of the temporal type 2 (by Khoroshilkina F.I. classifi-
bone cation). To normalize the incisor posi-
E. Zygomatic process of the maxilla tion, the Korkhaus appliance was used.
and the zygomatic process of the frontal What kind of movement is facilitated
bone by this appliance?
179. With the purpose of the 46 tooth A. Bodily movement
extraction, a 28-year-old male pati- B. Rotation
ent was given Weisbrem’s anesthesia. C. Intrusion
Specify the site of anesthetic injection D. Extrusion
with this anesthesia: E. Torque
183. A 24-year-old male patient
complains of spontaneous pain in the
24 tooth, which arose suddenly and
Кrok 2 Stomatology (англомовнi студенти) 2014 рiк 29

persists for about 15 minutes. Objecti- A. Exacerbation of chronic periodonti-


vely: the distal surface of the 24 tooth tis
exhibits a deep carious cavity with B. Acute serous periodontitis
overhanging walls. The cavity is fi- C. Acute suppurative periodontitis
lled with light softened dentin and D. Chronic granulating periodontitis
communicates with the tooth cavity. E. Acute serous periodontitis, intoxi-
The cold stimulus causes acute, slowly cation stage
abating pain. Percussion causes no pain
response. Select the best method of 186. A 5-month-old infant has acute
treatment: hematogenous osteomyelitis of the
maxilla, there are fistulae with purulent
A. Vital extirpation exudate at the medial angle of the ri-
B. Vital amputation ght eye. Specify the probable long-term
C. Biological method effects of the disease:
D. Devital amputation
E. Devital extirpation A. Bony ankylosis of the right TMJ
B. Chronic sinusitis
184. A 9-year-old girl has had gingi- C. Chronic rhinitis
val hemorrhages, cracks in the mouth D. Macrognathia
corners for a month. She has a hi- E. Macrogenia
story of frequent nosebleeds, rapid fati-
gability. Objectively: the facial skin 187. A 16-year-old boy has been di-
and oral mucosa are pale. In the agnosed with a fracture of the left
mouth corners there are cracks reachi- zygomatic bone with a fragment di-
ng the vermilion border. Gingiva in splacement. Select the most efficient
the region of the 11 and 12 teeth method of treatment:
is hyperemic and edematous, bleeds
when touched. DMF/dmf = 10. Back of A. Open reposition and osteosynthesis
the tongue is bright red, smooth, glossy. with metal plates
Submandibular lymph nodes are sli- B. Osteosynthesis with metal wires
ghtly enlarged, mobile, painless. What C. Any of these methods is applied in
specialist is it necessary to consult with children and adolescents
in the first place? D. Closed reposition
E. Bone suture
A. Hematologist
B. Neuropathist 188. A 5-year-old girl complains of pain
C. Endocrinologist in the mouth corners occurring when
D. Infectious disease specialist opening the mouth. She has a history of
E. Gastroenterologist acute bronchitis and prolonged antibi-
otic therapy. Objectively: the vermilion
185. A 25-year-old patient complains of border of lips is dry, the mouth corners
pain when biting on the 15 tooth. The are downturned. In the mouth corners
pain arose two days ago, has a constant there are cracks covered with a whi-
aching nature and increased signifi- te plaque and surrounded by a slightly
cantly over the last day. Objectively: the hyperemic elevation, tender on palpati-
crown of the 15 tooth is gray, the medial on and mouth opening. Oral mucosa
contact surface exhibits a deep carious exhibits no pathological alterations,
cavity communicating with the tooth dmf - 2, hygiene index - 2,0. What is
cavity. Percussion causes acute pain, the the most likely diagnosis?
gingival mucosa in the projection of the
25 tooth root apex is hyperemic. The A. Mycotic cheilitis
regional lymph node is tender. Radi- B. Streptococcal cheilitis
ograph shows an ill-defined zone of C. Meteorological cheilitis
periapical bone destruction. What is D. Glandular cheilitis
the most likely diagnosis? E. Exfoliative cheilitis
189. A female patient complains of a
moderate gum pain, tooth sensitivity
to the thermal stimuli. Objectively: the
Кrok 2 Stomatology (англомовнi студенти) 2014 рiк 30

gingiva is pale, dense, there is gingi- A. Eczematous cheilitis


val retraction in the region of the front B. Exfoliative cheilitis
mandibular teeth. Radiograph shows C. Glandular cheilitis
the resorption of the interalveolar D. Meteorological cheilitis
septa to 1/3 of the root length. What E. Chronic fissure of lips
is the most likely diagnosis?
193. Examination of a 7-year-old chi-
A. Periodontosis grade I ld revealed that all temporary maxi-
B. Periodontosis grade II llary molars were extracted. Mandi-
C. Periodontosis initial stage bular incisors were in contact with the
D. Catarrhal gingivitis palatal mucosa. What is the optimal
E. Localized periodontitis doctor’s tactics?
190. A 60-year-old patient needs a A. Fabrication of a removable laminar
partial removable denture for the maxi- claspless denture for the maxillary teeth
lla. The centric jaw relation has been restoration
determined. What is the next stage of B. Fabrication of an orthodontic device
prosthetics? for the treatment of deep overbite
C. Fabrication of a removable denture
A. Checking the wax denture constructi- with clasp fixation for the maxillary
on teeth restoration
B. Try-in of the denture D. Check-ups every six months until the
C. Fabrication of plaster models eruption of permanent teeth
D. Relining the denture E. Check-ups once a year until the
E. Taking impressions eruption of permanent teeth
191. A 40-year-old patient complai- 194. A 34-year-old dental patient has
ns of clicking in the region of his ri- an indication for the extraction of the
ght TMJ projection which occurs in 38 tooth for chronic fibrous periodonti-
the final phase of mouth opening. tis. The dental surgeon used torusal
The sounds in the joints arose after (Weisbrem’s) anesthesia. What nerves
prosthetic procedures. Mouth opening are blocked by this anesthesia?
is not restricted, the opening trajectory
is straight, the opening amplitude is 5 A. Superior alveolar, lingual, buccal
cm. What is the most likely diagnosis? B. Lingual, buccal
C. Superior alveolar, buccal
A. Subluxation of mandible D. Superior alveolar, lingual
B. Luxation of mandible E. Lingual, buccal, mental
C. Anterior dislocation of the articular
disc 195. A 40-year-old patient has an
D. Posterior dislocation of the articular indication for the extraction of
disc central maxillary incisors. What nerve
E. Perforation of the meniscus structures are blocked by the incisor
anesthesia?
192. A mother consulted a denti-
st about the lip lesions in her child. A. Nasopalatine nerve
Throughout the 1-2 year of life, the chi- B. Anterior superior alveolar branch of
ld had diathesis of moderate severity the infraorbital nerve
with affection of the face skin. Objecti- C. Middle superior alveolar branch of
vely: the lips are somewhat edematous, the infraorbital nerve
there are dry crusts on the vermilion D. Posterior superior alveolar branch of
border and the lip skin, in the mouth the infraorbital nerve
corners there are cracks and radial E. Dental nerve plexus
scars. The eyelids are edematous wi-
th hyperemic edges. What is the most 196. A 40-year-old male patient
likely diagnosis? complains of having experienced
seasonal pain (in spring and summer),
and a burning sensation in the lips for
3 years. Examination reveals edema
Кrok 2 Stomatology (англомовнi студенти) 2014 рiк 31

and hyperemia of the vermilion border A. Attack of bronchial asthma


of the lower lip, numerous erosions B. Coronary artery disease
sized 2-3 mm, crusts and cracks in C. Pulmonary artery thrombosis
the mouth corners. Palpation causes D. Bronchiectasis
moderate pain, regional lymph nodes E. Collapse
are not enlarged, the perioral skin is
not changed. What is most likely provi- 199. A 12-year-old girl complains of
sional diagnosis? acute pain in the left mandibular regi-
on, general weakness, fever up to
A. Actinic cheilitis 39o C , difficult swallowing. Objectively:
B. Exfoliative cheilitis, edematous form the face is asymmetric, the tongue is
C. Eczematous cheilitis plaqued, halitosis is present. Percussi-
D. Lichen planus, erosive form on reveals signs of acute periodontitis
E. Mycotic angular cheilitis of the 34, 36 teeth, as well as mobili-
ty of these teeth. Gums are hyperemic,
197. A 25-year-old male patient lives in edematous, there is infiltration on both
a locality where fluoride concentrati- sides of the alveolar process. Regi-
on in drinking water is 5,5 mg/l. Tooth onal lymph nodes are enlarged, tender
crowns are deformed due to destructi- on palpation. Vincent’s symptom is
on and wear of hard tissues. In the present. Blood test results: leukocytosis
22, 23, 32 teeth the enamel is chipped. with a left shift; urine contains traces
What form of fluorosis is observed in of protein. What is the most likely di-
this patient? agnosis?
A. Destructive A. Acute odontogenic osteomyelitis of
B. Streaked the mandible
C. Spotty B. Acute odontogenic purulent abscess
D. Chalky and speckled C. Exacerbation of chronic periodontitis
E. Erosive D. Acute odontogenic purulent
lymphadenitis
198. Prior to the preparation of a tooth E. Odontogenic submandibular abscess
for a porcelain-fused-to-metal crown,
a dentist anaesthetized a patient wi- 200. Parents of a 3-year-old child took
th a history of bronchial asthma wi- the child to a pedodontist for complex
th 2% solution of novocaine. A few dental care. The child has cerebral
minutes later, the patient complai- palsy. Objectively: there is multi-
ned of shortness of breath (more ple dental caries, gingival mucosa is
difficult expiration), there appeared hyperemic, oral hygiene is inadequate.
acrocyanosis, swollen neck veins, Ps- What method of dental care may be
100 bpm, rhythmic, AP- 180/110 mm appropriate in this case?
Hg. Percussion revealed box sound
over lungs. What is the most likely di- A. Under general anesthesia
agnosis? B. Under local injection anesthesia
C. Under local application anesthesia
D. Without anesthesia
E. Treatment after administration of
sedatives
Крок 2 Stomatology (англомовний варiант, iноземнi студенти) 2015 рiк 1

1. After examination a 5-year-old child was accident. The patient stayed conscious. He
diagnosed with acute general pulpitis of the 74 complains of headache, dizziness, general
tooth. What treatment will be most efficient in weakness, nausea. 12 hours after the injury
this case? the patient got "raccoon eyes"(periorbital
haematomas) within the limits of the circular
A. Pulp extirpation muscle of eye. What fracture does the victim
B. Vital amputation have?
C. Non-vital amputation
D. Biological method A. Fracture of skull base
E. Tooth extraction B. Le Fort II fracture of maxilla
C. Fracture of nasal bones
2. A 55-year-old female patient complains D. Bilateral fracture of zygomatic bones
of pain during eating food, erosions coveri- E. Fracture of frontal bones
ng oral mucosa. The disease onset was more
than one month ago. Objectively: the mucosa 7. In the process of root extraction of the 38th
of the gums, soft palate and dental arches is tooth with Lecluse root elevator a dentist has
unchanged, but has large bright-red erosions. pushed the root through into the perimandi-
The mucosa is easily damaged and exfoliates bular soft tissues. The root is situated under
when rubbed. The pain is slight. Impression the alveolar arch mucosa and can be clearly
smears contain Tzanck cells. What is the most palpated. What approach should the doctor
probable diagnosis? choose in the given case?
A. Pemphigus vulgaris A. Dissection of soft tissues covering the root
B. Lever’s bullous pemphigoid followed by root extraction
C. Benign nonacantholytic pemphigus B. To try and extract the root through the tooth
D. Erythema multiforme socket
E. Lichen ruber planus, bullous form C. Surgical root extraction through extraoral
submandibular way of access
3. A 17-year-old female patient complains of D. Further actions can be determined only after
gingival growth in the area of the upper and X-ray examination
lower front teeth. When examined, the gingi- E. To leave the root in the soft tissues
val papilla appear to be hyperemic, swollen,
cover the teeth crowns up to 1/2, bleed when 8. An 8-year-old girl complains of bleeding
touched. Pathology of the following system is gums. The child suffers from pancreatic di-
important in etiology of this disease: abetes. Objectively: gingival papillas in the
area of the upper and lower jaw teeth are
A. Endocrine hyperemic, swollen, cover 1/3 of the crown.
B. Cardio-vascular What tests are necessary to determine severity
C. Digestive of the illness?
D. Immune
E. Nervous A. X-ray radiography
B. Reoparodontography (Impedance
4. A 48-year-old male patient complains of
itching gums. Objectively: the gums are firm; phlebography of periodontium)
cold water causes short-time pain in the teeth. C. Capillary fragility test
X-ray imaging shows osteosclerosis of the D. PMA index
alveolar septum: close-meshed bone structure, E. Stomatoscopy
the alveolar septum height and intactness of 9. A 47-year-old female patient complains of
the cortical plates is retained. What is the most inability to eat hot and cold food, as well
probable diagnosis? as of intense pain caused by sour, sweet and
A. Initial parodontosis salty food. Objectively: there is a slight loss
B. Atrophic gingivitis of enamel on the molars and incisors. Probi-
C. Initial periodontitis ng and cold test cause acute pain. What is the
D. Periodontitis, stage I most likely diagnosis?
E. Parodontosis, stage I A. Hyperesthesia of dental hard tissues
5. A patient complains of spontaneous, B. Enamel erosion
paroxysmal, irradiating pain with short pain- C. Enamel necrosis
free intervals. The pain arose 2 days ago and D. Enamel hypoplasia
occurs only at night. Make a provisional di- E. Pathological abrasion of dental hard tissues
agnosis: 10. A 36-year-old patient complains of pain
A. Acute diffuse pulpitis under the dental bridge. After its removal the
B. Acute deep caries patient has been found to have an ulcer 0,3x0,5
C. Exacerbation of chronic periodontitis cm large on the alveolar process. The ulcer
D. Acute circumscribed pulpitis is slightly painful and soft, the surrounding
E. Acute purulent pulpitis mucosa is hyperaemic, submandibular lymph
nodes are not enlarged. What is a provisional
6. A 34-year-old patient had got in a car diagnosis?
Крок 2 Stomatology (англомовний варiант, iноземнi студенти) 2015 рiк 2

defect: the placement of the 23rd tooth is


A. Decubital ulcer incorrect. Objectively: the face is symmetrical.
B. Trophic ulcer The 24th tooth has buccal placement above the
C. Sutton aphtha occlusion area. The space between the 22nd
D. Cancerous ulcer and 24th is 3 mm. What additional examinati-
E. Tuberculous ulcer on is required?
11. A 45-year-old female patient presents to a A. All of the methods named below
dental clinic for prosthetics. Objectively: the B. Pont analysis
47, 46, 45, 35, 36, 37 teeth are missing. There C. X-ray examination
is dentoalveolar vertical displacement of the D. Measuring the length of the dentititon
17, 16, 26, 27 teeth; the alveolar process is E. Korkhaus analysis
hypertrophied, the cervical margins are not
exposed. When the teeth are closed, the di- 16. A 48-year-old female patient had been deli-
stance between the occlusal surfaces of teeth vered to a hospital with a cheek wound and
and the alveolar process is about 6 mm. Speci- severe headache, nausea, dizziness. It is known
fy the most efficient method of treatment of from the history that the patient had got a
secondary deformation in this patient: trauma as a result of a fall. After examination
she was diagnosed with a contused lacerated
A. Pulp removal and teeth grinding wound of cheek, closed craniocerebral injury,
B. Surgical treatment brain concussion. This trauma can be qualified
C. Grinding as:
D. Hardware-associated surgical treatment
E. Disocclusion A. Concominant trauma
B. Isolated trauma
12. A 58-year-old patient has made an appoi- C. Combined trauma
ntment to make a denture. Objectively: the D. Polytrauma
22nd tooth is stable and intact. The alveolar E. Single
crest is atrophied; palate is flat. Removable
denture is to be made. What approach regardi- 17. Parents of a 6-year-old child are concerned
ng the 22nd tooth should be chosen by a denti- about their child refusing to eat and havi-
st? ng grey-and-yellow coating in the oral cavi-
ty. The symptoms are observed over a peri-
A. To make a telescopic crown od of one year. The child complains of sharp
B. To retain the tooth dryness and burning pain in the mouth.
C. Removal of tooth pulp The anamnesis states chronic bronchitis
D. Tooth extraction and systematic treatment using anitibiotics.
E. To make a stump crown Objectively: swollen and hyperemic mucous
membranes of lips and cheeks are covered in
13. A 35-year-old patient has made an appoi- yellow and pale-grey coating. There is coati-
ntment with a doctor to have oral cavity sanati- ng on the tongue, which is connected with the
on. Objectively: the vestibular surface of the tissue, cannot be removed and has infiltration
22nd tooth has a hard tissue defect localised basis. What is the most probable diagnosis?
in the mantle dentin; the dentin is hard, dark-
coloured, the floor of the cavity is coarse. Cold A. Chronic candidiasis of mouth
stimulus and probing are painless. What is the B. Acute herpetic stomatitis
most probable diagnosis? C. Chronic recurrent aphthous stomatitis
D. Erythema multiforme
A. Chronic median caries E. Acute candidiasis of mouth
B. Enamel erosion
C. Cuneiform defect 18. A 24-year-old female patient has Ri-
D. Dental fluorosis, destructive form chmond crown being made to restore the
E. Hard tissue necrosis crown of the central maxillar incisor. The cap
is completed. What step should be the next?
14. A 12-year-old boy complains of a cavity in a
tooth on the lower left jaw. Objectively: 1/3 of A. To fit the cap on the tooth stump and place
the 36th tooth is destroyed, the carious cavity the post in the root canal
opens into the dental cavity; there is sensitivity B. To solder the post with the cap
to cold stimulus; probing and percussion are C. To fit the cap and the post to the tooth root
painless. X-ray imaging shows the periodontal D. Making of combination dental crown
fissure of the 36th tooth roots to be widened. E. Tooth fixation with cement
What is the provisional diagnosis?
19. A patient complaining of constant dull pain
A. Chronic fibrous periodontitis in the 38th tooth has made an appointment wi-
B. Chronic simple pulpitis th a dentist. Mouth opening is restricted to 1
C. Chronic gangrenous pulpitis sm. The patient is prescribed the extraction
D. Chronic granulating periodontitis of the 38th tooth. What kind of anaesthesia
E. Chronic granulomatous periodontitis should be administered?
15. An 11-year-old girl complains of a cosmetic
Крок 2 Stomatology (англомовний варiант, iноземнi студенти) 2015 рiк 3

A. Веrcher-Dubov anaesthesia A. Chronic fibrous pulpitis


B. Mandibular B. Exacerbation of chronic pulpitis
C. Torusal C. Acute deep caries
D. Extraoral administration of mandibular D. Chronic deep caries
anaesthesia E. Chronic fibrous periodontitis
E. Tuberal
24. A 32-year-old male patient complains of
20. A 67-year-old patient complains of dryness and burning pain in the back of the
recurrent erosion on the red border of the tongue. The symptoms have been lasting for
lower lip. Objectively: the erosion is oval 1 week. The pain increases when spicy food
in shape, 0,8х1,3 sm in size, covered in thin is taken. A day prior to that the patient had
scabs that reveal glossy surface with punctate suffered from pneumonia and had been taking
bleeding when removed. There are atrophic antibiotics. Objectively: the skin and mucosa
areas of the red border detected. Infiltrati- of oral cavity are pale. The tongue mucosa
on elements are absent. The submandibular is hyperemic and swollen; there is crumbling
lymph nodes are not enlarged. What diagnosis grey-and-white coating in the folds of the back
can be assumed? of the tongue; the lateral surfaces of the tongue
are desquamated. The saliva is thick and drags
A. Manganotti’s abrasive precancerous cheilitis in threads after a spatula. Choose the most
B. Leukoplakia, erosive ulcerative form effective drug complex for topical treatment.
C. Keratoacanthosis
D. Bowen’s disease A. Methylene blue + Pimafucin (Natamycin)
E. Cheilitis glandularis B. Hydrocortisone ointment + Solcoseryl
C. Decaminum (Dequalinium chloride) +
21. A 66-year-old patient complains of dense Hydrocortisone
raised induration at the tip of the nose. D. Furacilinum (Nitrofural) + Methylene blue
Protruding part of the induration often breaks
off and then continues to grow. Objectively: E. Furacilinum (Nitrofural) + Solcoseryl
there is an affected area with focus up to 1 25. A 37-year-old woman has made an appoi-
sm in diameter at the tip of the nose. There ntment to extraxt the 25th tooth. What kind of
is a protrusion growing from the focus; the anaesthesia is the most advisable?
protrusion is 1-1,2 sm high, brownish-grey in
colour, with dense consistency. What diagnosis A. Unilateral tuberal, infraorbital and palatinal
can be assumed? anaesthesia
B. Unilateral tuberal and palatinal anaesthesia
A. Cutaneous horn C. Unilateral infiltration anaesthesia from the
B. Keratoacanthoma vestibular surface
C. Rhinophyma D. Unilateral infraorbital and incisoral
D. Fibroma anaesthesia
E. Hard papilloma E. Unilateral infraorbital and palatinal
22. A 53-year-old patient complains of pain anaesthesia
and clicking in the left temporomandibular 26. A 70-year-old patient with all his lower jaw
joint. Objectively: the face is symmetrical, teeth lost is undergoing the process of fitti-
palpation of the lateral pterygoid muscles is ng individual dental spoon using Herbst tests.
painful on the left side. Mouth opening is When the mouth is opened slowly the spoon
restricted. Tomogram shows the bone outline rises in the front area. What area requires
of joint surfaces to be smooth. Which disease correction of the spoon placement?
of those given below corresponds with this cli-
nical presentation? A. Vestibular surface between canines
B. Behind the retromolar trigone to mandi-
A. Temporomandibular joint disfunction bulohyoid line
B. Rheumatic arthritis C. Frenulum of tongue area
C. Deforming arthrosis D. From the retromolar trigone to the place,
D. Acute posttraumatic arthritis where the 2nd molar will be placed
E. Joint ankylosis E. 1 cm from the midline on the sublingual end
23. A 38-year-old male patient complains 27. Preventive examination of a 4,5-year-
of carious cavity. Prior to that there were old child revealed some hidden cavities on
attacks of toothache with no external cause. the contact surfaces of the 54 and 55 teeth.
Objectively: the distal masticatory surface of After removal of the overhanging edges of
the 37th tooth has deep carious cavity filled the enamel the softened dentin could be
with softened pigmented dentine. Percussi- easily removed with an excavator within the
on is painless. Upon the extraction of the mantle dentin. Select the optimal material for
degenerated tissue cold water stimulus causes a permanent filling:
pain lasting about 1 minute. X-ray imaging
shows deformation of the periodontal fissure
in the area of the root apexes of the 37th tooth.
What is the most probable diagnosis?
Крок 2 Stomatology (англомовний варiант, iноземнi студенти) 2015 рiк 4

A. Compomer material A. Vankevych dental-gingival splint


B. Composite material B. Port gingival splint
C. Silicate cement C. Gunning gingival splint
D. Silicophosphate cement D. Weber dental-gingival splint
E. Polycarboxylate cement E. Zbarzh appliance
28. A 38-year-old female patient complains of 32. A 22-year-old patient complains of a pai-
pain in front of the external auditory meatus nful swelling in the right parotid gland. A week
projection, clicking when opening the mouth, earlier the patient got a cheek abrasion which
sensation of stuffiness in the ears. Objectively: healed under the purulent crust. Over the past
the face is symmetric, the trajectory of mouth two days the patient had observed progressi-
opening is straight. There is a I class Kennedy ng pain and fever up to 38, 6o C . Objectively:
defect, the 18, 17, 16, 26, 27, 28 teeth are missing. there is a soft tissue edema in the right parotid
Which anatomical structure takes the greatest region, the skin is slightly strained but has not
load in this situation? changed in colour. There is a dense painful
infiltration 2,5x3,5 cm large, the skin over it
A. Articular disc (meniscus) exhibits limited mobility. The mouth can be
B. Articular head fully opened, the mucous membrane around
C. Distal surface of the medial articular tubercle the orifice of the salivary duct is unchanged,
D. Bottom of the glenoid fossa of the temporal the saliva is transparent. What is the most li-
bone kely diagnosis?
E. Articular capsule
A. Acute lymphadenitis
29. An 11-year-old child complains of pain B. Exacerbation of chronic parotitis
and bleeding gums when eating, bad breath, C. Abscess of the parotid-masseteric region
weakness and headache that have been lasting D. Acute non-epidemic parotitis
for two days. Objectively: gingival mucosa in E. Epidemic parotitis
the area of front lower teeth is swollen, brightly
hyperemic, easily bleeds when touched. Gingi- 33. A 44-year-old patient complains of painful
val papillas are covered with dirty-grey coati- and restricted mouth closing, inability to take
ng; when coating is removed the underlaying food and sharp pain in the temporomandibular
surface is bleeding and painful. The occlusion joint. Objectively: the mouth is half-open, the
is deep. There are significant deposits of soft lower jaw is displaced in the distal projecti-
dental plaque. The regional lymph nodes are on, the lower jaw processes are displaced
slightly enlarged and painful when palpated. upwards; trismus of the masticatory muscles
Body temperature is 37, 6o C . What is the and swelling in the temporomandibular joint
provisional diagnosis? areas are observed. What is the most probable
diagnosis?
A. Acute ulcerative gingivitis
B. Acute catarrhal gingivitis A. Dystal mandibular dislocation
C. Chronic catarrhal gingivitis B. Bilateral mandibular fracture in the mandi-
D. Chronic hypertrophic gingivitis bular angle area
E. Generalized periodontitis C. Acute arthritis of temporomandibular joints
D. Bilateral ankylosis of temporomandibular
30. An 18-year-old student needs prosthetic joint
porcelain-fused-to-metal denture for the 11, E. Bilateral fractures of the condylar processes
21 teeth. There are no contraindications for of mandible
the use of such construction. What is the most
appropriate material for taking impressions? 34. A 58-year-old patient has significantly
narrowed mouth opening due to trauma and
A. Sielast burns of the face. He is prescribed partial
B. Stomalgin removable denture. What denture constructi-
C. Orthocor on is the most advisable in the given case?
D. Stens
E. Repin A. Folding denture
B. Clammer-stabilized clasp (bugel) prosthesis
31. A 67-year-old patient complains of pain, C. Attachment-stabilized clasp (bugel) dental
bleeding, chewing disorder. The anamnesis prosthesis
states domestic injuri of the lower jaw. Objecti- D. Jointed removable denture
vely: the upper jaw dentition is retained. The E. Simple removable partial laminar denture
lower jaw has no teeth, there is central fracture
of the lower jaw with bone defect, the place 35. A 40-year-old patient complains of pain in
of the fracture is swollen. What construction is the lower jaw, malocclusion. He sustained a
the most advisable in this case? trauma 6 hours ago. Examination revealed a
singular lower jaw fracture between the second
incisor and the left canine. Fragments di-
splacement is insignificant. All teeth are retai-
ned and stable. What is the optimal treatment?
Крок 2 Stomatology (англомовний варiант, iноземнi студенти) 2015 рiк 5

A. One arch dental braces Zbarzh set. Which construction provides the
B. Osteosynthesis with mini plate intra-oral fixation?
C. Full dental braces
D. Gartsatnikov splint A. Standard double arch
E. Vasilyev splint B. Aluminium arch bar
C. Plastic biteplate
36. Having recovered from acute respiratory D. Weber’s splint
disease, a patient has made an appointment E. Crown-supported soldered splint
with a dentist. The patient complains of pain
in the gums, bad breath, inability to take food, 41. A 32-year-old patient has made an appoi-
general weakness. Objectively: the gums are ntment with a dental surgeon to have oral
hyperemic, swollen, covered in necrotic dirty- cavity sanation performed prior to denture
grey coating; the gums bleed when the coati- installation. During examination the doctor
ng is removed. Microbiological study of ti- stated the following: the crown of the 25th
ssues revealed a great number of cocci, bacilli, tooth is destroyed. The tooth root is stable, and
fusobacteria, and spirochaete. Specify the drug its percussion is painless. Mucosa of alveolar
for etiotropic treatment. process is unchanged. X-ray imaging reveals
slight widening of periodontal fissure. What
A. Metronidazole provisional diagnosis can be made?
B. Galascorbinum
C. Potassium permanganate A. Chronic fibrous periodontitis of the 25th
D. Tripsin tooth
E. Carotolinum (Betacarotene) B. Chronic periodontitis of the 25th tooth
C. Chronic granulomatous periodontitis of the
37. A 4-year-old boy has been diagnosed wi- 25th tooth
th acute purulent periostitis of the upper D. Chronic granulating periodontitis of the 25th
jaw from the 64th tooth. Choose the correct tooth
approach in treatment. E. Cystogranuloma
A. The 64th tooth extraction, periosteotomy, 42. A 10-year-old girl complains of an aesthetic
pharmacotherapy flaw. The anamnesis states, that she had
B. The 64th tooth extraction, anti-inflammatory been sucking her right thumb up to the
pharmacotherapy age of 7. Objectively: the face lower thi-
C. Endodontological treatment of the 64th rd is somewhat reduced. The sagittal fissure
tooth, anti-inflammatory pharmacotherapy between the upper and lower incisors, is 9
D. Endodontological treatment of the 64th mm wide, class 2 according to the Angle
tooth, periosteotomy classification. Eshler-Bittner test leads to ini-
E. Periosteotomy, anti-inflammatory tial temporary improvement of the girl’s face,
pharmacotherapy followed by renewed deterioration. What cli-
nical malocclusion is the most probable in this
38. During the examinations and everyday case?
orthopaedic manipulations a dentist uses a
dental mirror. What is the way of its sterili- A. Maxillary macrognathia and mandibular
zation? micrognathia
B. Maxillary macrognathia
A. In the triple solution for 30 minutes C. Mandibular micrognathia
B. In the 0,5% ethyl chloride solution for 20 D. Maxillary prognathism with lateral
minutes compression
C. In the 6% hydrogen peroxide solution for 6 E. Mandibular retrognathia
hours
D. In the dry heat sterilizer at 180o C for 10 43. A 35-year-old patient complains about itch,
minutes burning and edema of lips. These presentati-
E. In the 0,01% chloramine solution for 10 ons occured a week ago. Objectively: there
minutes is reddening of vermilion border and skin,
especially in the region of mouth corners, there
39. A 45-year-old patient has central defect of are also vesicles, crusts, small cracks along wi-
the hard palate. The defect is 2х3 sm in si- th erythematous affection of vermilion border.
ze. Dentition is intact. What construction of What is the most likely diagnosis?
obturator is the the most advisable in this case?
A. Acute eczematous cheilitis
A. Palatal obturator B. Multiform exudative erythema
B. Pomerantseva-Urbanskaya obturator C. Acute herpetic cheilitis
C. Floating obturator D. Allergic contact cheilitis
D. Ilyina-Markosyan obturator E. Exudative form of exfoliative cheilitis
E. Laminar denture with obturating element
44. A 44-year-old male patient complains
40. A 32-year-old patient presented to a of fatigue and headache, limb numbness,
prosthetic dentistry clinic with a diagnosis of dry mouth, burning and pain in the tongue.
maxillary fracture.The fracture can be treated Objectively: skin and oral mucosa are pale.
by means of standard maxillo-mandibular There are painful cracks in the corners of
Крок 2 Stomatology (англомовний варiант, iноземнi студенти) 2015 рiк 6

mouth. Dorsum of tongue is smooth, glossy, A. Injection


with bright red striae. Blood count: Hb- 70 B. Audioanalgesia
g/l, RBCs - 1, 5 · 1012 /l, colour index - 1,6, C. Topical
leukopenia, thrombocytopenia, lymphocytosis. D. General anesthesia
What is the most likely diagnosis? E. Cryanesthesia

A. Addison-Biermer anemia 49. An 8-year-old child needs his 74th tooth


B. Chronic posthaemorrhagic anemia extracted due to exacerbation of chronic peri-
C. Late chlorosis odontitis. What kind of anaesthesia should be
D. Iron deficiency anemia used?
E. Aplastic anemia A. Mandibular anaesthesia
45. A 35-year-old male patient is prescribed B. Torusal anaesthesia
adhesive dental bridge. Objectively: the 15th C. Infiltration anaesthesia
tooth is lost. The 14th and 16th abutment D. Application anaesthesia
teeth are stable. The teeth crowns are of medi- E. Mental nerve block
um height. The teeth equator is pronounced. 50. A 25-year-old patient complains of pain
Occlusion is orthognathic. What area of the caused by eating sweet, hot and cold food;
abutment teeth MUST NOT be covered with pain ceases, when stimulation stops. Objecti-
denture framework? vely: the adjoining surface of the 36th tooth
A. Cervical circumference of the teeth has a deep carious cavity localised in the ci-
B. Tooth equators of the approximal surfaces rcumpulpar dentin. The dentin is softened.
C. Tooth equators of the oral surfaces Probing of the carious cavity floor is painful.
D. Fissures of masticatory surfaces What is the most probable diagnosis?
E. Below the equator of vestibular surfaces A. Acute deep caries
46. When a prosthodontist was preparing the B. Acute median caries
patient’s tooth, the patient had epileptic sei- C. Chronic fibrous pulpitis
zure that was subsequently terminated. What D. Chronic median caries
mistake had been made by the doctor? E. Chronic deep caries

A. No inquire into the patient anamnesis 51. A 24-year-old patient complains of aching
B. No inquire into the antecedent anamnesis pain in the 11 tooth that is getting worse during
C. No anaesthesia biting down on food. Two days ago the tooth
D. Crude preparation was filled for pulpitis. Objectively: the 11 tooth
E. Did not decline the appointment is filled. The thermal test causes no pain, verti-
cal percussion is slightly painful. X-ray picture
47. A 30-year-old patient complains of of the 11 tooth shows that the endodontic filli-
toothache caused by hot and cold stimuli. ng is 1 mm above the root apex. Which of the
The pain irradiates to the ear and temple. following methods will be most effective for
Previously there was spontaneous nocturnal eliminating this complication?
toothache. Objectively: on the occlusal surface
of the 37 tooth there is a deep carious cavi- A. Fluctuorization
ty communicating at one point with the tooth B. Ultrahigh frequency therapy
cavity. Probing at the communication point, as C. Relaxing incision
well as cold stimulus, cause acute pain. The D. Submucous injection of 1% solution of
pain persists for a long time. Electric pulp test hydrocortisone
result is 55 mA. What is the most likely di- E. Analgetics
agnosis? 52. A removable denture is being made for a
A. Exacerbation of chronic pulpitis patient. At the stage of designinig the denture
B. Acute diffuse pulpitis accessory material - isocol - is used. What
C. Exacerbation of chronic periodontitis group does it belong to?
D. Chronic concrementous pulpitis A. Insulation
E. Acute purulent pulpitis B. Impression
48. A 34-year-old-patient complains of hard ti- C. Modeling
ssues defect of the 21st and 22nd teeth. He is D. Forming
referred for porcelain-fused-to-metal crowns E. Polishing
installation for his 21st and 22nd teeth wi- 53. A 46-year-old patient, after his recovery
thout removal of tooth pulp. What kind of from flu, has suddenly started suffering from
anaesthesia administration should be used in pain in the 36th tooth, the crown of which had
tooth preparation? been destroyed. The soft tissues surrounding
the lower jaw are swollen. There are high fever
up to 39o C and chill observed. Objectively:
there is prominent swelling of the left lower
jaw alveolar mucosa (both oral and vestibular
surfaces). The 34th, 35th and 37th intact teeth
Крок 2 Stomatology (англомовний варiант, iноземнi студенти) 2015 рiк 7

are mobile. Percussion is painful. The gingival anterior palatal bars, tonsils. The submandi-
pockets produce purulent exudate. Vincent’s bular lymph nodes are slightly enlarged, pai-
symptom is observed. What diagnosis can be nless. Name the causative agent of this disease.
assumed?
A. Coxsackie virus
A. Acute odontogenic mandibular osteomyeli- B. Herpes simplex virus
tis starting from the 36th tooth C. Epstein-Barr virus
B. Exacerbation of chronic periodontitis of the D. Klebs-Loeffler bacillus
36th tooth E. Bordet-Gengou bacillus
C. Acute suppurative mandibular periostitis
starting from the 36th tooth 58. A 12-year-old girl has complaint of carious
D. Periodontitis cavity in a tooth. Objectively: there is class 1
E. Exacerbation of chronic mandibular carious cavity according to the Black’s classi-
osteomyelitis fication in the 36th tooth; it is localised in the
parapulpar dentin; the mouth of the cavity is
54. A doctor treating an AIDS-positive patient wide. The dentin is dense and pigmented. It
has accidentally pierced his own skin of the is sensitive to cold stimulus, percussion is pai-
palm with a root needle. What should be the nless. What is the most probable diagnosis?
first course of actions?
A. Chronic deep caries
A. Squeeze out blood, process the breach with B. Chronic median caries
70% alcohol solution C. Acute deep caries
B. Squeeze out blood, process the breach with D. Acute median caries
strong solution of potassium permanganate E. -
C. Apply tourniquet to the shoulder
D. Process the breach with 5% iodine solution 59. A 35-year-old female patient has complai-
E. Process the breach with 3% hydrogen nts of cosmetic defects of the front upper
peroxide solution teeth crowns. The defects have been increasi-
ng for the last 10 years. The patient suffers
55. A 45-year-old patient complains of mi- from unpleasant sensations when brushing her
ssing teeth in the lower jaw on the right. teeth, and when chemical stimuli are appli-
Objectively: the 46, 45, 38, 48 teeth are mi- ed. Objective examination revealed defects
ssing. The 47 tooth is mobile (1 grade mobi- localised in the enamel of the front upper teeth
lity), the crown of the 44 tooth is decayed vestibular surface. The defects are oval, saucer-
by 1/2. What orthopedic construction should shaped, and have clear margins. Probing and
be recommended for restoration of masti- cold stimuli sensitivity test resulted in positive
catory efficiency and prevention of periodontal reaction. Make the diagnosis.
overload of the 47, 44 teeth?
A. Enamel erosion
A. Bugel denture with clasp fixation for the 47, B. Enamel hypoplasia
44, 34 teeth C. Cuneiform defect
B. Soldered bridge supported by the 48, 44 D. Chemical necrosis of the tooth
teeth E. Hyperesthesia of tooth hard tissues
C. Porcelain-fused-to-metal bridge supported
by the 47, 46 teeth 60. A prosthodontist uses a mouth mirror for
D. Bridge supported by the 47 tooth examination and manipulation during regular
E. Small saddle denture with clasp fixation for appointments. How are mouth mirrors sterili-
the 47, 44 teeth zed?

56. A 53-year-old female patient complains A. With 6% hydrogen solution freshly prepared
of feeling of tightness and coarseness in the on the day of use for 6 hours
mucosa, the red border and the skin of the face. B. In a dry-heat sterilizer under 180o Celsius for
Objectively: the skin of the face has butterfly- 10 minutes
shaped damage, the red border is infiltrated C. With 1% chloramine solution for 10 minutes
and tightly covered in hyperkeratinized scales. D. With 0,5% sulfachlorantin solution for 20
When an attempt is made to remove them, minutes
they bleed and are painful. What disease is this E. With triple solution for 30 minutes
clinical presentation characteristic of? 61. A 56-year-old male patient complains of
A. Lupus erythematosus pain in the left parotid-masticatory area and
B. Chronic hyperplastic candidiasis progressing facial asymmetry first noticed
C. Lupus vulgaris (Tuberculosis luposa) one month ago. The diagnosis is the cancer
D. Lichen ruber planus of parotid gland - T2N2. What method of
E. Secondary syphilis treatment is optimal in the given case?

57. A 7-year-old boy complains of increased


body temperature, up to 38o C , headache, sore
throat. Objectively: there are erosions on the
slightly hyperemic mucosa of the soft palate,
Крок 2 Stomatology (англомовний варiант, iноземнi студенти) 2015 рiк 8

A. Combined method coarse. The anamnesis states short-time pain


B. Surgical extraction of neoplasm caused by chemical stimuli. What is the provi-
C. Radiation therapy sional diagnosis?
D. Chemotherapy
E. Lymph nodes removal A. Acute initial caries
B. Chronic initial caries
62. A 20-year-old male patient complains of C. Acute superficial caries
sharp pain in the mouth, increase of body D. Systemic hypoplasia of enamel
temeperature up to 38, 5o C , headache and E. Dental fluorosis
aching joints, general weakness. The disease
onset was 3 days ago due to overexposure 66. A 47-year-old patient is undergoing dental
to cold. Objectively: the lips red border has treatment of erosive/ulcerative form of li-
hemorrhagic scabs, oral mucosa has large chen ruber planus. 1-month-long conservati-
erosions and ulcers merging with each other ve therapy was uneffective. What physi-
and covered with greyish-white coating on the otherapeutic method is the most effective as
background of diffuse hyperemia. Conjuncti- a part of complex therapy?
vitis is observed. The skin of the forearms has
erythematous spots sized 1,5 cm in diameter, A. Cryodestruction
with blisters in their center. What is the most B. Ultra-high-frequency treatment
probable diagnosis? C. Electrophoresis
D. Ultraviolet irradiation
A. Stevens-Johnson syndrome E. D’arsonvalization
B. Erythema multiforme
C. Drug-induced stomatitis 67. A 21-year-old patient complains of a
D. Lyell’s syndrome constant progressing throbbing pain in the 27
E. Behcet’s syndrome tooth. Objectively: a large carious cavity is fi-
lled with softened dentin, the tooth cavity is
63. A 36-year-old female patient has complai- closed. Probing of the cavity floor is painless,
nt of the 23rd tooth filling constantly falling percussion causes acute pain, there is grade
out. Objectively: the 23rd tooth has carious II tooth mobility. Palpation of the mucous
cavity of the V class according to the Black’s membrane in the projection of the root apex of
classification of caries lesions. A dentist has the 27 tooth causes pain. Radiological changes
made a decision to restore anatomical shape of are absent. What is the most likely diagnosis?
the tooth with dental inlay. What is the speci-
fic feature of the cavity preparation for dental A. Acute purulent periodontitis
inlay in this case? B. Exacerbation of chronic periodontitis
C. Acute diffuse pulpitis
A. Designing oval-shaped cavity D. Acute serous periodontitis
B. Extention of the cavity floor E. Acute purulent pulpitis
C. Creation of retention points
D. Creation of additional platform 68. A child is 13 years old. The third upper
E. Making cavity outlet to the palatal surface tooth is fully cut, but situated slightly above
the occlusion surface. What kind of abnormal
64. A 68-year-old patient, having suffered from placement is it?
stroke, has time to time bouts of retching,
which lead to unilateral diclocation of the A. Supraversion
temporormandibular joint. Setting the joint B. Torsoversion
was possible only with anaesthesia. Objecti- C. Vestibular
vely: absent are the following teeth - from the D. Infraversion
21st to the 28th, 15th, 16th, 17th, 18th, 36th, E. Oral
37th, 44th, 46th, 47th. What kind of denture
construction will be optimal for the duration 69. A 47-year-old patient complains of ulcer on
of treatment? the back of the tongue. The anamnesis states
that the ulcer had been preceded for a long
A. Removable laminar denture with mouth term by protruding infiltrate that was gradually
opening restrictor growing until the ulcer appeared. Objectively:
B. Schroder appliance with sliding joint the back of the tongue has shallow oblong
C. Petrosov appliance with restrictor ulcer surrounded with bumps with undermined
D. Yadrova appliance edge. What is the most probable diagnosis?
E. Khodorovych-Burgonska appliance with
restrictor A. Tuberculous ulcer
B. Primary syphilis
65. Parents of a 12-year-old child are concerned C. Tongue actinomycosis
about the child having white spots on the D. Tongue abscess
frontal teeth of the upper jaw; the spots E. Decubitus ulcer
appeared half a year ago. Objectively: there
are chalky spots detected in the cervical zone 70. A 16-year-old patient has made an appoi-
of the 11th, 12th, 13th, 21st, 22nd, 23rd teeth ntment with a doctor due to the following
vestibular surface. The enamel in those spots complaints: enlarged lymph nodes and fistula
is dull; probing revealed it to be pliant and in the submandibular area; general weakness;
Крок 2 Stomatology (англомовний варiант, iноземнi студенти) 2015 рiк 9

low grade fever. Objectively: the submandi- brown exudate without cholesterol crystals.
bular lymph nodes are dense and slightly pai- What is the provisional diagnosis?
nful, with clearly detectable margins. Caseous
substance is produced from the fistulous tract. A. Osteoclastoma
What is the most probable diagnosis? B. Mandubular hemangioma
C. Adamantinoma
A. Tuberculosis of lymph nodes D. Mandubular cyst
B. Actinomycosis of lymph nodes E. Cholesteatoma
C. Syphilitic lymphadenitis
D. Chronic osteomyelitis 75. A 20-year-old female patient complains
E. Subcutaneous granuloma of pain caused by eating sweet or sour food.
Objectively: the vestibular surface of the 11th
71. A 42-year-old patient complains of pain tooth has a chalky spot, coarse when probed.
in the right side of her head, restricted The centre of the spot has defect localised in
movements of the lower jaw, clicking sound, the enamel. Fedorov-Volodkina index equals
periodic spasms of chewing muscles. Objecti- 2,5 (Oral Hygiene Index). What is the most
vely: the face is symmetric, mouth openi- probable diagnosis?
ng is restricted. On palpation of the right
temporo-mandibular joint (TMJ) there are A. Acute superficial caries
crepitation and clicking accompanying mandi- B. Acute median caries
ble movements. Examination of the oral cavity C. Enamel hypoplasia
revealed also a Kennedy’s class II defect on the D. Initial caries
right. What is the most likely diagnosis? E. Enamel necrosis
A. Pain dysfunction of the right TMJ 76. A 50-year-old male patient complains
B. Acute arthritis of the right TMJ of "small sore"on the sublingual mucosa.
C. Sclerosing osteoarthritis of the right TMJ Pathomorphologic study of tissue sampli-
D. Contracture of the right TMJ ng revealed the following: polymorphism
E. Myositis ossificans of spiny epithelial cells has progressed into
atypism; increased mitosis; giant and multi-
72. A 13-year-old child suffers from nucleate cells; acanthosis; hyperkeratosis and
odontogenic osteomyelitis starting from the parakeratosis; basal membrane and basal layer
36th tooth and complicated by abscess of the are retained. What is the most probable di-
pterygomandibular space. The 36th tooth is agnosis?
to be extracted; it is necessary to open the
pterygomandibular space. These operations A. Bowen’s disease
require the following type of anaesthesia: B. Leukoplakia, erosive form
C. Lichen ruber planus, hyperkeratotic form
A. General anaesthesia D. Lupus erythematosus, erosive/ulcerative
B. Веrcher-Dubov central anaesthesia form
C. Infiltration anesthesia E. Decubitus ulcer
D. Tuberal and palatinal anaesthesia
E. Torusal anaesthesia 77. A 15-year-old adolescent complains of hi-
gh body temperature up to 39, 5 − 40, 0o C ,
73. Clinical stage of checking the clasp (bugel) vomiting, headache and sore throat, especially
dental prosthesis framework as it should be when swallowing. Objectively: oral mucosa is
placed in the oral cavity revealed the denture swollen, hyperemic. The patient is diagnosed
arch to come off unevenly from the hard palate with acute catarrhal gingivitis. The tonsils are
and alveolar process creating the gap up to 0,8 hypertrophic and covered with yellow-grey
mm in width. What method can be applied to coating that does not spread from lymphoid
remove this defect? tissue and is easily removed. Submandibular,
cervical, and occipital lymph nodes are signi-
A. To make a new framework for clasp dental ficantly enlarged since the very first day of
prosthesis illness and are painful when palpated. The li-
B. To make adjustments to the framework using ver and spleen are enlarged. What is the most
crampon forceps probable diagnosis?
C. To make adjustments to the framework using
dental hammer and bench anvil A. Infectious mononucleosis
D. To heat the metal using a gasoline blow torch B. Scarlatina
and make adjustments by pressing to the model C. Diphtheria
E. This kind of defect does not require D. Herpetic angina
adjustments E. Measles
74. A 48-year-old man has an X-ray image of 78. A patient complains of loss of feeling in
his lower jaw made during sanation of the oral the left side of his lower lip and chin, whi-
cavity. Destruction of the bone tissue in the ch occurred after the complicated extraction
gonial angle area is detected, the pathology is of the 37th tooth. Electric pulp test (EPT)
not connected with tooth roots. The affected revealed decrease in electroexcitability of the
area has clear margins. Puncture consists of left lower jaw teeth. What is the most probable
Крок 2 Stomatology (англомовний варiант, iноземнi студенти) 2015 рiк 10

diagnosis? A. Lipoma
B. Brancial cleft cyst
A. Neuritis of the left inferior dental nerve C. Retention cyst of the left submandibular
B. Neuralgia of the left inferior dental nerve salivary gland
C. Alveolitis of the 37th tooth socket D. Hemangioma
D. Herpes Zoster n.Trigemini E. Chronic lymphadenitis
E. Acute osteomyelitis of the body of mandible
84. A 52-year-old patient complains of pain
79. During the extraction of the 17 tooth the di- and a swelling in the right parotid region.
stal buccal root broke off and remained in the These manifestations have been present for
socket. Select a tool to extract the root: about 2 years. Over the last month the swelli-
ng has grown bigger, pain has intensified.
A. Bayonet forceps, straight elevator Objectively: the face is asymmetric due to the
B. Straight forceps, straight elevator dense infiltrate in the right parotid region. The
C. S-shaped forceps poorly circumscribed, painful formation infi-
D. Beak-shaped forceps, angular elevator ltrates the surrounding tissues. At the right
E. Angular elevators side of neck in front and behind the sternoclei-
domastoid muscle there are enlarged, dense,
80. A partial removable denture is being made mobile lymph nodes. The right naso-buccal
for a 50-year-old patient. What impression groove is flattened, the corner of mouth is
trays are the more advisable? downturned. The mouth opens freely. The are
A. Split stock tray pronounced symptoms of the right facial nerve
B. Metal stock tray paresis. What disease can be suspected?
C. Disposable stock tray A. Adenocarcinoma of the parotid salivary
D. Perforated stock tray gland
E. Customized stock tray B. Chronic parotitis
81. An obturator is being made using Ilyina- C. Actinomycosis of the parotid-masseter
Markosyan technique for a 45-year-old-patient region
to compensate for the defect of the hard and D. Chronic lymphadenitis
soft palate. What type of connection between E. Pleomorphic adenoma of the parotid gland
the fixing and obturating parts will be used? 85. A 48-year-old patient complains of the
A. Button lower jaw teeth mobility. Van Thiel dental spli-
B. Band nt is to be made for prosthodontic treatment.
C. Clasp What construction elements are supposed to
D. Joint fix it in place?
E. Spring A. Whole piece proximal grip clasps
82. A 10-year-old child has fallen and hit stairs B. Full metal crowns
with his teeth. An appointment with a dentist C. Wire clasps
was made only 2 days later. There are complai- D. Parapulpar posts
ns of cold stimulus causing sharp pain. Objecti- E. Equator crowns
vely: the 11th tooth has transversal defect 86. A 38-year-old patient has made an appoi-
at 2/3 of the crown, the pulp is significantly ntment with a dentist to extract the 46th tooth
exposed. Probing causes sharp pain. Percussi- in preparation for prosthetics. Objectively:
on is painful, the tooth is mobile. The gums are the 46th tooth is destroyed by 2/3. What ki-
slightly hyperemic. What method of treatment nd of anaesthesia is the most advisable for the
should be used in this case? extraction of the 46th tooth?
A. Vital extirpation A. Torusal
B. Non-vital extirpation B. Веrcher-Dubov
C. Biological method C. Mandibular
D. Vital amputation D. Infiltration
E. Non-vital amputation E. Mental
83. A 33-year-old patient complains of slowly 87. A 13-year-old boy complains of general
growing tumor mass in the left submandi- weakness, high body temperature up to 39o C ,
bular area. Objectively: a tumor with soft lack of appetite, constant pain in the body of
elastic consistency can be palpated in the left the lower jaw. Objectively: observed is signifi-
submandibular area; the tumor changes its cant asymmetry of the face caused by soft ti-
configuration when squeezed with fingers and ssues swelling in the left buccal and submandi-
is painless. The skin covering the tumor has bular areas. Mouth opening is restricted.
no changes in colour, movable. There are no Intraoral examination revealed the following:
changes detected in the oral cavity. Salivary the 34th, 35th, 36th and 37th teeth are mobi-
glands function normaly. Shrinking-inflation le; teeth percussion is painful. The crown of
symptom is absent. What is the most probable the 36th tooth is completely destroyed. The
diagnosis? mucosa of those teeth is hyperemic and painful
when palpated. Muff-like enlargement of the
Крок 2 Stomatology (англомовний варiант, iноземнi студенти) 2015 рiк 11

lower jaw alveolar process is detected. What is immobile teeth. The marginal periodontium
the most probable diagnosis? in the area of the 14th-11th, 21st-23rd teeth
has inflammation symptoms; gingival and peri-
A. Acute mandibular odontogenic osteomyeli- odontal pockets are 4-5 mm deep. Whole piece
tis metal and plastic denture construction is to be
B. Acute mandibular hematogenous made for the patient. What are the specifics
osteomyelitis of the construction edge placement relative to
C. Acute mandibular odontogenic purulent the marginal periodontium in this case?
periostitis
D. Ewing’s sarcoma A. Circular garland covering necks of all
E. Abscess of the right submandibular area abutment teeth
B. Formation of gum-level circular ledge
88. A 25-year-old woman complains of pain C. Formation of subgingival circular ledge
in the 15th tooth, which is caused by thermal D. Construction should be composed of
stimuli and quickly ceases, when stimulation separate crowns
stops. Objectively: visually detected is a defect E. Construction should have neither lining nor
with smooth walls that join under an angle, circular ledge
which is located in the cervical zone of the 15th
tooth vestibular surface. Thermometry is pai- 92. Parents of an 8-year-old child with Down
nful; percussion is painless. What is the most syndrome took the child to a doctor for oral
probable diagnosis? cavity sanitation. After the examination entai-
ling great difficulties the child was found to
A. Cuneiform defect have four teeth decayed as a result of chronic
B. Enamel erosion periodontitis. What kind of anesthesia should
C. Enamel hypoplasia be chosen for surgical sanitation in one visit?
D. Fluorosis
E. Superficial caries A. Phlebonarcosis
B. Conduction anesthesia
89. A 40-year-old female patient complains of C. Mask anesthesia
short-term pain in the 34th tooth caused by D. Endotracheal anesthesia
thermal stimuli. Objectively: the distal surface E. Endotracheal anesthesia through a
of the 34th tooth has a carious cavity localised tracheostoma
in the circumpulpar dentin. The walls and floor
of the cavity are softened, faintly pigmented; 93. During examination of the 11-year-old chi-
probing of the floor is slightly painful. Cold sti- ld’s oral cavity the 23rd tooth vestibular posi-
mulus causes pain that ceases when stimulati- tion was detected. Correlation of the 16th and
on stops. What is the most probable diagnosis? 46th is Angle class 1, and 26th and 36th is
Angle class 2. The width of the 23rd crown
A. Acute deep caries is 8 mm. The dentition lacks 4 mm to place the
B. Acute median caries 23rd properly. Front teeth occlusion is normal.
C. Chronic deep caries Choose the optimal treatment approach.
D. Chronic median caries
E. Chronic fibrous pulpitis A. Move the upper lateral teeth on the right
side in distal direction, than move the canine
90. A 24-year-old patient complains of new into the correct place
growth in the right lateral surface of the neck. B. Extract the canine; move the 24th and 25th
The new growth was first noticed 4 months teeth to replace the 23rd
ago, no appointment with doctor was made. C. Widen the upper and lower dentitions, and
Objectively: the face is asymmetrical due to the move the canine into the correct place
new growth in the upper third part of the right D. Widen the upper dentition, and move the
lateral surface of the neck. Palpation allows to canine into the correct place
detect the painless rounded new growth sized E. Extract the first premolar, and move the
3х3,5 cm; its consistency is dence and elastic; canine into the correct place
the skin abowe the growth has no changes of
colour and can be folded. The new growth is 94. A 47-year-old patient complains of a burni-
not melded with its surrounding tissues and is ng sensation and pain in the mouth. Objecti-
situated along the front edge of the sternoclei- vely: on the mucous membrane of cheeks along
domastoid muscle. Puncture consists of pale the line of teeth contact and in the corners of
yellow viscous fluid. What is the most probable mouth there are multiple polygonal bright red
diagnosis? erosions 1,0-1,5 cm in diameter located on the
hyperkeratinized plaque and opaque whitish
A. Brancial cleft cyst mucosa. Cytological analysis revealed kerati-
B. Phlegmon of neck lateral surface nizing epithelial cells. What is the most likely
C. Chronic lymphadenitis of neck lateral diagnosis?
surface
D. Lipoma of neck lateral surface
E. Acute serous lymphadenitis of neck lateral
surface
91. A female patient is 51-year-old, with
Крок 2 Stomatology (англомовний варiант, iноземнi студенти) 2015 рiк 12

A. Leukoplakia, erosive form A. Local precancerous hyperkeratosis of the


B. Lichen planus, erosive form lower lip
C. Erythema multiforme B. Cheilitis abrasiva praecancerosa Manganotti
D. Secondary syphilis C. Verrucous precancer of the lower lip red
E. Lupus erythematosus, erosive form border
D. Bowen’s disease
95. A 24-year-old female patient complains E. Erythroplasia of Queyrat
of the 15th tooth being lost. The defect is of
aesthetic nature. Objectively: the 14th and 99. A 23-year-old male patient complains of
16th teeth are intact, stable, clinical crowns gum bleeding when he brushes his teeth or eats
are tall with pronounced equators and normal tough food. Objectively: the gums of the front
orthognathic overlapping; X-ray image shows lower jaw are hyperemic, swollen and bleed
no pathologies in the periapical tissues. What when palpated. The oral and gingival mucosa
kind of denture should be recommended for in other areas are not affected. The occlusi-
the patient? on is deep. The teeth are firm, except for the
41st and 31st (class 1 mobility). X-ray imagi-
A. Adhesive dental bridge ng shows resorption of the alveolar septum in
B. Metal ceramic dental bridge the area of the 41st, 42nd, 32nd and 31st teeth
C. Metal plastic dental bridge up to 1/3 of the root length. What is the most
D. Plastic dental bridge probable diagnosis?
E. Cantilever bridge
A. Localised periodontitis
96. A12-year-old child has a carious cavity B. Generalised periodontitis, initial stage
on the surface of the 12th tooth. The cavi- C. Generalised periodontitis, stage I
ty is localised in the mantle dentin; its floor D. Catarrhal gingivitis
and walls are covered with pale softened E. Parodontosis, stage I
dentin. Probing the enamel-dentin border is
painful. Thermal stimuli cause short-term pai- 100. A 78-year-old patient complains of diffi-
nful reaction. Vertical percussion is painless, culties when taking food and deformity of the
horisontal percussion is slightly painful. Gingi- dentition. The anamnesis states the lower jaw
val papilla between the 12th and 13th teeth fracture 2,5 month ago. Objectively: there is a
is swollen, hyperemic, bleeds when touched. deformity of the lower jaw front area, which
What is the most probable diagnosis? equals 19 mm (artificial joint). Both fragments
of the lower jaw have 3 stable teeth each. What
A. Acute median caries, gingival papillitis prosthesis construction is the most advisable in
B. Acute deep caries, gingival papillitis this case?
C. Acute partial pulpitis, gingival papillitis
D. Acute serous periodontitis A. Oxman joint fixed denture
E. Exacerbation of chronic periodontitis B. Oxman joint denture with one pivot-point
C. Jointless removable denture
97. A 27-year-old female patient has her 26th D. Kurlandsky’s denture with shock absorbing
tooth cavity accidentially breached along the ball-clasp
mesial buccal line angle during treatment of E. Oxman joint denture with two pivot-points
acute deep caries of the 26th. Choose the opti-
mal method of treatment. 101. A 3,5-year-old child has symmetrical face,
the middle part is predominant in proporti-
A. Biological method ons, swallowing is infantile, breathing is nasal.
B. Non-vital extirpation In the oral cavity the dentition corresponds
C. Non-vital amputation with the age norms, the sagittal fissure is 3
D. Vital amputation mm, every tooth in the lateral part has its
E. Vital extirpation anatagonist, the lower teeth touch the hard
98. A 54-year-old patient complaining of palate. Miogymnastics with Dassa orbicularis
tumor-like growth on the red border of the oris activator is recommended. What function
left lower lip has made an appointment with is normalised by this apparatus in the given
a dentist. Examination revealed the affected case?
area to be sized 1х1,5 cm, and have irregular A. Lips closure
shape with clear margins. Focus of the affected B. Breathing
area does not protrude from the surrounding C. Chewing
red border; the affected area is greyish-white D. Swallowing
in colour and covered in thin closely placed E. Speech
scales. What is the most probable diagnosis?
102. A patient consulted a dentist about pain
in the 47 tooth. X-ray examination revealed
fragments of endodontic instruments in the
canals of the medial and distal root. In the
periapical zone there are well-defined areas of
bone destruction. Select the most appropriate
method of treatment:
Крок 2 Stomatology (англомовний варiант, iноземнi студенти) 2015 рiк 13

A. Tooth extraction A. Korkhaus appliance


B. Radectomy B. Vasylenko appliance
C. Tooth replantation C. Simple cotton ligature
D. Tooth hemisection D. Kalvelis appliance
E. Conservative treatment E. Begg appliance
103. After unilateral resection of the upper jaw 108. An injured patient complains of limited
a 52-year-old patient received the immediate- opening of the mouth, nose bleeding, skin
insertion denture. What is the term of its use? numbness in the infraorbital and lower eyelid
region. Objectively: there is face deformation
A. 1-3 months due to the depression of soft tissues in the left
B. 1-2 months cheekbone region, step deformity in the mi-
C. 7-12 months ddle part of the inferior margin of the left orbit
D. 2-3 years and in the area of the zygomatic alveolar crest.
E. 4-5 years What is the most likely diagnosis?
104. A 57-year-old female patient complains of A. Zygomatic bone fracture with displacement
burning pain, dry mouth, feeling of foreign of bone fragments
body on the tongue back and lateral surfaces, B. Fracture of the right zygomatic bone without
which disappears in the process of eating. displacement of fragments
The first case of such symptoms was noted C. Le Fort I fracture of maxilla
a year ago after psychic trauma. The patient D. Le Fort II fracture of maxilla
suffers from pancreatic diabetes and sleep di- E. Fracture of the malar arch
sturbance. Examination revealed the followi-
ng: the tongue mucosa is without significant 109. A patient has sustained blunt object
changes, moderately moistened. What is the trauma to the face central area. Objectively:
most probable diagnosis? the nasal bones are mobile, there is signifi-
cant swelling of the left temporal region soft
A. Glossodynia tissues, uneven "stair-like"shape of bones is
B. Candidal glossitis observed in the lower edges of both eyesockets
C. Glossitis areata exfoliativa and maxillo-zygomatic sutures, bleeding, open
D. Hunter-Moeller’s glossitis bite. The central part of the face is elongated.
E. Glossitis with fissured tongue What is the most probable diagnosis?
105. A patient presented to a dental clinic for A. Le Fort fracture of maxilla, type II
complex dental care. Objectively: the 37 tooth B. Le Fort fracture of maxilla, type I
exhibits a deep carious cavity communicati- C. Left temporal bone fracture
ng with the tooth cavity. There is no response D. Nasal fracture
to stimuli. Radiography reveals widening and E. Le Fort fracture, type III
deformation of the periodontal ligament space
in the apical region. What is the most likely 110. A 44-year-old female patient has complai-
diagnosis? nts of the face swelling at the right lower
jaw area and teeth mobility. Objectively:
A. Chronic fibrous periodontitis soft tissues are without changes, the regional
B. Chronic granulating periodontitis lymph nodes cannot be palpated. The alveolar
C. Chronic granulomatous periodontitis process and the body of the lower jaw near the
D. Chronic fibrous pulpitis 46th, 47th and 48th teeth are thickened, pai-
E. Chronic gangrenous pulpitis nless when palpated, have bulges. The teeth in
the thickened area are mobile. Puncture consi-
106. A patient with post-resection upper jaw sts of brown fluid without cholesterol crystals.
defect that invades the nasal cavity has made What is the provisional diagnosis?
an appointment with a prosthodontic clinic.
What denture is recommended in the given A. Osteoclastoma
case? B. Adamantinoma
C. Osteoma
A. Replacement denture with obturating D. Follicular cyst
element E. Odontoma
B. Floating obturator
C. Mouthguard 111. A 49-year-old patient consulted a dental
D. Forming denture surgeon about the oral cavity sanitation. He
E. Replacement denture has an indication for the extraction of the 16
tooth. History: the tooth hasn’t been treated
107. An 18-year-old patient with complaint of before, it has decayed within the last 4 years.
large diastem has made an appointment with Objectively: the 16 tooth’s crown is decayed by
prosthodontics specialist. Objectively: there is over 2/3, the mucosa exhibits no pathological
full lateral displacement of central incisors due changes. Which tool is required for the tooth
to absence of the 12th and 22nd teeth. What extraction?
instrument is the most advisable for moving
the central incisors closer together?
Крок 2 Stomatology (англомовний варiант, iноземнi студенти) 2015 рiк 14

A. Bayonet root forceps is semi-impacted. Palpation of the internal


B. S-shaped forceps (right) surface of the mandibular angle is painful.
C. Straight elevator What is the most likely diagnosis?
D. S-shaped closed-beak forceps
E. Crown bayonet forceps A. Phlegmon of pterygomandibular space
B. Acute tonsillitis
112. A 39-year-old patient, who works in a C. Phlegmon of the infratemporal region
printing house, complains of burning pain, D. Acute pericoronitis of the 38 tooth region
metallic taste in the mouth and bad breath. E. Phlegmon of peripharyngeal space
Objectively: the skin is sallow in colour. The
oral mucosa has black-and-blue spots. The 117. A 35-year-old patient complains of a
gums are hyperemic and swollen. The edge of neoplasm on the tip of his tongue which he
the gums has black-and-blue band. The palate injures with his teeth. The neoplasm someti-
is of yellow shade. Intoxication symptoms mes increases, and sometimes decreases in si-
are observed. What is the most probable di- ze. Objectively: on the tip of tongue there is a
agnosis? roundish well-defined neoplasm 0,5 cm in di-
ameter with broad base. The neoplasm is the
A. Lead stomatitis same colour as the mucosa of tongue. What is
B. Mercurial stomatitis the most likely diagnosis?
C. Bismuthic stomatitis
D. Melanosis A. Papilloma of tongue
E. Addison’s disease B. Abscess of tongue
C. Lipoma of tongue
113. A 16-year-old female patient complains of D. Hemangiofibroma of tongue
a deformation and restricted mouth opening E. Fibroma of tongue
since birth. Objectively: the face is symmetri-
cal, disproportionate, there is a bird-like face 118. A 37-year-old patient complains of bleedi-
symptom. The mouth opens up to 1 cm. The ng gums, bad breath, loose teeth, difficulti-
patient has an occlusal abnormality, namely es with chewing food. Objectively: gums are
deep incisal overbite. What pathology is found bluish-red in colour; periodontal pockets are
in this patient? 6 mm deep and contain serous substance;
class II teeth mobility; moderate deposits of
A. Micrognathia with congenital ankylosis subgingival and supragingival dental calculus;
B. Micrognathia with acquired ankylosis Oral Hygiene Index equals 3 points; there is
C. Maxillary protraction traumatical occlusuion along all the length of
D. Prognathism the dental arches. What is the provisional di-
E. Mandibular protraction agnosis?
114. To perform a differentiated diagnostics of A. Generalized periodontitis, stage II, chronic
the lower jaw displacement a patient was asked B. Generalized periodontitis, stage I, acute
to open his mouth as wide as possible, and then C. Hypertrophic gingivitis
the lateral displacement of the lower jaw and D. Histiocytosis X
face asymmetry were measured. What clinical E. Generalized periodontitis, stage II, acute
test is it?
119. A 6,5-year-old child has closed non-
A. Ilyina-Markosyan clinical functional test 3 pigmented fissures in the first permanent
B. Ilyina-Markosyan clinical functional test 1 molar, which have been revealed during
C. Ilyina-Markosyan clinical functional test 2 preventive examination. Enamel transparency
D. Ilyina-Markosyan clinical functional test 4 is retained, its probing does not reveal any
E. Eschler-Bittner clinical diagnostic test coarseness. Choose the optimal method of
treatment in this case.
115. What measurements are necessary to
determine the width of dental arch accordi- A. Non-invasive sealing
ng to the Pont’s method of analysis? B. Invasive sealing
C. Preventive filling
A. Crown width of four upper incisors D. ART technique
B. Crown width of upper central incisors E. Regular medical check-up
C. Crown width of six upper front teeth
D. Crown width of upper central incisors and 120. A 3-year-old child suffers from extreme
the first premolars gingivitis with slight teeth mobility; teeth are
E. Size of dentition frontal segment intact. The skin is dry, the hair is brittle, the
skin of the palms and soles of the feet is rough
116. A patient complains of limited mouth and covered in small cracks. What is the provi-
opening, pain during swallowing, fever up to sional diagnosis?
38, 5o C , weakness, indisposition. Objectively:
the mouth opens up to 1 cm. After Berchet
anesthesia examination of the oral cavity
revealed edema, hyperemia, tenderness of the
pterygomandibular fold. Lateral and posteri-
or pharyngeal walls are intact. The 38 tooth
Крок 2 Stomatology (англомовний варiант, iноземнi студенти) 2015 рiк 15

A. Papillon-Lefevre syndrome A. Loosely fill the socket with iodoform tampon


B. Niemann-Pick disease B. Fill the socket with hemostatic sponge
C. Hand-Schuller-Christian disease C. Lavage the socket with microcide solution
D. Eosinophilic granuloma (Taratynov’s di- D. Lavage the socket with 0,1% chlorhexidine
sease) solution
E. Letterer-Siwe disease E. Fill the socket with antibiotic dusting powder
121. Parents of an 8-year-old child have made 125. A 24-year-old female patient consulted
and appointment with an orthodontist. There a dentist about pain in the 26 tooth. After
are complaints of their child having traumas a physical examination the patient was di-
of oral mucosa. Objectively: decreased hei- agnosed with exacerbation of chronic peri-
ght of the face lower part, everted lower lip, odontitis of the 26 tooth. During the tooth
deep labiomental furrow, milk occlusion. The extraction the coronal part of the tooth was
upper incisors fully cover the lower ones; cutti- accidentally broken. Further manipulations wi-
ng surface of the lower incisors make contact th bayonet forceps failed to extract the tooth
with the anterior third of the palate. Mesiodi- roots. What actions are to be taken for the
stal ratio of the canines and the first permanent successful root extraction?
molars is normal. Grouping of the upper and
lower front teeth is dissimilar. Make the di- A. To disjoin the roots by using a drill and
agnosis according to the Kalvelis classification. fissure burr
B. To send the patient to the hospital
A. Deep traumatic overbite C. To use Lecluse elevator
B. Deep incisor overbite D. To complete the tooth extraction during the
C. Deep neutral occlusion next visit
D. Deep prognatic (roof-shaped) occlusion E. To use angular elevator
E. -
126. Preventive examination of a 5-year-old
122. A 64-year-old patient with edentulous child revealed a habit of lower lip biting. What
jaws has a minor uniform atrophy of maxilla malocclusion may develop if the child keeps
and a pronounced atrophy of lateral parts of this habit?
mandible. The mucosa in these parts makes
longitudinal folds, the alveolar crest is mobile A. Anterior bite
in the anterior part. The condition of mandi- B. Prognathic bite
ble should be taken into consideration at the C. Open bite
following stage of fabrication of a complete D. Deep overbite
removable denture: E. Cross-bite

A. Taking differentiated functional impression 127. Parents of an 8-year-old girl are concerned
B. Taking anatomical impression that she is chewing food too slowly. Objecti-
C. Taking compression functional impression vely: it is a period of transitional dentition.
D. Taking decompression functional impression The first permanent molars has neutral relati-
E. Determining the central occlusion onship; sagittal fissure is 2 mm wide. The upper
front teeth cover the lower ones by 2/3. Name
123. A 46-year-old patient complains of masti- the pathology.
cation disruption caused by the lack of the
34th, 35th and 36th teeth. The antecedent A. Deepening of incisor overbite
anamnesis is as follows: the teeth were B. Widening of the sagittal fissure
extracted 3 months ago due to complication C. Tooth-alveolar lengthening of the front teeth
of cariosity. The patient anamnesis: the history D. Tooth-alveolar shortening of the lateral teeth
of tonsillitis, rheumatoid arthritis and Botki- E. Decrease of the lower face height
n’s disease. After the appointment with this 128. When checking construction of the
patient instruments should be sterilized in the soldered dental bridge with the 35th and 38th
following way: abutment teeth the following was detected:
A. Specialized procedure pores in the place of soldering together
B. Dry-heat sterilizer abutment crowns with the intermediate part;
C. Processing with lysol masticatory tubercules are sharply defined;
D. Processing with 0,1% chloramine solution there is early contact with tooth-antagonists;
E. General procedure the intermediate part makes tight contact with
the alveolar process mucosa. How can those
124. A 49-year-old patient has a tooth flaws be corrected?
removed from the left upper jaw under plexus
anaesthesia with articaine-forte (Articaine +
Epinephrine). After the operation the tooth
socket did not fill with blood clot. How to
prevent alveolitis in this case?
Крок 2 Stomatology (англомовний варiант, iноземнi студенти) 2015 рiк 16

A. Dental bridge shoold be remade ent?


B. Intermediate part should be corrected
C. Masticatory surface should be corrected, and A. Partial removable laminar metal-based
soldered places - filed denture
D. Masticatory surface should be corrected, and B. Dental bridge
the height of the flushing part is to be increased C. Partial removable laminar plastic denture
up to 2 mm with clasp-retainers
E. Tooth-antagonists sholud be filed down D. Partial removable laminar denture with
supporting-retaining clasps
129. A 14-year-old girl complains of indisti- E. Clasp (bugel) dental prosthesis
nct pronunciation that developed at the age
of 14 after the acute respiratory viral disease. 133. A 22-year-old patient presented to a
Examination revealed normal face and normal prosthetic dentistry clinic because of missing
teeth alignment, occlusal disharmony was not of the 21 tooth, the 11 and 22 teeth are intact.
found. Palpation didn’t reveal cleft palate. The 21 tooth was extracted 2 months ago. What
Uvula doesn’t move during pronunciation of construction is most suitable in this case?
sounds, its palpation does not cause gag reflex.
What is the reason for indistinct pronunciation A. Two-stage implantation
of sounds? B. One-stage implantation with simultaneous
fabrication of porcelain-fused-to-metal crown
A. Paresis of the soft palate and uvula muscles C. Partial removable denture
B. Adenoid vegetations D. Swaged and soldered denture supported by
C. Palatal slit the 11 and 22 teeth
D. Hypertrophy of lingual tonsil E. Metal-plastic denture supported by the 11
E. Deformation of the bite and 22 teeth
130. A 53-year-old male patient has made 134. A 48-year-old patient has undergone uni-
a repeat appointment after 3 days upon the lateral resection of the upper jaw. He needs
extraction of the 36th tooth. Spot X-ray of the a resection prosthesis. Objectively: the 21, 22,
36th tooth made from within the oral cavity 23, 24, 25, 26 teeth are stable, hard palate is
revealed the remaining mesial root. What tools broad, flat. What connection of clasps with the
should be used to extract this root? base will reduce the instability of the resection
prosthesis?
A. Inward bent elevator and upper beak-shaped
forceps without spike A. Articular
B. Outward bent elevator and upper beak- B. Stable
shaped forceps without spike C. Elastic
C. Inward bent elevator and lower beak-shaped D. Fixed
forceps without spike E. Does not matter
D. Outward bent elevator and lower beak-
shaped forceps without spike 135. A 15-year-old girl complaining of
E. Inward and outward bent elevators and moderate swelling of the left parotic-
upper beak-shaped forceps without spike masticatory area has made an appointment wi-
th a dental surgeon. Palpation revealed the left
131. A 35-year-old female patient seeks parotid gland to be bulgy, dense, and painless.
prosthetic dentistry. Objectively: the 18, 14, 13, Its duct secretes turbid saliva. The duct orifice
12, 11, 21, 22, 23, 24, 28 teeth need restoration. is dilated, the surrounding mucosa is cyanotic,
The crowns confining the defect are high and pastose, with teeth imprints. What disease is it?
stable. To restore the integrity of the dentition
it is planned to fabricate a cast clasp denture. A. Chronic parenchymatous sialoadenitis
What kind of mechanical fixator should be B. Chronic interstitial sialoadenitis
used? C. Calculous sialadenitis (sialolithiasis)
D. Mixed tumor
A. Bar fixation E. Cyst
B. Supporting-retaining clasps
C. Retaining clasps 136. The department of maxillofacial surgery
D. Telescopic fixation admitted a patient who needs repair of a post-
E. Dentoalveolar clasps traumatic defect of nose wing up to 3,0 cm in
diameter. The trauma occured six months ago.
132. A 42-year-old female patient has What kind of grafting is indicated in this clini-
made an appointment with a prosthodontic cal situation?
office to make a denture. Objecti-
vely: dental formula is as follows:
18 . . . . 13 12 11 21 22 23 . . . . 28
48 47 46 45 44 43 42 41 31 32 33 34 35 36 37 .
The patient has deep occlusion; clinical crowns
are low; equator is not pronounced. The pati-
ent suffers from epileptic seizures. What kind
of denture should be prescribed for this pati-
Крок 2 Stomatology (англомовний варiант, iноземнi студенти) 2015 рiк 17

A. Grafting with chondrocutaneous flap of the A. Mandibular anaesthesia with 3%


auricle Scandonest solution
B. Grafting with local tissues of nasolabial or B. Intravenous anaesthesia with thiopental
cheek regions sodium solution
C. Grafting with pedicle flap of frontal and C. Infiltration anesthesia with 2% lidocaine
buccal regions solution
D. Grafting with tubed pedicle (Filatov’s) flap D. Mandibular anaesthesia with Ultracain DS
E. Free grafting with dermal flap forte (epinephrine concentration 1:100 000)
E. -
137. A 25-year-old female patient consulted a
dentist about acute pain in the upper jaw on 141. During the surgical removal of a retention
the left. The pain occurs during eating. Objecti- cyst of the lower lip a 14-year-old boy complai-
vely: on the distal approximal surface of the ned of sudden weakness, dizziness, nausea.
26 tooth there is a cavity filled with light soft Objectively: the skin is covered with cold
dentin. Probing causes a slight pain along the sweat. Respiration is frequent, pulse is weak,
dentin-enamel junction, percussion is painless. AP is decreased (90/60 mm Hg), the hands are
Cold water causes quickly abating pain. What cold. What is the most likely diagnosis?
is the most likely diagnosis?
A. Syncope
A. Acute median caries B. Toxic shock
B. Chronic median caries C. Traumatic shock
C. Acute deep caries D. Anaphylactic shock
D. Chronic fibrous pulpitis E. Collapse
E. Chronic deep caries
142. A 10,5-year-old child complains of painful
138. A 40-year-old female patient has made an rash on his lips. Objectively: the red border
appointment with a dentist. She complains of of the lips is swollen, hyperemic, covered in
pain in her front teeth of the upper and lower fissures and numerous scabs of dried blood.
jaws, which is caused by thermal, chemical and The skin of the upper lip has small blisters
mechanical stimuli. Temporomandibular joint containing serous substance, which merge wi-
is also slightly painful. Objectively: the pati- th each other in some places. Maceration and
ent has deep occlusion; the enamel of lingual weeping skin also can be observed, especially
surface of the front upper teeth and vestbular in the corners of the mouth. What is the most
surface of the front lower teeth is rubbed off probable diagnosis?
exposing the glossy pigmented dentin. What is
the most probable diagnosis? A. Exematous cheilitis
B. Meteorological cheilitis
A. Pathologic brygmus C. Atopic cheilitis
B. Chemical necrosis of the tooth D. Actinic cheilitis
C. Physiological brygmus E. Exfoliative cheilitis
D. Hard tissues erosion
E. Stainton-Capdepont syndrome 143. An 11-year-old child complains of dryness
and tightness of the lips, which the child is li-
139. Whan conduction anaesthesia had been cking constantly. Objectively: red border of the
administerd by a dentist, in 2 seconds a lips is dry, infiltrated, hyperemic and covered
patient developed the following symptoms: in numerous thin scales. The skin of the lips is
loss of consciousness, seizures, dilated pupils, undamaged. What is the provisional diagnosis?
absent pupillary response to light, low arteri-
al pressure, laboured breathing, muffled heart A. Meteorological cheilitis
sounds. What is the most probable diagnosis? B. Atopic cheilitis
C. Cheilitis exfoliativa
A. Anaphylactic shock D. Allergic contact cheilitis
B. Syncope E. Exematous cheilitis
C. Collapse
D. Cardiac infarction 144. Parents of a 7,5-year old child brought him
E. Stroke to a dentist for oral cavity sanitation. Objecti-
vely: DEF (for primary teeth) + DMF (for
140. A 13-year-old boy needs his 46th tooth permanent teeth) index = 4, Green-Vermillion
extracted due to chronic granulomatous peri- index = 2,5. Fissures of the first permanent
odontitis. He has pancreatic diabetes as a molars are open, intact, non-pigmented. What
concomitant disease. Choose the most advi- method of primary prevention of dental caries
sable anaesthetic and its way of administration. may be appropriate in this case?
A. Non-invasive hermetization
B. Invasive hermetization
C. Application of fluoride varnish
D. Application of antibacterial varnish
E. Application of calcium gels
145. Parents of a 5-year-old child has made
Крок 2 Stomatology (англомовний варiант, iноземнi студенти) 2015 рiк 18

an appointment with a dentist for preventi-


ve examination of their child. Objectively: A. Osteosynthesis
the DMF index (Decayed, Missing, Filled)=5, B. One arch smooth dental braces
the gingival mucosa is pale pink in colour, C. Full dental splint
Fedorov-Volodkina index is 2,5. For oral hygi- D. Weber splint
ene the child should use a toothbrush with the E. Vankevytch splint
following kind of bristle:
150. A 37-year-old patient had an appointment
A. Soft with his orthopaedist about dental prosthetics.
B. Medium During the preparation of hard dental tissues
C. Hard for a fixed denture he was given conducti-
D. Extra-soft on anesthesia. During the injection the pati-
E. Extra-hard ent developed cold sweat, sudden pallor and
cyanosis, thready pulse. The patient stayed
146. A 28-year-old patient complains of experi- conscious, but became relaxed and lethargic.
encing gum discomfort, gingival haemorrhages What is the provisional diagnosis?
and pain, especially during eating, for a
week. Objectively: the gums are swollen and A. Collapse
markedly hyperemic, especially within the 43, B. Anaphylactic shock
42, 41, 31, 32, 33 teeth, soft plaque and tartar C. Intoxication
are present, gingival sulcus is up to 2 mm deep. D. Syncope
What is the most likely diagnosis? E. Hypertensive crisis

A. Acute deep gingivitis 151. A 48-year-old patient is in grave conditi-


B. Chronic catarrhal gingivitis on: body temperature is of 39, 8o C , the face
C. Hypertrophic gingivitis is asymmetric due to the infiltration of the
D. Initial stage of chronic generalized peri- submental and both submandibular triangles.
odontitis Speech is difficult as the tongue is displaced
E. Exacerbation of initial generalized peri- upwards. Swallowing is painful. Sublingual pli-
odontitis cae are markedly hyperemic and edematous.
What is the most likely diagnosis?
147. An 18-year-old patient complains of
chalky spot in the 23rd tooth, which is slightly A. Phlegmon of mouth floor
painful when cold stimulus is applied. Objecti- B. Abscess of tongue root
vely: the vestibular surface near dental cervix C. Phlegmon of submental triangle
of the 23rd tooth has a white spot 2-3 mm in D. Phlegmon of alveololingual groove
size. Probing is painless and reveals coarseness E. Phlegmon of submandibular triangle
of the surface. Thermal stimulus causes slightly
painful feeling. What is the most probable di- 152. A 32-year-old patient is hospitalised in
agnosis? an oral surgery department with perforating
wound of the cheek. The size of the wound
A. Acute initial caries opening is 1,5х4,0 mm. What kind of restorati-
B. Chronic initial caries ve surgical operation is the most advisable in
C. Fluorosis this case?
D. Local hypoplasia
E. Enamel necrosis A. Filatov-Gillies tube graft
B. Double-end flap
148. A 7-year-old child is diagnosed with C. Rotation flap
complete dislocation of the 11th tooth caused D. Rauer’s double shoulder-breast flap
by a trauma that happened 24 hours ago. E. Opposite triangular flaps (Z-plasty)
The tooth was brought along. Objectively: the
socket edges are slightly hyperemic, bone walls 153. A 60-year-old patient has been undergoi-
are intact; the tooth socket is filled with blood ng the procedure of checking the complete
clot. What approach is the most advisable for removable dentures construction and fixing
the therapy? teeth on wax bases. The following flaws have
been detected: fissure between the teeth on
A. Reimplantation of the 11th tooth preceded the frontal area and tubercule contact in the
by the canal filling lateral area. What mistake had been made?
B. Reimplantation of the 11th tooth followed
by the canal filling A. Anterior occlusion was determined instead
C. Extraction of the 11th tooth of central one
D. Reposition and fixation of the 11th tooth B. Posterior occlusion was determined instead
E. Socket closure with catgut of central one
C. Lateral occlusion was determined instead of
149. A 56-year-old patient suffering from central one
exacerbation of schizophrenia has been hospi- D. Models were plastered in a wrong way in an
talised in an oral in-patient department with a occluder
diagnosis of the lower jaw displaced fracture in E. Swabs were crushed, when central occlusion
the area of the 34th-35th teeth. What method was being determined
of treatment should be prescribed?
Крок 2 Stomatology (англомовний варiант, iноземнi студенти) 2015 рiк 19

154. A 20-year-old patient in the 8th month the following symptoms: tinnitus, weakness,
of pregnancy complains about an aesthetic paleness of skin. Vertigo was diagnosed. What
defect, speech disturbance due to the mi- first aid approach is the most advisable?
ssing upper front tooth. Objectively: the 12
tooth is missing. The 11, 13 teeth are intact, A. Put the patient in the Trendelenburg positi-
stable, of anatomical shape. The patient has on
orthognathic bite. What prosthesic design B. Intramuscular injection of 50% analgin
should be preferred? (mеtamisole sodium) solution
C. Intravenous injection of 0,1% epinephrine
A. Immediate prosthesis D. Subcutaneous injection of cordiamin
B. Plastic bridge E. Intravenous injection of prednisolone
C. Porcelain-fused-to-metal bridge
D. Plastic-to-metal bridge 159. A 15-year-old adolescent complains of a
E. Implant with a pocelain crown two-day long sharp pulsing pain in the tooth,
which intensifies when biting or touching
155. A 30-year-old patient complains of pain the tooth with the tongue. Objectively: the
and swelling in the area of the left parotid sali- 26th tooth contains composite material filli-
vary gland, which occurred 7 days after he had ng. Vertical and horisontal percussion of the
undergone abdominal cavity surgery. Objecti- tooth are painful, the tooth is slightly mobile
vely: body temperature equals 39o C , restricted in vestibular-oral projection. Gingival mucosa
mouth opening; dry mouth; when the gland of the affected area is hyperemic, swollen,
is massaged, there is purulent exudate being sharply painful when palpated. X-ray imaging
secreted from its duct. The patient can be di- shows no changes. What is the most probable
agnosed with the following disease: diagnosis?
A. Acute non-epidemic parotitis A. Acute purulent periodontitis
B. Acute epidemic parotitis B. Acute purulent pulpitis
C. Phlegmon of submasseteric space C. Acute serous periodontitis
D. Parenchymatous parotitis D. Acute serous pulpitis
E. Phlegmon of parotid-masticatory region E. Exacerbation of chronic periodontitis
156. A 16-year-old girl complains of cosmetic 160. A 5-year-old child has a cavity localised in
defect of the front teeth - there are dark the deep dentin on the masticatory surface of
spots and hard tissues defects. The spots were the 75th tooth. The cavity is filled with softened
detected in the process of teething, the defects pigmented dentin and food remains. The chi-
developed later. The following diagnosis was ld complains of dull pain when taking food or
made: erosive form of fluorosis of the 16th, cold liquids. Probing of the cavity floor is sli-
11th, 12th, 22nd, 26th, 31st, 32nd, 36th, 41st, ghtly painful; when the dentin is removed with
42nd, 46th teeth. Choose the method of excavator, pink pulp is visible through the cavi-
treatment. ty floor. What is the most probable diagnosis?
A. Restoration treatment A. Chronic fibrous pulpitis
B. Remineralization treatment B. Chronic deep caries
C. Prosthetics C. Acute deep caries
D. Surgical treatment D. Acute diffuse pulpitis
E. Regular medical check-up E. Chronic gangrenous pulpitis
157. An 8-year-old boy has thickening of the 161. A 9-year-old child complains of dull pain
lower jaw body in the area of the 83rd, 84th in a tooth, which increases when the tooth is
and 85th teeth. In the given area a rounded pressed. One month ago the tooth was treated
painless tissue protrusion can be detected; the for pulpitis. Objectively: the 36th tooth is fi-
Dupuytren’s symptom is positive. Deciduous lled; percussion is sharply painful; mucosa is
teeth are intact. X-ray image of the lower jaw hyperemic, swollen. X-ray imaging does not
made in lateral projection shows the focus of show any changes. What is the most probable
bone destruction with clear margins in the area diagnosis?
of the 83rd, 84th and 85th teeth. The 44th tooth
follicle is shifted down in distal direction, the A. Acute serous periodontitis
crown is projected on the destruction focus. B. Exacerbation of chronic pulpitis
What is the clinical diagnosis? C. Acute diffuse pulpitis
D. Acute suppurative pulpitis
A. Follicular cyst of mandibula from the 44th E. Exacerbation of chronic periodontitis
tooth
B. Fibrous dysplasia of mandible 162. A 50-year-old female patient complains
C. Mandibular ameloblastoma of pain in the upper left jaw, high fever up
D. Radicular cyst of mandibula from the 84th to 38, 5o C , worsening of her general conditi-
tooth on. The disease onset was 3 days ago. Objecti-
E. Mandibular osteoclastoma vely: the face is asymmetrical due to swelling
of the right cheek soft tissues, palpation is pai-
158. In the process of the 26th tooth extracti- nful, the skin is hyperemic. The crown of the
on a 34-year-old patient suddenly developed 16th tooth is destroyed completely; percussion
Крок 2 Stomatology (англомовний варiант, iноземнi студенти) 2015 рiк 20

sensitivity is positive. There is hyperemia and A. Oral rinsing with sodium bicarbonate soluti-
edema of the alveolar process soft tissues in the on, chlorhexidine solution
area of the 15th, 16th and 17th teeth vestibular B. Teeth cleaning with chewing gum
surface. What is the most probable diagnosis? C. Teeth flossing
D. Teeth brushing using soft-bristled toothbrush
A. Acute odontogenic maxillary periostitis in and anti-inflammatory toothpaste
the area of the 16th tooth E. Teeth cleaning with interdental stimulators
B. Exacerbation of chronic periodontitis of the and interdental brushes
16th tooth
C. Acute odontogenic maxillary osteomyelitis 167. A 42-year-old female patient presented to
starting from the 16th tooth a dentist for oral sanitation. She has a history
D. Acute odontogenic right-side maxillary of tonsillitis, rheumatoid arthritis, Botkin’s di-
sinusitis sease. What are the conditions of instrument
E. Chronic odontogenic maxillary osteomyelitis treatment after the patient’s visit?

163. A 60-year-old patient complains of a A. Under the special scheme


growth in the mental region, which is painless B. Under general conditions
and slowly grows in size. Objectively: there C. Dry-air sterilization
is rounded infiltrate with clear margins in the D. Processing with lysol
mental region; it is slightly painfiul, the skin E. Processing with 0,1% solution of chloramine
under it is very thin. Palpation in the vestibule
of mouth reveals dense band going from the 168. A 50-year-old patient complains of his
growth to the destroyed 33rd tooth. What is lower jaw teeth mobility. Objectively: the
the most probable diagnosis? dentition is intact; the necks of the teeth are
bared; the tooth crowns are tall; teeth mobility
A. Odontogenic granuloma of the face is class 1-2. What kind of dental splint is the
B. Odontogenic abscess of the mental region most advisable?
C. Atheroma of the mental region
D. Acute lymphadenitis the mental region A. Elbrecht splint
E. Chronic lymphadenitis of the mental region B. Mamlock splint
C. Full crown splint
164. A 43-year-old patient complains of mobi- D. Equator crown splint
lity of all his teeth. Upon examination and E. Cap splint
additional tests the following diagnosis is
made: generalized periodontitis. What kind of 169. A patient complains of an increasing
treatment does this patient need? new growth in the left sublingual area, whi-
ch he first noticed 2 months ago. Some ti-
A. Complex me after that he noticed this new growth to
B. Prosthodontic secrete large amount of clear viscous fluid,
C. Surgical which was colorless and tasteless; initially this
D. Therapeutic caused the new growth to become smaller but
E. Orthodontal subsequently it started to grow again. Objecti-
vely: there is a rounded new growth sized 3x3
165. A 38-year-old female complains of cm in the left sublingual area, its consistency is
experiencing pain in her lower lip for half soft and elastic, there is no pain. The mucosa
a year, difficult eating and talking. The pati- above is thinned out, semitransparent, bluish
ent sought medical help at place of residence, in shade. What is the most probable diagnosis?
but the administered treatment was ineffecti-
ve. Examination of the lower lip revealed A. Retention sublingual cyst
a deep fissure in the centre which affected B. Hemangioma of sublingual area
the vermilion border and partly the mucous C. Dermoid cyst of sublingual area
membrane of lips. The fissure was covered D. Post-traumatic cyst of sublingual area
with a brown crust, after its removal the fi- E. Pleomorphic adenoma of sublingual gland
ssure started bleeding. The fissure edges are
hyperkeratinized, infiltrated. Select the opti- 170. After extreme exposure to cold a 42-year-
mal treatment tactics: old patient complains of headache in the left
frontal lobe and the left upper jaw. Objecti-
A. Fissure excision vely: the face is symmetrical; left nasal meatus
B. Application of keratoplastic agents breathing is obstructed, and serous-purulent
C. Irradiation with helium-neon laser discharge is being produced; palpation of the
D. Application of corticosteroid ointment suborbital area and further along the mucogi-
E. Application of antibiotic ointment ngival fold in the 24th, 25th teeth projecti-
on reveals slight pain. Percussion of these
166. A 44-year-old male patient diagnosed with teeth is painless. The 24th tooth has a filli-
chronic generalized periodontitis, class II, has ng. The alveolar process mucosa has no visi-
had flap surgery of the lower jaw. What kind of ble changes. X-ray imaging shows decreased
oral hygiene procedures are recommended for pneumatization of the left maxillary sinus.
this patient during the first 2-3 days after the What is the provisional diagnosis?
surgery?
Крок 2 Stomatology (англомовний варiант, iноземнi студенти) 2015 рiк 21

A. Exacerbation of chronic odontogenic maxi- A. To file down the edges


llary sinusitis B. To insert parapaulpar posts
B. Acute periodontitis of the 24th C. To use extra hard material
C. Exacerbation of chronic periodontitis of the D. To make retention furrows in the area of
24th enamel-dentin joining
D. Acute rhinogenous maxillary sinusitis E. To shape a wide-angled slant of enamel
E. Acute albuminous periostitis of the left
maxilla 175. A 19-year-old young man complains of
unaesthetical look of his teeth, nearly all of
171. A 8,5-year-old child is practically in good which have spots and defects; his teeth have
health. There is a complaint of pain in the had such appearance since the moment of
upper left tooth, due to it having been physi- teething. The patient was born and had been
cally damaged 3 hours ago. Objectively: 1/2 of living until the age of 5 in the area, where the
the 21st tooth crown is destroyed, the pulp is fluorine content of drinking water was 3,2 mg/l.
significantly exposed, red, sharply painful and Objectively: the enamel of all his teeth is of
bleeding when probed. Percussion of the 21st dull chalky shade with separate dark-brown
tooth is sharply painful. Choose the optimal spots, and has multishaped defects with dense
method of the 21st tooth treatment. margins, that are the most pronounced on the
front upper teeth. What type of fluorosis does
A. Vital amputation this patient have?
B. Vital extirpation
C. Non-vital amputation A. Erosive
D. Non-vital extirpation B. Lined
E. Biological method C. Patchy
D. Chalky-speckled
172. A 34-year-old male patient complains of E. Destructive
a cosmetic defect, a cavity on the vestibular
surface in the cervical part of the 21 tooth. 176. To fill a medium depth carious cavity
Objectively: the carious cavity is within the in the 37th tooth (Black class 2) of a 35-
enamel, the floor and the walls are pigmented, year-old male patient a doctor has chosen a
probing and percussion and painless. There is technique of layer-by-layer tooth restoration.
no pain reaction to stimuli. What is the most What composite material should be applied for
likely diagnosis? covering the carious cavity walls and floor to
create the initial super adaptive layer?
A. Chronic surface caries
B. Acute surface caries A. Flowable
C. Necrosis of dental hard tissues B. Condensable
D. Acute median caries C. Macrofilled
E. Chronic median caries D. Microhybrid
E. Microfilled
173. A 6-year-old child complains of pain in the
right submandibular region. A day before he 177. A 13-year-old girl has bleeding gums and
complained of a sore throat. Objectively: the her front teeth are mobile. These symptoms
child is in moderately severe condition, body have been observed during the last month.
it is 37, 9o C . The face is asymmetric due to a Objectively: the gum mucosa in the area of
dense tender infiltrate in the right submandi- the lower incisors and canines is swollen,
bular region. The infiltrate is slightly mobi- hyperemic, bleeding when touched. There is
le, the overlying skin is hyperemic. There is class I teeth mobility, periodontal pockets are
fluctuation in the center. The teeth are intact. 3 mm deep. Orthopantomogram shows bone
What is the most likely diagnosis? tissue destruction of the alveolar septums up
to 1/3 of their height. Congestion of the lower
A. Acute suppurative non-odontogenic jaw front teeth can be observed. Oral Hygi-
submandibular lymphadenitis ene Index equals 4,2. What specialist must be
B. Acute serous non-odontogenic submandi- consulted first?
bular lymphadenitis
C. Phlegmonous adenitis A. Orthodontist
D. Acute suppurative odontogenic submandi- B. Neurologist
bular lymphadenitis C. Endocrinologist
E. Sialadenitis D. Hematologist
E. Gastroenterologist
174. A dentist treats a 35-year-old patient from
chronic deep caries of the 24th tooth that has 178. A 45-year-old patient complains of being
mesial-occlusal-distal cavity. What should the unable to chew food due to loss of the lateral
dentist do to prevent the cavity edges from chi- teeth. The rest of his teeth have mobility of the
pping? 1-2 type. What construction is the most advi-
sable for dental prosthetics?
Крок 2 Stomatology (англомовний варiант, iноземнi студенти) 2015 рiк 22

A. Removable denture splint A. Combination of photopolymer material with


B. Fixed dental bridge fiberglass framework
C. Metal-based prosthesis B. Immediate prosthesis
D. Clasp (bugel) dental prosthesis C. Adhesive dental bridge
E. Removable laminar denture D. Porcelain-fused-to-metal whole piece dental
bridge
179. A 65-year-old male patient complains of a E. Wire splint
long-lasting pain in a mandibular tooth on the
right. The pain increases on biting. The patient 183. A 65-year-old male patient complai-
has a history of this tooth treatment for cari- ns of crepitation and clicking in both
es. Objectively: the cavity on the masticatory temporomandibular joints; pain when movi-
surface of the 36 tooth is filled, percussion ng his mandible to the right; ear noise;
causes pain responce. Along the mucogingi- dry mouth; glossalgia. He has been using a
val junction in the projection of the 36 tooth complete removable denture for the mandi-
roots there is a fistula with purulent discharge. ble for 6 months. The patient has no history
What is the most likely diagnosis? of rheumatosis. Objectively: the lower third of
the face is shortened, mental fold is strongly
A. Chronic granulating periodontitis pronounced, mouth corners are downturned,
B. Radicular cyst of the mandible there are angular fissures and cracks. Palpation
C. Exacerbation of chronic periodontitis reveals crepitation during TMJ moving. What
D. Chronic fibrous periodontitis is the most likely diagnosis?
E. Chronic granulomatous periodontitis
A. Costen’s syndrome
180. During examination of an 8-year-old chi- B. Temporomandibular arthritis
ld there were spots detected on the vesti- C. Temporomandibular arthrosis
bular surface of the 11th, 12th, 21st and 22nd D. Temporomandibular dislocation
teeth. The spots are pearly-white, glossy, pai- E. Temporomandibular osteoarthritis
nless when probed, and gradually merge with
non-changed enamel. Ultraviolet irradiation 184. A 45-year-old patient undergoes teeth
revealed the spots to fluoresce blue. The child preparation. 15 minutes after anesthetization
has been living up to the age of 3 in the region, with 4% solution of Ubistesin forte the pati-
where fluorine content of drinking water was 2 ent developed hyperemia of skin, increased
mg/l. What kind of non-carious lesion is it? heart rate, headache, syncope. Previously the
patient had not exhibited such reaction to this
A. Fluorosis, speckled form anesthetic. What complication occurred in the
B. Fluorosis, lined form patient?
C. Fluorosis, erosive form
D. Local hypoplasia A. Arterial pressure rise
E. Systemic hypoplasia B. Anaphylactic shock
C. Syncope
181. A woman with complaints of restricted D. Collapse
mouth opening has made an appointment wi- E. Myocardial infarction
th a prosthodontic clinic. What muscles are
responsible for the downwards mowement of 185. An accident victim with an injury of face
the lower jaw? and the temporal region was diagnosed wi-
th a fracture of the zygomatic arch. Which
A. Mylohyoid muscle, digastric muscle, geni- processes of the skull bones are damaged?
ohyoid muscle
B. Masticatory muscle, medial pterygoid muscle A. Temporal process of the zygomatic bone and
C. Temporal muscle, lateral pterygoid muscle, the zygomatic process of the temporal bone
digastric muscle B. Zygomatic process of the frontal bone and
D. Orbicular muscle of mouth, mylohyoid the zygomatic process of the temporal bone
muscle C. Temporal process of the zygomatic bone and
E. Geniohyoid muscle, medial pterygoid muscle the zygomatic process of the frontal bone
D. Zygomatic process of the maxilla and the
182. A 45-year-old patient is diagnosed wi- zygomatic process of the temporal bone
th generalized periodontitis; there is class E. Zygomatic process of the maxilla and the
II mobility of the 13th, 12th, 21st, 22nd, zygomatic process of the frontal bone
23rd teeth. The patient has made an appoi-
ntment with dental clinic to make a prostesis 186. A 20-year-old male patient complains of
after the extraction of the 11th tooth. What spontaneous pain in the 24 tooth, which arose
prosthodontic construction is the most advi- suddenly and persists for about 15 minutes.
sable for combined therapy of generalized Objectively: the distal surface of the 24 tooth
periodontitis in this case? exhibits a deep carious cavity with overhangi-
ng walls. The cavity is filled with light softened
dentin and communicates with the tooth cavity.
The cold stimulus causes acute, slowly abati-
ng pain. Percussion causes no pain response.
Select the best method of treatment:
Крок 2 Stomatology (англомовний варiант, iноземнi студенти) 2015 рiк 23

A. Vital extirpation this case?


B. Vital amputation
C. Biological method A. Cast removable splint
D. Devital amputation B. Kurlyandsky splint bar
E. Devital extirpation C. Cap splint
D. Portion crown
187. A 25-year-old patient complains of pain E. Half-ring splint
when biting on the 15 tooth. The pain arose
two days ago, has a constant aching nature 191. A 7-year-old child has to undergo plastic
and increased significantly over the last day. surgery of the upper lip frenulum. What
Objectively: the crown of the 15 tooth is gray, operation should be performed to lengthen
the medial contact surface exhibits a deep cari- the frenulum of the upper lip?
ous cavity communicating with the tooth cavi-
ty. Percussion causes acute pain, the gingival A. Dieffenbach grafting
mucosa in the projection of the 25 tooth root B. Thiersch grafting with local flaps
apex is hyperemic. The regional lymph node is C. Relaxing incisions of the mucosa
tender. Radiograph shows an ill-defined zone D. Grafting with a pedicle flap
of periapical bone destruction. What is the E. Szymanowsky grafting with local flaps
most likely diagnosis?
192. A 12-year-old boy is in the dental offi-
A. Exacerbation of chronic periodontitis ce to complete treatment of the 36th tooth
B. Acute serous periodontitis chronic periodontitis exacerbation. There are
C. Acute suppurative periodontitis no complaints. Objectively: occlusive dressi-
D. Chronic granulating periodontitis ng of the 36th tooth is intact; tooth percussi-
E. Acute serous periodontitis, intoxication stage on is painless; there are no pathologies of the
mucosa surrounding the 36th; mucosa palpati-
188. A 27-year-old patient has been referred on is painless. What material is the most advi-
by a prosthodontist for endodontic treatment sable for root canal filling in this case?
of the 45th tooth. Objectively: the 45th tooth
crown is destroyed; the lateral surface of the A. Sealer with gutta-percha post
tongue and the buccal mucosa have patches B. Zinc oxide eugenol paste
of greyish macerated epithelium slightly risi- C. Resorcinol-formalin paste
ng above the mucosa surface at the points of D. Calcium-containing paste
direct contact with the 45th tooth. The uvula E. Phosphate cement
and palatal bars are stagnant-red in colour;
hard palate has papulae surrounded with red 193. A patient with complaints of toothache in
margin and covered in greyish epithelium. The the left upper jaw has made an appointment
submandibular, cervical, supraclavicular, and with a dental clinic. He was diagnosed with
subclavicular lymph nodes are enlarged and chronic periodontitis of the 24th tooth. What
painless. What is the provisional diagnosis? kind of anaesthesia is necessary for painless
extraction of the 24th tooth?
A. Secondary syphilis
B. Chronic recurrent aphthous stomatitis A. Infraorbital and palatinal anaesthesia
C. Lupus erythematosus, patch stage B. Tuberal and palatinal anaesthesia
D. Soft leukoplakia (leucoplakia mollis) C. Infraorbital and incisor anaesthesia
E. Lichen ruber planus D. Tuberal and incisor anaesthesia
E. Surface and tuberal anaesthesia
189. A 20-year-old male patient with insulin-
dependent diabetes mellitus has been given 194. A 60-year-old patient needs a partial
anesthesia with Ultracain DS-forte for surgi- removable denture for the upper jaw. The
cal oral sanitation. What complications may central occlusion has been determined. What
develop in this case? is the next stage of prosthetics?

A. Abnormal levels of blood glucose A. Checking the wax denture construction


B. Increased blood pressure B. Try-in of the denture
C. Clonus C. Fabrication of plaster models
D. Atopic dermatitis D. Relining the denture
E. Toxic shock E. Taking impressions

190. A 43-year-old patient complains of mobi- 195. A 12-year-old boy, due to trauma of
lity and significant neck exposure of the lower the 44th and 45th teeth area, suffers from
front teeth. Objectively: the gums in the regi- the following: pathologic displacement of the
on of the 44, 43, 42, 41, 31, 32, 33, 34 teeth mandibular alveolar process; rupture of the
are pale and cyanotic, non-bleeding. The 42, alveolar process mucosa. What additional
41, 31, 32 teeth exhibit the I-II grade mobility. examination is necessary to specify the di-
The overcrowding of the 42, 41, 31, 32 teeth is agnosis?
present. The necks of the 42, 41, 31, 32 teeth
are exposed by 1/2 of the root length, the necks
of the 43, 33 teeth are exposed by 1/4. What ki-
nd of dental prosthesis should be applied in
Крок 2 Stomatology (англомовний варiант, iноземнi студенти) 2015 рiк 24

A. X-ray radiography of mandibula in frontal is destroyed with cariosity. X-ray image of


and lateral projections the paranasal sinuses shows the left one to
B. X-ray radiography of cranium in axillary have veil-like shading with clear dome-shaped
projection margin. X-ray image of the crown of the 25th
C. X-ray radiography of mandibula in frontal tooth shows absence of the periodontal fissure
and Parma projection at the the palatal root apex. What is the most
D. Tomography of mandibula probable diagnosis?
E. -
A. Radicular cyst that invaded in the maxillary
196. A 5-year-old shild has sustained a trauma. sinus
Objectively: the crowns of the 51st and 61st B. Chronic rhinogenous maxillary sinusitis
teeth are shorter then their adjacent ones. The C. Chronic odontogenic maxillary sinusitis
mucosa surrounding the 51st and 61st teeth is D. Maxillary sinus mucosal cyst
hyperemic and swollen. X-ray imaging shows E. Maxillary cancer
the periodontal fissure to be absent in the api-
cal area of the 51st and 61st roots, 51st and 199. A 26 year-old female patient suffers from
61st roots apexes are submerged in the spongy bleeding gums. For 3 years she has been
bone of the body of the lower jaw. Choose the suffering from pancreatic diabetes. Objecti-
optimal treatment. vely: gingival papillas of all her teeth are
hyperemic, swollen and overlap with the teeth
A. Extraction of the 51st and 61st teeth crowns by 1/3 of the crowns height. What
B. Splinting of teeth method of examination must be conducted for
C. Regular medical check-up differentiated diagnostics?
D. Anti-inflammatory therapy
E. Resection of the 51st and 61st teeth and their A. X-ray radiography
splinting B. Schiller-Pysarev iodine test
C. Capillary fragility test
197. A 10-year-old child complains of pain and D. Reoparodontography (Impedance
swelling in the left submandibular region; eati- phlebography of periodontium)
ng causes pain. Objectively: the mouth can be E. Stomatoscopy
fully opened, the mucosa of the torus mandi-
bularis is hyperemic, excretory duct produces 200. Parents of a 3-year-old child took the child
pus. X-ray imaging reveals an oval shade in to a pedodontist for oral sanitation. The child
the left submandibular area. What is the most has cerebral palsy. Objectively: there is multi-
probable diagnosis? ple dental caries, gingival mucosa is hyperemic,
oral hygiene is inadequate. What method of
A. Calculous sialadenitis of submandibular dental health care may be appropriate in this
salivary gland case?
B. Mandibular osteoma
C. Foreign body A. Under general anesthesia
D. Chronic lymphadenitis B. Under local anesthesia injection
E. Chronic sialadenitis C. Under local application anesthesia
D. Without anesthesia
198. A patient complains of the alveolar E. Giving sedatives
process deformation of the left upper jaw.
Objectively: the crown of the 25th tooth
MINISTRY OF PUBLIC HEALTH OF UKRAINE

Department of human resources policy, education and science

Testing Board

Student ID Last name

Variant ________________

Test items for licensing examination

Krok 2
STOMATOLOGY
General Instruction
Every one of these numbered questions or unfinished statements in
this chapter corresponds to answers or statements endings. Choose the
answer (finished statements) that fits best and fill in the circle with the
corresponding Latin letter on the answer sheet.

ББК 54.1я73
УДК 61

Authors of items: Amosova L.I., Andrianova I.I., Avdieiev O.V., Babenko L.M.,
Babiychuk N.F., Bas O.A., Bedeniuk O.A., Bielikov O.B, Bondalietov V.O., Bondarenko V.S.,
Bosa L.F., Chaikivsky R.V., Cherepynska Yu.A., Chernov D.V., Chukhrai N.L.,
Chumachenko V.A., Chyhrynets V.N., Chyzhevsky I.V., Demkovych A.Ye., Derebaliuk L.Ya.,
Derkach L.Z., Devdera O.I., Dmytrenko R.R., Dmytriieva A.A., Dubrovina O.V., Duda K.M.,
Dzetsiukh T.I., Fastovets O.O., Havaleshko V.P., Halahdyna A.A., Han I.V., Hlushak A.A.,
Hodovanets O.I., Holik V.P., Holovko N.V., Holubieva I.M., Hrechko N.B., Hrekuliak V.V.,
Idashkina N.H., Isakova N.M., Ivchenko N.A., Karelina L.S., Kaskova L.F., Kharkov L.V.,
Khomych N.M., Kindii V.D., Konovalov M.F., Kosarieva L.I., Koval S.M., Kovalchuk L.P.,
Kril A.Y., Krupnyk N.M., Kryklias H.H., Kudinov V.O., Kuz V.S., Kuz H.M., Kuzniak N.B.,
Kyryliuk M.I., Larionov I.M., Lavrovska O.M., Levko V.P., Lokes K.P., Lomeks O.I.,
Lysenko Yu.H., Makarevych A.Yu., Mazurina I.O., Mikhalova A.O., Moiseitseva L.O.,
Muntian O.V., Muzychina H.A., Neiko N.V., Nesyn O.F., Novytska I.K., Odzhubeiska O.D.,
Oleniychuk V.V., Oliynyk O.V., Orlovsky V.O., Ovcharenko E.N., Ozhohan Z.R., Pankevych A.I.,
Pantus A.V., Parasochkina V.V., Pasko O.O., Pavelko N.M., Petrushanko T.O., Piuryk V.P.,
Plekhova M.M., Plotnikova V.H., Potiyko V.I., Prodanchuk A.I., Prots H.B., Pushkova T.M.,
Rekova L.P., Rimsha O.Ye., Riznyk S.S., Romanenko O.H., Romanovska A.P., Rubizova A.H.,
Ruda I.V., Ruzin H.P., Samsonov O.V., Semenenko I.P., Senchenko O.M., Shakhnovsky I.V.,
Sharan M.O., Shcherbyna I.M., Shubladze H.K., Shuklin V.A., Shutak O.V., Shuvalov S.M.,
Skakun L.M., Smahliuk L.V., Spichka I.A., Stakhanska O.O., Steblianko L.V., Svirchkov V.N.,
Sydorova A.I., Tyshchenko V.I., Tsentilo V.H., Udod O.O., Usenko S.A., Valchuk O.H.,
Vasylenko V.M., Voropaieva L.V., Yatsenko I.V., Zablotsky Ya.V., Zaitsev L.O., Zavoiko L.M.,
Zelinska N.A., Zhero N.I.. and Committees of professional expertise.

Item reviewers. Bezvushko E.V., Bulbyk O.I., Chyzhevsky I.V., Dmytriieva A.A., Fastovets O.O.,
Gerelyuk V.I., Ilenko N.M., Kaskova L.F., Lungu V.I., Muntian L.M., Novikov V.M., Ostapko O.I.,
Smagliuk L.V., Solovey S.I., Tril S.I., Tsentylo V.G., Volynets V.M., Volyak M.N.

The book includes test items for use at licensing integrated examination “Krok 2. Stomatology” and
further use in teaching.
The book has been developed for students of stomatological faculties and academic staff of higher
medical educational establishments.

Approved by Ministry of Public Health of Ukraine as examination and teaching


publication based on expert conclusions (Orders of MPH of Ukraine of
14.08.1998 №251, of 27.12.1999 №303, of 16.10.2002 №374, of 29.05.2003 №233).

© Copyright Testing Board.


Krok 2 Stomatology (англомовний варiант, iноземнi студенти) 2016 рiк 1

1. A prosthodontist plans to make a causes transient pain. What is the most li-
porcelain-fused-to-metal crown for the 23 kely diagnosis?
tooth. In order to correct its abnormal posi-
tion in the dental arch the prosthododntist A. Acute deep caries
is going to use a cast post and core. What is B. Acute median caries
the maximum permissible value (in degrees) C. Chronic deep caries
of the core deviation from the tooth axis? D. Chronic median caries
E. Chronic fibrous pulpitis
A. 15
B. 10 6. A 42-year-old patient complains of pain in
C. 6-8 the submaxillary and sublingual areas that
D. 4 aggravates during eating, body temperature
E. 20-25 rise up to 37, 6o C . He has been suffering
from this for 2 months. Objectively: infi-
2. What impression material is used to ltration along the right sublingual torus,
obtain impressions for making orthodontic hyperemia, soft tissue edema, acute pain
appliances? during palpation. The duct of the right
submandubular salivary gland excretes
A. Ypeen turbid saliva mixed with pus. What is the
B. Stomaflex most likely diagnosis?
C. Repin
D. Sielast A. Exacerbation of salivolithiasis
E. Orthocor B. Acute purulent lymphadenitis of submaxi-
llary area
3. A 45-year-old woman consults a denti- C. Adenophlegmon of submaxillary area
st for prosthetics. Objectively: the 47, 46, D. Abscess of maxillolingual groove
45, 35, 36, 37 teeth are missing. There is E. Retention cyst of sublingual salivary gland
dentoalveolar vertical displacement of the
17, 16, 26, 27 teeth; the alveolar process is 7. A patient consulted a dental surgeon
hypertrophied, the cervical margins are not about fever up to 37, 6o C , soft tissue swelli-
exposed. When the teeth are closed, the di- ng, pain in the 47 tooth on the lower jaw on
stance between the occlusal surfaces of the the left, the pain aggravates when the tooth
teeth and alveolar process is approximately is touched by its antagonist. Objectively:
6 mm. Specify the most efficient method of mucogingival fold of the alveolar process
treatment of secondary deformation in this is hyperemic and flat on the vestibular side
patient: of the decayed 47 tooth. What is the most
likely diagnosis?
A. Pulp removal and teeth grinding
B. Surgical treatment A. Acute purulent odontogenic periostitis
C. Grinding B. Acute serous periodontitis
D. Hardware-associated surgical treatment C. Exacerbation of chronic periodontitis
E. Disocclusion D. Acute odontogenic osteomyelitis
E. Periodontal abscess
4. A 35-year-old patient has made an appoi-
ntment with a doctor to have oral cavity 8. A 12-year-old boy complains of a cavity
sanation. Objectively: the vestibular surface in the tooth on the lower left jaw. Objecti-
of the 22 tooth has a hard tissue defect vely: 1/3 of the 36 tooth is destroyed, the
localized in the mantle dentin; the dentin is carious cavity opens into the dental cavity;
hard, dark-coloured, the floor of the cavity reaction to cold stimulus is positive; probing
is coarse. Cold stimulus and probing are pai- and percussion are painless. X-ray imagi-
nless. What is the most probable diagnosis? ng shows the periodontal fissure of the 36th
tooth roots to be widened. What is the provi-
A. Chronic median caries sional diagnosis?
B. Enamel erosion
C. Cuneiform defect A. Chronic fibrous periodontitis
D. Dental fluorosis, destructive form B. Chronic simple pulpitis
E. Hard tissue necrosis C. Chronic gangrenous pulpitis
D. Chronic granulating periodontitis
5. A 25-year-old female patient consulted E. Chronic granulomatous periodontitis
a dentist about acute pain in the mandi-
ble on the right that occurs during eating. 9. A 5-year-old child has been diagnosed wi-
Objectively: the approximate distal surface th chronic granulating periodontitis of the
of the 45 tooth exhibits a carious cavity filled 85 tooth. What is the optimal material for
with light softened dentin. Probing causes the root canal filling?
a minor pain response across the entire
floor. Percussion causes no pain. Cold water
Krok 2 Stomatology (англомовний варiант, iноземнi студенти) 2016 рiк 2

A. Zinc-eugenol cement A. Methylene blue + Pimafucin (Natamycin)


B. Zinc-phosphate cement B. Hydrocortisone ointment + Solcoseryl
C. Glass ionomer cement C. Decaminum (Dequalinium chloride) +
D. Calcium-containing paste Hydrocortisone
E. Resorcinol-formalin paste D. Furacilinum (Nitrofural) + Methylene
blue
10. A patient complaining of constant dull E. Furacilinum (Nitrofural) + Solcoseryl
pain in the 38 tooth has made an appoi-
ntment with a dentist. Mouth opening is 14. A 46-year-old patient complains of
restricted to 1 cm. The patient is prescri- spontaneous pain in the 36 tooth. Its crown
bed extraction of the 38 tooth. What kind of is decayed. The patient presented with
anesthesia should be administered? edema of soft tissues adjacent to the mandi-
ble. Body temperature rose up to 39, 0o C ,
A. Веrcher-Dubov anaesthesia the patient has chill. Examination of the
B. Mandibular oral cavity revealed a pronounced edema
C. Torusal of the left mandibular alveolar process on
D. Extraoral administration of mandibular
anaesthesia the vestibular and oral side (acute periosti-
E. Tuberal tis); the 35, 36, 37 teeth are mobile, subgi-
ngival pockets contain purulent exudate;
11. A 67-year-old patient consulted an their percussion causes acute pain. Positi-
orthodontist about missing of the 34, 35, ve Vincent’s symptom is present. What is
36, 37, 45, 46 teeth. Orthopantomogram the presumptive diagnosis?
of the remaining teeth shows the alveolar
bone resorption in the range of 1/3 of the A. Acute odontogenic osteomyelitis of
interdental septa height. What construction mandible from the 36 tooth
should be offered to the patient? B. Exacerbation of chronic periodontitis
from the 36 tooth
A. Clasp prosthesis with splinting elements C. Acute purulent periostitis of mandible
B. Bridges on the posterior teeth from the 36 tooth
C. Removable partial lamellar denture D. Exacerbation of generalized periodontitis
D. Cap splint E. Exacerbation of chronic mandibular
E. Van Thiel splint osteomyelitis
12. A 22-year-old student complains of gi- 15. An 18-year-old student needs prosthetic
ngival hemorrhage during tooth brushi- porcelain-fused-to-metal denture for the
ng. Objectively: hyperemia and edema of 11, 21 teeth. There are no contraindicati-
marginal gingiva of the front teeth on both ons for the use of such construction. What
jaws, periodontal pockets are absent. X-ray is the most appropriate material for taking
reveals no pathological changes. What is the impressions?
most likely diagnosis?
A. Sielast
A. Chronic catarrhal gingivitis B. Stomalgin
B. Localized periodontitis C. Orthocor
C. Ulcerative gingivitis D. Stens
D. Hypertrophic gingivitis E. Repin
E. Generalized periodontitis
16. A 20-year-old patient addressed a dentist
13. A 32-year-old male patient complains for treatment of the destroyed teeth. Exami-
of dryness and burning pain in the back of nation revealed round painless papulae wi-
the tongue. The symptoms have been lasting th hyperemic infiltrated crown 7 mm in di-
for 1 week. The pain increases when spicy ameter on the palatine arches, soft palate,
food is taken. A day prior to that the patient tonsils, tongue. Regional lymph nodes are
had suffered from pneumonia and had been enlarged, dense, elastic, painless and mobi-
taking antibiotics. Objectively: the skin and le. What tactics should a doctor choose?
mucosa of oral cavity are pale. The tongue
mucosa is hyperemic and swollen; there A. Refer for a consultation to a venereologist
is crumbling grey-and-white coating in the B. Perform bacteriological analysis
folds of the back of the tongue; the lateral C. Refer for a consultation to an ENT doctor
surfaces of the tongue are desquamated. D. Perform oral cavity sanation
The saliva is thick and drags in threads after E. Perform cytological analysis
a spatula. Choose the most effective drug 17. A week ago an 18-year-old girl complai-
complex for topical treatment: ned of pain in the 22 tooth which was treated
and filled several years ago. Over the past
two days the pain increased. Objectively:
the 22 tooth is filled, percussion is pai-
Krok 2 Stomatology (англомовний варiант, iноземнi студенти) 2016 рiк 3

nful, mucous membrane is hyperaemic and to 1 cm large. What prosthesis is indicated?


edematic. Spot-film roentgenograph of the
22 tooth shows an ill-defined periapical bone A. Oxman’s bridge-like prosthesis with pivot
rarefaction 0,4x0,5 cm large. What is the point
most likely diagnosis? B. Clasp denture
C. Lamellar prosthesis with Gavrilow’s pivot
A. Exacerbation of chronic periodontitis of point
the 22 tooth D. Lamellar prosthesis with Oxman’s pivot
B. Suppuration of the radicular cyst point
C. Acute odontogenic osteomyelitis E. Bridge-like prosthesis without a pivot
D. Acute purulent periodontitis of the 22 point
tooth
E. Acute maxillary periostitis 21. A 32-year-old patient has made an appoi-
ntment with a dental surgeon to have oral
18. A 22-year-old patient complains of a pai- cavity sanation performed prior to denture
nful swelling in the right parotid gland. A installation. During examination the doctor
week earlier the patient received a cheek stated the following: the crown of the 25
abrasion that healed under the purulent tooth is destroyed. The tooth root is stable,
crust. Over the past two days the patient and its percussion is painless. Mucosa of the
had observed progressing pain and fever up alveolar process is unchanged. X-ray reveals
to 38, 6o C . Objectively: there is a soft tissue slight widening of the periodontal fissure.
edema in the right parotid region, the skin What provisional diagnosis can be made?
is slightly strained but has not changed in
colour. There is a dense painful infiltrati- A. Chronic fibrous periodontitis of the 25th
on 2,5x3,5 cm large, the skin over it exhibi- tooth
ts limited mobility. The mouth can be fully B. Chronic periodontitis of the 25th tooth
opened, the mucous membrane around the C. Chronic granulomatous periodontitis of
orifice of the salivary duct is unchanged, the the 25th tooth
saliva is transparent. What is the most likely D. Chronic granulating periodontitis of the
diagnosis? 25th tooth
E. Cystogranuloma
A. Acute lymphadenitis
B. Exacerbation of chronic parotitis 22. A 45-year-old man complains about li-
C. Abscess of the parotid-masseteric region quid outpouring from his nose, inability to
D. Acute non-epidemic parotitis blow his nose, inflated cheeks. Objectively:
E. Epidemic parotitis there is a perforating defect (1х1,5 cm) of
alveolar process at a level of the extracted
19. A 3-year-old girl complains of pain and 26th tooth in the lateral part of his upper
swelling in the region of the decayed 51, jaw. Air inhalation through the nose with
52 teeth, body temperature rise up to 37,5- held nostrils is accompanied by generati-
37,9o C . Objectively: the face is asymmetric on of bubbles in the area of perforati-
because of a swelling in the upper lip region on. What denture construction should be
and right infraorbital region. The crown of recommended?
the 51 tooth is completely decayed. Mucous
membrane in the region of the 52, 51, 1 teeth A. Minor saddle denture with clasp fixation
is edematic, mucogingival fold is smoothed, B. Clasp denture with obturating part
palpation provokes pain, mobility of I-II C. Common partial removable denture
grade of the 51, 52 teeth is also present. D. Common dental bridge
What is the most likely diagnosis? E. Protective palatal bars
A. Acute purulent odontogenic maxillary 23. A 25-year-old patient presents with
periostitis fluorosis of vestibular surfaces of the upper
B. Acute albuminous odontogenic maxillary incisors. Which of the following constructi-
periostitis ons will provide the maximum aesthetic
C. Acute odontogenic maxillary osteomyeli- result?
tis
D. Odontogenic abscess of infraorbital region A. Veneers
E. Exacerbation of chronic periodontitis of B. Plastic crown
the 51 tooth C. Ceramic crown
D. Combined Kurylenko crown
20. A 50-year-old patient has median lower E. Metal-plastic crown
jaw fracture with formation of a false joint.
The 38, 32, 31, 41, 42, 48 teeth are missing. 24. A 6-year-old child complains of pain
The remaining teeth are intact, stable. There in the mandibular tooth on the left during
is no displacement of lower jaw fragments. eating. Objectively: masticatory surface of
X-ray picture shows a bone tissue defect up the 36 tooth exhibits a carious cavity within
Krok 2 Stomatology (англомовний варiант, iноземнi студенти) 2016 рiк 4

the mantle dentin. The cavity is full of light A. Adenocarcinoma


softened dentin which can be easily removed B. Mixed tumour
with an excavator. Probing of the cavity walls C. Cyst of the gland
causes pain response. The teeth are sensitive D. Glandular tuberculosis
to thermal stimuli, the pain is of short-term E. Chronic lymphadenitis
nature. There is no response to percussion.
Select the optimal filling material: 28. When a prosthodontist was preparing
the patient’s tooth, the patient had epileptic
A. Silver amalgam seizure that was subsequently terminated.
B. Zinc phosphate cement What mistake had been made by the doctor?
C. Silicate cement
D. Silicophosphate cement A. No inquire into the patient anamnesis
E. Polycarboxylate cement B. No inquire into the antecedent anamnesis
C. No anesthesia
25. A 48-year-old patient complains of D. Crude preparation
subfebrile temperature and a growing ulcer E. Did not decline the appointment
on the gingival mucosa around the molars;
teeth mobility in the affected area, cough. 29. A 13-year-old child complains of having
Objectively: gingival mucosa in the region a cavity in the front maxillary teeth. Contact
of the lower left molars has two superfici- medial surfaces of the 11 and 21 teeth
al extremely painful ulcers with undermined exhibit cavities found within the mantle
edges. The ulcers floor is yellowish, granular, dentin and filled with dense pigmented
covered with yellowish and isolated pink dentin. Probing of the cavity floor causes no
granulations. The ulcers are surrounded by pain response, neither does dental percussi-
the tubercles. Dental cervices are exposed, on. Select the best filling material for the
there is a pathological tooth mobility. Regi- permanent seals:
onal lymph nodes are enlarged and make A. Resin composite
dense matted together groups. What is the B. Silicate cement
most likely diagnosis? C. Silicophosphate cement
A. Tuberculosis D. Zinc phosphate cement
B. Syphilis E. Glass ionomer cement
C. Acute aphthous stomatitis 30. A 30-year-old patient complains of
D. Infectious mononucleosis toothache caused by hot and cold stimuli.
E. Decubital ulcer The pain irradiates to the ear and temple.
26. A 78-year-old patient is completely Previously there was spontaneous nocturnal
edentulous. He has been wearing dentures toothache. Objectively: on the occlusal
for 19 years. The patient complains of poor surface of the 37 tooth there is a deep cari-
fixation of the upper denture. Objectively: ous cavity communicating at one point with
the lower third of the face is shortened, the the tooth cavity. Probing at the communi-
alveolar processes of both jaws are markedly cation point, as well as cold stimulus, causes
atrophied, the palate is flat. Mucous acute pain. The pain persists for a long time.
membrane in the denture-supporting area Electric pulp test result is 55 microamperes.
is atrophied. How often should the dentures What is the most likely diagnosis?
be remodelled or restored? A. Exacerbation of chronic pulpitis
A. Every 3-4 years B. Acute diffuse pulpitis
B. Every 6 months C. Exacerbation of chronic periodontitis
C. Once a year D. Chronic concrementous pulpitis
D. Every 7 years E. Acute purulent pulpitis
E. Every 10-12 years 31. An 11-year-old boy complains of a cari-
27. A 56-year-old man complains of pain ous cavity in the mandibular tooth on the
in the left parotidomasticatory region, right. Objectively: the 46 tooth exhibits a
progressing face asymmetry that was noticed carious cavity within the mantle dentin. The
a month ago. Objectively: left-sided paresis dentin is dense, pigmented; there is pain
of mimic muscles. To the fore of earflap response to the cold stimulus; probing and
there is an ill-defined infiltration, the skin percussion cause no pain response. Make
above it is tense and cyanotic; left lymph the provisional diagnosis:
nodes are enlarged. Opening of mouth is li- A. Chronic median caries
mited down to 2,5 cm. The left parotid duct B. Acute median caries
doesn’t excrete saliva. What is the most li- C. Acute deep caries
kely diagnosis? D. Chronic deep caries
E. Chronic superficial caries
Krok 2 Stomatology (англомовний варiант, iноземнi студенти) 2016 рiк 5

32. During preventive examination an 11- bular tooth. Objectively: the medial surface
year-old girl presents with slight hyperemy, of the 36 tooth exhibits a carious cavity wi-
cyanosis, swollen gums, gingival papilla thin parapulpar dentin. The cavity is filled
are loose, not adjacent to teeth cervices. with light, softened dentin and does not
Fedorov-Volodkina dental hygiene index is communicate with the cavity of the tooth.
4 points. Schiller’s test is positive. Make the Probing the of the 36 tooth floor causes
diagnosis: pain response, the tooth is not sensitive to
percussion, the response to the cold stimulus
A. Chronic catarrhal gingivitis does not remain long after its removal. What
B. Hypertrophic gingivitis is the most likely diagnosis?
C. Acute catarrhal gingivitis
D. Necrotizing ulcerative gingivitis A. Acute deep caries
E. - B. Acute focal pulpitis
C. Acute diffuse pulpitis
33. A 22-year-old medical student complai- D. Chronic fibrous pulpitis
ns of the changed colour of his central upper E. Acute median caries
incisor on the right. Two years ago the tooth
became gray. Objectively: the 2 tooth is fi- 37. A 46-year-old patient after the recovery
lled, changed in colour, stable, percussion is from a case of influenza has suddenly started
painless. The patient has closed bite. What suffering from pain in the 36 tooth, the
is the absolute contraindication to making crown of which had been destroyed. The
porcelain crowns for this patient? soft tissues surrounding the lower jaw are
swollen. There are high fever up to 39o C and
A. Closed bite chills. Objectively: there is prominent swelli-
B. Defects of the anterior teeth that cannot ng of the left lower jaw alveolar mucosa
be restored with fillings (both oral and vestibular surfaces). The 34,
C. Enamel hypoplasia with a change of tooth 35 and 37 intact teeth are mobile. Percussi-
shape and colour on is painful. The gingival pockets produce
D. Abnormal tooth colour purulent exudate. Vincent’s symptom is
E. Defects of pulpless teeth that cannot be observed. What diagnosis can be suspected?
restored with inlays
A. Acute odontogenic mandibular
34. A 25-year-old patient complains of pain osteomyelitis starting from the 36th tooth
caused by eating sweet, hot and cold food; B. Exacerbation of chronic periodontitis of
pain ceases, when stimulation stops. Objecti- the 36th tooth
vely: the adjoining surface of the 36th tooth C. Acute suppurative mandibular periostitis
has a deep carious cavity localised in the ci- starting from the 36th tooth
rcumpulpar dentin. The dentin is softened. D. Periodontitis
Probing of the carious cavity floor is painful. E. Exacerbation of chronic mandibular
What is the most probable diagnosis? osteomyelitis
A. Acute deep caries 38. A doctor treating an AIDS-positive pati-
B. Acute median caries ent has accidentally pierced his own skin of
C. Chronic fibrous pulpitis the palm with a root needle. What should be
D. Chronic median caries the first course of actions?
E. Chronic deep caries
A. Squeeze out blood, process the breach
35. A 27-year-old patient complains of the with 70% alcohol solution
long-term pain in the 22 tooth caused by B. Squeeze out blood, process the
cold and hot food, as well as of spontaneous breach with strong solution of potassium
pain lasting for 30 minutes and occurring 3- permanganate
4 times per day, aggravating at night. Pain C. Apply tourniquet to the shoulder
arose 3 days ago after preparation of the D. Process the breach with 5% iodine soluti-
tooth for the acrylic crown. Objectively: the on
22 tooth is intact, the thermal test causes E. Process the breach with 3% hydrogen
acute long-lasting pain, percussion is pai- peroxide solution
nless. What is the optimal treatment tactics?
39. A 57-year-old patient complains of tooth
A. Vital pulp extirpation mobility, inability to eat. Objectively: the
B. Application of fluorine lacquer lower 35, 36, 37, 38, 44, 45, 46 and 48 teeth
C. Biological treatment of pulpitis are missing; the 31, 32, 33, 34, 41, 42, 43,
D. Devital pulp extirpation 47 teeth exhibit II grade mobility, their cli-
E. Vital pulp amputation nical crowns are low, tooth equator is not
pronounced. What is the optimal denture
36. An 11-year-old boy complains of a short- construction in this case?
term pain from the cold in the left mandi-
Krok 2 Stomatology (англомовний варiант, iноземнi студенти) 2016 рiк 6

A. Removable cast splint A. Phonetic tests


B. Removable partial denture B. Graphic
C. Kurlyandsky splint bar C. Myogymnastics
D. Removable Bynin splint D. Spectrographic
E. Removable splint with vestibulo-oral clasp E. Acoustic
40. A 7-year-old boy complains of increased 44. A 7,5-year-old boy fell down on his
body temperature up to 38o C , headache, face and damaged his front maxillary teeth.
sore throat. Objectively: there are erosions Objectively: the crowns of the 11 and 12
on the slightly hyperemic mucosa of the soft teeth are destroyed by 1/4. The tooth cavi-
palate, anterior pillars of the fauces, tonsils. ty is closed. Probing causes pain along the
The submandibular lymph nodes are slightly break-off line, percussion of the 11 and
enlarged, painless. Name the causative agent 12 teeth causes no pain response. Mucosa
of this disease: around the 11 and 12 teeth is slightly
hyperemic. Radiograph of the 11 and 12
A. Coxsackie virus teeth shows intactness of the dental tissues
B. Herpes simplex virus and alveolar process. What tactics of a denti-
C. Epstein-Barr virus st would be optimal in the first hours after
D. Klebs-Loeffler bacillus the injury?
E. Bordet-Gengou bacillus
A. Pulp monitoring
41. A patient complains of dull ache in the B. Fabrication of an artificial crown
16 tooth, which occurs during eating cold C. Photopolymer filling
food. Anamnesis: the tooth had been fi- D. Parapulpar pin reconstruction
lled due to deep caries, the filling was lost E. Composite filling
1 year ago. Objectively: a deep carious cavi-
ty that does not penetrate the tooth cavity 45. A 56-year-old man complains of pain
is present; percussion is painless, probing is in the left parotid-masticatory area and
painful along the whole floor of the carious progressing facial asymmetry first noticed
cavity. Electric pulp test - 50 microamperes. one month ago. The diagnosis is the cancer
Thermodiagnosis is painful. Make the di- of parotid gland - T2N2. What method of
agnosis: treatment is optimal in the given case?
A. Chronic fibrous pulpitis A. Combined method
B. Chronic deep caries B. Surgical extraction of neoplasm
C. Chronic fibrous periodontitis C. Radiation therapy
D. Acute deep caries D. Chemotherapy
E. Chronic gangrenous pulpitis E. Lymph nodes removal
42. A patient complains of acute short-term 46. A 20-year-old male patient complains
toothache caused by thermal and chemical of acute pain during eating, bad breath,
irritants. The tooth has not been treated general weakness. Objectively: interdental
before, the pain arose about a month ago. papillae and marginal gingiva are covered
Objectively: on the occlusal surface of the with dirty gray film, any touch is painful. The
36 tooth there is a deep carious cavity wi- body temperature is of 37, 5o C . Blood count:
thin circumpulpal dentin with overhanging RBC- 4, 8 · 1012 /l, Hb- 150 g/l, colour index
chalky enamel. Probing of the cavity floor - 0,9, WBC- 10, 5 · 109 /l, JG- 2%, BASO-
causes slight pain. There is also short-term 1%, EOS- 5%, segmented neut. - 8%, stabs
pain from the cold stimulus. The result of - 47%, LYM- 35%, PLT- 250 · 109 /l. ESR- 20
electric pulp test is 8 microamperes. What is mm/hour. What is the most likely diagnosis?
the most likely diagnosis?
A. Vincent’s gingivitis
A. Acute deep caries B. Agranulocytosis
B. Acute median caries C. Acute leukemia
C. Pulp hyperemia D. Acute herpetic stomatitis
D. Chronic deep caries E. Chronic leukemia
E. Chronic fibrous pulpitis
47. A 68-year-old patient, having suffered
43. A 65-year-old patient needs complete from stroke, suffers from periodical bouts of
removable dentures for both jaws. At the retching, which lead to unilateral dislocati-
stage of ”testing the denture constructi- on of the temporormandibular joint. Setting
on” the doctor checks the pronunciation the joint was possible only with anesthesia.
of sounds ”S” and ”Z”. Which method of Objectively the following teeth are absent:
normalization of speech functions should be from the 21 to the 28, 15, 16, 17, 18, 36, 37, 44,
applied in this case? 46, 47. What kind of denture construction wi-
ll be optimal for the duration of treatment?
Krok 2 Stomatology (англомовний варiант, iноземнi студенти) 2016 рiк 7

A. Mamlok splint
A. Removable laminar denture with mouth B. Ring adhesion splint
opening restrictor C. Soldered combined crowns
B. Schroder appliance with sliding joint D. Cap splint
C. Petrosov appliance with restrictor E. Brace
D. Yadrova appliance
E. Khodorovych-Burgonska appliance with 52. A 42-year-old patient has been hospitali-
restrictor zed with Le Fort II fracture of maxilla. Select
an appliance for the treatment of this pati-
48. A 9-year-old child complains of pain ent:
in the left mandibular molar that occurs
during eating. The masticatory surface of A. Zbarzh
the 75 tooth exhibits a carious cavitiy filled B. Kulagin
with softened dentin and localized within C. Rudko
parapulpar dentin. The cavity communi- D. Penn-Brown
cates with the tooth cavity. Probing at the E. Yadrova
site of communication causes acute pain
and moderate bleeding. Percussion of the 53. A 25-year-old patient complains of brief
tooth causes no pain response. Radiography pain attacks during eating sweet, hot or cold
revaled no periodontal alterations in the 75 food. Objectively: on the contact surface of
tooth. Select the method of treatment: the 36 tooth there is a carious cavity within
parapulpar dentin. The dentin is softened.
A. Devital amputation Probing of the cavity is painful. Vertical and
B. Devital extirpation horizontal percussion is painless. Electric
C. Vital extirpation pulp test - 9 microamperes. What diagnosis
D. Vital amputation is most likely?
E. Biological method
A. Acute deep caries
49. A 40-year-old patient complains of fever B. Acute median caries
up to 38o C , and a roundish infiltration on C. Chronic fibrous pulpitis
the upper lip. Objectively: the upper lip D. Chronic deep caries
on the left exhibits a round infiltrate, the E. Chronic median caries
overlying skin is deep crimson. The infiltrate
adheres to the surrounding tissues and has a 54. A 39-year-old patient suffers from mi-
necrotic core in the center. The upper lip is crostomia. What impression tray should be
hyperemic, edematous. What diagnosis can used?
be made? A. Demountable
A. Upper lip furuncle B. Standard
B. Acute abscess of the upper lip C. Standard for edentulous jaw
C. Retention cyst D. Custom plastic
D. Acute lymphadenitis E. Collapsible
E. Upper lip carbuncle 55. In a prostodontic clinic a partial lami-
50. A 25-year-old patient received a trauma nar denture for the upper jaw is being made
to the chin region. On the basis of X-ray he for a 53-year-old patient. Objectively: dental
was diagnosed with bilateral mental fracture formula is 14, 13, 12, 11, 21, 22, 23, 24, 27.
of mandible. Specify the direction of di- The teeth are firm, clinical crowns are tall
splacement of the minor fragment: with pronounced equator. X-ray shows no
periapical changes in the periodontium of
A. Downward and backward the abutment teeth. What clammer fixation
B. Upward and forward is optimal for this patient?
C. Upward and backward
D. Downward and forward A. Planar
E. There is no displacement B. Sagittal
C. Diagonal
51. A 38-year-old patient with chronic D. Transversal
generalized periodontitis has been referred E. Point
to orthopedic treatment. Objectively: denti-
tions are without gaps, the 12, 11, 21, 22 56. An intradental splint for the 33, 32, 31,
teeth are pulpless and exhibit I grade mobi- 41, 42, 43 teeth is being made for a 50-
lity. The other teeth are stable. What is the year-old patient. During its installation in
most aesthetic splint for the anterior teeth? the oral cavity hemorrhage occurred from
the 41 tooth root canal. What caused the
hemorrhage?
Krok 2 Stomatology (англомовний варiант, iноземнi студенти) 2016 рiк 8

A. Wall perforation of the root canal


B. Pulp trauma A. 18
C. Trauma of the gingival margin B. 16
D. Damaged root bifurcation C. 14
E. Crown perforation D. 12
E. 10
57. A 34-year-old male patient complains of
acute spasmodic pain in the region of his 61. A 47-year-old patient complains of a
upper jaw on the left that is aggravating burning sensation and pain in the mouth.
when affected by cold stimuli. Toothache Objectively: on the mucous membrane of
irradiates to the ear and temple. He had cheeks along the line of teeth contact and
acute toothache of the 37 tooth one year in the corners of mouth there are multi-
ago, but he did not consult a dentist. Pain ple polygonal bright red erosions 1,0-1,5
recurred three days ago. Objectively: the 37 cm in diameter located on the hyperkerati-
tooth has a carious cavity communicating nized plaque and opaque whitish mucosa.
with the dental cavity. Probing of the opened Cytological analysis revealed keratinizing
carious cavity is extremely painful. X-ray pi- epithelial cells. What is the most likely di-
cture shows widening of periodontal fissure agnosis?
at the root apex of the 37 tooth. What is the
most likely diagnosis? A. Leukoplakia, erosive form
B. Lichen planus, erosive form
A. Exacerbation of chronic pulpitis C. Erythema multiforme
B. Exacerbation of chronic granulating D. Secondary syphilis
periodontitis E. Lupus erythematosus, erosive form
C. Exacerbation of chronic fibrous peri-
odontitis 62. A 27-year-old female patient consulted
D. Acute diffuse pulpitis a dentist about pain in the 35 tooth. The
E. Acute purulent pulpitis pain is caused by cold stimuli and qui-
ckly abates after the stimulus is removed.
58. A 79-year-old female patient consulted a Objectively: in the cervical region of the 35
prosthodontist about denture replacement. tooth there is a dental tissue defect within
The patient has a history of a stroke. the enamel. The defect has white crumbling
Objectively: acute irregular atrophy of the edges. Electroexcitability of the pulp was 5
alveolar processes of both jaws is present; mkA. What is the most likely diagnosis?
mucous membrane of the oral cavity is
dry and nonmobile. The previous dentures A. Acute superficial caries
cannot be fixed. What is the most appropri- B. Local enamel hypoplasia
ate prosthetic construction? C. Acute initial caries
D. Enamel necrosis
A. Dentures with elastic lining E. Enamel erosion
B. Dentures with extended borders
C. Dentures with metal bases 63. A 27-year-old female patient has her 26
D. Dentures with shortened borders tooth cavity accidentially breached along the
E. Implant-supported dentures mesial buccal line angle during treatment of
acute deep caries of the 26. Choose the opti-
59. A 56-year-old patient complains of mal method of treatment:
frequent breakage of the basis of the partial
laminar denture of the upper jaw. Objecti- A. Biological method
vely: only the 23 tooth is retained on the B. Non-vital extirpation
upper jaw. Cracks in the deture basis appear C. Non-vital amputation
in the area of the 23 tooth. What should be D. Vital amputation
done to reinforce the denture basis? E. Vital extirpation

A. Make a new denture with telescopic 64. A 55-year-old patient consulted a dentist
fixation on the 23rd tooth about a roundish tumour-like formation of
B. Make the basis out of Ethacryl-02 plastic about 1 cm in diameter located within the
C. Make the basis out of Phtorax plastic vermilion border of his lower lip. Objecti-
D. Make the basis out of Prothacryl-M plastic vely: the tumor-like formation protrudes
E. Make the basis out of Redont plastic about 5 mm above the vermilion border, it is
dense and grayish-red in color. The surface
60. A patronage nurse visited a newborn of the formation is covered with thin scales
baby. Examination revealed the shortened that can hardly be removed. What is the
lower part of the face, the backward-sloping most likely diagnosis?
chin, missing teeth, the retroposed lower
jaw. What is the number of dental follicles
in each jaw of a newborn baby?
Krok 2 Stomatology (англомовний варiант, iноземнi студенти) 2016 рiк 9

A. Verrucous precancer of the vermilion


border of lip A. Glossodynia
B. Abrasive precancerous Manganotti’s B. Candidal glossitis
cheilitis C. Glossitis areata exfoliativa
C. Precancerous limited hyperkeratosis of D. Hunter-Moeller’s glossitis
the vermilion border of lip E. Glossitis with fissured tongue
D. Bowen’s disease
E. Erythroplasia of Queyrat 69. A patient presented to a dental cli-
nic for complex dental care. Objectively:
65. A 45-year-old man complains of dryness the 37 tooth exhibits a deep carious cavi-
and painfulness of the lower lip. On exami- ty communicating with the tooth cavity.
nation: the lower lip is swollen, dry, covered There is no response to stimuli. Radiography
in small scales and fissures. In the Klein area reveals widening and deformation of the
(wet-dry line) there are dilated openings periodontal ligament space in the apical
of salivatory glands observed as red dots region. What is the most likely diagnosis?
producing clear substance. The lower lip
mucosa is lumpy. What is the most likely A. Chronic fibrous periodontitis
diagnosis? B. Chronic granulating periodontitis
C. Chronic granulomatous periodontitis
A. Cheilitis glandularis D. Chronic fibrous pulpitis
B. Actinic cheilitis E. Chronic gangrenous pulpitis
C. Meteorological cheilitis
D. Exematous cheilitis 70. An 18-year-old patient complaining of
E. Cheilitis exfoliativa large diastem has made an appointment
with a prosthodontics specialist. Objecti-
66. A 50-year-old patient has a defect of the vely: there is full lateral displacement of the
lower dental arch. It is planned to make an central incisors due to absence of the 12 and
implant-supported bridge for its restoration. 22 teeth. What instrument is the most advi-
X-ray picture shows that the height of the sable for moving the central incisors closer
bone mass from projection of mandibular together?
canal up to the top of alveolar crest is 2 cm.
What type of implant should be applied? A. Korkhaus appliance
B. Vasylenko appliance
A. Threaded C. Simple cotton ligature
B. Endodontic-endoosseous D. Kalvelis appliance
C. Plate-form E. Begg appliance
D. Subperiosteal
E. Conical 71. An injured patient complains of limi-
ted opening of the mouth, nose bleedi-
67. A 40-year-old man, a chemical industry ng, skin numbness in the infraorbital and
worker, notes the development of soreness lower eyelid region. Objectively: there is
of the mouth, pain response to thermal and face deformation due to the depression of
chemical stimuli. On examination: on the soft tissues in the left cheekbone region, step
vestibular surface and cutting edge of the deformity in the middle part of the inferior
front teeth there are enamel defects wi- margin of the left orbit and in the area of the
th uneven scalloped margins, chalk-like in zygomatic alveolar crest. What is the most li-
color. Make the diagnosis: kely diagnosis?
A. Acidic necrosis of enamel A. Zygomatic bone fracture with di-
B. Superficial caries splacement of bone fragments
C. Enamel hypoplasia (erosive form) B. Fracture of the right zygomatic bone
D. Fluorosis (erosive form) without displacement of fragments
E. Median caries C. Le Fort I fracture of maxilla
D. Le Fort II fracture of maxilla
68. A 57-year-old female patient complains E. Fracture of the malar arch
of burning pain, dry mouth, sensation of a
foreign body on the tongue back and lateral 72. A 60-year-old patient addressed a denti-
surfaces, which disappears in the process of st with complaints of a conic protrusion
eating. The first case of such symptoms was appearing on the skin of his chin on the
noted a year ago after psychological trauma. left. Height of the protrusion is 1,5 cm, wi-
The patient suffers from pancreatic di- dth - 0,8 cm. Palpation is painless; there are
abetes and sleep disturbance. Examination clear margins; the protrusion is gray-brown
revealed the following: the tongue mucosa in color; on palpation of the soft tissues
is without significant changes, moderately near the protrusion base a thickening can
moistened. What is the most probable di- be detected. What preliminary diagnosis can
agnosis? be made?
Krok 2 Stomatology (англомовний варiант, iноземнi студенти) 2016 рiк 10

the provisional diagnosis?


A. Cutaneous horn
B. Bowen’s disease A. Papillon-Lefevre syndrome
C. Papilloma B. Niemann-Pick disease
D. Keratoacanthoma C. Hand-Schuller-Christian disease
E. Melanoma D. Eosinophilic granuloma (Taratynov’s
disease)
73. A 30-year-old patient needs to have his E. Letterer-Siwe disease
26 tooth extracted because of exacerbati-
on of chronic periodontitis. Objectively: the 77. A 49-year-old patient has a tooth
crown of the 26 tooth is decayed by 1/3. removed from the left upper jaw under
What forceps can be used for this tooth plexus anesthesia with articaine-forte (Arti-
extraction? caine + Epinephrine). After the operation
the tooth socket did not fill with blood clot.
A. S-shaped forceps with a projecting tip on How to prevent alveolitis in this case?
the left beak
B. S-shaped forceps with a projecting tip on A. Loosely fill the socket with iodoform
the right beak tampon
C. Straight forceps B. Fill the socket with hemostatic sponge
D. Straight elevator C. Lavage the socket with microcide solution
E. S-shaped forceps without projecting tips D. Lavage the socket with 0,1% chlorhexidi-
ne solution
74. A patient complains of restricted mouth E. Fill the socket with antibiotic dusting
opening, pain during swallowing, fever up powder
to 38, 5o C , weakness, indisposition. Objecti-
vely: the mouth opens up to 1 cm. After 78. A 24-year-old woman consulted a denti-
Berchet anesthesia examination of the st about pain in the 26 tooth. After a physi-
oral cavity revealed edema, hyperemia, cal examination the patient was diagnosed
tenderness of the pterygomandibular fold. with exacerbation of chronic periodontitis
Lateral and posterior pharyngeal walls of the 26 tooth. During the tooth extracti-
are intact. The 38 tooth is semi-impacted. on the coronal part of the tooth was acci-
Palpation of the internal surface of the dentally broken. Further manipulations wi-
mandibular angle is painful. What is the th bayonet forceps failed to extract the tooth
most likely diagnosis? roots. What actions are to be taken for the
successful root extraction?
A. Phlegmon of the pterygomandibular
space A. To disjoin the roots by using a drill and
B. Acute tonsillitis fissure burr
C. Phlegmon of the infratemporal region B. To send the patient to the hospital
D. Acute pericoronitis of the 38 tooth region C. To use Lecluse elevator
E. Phlegmon of the peripharyngeal space D. To complete the tooth extraction during
the next visit
75. A 37-year-old patient complains of E. To use angular elevator
bleeding gums, bad breath, loose teeth, di-
fficulties with chewing food. Objectively: 79. Preventive examination of a 5-year-old
gums are bluish-red in colour; periodontal child revealed a habit of lower lip biting.
pockets are 6 mm deep and contain serous What malocclusion may develop if the child
substance; class II teeth mobility; moderate keeps this habit?
deposits of subgingival and supragingival
dental calculus; Oral Hygiene Index equals 3 A. Anterior bite
points; there is traumatical occlusuion along B. Prognathic bite
all the length of the dental arches. What is C. Open bite
the provisional diagnosis? D. Deep overbite
E. Cross-bite
A. Generalized periodontitis, stage II,
chronic 80. A 7-year-old child has protruding chin,
B. Generalized periodontitis, stage I, acute the lower lip overlaps the upper one. There
C. Hypertrophic gingivitis are diastemas and tremas between the lower
D. Histiocytosis X incisors, the lower incisors overlap the upper
E. Generalized periodontitis, stage II, acute incisors by 2/3 of the crown height. Fi-
rst permanent molars demonstrate Angle’s
76. A 3-year-old child suffers from extreme class III relation. Sagittal gap is 3 mm. The
gingivitis with slight teeth mobility; the teeth correct doctor’s tactics will be to:
are intact. The skin is dry, the hair is brittle,
the skin of the palms and soles of the feet is
rough and covered in small cracks. What is
Krok 2 Stomatology (англомовний варiант, iноземнi студенти) 2016 рiк 11

A. Use Bruckl’s appliance the duct. The duct orifice is dilated, the
B. Recommend a complex of myogymnastic surrounding mucosa is cyanotic, pastose, wi-
exercises th teeth imprints. What disease is it?
C. Use Angle’s apparatus
D. Use Bynin appliance A. Chronic parenchymatous sialoadenitis
E. Use Schwartz appliance B. Chronic interstitial sialoadenitis
C. Sialolithiasis
81. A 5-year-old child was found to have D. Mixed tumor
missing upper molars. Lower incisors are E. Cyst
in contact with the mucous membrane of
palate. Specify the doctor’s tactics: 85. A 39-year-old patient complains of
experiencing pain in the region of the 21
A. Fabricate a removable laminar denture tooth for 2 days. It is known from the history
B. Examine the child every six months until that the indicated tooth had been treated
the eruption of permanent teeth before for caries. Objectively: the 21 tooth is
C. Examine the child once a year until the covered with metal-ceramic crown, mucous
eruption of permanent teeth membrane in apex projection is edematic
D. Fabricate an orthodontic appliance for the and hyperemic. Percussion of the tooth
treatment of closed bite is extremely painful. X-ray picture shows
E. Medical intervention is not needed improperly filled root canal. It is planned to
remove the 21 tooth crown. What kind of
82. A 14-year-old girl complains of indisti- anesthesia should be administered?
nct pronunciation that showed up at the
age of 14 after the acute respiratory vi- A. Field block anaesthesia
ral disease. Examination revealed normal B. Infiltration anaesthesia
face and normal teeth alignment, occlusal C. Intraligamentous anaesthesia
disharmony was not found. Palpation did D. Application anaesthesia
not reveal cleft palate. Uvula does not move E. Intraosseous anaesthesia
during pronunciation of sounds, its palpati-
on does not cause gag reflex. What is the 86. At a prosthetics dentistry clinic a 35-
cause of indistinct pronunciation of sounds? year-old patient received a porcelain-fused-
to-metal crown for the 21 tooth. What is
A. Paresis of the soft palate and uvula the minimum warranty period for porcelain-
muscles fused-to-metal crowns under the current
B. Adenoid vegetations legislation?
C. Palatal slit
D. Hypertrophy of lingual tonsil A. 12 months
E. Deformation of the bite B. 24 months
C. 36 months
83. A 42-year-old woman has made D. 6 months
an appointment with a prosthodontic E. 3 months
office to make a denture. Objecti-
vely the dental formula is as follows: 87. During the preparation of a tooth a pati-
18 . . . . 13 12 11 21 22 23 . . . . 28 ent had an epileptic seizure. The seizure was
48 47 46 45 44 43 42 41 31 32 33 34 35 36 37 . arrested. What mistake did the orthopaedist
The patient has deep occlusion; clinical make?
crowns are low; equator is not pronounced.
The patient suffers from epileptic seizures. A. Did not collect complete history data
What kind of denture should be prescribed B. Did not apply one of the types of local
for this patient? anesthesia
C. Violated the rules of preparation
A. Partial removable laminar metal-based D. Skipped psychological preparation of the
denture patient
B. Dental bridge E. Did not apply general anaesthesia
C. Partial removable laminar plastic denture
with clasp-retainers 88. During the surgical removal of a retenti-
D. Partial removable laminar denture with on cyst of the lower lip a 14-year-old boy
supporting-retaining clasps complained of sudden weakness, dizziness,
E. Bugel dental prosthesis nausea. Objectively: the skin is covered with
cold sweat. Respiration is frequent, pulse is
84. A 15-year-old girl complaining of weak, BP is low (90/60 mm Hg), the hands
moderate swelling of the left parotic- are cold. What is the most likely diagnosis?
masticatory area is being examined by a
dental surgeon. Palpation revealed the left
parotid gland to be bulgy, dense, and pai-
nless. Turbid saliva is being secreted from
Krok 2 Stomatology (англомовний варiант, iноземнi студенти) 2016 рiк 12

A. Syncope 93. Objective examination of a 65-year-


B. Toxic shock old patient with the completely edentulous
C. Traumatic shock mandible revealed a marked uniform
D. Anaphylactic shock atrophy of the alveolar bone; bony promi-
E. Collapse nences on the lingual surface in the regi-
on where premolars had previously been.
89. A 56-year-old patient addressed a dental Mucosa was unevenly pliable; alveolar crest
surgeon for extraction of the 27 tooth. What was mobile in the frontal region. The clinical
anesthesia should be applied? condition of the mandible should be taken
A. Tuberal, palatal into consideration at the following stage of
B. Palatal, infraorbital denture fabrication:
C. Infiltration, incisive A. Taking a differentiated impression
D. Mandibular B. Taking an anatomical impression
E. Torusal C. Taking a positive pressure impression
90. A 28-year-old patient has been suffering D. Taking a decompression impression
from gum discomfort, gingival hemorrhages E. Measuring the centric relation of jaws
and pain, especially during eating, for a 94. A 62-year-old patient diagnosed with
week. Objectively: the gums are swollen and TMJ arthritis presented to a clinic. The pati-
markedly hyperemic, especially in the area ent underwent radiography. Specify the radi-
of the 43, 42, 41, 31, 32, 33 teeth, soft plaque ological presentations of TMJ arthritis:
and tartar are present, gingival sulcus is up
to 2 mm deep. What is the most likely di- A. Changed shape of the bone elements of
agnosis? the joint
B. Joint space narrowing
A. Acute deep gingivitis C. Joint space widening
B. Chronic catarrhal gingivitis D. No joint space
C. Hypertrophic gingivitis E. Compaction of the articular head cortex
D. Initial stage of chronic generalized peri-
odontitis 95. A 45-year-old patient has a bilateral
E. Exacerbation of initial generalized peri- mental mandibular fracture with a typical
odontitis displacement of fragments. What actions
should be taken to eliminate asphyxia?
91. An 18-year-old patient complains of
chalky spot in the 23 tooth, which is sli- A. Pull out and pierce the tongue
ghtly painful when cold stimulus is appli- B. Artificial pulmonary ventilation
ed. Objectively: the vestibular surface near C. Tracheal intubation through the nose
dental cervix of the 23 tooth has a white D. Tracheal intubation through the mouth
spot 2-3 mm in size. Probing is painless and E. Tracheostomy
reveals coarseness of the surface. Thermal
stimulus causes slight pain. What is the most 96. A 53-year-old patient complains of
probable diagnosis? absent 12, 11, 21 and 22 teeth. On exami-
nation it is decided to make a porcelain-
A. Acute initial caries fused-to-metal bridge with the 13 and 23 as
B. Chronic initial caries abutment teeth. During preparation ledges
C. Fluorosis are created in abutment teeth. Ledge width
D. Local hypoplasia can vary within the following range:
E. Enamel necrosis
A. 1,5-2,0 mm
92. A 12-year-old child presents with body B. 0,5-0,8 mm
temperature of 38o C , chills, nausea, vomiti- C. 2,0-2,5 mm
ng, delirium, weakness. In the middle third D. 2,5-3,0 mm
of the face there is butterfly-shaped rash. E. 0,1-0,2 mm
Regional lymph nodes are enlarged, sli-
ghtly painful. Blood count: WBC- 12 · 109 /l, 97. A 16-year-old girl complains of cosmetic
lymphocytes - 8, 0 · 109 /l, ESR- 26 mm/hour. defect of the front teeth - there are dark
Make the diagnosis: spots and hard tissue defects. The spots
were detected in the process of teething, the
A. Erysipelas defects developed later. The following di-
B. Facial vein phlebitis agnosis was made: erosive form of fluorosis
C. Cutaneous actinomycosis of the 16, 11, 12, 22, 26, 31, 32, 36, 41, 42, 46
D. Streptoderma. Submandibular teeth. Choose the method of treatment:
lymphadenitis
E. Acute nonodontogenic sinusitis
Krok 2 Stomatology (англомовний варiант, iноземнi студенти) 2016 рiк 13

A. Restoration treatment A. Cancer of the tongue lateral surface


B. Remineralization treatment B. Trophic ulcer
C. Prosthetics C. Traumatic ulcer
D. Surgical treatment D. Vincent’s necrotizing ulcerative stomatitis
E. Regular medical check-ups E. Tuberculous ulcer
98. Parents of a 6-year-old boy complain of 101. An 8-year-old child has been clini-
his high body temperature, decrease of his cally diagnosed with exacerbation of chronic
general well-being, painful swelling of the periodontitis of the 84 tooth. The crown is
right cheek. Objectively: the skin is pale, the decayed by 1/2. What is the dentist’s optimal
face is asymmetric due to swelling of the tactics?
soft tissues of the right cheek. The maxillary
alveolar process is bilaterally thickened in A. Extraction
the area of the 55, 54 and 53 teeth, pai- B. Endodontic treatment
nful on palpation, these teeth are partially C. Endodontic treatment and drug therapy
destroyed by caries, mobile (II-III degree), D. Drug therapy
their dentogingival pockets produce pus. E. Opening along the mucogingival fold,
What is the most likely diagnosis? drug thrapy

A. Acute odontogenic osteomyelitis from the 102. A laminar denture for the lower jaw is
53, 54, 55 being made for a 54-year-old patient. Base
B. Acute odontogenic purulent maxillar plate wax is used during laboratory stage
periostitis from the 53, 54, 55 for wax templates. What group of accessory
C. Odontogenic buccal abscess from the 53, materials does such wax belong to?
54, 55
D. Acute odontogenic albuminous maxillar A. Modeling
periostitis from the 53, 54, 55 B. Abrasive
E. Odontogenic buccal phlegmon from the C. Fixing
53, 54, 55 D. Impression
E. Forming
99. In the process of the 26th tooth
extraction a 34-year-old patient suddenly 103. A 28-year-old patient has been
developed the following symptoms: tinni- scheduled for the surgical resection of
tus, weakness, paleness of skin. Vertigo was the root apex of the 12 tooth for chronic
diagnosed. What is the most advisable first granulomatous periodontitis. What kind of
aid approach? anesthesia should be administered before
the surgery?
A. Put the patient in the Trendelenburg
position A. Bilateral infraorbital and incisive
B. Intramuscular injection of 50% analgin B. Infraorbital and palatine
C. Plexual and palatine
(mеtamisole sodium) solution D. Tuberal and palatine
C. Intravenous injection of 0,1% adrenaline E. Terminal and incisive
D. Subcutaneous injection of cordiamin
E. Intravenous injection of prednisolone 104. A 11-year-old child complains of pain
during eating food, especially hot, in the
100. A 53-year-old patient complains of an lower right lateral tooth. On the masti-
ulcer on the lateral surface of the tongue. catory surface of the 46 tooth there is a
The ulcer appeared 6 months ago in the large carious cavity filled with softened
result of a trauma caused by sharp tip of the light-brown dentin. The cavity is located wi-
37 tooth metal crown. A dentist replaced thin parapulpar dentin. In the projection of
the crown with the one of better quality medial buccal pulp horn the carious cavity
and prescribed keratoplastic drugs. Despite communicates with the pulp chamber. Deep
these measures the ulcer continues to grow. probing is painful. Electric pulp test - 60 mi-
Lately there has been pain during talking, croamperes. Make the diagnosis:
chewing, swallowing; sometimes the pain
irradiates to the pharynx. Objectively: on A. Chronic gangrenous pulpitis
the lateral surface of the tongue there is B. Chronic hypertrophic pulpitis
a painful ulcer with uneven raised dense C. Acute diffuse pulpitis
margins and lumpy floor covered with grayi- D. Chronic fibrous pulpitis
sh necrotic coating. What diagnosis is most E. Acute focal pulpitis
likely?
105. A 34-year-old male patient complains
of a cosmetic defect, a cavity on the vesti-
bular surface in the cervical part of the 21
tooth. Objectively: the carious cavity is wi-
thin the enamel, the floor and the walls are
Krok 2 Stomatology (англомовний варiант, iноземнi студенти) 2016 рiк 14

pigmented, probing and percussion and pai- Select the best method of treatment:
nless. There is no pain reaction to stimuli.
What is the most likely diagnosis? A. Vital extirpation
B. Vital amputation
A. Chronic surface caries C. Biological method
B. Acute surface caries D. Devital amputation
C. Necrosis of dental hard tissues E. Devital extirpation
D. Acute median caries
E. Chronic median caries 110. A 9-year-old girl has been suffering
from gingival hemorrhages, cracks in the
106. A 12-year-old boy complains of dull mouth angles for a month. She has a hi-
ache in the upper right tooth. The pain story of frequent nosebleeds, rapid fatigabi-
aggravates during biting. Objectively: in lity. Objectively: the skin of the face and
the 16 tooth there is a deep carious cavity oral mucosa are pale. In the mouth angles
penetrating into the tooth cavity, cold test there are cracks reaching the red border. Gi-
and probing are painless, mucosa is swollen, ngiva in the region of the 11 and 12 teeth
hyperemic. X-ray reveals areas of bone ti- is hyperemic and edematous, bleeds when
ssue destruction with blurred margins near touched. DMF/dmf = 10. Back of the tongue
root apices. What diagnosis is most likely? is bright red, smooth, glossy. Submandi-
bular lymph nodes are slightly enlarged,
A. Exacerbation of chronic granulating mobile, painless. What specialist should be
periodontitis consulted in the first place?
B. Exacerbation of chronic granulomatous
periodontitis A. Hematologist
C. Exacerbation of chronic fibrous peri- B. Neuropathist
odontitis C. Endocrinologist
D. Acute purulent pulpitis D. Infectious disease specialist
E. Acute purulent periodontitis E. Gastroenterologist
107. To fill a medium depth carious cavi- 111. A 25-year-old patient complains of pain
ty in the 37 tooth (Black class II) of a 35- when biting on the 15 tooth. The pain arose
year-old male patient a doctor has chosen a two days ago, has a constant aching nature
technique of layer-by-layer tooth restorati- and increased significantly over the last day.
on. What composite material should be Objectively: the crown of the 15 tooth is gray,
applied for covering the carious cavity walls the medial contact surface exhibits a deep
and floor to create the initial super adaptive carious cavity communicating with the tooth
layer? cavity. Percussion causes acute pain, the gi-
ngival mucosa in the projection of the 25
A. Flowable tooth root apex is hyperemic. The regional
B. Condensable lymph node is tender. Radiograph shows an
C. Macrofilled ill-defined zone of periapical bone destructi-
D. Microhybrid on. What is the most likely diagnosis?
E. Microfilled
A. Exacerbation of chronic periodontitis
108. A patient suffered a facial trauma. B. Acute serous periodontitis
On examination and X-ray the patient was C. Acute suppurative periodontitis
diagnosed with fracture of the maxillar D. Chronic granulating periodontitis
alveolar process. What treatment would be E. Acute serous periodontitis, intoxication
most advisable in this case? stage
A. Smooth oral splint 112. A 27-year-old patient has been
B. Weber splint referred by a prosthodontist for endodontic
C. Gunning splint treatment of the 45 tooth. Objectively: the
D. Intermaxillary ligature 45 tooth crown is destroyed; the lateral
E. Zbarzh apparatus surface of the tongue and the buccal mucosa
have patches of grayish macerated epi-
109. A 20-year-old male patient complains thelium slightly rising above the mucosa
of spontaneous pain in the 24 tooth, whi- surface at the points of direct contact wi-
ch arose suddenly and persists for about th the 45 tooth. The uvula and palati-
15 minutes at a time. Objectively: the distal ne arches are stagnant-red in color; hard
surface of the 24 tooth exhibits a deep cari- palate has papulae surrounded with red
ous cavity with overhanging walls. The cavi- margin and covered in grayish epithelium.
ty is filled with light-colored softened dentin The submandibular, cervical, supraclavi-
and communicates with the tooth cavity. The cular, and subclavicular lymph nodes are
cold stimulus causes acute, slowly abating enlarged and painless. What is the provisi-
pain. Percussion causes no pain response. onal diagnosis?
Krok 2 Stomatology (англомовний варiант, iноземнi студенти) 2016 рiк 15

A. Cast removable splint


A. Secondary syphilis B. Kurlyandsky splint bar
B. Chronic recurrent aphthous stomatitis C. Cap splint
C. Lupus erythematosus, patch stage D. Portion crown
D. Soft leukoplakia (leucoplakia mollis) E. Half-ring splint
E. Lichen ruber planus
117. A patient complains of fever up to 38o C ,
113. A 20-year-old male patient with insulin- headache, pain in the joints anf muscles,
dependent diabetes mellitus has been given vesicles in the oral cavity, mainly in the
anesthesia with Ultracain DS-forte for surgi- frontal part. Eating is sharply painful. For
cal oral sanitation. What complications may the last several years the disease recurred
develop in this case? during wet anf windy weather. The patient
often suffers from cases of URTI. Objecti-
A. Abnormal levels of blood glucose vely: on the buccal, lingual, labial mucosa
B. Increased blood pressure there are confluent erosions against the
C. Clonus erythematous background, covered in gray-
D. Atopic dermatitis white coating. There are bloody scabs on the
E. Toxic shock red border and in the angles of the mouth.
Make the diagnosis:
114. As a result of an injury the 51and 61
teeth of a 3-year-old child have completely A. Erythema multiforme
cut into the alveolar bone. Medical history B. Syphilis
record: the teeth were partially decayed, C. Acute aphthous stomatitis
changed in colour. Select an appropriate D. Acute necrotizing ulcerative stomatitis
treatment tactics: E. Chronic recurrent aphthous stomatitis
A. Extraction of the injured teeth 118. A 14-year-old adolescent complains of
B. Follow-up as the teeth may erupt again bleeding and pain during eating occurring
C. Reposition of teeth, endodontic treatment in the 36 tooth. Anamnesis states intense
D. Extraction of teeth, endodontic treatment, pain in the past. Objectively: the crown is
reposition severely damaged; carious cavity is large,
E. Treatment is not required penetrates into the tooth cavity, and is parti-
ally filled with overgrown pulp. The tissues
115. A 5-year-old girl complains of pain demonstrate low sensitivity to touch, bleed
in the mouth angles occurring during upon touching. There are significant soft
mouth opening. She has a history of acute dental deposits on the 35 and 34 teeth. Make
bronchitis and prolonged antibiotic therapy. the diagnosis:
Objectively: the red border is dry, the mouth
angles are downturned. In the mouth angles A. Chronic hypertrophic pulpitis
there are cracks covered with white plaque B. Chronic gangrenous pulpitis
and surrounded by slightly hyperemic C. Chronic papillitis
elevation, painful on palpation and mouth D. Chronic simple pulpitis
opening. Oral mucosa exhibits no pathologi- E. Chronic granulating periodontitis
cal changes, decay-missing-filled index - 2,
hygiene index - 2,0. What is the most likely 119. The 47, 46, 45, 35, 36, 37, 38 teeth of a
diagnosis? 57-year-old patient are missing. It is planned
to make a clasp denture. The 48 tooth incli-
A. Mycotic cheilitis nes to the lingual side and forwards. On the
B. Streptococcal cheilitis lingual side of the 48 tooth the border line
C. Meteorological cheilitis is diagonal, on the buccal side it runs on a
D. Glandular cheilitis level with gingival edge. What type of Ney’s
E. Exfoliative cheilitis clasp should be applied?
116. A 43-year-old patient complains of A. V type clasp
mobility and significant neck exposure of B. I type clasp
the lower front teeth. Objectively: the gums C. IV type clasp (reverse back-action)
in the region of the 44, 43, 42, 41, 31, 32, 33, D. I-II type clasp
34 teeth are pale and cyanotic, non-bleeding. E. II type clasp
The 42, 41, 31, 32 teeth exhibit the I-II grade
mobility. The overcrowding of the 42, 41, 120. A 12-year-old boy due to trauma of
31, 32 teeth is present. The necks of the 42, the 44 and 45 teeth area suffers from the
41, 31, 32 teeth are exposed by 1/2 of the following: pathologic displacement of the
root length, the necks of the 43, 33 teeth mandibular alveolar process; rupture of the
are exposed by 1/4. What kind of dental alveolar process mucosa. What additional
prosthesis should be applied in this case? examination is necessary to specify the di-
agnosis?
Krok 2 Stomatology (англомовний варiант, iноземнi студенти) 2016 рiк 16

125. An orthodontist has been addressed


A. X-ray of the mandibula in frontal and by parents of a 5-year-old child. The chi-
lateral projections ld has the 54 tooth extracted, all the other
B. X-ray of the cranium in axillary projection deciduous teeth are present. The doctor
C. X-ray of the mandibula in frontal and made a thin-wall crown for the 55 tooth with
Parma projection interdental wedge to the 53 tooth. What is
D. Tomography of the mandibula the purpose of such treatment?
E. -
A. Prevention of dentition malformation
121. A 5-year-old child has sustained a B. Aesthetical restoration
trauma. Objectively: the crowns of the 51 C. Restoration of masticatory efficiency
and 61 teeth are shorter then the adjacent D. Acceleration of permanent tooth eruption
crowns. The mucosa surrounding the 51 E. Deceleration of permanent tooth eruption
and 61 teeth is hyperemic and swollen. X-
ray imaging shows the periodontal fissure 126. A 30-year-old patient, a lecturer,
to be absent in the apical area of the 51 complains of periodical dislocations of the
and 61 roots, the 51 and 61 roots apexes lower jaw. Objectively: direct occlusion,
are submerged in the spongy bone of the continuous dentition is retained. During
body of the lower jaw. Choose the optimal mouth opening there is a clicking sound
treatment: in the temporomandibular joint. What
apparatus can prevent recurrent dislocati-
A. Extraction of the 51st and 61st teeth on?
B. Splinting of teeth
C. Regular medical check-ups A. Petrosov apparatus
D. Anti-inflammatory therapy B. Oxman apparatus
E. Resection of the 51st and 61st teeth and C. Limberg apparatus
their splinting D. Shur apparatus
E. Zbarzh apparatus
122. During teeth preparation a pati-
ent developed an epileptic attack. What 127. A 37-year-old patient complains of
measures should a prostodontist take? severe pain in the gums, difficult chewing,
smell of rot from the mouth, bleeding gums,
A. To take measures to prevent tongue biting along with general fatigue, headaches, body
B. To continue working temperature up to 37, 9o C . Objectively: large
C. To call in an emergency aid team amout of soft dental deposit, shape of the
D. To measure blood pressure gingival margin is abnormal, the gingival
E. To introduse an anesthetic margin is covered with gray coating, regi-
onal lymph nodes are painful on palpation.
123. A 34-year-old dental patient has an What is the most likely diagnosis?
indication for the extraction of the 38
tooth for chronic fibrous periodontitis. The A. Ulcerative gingivitis
dental surgeon used torusal (Weisbrem’s) B. Chronic recurrent aphthous stomatitis
anesthesia. What nerves are blocked by this C. Chemical trauma
anesthesia? D. Herpetic stomatitis
E. Thermal trauma
A. Superior alveolar, lingual, buccal
B. Lingual, buccal 128. Parents of a 5-year-old boy addressed
C. Superior alveolar, buccal a dental surgeon with complaints of
D. Superior alveolar, lingual hemorrhage occurring from the boy’s
E. Lingual, buccal, mental mouth. According to the mother the boy
had accidentally bit his tongue. Objectively:
124. A 26 year-old woman suffers from the tongue apex is damaged, the wound is
bleeding gums. For 3 years she has been 5 mm deep and is bleeding profusely. What
suffering from pancreatic diabetes. Objecti- aid should be provided?
vely: gingival papillas of all the teeth are
hyperemic, swollen and overlap with the A. Perform primary surgical treatment of the
teeth crowns by 1/3 of the crowns hei- wound, apply blind sutures
ght. What method of examination must be B. Prescribe antibacterial treatment and
conducted for differentiated diagnostics? antiseptic oral rinsing
C. Perform antiseptic treatment of the wound
A. X-ray radiography and stop the hemorrhage by pressing the
B. Schiller-Pysarev iodine test tissues
C. Capillary fragility test D. Stop the hemorrhage by ligation of the
D. Rheoparodontography lingual artery
E. Stomatoscopy E. Prescribe hemostatics intravenously
Krok 2 Stomatology (англомовний варiант, iноземнi студенти) 2016 рiк 17

129. A 49-year-old patient was hospitali- the submandibular gland produces no saliva.
sed to an oral surgery department with a What preliminary diagnosis can be made?
gunshot wound of the left side of the face.
There is a 3,5 cm long defect of the body A. Sialolithiasis of the submandibular gland
of the mandible on the left. After the ini- on the right
tial surgical d-bridement and stabilization B. Acute serous lymphadenitis of the
of the patient’s general state it is planned submandibular area on the right
to perform plastic correction of the defect. C. Abscess of the mental region on the right
What treatment method is optimal in the gi- D. Exacerbation of sialolithiasis of the
ven case? submandibular gland on the right
E. Abscess of the submandibular region on
A. Shvyrkov’s compression-distraction the right
osteosynthesis
B. Osteosynthesis with titanium reconstructi- 133. Parents complain of painfulness and
on plate mobility of the tooth of their 4-year-old chi-
C. Rudko’s apparatus application ld, which developed after the impact with
D. Galmosh polyfunctional apparatus appli- a wooden object. Objectively: the face is
cation asymmetrical due to swollen tissues of the
E. Substitution of the defect with an autograft upper lip. The 51 tooth is intact, with vesti-
bular displacement and the II degree mobili-
130. Before performing a surgery a dental ty, gums around the 51 tooth are hyperemic.
surgeon treats his hands according to What preliminary diagnosis can be made?
Spasokukotsky-Kochergin method. What
antiseptic is basic in this kind of treatment A. Incomplete dislocation of the 51 tooth
and how is it applied? B. Complete dislocation of the 51 tooth
C. Contusion of the 51 tooth
A. 0,5% liquid ammonia solution in two D. Acute albuminous periostitis
washbasins, 3-5 minutes in each E. Acute purulent periostitis
B. Tissue with 98% ethanol, 5 minutes
C. Two tissues with 96% ethanol, 5 minutes 134. During preventive examination of a 19-
each year-old man it was revealed that there are
D. Two tissues with 5% liquid ammonia carious lesions in the fissures of the 37 and 47
solution, 5 minutes each teeth, sealant is fully retained in the 16 and
E. Tissue with 0,5% chlorhexidine di- 26 but absent in the closed intact fissures of
gluconate, 5 minutes the 36. The 46 was extracted due to compli-
cated caries. Determine the degree of caries
131. A 37-year-old patient addressed a in this patient:
dental surgeon with complaints of deteri-
orating welbeing, high fever up to 38, 1o C , A. 3
headache, painful swelling on the chin. B. 2
Objectively: there is a dense painful infi- C. 4
ltration 3x4 cm in size on the skin of the D. 5
chin. The skin over the infiltration is bluish E. 6
in color. There are four necrosis areas in the
focal center around the hair follicles. The 135. A dentist performs endodontic
submental lymph nodes are enlarged and treatment of the 31 tooth of a 62-year-old
painful on palpation. Make the diagnosis: patient. The root canal is narrow, extremely
calcificated. Choose the optimal tool to pass
A. Carbuncle through the root canal in this case:
B. Furuncle
C. Erysipelas A. Pathfinder
D. Dermal actinomycosis B. Gates-glidden drill
E. Festered atheroma C. K-Reamer
D. K-File Nitiflex
132. A 43-year-old woman complains of E. H-File
periodical facial assymetry and bursting
stabbing pain in the submandibular area 136. A crown inlay for the patient’s 46 tooth
on the right, which increases after eati- is being prepared. At the first clinical stage
ng spicy and sour food. Eventually these the cavity preparation is completed with
clinical presentations disappear. The pati- design of an incline. What material can be
ent notes periodical exacerbations. Objecti- used for the inlay?
vely: the face is symmetrical, mouth openi-
ng is unrestricted, mucosa of the oral cavi-
ty floor is without pathologies. In the area
of the sublingual fold a thickening of soft
tissues can be palpated. Excretory duct of
Krok 2 Stomatology (англомовний варiант, iноземнi студенти) 2016 рiк 18

A. Metal alloy A. Angina pectoris attack


B. Sinma M plastic B. Heart failure
C. Porcelain C. Bronchial asthma attack
D. Composite D. Hypertensic crisis
E. Pressed ceramics E. -
137. A 54-year-old patient complains of 141. During administration of Ultracain
frequent crunching sound in the right solution for infiltration anesthesia the
temporomandibular joint, which developed condition of a 22-year-old patient sharply
one month ago. In the morning the deteriorated. The patient became inert, si-
crunching is more frequent and decreases lent, developed cough attacks, expiratory
towards the evening. Objectively: the face is dyspnea, wheezing. Viscous slimy sputum is
symmetrical, the skin above the joint is wi- expectorated. What urgent condition did the
thout changes, the mouth opens at 2,9 mm. patient develop?
What diagnosis is most likely in this case?
A. Bronchial asthma attack
A. Arthrosis B. Anaphylactic shock
B. Acute arthritis C. Hypertensic crisis
C. Temporomandibular joint dislocation D. Asphyxia
D. Chronic arthritis E. Collapse
E. Temporomandibular joint pain dysfuncti-
on syndrome 142. A 2,5-year-old boy has been delivered
to an oral surgery. The patient has the upper
138. On examination of a 27-year-old patient lip cleft into three fragments, the palate is
the tip of the dental probe caught on the fi- whole. Specify the period when cheiloplasty
ssures of the 36, 37, 38 teeth. Margins of the should be performed:
enamel defect are dark, the surface is coarse.
Teeth transillumination with photopolymer A. 6-10 months
lamp revealed the defect to be limited to the B. 3-5 months
enamel. What diagnosis is most likely? C. 12-14 months
D. 15-19 months
A. Chronic superficial caries E. 20-24 months
B. Chronic median caries
C. Acute superficial caries 143. A 7-year-old boy has hyperemic gums;
D. Chronic primary caries bleeding can be detected. The teeth are
E. Acute primary caries mobile, with roots exposed and covered wi-
th a layer of coating. Periodontal pockets
139. A 21-year-old woman addressed a demonstrate granulation. The child is regi-
doctor with complaints of bleeding gums stered with a pediatrician for diabetes insi-
and foul smell from her mouth. Objectively: pidus. The child’s mother states that her
the gums are congestively hyperemic, with son’s condition has deteriorated lately. The
cyanotic color. X-ray revealed resorption of child suffers from periodontal syndrome
interalveolar septa up to 1/3 of their height. characteristic of:
What is the most likely diagnosis?
A. Hand-Schuller-Christian disease
A. Chronic generalized periodontitis, I B. Diabetes mellitus
degree C. Gaucher’s disease
B. Chronic generalized periodontitis, II D. Cushing’s disease
degree E. Letterer-Siwe disease
C. Acute generalized periodontitis, I degree
D. Chronic generalized periodontitis, III 144. A 40-year-old patient addressed a
degree prosthodontist with complaints of absent
E. Acute generalized periodontitis, II degree crown of the 24 tooth due to trauma. If the
crown is broken off at the gum level, the
140. A 68-year-old patient addressed a tooth should be restored with:
surgical department of a dental clinic for
extraction of the 45 tooth. During procedure A. Pivot crown
the patient developed burning retrosternal B. Semi-crown
pain attack irradiating to the left shoulder, C. Equator crown
scapula, hand. The skin is pale, BP is 140/100 D. Partial denture
mm Hg, heart rate is rapid. Skin hyperplasia E. Intradental inlay
can be observed in the Zakharin-Head’s
zones. What emergency condition did the 145. During extraction of the 36 tooth the
patient develop? tooth crown was fractured. What tool should
be applied to extract tooth roots?
Krok 2 Stomatology (англомовний варiант, iноземнi студенти) 2016 рiк 19

A. Bayonet forceps is covered in scarce vesicles with clear


B. Beak-shaped forceps left side content. Within the oral cavity on the buccal
C. Beak-shaped straight forceps and lingual mucosa there are sharply pai-
D. S-shaped forceps left side nful erosions, 2-3 mm in size, with white
E. S-shaped forceps right side coating and hyperemic crown. The gums
are swollen, hyperemic. The submandi-
146. A patient after the surgery for bular lymph nodes are enlarged, painful on
phlegmon of the pterygomandibular palpation. Make the diagnosis:
space was prescribed compresses with a
membrane perforating drug. Which of the A. Acute herpetic stomatitis
given below is a membrane perforating B. Stevens-Johnson syndrome
drug? C. Erythema multiforme
D. Stomatitis with the background of infecti-
A. Dimexid (Dimethyl sulfoxide) ous mononucleosis
B. Ectericidum E. Stomatitis with the background of chi-
C. Streptocid (Sulfidine) ckenpox
D. Lincomycin
E. Oflocain 151. A 9-year-old girl complains of peeli-
ng lips and sensation of dryness, strain and
147. A 64-year-old patient complains of burning in them. Objectively: on the red
spots on the mucosa of the cheeks, mouth border from angle to angle of the mouth
angles, palate. Objectively: on the oral cavity and from Klein area (wet-dry line) to the
mucosa there are white growths that cannot red border middle there are gray scales.
be removed with spatula. The mucosa The scales can be removed without erosi-
around the spots is pale pink in color. The ons. Make the preliminary diagnosis:
patient has been smoking for 40 years. What
is the most likely diagnosis? A. Exfoliative cheilitis, dry form
B. Exfoliative cheilitis, exudative form
A. Leukoplakia C. Meteorological cheilitis
B. Lupus erythematosus D. Atopic cheilitis
C. Bowen’s disease E. Allergic contact cheilitis
D. Candidiasis
E. Lichen ruber planus 152. A patient addressed a dental
surgeon with complaints of increased body
148. An 8-year-old girl complains of burni- temperature up to 37, 6o C , swelling of the
ng pain in her tongue during eating spicy soft tissues, pain in the 65 tooth, whi-
and sour food. Objectively: there are red ch aggravates upon touching it with its
irregular-shaped spots on the tongue apex antagonist tooth. On examination there are
and dorsum. Filiform papillae are absent in hyperemy and smoothing out of the mucogi-
the affected areas. The girl states that the ngival fold of the alveolar process in the area
spots periodically grow and migrate. Make of the destroyed 65 tooth. What diagnosis is
the diagnosis: most likely?
A. Benign migratory glossitis A. Acute odontogenic purulent periostitis
B. Median rhomboid glossitis B. Acute serous periodontitis
C. Fissured tongue C. Exacerbation of chronic periodontitis
D. Hairy tongue D. Acute odontogenic osteomyelitis
E. Raspberry tongue E. Parodontal abscess
149. A 70-year-old patient addressed a 153. A 17-year-old patient has been suffering
hospital with complaints of poorly stabi- for 2 years from periodical painful swelli-
lized complete dentures of the upper and ng and fistula appearing on the skin of
lower jaws. What method of artificial teeth gonial angle area. After cleaning the fi-
arrangement is preferable in making of a stula of its purulent content pain and swelli-
new denture? ng disappear leaving a small concave scar.
A. According to individual occlusion curves Objectively: there is a small scar on the skin
B. According to disocclusion planes in the gonial angle area. Palpation of the
C. According to standard occlusion curves oral cavity floor reveals a dense band in the
D. According to spherical occlusion curves buccal soft tissues, which connects fistula wi-
E. According to prothetic occlusion planes th projection of the 47 tooth roots. Crown of
the 47 is destroyed. What diagnosis is most
150. A 2,5-year-old child has fever up to likely?
38, 5o C , low appetite, rash inside the oral
cavity. The disease onset was 3 days ago.
Objectively: the skin of the perioral area
Krok 2 Stomatology (англомовний варiант, iноземнi студенти) 2016 рiк 20

A. Migrating subcutaneous granuloma and swelling of the gums, gingival margin


B. Festered buccal atheroma hypertrophy of the 12, 13, 14 teeth up to 1/2
C. Mandibular tuberculosis of the crown height. Formalin test is pai-
D. Actinomycosis of the parotid-masticatory nless. What diagnosis is most likely?
area
E. Chronic osteomyelitis of the left mandi- A. Hypertrophic gingivitis
bular angle B. Chronic generalized periodontitis, II
degree
154. A 30-year-old woman addressed C. Catarrhal gingivitis
a dental surgeon with complaints of D. Ulcerative gingivitis
uncomfortable sensation of pressure in her E. Acute generalized periodontitis, I degree
upper right tooth, which aggravates due to
hot stimulus, and foul smell from the mouth. 158. A 48-year-old patient has made an
Objectively: there is a deep carious cavity appointment for extraction of the 35 tooth
in the 17 tooth, which penetrates into the due to exacerbation of chronic periodontitis.
tooth cavity. Deep probing causes severe Specify the most advisable method of peri-
pain, percussion of the 17 tooth is painful. pheral conduction anesthesia in this case:
X-ray: there is slight widening of the peri-
odontal fissure near the root apex. Electric A. Torusal
pulp test - 70 microamperes. What final di- B. Infiltration
agnosis can be made? C. Extraoral mandibular
D. Mental
A. Chronic gangrenous pulpitis E. Plexus
B. Chronic fibrous pulpitis
C. Acute purulent pulpitis 159. During application of an impression
D. Chronic fibrous periodontitis tray to the upper jaw of a 62-year-old patient
E. Exacerbation of chronic fibrous peri- the tray falls out when the patient opens his
odontitis mouth wide. What edge of the tray should
be shortened?
155. A 20-year-old patient has made an
appointment with a dentist. On external A. In posterolateral area of the maxillary
examination the patient is pale, complai- tuberosity
ns of long-term fever, insomnia, weight B. Along the A line
loss of 13 kg within 5 months. The pati- C. In the front area
ent is homeless. The therapist’s consultati- D. In the palatine torus area
on states: all groups of the lymph nodes E. In the buccal-alveolar fold area
are enlarged, hepatolienal syndrome. Blood
160. During objective examination of the
test: leukocytes - 2, 2 · 109 /l. Make the di- patient’s oral cavity the lower jaw is revealed
agnosis: to be toothless, in the front area there is a
A. HIV infection mobile band of mucosa, irregular atrophy of
B. Chroniosepsis the alveolar process. What method of obtai-
C. Tuberculosis ning functional impression would be most
D. Infectious mononucleosis advisable in the given case?
E. Lymphogranulomatosis A. Differentiated impression
156. A 45-year-old patient complains of B. Decompression impression
itching gums, pain in the area of the 43, C. Compression impression
42, 41, 31, 32, 33 teeth caused by chemi- D. Selective pressure impression during
cal and thermal stimuli. Objectively: the mastication
gums are pale, no bleeding, the roots of E. Gradual pressure impression
affected teeth are exposed by 0,4 mm. The 161. A 46-year-old man complains of
43 and 33 demonstrate cuneiform defect. constant losing of a filling in his lower ri-
The teeth are intact. X-ray: regular resorpti- ght tooth. Objectively: in the 16 tooth on the
on of interalveolar septa at 1/4 of their hei- approximal masticatory surface there is a
ght. Make the diagnosis: defect of crown hard tissues at 1/3. The tooth
A. Parodontosis, I degree has no discoloration, percussion is painless.
B. Generalized periodontitis, I degree What construction should be prescribed?
C. Catarrhal gingivitis A. Inlay
D. Hypertrophic gingivitis B. Porcelain-fused-to-metal crown
E. Ulcerative gingivitis C. Semi-crown
157. An 18-year-old woman complains of D. 3/4 crown
gum growth, pain and bleeding during E. Plastic crown
eating solid food. Objectively: hyperemy 162. A 37-year-old patient complains of an
Krok 2 Stomatology (англомовний варiант, iноземнi студенти) 2016 рiк 21

aesthetic defect. Objectively: the 13 tooth is when is the optimal time for such a surgery?
destroyed by 2/3. The tooth is pulpless, the
root canal is filled. How deep should be the A. Uranostaphyloplasty, 4-6 years
root canal opened for pivot crown in this B. Cheiloplasty, 4-6 months
patient? C. Uranostaphyloplasty, 6-7 months
D. Staphyloplasty, 10 years
A. 2/3 of the root canal E. Uranoplasty, 1 year
B. 1/3 of the root canal
C. 3/4 of the root canal 167. A 19-year-old girl addressed an
D. 1/2 of the root canal oncologist with complaints of slowly growi-
E. Full length of the root canal ng tumor-like mass on the tip of her tongue.
The mass was first noticed 5-6 years ago. The
163. A 7-year-old girl hit her forehead one patient requested no medical help. Objecti-
day ago. Several hours after the sustained vely: there is a pale pink round growth with
trauma a swelling developed in the left wide pedicle on the tongue apex; the growth
superciliary area. General condition of the is painless, elastic; there are no changes of
child is not disrupted. Objectively: swelling mucosa surrounding the pedicle. Submandi-
of forehead tissues spreading towards the bular lymph nodes cannot be palpated.
left eyelids; the swelling is soft, fluctuati- What kind of tumor is it?
on sign is present. Make the preliminary di-
agnosis: A. Papilloma
B. Atheroma
A. Hematoma of the left superciliary area C. Fibroma
B. Hemangioma of the right superciliary area D. Lipoma
C. Fracture of the temporal bone E. Keratoma
D. Fracture of the frontal bone
E. Hematic abscess of the left superciliary 168. A 48-year-old patient has addressed a
area hospital with complaints of defects in the
paragingival area and slight sensitivity to
164. Mother of a 10-year-old girl complai- thermal stimuli. Objectively: there are hard
ns of a cosmetic defect of the child’s 22 tissue defects that resemble a wedge with
tooth that erupted with damaged enamel. smooth polished walls on precervical vesti-
Anamnesis states premature extraction of bular surface of the 23 and 24 teeth. Thermal
the 62 tooth due to caries complication. test is slightly positive. What is the most li-
There is a white-yellow spot with clear kely diagnosis?
margins on the vestibular surface of the 22
tooth. Enamel retains glossiness, no surface A. Cuneiform defect
roughness can be detected on probing. Make B. Enamel necrosis
the diagnosis: C. Acute deep caries
D. Enamel erosion
A. Local enamel hypoplasia E. Endemic fluorosis
B. Fluorosis
C. Acute superficial caries 169. Parents of a 7-year-old child addressed
D. Chronic superficial caries a hospital with complaints of the child havi-
E. Systemic enamel hypoplasia ng no permanent teeth in the front area of
the mandible. Anamnesis states that the fi-
165. A 2-year-old child has suffered a teeth rst deciduous teeth erupted at the age of
trauma. Objectively: the crowns of the 51 11 months. Objective clinical examination
and 61 teeth are by 1/3 shorter than the revealed the following: appearance is wi-
others. Mucosa in the area of the 51 and 61 thout changes; milk occlusion; there are
is hyperemic, swollen. X-ray reveals absence physiological diastemas and tremas; edge-
of the periodontal fissure in the root apex to-edge incisor contact. What preliminary
areas of the 51 and 61. What treatment tacti- diagnosis can be made according to Kalvelis
cs would be optimal? classification?
A. Regular medical check-ups A. Retarded eruption
B. Extraction of the 51 and 61 B. Supernumerary tooth
C. Reduction of the 51 and 61 C. Adentia
D. Ligature splint D. Dystopia
E. Dental reimplantation E. Hypoplasia
166. A mother of a newborn complains that 170. Parents of a 4-year-old child have made
the child cannot be breastfed. After objecti- an appointment with an orthodontist for
ve examination the child was diagnosed wi- preventive examination of the child. In the
th complete submucous cleft palate. What oral cavity: scissor bite is observed in the
surgical procedure should be prescribed and front area in the place of inscisor contact;
Krok 2 Stomatology (англомовний варiант, iноземнi студенти) 2016 рiк 22

distal surfaces of the second temporary ve examination. No complaints from the


molars are situated in one plane. Teeth in patient. Objectively: on the 36 masticatory
the dental arch are placed tightly, without surface there is a carious cavity within
intervals. Determine, what period of occlusi- enamel with wide inlet. Enamel of the cavi-
on development is described: ty floor and walls is dense and pigmented.
Probing of the carious lesion is painless. No
A. Stability of temporary occlusion reaction to thermal stimuli. Tooth percussi-
B. Formation of temporary occlusion on is painless. The girl belongs to the I group
C. Involution of temporary occlusion according to her health status. Make the di-
D. Early stage of developing occlusion agnosis:
E. Late stage of developing occlusion
A. Chronic superficial caries
171. A 22-year-old patient addressed an B. Acute median caries
orthodontist with complaints of incorrect C. Acute initial caries
teeth placement on the upper jaw. The D. Chronic initial caries
appearance is without changes. Occlusion is E. Acute superficial caries
permanent. The first molar relation is of the
1st type according to Angle’s classification. 175. A 57-year-old patient adressed a
Analysis of the control diagnostic models prosthodontic clinic to have a denture made
revealed the disturbance of the Andrews’ for him. After objective examination the
second key of optimal occlusion of the 12 patient was prescribed single-unit crowns
and 22 teeth. The patient suffers from: for the 46, 47, 36, 37 teeth. What maximal
angle of a tooth stump can be allowed duri-
A. Angulation of the maxillary teeth ng preparation?
B. Cusp-to-fissure contact between the first
permanent maxillary and mandibular molars A. 3-5o
C. Maxillary teeth contact B. 5-6o
D. Axial rotation of the teeth C. 10-12o
E. Spee curve concavity D. Stump walls should be parallel to each
other
172. A 15-year-old girl complains of brief E. 7-8o
pain attacks in her teeth due to chemical sti-
muli. Objectively: on the contact surfaces of 176. A 25-year-old woman, a teacher,
the 11, 21, 22 teeth there are enamel areas addressed a prosthodontics clinic with
matt white in color, with lost shine, covered complaints of cosmetic defect, inability to
in large amount of dental deposit. Enamel take a bite, disrupted enunciation. The teeth
is softened and can be easily chipped off wi- are lost due to trauma. Dental formula:
th excavator. Probing of lesions is painless. 87654300!00345678. 3!3 are firm, intact,
Percussion is painless. No reaction to cold percussion is painless. What anesthesia
stimuli. Make the diagnosis: should be administered for this group of
teeth in preparation for making a porcelain-
A. Acute superficial caries fused-to-metal bridge?
B. Acute median caries
C. Acute initial caries A. Infiltration
D. Chronic initial caries B. Application
E. Chronic superficial caries C. Central conduction anesthesia
D. Tuberal
173. A 5,5-year-old child is undergoi- E. Intraosseous (into spongy tissue)
ng preventive examination. No complai-
nts from the patient. Objectively: in the 177. A 58-year-old patient after examination
precervical area of buccal surface of the in a prostodontic clinic has been advised to
75 tooth there is an enamel patch with loss make a full-arch prosthesis. The treatment
of natural glossiness. The enamel surface is plan was developed, the impressions of both
coarse, painless on probing. Tooth percussi- jaws were obtained, central occlusion was
on is painless. Thermodiagnosis is negati- determined with block method. What stage
ve. Damaged enamel stains with 2% water is next?
solution of methylene blue. Make the di-
agnosis: A. Parallelometry
B. Model duplication
A. Acute initial caries C. Restoration of the abutment crowns
B. Enamel hypoplasia D. Frame design of a full-arch prosthesis
C. Fluorosis E. Preparation for model duplication
D. Acute superficial caries
E. Chronic initial caries 178. A 25-year-old patient addressed a
hospital and upon examination was di-
174. A 13-year-old girl undergoes preventi- agnosed with unilateral fracture of the body
Krok 2 Stomatology (англомовний варiант, iноземнi студенти) 2016 рiк 23

of the mandible in the area of 43, 44 teeth belong to?


with persistent vertical displacement of the
bone ends. Choose the functional type of A. First
prosthodontic apparatus required for the B. Second
patient’s treatment: C. Third
D. Fourth
A. Reponating E. Fifth
B. Directing
C. Forming 183. Preventive examination of the oral cavi-
D. Substituting ty of a 9,5-year-old child revealed bilateral
E. Fixing cross bite without displacement of the lower
jaw. What biometric method of dentition
179. A 32-year-old patient addressed a denti- model investigation should be applied in this
st with complaints of inability to close his case?
mouth. Objectively the mouth is half-open,
the chin is protruding forwards and is di- A. Pont index
splaced to the left. Such condition occurred B. Gerlach index
after the mouth was opened wide. What di- C. Schwarz index
agnosis is most likely? D. Korkhaus index
E. Tonn index
A. Anterior right-sided mandibular dislocati-
on 184. For a 30-year-old patient a composite
B. Anterior left-sided mandibular dislocation inlay for the 37 tooth is being made. Objecti-
C. Anterior bilateral mandibular dislocation vely: there is a carious cavity of medium size
D. Posterior right-sided mandibular dislocati- (Black’s classification I class) on the masti-
on catory surface of the 37 tooth. What specifics
E. Posterior left-sided mandibular dislocation are required for the tooth preparation in this
case?
180. A 37-year-old patient has symmetrical
face; the mucosa in the area of the 12 tooth A. Creating a flat floor
root apex projection is pale pink; palpation B. Creating an auxiliary ledge
is painless; the tooth crown is destroyed by C. Creating an incline
1/3; percussion is painless. X-ray: the root D. Widening of the cavity floor
canal of the 12 tooth is filled to the apex; E. Creating an auxiliary cavity
granuloma 4 mm in diameter surrounds the
root apex. Choose the method of surgical 185. A bugle denture for the lower jaw is
treatment: planned to be made for a 53-year-old pati-
ent. Objectively: the 38, 37, 35, 34, 45, 46, 47
A. Granuloma removal with root apex teeth are absent. The retained teeth are fi-
resection rm, with low clinical crowns. What fixation
B. Root hemisection method of bugle denture would be optimal
C. Coronary radicular tooth separation in this case?
D. Root amputation
E. Tooth extraction A. Telescopic system
B. Supporting retentive clammers
181. During preventive examination a C. Beam system
patient was diagnosed with precancerous D. Attachments
hyperkeratosis of the lower lip red border. E. Ball joint attachment
What treatment should be prescribed?
186. A 30-year-old patient complains of
A. Surgical removal of the focus within spontaneous pain attacks in the lower left
healthy tissues tooth, which occurred 2 hours ago duri-
B. Surgical removal of the focus wi- ng travel by plain. Objectively: there is
thin healthy tissues + close-focus pathological abrasion of the 36 tooth, tooth
roentgenotherapy percussion is slightly painful. After addi-
C. No treatment is required tional investigation the diagnosis is made:
D. Surgical removal of the focus within chronic pulpitis. What investigation allows
healthy tissues + chemotherapy to make such a diagnosis?
E. Palliative treatment
A. X-ray radiography
182. In the district dispensary center for B. Probing
children with cleft lip and palate a child C. Thermal tests
was diagnosed with congenital incomplete D. Luminescence diagnostics
cleft upper lip on the left with malformed E. Electric pulp test
cutaneo-cartilaginous portion of the nose.
What dispensary group does the child 187. A 22-year-old student arrived for an
Krok 2 Stomatology (англомовний варiант, iноземнi студенти) 2016 рiк 24

appointment with a dentist in the afternoon 191. A 24-year-old patient addressed a


after his classes were over. The patient is first-aid center with complaints of facial
registered for regular check-ups with an pain on the left, restricted mouth openi-
endocrinologist. During treatment the pati- ng. Objectively: there is moderate swelling
ent developed excited state followed by loss and hematoma in the left malar arch area.
of consciousness. Objectively: tremor of the On palpation there is bone tissue retracti-
extremities, moist skin, heart rate - 100/min., on and moderate painfulness. Occlusion is
BP- 100/60 mm Hg. What diagnosis is most intact. Mouth opening is 1-1,5 cm. Upon
likely? attempt to open the mouth further there are
a sensation of mechanical obstructon and
A. Hypoglycemic coma pain aggravation. What is the most likely di-
B. Syncope agnosis?
C. Anaphylactic shock
D. Hyperglycemic coma A. Fracture of the left malar arch
E. Epileptic attack B. Fracture of the left zygomatic bone
C. Le Fort maxillary fracture, III grade
188. A 32-year-old patient consulted a D. Le Fort maxillary fracture, II grade
dentist with complaints of tooth crown E. -
discoloration on the upper right jaw.
Anamnesis: 3 weeks ago the patient suffered 192. A 22-year-old patient has suffered uni-
a trauma of the upper jaw. On examination: lateral linear fracture in the area of the
crown of the 11 tooth is intact, gray, pai- gonial angle. Immobilization was provi-
nless on percussion, no changes of mucosa ded with full dental brace with loops and
in the area of root apex projection of the 11 intermaxillary elastic expansion. Recovery
tooth. Electric pulp test - 100 microamperes. was uncomplicated. The brace should be
The patient was diagnosed with traumatic removed after:
periodontitis of the 11 tooth. What method
of removing tooth discoloration would be A. 3 weeks
advisable after endodontic treatment? B. 2 weeks
C. 1 week
A. Intracrown bleaching D. 10 days
B. External bleaching E. -
C. Custom teeth whitening with a bleaching
tray (kappa) 193. A 19-year-old woman complains of
D. Microabrasion method aesthetic defects, dry lips, unpleasant
E. Air-abrasive method sensation of strain in the lower lip and its
constant peeling. The patient often bites her
189. A 39-year-old patient was diagnosed wi- lips, rips and bites off peeling flakes, which
th chronic generalized periodontitis, initial sometimes results in bleeding; she notes her
stage. After undergoing complex treatment increased irritability. On examination there
the patient received instructions and was are numerous transparent scales between
registered for regular check-ups. How often the red border and mucosa. The scales are
should the patient undergo these regular tightly adjacent in the center and loose at
check-ups? the periphery. They can be easily removed,
produce no erosion. Regional lymph nodes
A. Once every 6 months cannot be palpated. What diagnosis is most
B. Once every 3 months likely?
C. Once every 9 months
D. Once every 12 months A. Cheilitis exfoliativa
E. Once every 18 months B. Mycotic cheilitis
C. Meteorological cheilitis
190. A 22-year-old patient is diagnosed wi- D. Allergic contact cheilitis
th chronic granulomatous periodontitis of E. Actinic cheilitis
the 46 tooth. During the first appointment
the patient was prescribed a temporary filli- 194. A 73-year-old man is registered for
ng made of antibacterial mateterial that sti- regular check-ups in an oncological clinic
mulates osteo-, dentino- and cement genesis. after completion of the combined treatment
Specify the material that satisfies this requi- for oral mucosa cancer stage II (radiati-
rements: on therapy and surgery). During one of
the routine check-ups an area of exposed
A. Calcium hydroxide mandibular bone is detected. There are
B. Iodoform no inflammatory changes of surrounding
C. Dexamethasone mucosa. A sinus tract with weak granulati-
D. Thymol on is observed. Mandibular X-ray shows
E. Camphor sequestrum without clear margin between
healthy and necrotic bone. What diagnosis
Krok 2 Stomatology (англомовний варiант, iноземнi студенти) 2016 рiк 25

is most likely? A. Interarticular disk


B. Articular capsule
A. Mandibular osteoradionecrosis C. Articular head
B. Acute purulent mandibular osteomyelitis D. Distal slope of the articular tubercle
C. Posttraumatic mandibular osteomyelitis E. Socket floor of the temporal bone
D. Relapse of oral mucosa cancer
E. Chronic mandibular periostitis 198. A 34-year-old patient complains of pain
in the lower right jaw, reduced mouth openi-
195. A patient with odontogenic phlegmon ng. Objectively: dentition is intact both on
of the oral cavity floor and neck has been the upper and lower jaw, there is premature
delivered into an oral surgery department. contact between the 46 and 47 teeth. Di-
The patient’s general condition is severe, agnosis: traumatic fracture of the mandible
labored breating, inflammatory masticatory in the area of the 46 tooth with fragment
muscles contracture II degree, swallowi- displacement. What construction would be
ng is impossible. There can be determi- advisable for temporary immobilization of
ned dense sharply painful infiltration of the the mandible fragments?
lower face, oral cavity floor and neck; the
skin is strained and cannot be pinched in a A. Tigerstedt’s splint
fold; fluctuation can be palpated in the oral B. Temporary plastic splint
cavity. After 30 minutes of the patient’s stay C. Entin’s head-chin strap
in the in-patient department the patient’s D. Plastic kappa
neck swelling significantly enlarged, breati- E. Weber’s splint
ng deteriorated. What kind of asphyxia can
develop in the patient? 199. A 55-year-old patient addressed a
prostodontic clinic to have a denture made.
A. Stenotic During preparation of the 44 and 47 teeth
B. Dislocational for crown installation the patient became
C. Aspiration unwell: he developed vertigo, nausea, ti-
D. Valvular nnitus, blurred vision, dyspnea. Unconsci-
E. Obturative ousness followed, the pupils were dilated,
heart rate was weak. After 1 minute the
196. A 16-year-old girl suffers from enlarged patient’s condition improved. What condi-
gums. Objectively: gingival margin of the tion occurred in the patient?
vestibular surface of the gums is diffusely
thickened, dense, pale pink in color. Gingi- A. Dizziness
val papilla cover crowns of all the teeth up to B. Anaphylactic shock
1/2 of their height, hemorrhage is absent. X- C. Collapse
ray reveals no pathological changes of peri- D. Hypoglycemic coma
odontal tissues. Make the diagnosis: E. Bronchial asthma
A. Chronic hypertrophic gingivitis 200. A 23-year-old woman complains of
B. Exacerbation of catarrhal gingivitis carious cavity present in the 27 tooth,
C. Acute catarrhal gingivitis where food particles are retained. Objecti-
D. Chronic catarrhal gingivitis vely: there is a deep carious cavity on the
E. Gingival fibromatosis masticatory surface of the 27 tooth filled
with dense dark-brown dentin. Probing
197. A 40-year-old patient complains of pain of the cavity floor and walls is painless,
in the tragus area, clicking during mouth thermometry and percussion are painless.
opening, stuffed ears. Objectively: the face Electric pulp test - 10 microamperes. X-ray
is symmetrical, mouth opening path is strai- reveals no pathological changes. Make the
ght. Dentition defect can be estimated as the diagnosis:
I class by Kennedy; the 18, 17, 16, 26, 27, 28
teeth are absent. In this case the load would A. Chronic deep caries
be the most traumatizing for the following B. Chronic median caries
anatomical structure: C. Acute deep caries
D. Chronic fibrous pulpitis
E. Chronic periodontitis
INSTRUCTIONAL BOOK
Ministry of public health of Ukraine (MPH of Ukraine)
Department of human recources policy, education and science
Testing Board

TEST ITEMS FOR LICENSING EXAMINATION: KROK 2. STOMATOLOGY.

Kyiv. Testing Board.


(English language).

Approved to print 01.03./№24. Paper size 60х84 1/8


Offset paper. Typeface. Times New Roman Cyr. Offset print.
Conditional print pages 24. Accounting publishing pages 28.
Issue. 387 copies.
List of abbreviations

A/G Albumin/globulin ratio HR Heart rate


A-ANON Alcoholics anonymous IDDM Insulin dependent diabetes mellitus
ACT Abdominal computed tomography IFA Immunofluorescence assay
ADP Adenosine diphosphate IHD Ischemic heart disease
ALT Alanin aminotranspherase IU International unit
AMP Adenosine monophosphate LDH Lactate dehydrogenase
AP Action potential MSEC Medical and sanitary expert committee
ARF Acute renal failure NAD Nicotine amide adenine dinucleotide
AST Aspartat aminotranspherase NADPH Nicotine amide adenine dinucleotide
phosphate restored
ATP Adenosine triphosphate NIDDM Non-Insulin dependent diabetes
mellitus
BP Blood pressure PAC Polyunsaturated aromatic
carbohydrates
bpm Beats per minute PAS Periodic acid & shiff reaction
C.I. Color Index pCO2 CO2 partial pressure
CBC Complete blood count pO2 CO2 partial pressure
CHF Chronic heart failure pm Per minute
CT Computer tomography Ps Pulse rate
DIC Disseminated intravascular coagualtion r Roentgen
DCC Doctoral controlling committee RBC Red blood count
DM-2 Non-Insulin dependent diabetes mellitus RDHA Reverse direct hemagglutination assay
DTP Anti diphtheria-tetanus vaccine Rh Rhesus
ECG Electrocardiogram (R)CFT Reiter's complement fixation test
ESR Erythrocyte sedimentation rate RIHA Reverse indirect hemagglutination
assay
FC Function class RNA Ribonucleic acid
FAD Flavin adenine dinucleotide RR Respiratory rate
FADH2 Flavin adenine dinucleotide restored S1 Heart sound 1
FEGDS Fibro-esphago-gastro-duodenoscopy S2 Heart sound 2
FMNH2 Flavin mononucleotide restored TU Tuberculin unit
GIT Gastrointestinal tract U Unit
Gy Gray USI Ultrasound investigation
GMP Guanosine monophosphate V/f Vision field
Hb Hemoglobin WBC White blood count
HbA1c Glycosylated hemoglobin X-ray Roentgenogram
Hct Hematocrit
HIV Human immunodeficiency virus
MINISTRY OF PUBLIC HEALTH OF UKRAINE

Department of human resources policy, education and science

Testing Board

Student ID Last name

Variant ________________

Test items for licensing examination

Krok 2
STOMATOLOGY
General Instruction
Every one of these numbered questions or unfinished statements in
this chapter corresponds to answers or statements endings. Choose the
answer (finished statements) that fits best and fill in the circle with the
corresponding Latin letter on the answer sheet.

ББК 54.1я73
УДК 61

Authors of items: Aksonova Ye.A., Amosova L.I., Andrianova I.I., Artiukh V.M., Avdieiev O.V.,
Babenko L.M., Bas O.A., Bedeniuk O.A., Beliaieva L.H., Bielikov O.B., Bik Ya.H., Bilous I.V.,
Bobkova S.A., Bosa L.F., Chernov D.V., Chyhrynets V.N., Chyzhevsky I.V., Denysova O.H.,
Derkach L.Z., Dmytrenko R.R., Dontsova D.O., Dubrovina O.V., Dubrovina-Parus T.A.,
Duda K.M., Eismund A.P., Fastovets O.O., Flis P.S., Furdychko A.I., Haiduk R.V., Hanchev K.S.,
Herasym L.M., Hirchak H.V., Holik V.P., Holovko N.V., Holubieva I.M., Honcharenko V.A.,
Honcharuk L.V., Hordiychuk M.O., Horlenko O.V., Hrad A.O., Hrekuliak V.V., Hrynovets V.S.,
Hubanova O.I., Hurzhiy O.V., Ivanitska O.V., Ivchenko N.A., Kalinichenko Yu.A., Kaskova L.F.,
Katurova H.F., Kharchenko O.I., Kharkov L.V., Kobryn O.P., Kolesnyk V.M., Kolomeichuk B.Ya.,
Konovalov M.F., Kosarieva L.I., Koval O.V., Kovalchuk L.P., Kril A.Y., Krupnyk N.M.,
Kryzhanivska O.O., Kuchyrka L.I., Kutelmakh O.I., Kyrychenko V.M., Kyryliuk L.M.,
Kyryliuk M.I., Larionov I.M., Larionova L.V., Lokes K.P., Lunhu V.I., Lysenko Yu.H.,
Lysiuk S.V., Malakhovska A.O., Marchenko N.V., Mazurina I.O., Mikhalova A.O., Miziuk L.V.,
Moiseitseva L.O., Morozova M.M., Muzychina H.A., Nemish T.Yu., Nesyn O.F.,
Odzhubeiska O.D., Onyshchenko S.I., Orlovsky V.O., Ostafiychuk M.O., Ozhohan Z.R.,
Palis S.Yu., Pantus A.V., Parasochkina V.V., Pasechnyk A.M., Pidlubna L.S., Piuryk V.P.,
Posolenyk L.Ya., Prodanchuk A.I., Prots H.B., Raida A.I., Riaboshapko O.A., Riznyk S.S.,
Romanenko I.H., Rozumenko O.P., Ruzin H.P., Samsonov O.V., Senchenko O.M.,
Shcherbyna I.M., Shmat S.M., Shubladze H.K., Shuvalov S.M., Shuturminsky V.H., Siry O.M.,
Stavytsky S.O., Strelchenia T.M., Svirchkov V.N., Sydorchuk S.H., Sydorenko I.V., Sydorova A.I.,
Sylenko Yu.I., Tereshyna Z.L., Tiuhashkina Ye.H., Tkachenko P.I., Tril S.I., Tsentilo V.H.,
Udod O.O., Valchuk O.H., Yatsenko I.V., Yermakova I.D., Yeroshenko A.V., Yevtushenko L.H.,
Yudina O.O., Zavoiko L.M. and Committees of professional expertise.

Item reviewers. Bezvushko E.V., Bulbyk O.I., Chyzhevsky I.V., Dmytriieva A.A.,
Fastovets O.O., Gerelyuk V.I., Ilenko N.M., Kaskova L.F., Lungu V.I., Muntian L.M.,
Novikov V.M., Ostapko O.I., Smagliuk L.V., Solovey S.I., Tril S.I., Tsentylo V.G., Volynets V.M.,
Volyak M.N.

The book includes test items for use at licensing integrated examination “Krok 2. Stomatology” and
further use in teaching.
The book has been developed for students of stomatological faculties and academic staff of higher
medical educational establishments.

Approved by Ministry of Public Health of Ukraine as examination and teaching


publication based on expert conclusions (Orders of MPH of Ukraine of
14.08.1998 №251, of 27.12.1999 №303, of 16.10.2002 №374, of 29.05.2003 №233).

© Copyright Testing Board.


Krok 2 Stomatology (англомовний варiант, iноземнi студенти) 2017 рiк 1

1. A 27-year-old man complains of teeth filled. Gradually the tooth assumed grayish
mobility in his upper and lower jaws. color. Objectively the 11 is filled, discolored,
Objectively: dentition is intact. Central stable, painless on percussion. Deep occlusi-
occlusion is determined. What examinati- on is observed. What part of the clinical
on methods should be applied? presentation contraindicates installation of
an all-porcelain crown?
A. Analysis of diagnostic models of the jaws
B. Gnathodynamometry A. Deep occlusion
C. Masticatiography B. Front teeth defects that cannot be
D. X-ray corrected with fillings
E. Electromyography C. Enamel hypoplasia with tooth deformati-
on and discoloration
2. A 47-year-old man complains of mobility D. Tooth discoloration
of the artificial crown on the 36 tooth, whi- E. Devitalized teeth defects that cannot be
ch was made 2 years ago. Objectively: the corrected with dental inlays
36 tooth is covered with full metal swaged
crown. Crown decementation and dentin 6. A 18-year-old woman needs a denture.
demineralization are observed. What is the Objectively: the 21 tooth is dark gray in
cause of such complication? color, devitalized; orthognathic occlusion is
observed. The tooth is to be covered with
A. The crown is loose at the tooth cervix plastic crown. What plastic should be used
B. The crown edge is embedded into the to make the crown?
gingival pocket
C. There are interdental contacts A. Sinma-M
D. The crown contacts with antagonistic B. Phtorax
teeth C. Protacryl-M
E. Useful life of the crown is exceeded D. Bacryl
E. Etacryl
3. A 48-year-old man complains of itchi-
ng gums. Objectively: the gums are firm; 7. A 58-year-old patient has made an appoi-
cold water causes short-time pain in the ntment to make a denture. Objectively: the
teeth. X-ray imaging shows osteosclerosis 22 tooth is firm and intact. The alveolar crest
of the alveolar septum: close-meshed bone is atrophied; the palate is flat. Removable
structure, the alveolar septum height and denture is to be made. What approach
intactness of the cortical plates is retained. regarding the 22 tooth should be chosen by
What is the most likely diagnosis? a dentist?
A. Initial periodontosis A. To make a telescopic crown
B. Atrophic gingivitis B. To retain the tooth
C. Initial periodontitis C. Removal of tooth pulp
D. Periodontitis, stage I D. Tooth extraction
E. Periodontosis, stage I E. To make a stump crown
4. A 60-year-old man complains of stabbi- 8. A patient needs his 26 tooth extracted.
ng pain near the root of the tongue on the After application of tuberal anaesthesia
right, which develops during eating, especi- he developed general fatigue, nausea, and,
ally sour food. Objectively: there is a swelli- later, severe itching and skin rashes. What
ng in the right submandibular area. On complication occurred in the patient?
palpation the submandibular gland is dense
and enlarged. Excretory duct orifice of the A. Urticaria
right submandibular gland is dilated and B. Anaphylactic shock
produces mucopurulent secretion. What is C. Collapse
the most likely diagnosis? D. Quincke’s edema
E. Unconsciousness
A. Sialolithiasis of the submandibular gland
B. Calculous sialadenitis of the sublingual 9. For a 24-year-old woman a Richmond
gland crown is being made to restore the crown
C. Acute suppurative lymphadenitis of the central maxillar incisor. The cap is
D. Acute sialodochitis completed. What is the next step?
E. Adenophlegmon of the right submandi-
bular area
5. A 19-year-old woman, an actress, complai-
ns of discoloration of her left maxillary
central incisor. One year ago the pulp of
this tooth was removed and the tooth was
Krok 2 Stomatology (англомовний варiант, iноземнi студенти) 2017 рiк 2

A. To fit the cap on the tooth stump and fever up to 38, 6o C . Objectively: there is a
place the post in the root canal soft tissue edema in the right parotid regi-
B. To solder the post with the cap on, the skin is slightly strained, without di-
C. To fit the cap and the post to the tooth scoloration. There is a dense painful infi-
root ltration 2,5x3,5 cm large, the skin over it
D. To make the combination dental crown exhibits limited mobility. The mouth can be
E. To fixate the tooth with cement fully opened, the mucous membrane around
the orifice of the salivary duct is unchanged,
10. A 25-year-old patient complains of saliva is transparent. What is the most likely
profuse gingival hemorrhages, pain in the diagnosis?
oral cavity, weakness, fatigue, fever up
to 38o C . These presentations appeared a A. Acute lymphadenitis
week ago. Objectively: the patient is pale, B. Exacerbation of chronic parotitis
adynamic. Examination of the oral mucosa C. Abscess of the parotid-masseteric region
reveals multiple hemorrhages, friable gums, D. Acute non-epidemic parotitis
necrotic areas on the tops of gingival papi- E. Epidemic parotitis
llae, as well as enlarged, soft, painless lymph
nodes. The oral mucosal lesion can be a 14. A 33-year-old woman has been admi-
symptom of the following disease: tted to the dentofacial department with
complaints of pain and edema in the right
A. Acute leukemia submandibular region, body temperature
B. Chronic leukemia rise up to 39, 5o C . Objectively: the patient
C. Vincent’s stomatitis has asymmetric face because of soft tissue
D. Hypovitaminosis C edema of the right submandibular region,
E. Intoxication with heavy metal salts palpation reveals a dense infiltration, the
skin over it is hyperemic, cannot make a
11. A 67-year-old patient complains of fold. The 46 tooth has a deep carious cavity.
recurrent erosion on the red border of What is the most likely diagnosis?
the lower lip. Objectively: the erosion is
oval in shape, 0,8х1,3 sm in size, covered A. Submandibular phlegmon on the right
in thin scabs that reveal glossy surface B. Acute submandibular sialadenitis
with punctate bleeding, when removed. C. Acute suppurative periostitis of the
There are atrophic areas of the red border mandible
detected. Infiltration elements are absent. D. Acute suppurative submandibular
The submandibular lymph nodes are not lymphadenitis
enlarged. What is the provisional diagnosis? E. Acute right-sided osteomyelitis of the
mandible
A. Manganotti’s abrasive precancerous chei-
litis 15. Having recovered from acute respiratory
B. Leukoplakia, erosive ulcerative form disease, a patient has made an appointment
C. Keratoacanthosis with a dentist. The patient complains of
D. Bowen’s disease pain in the gums, foul breath, inability to
E. Cheilitis glandularis take food, general weakness. Objectively:
the gums are hyperemic, swollen, covered
12. A 53-year-old patient complains of pain with necrotic dirty-gray coating; the gums
and clicking in the left temporomandibular bleed when the coating is removed. Microbi-
joint. Objectively: the face is symmetrical, ological study of tissues revealed a great
palpation of the lateral pterygoid muscles is number of cocci, bacilli, fusobacteria, and
painful on the left side. Mouth opening is spirochaete. Specify the drug for etiotropic
reduced. Tomography shows the bone outli- treatment:
ne of joint surfaces to be smooth. Which di-
sease of those listed below corresponds with A. Metronidazole
this clinical presentation? B. Galascorbinum
C. Potassium permanganate
A. Temporomandibular joint disfunction D. Tripsin
B. Rheumatic arthritis E. Carotolinum (Betacarotene)
C. Deforming arthrosis
D. Acute posttraumatic arthritis 16. A 35-year-old patient complains of itch,
E. Joint ankylosis burning and edema of lips. These presentati-
ons occured a week ago. Objectively: there is
13. A 22-year-old patient complains of a pai- reddening of the red border and skin, especi-
nful swelling in the right parotid gland. A ally in the area of the mouth corners, there
week earlier the patient received a cheek are also vesicles, scabs, small cracks against
abrasion that healed under the purulent the background of erythematous affection
crust. Over the past two days the patient of the red border. What is the most likely
had been observing progressing pain and diagnosis?
Krok 2 Stomatology (англомовний варiант, iноземнi студенти) 2017 рiк 3

A. Deep overbite
A. Acute eczematous cheilitis B. Open bite
B. Multiform exudative erythema C. False prognathism
C. Acute herpetic cheilitis D. True prognathism
D. Allergic contact cheilitis E. Cross bite
E. Exudative form of exfoliative cheilitis
21. A 43-year-old woman complains of
17. A 47-year-old patient presents with mobility and displacement of her upper
rounded bone protrusions 0,7-0,8 cm in si- front teeth. Objectively: dental formula is
ze on the inner surface of the edentulous 17 16 15 14 13 12 11 21 22 23 24 25 26 27
mandible in the premolar area. The denture 47 46 45 44 43 42 41 31 32 33 34 35 36 37 .
for this patient should have: Teeth 12 11 21 22 are slanted towards
the vestibular side, diastema and tremata
A. Elastic liner are observed, I-II degree teeth mobility is
B. Kemeny clasps detected. Select the orthodontic appliance
C. Metal base for correction of teeth misalignment as a
D. Orifices for the exostoses part of complex treatment of periodontal di-
E. Dentogingival clasps sease:
18. A 78-year-old patient is completely A. Palatal plate with vestibular arch
edentulous. He has been wearing dentures B. Bynin appliance
for 19 years. The patient complains of poor C. Schwartz appliance
fixation of the upper denture. Objectively: D. Katz crown
the lower third of face is shortened, the E. Palatal plate with inclined plane
alveolar processes of both jaws are markedly
atrophied, the palate is flat. Mucous 22. A 44-year-old patient consulted a dental
membrane in the denture-supporting area surgeon about constant acute pain in the
is atrophied. How often should the dentures upper jaw region on the left that aggravates
be remodelled or restored? during teeth joining. The pain appeared 3
days ago. Objectively: the face is symmetric,
A. Every 3-4 years mouth opening is not limited. The crown
B. Every 6 months of the 26 tooth is half-decayed. Probing
C. Once a year of the carious cavity is painless. Percussi-
D. Every 7 years on of the 26 tooth provokes acute pain.
E. Every 10-12 years Mucous membrane of the alveolar process
19. A 12-year-old boy complains of pai- is edematic, hyperemic at the level of the 26
nful and bleeding gums on his upper jaw. tooth. The 26 tooth had been treated before.
Objectively the gingival margin in the area What is your provisional diagnosis?
of the 13, 12, 11, 21, 22, 23 teeth is swollen, A. Exacerbation of chronic periodontitis of
hyperemic, deformed due to overgrowths. the 26 tooth
Gingival papilla cover the crowns by 1/3 of B. Acute suppurative periodontitis of the 26
their height, bleed on touch. Upper front tooth
teeth are overcrowded. X-ray shows no C. Acute pulpitis of the 26 tooth
pathological changes of the periodontium. D. Acute suppurative periostitis of the left
What drugs should be administered for topi- upper jaw extending from the 26 tooth
cal treatment in the first place? E. Periodontitis of the 26, 27, and 28 teeth
A. Nonsteroidal antiinflammatory drugs 23. When a prosthodontist was preparing
B. Sclerosants the patient’s tooth, the patient had epileptic
C. Steroidal antiinflammatory drugs seizure that was subsequently terminated.
D. Keratoplastic agents What mistake had been made by the doctor?
E. Cytostatic agents
A. No inquire into the patient anamnesis
20. A girl is 8 years old. She complains B. No inquire into the antecedent anamnesis
of impaired mastication. Objectively: on C. No anaesthesia
examination of the oral cavity the cutting D. Crude preparation
edges of her lower incisors touch the palati- E. Did not decline the appointment
ne mucosa in the frontal area; the upper
frontal teeth overlap with the lower ones 24. Parents of a 3-year-old child report that
by full height of their crowns. On the lower the child suffers from constant pain in the
jaw the occlusal curve of the front teeth is upper front teeth. Objectively: the coronal
markedly concave. Make the provisional di- part of the 61 tooth is gray and decayed.
agnosis: Probing of the root canal orifice is painful
and accompanied by bleeding. The tooth
percussion provokes acute pain. Mucosa is
Krok 2 Stomatology (англомовний варiант, iноземнi студенти) 2017 рiк 4

hyperemic, edematic and painful. Palpation cles and erosions covered in grayish fibri-
in the region of the 61 and 62 teeth reveals a nous coating on the hyperemic and swollen
fistula. What is your provisional diagnosis? labial and buccal mucosa. Nikolsky’s sign is
negative. What is the most likely diagnosis?
A. Exacerbation of chronic periodontitis
B. Acute suppurative periodontitis A. Erythema multiforme
C. Acute diffuse pulpitis B. Pemphigus vulgaris
D. Chronic granulating periodontitis C. Acute herpetic stomatitis
E. Exacerbation of chronic pulpitis D. Nonacantholytic pemphigus
E. Dermatitis herpetiformis (Duhring’s
25. A 30-year-old patient complains of disease)
a toothache caused by hot and cold sti-
muli. The pain irradiates to the ear and 29. A 56-year-old man complains of
temple. Previously this tooth presented with enlarged lower lip, pain induced by hot,
spontaneous nocturnal toothache. Objecti- sour, salty, and bitter foods, and lips glui-
vely: on the occlusal surface of the 37 tooth ng together in the morning. The lower lip
there is a deep carious cavity communicating has been gradually enlarging and developi-
at one point with the tooth cavity. Probing ng lumps his whole life. On examination:
at the communication point, as well as cold the lower lip is enlarged. The middle thi-
stimulus, causes acute pain. The pain persi- rd of the Klein’s zone presents with several
sts for a long time. Electric pulp test result small red dots with openings that discharge
is 5 microamperes. What is the most likely clear drops. The red border is dry and peeli-
diagnosis? ng, there are fissures and erosions. What is
the most likely diagnosis?
A. Exacerbation of chronic pulpitis
B. Acute diffuse pulpitis A. Cheilitis glandularis
C. Exacerbation of chronic periodontitis B. Cheilitis exfoliativa
D. Chronic concrementous pulpitis C. Cheilitis actinica
E. Acute suppurative pulpitis D. Allergic contact cheilitis
E. Atopic cheilitis
26. A 52-year-old woman complains of peri-
odical appearance of a gingival fistula in the 30. A 57-year-old retired man complains
area of the 15 tooth. The tooth had been of attacks of burning pain and rashes on
treated 1,5 years ago for caries. Objecti- the skin of his face and oral mucosa on
vely: the 15 tooth is filled. In the root apex the right. Anamnesis: a course of radiati-
projection there is a fistula; purulent exudate on therapy for treatment of the gastric di-
discharges on pressure. Tooth percussion is sease, past case of chickenpox. Objecti-
painless. On X-ray: the root canal is not fi- vely: along the third branch of the trigemi-
lled, there is a destruction focus with blurred nal nerve the skin of the face presents wi-
margins near the root. Make the diagnosis: th isolated erosions covered in fibrinous
coating. There are multiple vesicles on the
A. Chronic granulating periodontitis hyperemic and swollen oral mucosa. Right-
B. Exacerbation of chronic granulating sided lymphadenitis is observed. What di-
periodontitis agnosis is the most likely?
C. Chronic fibrous periodontitis
D. Periapical cyst A. Herpes zoster
E. Chronic granulomatous periodontitis B. Neuralgia
C. Murrain
27. A man was diagnosed with hard palate D. Acute recurrent herpes
abscess. What approach should be chosen E. Neuritis
for abscess dissection?
31. A 42-year-old woman complains of acute
A. Triangular dissection of the hard palate pain in her lower jaw, teeth mobility, high
area fever. The condition persists for 2 days. On
B. Linear dissection parallel to the hard clinical examination a doctor diagnosed her
palate raphe with acute mandibular osteomyelitis. What
C. Linear dissection perpendicular to the tactics regarding the mobile teeth should the
hard palate raphe doctor choose?
D. Pus aspiration with a syringe
E. Abscess puncture
28. A 32-year-old patient presents with
body temperature of 38, 9o C , general fati-
gue, impaired speech, inability to eat. This
condition has been recurring for the last 4
years in autumn and spring. There are vesi-
Krok 2 Stomatology (англомовний варiант, iноземнi студенти) 2017 рiк 5

A. Extraction of the causative tooth only A. Prognathism, distal mandibular di-


B. Extraction of all mobile teeth splacement
C. Conservative treatment of the causative B. Prognathism, maxillary macrognathia
tooth C. Retrusion of the lower jaw frontal area
D. Conservative treatment of all mobile teeth D. Progenia, mandibular macrognathia
E. Extraction of the causative tooth, E. Protrusion of the upper jaw frontal area
trepanation and treatment of all mobile
teeth 36. A 10-month-old child is fussy, refuses
to eat. Disease onset was 2 days ago. The
32. A 7-year-old boy complains of increased child is been treated by a pediatrician for
body temperature, up to 38o C , headache, pneumonia, receives antibiotics and sulfani-
sore throat. Objectively: there are erosions lamides. Objectively: the oral mucosa is
on the slightly hyperemic mucosa of the hyperemic, swollen; there is whitish coati-
soft palate, anterior palatal bars, and tonsils. ng on the mucosa of the cheeks, lips, soft
The submandibular lymph nodes are slightly and hard palate; coating removal can cause
enlarged, painless. Name the causative agent erosions. Submandibular lymph nodes are
of this disease: enlarged. What is the most likely diagnosis?
A. Coxsackie virus A. Acute candidal stomatitis
B. Herpes simplex virus B. Acute herpetic stomatitis
C. Epstein-Barr virus C. Geographic tongue
D. Klebs-Loeffler bacillus D. Allergic contact stomatitis
E. Bordet-Gengou bacillus E. Chronic candidal stomatitis
33. A 6-year-old boy complains of a cavi- 37. A 20-year-old man complains of sharp
ty in the prevoiusly treated tooth. Objecti- pain in the mouth, increase of body
vely: in the 85 tooth there is a carious cavity temeperature up to 38, 5o C , headache and
within mantle dentin; the dentin is dense aching joints, general weakness. The disease
and pigmented; probing of the cavilty floor onset was 3 days ago due to overexposure to
and walls, thermal stimuli, and percussi- cold. Objectively: the red border is covered
on are painless. Sensitivity of the dentin- with hemorrhagic scabs, oral mucosa has
enamel junction is observed during the cavi- large erosions and ulcers merging with
ty preparation. Make the diagnosis: each other and covered with grayish-white
coating against the background of diffuse
A. Chronic median caries hyperemia. Conjunctivitis is observed. The
B. Chronic deep caries skin of the forearms has erythematous spots
C. Acute median caries 1,5 cm in diameter, with blisters in their
D. Acute deep caries center. What is the most likely diagnosis?
E. Chronic superficial caries
A. Stevens-Johnson syndrome
34. A patient consulted a dentist about a B. Erythema multiforme
cosmetic defect in the cervical region of C. Drug-induced stomatitis
the upper and lower canines. Various sti- D. Lyell’s syndrome
muli cause no pain. Objectively: there are E. Behcet’s syndrome
V-shaped defects on the vestibular surface
in the cervical area of the upper and lower 38. Parents of a 12-year-old child are
canines. Their surface is smooth, glossy, concerned about the child having white
hard. There is no reaction to probing and spots on the frontal teeth of the upper jaw;
cold stimuli. What treatment should be the spots appeared half a year ago. Objecti-
administered? vely: there are chalky spots detected in the
cervical zone vestibular surfaces of the 13,
A. Filling of the defects 12, 11, 21, 22, and 23 teeth. The enamel in
B. Metal crowns those spots is dull; probing revealed it to
C. Applications with 10% solution of calcium be pliant and coarse. The anamnesis states
gluconate short-time pain caused by chemical stimuli.
D. Application of fluorine lacquer What is the provisional diagnosis?
E. Medical intervention is unnecessary
A. Acute initial caries
35. A boy is 10 years old. He complains B. Chronic initial caries
of sloped chin and impaired mastication. C. Acute superficial caries
Anamnesis states formula feeding. Objecti- D. Systemic hypoplasia of enamel
vely: corellation of the 6th teeth is of the E. Dental fluorosis
Angle’s II class. Sagittal fissure is 7 mm.
Eschler-Bittner test is positive. What is the 39. A 22-year-old woman complains of pain
most likely diagnosis? caused by hot food and bursting sensation
in the tooth. Half a year ago she presented
Krok 2 Stomatology (англомовний варiант, iноземнi студенти) 2017 рiк 6

with brief bouts of pain in the night, whi- The rest of her teeth present with the I-II
ch over time increased in duration. Objecti- degree of mobility. Generalized periodonti-
vely: there is a large carious cavity in the tis is observed. What denture construction
24 tooth, which opens to the dental cavity, would be optimal in this case?
deep probing is painful. Electric pulp test
is 80 microamperes. What is the most likely A. Removable dental splint
diagnosis? B. Clasp-retained (bugel) removable partial
denture
A. Chronic gangrenous pulpitis C. Removable laminar denture
B. Chronic concrementous pulpitis D. Fixed dental bridge
C. Chronic hypertrophic pulpitis E. Metal-based denture
D. Acute suppurative pulpitis
E. Chronic fibrous pulpitis 44. A 55-year-old patient requires a denture.
Objectively: Kennedy’s I class dentition
40. A 14-year-old girl complains of bleedi- defect; the 16, 17, 18, 26, 27, and 28 teeth
ng gums and foul smell from her mouth. are missing. The patient presents with fi-
Objectively: gingival mucosa is hyperemic, xed occlusion. The 15 and 25 teeth have
pastose, hemorrhaging. Schiller-Pisarev test low crowns with poor anatomic contours,
is positive. Papillary marginal alveolar index intact. Clasp-retained (bugel) removable
is 70%. Fedorov-Volodkina Hygiene Index partial denture is being made for the pati-
equals 3. X-ray of the frontal area of jaws ent. What fixation system would be optimal
demonstrates retained cortical plate. Make in this case?
the diagnosis:
A. Telescopic fixation
A. Chronic generalized catarrhal gingivitis B. Attachments
B. Chronic generalized periodontitis C. Roach clasp (clammer)
C. Acute generalized catarrhal gingivitis D. Aker-Roach combined clasp (clammer)
D. Chronic generalized hypertrophic gingivi- E. Continuous clasp (clammer)
tis
E. Exacerbation of chronic generalized 45. A patient needs a clasp-retained (bugel)
periodontitis removable partial denture. It is planned
to study the jaw model by means of a
41. A 40-year-old man had his root canal parallelometer in order to determine the
of the 34 tooth filled due to chronic fi- required depth of the undercuts on the
brous periodontitis. Soon the treated place abutment teeth. Specify the length of the
became painful. On X-ray the root canal of measuring rods used for this purpose:
the 34 tooth is filled to the root apex. What
tactics should the dentist choose to manage A. 0,25 0,50 0,75
the pain? B. 0,15 0,40 0,65
C. 0,20 0,45 0,70
A. To prescribe physiotherapeutic procedures D. 0,30 0,55 0,80
B. To rinse with antiseptic mouthwash E. 0,35 0,60 0,85
C. To make insicion along the mucogingival
fold 46. In a prostodontic clinic a partial lami-
D. To provide conduction anesthesia nar denture for the upper jaw is being made
E. To provide infiltration anesthesia for a 53-year-old patient. Objectively: dental
formula is 14, 13, 12, 11, 21, 22, 23, 24, 27.
42. A 38-year-old patient with chronic The teeth are firm, clinical crowns are tall
generalized periodontitis has been referred with pronounced equator. X-ray shows no
for orthopedic treatment. Objectively: periapical changes in the periodontium of
dentitions are without gaps, the 12, 11, 21, the abutment teeth. What clasp fixation is
and 22 teeth are pulpless and exhibit I grade optimal for this patient?
mobility. The other teeth are firm. What
is the most aesthetic dental splint for the A. Planar
anterior teeth? B. Sagittal
C. Diagonal
A. Mamlok’s splint D. Transversal
B. Ring splint E. Point
C. Soldered combined crowns
D. Cap splint 47. A 20-year-old man complains of missi-
E. Mouthguard ng tooth on the upper right jaw, aesthetic
defect. Objectively: the 12 tooth is absent,
43. The 40-year-old woman complains of adjacent teeth are intact, firm, with disti-
inability to properly masticate due to the nct anatomical shape and tall crowns. Direct
loss of the following lateral teeth: 18, 16, occlusion is observed. During the interview
15, 25, 26, 28, 38, 35, 36, 44-46, and 48. the patient was found out to have congeni-
Krok 2 Stomatology (англомовний варiант, iноземнi студенти) 2017 рiк 7

tal heart disease. What denture construction region. The poorly circumscribed, painful
would be optimal in this case? formation infiltrates the surrounding tissues.
At the right side of neck in front and behi-
A. Adgesive dental bridge nd the sternocleidomastoid muscle there are
B. Plastic dental bridge enlarged, dense, mobile lymph nodes. The
C. Porcelain-fused-to-metal dental bridge right naso-buccal groove is flattened, the
with 14 and 13 abutment teeth corner of the mouth is downturned. The
D. Plastic-fused-to-metal dental bridge mouth opens freely. The are pronounced
E. Swaged-soldered dental bridge symptoms of the right facial nerve paresis.
What disease can be suspected?
48. The 15 tooth must be extracted. The
tooth crown is retained. What instrument A. Adenocarcinoma of the parotid salivary
should be used in this case? gland
B. Chronic parotitis
A. Forceps with S-shaped handles C. Actinomycosis of the parotid-masseteric
B. Straight forceps region
C. Bayonet forceps D. Chronic lymphadenitis
D. Left-sided forceps with S-shaped handles E. Pleomorphic adenoma of the parotid
E. Right-sided forceps with S-shaped handles gland
49. A 7-year-old child complains of pain and 52. A 57-year-old woman came to a denti-
swelling in the left submandibular region. st for extraction of the 34 tooth due to
The swelling in this region developed 2 days exacerbation of chronic periodontitis. What
ago. Objectively: the child is in a satisfactory instrument would be optimal for tooth
condition, body temperature is of 37, 3o C . extraction in the given case?
Face is asymmetrical due to the soft tissue
swelling in the left submandibular region. A. Beak-shaped non-crushing forceps
Palpation reveals a round formation 2x2 cm B. Beak-shaped crushing forceps
in size. The formation is mobile, painful, C. Beak-shaped curved forceps
unattached to the skin. The 74 tooth is di- D. Straight elevator
scolored, percussion is painful. What is the E. Curved elevators
provisional diagnosis?
53. A 35-year-old patient has been di-
A. Acute serous odontogenic lymphadenitis agnosed with chronic median caries of the
of the left submandibular region 36 tooth. There is a Black’s class II cavity
B. Acute serous nonodontogenic affecting masticatory surface. What material
lymphadenitis of the left submandibular should be chosen for the tooth filling?
region
C. Acute suppurative odontogenic A. Light-cure microhybrid composite
lymphadenitis of the left submandibular B. Glass ionomer cement
region C. Silicophosphate cement
D. Phlegmonous adenitis of the right D. Light-cure fluid composite
submandibular region E. Light-cure microfilled composite
E. Lateral cervical cyst
54. A cast clasp-retained (bugel) removable
50. A 32-year-old woman complains of partial denture is being made for a 58-year-
tumor-like growth in the mucosa of her left old patient. Impressions are made, centric
cheek. Locally: buccal mucosa is of normal jaw relation is determined, plaster casts are
color. In the distal area there is a rounded obtained. What is the next stage?
elongated growth, soft and elastic, attached
to a pedicle sized 0,5х1,5 cm. Make the A. Examination of the working model with a
provisional diagnosis: parallelometer
B. Transfer of denture frame pattern to the
A. Papilloma working model
B. Lipoma C. Wax modelling of the denture frame
C. Hemangioma D. Duplication of the working model
D. Pleomorphic adenoma E. Marking the border seal
E. Fibroma
55. A 13-year-old boy complains of general
51. A 52-year-old patient complains of pain weakness, high body temperature up to
and swelling in the right parotid region. 39o C , lack of appetite, constant pain in the
These manifestations have been present body of the lower jaw. Objectively: observed
for about 2 years. Over the last month the is significant asymmetry of the face caused
swelling has enlarged, pain has intensifi- by soft tissues swelling in the left buccal
ed. Objectively: the face is asymmetric due and submandibular areas. Mouth opening
to the dense infiltrate in the right parotid is reduced. Intraoral examination revealed
Krok 2 Stomatology (англомовний варiант, iноземнi студенти) 2017 рiк 8

the following: the 34, 35, 36, and 37 teeth Cytological analysis revealed keratinizing
are mobile; teeth percussion is painful. epithelial cells. What is the most likely di-
The crown of the 36 tooth is completely agnosis?
destroyed. The mucosa of those teeth is
hyperemic and painful when palpated. Muff- A. Leukoplakia, erosive form
like enlargement of the lower jaw alveolar B. Lichen ruber planus, erosive form
process is detected. What is the most likely C. Erythema multiforme
diagnosis? D. Secondary syphilis
E. Lupus erythematosus, erosive form
A. Acute mandibular odontogenic
osteomyelitis 60. A 28-year-old man complains of pain in
B. Acute mandibular hematogenous the infraorbital and parotid region on the
osteomyelitis left. On examination: hemorrhage occurs in
C. Acute mandibular odontogenic suppurati- the lower eyelid and conjunctiva of the left
ve periostitis eye, there are signs of crepitation and step
D. Ewing’s sarcoma deformity of the eyesocket lower edge. The
E. Abscess of the right submandibular area mouth opens by 1 cm. Make the diagnosis:
56. What denture constructions should be A. Zygomatic bone fracture
chosen in the cases of multiple adentia duri- B. Malar arch fracture
ng the initial period of occlusion change? C. Left articular process fracture
D. Traumatic arthritis of the temporomandi-
A. Removable partial denture bular joint
B. Dental bridge E. Hematoma of the infraorbital region
C. Clasp-retained (bugel) removable partial
denture 61. A child is 8 years old. There are complai-
D. No denture is necessary nts of congested upper incisors. Objectively:
E. Removable complete denture the first molars closure is of Angle’s I class,
frontal overbite is orthognathic. The 12 and
57. On objective examination a 59-year-old 22 teeth erupt palatinally with space defici-
man with the edentulous mandible presents ency of 2/3 of the tooth crown. The 11 and 21
with bone protrusions and mobile areas of teeth are 10 mm each in cross-section. The
the alveolar crest. To ensure proper fixation child has inherited father’s facial type with
of the denture and even load distribution prognathism and macrodontia of the central
the following functional impression should incisors. Choose the preventive treatment,
be made: considering this hereditary pathology:
A. Differentiated A. Hotz serial extraction to reduce the dental
B. Complete anatomical arch
C. Compression B. Jaw expansion to provide the space for the
D. Decompression 12 and 21 teeth
E. Combined C. Massage of the 12 and 21 teeth area to
stimulate their eruption
58. A 49-year-old woman complains of D. Extraction of the 12 and 21 teeth to reduce
cosmetic defect of the 11, 21, and 22 teeth, the dental arch
which developed over a year ago. Objecti- E. Filing down of the 11 and 21 approximal
vely: on the vestibular surface at the equator surfaces to provide the space for the 12 and
of the 11, 21, and 22 teeth there are shallow 22 teeth
cup-shaped enamel defects that are dense on
probing. Cold water induces no pain. Make 62. A 65-year-old woman complains of a
the provisional diagnosis: neoplasm in the area of the nasolabial fold
on the left, which appeared one month ago.
A. Enamel erosion Objectively: there is a gray neoplasm on
B. Cuneiform defect the skin of the nasolabial fold on the left,
C. Superficial caries markedly keratotic, 3,0х0,5х0,3 cm in si-
D. Hypoplasia ze. Neoplastic base is painless, dense, and
E. Fluorosis elastic. What is the most likely pathology
that results in such clinical presentation?
59. A 47-year-old patient complains of a
burning sensation and pain in the mouth. A. Cutaneous horn of the left nasolabial fold
Objectively: on the mucous membrane of B. Common wart of the left nasolabial fold
cheeks along the line of teeth contact and in C. Senile keratosis of the left nasolabial fold
the corners of the mouth there are multi- D. Keratoacanthoma of the left nasolabial
ple polygonal bright red erosions 1,0-1,5 fold
cm in diameter located on the hyperkerati- E. Lupus
nized plaque and opaque whitish mucosa.
Krok 2 Stomatology (англомовний варiант, iноземнi студенти) 2017 рiк 9

63. A 55-year-old patient consulted a denti-


st about a rounded tumor-like formation A. Treatment and prevention
about 1 cm in diameter located within the B. Prevention
red border of his lower lip. Objectively: the C. Retention
tumor-like formation protrudes about 5 mm D. Passive
above the red border, is dense and grayish- E. Treatment
red. The surface of the formation is covered
with thin scales that can hardly be removed. 67. A 40-year-old man, a chemical industry
What is the most likely diagnosis? worker, notes the development of sour taste
of the mouth, pain response to thermal and
A. Verrucous precancer of the red border chemical stimuli. On examination: on the
B. Manganotti’s abrasive precancerous chei- vestibular surface and cutting edge of the
litis front teeth there are enamel defects wi-
C. Precancerous limited hyperkeratosis of th uneven scalloped margins, chalk-like in
the red border color. Make the diagnosis:
D. Bowen’s disease
E. Erythroplasia of Queyrat A. Acidic necrosis of enamel
B. Superficial caries
64. A 38-year-old woman complains of C. Enamel hypoplasia (erosive form)
burning pain in her lips and angles of her D. Fluorosis (erosive form)
mouth, their dryness. Anamnesis states E. Median caries
that she has been suffering from diabetes
mellitus for the last 8 years. Objecti- 68. A patient with post-resection upper jaw
vely: the red border is dry, congestively defect that invades the nasal cavity has come
hyperemic, covered in scales of variable si- to a prosthodontic clinic. What denture is
ze. In the angles of the mouth there are recommended in the given case?
fissures covered in white coating, the skin
is macerated. What ointment should be A. Replacement denture with obturating
prescribed for topical treatment in the gi- element
ven case? B. Floating obturator
C. Mouthguard
A. Clotrimazol D. Forming denture
B. Interferon E. Replacement denture
C. Prednisolone
D. Lanolin 69. An injured patient complains of reduced
E. Erythromycin opening of the mouth, nose bleeding, skin
numbness in the infraorbital and lower
65. A 35-year-old woman complains of lips eyelid region. Objectively: there is face
enlargement. The first incident occurred one deformation due to the depression of soft
year ago, when she developed lip edema that tissues in the left cheekbone region, step
abated quickly, but the lips remained slightly deformity in the middle part of the inferi-
enlarged. Three days ago after overexposure or margin of the left orbit and in the area
to cold her lips enlarged again. Objectively: of the zygomatic alveolar crest. What is the
ptosis, upper and lower lips are markedly most likely diagnosis?
enlarged, more on the left, soft, elastic, and
painless on palpation; no impressions on the A. Zygomatic bone fracture with di-
lip surface are left after pressing it with a splacement of the bone fragments
finger. The tongue is swollen, with tuberous B. Fracture of the right zygomatic bone
surface and folds on its back. What is the without displacement of the bone fragments
most likely diagnosis? C. Le Fort I fracture of maxilla
D. Le Fort II fracture of maxilla
A. Melkersson-Rosenthal syndrome E. Fracture of the malar arch
B. Miescher granulomatous cheilitis
C. Quincke’s edema 70. A 30-year-old woman came to a dentist
D. Achard’s syndrome with complaints of a slightly bleeding ”sore”
E. Meige’s trophedema on her lower lip, which is located at its medi-
an and bisects the lip into two even parts. On
66. A 5-year-old child has bad habit of sucki- palpation the lip is swollen and slightly pai-
ng on his tongue. At the front area there is nful. What diagnosis corresponds with the
a small vertical fissure up to 2 mm in size. given clinical presentation?
Neutral closure is observed in the lateral
areas of the jaws. The child is diagnosed wi-
th open traumatic bite of the I degree. A
vestibulo-buccal shield was prescribed for
treatment. What is the function of the appli-
ance in the given case?
Krok 2 Stomatology (англомовний варiант, iноземнi студенти) 2017 рiк 10

A. Chronic labial fissure st. There are complaints of their child havi-
B. Tappeiner’s leukoplakia ng traumas of oral mucosa. Objectively:
C. Erosive-ulcerative leukoplakia decreased height of the lower face, everted
D. Lichen ruber planus, erosive-ulcerative lower lip, deep labiomental furrow, milk
form occlusion. The upper incisors fully cover the
E. Meteorological cheilitis lower ones; cutting surface of the lower inci-
sors make contact with the anterior third of
71. A 30-year-old patient needs to have his the palate. Mesiodistal ratio of the canines
26 tooth extracted because of exacerbati- and the first permanent molars is normal.
on of chronic periodontitis. Objectively: the Grouping of the upper and lower front teeth
crown of the 26 tooth is decayed by 1/3. is dissimilar. Make the diagnosis according
What forceps can be used for this tooth to the Kalvelis classification:
extraction?
A. Deep traumatic overbite
A. S-shaped forceps with a projecting tip on B. Deep incisor overbite
the left beak C. Deep neutral occlusion
B. S-shaped forceps with a projecting tip on D. Deep prognatic (roof-shaped) occlusion
the right beak E. -
C. Straight forceps
D. Straight elevator 76. A 46-year-old patient complains of
E. S-shaped forceps without projecting tips mastication disorder caused by the lack of
the 34, 35, and 36 teeth. The antecedent
72. A 62-year-old patient came to a dental anamnesis is as follows: the teeth were
clinic with complaints of facial swelling, pain extracted 3 months ago due to complication
in the lower left jaw, and numb lower lip. On of cariosity. The patient anamnesis: the hi-
clinical examination he was diagnosed with story of tonsillitis, rheumatoid arthritis and
fracture of the body of mandible on the left, Botkin’s disease. After the appointment wi-
edentulous jaws, microstomia. Choose the th this patient the instruments should be
optimal construction: sterilized in the following way:
A. Limberg’s dental splint A. Specialized procedure
B. Weber’s dental splint B. Dry-heat sterilizer
C. Guning-Port’s dental splint C. Processing with lysol
D. Elbrecht’s dental splint D. Processing with 0,1% chloramine solution
E. Vankevych dental splint E. General procedure
73. Prior to dental treatment a 13-year-old 77. Preventive examination of a 5-year-old
patient had been administered anaesthesia. child revealed a habit of lower lip biting.
The patient complained of itching, tingli- What malocclusion may develop if the child
ng skin of the face, vertigo, nausea, labored keeps this habit?
respiration, spontaneous vision impairment.
Objectively: pale face, swollen eyelids and A. Anterior bite
red border, dilated pupils, thready pulse, B. Prognathic bite
and rapid labored respiration with crackles. C. Open bite
Make the diagnosis: D. Deep overbite
E. Cross-bite
A. Anaphylactic shock
B. Syncope 78. A 7-year-old child has protruding chin,
C. Collapse the lower lip overlaps the upper one. There
D. Epileptic attack are diastema and tremata between the lower
E. Quincke’s edema incisors, the lower incisors overlap the upper
incisors by 2/3 of the crown height. Fi-
74. A 48-year-old patient came to a denti- rst permanent molars demonstrate Angle’s
st after the maxillectomy on one side class III relation. Sagittal gap is 3 mm. The
conducted 3 days ago. Remaining teeth are correct treatment tactics would be to:
firm. Treatment plan foresees making an
Oxman’s denture for the patient. What part A. Use Bruckl’s appliance
of the denture should be produced first? B. Recommend a complex of myogymnastic
exercises
A. Fixating C. Use Angle’s slider appliance
B. Obturating D. Use Bynin’s appliance
C. Resection E. Use Schwartz’s appliance
D. Forming
E. Substituting 79. A 62-year-old man had been wearing
a full removable upper jaw denture. He
75. Parents of an 8-year-old child have complains of inability to use this denture
made and appointment with an orthodonti-
Krok 2 Stomatology (англомовний варiант, iноземнi студенти) 2017 рiк 11

due to constant burning sensation in the occlusion. X-ray shows a median mandi-
hard palate and dryness of the oral cavi- bular fracture. What dental splint would be
ty. Objectively: on the hard palate under optimal?
the denture base there are an edema
and marked hyperemia. What means of A. Flat occlusal splint
prevention would be optimal in the given B. Soldered splint on rings
case? C. Cap splint
D. Weber’s splint
A. Shielding of denture base E. Plastic mouthguard
B. Strict adherence to polymerization
procedure 83. The department of dentofacial surgery
C. Careful selection of impression paste admitted a patient who needs repair of a
D. Determination of risk group during post-traumatic nose wing defect up to 3,0 cm
examination in diameter. The trauma occured six months
E. Strict indications for the choice of material ago. What kind of grafting is indicated in this
clinical situation?
80. A 42-year-old woman has made
an appointment with a prosthodontic A. Grafting with chondrocutaneous flap of
office to make a denture. Objecti- the auricle
vely: dental formula is as follows: B. Grafting with local tissues of nasolabial or
18 . . . . 13 12 11 21 22 23 . . . . 28 cheek regions
48 47 46 45 44 43 42 41 31 32 33 34 35 36 37 . C. Grafting with pedicle flap of frontal and
The patient has deep occlusion; clinical buccal regions
crowns are low; equator is not pronounced. D. Grafting with tubed pedicle flap (Filatov’s
The patient suffers from epileptic seizures. flap)
What kind of denture should be prescribed E. Free grafting with dermal flap
for this patient?
84. A 25-year-old woman consulted a denti-
A. Removable partial laminar metal-based st about acute pain in her upper jaw on the
denture left. The pain occurs during eating. Objecti-
B. Dental bridge vely: on the distal approximal surface of the
C. Removable partial laminar plastic denture 26 tooth there is a cavity filled with light
with retainers (clammers) soft dentin. Probing causes slight pain along
D. Removable partial laminar denture with the dentin-enamel junction, percussion is
supporting-retaining clasps (clammers) painless. Cold water causes quickly abati-
E. Clasp-retained (bugel) removable partial ng pain. What is the most likely diagnosis?
denture A. Acute median caries
81. A 27-year-old woman complains of pai- B. Chronic median caries
nful and bleeding gums, with the signs C. Acute deep caries
aggravating during eating, and indisposition. D. Chronic fibrous pulpitis
One week ago she had a case of URTI. Wi- E. Chronic deep caries
thin the last 5 years she periodicaly presents 85. A 27-year-old woman complains of
with gingival hemorrhages. Objectively: the recurrent loss of a tooth filling in the lower
gums are bright red, markedly swollen; gi- right jaw. Objectively: in the 46 tooth on
ngival papillae are friable, bleed on the sli- the masticatory approximal surface there is
ghtest touch. There is moderate accumulati- a defect of hard tooth tissues affecting 1/3
on of dental calculus, large amount of of the tooth crown, no tooth discoloration;
soft dental deposit. Submaxillary lymph positive, quickly abating reaction to cold sti-
nodes are enlarged, painful on palpation. mulus is observed. What denture constructi-
On X-ray: osteoporosis of alveolar septa is on would be optimal in this case?
observed. What is the most likely diagnosis?
A. Dental inlay
A. Exacerbation of chronic catarrhal gingivi- B. Combined crown
tis C. Porcelain-fused-to-metal crown
B. Exacerbation of initial generalized peri- D. Plastic crown
odontitis E. Partial crown
C. Hypertrophic gingivitis, edematous form
D. Hypertrophic gingivitis, fibrous form 86. A 10,5-year-old child complains of pai-
E. Acute catarrhal gingivitis nful rash on his lips. Objectively: the red
border of the lips is swollen, hyperemic,
82. A 30-year-old man complains of pain in covered in fissures and numerous scabs of
his front lower teeth, which he attributes dried blood. The skin of the upper lip has
to a trauma to the mental region. Objecti- small blisters containing serous substance,
vely: continuous dentition, orthognathic which merge with each other in some places.
Krok 2 Stomatology (англомовний варiант, iноземнi студенти) 2017 рiк 12

Maceration and weeping skin also can be A. According to the scheme


observed, especially in the corners of the B. The impressions should be desinfected
mouth. What is the most likely diagnosis? under a quartz lamp
C. The patient should be refused appoi-
A. Exematous cheilitis ntment
B. Meteorological cheilitis D. No special precautions are required
C. Atopic cheilitis E. The orthopaedist should work in gloves
D. Actinic cheilitis and a mask
E. Exfoliative cheilitis
91. A 3-year-old child received an injury of
87. A 27-year-old man complains of achi- the upper teeth as a result of a fall. Objecti-
ng long-lasting pain in the 15 tooth during vely: crowns of the 51 and 61 teeth are
eating, especially cold food. Sometimes the embedded deep into the surrounding tissues
pain occurs when the temperature changes. with only their cutting edge visible, the gi-
Objectively: on the distal surface of the 15 ngival margin is hyperemic, edematic. What
tooth there is a cavity filled with softened is the treatment tactics?
dentin. Probing is painful. Electroexcitabili-
ty of the pulp is 35 microamperes. What is A. Tooth extraction
the most likely diagnosis? B. Monitoring
C. Reposition
A. Chronic fibrous pulpitis D. Endodontic treatment
B. Acute deep caries E. Anti-inflammatory therapy
C. Chronic deep caries
D. Hyperemia of the pulp 92. A 60-year-old patient has been undergoi-
E. Exacerbation of chronic pulpitis ng the procedure of checking the complete
removable dentures construction and fixing
88. A 56-year-old patient suffering from teeth on wax bases. The following flaws have
exacerbation of schizophrenia has been been detected: fissure between the teeth in
hospitalised in an oral in-patient department the frontal area and tubercule contact in the
with a diagnosis of the lower jaw displaced lateral area. What mistake had been made?
fracture in the area of the 34-35 teeth. What
method of treatment should be prescribed? A. Anterior occlusion was determined
instead of central one
A. Osteosynthesis B. Posterior occlusion was determined
B. One arch flat occlusal splint instead of central one
C. Dual splint C. Lateral occlusion was determined instead
D. Weber’s dental splint of central one
E. Vankevych dental splint D. Models were plastered in a wrong way in
an occluder
89. During preventive examination a 5-year- E. Swabs were crushed, when central occlusi-
old child was found to have insufficient on was being determined
physiological attrition of tubercles of the
deciduous canines. What treatment tactics 93. Dental splint is being designed in a
should a doctor choose? prosthodontics clinic for a 39-year-old pati-
ent with generalized periodontitis, II degree.
A. To shave off the retained canine tubercles Gnathodynamometer is used to measure
B. Medical examination once a month until the periodontium load resistance. What
the incisors are replaced anatomico-functional data are obtained wi-
C. Medical examination every 6 months until th this method?
the incisors are replaced
D. Medical examination every 6 months until A. Masticatory pressure
the canines are replaced B. Masticatory force
E. No medical intervention is necessary C. Periodontium pliancy
D. Masticatory muscles tone
90. A 25-year-old HIV-infected patient came E. Masticatory efficiency
to a clinic of prosthetic dentistry to have
a denture made for him. What aseptic and 94. A 45-year-old man complains of impai-
antiseptic precautions should be taken? red chewing due to pathologic lower jaw
mobility. The patient was diagnosed wi-
th false joint in the area of absent 33 and
34 teeth. On X-ray: defect of the mandi-
bular body is 0,8 cm in size. The teeth
on the fragments on both sides of the
defect are intact. What denture would be
recommended in this case?
Krok 2 Stomatology (англомовний варiант, iноземнi студенти) 2017 рiк 13

A. Oxman’s fixed dental bridge lls, fatigue, fever up to 38o C , muscle pain,
B. Tigerstedt’s flat occlusal splint sensations of dryness, burning, and pain in
C. Weber’s dental splint the oral cavity, excessive salivation, vesicles
D. Entin’s stiff head-chin strap in the interdigital folds, on the lips, oral and
E. Tigerstedt’s wire anchor splint nasal mucosa. On examination of the oral
cavity there were detected painful bright red
95. A 30-year-old patient complains of pain erosions with polycyclic contours against the
and swelling in the area of the left parotid background of inflammation. The following
salivary gland, which occurred 7 days after was observed: scabs on the lips, enlarged
he had undergone abdominal cavity surgery. tongue, impaired speech, salivation up to 4
Objectively: body temperature equals 39o C , liters per day. Make the diagnosis:
reduced mouth opening; dry mouth; when
the gland is massaged, there is purulent A. Murrain
exudate being secreted from its duct. The B. Acute herpetic stomatitis
patient can be diagnosed with the following C. Chickenpox
disease: D. Measles
E. Erythema multiforme
A. Acute non-epidemic parotitis
B. Acute epidemic parotitis 99. During examination of a 5-year-old child
C. Phlegmon of submasseteric space the orthodontist revealed no wear of teeth,
D. Parenchymatous parotitis no tremata and diastemata, orthogenic
E. Phlegmon of parotid-masseteric region occlusion. Which of the following symptoms
detected in the 5-year old child is a sign of
96. A 53-year-old patient complains of an future teeth overcrowding?
ulcer on the lateral surface of the tongue.
The ulcer appeared 6 months ago in the A. Absence of tremata and diastemata
result of a trauma caused by sharp tip of the B. Absence of wear of teeth
37 tooth metal crown. A dentist replaced C. Orthogenic occlusion
the crown with the one of better quality D. Orthognathic bite
and prescribed keratoplastic drugs. Despite E. Absence of mesial step in the region of
these measures the ulcer continues to grow. second temporary molars
Lately there has been pain during talking,
chewing, and swallowing, with occasional 100. An 8-year-old child has been clini-
irradiation to the pharynx. Objectively: on cally diagnosed with exacerbation of chronic
the lateral surface of the tongue there is periodontitis of the 84 tooth. The crown is
a painful ulcer with uneven raised dense decayed by 1/2. What is the optimal tactics
margins and lumpy floor covered with grayi- of dental treatment?
sh necrotic coating. What is the most likely
diagnosis? A. Extraction
B. Endodontic treatment
A. Cancer of the tongue lateral surface C. Endodontic treatment and drug therapy
B. Trophic ulcer D. Drug therapy
C. Traumatic ulcer E. Opening along the mucogingival fold,
D. Vincent’s necrotizing ulcerative stomatitis drug thrapy
E. Tuberculous ulcer
101. A 7-year-old child is diagnosed with
97. A 15-year-old girl complains of chronic granulating periodontitis of the 55
toothache that persists for a day and tooth. Additionally accompanying diagnosis
increases on biting. Objectively: in the 36 of rheumatic endocarditis is made. What
tooth there is a deep carious cavity non- treatment tactics should a dentist choose?
communicating with the dental cavity. No
reaction to the thermal stimuli is observed, A. Tooth extraction
probing of the carious cavity floor is painless. B. Endodontic treatment
Vertical percussion is markedly painful. Gi- C. Endodontic treatment and physical
ngival mucosa in the area of the 36 tooth is therapy
unaltered. X-ray presents with no alterati- D. Case monitoring
ons. Make the diagnosis: E. Endodontic treatment and case monitori-
ng
A. Acute serous periodontitis
B. Acute suppurative pulpitis 102. Parents of an 8-year-old child complain
C. Acute suppurative periodontitis of rashes in the child’s oral cavity. Lately the
D. Acute serous pulpitis child has been inert, refused to eat. On the
E. Exacerbation of chronic periodontitis oral mucosa there are small round erosions
with clear margins. There are vesicles with
98. A 35-year-old patient, a veterinarian, turbid content on the child’s face and scalp.
came to a dentist with complaints of chi- Make the provisional diagnosis:
Krok 2 Stomatology (англомовний варiант, iноземнi студенти) 2017 рiк 14

107. A laminar denture for the lower jaw is


A. Chickenpox being made for a 54-year-old patient. Base
B. Measles plate wax is used during the laboratory stage
C. Hypertensive-hydrocephalic syndrome for wax templates. What group of accessory
D. Infectious mononucleosis materials does such wax belong to?
E. Scarlet fever
A. Modeling
103. A 45-year-old patient is prescri- B. Abrasive
bed a dental bridge supported with C. Fixing
implants. In the process of preparati- D. Impression
on to the prosthodontic treatment there E. Forming
were intraosseous screw two-stage implants
placed in the area of the 34 and 36 teeth. 108. A man complains of short-term pain
How long is the period necessary for implant attacks caused by cold stimuli in the tooth on
integration in this case? his upper left jaw, which have been observed
for the last 3 weeks. Objective examinati-
A. 3 months on of the 25 revealed a Black’s I class cari-
B. 2 weeks ous cavity located within mantle dentin. The
C. 6 months cavity has narrow opening, its walls and floor
D. 10 months are covered with softened dentin. Probing
E. 1 year is painful along the dentin-enamel border,
percussion is painless, thermometry is pai-
104. A 1-month-old child has problems wi- nful, the pain quickly abates after removal
th breastfeeding, insufficiently gains wei- of a stimulus. Electric pulp test is 6 mi-
ght. On examination a doctor made the croamperes. Make the diagnosis:
diagnosis of abnormal attachment of the
tongue frenulum. What method of surgical A. Acute median caries
treatment should be chosen in this case? B. Acute deep caries
C. Chronic median caries
A. Transversal dissection of the frenulum D. Chronic fibrous pulpitis
B. Lingual frenulectomy E. Chronic deep caries
C. Vestibuloplasty
D. Limberg’s relocation of triangular flaps 109. A 54-year-old man complains of tooth
E. Dieffenbach’s frenuloplasty hard tissue defects on the upper and lower
jaws. Objectively: in the precervical area of
105. A 5-year-old child complains of the upper and lower premolars within the
spontaneous pain in an upper jaw tooth on external layer of dentin there are hard ti-
the right that aggravates at night and during ssue defects with surfaces meeting under
eating cold food. Objectively: the 65 tooth the angle. Surface of the defects is smooth,
has a deep cavity communicating with the glossy, and dense. What measure should be
tooth cavity. Probing is painful, percussion is taken to prevent further progression of the
painless. Cold water causes long-term pain. morbid process in the patient?
What is your provisional diagnosis?
A. To train him in efficient toothbrushing
A. Exacerbation of chronic pulpitis B. To perform functional examination of the
B. Acute periodontitis thyroid
C. Exacerbation of chronic periodontitis C. To prescribe calcium preparations intake
D. Acute serous pulpitis D. To limit sour foods in the diet
E. Acute suppurative pulpitis E. To limit sweet foods in the diet
106. A 10-year-old girl complains of sensati- 110. A 35-year-old man has been hospitali-
ons of dryness and pain in her lips, whi- zed into a dentofacial unit with complaints
ch develop in the summer. On examinati- of mobility of the 38, 37, and 36 teeth and a
on: the red border has areas of congestive fistulous tract in the socket of the extracted
hyperemia and infiltration, scales and scabs, 35 tooth. The condition has been persisti-
that, when removed, result in hemorrhagi- ng for 3 months. Insertion of a grooved
ng erosions. The skin surrounding lips is probe into the fistulous tract palpated
unaltered. No rashes are detected on the bared coarse bone fragment that easily
oral mucosa. What is the most likely di- moved under pressure. X-ray of the lower
agnosis? jaw demonstrates a focus of bone tissue
A. Actinic cheilitis, dry form destruction, with a spot of dense bone ti-
B. Actinic cheilitis, exudative form ssue 0,5х0,3 cm in size. Make the diagnosis:
C. Atopic cheilitis
D. Allergic contact cheilitis
E. Meteorological cheilitis
Krok 2 Stomatology (англомовний варiант, iноземнi студенти) 2017 рiк 15

A. Chronic osteomyelitis pain and bleeding of gums in the frontal


B. Acute osteomyelitis part of the upper jaw on the left. Two
C. Exacerbation of chronic osteomyelitis years ago, the 22 tooth was covered with
D. Chronic periostitis a porcelain-fused-to-metal crown. Objecti-
E. Actinomycosis vely: interdental papilla between the 21
and 22 tooth is hypertrophied, markedly
111. After extreme overexposure to cold a hyperemic, overlaps the crown of the 22
42-year-old patient complains of headache by 1/3 of its height, bleeds when touched.
in the left frontal lobe and the left upper Periodontal pocket between the 21 and
jaw. Objectively: the face is symmetrical; 22 tooth is 4 mm deep. Artificial crown is
left nasal meatus breathing is obstructed, located on the gingival margin. Radiography
and serous-purulent discharge is being reveals resorption of the interalveolar septa
produced; palpation of the suborbital area between the 21 and 22 tooth by 1/3 of
and further along the mucogingival fold in their height. Specify a priority action in the
the 24 and 25 teeth projection reveals sli- treatment of this patient:
ght pain. Percussion of these teeth is pai-
nless. The 24 tooth is filled. The alveolar A. Removal of the artificial crown
process mucosa has no visible alterations. B. Gingivectomy
X-ray imaging shows decreased pneumati- C. Anti-inflammatory therapy
zation of the left maxillary sinus. What is the D. Curettage of the periodontal pocket
provisional diagnosis? E. Sclerotherapy
A. Exacerbation of chronic odontogenic 115. A 45-year-old patient came to a
maxillary sinusitis prosthodontics clinic. During the objective
B. Acute periodontitis of the 24 examination the doctor checked the sagittal
C. Exacerbation of chronic periodontitis of movements of the lower jaw. What muscles
the 24 are responsible for sagittal movements of
D. Acute rhinogenous maxillary sinusitis the lower jaw?
E. Acute albuminous periostitis of the left
maxilla A. Lateral pterygoid muscles
B. Medial pterygoid muscles
112. A 8,5-year-old child is mostly healthy. C. Mandibulohyoid muscle
There is a complaint of pain in the upper D. Digastric muscle
left tooth, due to it having been physically E. Mentohyoid muscle
damaged 3 hours ago. Objectively: 1/2 of the
21 tooth crown is destroyed, the pulp is si- 116. An 18-year-old patient complains of a
gnificantly exposed, red, sharply painful and white spot on the vestibular surface of the 21
bleeding when probed. Percussion of the 21 tooth. Objectively: the white spot is located
tooth is sharply painful. Choose the optimal near the cutting edge. The spot surface is
method of the 21 tooth treatment: glossy, its size remains unaltered on drying.
Make the diagnosis:
A. Vital amputation
B. Vital extirpation A. Local hypoplasia
C. Devital amputation B. Fluorosis
D. Devital extirpation C. Initial caries
E. Biological method D. Enamel necrosis
E. Amelogenesis imperfecta
113. A 11-year-old child complains of pain
during eating food, especially hot, in the 117. A 20-year-old man complains of
lower right lateral tooth. On the masti- spontaneous pain in the 24 tooth, which
catory surface of the 46 tooth there is a arose suddenly and persists for about 15 mi-
large carious cavity filled with softened nutes. Objectively: the distal surface of the
light-brown dentin. The cavity is located wi- 24 tooth exhibits a deep carious cavity with
thin parapulpar dentin. In the projection of overhanging walls. The cavity is filled with
mesiobuccal pulp horn the carious cavity light softened dentin and communicates wi-
communicates with the pulp chamber. Deep th the tooth cavity. The cold stimulus causes
probing is painful. Electric pulp test - 60 mi- acute, slowly abating pain. Percussion causes
croamperes. Make the diagnosis: no pain response. Select the best method of
treatment:
A. Chronic gangrenous pulpitis
B. Chronic hypertrophic pulpitis A. Vital extirpation
C. Acute diffuse pulpitis B. Vital amputation
D. Chronic fibrous pulpitis C. Biological method
E. Acute focal pulpitis D. Devital amputation
E. Devital extirpation
114. A 28-year-old patient complains of
Krok 2 Stomatology (англомовний варiант, iноземнi студенти) 2017 рiк 16

118. A 25-year-old patient complains of pain ckened, Dupuytren’s symptom is observed


when biting on the 15 tooth. The pain arose on the vestibular surface of the mucogingi-
two days ago, has a constant aching nature val fold in the area of the 36 and 37 teeth.
and increased significantly over the last day. X-ray of the lower left jaw demonstrates the
Objectively: the crown of the 15 tooth is gray, rounded focus of bone tissue destruction wi-
the medial contact surface exhibits a deep th clear margins. The roots of the 36 and 37
carious cavity communicating with the tooth teeth are resorbed. Puncture yielded brown
cavity. Percussion causes acute pain, the gi- liquid. What is the most likely diagnosis?
ngival mucosa in the projection of the 25
tooth root apex is hyperemic. The regional A. Giant cell tumor of the body of mandible
lymph node is tender. Radiograph shows an B. Ameloblastoma of the body of mandible
ill-defined zone of periapical bone destructi- C. Fibrous dysplasia of bone
on. What is the most likely diagnosis? D. Cancer of the body of mandible
E. Sarcoma of the body of mandible
A. Exacerbation of chronic periodontitis
B. Acute serous periodontitis 122. A patient complains of the alveolar
C. Acute suppurative periodontitis process deformation on the left upper jaw.
D. Chronic granulating periodontitis Objectively: the crown of the 25 tooth is
E. Acute serous periodontitis, intoxication destroyed with cariosity. X-ray image of the
stage paranasal sinuses shows the left one to have
veil-like shading with clear dome-shaped
119. A 27-year-old patient has been margin. X-ray image of the crown of the
referred by a prosthodontist for endodontic 25 tooth shows absence of the periodontal
treatment of the 45 tooth. Objectively: the fissure at the the palatal root apex. What is
45 tooth crown is destroyed; the lateral the most likely diagnosis?
surface of the tongue and the buccal mucosa
have patches of grayish macerated epi- A. Radicular cyst that invaded in the maxi-
thelium slightly rising above the mucosa llary sinus
surface at the points of direct contact wi- B. Chronic rhinogenous maxillary sinusitis
th the 45 tooth. The uvula and palatal C. Chronic odontogenic maxillary sinusitis
bars are stagnant-red in color; hard palate D. Maxillary sinus mucosal cyst
presents with papulae surrounded with red E. Maxillary cancer
margin and covered in grayish epithelium.
The submandibular, cervical, supraclavi- 123. A patient with ischemic heart disease
cular, and subclavicular lymph nodes are has developed ventricular fibrillation. What
enlarged and painless. What is the provisi- is the first-priority therapeutic action?
onal diagnosis? A. Electric defibrillation
A. Secondary syphilis B. Lidocaine injection
B. Chronic recurrent aphthous stomatitis C. Adrenaline injection
C. Lupus erythematosus, patch stage D. Potassium chloride injection
D. Soft leukoplakia (leucoplakia mollis) E. Novocaine amide injection
E. Lichen ruber planus 124. A 7-year-old boy came to a dental
120. A 43-year-old patient complains of surgeon with complaints of painful
mobility and significant neck exposure of swelling of his right cheek and high
the lower front teeth. Objectively: the gums body temperature. Objectively the body
in the area of the 44, 43, 42, 41, 31, 32, 33, and temperature is 38,2o C , facial asymmetry
34 teeth are pale and cyanotic, non-bleeding. caused by the right cheek edema, no skin di-
The 42, 41, 31, and 32 teeth exhibit the I-II scoloration, skin can be pinched in a fold, no
grade mobility. The overcrowding of the 42, mouth opening reduction. In the oral cavi-
41, 31, and 32 teeth is present. The necks of ty the mucogingival fold in the area of the
the 42, 41, 31, and 32 teeth are exposed by 84 and 85 teeth is smoothed out, the fold is
1/2 of the root length, the necks of the 43 hyperemic, fluctuation is observed. The 85
and 33 teeth are exposed by 1/4. What kind tooth is destroyed by cariosity, painless on
of denture should be applied in this case? percussion. Make the diagnosis:

A. Cast removable splint


B. Kurlyandsky’s bar splint
C. Cap splint
D. Partial crown
E. Half-ring splint
121. A 35-year-old woman complains of
toothache and thickened body of the mandi-
ble. Objectively: the lower left jaw is thi-
Krok 2 Stomatology (англомовний варiант, iноземнi студенти) 2017 рiк 17

A. Acute odontogenic suppurative mandi-


bular periostitis originating from the 85 tooth A. Catarrhal gingivitis
B. Acute odontogenic albuminous mandi- B. Hypertrophic gingivitis
bular periostitis originating from the 85 tooth C. Atrophic gingivitis
C. Acute odontogenic mandibular D. Periodontitis
osteomyelitis originating from the 85 tooth E. Ulcerative gingivitis
D. Suppuration of the periapical cyst of the
85 tooth 129. A 22-year-old woman came to a dentist
E. Odontogenic abscess of the buccal area for preventive examination. During exami-
originating from the 85 tooth nation of the oral cavity the dentist detected
a defect of hard tooth tissues in the cervi-
125. A 15-year-old adolescent boy complai- cal area of the 22 tooth within mantle
ns of pain in the lower right jaw, which dentin. The dentin is dense and pigmented.
increases during chewing, and impaired No reaction to percussion and probing is
closure of the teeth. Anamnesis: 2 days ago observed. Make the diagnosis:
a trauma was received. Based on objecti-
ve examination results and X-ray data the A. Chronic median caries
patient was diagnosed with open fracture of B. Cuneiform defect
the mandible between the 45 and 46 teeth. C. Acute deep caries
Choose the method of treatment: D. Acute median caries
E. Necrosis of hard tooth tissues
A. Removable braces
B. Temporary immobilization 130. Spot X-ray is performed for a 12-year-
C. Port’s dental splint old child. The X-ray film shows the 35 tooth
D. Rudko’s appliance root to be of nornal length, its walls are
E. Ligature parallel to each other and gradually thin
out towards the apex. At the entrance the
126. A woman came to a dentist for root canal is slightly smaller in diameter
consultation. She is 4 months pregnant. than near the forming apex. The periodontal
Objectively: marginal gingiva presents with fissure is uniformly wide along the whole
dense gingival papillae, torus-shaped, oval, length of the formed part of the root. In the
enlarged up to 1/3 of the tooth crowns. Make apical area of the root it merges with growth
the provisional diagnosis: zone. What stages of root and periodonti-
um development correspond with this X-ray
A. Hypertrophic gingivitis image?
B. Catarrhal gingivitis
C. Periodontitis A. Unformed apex
D. Periodontosis B. Lengthwise growth of a root
E. Ulcerative gingivitis C. Open apical foramen
D. Unformed periodontium
127. A 53-year-old man complains of E. Fully formed root and periodontium
increased teeth sensitivity to chemical sti-
muli. Objectively: the gums are pale pi- 131. A 10-year-old boy complains of painful
nk, roots are bared by 1/3 of their length. sore in the mouth, which has been persi-
Small amount of dental deposit is observed. sting and increasing in size for 1,5 months.
The 15, 14, and 24 present with cuneiform Objectively: on the buccal mucosa there is
defects. Probing of the bared cervices and a soft shallow ulcer 2 cm in diameter wi-
defects is painful. What is the most likely di- th uneven undermined edges. The floor of
agnosis? the ulcer is tuberous, covered in yellow-
gray coating. The ulcer is surrounded with
A. Periodontosis, I degree numerous yellowish tubercles. The regional
B. Catarrhal gingivitis lymph nodes are elastic, painful, and matted
C. Periodontitis, II degree together. These symptoms are characteristic
D. Periodontitis, I degree of the following disease:
E. Ulcerative gingivitis
A. Tuberculosis
128. A 24-year-old man complains of pai- B. Lichen ruber planus
nful and bleeding gums. The condition onset C. Necrotizing ulcerative stomatitis
was 3 days ago after the patient had a case D. Cancer
of acute respiratory disease. Objectively E. Syphilis
the gingival mucosa is swollen, bright-red,
bleeds on probing, painful on palpation; ti- 132. A 47-year-old man complains of persi-
ps of the gingival papilla are rounded; soft sting dull pain in his upper right jaw.
dental deposit is observed. X-ray shows no Anamnesis states that the 13 tooth has been
changes in the bone. What is the most likely treated repeatedly to no effect. Objecti-
diagnosis? vely: oral mucosa in the area of the 13
Krok 2 Stomatology (англомовний варiант, iноземнi студенти) 2017 рiк 18

and 14 teeth is hyperemic and painless on A. Extracoronal attachments


palpation. The crown of the 13 tooth is B. Telescopic crowns
destroyed to the gingival margin. Percussi- C. Wire clasps (clammers)
on is sharply painful. What conduction D. Cast clasps (clammers)
anaesthesia should be applied to extract the E. Intracoronal attachments
13 tooth?
136. A patient diagnosed with rheumatoid
A. Infraorbital, incisor, palatal arthritis came to a dental surgeon complai-
B. Infraorbital, incisor ning of painful mouth opening characterized
C. Infraorbital, palatal, tuberal by pain on both sides of the jaw. The followi-
D. Infraorbital, palatal ng is characteristic of rheumatoid arthritis of
E. - the temporomandibular joint:
133. A 29-year-old man came to an A. Pain is observed on both sides of the
oral surgery department to extract the temporomandibular joint
38 tooth. There are complaints of pain B. Pain is observed on one side of the
and reduced mouth opening. Objectively: temporomandibular joint
body temperature is 38o C , general condi- C. Pain can be observed on one or both sides
tion is satisfactory, slight facial asymmetry of the temporomandibular joint
is observed due to soft tissue swelli- D. Clicking is observed in the joint on the
ng under the gonial angle on the left. right
Inflammatory contracture of the III degree E. Clicking is observed in the joint on the left
is observed. In the oral cavity there are
edema and hyperemia of mucosa along the 137. A 38-year-old man after a domestic acci-
pterygomandibular fold on the left. The dent complains of pain and mobility of his
submandibular lymph nodes are enlarged upper teeth, problems with eating. Objecti-
and painful on palpation. What conduction vely: soft tissues edema. The 11 and 21 teeth
anaesthesia should be applied prior to the are displaced towards the palate, mobile
extraction of the 38 tooth? (II degree), painful on percussion. Mucosa
surrounding the affected teeth is hyperemic
A. Berchet-Dubov anaesthesia and swollen. X-ray demonstrates widened
B. Mandibular, lingual periodontal fissure of the 11 and 21. Choose
C. Buccal, lingual the treatment method:
D. Infiltration
E. Tuberal A. Setting of the teeth and their fixation with
a flat occlusal splint
134. A 52-year-old man was referred to B. Extraction of the 11 and 21 teeth
an in-patient dentofacial department wi- C. Reimplantation of the 11 and 21 teeth
th complaints of pain in the submandi- D. Immobilization or mouthguard
bular area, aggravating during eating. E. Removal of tooth pulp in the 11 and 21
Anamnesis states frequent exacerbations of teeth
inflammatory processes. Main and additi-
onal investigations resulted in provisional 138. Parents complain of painfulness and
diagnosis of submandibular sialolithiasis. A mobility of the tooth of their 4-year-old chi-
concrement 1,5 cm in diameter is localized ld, which developed after the impact with
in the body of the gland. What tactics should a wooden object. Objectively: the face is
a dental surgeon choose? asymmetrical due to swollen tissues of the
upper lip. The 51 tooth is intact, with vesti-
A. Gland extirpation bular displacement and the II degree mobili-
B. Saliva-producing diet ty, gums around the 51 tooth are hyperemic.
C. Physiotherapeutic procedures What provisional diagnosis can be made?
D. Puncture biopsy
E. Extract the concrement from the gland A. Incomplete dislocation of the 51 tooth
and place a blind suture B. Complete dislocation of the 51 tooth
C. Contusion of the 51 tooth
135. A 70-year-old patient complains of D. Acute albuminous periostitis
impaired mastication. Objectively: the 33 E. Acute suppurative periostitis
and 43 teeth on the lower jaw are retai-
ned, firm, but have significantly destroyed 139. A 7-year-old child is in a grave condi-
crowns. Removable partial denture is requi- tion, teeth of the upper left jaw are pai-
red. Choose the optimal method of denture nful, body temperature is 39, 3o C . The chi-
fixation: ld is pale, adynamic; the face is asymmetri-
cal due to infiltration in the upper left jaw.
The 64 tooth is filled, painful on percussi-
on. The 63 and 65 teeth are intact, painful
on percussion. The I degree of tooth mobili-
Krok 2 Stomatology (англомовний варiант, iноземнi студенти) 2017 рiк 19

ty is observed; pus is being discharged from one month ago. In the morning the
under the marginal gingiva of the 64 tooth. crunching is more frequent and decreases
The alveolar process is deformed at its vesti- towards the evening. Objectively: the face
bular and palatine surfaces. Make the provi- is symmetrical, the skin above the joint is
sional diagnosis: unaltered, the mouth opens by 2,9 mm.
What is the most likely diagnosis in this
A. Acute odontogenic osteomyelitis case?
B. Acute albuminous periostitis
C. Acute suppurative periostitis A. Arthrosis
D. Ossification periostitis B. Acute arthritis
E. Ewing’s sarcoma C. Temporomandibular joint dislocation
D. Chronic arthritis
140. A 4-year-old child has developed E. Pain dysfunction syndrome of the
acute spontaneous pain in the tooth on the temporomandibular joint
lower right jaw, which aggravates on biti-
ng. Objectively: in the 85 tooth there is a 144. A 34-year-old man came to a dental
deep carious cavity non-penetrating to the clinic for extraction of the 26 tooth. After
dental cavity. Probing is sharply painful at application of 1,7 ml of Ultracain (Articai-
all points of the cavity floor. Painful reacti- ne) solution for local anaesthesia the pati-
on to cold water stimulus and percussion ent developed general fatigue and nausea.
is observed; mucosa surrounding the 85 is Objectively: the skin is pale, cold, cyanotic,
hyperemic. Submandibular lymphadenitis is covered in clammy sweat; BP is 60/40 mm
detected. Make the provisional diagnosis: Hg. What urgent condition did the patient
develop?
A. Acute pulpitis complicated with peri-
odontitis A. Collapse
B. Acute albuminous periostitis B. Anaphylactic shock
C. Acute serous periodontitis C. Loss of consciousness
D. Acute suppurative pulpitis D. Bronchial asthma
E. Exacerbation of chronic periodontitis E. Urticaria
141. An 18-year-old girl came to a dentist to 145. Carious cavities of the 11 and 21 teeth
check the quality of fissure sealing that had were detected during the preventive exami-
been performed one year ago. Objectively: nation of a 20-year-old patient. What materi-
the sealant is completely retained in the 17, al should be used to fill the detected caviti-
16, 26, 27, 37, and 47. No sealant was detected es?
in the 36 and 46, in the distal longitudi-
nal fissures of these teeth there is softened A. Microhybrid composite
enamel. Fedorov-Volodkina Hygiene Index B. Macrofilled composite
is 2,5. What tactics regarding the 36 and 46 C. Amalgam
should the dentist choose in this case? D. Phosphate cement
E. Plastic
A. Preventive filling
B. Repeated noninvasive sealing 146. A 14-year-old boy complains of rapid
C. Invasive sealing wearing-off of tooth crowns. Objectively:
D. Applications with fluorine-containing gel tooth crowns are worn-off by 1/3. Enamel
E. Electrophoresis of calcium-containing easily chips off and is pale gray in color.
solution Make the diagnosis:
142. A patient needs the 36 tooth extracted. A. Stainton-Capdepont syndrome
After administering anaesthesia the doctor B. Dentinogenesis imperfecta
started applying the elevator. However, C. Fluorosis
immediately after that the patient suddenly D. Systemic hypoplasia
paled, complained of dizziness, ear noise, E. Focal hypoplasia
and blackout and slid down in the chair.
What is the most likely diagnosis? 147. A patient complains of periodical gi-
ngival hemorrhages during tooth brushing
A. Unconsciousness and increased teeth sensitivity to thermal
B. Anaphylactic shock and chemical stimuli, which persist for the
C. Collapse last 6 years. On examination the gums are
D. Shock swollen and hyperemic. Periodontal pockets
E. Hypoglycemic coma are 5 mm deep with serous purulent content,
tooth cervices are bared, I degree tooth
143. A 54-year-old patient complains of mobility is observed. On X-ray: irregular
frequent crunching sound in the right resorption of of alveolar septa up to their
temporomandibular joint, which developed 1/2. What diagnosis corresponds with the gi-
Krok 2 Stomatology (англомовний варiант, iноземнi студенти) 2017 рiк 20

ven clinical presentation?


A. Acute odontogenic maxillary sinusitis
A. Exacerbation of generalized periodontitis, B. Acute rhinogenic maxillary sinusitis
II degree C. Chronic odontogenic maxillary sinusitis
B. Papillon-Lefevre syndrome D. Cyst of the maxillary sinus
C. Periodontosis, I degree E. Acute ethmoiditis
D. Exacerbation of generalized periodontitis,
I degree 152. During application of tuberal
E. Exacerbation of severe catarrhal gingivitis anaesthesia the patient developed rapidly
increasing tissue edema and reduced mouth
148. A 63-year-old man complains of pain opening. What resulted in such a condition?
in the area of maxillary mucogingival
fold caused by using a removable lami- A. Vascular trauma
nar denture. Objectively: in the area of the B. Muscle trauma during anaesthesia appli-
mucogingival fold there is a trophic ulcer wi- cation
th swollen margins and hemorrhaging floor. C. Nerve trunk trauma
Make the diagnosis: D. Intolerance to the anaesthetic
E. Anaphylactic shock
A. Denture-related stomatitis
B. Toxic chemical stomatitis 153. During application of infraorbital
C. Toxic infectious stomatitis anaesthesia the patient developed a posti-
D. Allergic contact stomatitis njection hematoma. What vessel had been
E. Greenhouse effect damaged?
149. A 64-year-old man complains of pain A. Infraorbital artery
and mobility of his front lower teeth. B. Maxillary artery
Objectively: the 43, 42, 41, and 31 teeth C. Pterygoid venous plexus
demonstrate the III degree mobility. These D. Temporal artery
teeth are planned to be extracted and E. Palatine artery
immediate-insertion denture is to be made.
When should this denture be put in place? 154. A patient came to a dental surgeon with
complaint of periodical pains in the 22 tooth.
A. On the day of the teeth extraction On X-ray examination the patient was di-
B. In 2-4 days after the teeth extraction agnosed with granulomatous periodontitis
C. In 6-8 days after the teeth extraction developed due to filling material penetrati-
D. In 3-6 days after the teeth extraction ng the space behind the root apex. What
E. In 1-2 days after the teeth extraction further treatment tactics should be chosen?
150. During the oral cavity sanation on the A. Resection of the root apex
vestibular surface of the 21 and 12 teeth in B. Extraction of the 22 tooth
the cervical area there were detected chalky C. Prescription of anaesthetics
spots. Enamel surface is dull and coarse, D. Referral to an oncologist
no reaction to thermal stimuli. Decayed- E. Recurrent endodontic treatment
Missing-Filled Index (dmft/DMFT) is 6,
Hygiene Index is 2. Mesial occlusion is 155. After the inflammatory process in the
observed. Within the first year of life the parotid area a woman developed frequent
patient had been suffering from frequent pain attacks resembling electric current in
cases of URTI and a case of chicken her face on the right. The attacks last for 15-
pox. What additional investigation methods 20 minutes. The most likely diagnosis is:
would be useful for the diagnosis-making in A. Trigeminal neuralgia
this case? B. Tympanic plexus neuralgia
A. Vital staining C. Trigeminal neuritis
B. Anamnesis data D. Exacerbation of chronic maxillary sinusi-
C. Electric pulp test tis
D. X-ray E. Exacerbation of chronic osteomyelitis
E. Stomatoscopy 156. A 24-year-old woman came to a denti-
151. A patient complains of pain and sensati- st to receive sanation. Objectively on the
on of heaviness in the left side of his face masticatory surface of the 37 tooth there
and mucous discharge from the nose. On is a deep carious cavity connected with the
examination: left cheek edema, destroyed dental cavity. The cavity probing is painless,
26 tooth. Tooth percussion is sharply painful. no reaction to thermal stimuli is observed
X-ray demonstrates shadowed left maxillary in the tooth, percussion is painless. EOD is
sinus. What disease corresponds with the gi- 108 microamperes. X-ray shows traces of fi-
ven clinical presentation? lling material in the rooth canal of the 37
tooth, periodontal fissure is enlarged and
Krok 2 Stomatology (англомовний варiант, iноземнi студенти) 2017 рiк 21

deformed. Make the diagnosis: of incisors. What preventive appliance


would be optimal in this case?
A. Chronic fibrous periodontitis of the 37
tooth A. Vestibulo-buccal shield
B. Chronic granulating periodontitis of the B. Schonherr’s standard vestibular plate
37 tooth C. Frankel’s functional regulator
C. Chronic granulomatous periodontitis of D. Janssen’s bionator
the 37 tooth E. Rudolf’s plate with loops
D. Chronic fibrous pulpitis of the 37 tooth
E. Exacerbation of chronic granulomatous 161. A 45-year-old man complains of pain
periodontitis of the 37 tooth and crepitation in the temporomandibular
joint during the movements of the lower
157. A 40-year-old man came to an admi- jaw. Objectively: the face is symmetrical, the
ssion room with an incised wound of the mouth opens with slight displacement to the
infraorbital region received 8 hours ago. left. Dentition is intact. To clarify the di-
On examination the wound underwent pri- agnosis X-ray of the temporomandibular joi-
mary surgical treatment. In case of an inci- nt was performed. Where should the heads
sed wound its edges: of the mandible be located normally during
maximum mouth opening?
A. Should not be excised
B. Should be closed with secondary sutures A. At the top of the articular tubercle
C. Should be closed with primary delayed B. In the center of the glenoid fossa
sutures C. In front of the articular tubercle
D. Should be processed with antibiotic D. In the center of the articular tubercle
solution E. Closer to the distal part of the glenoid
E. Should be cleaned and drained fossa
158. An ambulance has delivered an 8- 162. A 45-year-old man complains of pain
year-old child to an admission room. An and crepitation in the temporomandibular
oral surgeon has made the following di- joint during the movements of the lower
agnosis: odontogenic phlegmon of the ri- jaw. Objectively: the face is symmetrical,
ght submandibular area. What surgical the mouth opens with slight displacement
approach would be advicable for open to the left. Dentition is intact. On occlusi-
treatment of this phlegmon? ography there were detected centric and
eccentric supracontacts. What treatment
A. Dissection in the submandibular area, methods should be applied in the first place?
parallel to the mandible
B. Dissection parallel to the torus mandi- A. Selective teeth shaving
bularis B. Mouthguard for muscle relaxation
C. Dissection around the mandibular angle C. Appliances that limit mouth opening
D. Dissection along the lower neck fold D. Mouthguards that increase the height of
E. Dissection in the area of pterygomandi- central occlusion
bular fold E. Lower jaw immobilization
159. A 14-year-old child complains of 163. A 65-year-old man during the
throbbing undulating pain in the lower tooth extraction suddenly felt unwell, he
left teeth, which aggravates due to hot developed severe pain irradiating to the
stimuli. Objectively: on the masticatory left scapula and numbness of the left hand.
surface of the 36 tooth there is a cari- Objectively: the patient is pale, beads of
ous cavity within parapulpar dentin, whi- perspiration appeared on his forehead, BP
ch is non-communicating with the dental is 170/90 mm Hg, heart rate is 86/min.,
cavity. The cavity floor probing is painless, rhythmical. The dentist stopped the mani-
tooth percussion is painful. What treatment pulations in the oral cavity. What drug
method would be optimal in the given case? should be administered in this case?
A. Vital extirpation A. Nitroglycerine
B. Devital extirpation B. Zelenin drops
C. Devital amputation C. Valerian tincture
D. Vital amputation D. Valocordin
E. Biological method E. Analgin (Metamizole)
160. An orthodontist has registered for 164. A 45-year-old patient after admini-
regular check-ups a 3,5-year-old child, stration of local anaesthesia in preparati-
who has a bad habit of finger sucking on to oral surgery has suddenly felt unwell,
and presents with infantile swallowing. On developed increasing edema of laryngeal
examination: milk occlusion, direct contact mucosa and respiration disorder. The dentist
Krok 2 Stomatology (англомовний варiант, iноземнi студенти) 2017 рiк 22

stopped the manipulations in the oral cavi- man. At the stage of placing the artificial
ty. What type of asphyxia developed in the teeth in the dental articulator it is necessary
patient? to determine sagittal articular angle. This
angle equals:
A. Stenotic
B. Dislocational A. 20-40
C. Valvular B. 5-15
D. Obturative C. 15-20
E. Aspiration D. 20-25
E. 40-50
165. A 19-year-old young man complains of
a fistula on the neck anterior surface, whi- 169. In a new neighbourhood unit of a large
ch periodically reappears at the same place. city a dental clinic is being opened. The
Objectively: at the neck midline between clinic will employ 3 prosthodontists. How
the hyoid bone and thyroid cartilage there many positions of dental technicians, dental
is a fistula; the skin of the affected area is nurses, and orderlies should be provided?
scarred, drawn-in, and macerated. In the
surrounding tissues a dense band extending A. 6 dental technicians, 1 dental nurse, 1
from the fistula opening to hyoid bone can orderly
be palpated. A doctor has made a provisi- B. 3 dental technicians, 1 dental nurse, 1
onal diagnosis of thyroglossal fistula. Specify orderly
the additional method of investigation: C. 3 dental technicians, 1,5 position of a
dental nurse, 1 orderly
A. Contrast radiography D. 6 dental technicians, 1 dental nurse, 0,5
B. Probing position of an orderly
C. Computer tomography E. 1,5 position of a dental technician, dental
D. Ultrasound nurse, and an orderly
E. -
170. A 25-year-old man complains of short-
166. Mother of an 8-month-old girl came to term pain in the tooth on the lower ri-
a clinic with complaints of the child’s anxi- ght jaw during eating sweet, hot, and cold
ety, fussiness, high fever up to 38, 5o C , si- food. Objectively: in the 36 tooth on the
gns of alimentary canal irritation, vomiti- distal surface there is a carious cavity
ng and refusal to eat. On objective exami- non-communicating with the dental cavi-
nation the child is pale, crying, presents wi- ty, dentin is softened. Probing of the cavi-
th hyperemia, edema, gingival pain in the ty floor is painful, percussion is painless.
frontal area of the upper jaw, no erupted Electric pulp test is 16 microamperes. Make
teeth can be detected. Make the diagnosis: the final diagnosis:
A. Hindered tooth eruption A. Acute deep caries
B. Acute herpetic stomatitis B. Acute median caries
C. Food poisoning C. Hyperemia of the pulp
D. Hematogenous osteomyelitis of the maxi- D. Chronic gangrenous pulpitis
lla E. Chronic fibrous periodontitis
E. Odontogenic osteomyelitis of the maxilla
171. A 30-year-old woman complains of a
167. A 45-year-old patient complains of carious cavity in the 16 tooth, food retention
inability to properly masticate due to the in the gap between the 16 and 17 teeth. Duri-
loss of lateral teeth. The 17, 16, 15, 25, 26, ng examination there was detected a cari-
27, 37, 36, 35, 44, 45, and 46 teeth are mi- ous cavity within mantle dentin with wide
ssing. The retained teeth exhibit the I-II opening on the approximal-medial surface
degree of mobility. The patient is diagnosed of the 16 tooth. The cavity floor and walls
with generalized periodontitis. Kennedy are pigmented, dense, painless on probing.
class I dentition defects are observed. What Percussion of the 16 tooth is painless. On
construction would be optimal in the given thermodiagnostics a short-term reaction can
case? be observed. Make the diagnosis:
A. Clasp-retained (bugel) removable partial A. Chronic deep caries
denture with splinting elements B. Chronic fibrous pulpitis
B. Partial laminar denture C. Chronic median caries
C. Elbrecht’s dental splint D. Chronic fibrous periodontitis
D. Mamlok’s dental splint E. Chronic concrementous pulpitis
E. Cantilever dental bridge
172. A 40-year-old patient requires surgical
168. Removable complete laminar denture sanation of the oral cavity. Objectively: the
is being made for a 63-year-old edentulous 36 tooth is completely destroyed. Mouth can
Krok 2 Stomatology (англомовний варiант, iноземнi студенти) 2017 рiк 23

be fully opened. What anaesthesia would be superciliary area. General condition of the
optimal for extraction of the 36 tooth? child is unaffected. Objectively: swelling of
the forehead tissues spreading towards the
A. Torusal left eyelids; the swelling is soft, fluctuati-
B. Mandibular on sign is present. Make the preliminary di-
C. Mental agnosis:
D. Infiltration
E. Berchet-Dubov A. Hematoma of the left superciliary area
B. Hemangioma of the right superciliary area
173. A 45-year-old woman needs a denture. C. Fracture of the temporal bone
Objectively: the 17, 16, 15, 14, 12, 25, and D. Fracture of the frontal bone
26 teeth are missing. Specify the Kennedy’s E. Hematic abscess of the left superciliary
class of dentition defects in the given case: area
A. II class, 2 subclass 178. Mother of a 10-year-old girl complai-
B. II class, 4 subclass ns of a cosmetic defect of the child’s 22
C. III class, 1 subclass tooth that erupted with damaged enamel.
D. III class, 3 subclass Anamnesis states premature extraction of
E. II class, 3 subclass the 62 tooth due to caries complication.
There is a white-yellow spot with clear
174. A 37-year-old patient complains of an margins on the vestibular surface of the 22
aesthetic defect. Objectively: the 13 tooth is tooth. Enamel retains glossiness, no surface
destroyed by 2/3. The tooth is pulpless, the roughness can be detected on probing. Make
root canal is filled. How deep should the root the diagnosis:
canal be opened for pivot crown installation
in this patient? A. Local enamel hypoplasia
B. Fluorosis
A. 2/3 of the root canal C. Acute superficial caries
B. 1/3 of the root canal D. Chronic superficial caries
C. 3/4 of the root canal E. Systemic enamel hypoplasia
D. 1/2 of the root canal
E. Full length of the root canal 179. A 48-year-old patient has come to a
hospital with complaints of defects in the
175. A 10-year-old boy complains of acute paragingival area and slight sensitivity to
pain attacks in the area of his upper left thermal stimuli. Objectively: there are hard
teeth. The toothache persisted for a night. tissue defects that resemble a wedge with
Objective examination revealed a carious smooth polished walls on the precervical
cavity on the masticatory surface of the 26 vestibular surface of the 23 and 24 teeth.
tooth within parapulpar dentin. Probing is Thermal test is slightly positive. What is the
sharply painful at all points of the cavity most likely diagnosis?
floor. Markedly positive reaction to cold
water stimulus is observed. Select the most A. Cuneiform defect
likely diagnosis: B. Enamel necrosis
C. Acute deep caries
A. Acute diffuse pulpitis D. Enamel erosion
B. Acute serous periodontitis E. Endemic fluorosis
C. Acute suppurative pulpitis
D. Acute suppurative periodontitis 180. Parents of a 3-year-old child complain
E. Acute local pulpitis that the child has a neck growth that
developed 3 months after the birth. Objecti-
176. A 15-year-old patient complains of cari- vely: in the upper lateral neck area there is a
ous cavity and short-term ”lightning-fast” semicircular neoplasm with limited mobility,
pain attacks in the 26 tooth. The pain attacks soft elastic consistency, no skin alterations,
cease in 1-2 minutes after eating. Objecti- painless on palpation. Puncture yielded pus-
vely: there is a deep carious cavity filled with like clear yellow substance. Make the provi-
softened dentin. The cavity floor is painful sional diagnosis:
on probing. Make the diagnosis:
A. Branchial cleft cyst
A. Pulpal hyperemia B. Chronic lymphadenitis
B. Acute traumatic pulpitis C. Lymphangioma
C. Acute suppurative pulpitis D. Specific lymphadenitis
D. Acute local pulpitis E. Hemangioma
E. Acute diffuse pulpitis
181. A 16-year-old adolescent girl complai-
177. A 7-year-old girl hit her forehead one ns of pain caused by cold stimuli and food
day ago. Several hours after the sustained particles retained in her upper jaw tooth.
trauma a swelling developed in the left
Krok 2 Stomatology (англомовний варiант, iноземнi студенти) 2017 рiк 24

Objectively: on the contact surface of the 185. During preventive examination a


24 tooth there is a carious cavity within patient was diagnosed with precancerous
parapulpar dentin. The cavity floor and walls hyperkeratosis of the red border of the
are covered with light softened dentin. The lower lip. What treatment should be prescri-
carious cavity floor is sensitive to probing, bed?
percussion of the 24 is painless. Cold water
stimulus is painful, the pain quickly abates A. Surgical removal of the focus within
after the stimulus is removed. Make the di- healthy tissues
agnosis: B. Surgical removal of the focus wi-
thin healthy tissues + close-focus
A. Acute deep caries roentgenotherapy
B. Acute median caries C. No treatment is required
C. Acute diffuse pulpitis D. Surgical removal of the focus within
D. Chronic fibrous pulpitis healthy tissues + chemotherapy
E. Chronic deep caries E. Palliative treatment
182. A 15-year-old girl complains of brief 186. A 23-year-old patient is hospitalized
pain attacks in her teeth due to chemical into a dentofacial unit with provisional di-
stimuli. Objectively: on the contact surfaces agnosis of the II degree thermal burns of the
of the 11, 21, and 22 teeth there are enamel right buccal and parotid-masseter region.
areas matt white in color, with lost shine, What scar tissue will develop in this case?
covered in large amount of dental deposit.
Enamel is softened and can be easily chi- A. Healing without a scar
pped off with excavator. Probing of lesions B. Atrophic scar
is painless. Percussion is painless. No reacti- C. Hypertrophic scar
on to cold stimuli. Make the diagnosis: D. Hypotrophic scar
E. Keloid scar
A. Acute superficial caries
B. Acute median caries 187. A 56-year-old man complains of pain
C. Acute initial caries in the gonial angle. Objectively: the 45, 46,
D. Chronic initial caries 34, 35, and 36 teeth are missing; there are
E. Chronic superficial caries slight swelling and reduced mouth opening
observed. X-ray: right-sided fracture of the
183. A 32-year-old patient addressed a body of mandible in the area of the 45 and
dentist with complaints of inability to close 46 teeth without bone defect. What dental
his mouth. Objectively the mouth is half- splint should be used for treatment?
open, the chin is protruding forwards and
is displaced to the left. Such condition A. Weber
occurred after the mouth was opened wi- B. Limberg
de. What is the most likely diagnosis? C. Tigerstedt
D. Zbarzh
A. Anterior right-sided mandibular dislocati- E. Vankevych
on
B. Anterior left-sided mandibular dislocation 188. A clasp-retained (bugel) removable
C. Anterior bilateral mandibular dislocation partial denture for the lower jaw is to be
D. Posterior right-sided mandibular dislocati- made for a 53-year-old patient. Objectively:
on the 38, 37, 35, 34, 45, 46, and 47 teeth are mi-
E. Posterior left-sided mandibular dislocation ssing. The retained teeth are firm, with low
clinical crowns. What fixation method of the
184. A 37-year-old patient has symmetrical denture would be optimal in this case?
face; the mucosa in the area of the 12 tooth
root apex projection is pale pink; palpation A. Telescopic system
is painless; the tooth crown is destroyed by B. Supporting-retaining clasps (clammers)
1/3; percussion is painless. X-ray: the root C. Bar system
canal of the 12 tooth is filled to the apex; D. Attachments
granuloma 4 mm in diameter surrounds the E. Ball joint attachments
root apex. Choose the method of surgical
treatment: 189. A 28-year-old woman is diagnosed with
chronic generalized periodontitis, II degree.
A. Granuloma removal with root apex The doctor prescribed her a mouthwash with
resection chlorhexidine gluconate as a part of complex
B. Root hemisection therapy. This drug belongs to the following
C. Coronary radicular tooth separation group of antiseptics:
D. Root amputation
E. Tooth extraction
Krok 2 Stomatology (англомовний варiант, iноземнi студенти) 2017 рiк 25

A. Detergents
B. Dyes A. II
C. Halogens B. I
D. Oxidants C. III A
E. Acids and alkalis D. III B
E. IV
190. A 22-year-old patient has suffered uni-
lateral linear fracture in the area of the 194. A 19-year-old patient came to a
gonial angle. Immobilization was provi- dentofacial clinic with complaints of pain
ded with full dental brace with loops and in the gonial angle on the right, impaired
intermaxillary elastic expansion. Recovery mouth opening and painful chewing. The si-
was uncomplicated. The brace should be gns had been persisting for 5 days, emerged
removed after: spontaneously and had been aggravating
gradually. Mandibular contracture is of the
A. 3 weeks III degree. On examination of the oral cavi-
B. 2 weeks ty: hyperemia, edema of the retromolar
C. 1 week space on the right, hood-shaped mucosa
D. 10 days from under which pus is being discharged
E. - and 2 tooth tubercles can be detected. X-ray
shows oblique medial tooth position. Make
191. A 33-year-old man, a metalworker, the diagnosis:
complains of pain and itching in the gums,
gingival hemorrhages intensifying during A. Acute suppurative pericoronitis of the 48
tooth brushing. The onset of the disease tooth
was 1 year ago. Objectively: the gums in the B. Acute suppurative periostitis from the 48
area of upper and lower frontal teeth are tooth
hyperemic, swollen, and cyanotic. There are C. Chronic local mandibular osteomyelitis
significant mineralized deposits on the teeth; D. Fracture of the gonial angle
the periodontal sockets are 3 mm deep and E. Acute submandibular sialadenitis
produce small amount of serous discharge.
What is the most likely diagnosis? 195. A 45-year-old man came to a dentist
with complaints of a massive and extremely
A. Chronic generalized periodontitis, I class dense (resembling wood) infiltration in the
B. Chronic localized periodontitis, I class parotid and retromandibular areas, which
C. Exacerbation of chronic generalized persists for 1,5 months. The patient’s general
periodontitis, II class condition remains largely undisturbed, signs
D. Acute localized periodontitis, II class of inflammatory process are vague and indi-
E. Generalized periodontosis, I class stinct. Periodically in the infiltration area
the skin assumes cyanotic-purple color, a
192. A 34-year-old man complains of pain in soft patch appears in the center, where 1-2
the area of his right eye, headache, and body fistulae develop and discharge pus with whi-
temperature rise up to 38,6o C . Two days ago te granules. Periodically fistulae close and
the patient developed an infiltration in the reopen. Make the diagnosis:
lower eyelid of the right eye. Objectively
the eyelids are markedly swollen, palpebral A. Parotid actinomycosis
fissure is closed, conjunctiva is swollen. B. Parotid tuberculosis
Exophthalmos is observed. The eyeball is C. Parotid erysipelas
immobile, vision is impaired. Make the di- D. Chronic parotid sialadenitis
agnosis: E. Adenocarcinoma of the parotid gland
A. Orbital phlegmon 196. Parents of a 6-year-old child complain
B. Eyelid phlegmon of their child having a gradually enlargi-
C. Purulent maxillary sinusitis ng neoplasm in the left parotid-masticatory
D. Angular vein trombophlebitis region. Skin over the tumor is without di-
E. Lower eyelid abscess scoloration. The tumor is painless, but when
the head bends down the tumor increases in
193. A 35-year-old patient complains of size and assumes bluish coloring. What di-
burns of the face and neck, swelling and sease can be suspected in the child?
burning pain in the affected area. On
examination: edema of the face and neck, A. Hemangioma
palpebral fissure is narrowed due to swelli- B. Fibroma
ng, affected skin is hyperemic and covered C. Atheroma
with strained thin-walled blisters filled wi- D. Lymphangioma
th clear content. Where blisters are broken, E. Cyst of the parotid gland
there are pink wounds, sharply painful to
touch. Determine the degree of the burns: 197. A 40-year-old patient complains of pain
Krok 2 Stomatology (англомовний варiант, iноземнi студенти) 2017 рiк 26

in the tragus area, clicking sound during tonsils, and posterior wall of the pharynx.
mouth opening, stuffed ears. Objectively: Submandibular, submental and deep cervi-
the face is symmetrical, mouth opening path cal lymph nodes have been enlarged for
is straight. Dentition defect can be estimated 4 months. Two weeks ago the patient
as Kennedy I class; the 18, 17, 16, 26, 27, developed intermittent fever and general
and 28 teeth are missing. In this case the fatigue. Select the correct sequence of HIV
load would be the most traumatizing for the diagnosing:
following anatomical structure:
A. Enzymoimmunoassay, immunoblotting
A. Interarticular disk (Western-Blot)
B. Articular capsule B. CD4 cell count, enzymoimmunoassay
C. Articular head C. Complete blood count, viral load
D. Distal slope of the articular tubercle D. Complete blood count, enzymoi-
E. Socket floor of the temporal bone mmunoassay
E. Viral cultivation, enzymoimmunoassay
198. A 50-year-old man complains of bared
dental cervices on his upper and lower 200. A 38-year-old man complains of
jaws. Objectively: the teeth and dentition sensation of a foreign body on his tongue
are intact, clinical crowns are elongated, and development of gag reflex during talki-
the teeth have no pathologic mobility, are ng. The signs appeared after the prolonged
worn off within the physiological norm. taking of antibiotics. Objective examinati-
To remove supracontacts it is planned to on detected thickened and pigmented fi-
perform selective teeth shaving. What addi- liform papillae enlarged to 2-3 cm in si-
tional investigation is necessary in the given ze. Histologically papillar hyperplasia and
case? marked keratinization without alteration of
the surrounding tissues were detected. What
A. Occlusiography is the most likely diagnosis?
B. X-ray
C. Masticatiography A. Black hairy tongue
D. Gnathodynamometry B. Median rhomboid glossitis
E. Mastication tests C. Fissured tongue
D. Glossitis areata exfoliativa
199. A 28-year-old man presents with E. Geographic tongue
profuse caseous coating on the posterior
third of the back of his tongue, soft palate,
INSTRUCTIONAL BOOK

Ministry of public health of Ukraine (MPH of Ukraine)


Department of human recources policy, education and science
Testing Board

TEST ITEMS FOR LICENSING EXAMINATION: KROK 2. STOMATOLOGY.

Kyiv. Testing Board.


(English language).

Approved to print 03.04./№47. Paper size 60х84 1/8


Offset paper. Typeface. Times New Roman Cyr. Offset print.
Conditional print pages 24. Accounting publishing pages 28.
Issue. 297 copies.
List of abbreviations

A/G Albumin/globulin ratio HR Heart rate


A-ANON Alcoholics anonymous IDDM Insulin dependent diabetes mellitus
ACT Abdominal computed tomography IFA Immunofluorescence assay
ADP Adenosine diphosphate IHD Ischemic heart disease
ALT Alanin aminotranspherase IU International unit
AMP Adenosine monophosphate LDH Lactate dehydrogenase
AP Action potential MSEC Medical and sanitary expert committee
ARF Acute renal failure NAD Nicotine amide adenine dinucleotide
AST Aspartat aminotranspherase NADPH Nicotine amide adenine dinucleotide
phosphate restored
ATP Adenosine triphosphate NIDDM Non-Insulin dependent diabetes
mellitus
BP Blood pressure PAC Polyunsaturated aromatic
carbohydrates
bpm Beats per minute PAS Periodic acid & shiff reaction
C.I. Color Index pCO2 CO2 partial pressure
CBC Complete blood count pO2 CO2 partial pressure
CHF Chronic heart failure pm Per minute
CT Computer tomography Ps Pulse rate
DIC Disseminated intravascular coagualtion r Roentgen
DCC Doctoral controlling committee RBC Red blood count
DM-2 Non-Insulin dependent diabetes mellitus RDHA Reverse direct hemagglutination assay
DTP Anti diphtheria-tetanus vaccine Rh Rhesus
ECG Electrocardiogram (R)CFT Reiter's complement fixation test
ESR Erythrocyte sedimentation rate RIHA Reverse indirect hemagglutination
assay
FC Function class RNA Ribonucleic acid
FAD Flavin adenine dinucleotide RR Respiratory rate
FADH2 Flavin adenine dinucleotide restored S1 Heart sound 1
FEGDS Fibro-esphago-gastro-duodenoscopy S2 Heart sound 2
FMNH2 Flavin mononucleotide restored TU Tuberculin unit
GIT Gastrointestinal tract U Unit
Gy Gray USI Ultrasound investigation
GMP Guanosine monophosphate V/f Vision field
Hb Hemoglobin WBC White blood count
HbA1c Glycosylated hemoglobin X-ray Roentgenogram
Hct Hematocrit
HIV Human immunodeficiency virus
MINISTRY OF PUBLIC HEALTH OF UKRAINE

Department of human resources policy, education and science

Testing Board

Student ID Last name

Variant ________________

Test items for licensing examination

Krok 2
STOMATOLOGY
General Instruction
Every one of these numbered questions or unfinished statements in
this chapter corresponds to answers or statements endings. Choose the
answer (finished statements) that fits best and fill in the circle with the
corresponding Latin letter on the answer sheet.

ББК 54.1я73
УДК 61

Authors of items: Aksonova Ye.A., Amosova L.I., Babenko L.M., Babushkina N.S.,
Bahlyk T.V., Bas O.A., Beliaiev E.V., Bielikov O.B., Bik Ya.H., Bosa L.F., Chaikivsky R.V.,
Chernov D.V., Chumachenko V.A., Chyrkin V.I., Chyzhevsky I.V., Derkach L.Z., Dmitriyev M.O.,
Dorubets A.D., Dubrovina O.V., Dvornyk V.M., Dzetsiukh T.I., Eismund A.P., Fedorovych O.A.,
Flis P.S., Haiduk R.V., Hanchev K.S., Haranina T.S., Hembarovsky M.V., Herasym L.M.,
Hirchak H.V., Hladka O.M., Hodovanets O.I., Holik V.P., Holovko N.V., Holubieva I.M.,
Hordiychuk M.O., Hrad I.V., Hrechko N.B., Hrekuliak V.V., Huliuk A.H., Humetsky R.I.,
Hurzhiy O.V., Hrynkov Ye.I., Idashkina N.H., Ilenko N.M., Ilnytsky Ya.M., Ivchenko N.A.,
Karasiunok Ye.O., Karelina L.S., Kaskova L.F., Katurova H.F., Khalmatov B.D., Kharchenko O.I.,
Kharkov L.V., Kirsanova O.V., Klomin V.A., Konovalov M.F., Kopelian N.M., KosarievaL.I.,
Kotelevska N.V., Koval O.V., Kril A.Y., Kryzhanivska O.O., Kuz H.M., Kuchyrka L.I.,
Kyrychenko V.M., Larionov I.M., Lavrovska O.M., Levko V.P., Lokes K.P., Lunhu V.I.,
Lysiuk S.V., Lytovchenko Yu.O., Makarevych A.Yu., Malakhovska A.O., Mikhalova A.O.,
Moiseitseva L.O., Morozova M.M., Morozova N.P., Nemish T.Yu., Nesin O.F., Odzhubeiska O.D.,
Oktysiuk Yu.V., Onyshchenko S.I., Orlovsky V.O., Ozhohan Z.R., Parasochkina V.V.,
Pasechnyk A.M., Pavelko N.M., Petrushanko V.M., Polyshchuk L.F., Potiyko V.I.,
Prodanchuk A.I., Pryshko Z.R., Raida A.I., Romanenko I.H., Romankov I.O., Romashkina O.A.,
Rozumenko O.P., Ruzin H.P., Saiapina L.M., Samsonov O.V., Shakhnovsky I.V., Shcherbyna I.M.,
Shmat S.M., Shubladze H.K., Shuvalov S.M., Semenova O.O., Sidlak O.Ya., Skakun L.M.,
Skvortsova I.H., Smahliuk L.V., Stasiuk N.O., Svirchkov V.N., Sydorenko A.Yu., Sydorenko I.V.,
Sydorova A.I., Teslenko O.I., Tsilenko O.L.,Tyuhashkina Ye.H., Ushych A.H., Vasylenko V.M.,
Voliak M.N., Volkova O.S., Yermakova I.D., Yeroshenko A.V., Yevtushenko L.H., Zinchenko T.P.
and Committees of professional expertise.

Item reviewers. Bezvushko E.V., Bulbyk O.I., Chyzhevsky I.V., Dmytriieva A.A.,
Fastovets O.O., Gerelyuk V.I., Ilenko N.M., Kaskova L.F., Lungu V.I., Muntian L.M.,
Novikov V.M., Ostapko O.I., Smagliuk L.V., Solovey S.I., Tril S.I., Tsentylo V.G., Volynets V.M.,
Volyak M.N.

The book includes test items for use at licensing integrated examination “Krok 2. Stomatology” and
further use in teaching.
The book has been developed for students of stomatological faculties and academic staff of higher
medical educational establishments.

Approved by Ministry of Public Health of Ukraine as examination and teaching


publication based on expert conclusions (Orders of MPH of Ukraine of
14.08.1998 №251, of 27.12.1999 №303, of 16.10.2002 №374, of 29.05.2003 №233).

© Copyright Testing Board.


Krok 2 Stomatology (англомовний варiант, iноземнi студенти) 2018 рiк 1

1. A removable full denture for the lower A. Deep occlusion


jaw is being made for a 75-year-old man. B. Front teeth defects that cannot be corrected
Objectively the alveolar process is slightly with fillings
atrophied. Herbst tests are performed during C. Enamel hypoplasia with tooth deformation
fitting of an impression tray. When lips are and discoloration
stretched forwards the tray slips off. Where D. Tooth discoloration
should the tray edge be shortened in this E. Devitalized teeth defects that cannot be
case? corrected with dental inlays
A. From canine to canine on the vestibular 5. A 32-year-old woman needs a denture. On
side objective examination the decision was made
B. From canine to canine on the lingual side in favor of porcelain-fused-to-metal crown.
C. From behind the mandibular tuberosity to What material should be used in this case to
the mylohyoid line obtain the impression?
D. Along the mylohyoid line
E. In the premolar area on the lingual side A. Stomaflex
B. Repin
2. A 18-year-old boy complains of bleeding C. Stomalgin
and pain in her gums. The disease onset was D. Stens
4 days ago. Objectively the skin is pale, body E. Orthocor
temperature is 38.5o C . Her submandibular
lymph nodes on the left are enlarged, painful, 6. Parents of a 6-month-old child complain of
non-fused with the surrounding tissues. The their child having a large amount of dental
gingival papillae and gingival margin in the deposit in the oral cavity. Objectively the oral
area of 33, 34, 35, 36, and 37 are hyperemic, mucosa is hyperemic, covered in white easily
ulcerated, and covered in necrotic deposit. removed deposit that resembles curdled mi-
Teeth present with soft dental plaque. Make lk. What is the causative agent of this disease?
the diagnosis: A. Candida fungi
A. Necrotizing ulcerative gingivitis B. Herpes simplex virus
B. Acute catarrhal gingivitis C. Klebs-Loeffler bacillus
C. Chronic hypertrophic gingivitis D. Coxsackie virus
D. Chronic catarrhal gingivitis E. Epstein-Barr virus
E. - 7. A 55-year-old man complains of heartburn
3. A 60-year-old man complains of stabbing and unpleasant sensations in his tongue. The
pain near the root of the tongue on the ri- patient has been suffering from gastritis with
ght, which develops during eating, especially low acidity for approximately 5 years. What
sour food. Objectively: there is a swelling in changes in the tongue are the most likely to
the right submandibular area. On palpati- be detected in this patient?
on the submandibular gland is dense and A. Atrophied and smoothed out lingual papi-
enlarged. Excretory duct orifice of the right llae
submandibular gland is dilated and produces B. Hypertrophic lingual papillae
mucopurulent secretion. What is the most li- C. Erosions on the lateral surfaces of the
kely diagnosis? tongue
A. Sialolithiasis of the submandibular gland D. Coated tongue
B. Calculous sialadenitis of the sublingual E. Fissured tongue
gland 8. A 10-year-old child is referred by the
C. Acute suppurative lymphadenitis orthodontist for extraction of tooth 53.
D. Acute sialodochitis Objectively the crown of 53 is retained, the
E. Adenophlegmon of the right submandi- tooth is immobile. X-ray of tooth 53 shows
bular area root resorption by less than 1/3. Choose the
4. A 19-year-old woman, an actress, complai- best instrument for extraction of tooth 53:
ns of discoloration of her left maxillary A. Straight crown forceps
central incisor. One year ago the pulp of B. Straight elevator
this tooth was removed and the tooth was C. Root bayonet forceps
filled. Gradually the tooth assumed grayish D. Crown forceps with S-shaped handles
color. Objectively the 11 is filled, discolored, E. Beak-shaped root forceps
firm, painless on percussion. Deep occlusi-
on is observed. What part of the clinical 9. A 24-year-old woman has Richmond crown
presentation contraindicates installation of being made to restore the crown of the central
an all-porcelain crown? maxillar incisor. The cap is completed. What
is the next step of prosthesis-making?
Krok 2 Stomatology (англомовний варiант, iноземнi студенти) 2018 рiк 2

A. To fit the cap on the tooth stump and place 14. During preventive examination a 40-year-
the post in the root canal old man presents with the following changes:
B. To solder the post with the cap marginal gingiva is enlarged, torus-shaped,
C. To fit the cap and the post to the tooth root cyanotic, slightly bleeding when touched wi-
D. Making of combination dental crown th a dental probe; there is no pain. Staining
E. Tooth fixation with cement the gums with Lugol’s iodine solution results
in light-brown coloring of mucosa. Make the
10. Removable full dentures are being made diagnosis:
for a 65-year-old man. Progenic occlusion is
determined. What are the specifics of teeth A. Chronic catarrhal gingivitis
placement in case of progenic occlusion of B. Acute catarrhal gingivitis
edentulous jaws? C. Exacerbation of chronic catarrhal gingivitis
D. Chronic hypertrophic gingivitis
A. Superior dental arch is shortened by two E. Generalized periodontitis
premolars
B. Anterior teeth are placed in direct occlusi- 15. A 4-year-old boy has been diagnosed wi-
on th acute purulent periostitis of the upper jaw
C. Inferior dental arch is shortened by two originating from the 64 tooth. Choose the
premolars optimal treatment tactics:
D. Anterior teeth are placed in orthognathic
occlusion A. The 64 tooth extraction, periosteotomy,
E. Short-bite anterior teeth pharmacotherapy
B. The 64 tooth extraction, anti-inflammatory
11. A 53-year-old patient complains of pain pharmacotherapy
and clicking in the left temporomandibular C. Endodontological treatment of the 64
joint. Objectively: the face is symmetrical, tooth, anti-inflammatory pharmacotherapy
palpation of the lateral pterygoid muscles is D. Endodontological treatment of the 64
painful on the left side. Mouth opening is tooth, periosteotomy
reduced. Tomography shows smooth bone E. Periosteotomy, anti-inflammatory
outline of joint surfaces. Which disease of pharmacotherapy
those listed below corresponds with this clini-
cal presentation? 16. An adolescent complains of reduced and
painful mouth opening, difficulties when eati-
A. Temporomandibular joint dysfunction ng, and swelling in the left mandibular angle
B. Rheumatic arthritis that developed after tooth 37 was extracted 3
C. Deforming arthrosis days ago. Objectively the face is asymmetric
D. Acute posttraumatic arthritis due to soft tissue swelling in the area of the
E. Joint ankylosis left mandibular angle. Mouth opening is pai-
nful and reduced to 2.0 cm. Disturbed occlusi-
12. A 47-year-old man complains of partial on is observed. Palpation of the left mandi-
loss of his upper teeth. The patient’s medi- bular angle is painful, the tissues are soft,
cal history states loss of teeth due to trauma bone crepitus is detected. ”Indirect load to
sustained 3 months ago. 11 and 12 are lost. 13, the chin” symptom is positive in the area
21, and 22 are destroyed by 2/3 and restored of the left mandibular angle. The socket of
with fillings. Occlusion is orthognathic. What the extracted tooth is packed with iodoform
denture construction would be optimal for gauze. What is the most likely diagnosis?
this patient, considering his occupation as a
lecturer? A. Left mandibular angle fracture
B. Mandibular alveolar fracture
A. Porcelain-fused-to-metal dental bridge C. Anterior mandibular fracture
B. Plastic dental bridge D. Mandibular periostitis on the left
C. Clasp-retained (bugel) removable partial E. Odontogenic mandibular osteomyelitis
denture with attachments
D. Removable partial laminar denture for the 17. A 32-year-old man has metallic inlay made
upper jaw for him. The denture is being made for tooth
E. Swaged-soldered metal dental bridge with 36 with Black’s class I carious cavity. What
faceted intermediate part surfaces of the inlay should be filed down and
polished before fixing the denture?
13. A 7-year-old boy is diagnosed with epi-
demic parotitis (mumps). Name the most li- A. Occlusal surface
kely complication of this disease: B. Lateral surfaces
C. All surfaces
A. Orchitis D. Inlay bottom
B. Colitis E. Lateral surfaces and inlay bottom
C. Dermatitis
D. Pneumonia 18. A 12-year-old boy complains of pai-
E. Cholecystitis nful and bleeding gums on his upper jaw.
Objectively the gingival margin in the area
Krok 2 Stomatology (англомовний варiант, iноземнi студенти) 2018 рiк 3

of the 13, 12, 11, 21, 22, and 23 teeth is A. Adenocarcinoma of the right parotid gland
swollen, hyperemic, deformed due to gingi- B. Chronic non-epidemic parotitis
val overgrowths. Gingival papillae cover the C. Chronic lymphadenitis of the right parotid
crowns by 1/3 of their height, bleed on touch. area
Upper front teeth are overcrowded. X-ray D. Mixed tumor of the right parotid gland
shows no pathological changes of the peri- E. Actinomycosis of the right parotid gland
odontium. What drugs should be admini-
stered for topical treatment in the first place? 22. Parents of a 3-year-old child report that
the child suffers from constant pain in the
A. Nonsteroidal anti-inflammatory drugs upper front teeth. Objectively: the coronal
B. Sclerosants part of the 61 tooth is gray and decayed.
C. Steroidal anti-inflammatory drugs Probing of the root canal orifice is painful and
D. Keratoplastic agents accompanied by bleeding. The tooth percussi-
E. Cytostatic agents on provokes acute pain. Mucosa is hyperemic,
edematic and painful. Palpation in the region
19. A 43-year-old woman complains of of the 61 and 62 teeth reveals a fistula. What
mobility and displacement of her upper is the provisional diagnosis?
front teeth. Objectively: dental formula is
17 16 15 14 13 12 11 21 22 23 24 25 26 27 A. Exacerbation of chronic periodontitis
47 46 45 44 43 42 41 31 32 33 34 35 36 37 . B. Acute suppurative periodontitis
Teeth 12 11 21 22 are slanted towards C. Acute diffuse pulpitis
the vestibular side, diastema and tremata D. Chronic granulating periodontitis
are observed, I-II degree teeth mobility is E. Exacerbation of chronic pulpitis
detected. Select the orthodontic appliance 23. Puncture sample taken from a 13-year-
for correction of teeth misalignment as a part old child contains giant Reed-Sternberg cells.
of complex treatment of periodontal disease: What diagnosis can be confirmed by the cell
A. Palatal plate with vestibular arch content of this puncture material?
B. Bynin appliance A. Lymphogranulomatosis
C. Schwartz appliance B. Tuberculous lymphadenitis
D. Katz crown C. Lymphocytic leukemia
E. Palatal plate with inclined plane D. Lymph node actinomycosis
20. A 45-year-old man complains of E. Infectious mononucleosis
toothache and mobility of his upper front 24. A 25-year-old man complains of genelal
teeth. Objectively his dental formula is as malaise, high body temperature, acute gi-
follows: ngival bleeding, and gingival enlargement.
17 16 15 14 13 12 11 21 22 23 24 25 26 27
47 46 45 44 43 42 41 31 32 33 34 35 36 37 . He has a history of nosebleeds. Objectively
the patient presents with systemic lymphoid
Dental cervices of 13 12 11 21 22 are hyperplasia, pallor of skin and mucosa, II-III
exposed and demonstrate mobility of the degree hyperplasia of the gingival mucosa,
III degree. Mobile teeth are to be extracted hemorrhages into the buccal mucosa, and
and immediate denture is to be made for the ulcers covered with gray deposit. What
patient. How soon after the teeth extraction examination method would be optimal for
should such dentures be inserted? diagnosis-making in this case?
A. On the day of teeth extraction A. Complete blood test panel
B. In 1-2 days B. Yasynsky test
C. In 3-4 days C. Bacterioscopy
D. In 5-6 days D. Immunoassay
E. In 6-7 days E. Blood glucose test
21. A 56-year-old man complains of swelli- 25. A 19-year-old young man complains of
ng and pain in his right parotid area. The constant pain in tooth 22, which intensifies on
swelling was noticed 5-6 months ago. Objecti- biting with this tooth, sensation of ”protrudi-
vely right-sided paresis of the facial muscles ng” tooth, and upper lip edema. The pati-
can be determined. Palpation reveals there ent has history of upper jaw trauma. Objecti-
a modrately painful tuberous tumor fused vely tooth 22 is intact. Vertical percussion
with surrounding tissues. In the center of is acutely painful. The upper lip is swollen,
the tumor there is an area of softening. mucogingival fold in the area of tooth 22 is
Submandibular and cervical lymph nodes on red and painful on palpation. What exami-
the right are enlarged and dense. The mouth nation method is necessary for diagnosis-
can be opened without restriction. There is making in this case?
no saliva outflow from the opening of the ri-
ght parotid gland. What provisional diagnosis
can be made?
Krok 2 Stomatology (англомовний варiант, iноземнi студенти) 2018 рiк 4

A. X-ray A. Erythema multiforme exudativum


B. Dental pulp test B. Pemphigus vulgaris
C. Rheodentography C. Acute herpetic stomatitis
D. Transillumination D. Nonacantholytic pemphigus
E. Thermometry E. Dermatitis herpetiformis (Duhring’s di-
sease)
26. A patient complains of carious cavity in
tooth 11. The filling was lost one week ago. 30. A 28-year-old woman complains of persi-
The tooth crown is dark, there is residual fi- sting pain in tooth 34, which intensifies on
lling material at the bottom of the carious biting. Four days ago arsenic paste was left
cavity. Vertical percussion is painless. X-ray in the 34. The patient missed her appoi-
shows an oval area of bone tissue resorpti- ntment with the dentist. Objective examinati-
on with clear margins, 0.4х0.3 cm in size. The on detected occlusive dressing on the distal
root canal is filled by 2/3 of its length. What is masticatory surface of tooth 34, percussion is
the most likely diagnosis? acutely painful. What treatment tactics would
be the most advisable in this case?
A. Chronic granulomatous periodontitis
B. Chronic fibrous periodontitis A. Arsenic antidote is placed into the root
C. Chronic granulating periodontitis canal under occlusive dressing
D. Radicular cyst B. Dentin dressing is removed, electrophoresis
E. Exacerbation of chronic periodontitis with antidote along the mucogingival fold is
prescribed
27. A 28-year-old man is referred for oral C. The root canal is lavaged with antidote, the
cavity sanation. On examination there is a tooth remains uncovered
filling on the masticatory surface of tooth D. The root canal is lavaged with antidote and
17, percussion is painless. Mucosa in the root filled
apex projection of 17 is cyanotic, vasoparesis E. -
symptom is positive. X-ray shows foci of bone
tissue destruction with fuzzy margins in the 31. A 35-year-old man came to the
area of root apices, root canals are not filled. prosthodontic clinic with complaints of teeth
What is the most likely diagnosis? mobility on his lower jaw. What type of
occlusion stabilization is recommended in
A. Chronic granulating periodontitis this case?
B. Chronic fibrous periodontitis
C. Chronic granulomatous periodontitis A. Arch
D. Radicular cyst B. Sagittal
E. Chronic fibrous pulpitis C. Frontal
D. Parasagittal
28. A 30-year-old woman complains of mi- E. Frontosagittal
ld burning sensation in her lower lip and
its dryness. She peels skin scales off with 32. A 57-year-old patient complains of tooth
her teeth. She has been presenting with this mobility and inability to eat. Objectively: the
condition for 10 years. On examination the lower 35, 36, 37, 38, 44, 45, 46 and 48 teeth are
skin scales are gray and located on the lip missing; the 31, 32, 33, 34, 41, 42, 43, 47 teeth
from the Klein’s line to the center of the exhibit grade II mobility, their clinical crowns
vermillion border from angle to angle of the are low, tooth equator is not pronounced.
mouth. The scales are firmly attached in the What is the optimal denture construction in
center and are loose on the periphery. Their this case?
forcible removal does not result in erosions.
What is the most likely diagnosis? A. Removable cast splint
B. Removable partial denture
A. Exfoliative cheilitis C. Kurlyandsky splint bar
B. Lupus erythematosus D. Bynin removable splint
C. Meteorological cheilitis E. Removable splint with vestibulo-oral clasp
D. Allergic contact cheilitis
E. Eczematous cheilitis 33. A 16-year-old young man complai-
ns of temperature increase up to 38.7o C ,
29. A 32-year-old patient presents with pain when eating and swallowing, foul
body temperature of 38.9o C , general fatigue, acrid smell from his mouth. Lymph nodes,
impaired speech, inability to eat. This condi- especially cervical ones, are enlarged, mobi-
tion has been recurring for the last 4 years in le, and painless. Objectively the patient
autumn and spring. There are vesicles and presents with generalized hyperemia of the
erosions with grayish fibrinous coating on oral mucosa, multiple petechiae, necrotic
the hyperemic and swollen labial and buccal spots, and profuse coating of the anteri-
mucosa. Nikolsky’s sign is negative. What is or pharynx. Blood test: increased ESR,
the most likely diagnosis? marked leukocytosis, monocytosis, atypical
mononuclear cells, thrombocytopenia. What
is the most likely diagnosis?
Krok 2 Stomatology (англомовний варiант, iноземнi студенти) 2018 рiк 5

painful along the whole floor of the carious


A. Infectious mononucleosis cavity. Electric pulp test - 50 microamperes.
B. Vincent stomatitis (acute necrotizing Thermodiagnosis is painful. Make the di-
ulcerative gingivitis) agnosis:
C. Herpetic angina
D. Monoblastic leukemia A. Chronic fibrous pulpitis
E. Acute herpetic stomatitis B. Chronic deep caries
C. Chronic fibrous periodontitis
34. A 57-year-old retired man complains of D. Acute deep caries
attacks of burning pain and rashes on the E. Chronic gangrenous pulpitis
skin of his face and oral mucosa on the right.
Anamnesis: a course of radiation therapy for 38. A 35-year-old man complains of thickeni-
treatment of gastric disease, past case of chi- ng of his maxillary alveolar process. Prelimi-
ckenpox. Objectively: along the third branch nary diagnosis of maxillary radicular cyst was
of the trigeminal nerve the skin of the face made. What substance will be obtained as the
presents with isolated erosions covered in fi- result of the puncture of the alveolar process
brinous coating. There are multiple vesicles in the area of buccal thickening?
on the hyperemic and swollen oral mucosa.
Right-sided lymphadenitis is observed. What A. Yellowish liquid
is the most likely diagnosis? B. Blood
C. Pus
A. Herpes zoster D. Epithelium
B. Neuralgia E. Turbid infiltration
C. Murrain
D. Acute recurrent herpes 39. Six months ago a 40-year-old man had his
E. Neuritis tooth 26 extracted; afterwards his oral cavity
and maxillary sinus became communicating
35. A 12-year-old girl has complaint of a cari- and the patient developed the first signs of
ous cavity in her tooth. Objectively: there is maxillary sinusitis. What surgical procedure
Black’s class 1 carious cavity in the 36 tooth; should be performed in this case?
it is localized in the parapulpar dentin; the
mouth of the cavity is wide. The dentin is A. Maxillary sinusotomy with simultaneous
dense and pigmented. It is sensitive to cold plastic surgery for repair of the fistula
stimulus, percussion is painless. What is the B. Caldwell-Luc surgery
most likely diagnosis? C. Fistula plication
D. Fistula packing with Iodoform gauze
A. Chronic deep caries E. Osteotomy of the alveolar process
B. Chronic median caries
C. Acute deep caries 40. A 46-year-old woman complains of
D. Acute median caries bleeding gums, suppuration, teeth mobili-
E. - ty. She has been presenting with these signs
for 10 years. On examination her upper and
36. A 35-year-old woman has complaints of lower gums are hyperemic, swollen, bleed on
cosmetic defects of the front upper teeth touch. In the area of 42, 41, 31, and 32 peri-
crowns. The defects have been aggravati- odontal pockets are up to 8 mm deep, contain
ng for the last 10 years. The patient suffers purulent discharge; these teeth demonstrate
from unpleasant sensations when brushi- mobility of the II degree, other teeth present
ng her teeth, and when chemical stimuli with mobility of the I degree. In the area of
are applied. Objective examination revealed 42, 41, 31, and 32 X-ray shows interalveolar
defects localized in the enamel of the front septa resorption by 1/2 of the root length and
upper teeth vestibular surface. The defects signs of osteoporosis. What is the most advi-
are oval, saucer-shaped, and have clear margi- sable method of surgical treatment in this
ns. Response to probing and cold stimuli was case?
positive. Make the diagnosis:
A. Osteoplasty
A. Enamel erosion B. Curettage
B. Enamel hypoplasia C. Gingivotomy
C. Cuneiform defect D. Gingivectomy
D. Chemical necrosis of the tooth E. Flap surgery
E. Hyperesthesia of tooth hard tissues
41. A 40-year-old man had his root canal of
37. A patient complains of dull ache in the the 34 tooth filled due to chronic fibrous peri-
16 tooth, which occurs during eating cold odontitis. Soon the treated place became pai-
food. Previously the tooth had been filled nful. On X-ray the root canal of the 34 tooth
due to deep caries, the filling was lost 1 year is filled to the root apex. What tactics should
ago. Objectively: a deep carious cavity that the dentist choose to manage the pain?
does not communicate with the tooth cavity
is present; percussion is painless, probing is
Krok 2 Stomatology (англомовний варiант, iноземнi студенти) 2018 рiк 6

A. To prescribe physiotherapeutic procedures her. What material would be optimal for this
B. To rinse with antiseptic mouthwash splint?
C. To make insicion along the mucogingival
fold A. Cobalt nickel chromium alloy
D. To provide conduction anesthesia B. ”EI-95” alloy
E. To provide infiltration anesthesia C. Stainless steel
D. ”PD-250” alloy (silver palladium alloy)
42. A 49-year-old man was diagnosed wi- E. Gold alloy of 900 millesimal fineness
th recurrence of lower lip cancer two years
after he had undergone radiation therapy. 47. A 4-year-old practically healthy child
Objectively in the area of his lower right came for oral cavity sanation. Objectively on
lip there is a neoplasm 1x2 cm in size with the masticatory surface of 75 there is a cari-
an ulcer in its center. In the right submandi- ous cavity within mantle dentin. The cavity
bular area there are 2 round, enlarged, dense, is filled with softened dentin. Dentinoenamel
painless lymph nodes. What approach to the junction is painful on probing. What material
treatment would be optimal in this case? would be optimal for permanent filling?
A. Combined treatment A. Glass ionomer cement
B. Wedge resection of the lower lip B. Phosphate cement
C. Rectangular resection of the lower lip C. Silicophosphate cement
D. Trapezial resection of the lower lip D. Composite material
E. Vanakh’s operation E. Silicate cement
43. During regular check-up a 6.5-year-old 48. A 7.5-year-old practically healthy child
child presents with carious cavity on the di- complains of crown fracture and pain in the
stal proximal surface of 65 within mantle upper right incisor. Objectively 2/3 of crown
dentin. Cavity walls and bottom are pi- of 11 is absent, the pulp is exposed and red;
gmented, dense, painless on probing; there on probing it is acutely painful and bleedi-
is no response to cold stimulus; percussion ng; tooth percussion is painful. The trauma
is painless. During tooth preparation there occurred 2 hours ago. What would be the
is tenderness at the level of dentinoenamel optimal treatment method in this case?
junction. What is the most likely diagnosis?
A. Vital amputation
A. Chronic median caries B. Devital amputation
B. Acute median caries C. Vital extirpation
C. Chronic deep caries D. Devital extirpation
D. Chronic fibrous pulpitis E. Biological approach
E. Chronic granulating periodontitis
49. Parents of a 2.5-year-old child complain of
44. After adenotonsillectomia it is necessary gradual destruction of the upper front teeth
to break the mouth breathing habit in a 4- of their child for the last several months.
year-old child. The orthodontist recommends Objectively there are carious cavities within
application of an oral vestibular shield mantle dentin on the contact and vestibular
(Kerbitz’ vestibular plate). Vestibular shield surfaces of 52, 51, 61, and 62. The cavities are
facilitates training of the following muscle: filled with softened pigmented dentin that
can be easily removed with dental excavator.
A. Orbicular muscle Make the provisional diagnosis:
B. Temporal muscle
C. Masseter muscle A. Acute median caries
D. Lateral pterygoid muscle B. Chronic deep caries
E. Medial pterygoid muscle C. Acute deep caries
D. Chronic median caries
45. A 30-year-old man presents with fresh E. Chronic superficial caries
median mandibular fracture without visible
displacement of the fragments. What will be 50. A 49-year-old man complains of progressi-
the function of the dental apparatus to be ng reduction of mouth opening, pain on
prescribed in this case? the left when swallowing, severe deteriorati-
on of his general well-being, temperature
A. Fixation increase up to 39.3o C . Destroyed tooth 38
B. Setting presents with acute pain. Objectively the face
C. Directing is symmetrical, the submandibular lymph
D. Replacement nodes on the left are enlarged and painful
E. Formation on palpation. Palpation under the left mandi-
bular angle and in the the left retromandi-
46. A 43-year-old woman complains of her bular area provokes sharp pain. Mouth
lower teeth mobility. Objectively the teeth opening and movement of the mandible to
mobility is of the I-II degree. It is planned to the left are significantly reduced. The left
make a full-cast removable occlusal splint for pterygomandibular fold is hyperemic and
Krok 2 Stomatology (англомовний варiант, iноземнi студенти) 2018 рiк 7

infiltrated. What is the most likely diagnosis? transport immobilization of the fracture?
A. Phlegmon of the pterygomandibular space A. Patient’s dentures
B. Phlegmon of the parapharyngeal space B. Weber splint
C. Phlegmon of the retromandibular area C. Vankevych splint
D. Phlegmon of the submandibular space D. Zbarzh apparatus
E. Abscess of the sublingual fossa E. Vasiliev splint
51. A 40-year-old man presents with 55. A 57-year-old woman came to a dentist for
pathologic teeth grinding caused by their extraction of the 34 tooth due to exacerbati-
functional overload due to the loss of many on of chronic periodontitis. What instrument
antagonist teeth. With direct occlusion, verti- would be optimal for tooth extraction in the
cal grinding of the front teeth resulted given case?
in protrusion of the patient’s lower jaw
forwards. Interalveolar space is diminished, A. Beak-shaped non-crushing forceps
the lower third of the face is shortened. What B. Beak-shaped crushing forceps
would be the most advisable treatment in this C. Beak-shaped curved forceps
case? D. Straight elevator
E. Curved elevators
A. Prosthetics that increase interalveolar
height 56. A cast clasp-retained (bugel) removable
B. Prothetic treatment partial denture is being made for a 58-year-
C. Teeth shortening old patient. Impressions are made, centric jaw
D. Prosthetic treatment relation is determined, plaster casts are obtai-
E. Instrumental surgical treatment ned. What is the next stage?
52. A 7-year-old child complains of pain and A. Examination of the working model with a
swelling in the left submandibular region. parallelometer
The swelling in this region developed 2 days B. Transfer of denture frame pattern to the
ago. Objectively: the child is in a satisfactory working model
condition, body temperature is of 37.3o C . C. Wax modelling of the denture frame
Face is asymmetrical due to the soft tissue D. Duplication of the working model
swelling in the left submandibular region. E. Marking the border seal
Palpation reveals a round formation 2x2 cm
in size. The formation is mobile, painful, non- 57. A 7-year-old boy underwent fissure seali-
fused with the skin. The 74 tooth is discolored, ng in teeth 36 and 46. Fissure sealing would
percussion is painful. What is the provisional be most effective:
diagnosis? A. Immediately after eruption of the
A. Acute serous odontogenic lymphadenitis permanent tooth
of the left submandibular region B. After permanent occlusion is formed
B. Acute serous nonodontogenic lymphadeni- C. If permanent teeth are affected with caries
tis of the left submandibular region D. In 1-2 years after tooth eruption
C. Acute suppurative odontogenic E. In 3-4 years after tooth eruption
lymphadenitis of the left submandibular 58. A 6-year-old girl took paracetamol to
region treat a case of URTI two days ago, whi-
D. Phlegmonous adenitis of the right ch resulted in the development of her
submandibular region present condition. The disease onset was
E. Lateral cervical cyst acute with temperature increase up to
53. A 48-year-old patient complains of the 39.8o C . Objectively there are cockade-shaped
lower jaw teeth mobility. Van Thiel dental maculopapular rashes on her face. The vermi-
splint is to be made for prosthodontic lion border is swollen, hyperemic, covered
treatment. What construction elements are in massive brown crusts, and presents with
supposed to fix it in place? bleeding cracks. Conjunctivitis is detected.
Swollen and hyperemic oral mucosa presents
A. Whole piece proximal grip clasps with numerous erosions covered with fibri-
B. Full metal crowns nous incrustations; the erosions are sharply
C. Wire clasps painful on palpation. What is the most likely
D. Parapulpar posts diagnosis?
E. Equator crowns
A. Stevens-Johnson syndrome
54. A 55-year-old man suffered a blow to the B. Erythema multiforme exudativum
frontal mandibular area. He is diagnosed with C. Acute herpetic stomatitis
mandibular fracture. Prior to trauma he was D. Chronic recurrent aphthous stomatitis
wearing removable dentures (partial laminar E. Pemphigus
denture for the lower law and full denture
for the upper jaw). What can be used for 59. How often should the dentures be
Krok 2 Stomatology (англомовний варiант, iноземнi студенти) 2018 рiк 8

replaced in children during the period of milk covered in scales of varying size. In the angles
occlusion according to Ilyina-Markosian? of the mouth there are fissures covered in
white coating, the skin is macerated. What
A. Every 6-8 months ointment should be prescribed for topical
B. Every 8-10 months treatment in the given case?
C. Every 10-12 months
D. Every 12-16 months A. Clotrimazol
E. Every 16 months B. Interferon
C. Prednisolone
60. On objective examination a 59-year-old D. Lanolin
man with the edentulous mandible presents E. Erythromycin
with bone protrusions and mobile areas of
the alveolar crest. To ensure proper fixati- 64. A 35-year-old woman complains of lips
on of the denture and even load distribution enlargement. The first incident occurred one
the following functional impression should be year ago, when she developed lip edema that
made: abated quickly, but the lips remained slightly
enlarged. Three days ago after overexposure
A. Differentiated to cold her lips enlarged again. Objectively:
B. Complete anatomical ptosis, upper and lower lips are markedly
C. Compression enlarged, more on the left, soft, elastic, and
D. Decompression painless on palpation; no impressions on the
E. Combined lip surface are left after pressing it with a fi-
nger. The tongue is swollen, with tuberous
61. A 28-year-old man complains of pain in surface and folds on its back. What is the
the infraorbital and parotid region on the most likely diagnosis?
left. On examination: hemorrhage occurs in
the lower eyelid and conjunctiva of the left A. Melkersson-Rosenthal syndrome
eye, there are signs of crepitation and step B. Miescher’s granulomatous cheilitis
deformity of the eyesocket lower edge. The C. Quincke’s edema
mouth opens by 1 cm. Make the diagnosis: D. Achard’s syndrome
E. Meige’s trophedema
A. Zygomatic bone fracture
B. Malar arch fracture 65. A 45-year-old man complains of dryness
C. Left articular process fracture and pain in the lower lip. On examination:
D. Traumatic arthritis of the temporo- mandi- the lower lip is swollen, dry, covered in small
bular joint scales and fissures. In the Klein area (wet-dry
E. Hematoma of the infraorbital region line) there are dilated openings of salivatory
62. A child is 8 years old. There are complai- glands observed as red dots producing clear
nts of congested upper incisors. Objectively: substance. The lower lip mucosa is lumpy.
the first molars closure is of Angle’s I class, What is the most likely diagnosis?
frontal overbite is orthognathic. The 12 and A. Glandular cheilitis
22 teeth erupt palatinally with space defici- B. Actinic cheilitis
ency of 2/3 of the tooth crown. The 11 and C. Meteorological cheilitis
21 teeth are 10 mm each in cross-section. The D. Eczematous cheilitis
child has inherited father’s facial type with E. Exfoliative cheilitis
prognathism and macrodontia of the central
incisors. Choose the preventive treatment, 66. A 23-year-old man complains of gum
considering this hereditary pathology: bleeding when he brushes his teeth or eats
solid food. Objectively: the gums of the front
A. Hotz serial extraction to reduce the dental lower jaw are hyperemic, swollen and bleedi-
arch ng when palpated. Oral and gingival mucosa
B. Jaw expansion to provide the space for the in other areas are not affected. The occlusi-
12 and 21 teeth on is deep. The teeth are firm, except for the
C. Massage of the 12 and 21 teeth area to 41 and 31 (degree 1 mobility). X-ray shows
stimulate their eruption resorption of the alveolar septum in the area
D. Extraction of the 12 and 21 teeth to reduce of the 41, 42, 32, and 31 teeth up to 1/3 of the
the dental arch root length. What is the most likely diagnosis?
E. Shave off the approximal surfaces of the 11
and 21 to provide the space for the 12 and 22 A. Localized periodontitis
teeth B. Generalized periodontitis, initial stage
C. Generalized periodontitis, stage I
63. A 38-year-old woman complains of burni- D. Catarrhal gingivitis
ng pain in her lips and angles of her mouth, E. Parodontosis, stage I
their dryness. Anamnesis states that she has
been suffering from diabetes mellitus for 67. A 40-year-old man, a chemical industry
the last 8 years. Objectively: the vermilli- worker, notes the sour sensation in his mouth,
on border is dry, congestively hyperemic, pain response to thermal and chemical sti-
Krok 2 Stomatology (англомовний варiант, iноземнi студенти) 2018 рiк 9

muli. On examination: on the vestibular A. Osteoclastoma


surface and cutting edge of the front teeth B. Adamantinoma
there are chalky enamel defects with uneven C. Osteoma
scalloped margins. Make the diagnosis: D. Follicular cyst
E. Odontoma
A. Acidic necrosis of enamel
B. Superficial caries 72. A 6.5-year-old child has closed non-
C. Enamel hypoplasia (erosive form) pigmented fissures in the first permanent
D. Fluorosis (erosive form) molar, which have been revealed duri-
E. Median caries ng preventive examination. Enamel
transparency is retained, its probing reveals
68. A 42-year-old woman complains of acute no coarseness. Choose the optimal method of
lip enlargement, itching, and bursting sensati- treatment in this case:
on. She ascribes her condition to introduction
of a new lipstick. On examination the lips are A. Non-invasive sealing
significantly enlarged and turgid, on palpati- B. Invasive sealing
on they are firm, elastic, and painless. Regi- C. Preventive filling
onal lymph nodes are without changes. What D. ART technique
is the most likely diagnosis? E. Regular medical check-ups

A. Allergic contact cheilitis 73. A man complains of gingival pain in


B. Meteorological cheilitis his upper left jaw and bleeding that occurs
C. Exfoliative cheilitis when he brushes his teeth or eats solid food.
D. Glandular cheilitis Objectively on the upper jaw he wears a
E. Actinic cheilitis swaged-soldered metal bridge with 14 and
16 as abutment teeth. The crown edge is
69. A 50-year-old patient, an employee of the pushed under the gingival margin by 0.3
print shop, complains of foul smell from his mm. Intermediate part is closely fitted to
mouth and excessive salivation. Objectively the gums. The mucosa is hyperemic, swollen,
against the background of hyperemic and interdental papillae are smoothed out;
slightly swollen gums there is a blue-black touching mucosa with a dental instrument
border along the gum margin of the lower provokes bleeding. What medical tactics
jaw and upper front teeth. There is a large should the dentist choose in the first place?
amount of dental deposit observed. Name
the type of stomatitis in this patient: A. Remove the dental bridge
B. Refer the patient for cosultation with the
A. Lead dental therapist
B. Mercury C. Refer the patient for X-ray
C. Bismuth D. Refer the patient for clinical blood test
D. Catarrhal E. Make metal-fused-to-porcelain dental
E. Necrotizing ulcerative bridge
70. A 25-year-old man complains of 74. When checking construction of the
incorrectly positioned maxillary left central soldered dental bridge with the 35 and 38
incisor due to trauma sustained 2 months ago. abutment teeth the following was detected:
Objectively tooth 21 is rotated around its axis pores in the place where abutment crowns
into palatal position. What would be the most and intermediate part are soldered together;
advisable treatment method for correction of masticatory cusps are sharply defined; there
this defect? is early contact with antagonist teeth; the
intermediate part makes tight contact with
A. Orthodontic treatment the alveolar process mucosa. How can those
B. Surgical treatment flaws be corrected?
C. Instrumental surgical treatment
D. - A. Dental bridge should be remade
E. Splinting followed by prosthetic treatment B. Intermediate part should be corrected
C. Masticatory surface should be corrected,
71. A 44-year-old woman complains of the and soldered places - polished
face swelling in the right lower jaw area and D. Masticatory surface should be corrected,
teeth mobility. Objectively: soft tissues are and the height of the flushing part is to be
without changes, the regional lymph nodes increased up to 2 mm
cannot be palpated. The alveolar process and E. Tooth-antagonists should be shaved off
the body of the lower jaw near the 46, 47,
and 48 teeth are thickened, painless when 75. A 50-year-old man was diagnosed with
palpated, and lumpy. The teeth in the thi- sialolithiasis with the salivary gland stone
ckened area are mobile. Puncture consists located deep within the salivary gland.
of brown fluid without cholesterol crystals. Choose the optimal treatment tactics:
What is the provisional diagnosis?
Krok 2 Stomatology (англомовний варiант, iноземнi студенти) 2018 рiк 10

A. Submandibular gland excision A. Plaster


B. Radiation therapy B. Dentafol
C. Sclerotherapy C. Stens
D. Removal of the sialolith while retaining the D. Stomaflex
gland E. Repin
E. Conservative pharmacotherapy
81. A 19-year-old young man complains
76. Due to lack of timely specialized of cosmetic defect of all his teeth, which
treatment a 44-year-old man presents wi- developed immediately after the teeth erupti-
th incorrectly healed displaced mandibular on. Objectively on the vestibular and masti-
fracture. Objectively the lower jaw narrows catory surfaces of all patient’s teeth there
sharply, vestibular cusps of the lower teeth are enamel defects, tooth crowns present
contact with oral cusps of the upper teeth. with dark brown discoloration. Percussion
The patient declined surgical treatment. and probing are painful. In this area fluori-
What treatment tactics should be chosen by de levels in water are 2.6 mg/L. Make the
the dentist in this case? provisional diagnosis:
A. Make a prosthesis with double dentition A. Fluorosis
B. Make a non-removable dental bridge with B. Systemic hypoplasia
movable joint C. Chronic initial caries
C. Correct the malocclusion by filing down the D. Enamel erosion
patient’s teeth E. Chronic superficial caries
D. Correct the malocclusion via instrumental
surgical method 82. A 14-year-old teenager complains of dry
E. Make a dentogingival laminar denture and chapped lips especially in autumn and
winter. Objetively the vermillion border is
77. A 57-year-old man presents with habi- dry, infiltrated, and covered in numerous
tual mandibular dislocation. To reduce mouth scales. Skin of the lips is dense, pigmented,
opening, Yadrova apparatus was made. How with pronounced pattern, peeling, and radial
long should the treatment last in this case? cracks. At the same time the skin of the face
is dry, lichenified, and excoriated. What is the
A. 3 months provisional diagnosis?
B. 6 months
C. 9 months A. Atopic cheilitis
D. 12 months B. Exfoliative cheilitis
E. 18 months C. Actinic cheilitis
D. Meteorological cheilitis
78. An HIV-infected patients needs a dental E. Allergic contact cheilitis
prosthesis. The dentist plans to make dental
bridges for this patient. How should the 83. A 56-year-old patient suffering from
instruments be processed after the appoi- exacerbation of schizophrenia has been
ntment? hospitalized in an oral in-patient department
with a diagnosis of the displaced mandibular
A. According to the special scheme fracture in the area of teeth 34 and 35. What
B. According to the usual scheme method of treatment should be prescribed?
C. In a hot air sterilizer
D. With lysoformin A. Osteosynthesis
E. With 3% chloramine solution B. One arch smooth dental braces
C. Full dental splint
79. A 30-year-old man complains of pain in D. Weber splint
his front lower teeth, which he attributes to E. Vankevych splint
a trauma to the mental region. Objectively:
continuous dentition, orthognathic occlusion. 84. During preventive examination a 5-year-
X-ray shows a median mandibular fracture. old child was determined to have insufficient
What dental splint would be optimal? physiological attrition of the cusps of the deci-
duous canines. What treatment tactics should
A. Flat occlusal splint the doctor choose?
B. Soldered splint on rings
C. Cap splint A. To shave off the retained canine cusps
D. Weber’s splint B. Medical examination once a month until
E. Plastic mouthguard the incisors are replaced
C. Medical examination every 6 months until
80. To make the external prosthesis for a 62- the incisors are replaced
year-old man it is necessary to obtain a Hi- D. Medical examination every 6 months until
ppocrates facial moulage of this patient. What the canines are replaced
impression material should be used? E. No medical intervention is necessary
85. A 60-year-old patient has been undergoi-
Krok 2 Stomatology (англомовний варiант, iноземнi студенти) 2018 рiк 11

ng the procedure of checking the complete A. Periosteotomy and pharmacotherapy


removable dentures construction and fixi- followed by treatment of the causative tooth
ng teeth on wax bases. The following flaws B. -
have been detected: fissure between the teeth C. Extraction of the causative tooth, physi-
on the frontal area and cusp-to-cusp contact otherapy
in the lateral area. What mistake had been D. Extraction of the causative tooth,
made? pharmacotherapy
E. Extraction of the causative tooth, peri-
A. Anterior occlusion had been determined osteotomy
instead of central one
B. Posterior occlusion had been determined 89. A 30-year-old patient complains of pain
instead of central one and swelling in the area of the left parotid
C. Lateral occlusion had been determined salivary gland, which occurred 7 days after
instead of central one he had undergone abdominal cavity surgery.
D. Models had been plastered in a wrong way Objectively: body temperature is 39o C ,
in an occluder reduced mouth opening, dry mouth; when the
E. Swabs had been crushed during determi- gland is massaged, there is purulent exudate
nation of central occlusion being secreted from its duct. The patient can
be diagnosed with the following disease:
86. A 35-year-old man complains of sour
sensation in his mouth and front teeth A. Acute non-epidemic parotitis
sensitivity to thermal and mechanical sti- B. Acute epidemic parotitis
muli. Objective examination revealed visi- C. Phlegmon of submasseteric space
ble changes in the enamel of 13, 12, 11, 21, D. Parenchymatous parotitis
22, and 23. The enamel is dull, rough, mi- E. Phlegmon of parotid-masticatory region
ssing on the cutting edge. Probing of the
vestibular surface of these teeth is painful, 90. A 45-year-old man presents with facial
response to thermal stimuli is positive. The asymmetry due to a dense isolated infiltrati-
patient’s medical record states his occupati- on in his right buccal area; the skin over the
on in industrial production of inorganic acids. infiltration is cyanotic, thinned out; in the
What is the most likely diagnosis? center of the infiltration there is a fistula. In
the oral cavity the crown of 46 is destroyed
A. Necrosis of dental hard tissues by 2/3, along the mucogingival fold the band
B. Fluorosis connecting the tooth with the fistula can be
C. Enamel erosion palpated. Make the diagnosis:
D. Pathologic teeth grinding
E. Enamel hypoplasia A. Migrating facial granuloma
B. Chronic mandibular osteomyelitis
87. A 55-year-old man was delivered into the C. Cheek furuncle
hospital with bilateral mandibular fracture D. Odontogenic lymphadenitis
within the dentition. Objectively teeth 34, 35, E. Actinomycosis
36, 45, and 46 are missing. Lower incisors are
mobile (I-II degree). Fragment displacement 91. A 53-year-old patient complains of an
is insignificant. What splint should be made ulcer on the lateral surface of the tongue. The
for this patient? ulcer appeared 6 months ago as the result of
a trauma caused by sharp tip of the 37 tooth
A. Weber metal crown. A dentist replaced the crown
B. Vankevych with the one of better quality and prescribed
C. Tigerstedt keratoplastic drugs. Despite these measures
D. Flat occlusal splint the ulcer continues to grow. Lately there has
E. Gunning-Port been pain observed during talking, chewi-
ng, swallowing; sometimes the pain irradi-
88. A 30-year-old patient is diagnosed with ates to the pharynx. Objectively on the lateral
acute suppurative odontogenic periostitis of surface of the tongue there is a painful ulcer
the upper left jaw originating from tooth 23. with uneven raised dense margins and lumpy
The crown of 23 on the left is destroyed with floor with grayish necrotic coating. What is
caries by 1/3. Teeth 22 and 24 are intact. Spot- the most likely diagnosis?
film X-ray shows widening of the periodontal
fissure of 23. What treatment would be the A. Cancer of the lateral surface of the tongue
most advisable in this case? B. Trophic ulcer
C. Traumatic ulcer
D. Vincent’s necrotizing ulcerative stomatitis
E. Tuberculous ulcer
92. A 13-year-old child complains of peri-
odical gingival bleeding during teeth brushi-
ng, which has been observed for half a year.
Objectively the gingival mucosa in the frontal
Krok 2 Stomatology (англомовний варiант, iноземнi студенти) 2018 рiк 12

mandibular area presents with congestive A. Calcium hydroxide paste


hyperemia and edema. Decay-missing-filled B. Zinc phosphate cement
(DMF) index equals 4. Oral cavity hygiene is C. Resorcinol-formalin paste
unsatisfactory. In this case it is necessary to D. Glass ionomer cement
recommend the patient the toothpastes with: E. -

A. Herbal extracts 97. The medical station of a regiment received


B. Zinc citrate a patient with signs of bilateral mandibular
C. Calcium glycerophosphate fracture. What is the main task of first aid in
D. Amine fluorides this case?
E. Salt additives
A. To control shock, bleeding, and asphyxia
93. A 70-year-old man has edentulous maxi- and to provide transport immobilization
lla. Objectively maxillary tuberosity and B. To check and correct previously applied
alveolar processes are completely atrophi- bandages
ed; palatine vault is flat, its mucosal layer is C. To administer analgesics and cardiac medi-
moderately pliant. In this case the patient’s cations
atrophic edentulous maxilla can be classified D. To clean oral cavity from blood clots, tooth
as: shards, and bone fragments
E. To provide symptomatic therapy and care
A. Schroeder class III
B. Keller class III 98. A 35-year-old man has been hospitali-
C. Schroeder class II zed into a dentofacial unit with complaints of
D. Keller class II mobility of the 38, 37, and 36 teeth and a fi-
E. - stulous tract in the socket of the extracted 35
tooth. The condition has been persisting for 3
94. Teeth 71 and 81 erupted in a 6-year-old months. Insertion of a grooved probe into the
child, the lower jaw is retrogenic, the palate is fistulous tract palpated a bared coarse bone
flat with pronounced cross-folds. Determine fragment that easily moved under pressure.
the condition of the oral cavity: X-ray of the lower jaw demonstrates a focus
of bone tissue destruction, with a spot of
A. Physiologic dense bone tissue 0.5х0.3 cm in size. Make
B. Pathologic the diagnosis:
C. Abnormal
D. Subcompensated A. Chronic osteomyelitis
E. Decompensated B. Acute osteomyelitis
C. Exacerbation of chronic osteomyelitis
95. A 12-year-old girl complains of pain in her D. Chronic periostitis
mouth that occurs during eating. Accordi- E. Actinomycosis
ng to her medical history these symptoms
reemerge once or twice per year. Objectively 99. A 10-year-old child complains of persi-
on the mucogingival fold there are 3 aphthae sting throbbing pain in tooth 36, whi-
5-7 mm in size, they have yellowish coati- ch appeared one day ago. Hot stimulus
ng and inflamed red border and are acutely aggravates the pain, while cold mitigates it
painful on touch. Name the most likely di- slightly. Objectively on the masticatory and
agnosis: medial surfaces of tooth 36 there is a deep
carious cavity non-communicating with the
A. Chronic recurrent aphthous stomatitis dental cavity. Probing of the cavity bottom
B. Acute herpetic stomatitis and percussion are painful. X-ray shows no
C. Chronic recurrent herpetic stomatitis pathologic changes of the periodontium.
D. Erythema multiforme exudativum What treatment method shold be chosen in
E. Toxic-allergic drug-induced stomatitis this case?
96. An 8-year-old girl complains of tooth A. Vital extirpation
21 discoloration and pain response to hot B. Devital extirpation
stimulus. Several months ago tooth 21 was C. Vital amputation
treated for acute diffuse pulpitis by means of D. Devital amputation
vital amputation. Objectively tooth 21 is fi- E. Conservative treatment
lled, percussion is painless. X-ray shows the
rooth to be formed by 2/3, cortical plate of 100. A 11-year-old child complains of pain in
the tooth socket remains intact in the area of the lower right lateral tooth, which occurs
root radix. What material should be used for when eating, especially hot food. On the
root canal filling in this case? masticatory surface of the 46 tooth there is a
large carious cavity filled with softened light-
brown dentin. The cavity is located within
parapulpar dentin. In the projection of the
medial buccal pulp horn the carious cavity
communicates with the pulp chamber. Deep
Krok 2 Stomatology (англомовний варiант, iноземнi студенти) 2018 рiк 13

probing is painful. Electric pulp test - 60 mi- A. Make a temporary removable denture
croamperes. Make the diagnosis: B. Make a clasp-retained (bugel) removable
partial denture
A. Chronic gangrenous pulpitis C. Make a dental bridge with 12 and 21 as
B. Chronic hypertrophic pulpitis abutment teeth
C. Acute diffuse pulpitis D. Temporarily refrain from making a denture
D. Chronic fibrous pulpitis E. Perform implantation
E. Acute focal pulpitis
105. A 23-year-old man complains of acute
101. A 7-year-old practically healthy child was gingival bleeding and unpleasant smell from
undergoing the carious cavity preparation of the mouth that appeared 5 days ago. Objecti-
tooth 46 due to acute median caries. During vely gingival papillae and marginal gingi-
this procedure the mesio-buccal pulp horn va are friable, bright red, swollen, painful,
was accidentally exposed. What treatment and bleed profusely on palpation. Gingival
would be optimal in this case? pockets are 3 mm deep. X-ray shows marked
osteoporosis of the interalveolar septa, peri-
A. Biological approach odontal fissure in the apical areas of the
B. Devital amputation interalveolar septa is enlarged. Cortical plate
C. Devital extirpation is intact. Make the diagnosis:
D. Vital amputation
E. Vital extirpation A. Acute catarrhal gingivitis
B. Acute leukemia
102. A woman complains of pain in her C. Acute necrotizing ulcerative gingivitis
gums, unpleasant smell from her mouth, D. Generalized periodontitis, stage II,
difficult eating, general weakness, low- exacerbated development
grade fever. Objectively her gums are E. Hypovitaminosis C
hyperemic, with areas of ulceration, covered
in necrotic deposit. Microscopy revealed 106. A 27-year-old patient has been referred
fusospirochetosis. Choose the medication for by a prosthodontist for endodontic treatment
etiotropic treatment: of the 45 tooth. Objectively: the 45 tooth
crown is destroyed; the lateral surface of
A. Metronidazole the tongue and the buccal mucosa have
B. Keratoline patches of grayish macerated epithelium sli-
C. Galascorbin ghtly protruding above the mucosa surface at
D. Chlorhexidine the points of direct contact with the 45 tooth.
E. Chymotrypsin The uvula and palatal bars are stagnant-
red in colour; hard palate has papulae
103. A man complains of gingival bleeding surrounded with red margin and covered
that has been persisting for the last 2 years. in grayish epithelium. The submandibular,
Objectively he presents with chronic diffuse cervical, supraclavicular, and subclavicular
catarrhal gingivitis, teeth mobility is of the lymph nodes are enlarged and painless. What
I degree, periodontal pockets are 2-3 mm is the provisional diagnosis?
deep with small amount of serous exudate,
occlusion is markedly traumatic. X-ray shows A. Secondary syphilis
damaged cortical plate, enlarged periodontal B. Chronic recurrent aphthous stomatitis
fissure in the apical areas of the interalveolar C. Lupus erythematosus, patch stage
septa, osteoporosis, and interalveolar septa D. Soft leukoplakia (leucoplakia mollis)
resorption by 1/3 of their height. Make the E. Lichen ruber planus
diagnosis:
107. A 6-year-old boy with congenital heart
A. Generalized periodontitis, stage I, chronic disease (pulmonary artery stenosis) presents
development with suppurative periostitis of the maxilla.
B. Chronic catarrhal gingivitis The child needs surgical treatment. What unit
C. Parodontosis, stage I should he be referred to?
D. Generalized periodontitis, stage I,
exacerbated development A. The pediatric maxillofacial unit
E. Generalized periodontitis, early stage, B. The out-patient unit, no precautions are
chronic development necessary
C. The out-patient unit after preliminary
104. A 55-year-old man came to the cardiological treatment
prosthodontic clinic to have a denture made D. The cardiology unit
for him. Tooth 11 is missing in the pati- E. The out-patient or in-patient unit at the
ent. Two days ago he was released from the discretion of the child’s parents
in-patient unit after a case of myocardial
infarction. What tactics should the dentist 108. Parents of a 3-year-old child complain of
choose? food periodically getting into the child’s nasal
cavity during feeding. Objectively there is a
fissure in the area of the soft palate. Make the
Krok 2 Stomatology (англомовний варiант, iноземнi студенти) 2018 рiк 14

diagnosis: A. Temporary molars


B. Temporary central incisors
A. Isolated partial nonunion of the soft palate C. Temporary lateral incisors
B. Congenital hidden cleft palate D. Temporary canines
C. Isolated nonunion of the hard and soft E. Front teeth
palate
D. Cleft hard palate 113. Due to trauma of the area of teeth 44
E. Combined cleft palate and 45, a 12-year-old boy suffers from the
pathologic displacement of the mandibular
109. A 9-year-old girl complains of persisting alveolar process and rupture of the alveolar
pain in tooth 11 that one month ago sustained process mucosa. What additional examinati-
a trauma resulting in broken crown. The tooth on is necessary to specify the diagnosis?
received no treatment. Objectively the 1/4 of
the tooth crown of 11 is broken off, the remai- A. X-ray of the mandible in frontal and lateral
ning crown is grayish, the dental cavity is not projections
exposed. Percussion is acutely painful. The B. Skull X-ray in axillary projection
mucogingival fold is hyperemic and acutely C. X-ray of the mandible in frontal and Parma
painful on palpation. Make the diagnosis: projection
D. Computed tomography of the mandible
A. Acute traumatic periodontitis E. -
B. Acute diffuse suppurative pulpitis
C. Acute suppurative periodontitis 114. A 12-year-old child presents with
D. Exacerbation of chronic periodontitis temperature 38o C , chills, nausea, vomiti-
E. Chronic granulating periodontitis ng, delirium, and weakness. On the middle
third of the face there is hyperemia in a
110. A patient complains of fever up to 38o C , butterfly-shaped pattern. Regional lymph
headache, pain in the joints anf muscles, vesi- nodes are enlarged and mildly painful. In
cles in the oral cavity, mainly in the frontal blood: leukocytes - 12 · 109 /L, lymphocytes -
part. Eating is sharply painful. For the last 8.0 · 109 /L, ESR- 26 mm/hour. What diagnosis
several years the disease has been recurring should be made?
during wet and windy weather. The patient
often suffers from cases of URTI. Objecti- A. Erysipelatous inflammation
vely: on the buccal, lingual, labial mucosa B. Facial vein thrombophlebitis
there are confluent erosions against the C. Cutaneous actinomycosis
erythematous background, with gray-white D. Streptoderma, submandibular
coating. There are bloody scabs on the vermi- lymphadenopathy
llion border and in the angles of the mouth. E. Acute non-odontogenic maxillary sinusitis
Make the diagnosis:
115. X-ray of the patient shows a focus
A. Erythema multiforme exudativum of bone destruction 3x4 cm in size in the
B. Syphilis mandibular body. The focus is structured
C. Acute aphthous stomatitis as numerous small cavities different in size
D. Acute necrotizing ulcerative stomatitis and shape and separated by septa. Tumor
E. Chronic recurrent aphthous stomatitis puncture yielded brown liquid. What is the
most likely diagnosis?
111. A patient with complaints of toothache
in the left upper jaw has made an appoi- A. Giant cell tumor of the mandible
ntment with a dental clinic. He was diagnosed B. Radicular cyst of the mandible
with chronic periodontitis of the 24 tooth. C. Carcinoma of the mandible
What kind of anesthesia is necessary for pai- D. Soft odontoma of the mandible
nless extraction of the 24 tooth? E. Mandibular ameloblastoma
A. Infraorbital and palatinal anesthesia 116. An orthodontist has been addressed by
B. Tuberal and palatinal anesthesia parents of a 5-year-old child. The child has the
C. Infraorbital and incisor anesthesia 54 tooth extracted, all the other deciduous
D. Tuberal and incisor anesthesia teeth are present. The doctor made a thin-
E. Surface and tuberal anesthesia wall crown for the 55 tooth with interdental
wedge to the 53 tooth. What is the purpose of
112. A 2.5-year-old child is registered for such treatment?
regular check-ups with the orthodontist. The
I stage of physiogical occlusion development A. Prevention of dentition malformation
corresponds with eruption of the following B. Aesthetic restoration
group of temporary teeth: C. Restoration of masticatory efficiency
D. Acceleration of permanent tooth eruption
E. Deceleration of permanent tooth eruption
117. Removable partial dentures for upper
and lower teeth are being made for a 45-year-
Krok 2 Stomatology (англомовний варiант, iноземнi студенти) 2018 рiк 15

old man. Complete anatomical impressions should the dental surgeon give to the patient
were made using ”Ypeen” alginate material. for the procedure of tooth extraction?
What should be used for disinfection of obtai-
ned impressions? A. Mandibular and buccal anesthesia
B. Intraoral infraorbital nerve block
A. 2.5% glutaraldehyde with рH- 7.0 - 8.7 C. Tuberal anesthesia
B. 0.1% desoxone solution D. Mandibular anesthesia
C. - E. Mental nerve block
D. Phenol solution in proportion 1:20
E. 6% hydrogen peroxide solution 122. A 34-year-old man complains of soft ti-
ssues edema in his lower left jaw and fistulae
118. A 48-year-old man complains of gingival in the submandibular area. Teeth 36 and 37
overgrowth (”gums cover the teeth”). The are destroyed. Alveolar mucosa is swollen
patient suffers from epilepsy and takes anti- and hyperemic at the level of 36 and 37. X-ray
convulsant agents. Objectively gingival papi- detected sequestra in the mandibular body
llae are of normal color, dense, with lumpy on the left. What treatment method should
surface; they do not bleed on probing and be chosen in this case?
cover the lower front teeth up to their incisal
surfaces. What is the most likely diagnosis? A. Extraction of teeth 36 and 37 and mandi-
bular sequestrectomy
A. Hypertrophic gingivitis, fibrous form, B. Extraction of teeth 36 and 37
degree III C. Mandibular sequestrectomy
B. Hypertrophic gingivitis, edematous form, D. Puncture of the inflamed area
degree III E. Antibacterial treatment
C. Gingival fibromatosis
D. Hypertrophic gingivitis, fibrous form, 123. A 42-year-old man was delivered to the
degree II hospital in the severe condition: inert, body
E. Hypertrophic gingivitis, edematous form, temperature is 39.1o C , there is acutely painful
degree II infiltration of the mouth floor and submandi-
bular area on the right. The skin over the
119. A 21-year-old man came to the denti- infiltration is turgid and cyanotic. Palpati-
st complaining of general weakness, muscle on detects crepitus under the skin. What di-
pain, body temperature up to 38.3o C , indi- agnosis can be made in this case?
gestion, excessive salivation, and rashes in
the oral and nasal cavities, urethra, on the wi- A. Ludwig’s angina (suppurative-necrotic
ngs of the nose, and in the interdigital folds. phlegmon of the mouth floor)
These symptoms appeared after ingestion of B. Adenophlegmon of the mouth floor
milk during the patient’s stay in the village. C. Malignant tumor of the mouth floor
What is the most likely diagnosis? D. Actinomycosis of the mouth floor
E. Odontogenic phlegmon of the mouth floor
A. Murrain
B. Herpetic stomatitis 124. After a blow to the temporomandibular
C. Herpes zoster joint the patient developed facial hematoma,
D. Behcet’s disease the joint is difficult to move, mandibular
E. Infectious mononucleosis mobility is reduced. Attempts to open the
mouth wide are painful. What examination
120. A 28-year-old man complains of pai- should be performed to make the diagnosis?
nless sore in his mouth that persists despi-
te the attempts at self-treatment. Objecti- A. Bilateral X-ray of the temporomandibular
vely the regional lymph nodes on the left joint with mouth open and closed
are enlarged and painless. Mucosa of the left B. Limit the joint mobility
cheek presents with round ulcer, 1 cm in di- C. X-ray and consultation with the neurologist
ameter, with raised margins and cartilage-like D. Rheoencephalography and consultation
infiltration in its basis. The surface of the ulcer with the neurologist
is colored meat red and painless on palpation. E. Panoramic dental X-ray
What is the most likely diagnosis?
125. A 38-year-old man after a domestic acci-
A. Primary syphilis dent complains of pain and mobility of his
B. Cancer upper teeth, problems with eating. Objecti-
C. Secondary syphilis vely: soft tissues edema. The 11 and 21 teeth
D. Lupus vulgaris are displaced towards the palate, mobile
E. Decubitus ulcer (II degree), painful on percussion. Mucosa
surrounding the affected teeth is hyperemic
121. A 25-year-old woman made an appoi- and swollen. X-ray demonstrates widened
ntment with the dental surgeon for oral cavi- periodontal fissure of the 11 and 21. Choose
ty sanation. Objectively the crown of tooth 37 the treatment method:
is destroyed by 2/3. Gingival mucosa around
tooth 37 is without changes. What anesthesia
Krok 2 Stomatology (англомовний варiант, iноземнi студенти) 2018 рiк 16

A. Setting of the teeth and their fixation with is NOT a part of typical procedure of tooth
a flat occlusal splint extraction with forceps?
B. Extraction of the 11 and 21 teeth
C. Reimplantation of the 11 and 21 teeth A. Applying tip of forceps jaw to the edge of
D. Immobilization or mouthguard alveolar process
E. Removal of tooth pulp in the 11 and 21 B. Applying forceps jaw to the tooth
teeth C. Pushing forceps jaw to the cementoenamel
junction
126. A 4-year-old child has developed acute D. Closure of forceps handles
spontaneous pain in the tooth on the lower E. Tooth dislocation and extraction from the
right jaw, which aggravates on biting. Objecti- socket
vely: in the 85 tooth there is a deep carious
cavity non-communicating with the dental 131. A patient needs the 36 tooth extracted.
cavity. Probing is sharply painful at all poi- After administering anesthesia the doctor
nts of the cavity floor. Painful reaction to cold started applying the elevator. However,
water stimulus and percussion is observed; immediately after that the patient suddenly
mucosa surrounding the 85 is hyperemic. paled, complained of dizziness, ear noise, and
Submandibular lymphadenitis is detected. blackout and slid down in the chair. What is
Make the provisional diagnosis: the most likely diagnosis?
A. Acute pulpitis complicated with peri- A. Unconsciousness
odontitis B. Anaphylactic shock
B. Acute serous periostitis C. Collapse
C. Acute serous periodontitis D. Shock
D. Acute suppurative pulpitis E. Hypoglycemic coma
E. Exacerbation of chronic periodontitis
132. A child is diagnosed with congeni-
127. During Eschler-Bittner test the profile tal cleft in the soft palate and posteri-
of a 12-year-old girl with posterior occlusion or part of the hard palate. What type of
has shown some improvement. Specify the anesthesia should be given to the patient for
condition that resulted in the development of uranostaphyloplasty?
posterior occlusion in this patient:
A. Intubation narcosis
A. Mandibular underdevelopment B. Intravenous narcosis
B. Maxillary overdevelopment C. Anesthesia mask
C. Mandibular underdevelopment and maxi- D. Infiltration anesthesia
llary overdevelopment E. Conduction anesthesia
D. Mandibular overdevelopment
E. Maxillary underdevelopment 133. A 6-year-old child complains of pain
and edema in the upper right jaw, body
128. Parents of an 8-year-old girl complain of temperature up to 37.9o C , and deterioration
their child having an aesthetic defect of her of general well-being. Symptom onset was 3
teeth. Objectively the patient’s lower face is days ago. Objectively the face is asymmetric
shortened. Her chin protrudes forwards and due to soft tissue edema of buccal and
her upper lip is sunken. During teeth closure infraorbital regions on the right. The crown
the deep underbite becomes apparent. of 54 is destroyed by 1/2, percussion is pai-
Mesio-occlusion is observed in the lateral nful; the tooth previously had been treated
areas. Choose the apparatus optimal for the for complicated caries. On the palatine side
treatment: of the affected tooth area there is a painful
infiltration with fluctuation in its center; the
A. Frankel functional regulator - 3 tissues over the infiltration are hyperemic.
B. Frankel functional regulator - 2 Make the provisional diagnosis:
C. Osadchy apparatus
D. Andresen-Haupl activator A. Acute suppurative periostitis of the maxilla
E. Frankel functional regulator - 1 originating from tooth 54
B. Acute serous periostitis of the maxilla
129. During or immediately after an injecti- originating from tooth 54
on, certain local complications can develop. C. Acute odontogenic osteomyelitis of the
What is NOT one of those complications? maxilla
D. Exacerbation of chronic periodontitis of 54
A. Mucosal necrosis E. Chronic odontogenic osteomyelitis of the
B. Dermal ischemia maxilla
C. Diplopia
D. Functional paralysis or paresis of facial 134. A 27-year-old man presents with missi-
muscles ng crown of 11. Objectively teeth 21 and 12
E. Damage to a blood vessel by the needle are intact; intraoral spot film X-ray shows the
root of 11 to be filled to the apex, no changes
130. What manipulation of those listed below in the periapical tissues, no pathologic mobi-
Krok 2 Stomatology (англомовний варiант, iноземнi студенти) 2018 рiк 17

lity. What construction of the denture should socket and in the area of zygomaticoalveolar
be recommended for this patient? crest. What is the most likely diagnosis?
A. Metal stump inlay with overlaying A. Displaced fracture of the temporal bone
porcelain-fused-to-metal crown B. Le Fort II maxillary fracture
B. Stump inlay with overlaying swaged crown C. Nondisplaced fracture of the temporal bone
C. Stump inlay with overlaying plastic crown D. Le Fort I maxillary fracture
D. Stump inlay with overlaying full cast metal E. Zygomatic arch fracture
crown
E. Remove the root of 11 and perform 139. A 14-year-old boy complains of rapid
implantation wearing-off of tooth crowns. Objectively:
tooth crowns are worn-off by 1/3. Enamel
135. A 54-year-old patient complains of easily chips off and is pale gray in color. Make
frequent crunching sound in the right the diagnosis:
temporomandibular joint, which developed
one month ago. In the morning the crunching A. Stainton-Capdepont syndrome
is more frequent and decreases towards the B. Dentinogenesis imperfecta
evening. Objectively: the face is symmetrical, C. Fluorosis
the skin over the joint is without changes, the D. Systemic hypoplasia
mouth opens by 2.9 mm. What is the most E. Focal hypoplasia
likely diagnosis in this case?
140. A 63-year-old man complains of pain
A. Arthrosis in the area of maxillary mucogingival fold
B. Acute arthritis caused by using a removable laminar denture.
C. Temporomandibular joint dislocation Objectively: in the area of the mucogingi-
D. Chronic arthritis val fold there is a trophic ulcer with swollen
E. Temporomandibular joint pain dysfunction margins and hemorrhaging floor. Make the
syndrome diagnosis:

136. On examination of a 27-year-old pati- A. Denture-related stomatitis


ent the tip of the dental probe caught on B. Toxic chemical stomatitis
the fissures of the 36, 37, and 38 teeth. C. Toxic infectious stomatitis
Margins of the enamel defect are dark, the D. Allergic contact stomatitis
surface is coarse. Teeth transillumination wi- E. Greenhouse effect
th photopolymer lamp revealed the defect to
be limited to the enamel. What is the most 141. Objective examination of a 10-year-old
likely diagnosis? child revealed slight hyperemia, infiltration,
and dryness of the whole surface of the vermi-
A. Chronic superficial caries llion border. Architectonics of the lips is di-
B. Chronic median caries sturbed. Dryness and contracted sensation
C. Acute superficial caries are observed in the lips, especially during cold
D. Chronic initial caries seasons. Make the provisional diagnosis:
E. Acute initial caries
A. Meteorological cheilitis
137. A 68-year-old patient addressed a surgi- B. Atopic cheilitis
cal department of a dental clinic for extracti- C. Allergic contact cheilitis
on of the 45 tooth. During procedure the D. Exfoliative cheilitis
patient developed burning retrosternal pain E. Cheilitis of microbial origin
attack irradiating to the left shoulder, scapula,
hand. The skin is pale, BP is 140/100 mm 142. A 24-year-old woman made an appoi-
Hg, heart rate is rapid. Skin hyperplasia can ntment with the dental surgeon for extraction
be observed in the Zakharin-Head’s zones. of tooth 38. What anesthesia should be gi-
What emergency condition did the patient ven to the patient for the procedure of tooth
develop? extraction?

A. Angina pectoris attack A. Torusal


B. Heart failure B. Mandibular
C. Bronchial asthma attack C. Tuberal
D. Hypertensic crisis D. Infiltration
E. - E. Plexus

138. After sustained trauma a man developed 143. During application of tuberal anesthesia
nose bleeding, reduced mouth opening, the patient developed rapidly increasing ti-
sensation of paresthesia in the right infraorbi- ssue edema and reduced mouth opening.
tal region and lower eyelid. Objectively the What resulted in such a condition?
face is asymmetric due to concave right
temporal region; step deformity symptom is
observed in the middle of the lower right eye
Krok 2 Stomatology (англомовний варiант, iноземнi студенти) 2018 рiк 18

A. Vascular trauma
B. Muscle trauma during anesthesia applicati- A. Collapse
on B. Anaphylactic shock
C. Nerve trunk trauma C. Allergic response to the anesthetic
D. Intolerance to the anesthetic D. Pain shock
E. Anaphylactic shock E. Vertigo
144. A woman came to the dental surgeon wi- 149. An ambulance has delivered an 8-
th complaints of teeth mobility. After objecti- year-old child to an admission room. An
ve examination and X-ray analysis she was oral surgeon has made the following di-
diagnosed with generalized periodontitis of agnosis: odontogenic phlegmon of the right
stages I and II. Which teeth of those affected submandibular area. What surgical approach
by periodontitis should be extracted? would be advicable for surgical treatment of
this phlegmon?
A. With degrees 2-3 of tooth mobility
B. Intact teeth A. Dissection in the submandibular area,
C. Teeth with painful percussion parallel to the mandible
D. With degree 1 of tooth mobility B. Dissection parallel to the torus mandi-
E. Carious teeth bularis
C. Dissection around the mandibular angle
145. After the inflammatory process in the D. Dissection along the lower neck fold
parotid area a woman developed frequent E. Dissection in the area of pterygomandi-
pain attacks resembling electric current in bular fold
her face on the right. The attacks last for 15-
20 minutes. The most likely diagnosis is: 150. A 42-year-old man came to the
prosthodontics clinic to have dental
A. Trigeminal neuralgia prosthesis made for him. Objectively teeth
B. Tympanic plexus neuralgia 34, 35, 36, 43, and 16 are absent in this pati-
C. Trigeminal neuritis ent. During examination alginate impressions
D. Exacerbation of chronic maxillary sinusitis for diagnostic models were obtained. What
E. Exacerbation of chronic osteomyelitis should be used for disinfection of obtained
impressions?
146. Mother and her 11-year-old daughter
came to the medical station. According to the A. 0.5% sodium hypochlorite solution
mother her daughter has fallen when playing B. 3% hydrogen peroxide solution
outdoors. The doctor determined the patient C. 10% hydrogen peroxide solution
to have an isolated mechanical injury of soft D. Sterillium
tissues in her cheek with damaged skin. Make E. 3% sodium hypochlorite solution
the diagnosis:
151. A 36-year-old woman needs a dental
A. Abrasion prosthesis. Objectively there is a carious cavi-
B. Contusion ty on the mesial masticatory surface of tooth
C. Bruice 46, interdental contact is disturbed. Dental
D. Hematoma inlay is to be made for this patient. According
E. Wound to Black’s classification of dental caries this
cavity is class:
147. The patient with shallow vestibule of
mouth and edentulous mandible underwent a A. 2
surgery: a mucoperiosteal flap was relocated B. 3
from the alveolar ridge to the body of the C. 4
mandible and fixed with denture acting as a D. 5
bandage. What surgical procedure was used E. 1
for vestibular deepening?
152. A 45-year-old man complains of pain
A. Rumpel and crepitation in the temporomandibular
B. Kazanjian joint during the movements of the lower
C. Thiersch jaw. Objectively: the face is symmetrical,
D. Trauner the mouth opens with slight displacement
E. Rhermann to the left. Dentition is intact. On occlusi-
ography there were detected centric and
148. A 7-year-old girl received conduction eccentric supracontacts. What treatment
anesthesia with 2% articaine solution for methods should be applied in the first place?
extraction of tooth 16. She has no history of
allergies. After receiving anesthesia the pati-
ent complained of weakness, she developed
skin pallor, cyanosis, and nausea. Her blood
pressure dropped significantly. The patient is
conscious. What is the provisional diagnosis?
Krok 2 Stomatology (англомовний варiант, iноземнi студенти) 2018 рiк 19

A. Selective teeth shaving A. Fixation of the lower jaw and tongue with
B. Mouthguard for muscle relaxation standard Entin’s head-chin strap
C. Appliances that limit mouth opening B. Fixation of the tongue to the patient’s
D. Mouthguards that increase the height of collar
central occlusion C. Tracheostomy and artificial pulmonary
E. Lower jaw immobilization ventilation
D. Removal of foreign bodies from the oral
153. A 45-year-old patient after administrati- cavity
on of local anesthesia in preparation for oral E. Excision of injured mucosal flaps
surgery has suddenly felt unwell, developed
increasing edema of laryngeal mucosa and 157. The hospital received a 19-year-old pati-
respiration disorder. The dentist stopped the ent injured in a landmine explosion wi-
manipulations in the oral cavity. What type of th an open displaced mandibular fracture
asphyxia developed in the patient? in the area of the left mandibular angle
and a contused lacerated wound of the
A. Stenotic left submandibular area. The patient is in
B. Dislocational moderately severe condition, pulse is 80/min.,
C. Valvular blood pressure is 110/80 mm Hg. What
D. Obturative anesthesia should be given to the patient for
E. Aspiration initial surgical d-bridement?
154. A 19-year-old young man, who was wai- A. Endotracheal anesthesia
ting for the appointed time at the denti- B. Intravenous anesthesia
st’s, suddenly developed an attack: his face C. Neuroleptanalgesia
became purple, bloody foam flowed from D. Conduction anesthesia with premedication
his mouth (bitten tongue), pupils were di- E. Conduction anesthesia
lated and unresponsive to light, the patient
developed first tonic then clonic convulsi- 158. A 5-year-old child developed a
ons that stopped spontaneously, after that he hemorrhage after pulp extirpation of 74 due
calmed down and fell asleep quickly. What to exacerbated chronic pulpitis. The child
happened with the patient? suffers from Von Willebrand disease. What
actions should be taken by the dental surgeon
A. Epileptic seizure to stop bleeding?
B. Sympathoadrenal crisis
C. Spasmophilia A. To hospitalize the child to the hematologi-
D. Morgagni-Adams-Stokes syndrome cal unit
E. Pulmonary embolism B. To hospitalize the child to the maxillofacial
unit
155. A victim of a traffic accident was deli- C. To place sutures on the mucosa
vered into the admission room. The patient is D. To plug the cavity with epsilon-
supine and unconscious. His skin is cyanotic, aminocaproic acid dressing
respiration is extremely labored, mucosa is E. To plug the cavity with hemostatic sponge
pale, blood clots are accumulated in the oral
cavity. The patient is diagnosed with displaced 159. Parents of a 9-year-old child came to the
bilateral mandibular fracture. How should dentist complaining that their child presents
this patient be transported? with enlarged cervical lymph nodes on the ri-
ght. During examination palpation revealed
A. In the prone position on a soft stretcher the lymph nodes in the right submandibular,
B. Positioned on the side on a rigid stretcher cervical, supraclavicular, and infraclavicular
C. In the sitting position with the head thrown areas to be enlarged up to 2-2.5 cm in di-
back ameter, painless, non-matted together, and
D. In the sitting position with air tube inserted non-fused to the skin (resemble ”potatoes
into the upper airways in a sack”). The parents note rapid fatigabi-
E. Positioned on the side on a soft stretcher lity and night sweats in their child. What
additional examinations should the child be
156. A victim of a traffic accident was deli- referred for?
vered into the admission room. The patient is
supine and unconscious. His skin is cyanotic, A. Puncture biopsy of the lymph nodes
respiration is extremely labored, mucosa is B. Pirquet and Mantoux tests
pale, blood clots are accumulated in the oral C. Wassermann test
cavity. The patient is diagnosed with displaced D. CT of the cervical spine
bilateral mandibular fracture. What measures E. Clinical blood and urine tests
should be taken to prevent complications in
this case? 160. A 45-year-old patient complains of inabi-
lity to properly masticate due to the loss of
lateral teeth. The 17, 16, 15, 25, 26, 27, 37, 36,
35, 44, 45, and 46 teeth are missing. The retai-
ned teeth exhibit the I-II degree of mobi-
Krok 2 Stomatology (англомовний варiант, iноземнi студенти) 2018 рiк 20

lity. The patient is diagnosed with generali- probing causes severe pain, percussion of the
zed periodontitis. Kennedy class I dentiti- 17 tooth is painful. X-ray: there is slight wi-
on defects are observed. What construction dening of the periodontal fissure near the root
would be optimal in the given case? apex. Electric pulp test - 70 microamperes.
What final diagnosis can be made?
A. Clasp-retained (bugel) removable partial
denture with splinting elements A. Chronic gangrenous pulpitis
B. Partial laminar denture B. Chronic fibrous pulpitis
C. Elbrecht’s dental splint C. Acute purulent pulpitis
D. Mamlok’s dental splint D. Chronic fibrous periodontitis
E. Cantilever dental bridges E. Exacerbation of chronic fibrous periodonti-
tis
161. A 70-year-old patient addressed a hospi-
tal with complaints of poorly stabilized 165. Medical committee registers the patients,
complete removable dentures of the upper who for a long time lived in an area polluted
and lower jaws. What method of artificial with radiation. The patients are advised on
teeth arrangement is preferable in making the diet that will quickly purge the body from
of a new complete removable denture? radionuclides. The portion of products rich
in pectine should be increased in their diet.
A. According to individual occlusal curves Name these products:
B. According to disocclusal planes
C. According to standard occlusal curves A. Fruits and vegetables
D. According to spherical occlusal curves B. Meat products
E. According to prothetic occlusal planes C. Pasta
D. Dairy products
162. A 2.5-year-old child has fever up to E. Baked goods
38.5o C , low appetite, rashes in the oral cavity.
The disease onset was 3 days ago. Objecti- 166. A 44-year-old man came to extract
vely: the skin of the perioral area is covered in destroyed tooth 24. Objectively his face is
scarce vesicles with clear content. Within the symmetrical, the crown of 24 is destroyed by
oral cavity on the buccal and lingual mucosa 2/3, percussion is painless. Gingival mucosa
there are sharply painful erosions, 2-3 mm surrounding the tooth is unchanged. X-ray
in size, with white coating and hyperemic shows enlarged periodontal fissure in the area
crown. The gums are swollen, hyperemic. The of the root apex. What is the most likely di-
submandibular lymph nodes are enlarged, agnosis?
painful on palpation. Make the diagnosis:
A. Chronic fibrous periodontitis of 24
A. Acute herpetic stomatitis B. Chronic granulating periodontitis of 24
B. Stevens-Johnson syndrome C. Chronic granulomatous periodontitis of 24
C. Erythema multiforme exudativum D. Exacerbation of chronic periodontitis of 24
D. Stomatitis with the background of infecti- E. Chronic gangrenous periodontitis of 24
ous mononucleosis
E. Stomatitis with the background of chi- 167. A 46-year-old man complains of constant
ckenpox losing of a filling in his lower right tooth.
Objectively: in the 16 tooth on the approxi-
163. A 25-year-old man complains of short- mal masticatory surface there is a defect of
term pain in the tooth on the lower ri- crown hard tissues at 1/3. The tooth has no
ght jaw during eating sweet, hot, and cold discoloration, percussion is painless. What
food. Objectively: in the 36 tooth on the construction should be prescribed?
distal surface there is a carious cavity non-
communicating with the dental cavity, dentin A. Inlay
is softened. Probing of the cavity floor is pai- B. Porcelain-fused-to-metal crown
nful, percussion is painless. Electric pulp test C. Semi-crown
is 16 microamperes. Make the final diagnosis: D. 3/4 crown
E. Plastic crown
A. Acute deep caries
B. Acute median caries 168. A 10-year-old boy complains of acute
C. Pulpal hyperemia pain attacks in the area of his upper left teeth.
D. Chronic gangrenous pulpitis The toothache persisted for a night. Objecti-
E. Chronic fibrous periodontitis ve examination revealed a carious cavity on
the masticatory surface of the 26 tooth within
164. A 30-year-old woman came to the dentist parapulpar dentin. Probing is sharply painful
with complaints of uncomfortable sensation at all points of the cavity floor. Markedly
of pressure in her upper right tooth, which positive reaction to cold water stimulus is
aggravates in response to hot stimulus, and observed. Select the most likely diagnosis:
foul smell from the mouth. Objectively: there
is a deep carious cavity in the 17 tooth, whi-
ch communicates with the tooth cavity. Deep
Krok 2 Stomatology (англомовний варiант, iноземнi студенти) 2018 рiк 21

A. Acute diffuse pulpitis patient?


B. Acute serous periodontitis
C. Acute suppurative pulpitis A. Biological approach
D. Acute suppurative periodontitis B. Vital amputation
E. Acute local pulpitis C. Vital extirpation
D. Devital amputation
169. In a state of inebriation a 36-year-old E. Devital extirpation
woman suffered a domestic accident 4 hours
ago. She was delivered to the maxillofacial 173. A 37-year-old woman came to the dentist
surgery unit. On examination she was provisi- with complaints of brief attacks of toothache
onally diagnosed with a deep perforating inci- caused by eating sweets. Objectively there is
sed wound of the lateral surface of her face a shallow carious cavity within enamel. On
with injured perioral area and parotid gland. probing cavity walls and bottom are coarse;
The woung edges are gaping. This gaping is there is no response to thermal stimuli. Make
caused by: the diagnosis:
A. Location of facial muscles A. Acute superficial caries
B. Wound communication with the oral cavity B. Endemic fluorosis
C. Specifics of facial innervation C. Enamel hypoplasia
D. Location of masticatory muscles D. Acute median caries
E. Depth of the wound E. Chronic median caries
170. The maxillofacial surgery unit recei- 174. A 24-year-old patient came to the dentist
ved a patient with complaints of inability complaining of chalky lesions on the front
to close his mouth. This condition occurred teeth. Objectively teeth 13, 12, 11, 21, 22,
when the patient was biting an apple. Objecti- and 23 present with chalky lesions separated
vely there is a frightened expression on the by areas of healthy unchanged enamel. Lesi-
patient’s face, the mouth is open wide, the on surface is coarse; there is no response to
chin is displaced to the left, salivation is thermal stimuli. Childhood years of the pati-
observed. Palpation through the external ent were spent in the area with fluorine level
acoustic meatus detected no movements of of 1.8 mg/L in drinking water. Make the di-
the right articular head. What is the most li- agnosis:
kely diagnosis?
A. Endemic fluorosis
A. Right temporomandibular joint dislocation B. Enamel hypoplasia
B. Fracture of the mandibular process C. Enamel hyperplasia
C. Acute temporomandibular arthritis D. Acute superficial caries
D. Temporomandibular joint pain dysfunction E. Chronic superficial caries
syndrome
E. Bilateral temporomandibular joint di- 175. A 48-year-old patient has addressed a
slocation hospital with complaints of defects in the
paragingival area and slight sensitivity to
171. A 19-year-old girl addressed an oncologi- thermal stimuli. Objectively there are hard
st with complaints of slowly growing tumor- tissue defects that resemble a wedge with
like mass on the tip of her tongue. The mass smooth polished walls on the precervical
was first noticed 5-6 years ago. The pati- vestibular surface of the 23 and 24 teeth.
ent requested no medical help. Objectively: Thermal test is slightly positive. What is the
there is a pale pink round growth with wi- most likely diagnosis?
de pedicle on the tongue apex; the growth
is painless, elastic; there are no changes of A. Cuneiform defect
mucosa surrounding the pedicle. Submandi- B. Enamel necrosis
bular lymph nodes cannot be palpated. What C. Acute deep caries
kind of tumor is it? D. Enamel erosion
E. Endemic fluorosis
A. Papilloma
B. Atheroma 176. Parents of a 7-year-old child addressed a
C. Fibroma hospital with complaints of their child havi-
D. Lipoma ng no permanent teeth in the front area of
E. Keratoma the mandible. Anamnesis states that the fi-
rst deciduous teeth erupted at the age of
172. During carious cavity preparation in a 11 months. Objective clinical examination
20-year-old man, the pulp-chamber floor was revealed the following: appearance is without
accidentally perforated and horn of the pulp changes; milk occlusion; there are physiologi-
was exposed. On the carious cavity floor there cal diastema and tremata; edge-to-edge inci-
is a point-like puncture surrounded with whi- sor contact. What preliminary diagnosis can
te predentin. Pink pulp can be seen through be made according to Kalvelis classification?
the perforation, pulp probing is acutely pai-
nful. What treatment should be given to the
Krok 2 Stomatology (англомовний варiант, iноземнi студенти) 2018 рiк 22

A. Retarded eruption
B. Supernumerary tooth A. Mamlok’s dental splint
C. Adentia B. Removable segmented splint for the front
D. Dystopia teeth
E. Hypoplasia C. Cap splint
D. Splint with embrasure clasps
177. A 5.5-year-old child is undergoi- E. Semicrown splint
ng preventive examination. There are no
complaints from the patient. Objectively: in 182. A woman complains of spontaneous
the precervical area of buccal surface of the attacks of acute pain, with practically no
75 tooth there is an enamel patch with loss intermissions and irradiation into the temple;
of natural glossiness. The enamel surface is cold water slightly mitigates the pain. In tooth
coarse, painless on probing. Tooth percussi- 26 examiantion revealed deep carious cavity
on is painless. Thermodiagnosis is negative. non-communicating with the dental cavity.
Damaged enamel stains with 2% water soluti- Probing of the cavity bottom is acutely pai-
on of methylene blue. Make the diagnosis: nful, the tooth is tender on vertical percussi-
on. Make the provisional diagnosis regarding
A. Acute initial caries tooth 26:
B. Enamel hypoplasia
C. Fluorosis A. Acute suppurative pulpitis
D. Acute superficial caries B. Acute diffuse pulpitis
E. Chronic initial caries C. Pulpal hyperemia
D. Chronic fibrous pulpitis
178. A 68-year-old man has removable parti- E. Chronic hypertrophic pulpitis
al dentures made for him. At the stage of
checking the denture construction, occlusal 183. A 78-year-old man complains of a painful
contact in the lateral areas is cusp-to-cusp sore in his mouth that has been persisting for
and frontal area has sagittal fissure. What mi- 2 months already. The patient is a smoker.
stake was made by the doctor? Objectively on the buccal mucosa on the ri-
ght there is a shallow ulcer up to 1.5 cm in
A. Anterior occlusion is fixed size with lumpy floor and uneven margins.
B. Lateral occlusion is fixed There are yellowish granules on its periphery.
C. Prothetic plane is designed incorrectly Palpation is painful, the lesion is soft. Regi-
D. Height of centric occlusion is understated onal lymph nodes are enlarged, painful, and
E. Height of centric occlusion is overstated matted together. What is the provisional di-
agnosis?
179. An 80-year-old man needs a removable
partial denture for the lower jaw. How should A. Tuberculous ulcer
the denture boder be located relative to the B. Cancerous ulcer
natural teeth? C. Decubitus ulcer
D. Trophic ulcer
A. Covers the crowns by 2/3 of their height E. Hard chancre
B. Covers the crowns by 1/3 of their height
C. Fully covers the crowns 184. A 37-year-old patient has symmetrical
D. Reaches dental cervices face; the mucosa in the area of the 12 tooth
E. Located below dental cervices root apex projection is pale pink; palpation is
painless; the tooth crown is destroyed by 1/3;
180. A 52-year-old woman needs a dental percussion is painless. X-ray: the root canal of
prosthesis. Upon objective examination a the 12 tooth is filled to the apex; granuloma 4
removable partial laminar denture is determi- mm in diameter is located near the root apex.
ned to be optimal denture construction in her Choose the method of surgical treatment:
case. What material should be applied to the
impression tray edges? A. Granuloma removal with root apex resecti-
on
A. Orthocor B. Root hemisection
B. Plaster C. Coronary radicular tooth separation
C. Wax D. Root amputation
D. Silast E. Tooth extraction
E. Stens
185. A 48-year-old man presents with verruci-
181. A 46-year-old man, a teacher, complai- form, dense, gray-white growths on the buccal
ns of lower teeth mobility that impedes the mucosa. The growths protrude above the
process of biting. Objectively the dentition is neighbouring tissues and are surrounded by
uninterrupted, front teeth demonstrate the II keratinized gray-white spots that cannot be
degree of mobility. X-ray shows straight and scraped off. Make the provisional diagnosis:
filled root canals of 32, 31, 41, and 42. What
appliance will stabilize front teeth while retai-
ning their aesthetic appearance?
Krok 2 Stomatology (англомовний варiант, iноземнi студенти) 2018 рiк 23

A. Verrucous leukoplakia response to thermal stimuli. Percussion of


B. Erosive leukoplakia tooth 24 is acutely painful. X-ray shows no
C. Bowen’s disease pathologic changes of periapical tissues in
D. Erythroplasia of Queyrat the area of root apices of 24. What is the most
E. Papillomatosis likely diagnosis?
186. During preventive examination a A. Acute serous periodontitis
patient was diagnosed with precancerous B. Acute suppurative periodontitis
hyperkeratosis of the lower lip vermillion C. Acute suppurative pulpitis
border. What treatment should be prescri- D. Acute diffuse pulpitis
bed? E. Exacerbation of chronic periodontitis
A. Surgical removal of the focus within 190. A 22-year-old man complains of tearing,
healthy tissues throbbing, constant, intensifying pain in the
B. Surgical removal of the focus within healthy tooth on the upper left jaw. The pain has been
tissues + close-focus roentgenotherapy persisting for 4 days. Objectively tooth 26 has
C. No treatment is necessary deep carious cavity non-communicating with
D. Surgical removal of the focus within healthy the dental cavity. Probing is painless. Percussi-
tissues + chemotherapy on is acutely painful. The tooth is mobile.
E. Palliative treatment Mucogingival fold in the area of tooth 26 is
painful on palpation. Make the diagnosis:
187. A 63-year-old man complains of fever
and multiple painful rashes in his oral cavi- A. Acute suppurative periodontitis
ty and on his face and torso. 3-4 days before B. Acute serous periodontitis
the rashes appeared he had noticed a burni- C. Acute suppurative pulpitis
ng sensation followed by sharp shooting D. Exacerbation of chronic periodontitis
pain resembling that which occurs during E. Acute local pulpitis
lumbago. The patient takes cytotoxic drugs
for leukemia treatment. On examination 191. A 22-year-old patient has suffered uni-
there are multiple aphthae located in a row lateral linear fracture in the area of the gonial
on the vermillion border and labial, lingual, angle. Immobilization was provided with full
and buccal mucosa on the right. The aphthae dental brace with loops and intermaxillary
have hyperemic borders, they are not fused elastic expansion. Recovery was uncompli-
together and are sharply painful on palpati- cated. The brace should be removed after:
on. The right side of the face presents with
erythematous spots, vesicles, and erosions. A. 3 weeks
Make the provisional diagnosis: B. 2 weeks
C. 1 week
A. Herpes zoster D. 10 days
B. Toxic allergic dermatostomatitis E. -
C. Erythema multiforme exudativum
D. Secondary syphilis 192. A 27-year-old patient was provisi-
E. Chronic recurrent herpes onally diagnosed with acute suppurative
odontogenic maxillary sinusitis. What radi-
188. A 34-year-old man presents with persisti- ology method would be the most informative
ng dull pain in his tooth, which aggravates on in this case?
biting. One week ago the tooth was treated
for deep caries. Objectively on the masti- A. Computed tomography
catory surface of tooth 36 there is a filling, B. X-ray
percussion is painful, there is a supracontact C. Panoramic radiography
observed in the area of 36. X-ray shows D. Spot-film radiography
unchanged periodontium. What mistake was E. -
made when tooth 36 was filled?
193. A 22-year-old man presents with swollen
A. High filling and hyperemic mucosa of the retromolar
B. Filling without insulation layer area; tooth 38 is covered with hood-shaped
C. Gingival attachment is disturbed gingival flap that discharges pus on palpati-
D. Insulation layer exceeds borders of the on; body temperature is 37.5o C . What urgent
dentinoenamel junction aid should be given to the patient in this case?
E. Filling without medicinal substance sealed
inside A. Gingival flap incision and antibacterial
treatment
189. A 35-year-old man complains of persi- B. Gingival flap excision
sting pain in tooth 24, which intensifies on C. Extraction of tooth 38
biting. Objectively on the distal masticatory D. Antibiotic treatment
surface of tooth 24 there is a deep carious E. Gingival flap incision
cavity filled with food debris. Percussion of
cavity bottom is painless, there is no pain 194. A 43-year-old man came to the maxi-
Krok 2 Stomatology (англомовний варiант, iноземнi студенти) 2018 рiк 24

llofacial surgeon with complaints of aesthetic neoplasm in the left parotid-masticatory regi-
defect. Examination revealed excessive on. Skin over the tumor is without discolorati-
accumulation of adipose tissue in the pati- on. The tumor is painless, but when the head
ent’s neck and upper torso, which resembles bends down the tumor increases in size and
collar with unclear magins; neck mobility assumes bluish coloring. What disease can be
is reduced. Family history shows the same suspected in the child?
symptoms to be present in the patient’s
father. Make the provisional diagnosis: A. Hemangioma
B. Fibroma
A. Madelung’s deformity C. Atheroma
B. Lipoma D. Lymphangioma
C. Neurofibromatosis E. Cyst of the parotid gland
D. Fibroma
E. Lymphangioma 198. An 11-year-old child presents with
protrusion of the upper front teeth, there
195. A 73-year-old man is registered for are tremata and diastema between the chi-
regular check-ups in an oncological clinic ld’s teeth. What apparatus can be used for
after completion of the combined treatment treatment of this pathology?
for oral mucosa cancer stage II (radiation
therapy and surgery). During one of the routi- A. Osadchy
ne check-ups an area of exposed mandibular B. Brukle
bone is detected. There are no inflammatory C. Frankel, type 3
changes of surrounding mucosa. A fistula D. Vasylenko
tract with soft granulation is detected. Mandi- E. Mershon
bular X-ray shows a sequestrum without clear 199. A 40-year-old patient complains of pain
margin between healthy and necrotic bone. in the tragus area, clicking during mouth
What is the most likely provisional diagnosis? opening, stuffed ears. Objectively the face
A. Mandibular osteoradionecrosis is symmetrical, mouth opening path is strai-
B. Acute purulent mandibular osteomyelitis ght. Dentition defect can be estimated as the
C. Posttraumatic mandibular osteomyelitis I class by Kennedy; the 18, 17, 16, 26, 27, 28
D. Relapse of oral mucosa cancer teeth are missing. In this case the load would
E. Chronic mandibular periostitis be the most traumatizing for the following
anatomical structure:
196. A 19-year-old patient came to a
dentofacial clinic with complaints of pain A. Interarticular disk
in the gonial angle on the right, impaired B. Articular capsule
mouth opening and painful chewing. The si- C. Articular head
gns had been persisting for 5 days, emerged D. Distal slope of the articular tubercle
spontaneously and had been aggravating E. Floor of the temporal bone socket
gradually. Mandibular contracture is of the 200. A 38-year-old man complains of sensati-
III degree. On examination of the oral cavity: on of a foreign body on his tongue and
hyperemia, edema of the retromolar space on development of gag reflex during talking.
the right, hood-shaped mucosa from under The signs appeared after the prolonged taki-
which pus is being discharged and 2 tooth ng of antibiotics. Objective examination
tubercles can be detected. X-ray shows obli- detected thickened and pigmented filiform
que medial tooth position. Make the di- papillae enlarged to 2-3 cm in size. Histologi-
agnosis: cal analysis detected papillar hyperplasia and
A. Acute suppurative pericoronitis of the 48 marked keratinization without alteration of
tooth the surrounding tissues. What is the most li-
B. Acute suppurative periostitis from the 48 kely diagnosis?
tooth A. Black hairy tongue
C. Chronic local mandibular osteomyelitis B. Median rhomboid glossitis
D. Mandibular angle fracture C. Fissured tongue
E. Acute submandibular sialadenitis D. Glossitis areata exfoliativa
197. Parents of a 6-year-old child complain E. Geographic tongue
of their child having a gradually enlarging
INSTRUCTIONAL BOOK
Ministry of public health of Ukraine (MPH of Ukraine)
Department of human recources policy, education and science
Testing Board

TEST ITEMS FOR LICENSING EXAMINATION: KROK 2. STOMATOLOGY.

Kyiv. Testing Board.


(English language).

Approved to print 10.04./№32. Paper size 60х84 1/8


Offset paper. Typeface. Times New Roman Cyr. Offset print.
Conditional print pages 24. Accounting publishing pages 28.
Issue. 306 copies.
List of abbreviations

A/G Albumin/globulin ratio HR Heart rate


A-ANON Alcoholics anonymous IDDM Insulin dependent diabetes mellitus
ACT Abdominal computed tomography IFA Immunofluorescence assay
ADP Adenosine diphosphate IHD Ischemic heart disease
ALT Alanin aminotranspherase IU International unit
AMP Adenosine monophosphate LDH Lactate dehydrogenase
AP Action potential MSEC Medical and sanitary expert committee
ARF Acute renal failure NAD Nicotine amide adenine dinucleotide
AST Aspartat aminotranspherase NADPH Nicotine amide adenine dinucleotide
phosphate restored
ATP Adenosine triphosphate NIDDM Non-Insulin dependent diabetes
mellitus
BP Blood pressure PAC Polyunsaturated aromatic
carbohydrates
bpm Beats per minute PAS Periodic acid & shiff reaction
C.I. Color Index pCO2 CO2 partial pressure
CBC Complete blood count pO2 CO2 partial pressure
CHF Chronic heart failure pm Per minute
CT Computer tomography Ps Pulse rate
DIC Disseminated intravascular coagualtion r Roentgen
DCC Doctoral controlling committee RBC Red blood count
DM-2 Non-Insulin dependent diabetes mellitus RDHA Reverse direct hemagglutination assay
DTP Anti diphtheria-tetanus vaccine Rh Rhesus
ECG Electrocardiogram (R)CFT Reiter's complement fixation test
ESR Erythrocyte sedimentation rate RIHA Reverse indirect hemagglutination
assay
FC Function class RNA Ribonucleic acid
FAD Flavin adenine dinucleotide RR Respiratory rate
FADH2 Flavin adenine dinucleotide restored S1 Heart sound 1
FEGDS Fibro-esphago-gastro-duodenoscopy S2 Heart sound 2
FMNH2 Flavin mononucleotide restored TU Tuberculin unit
GIT Gastrointestinal tract U Unit
Gy Gray USI Ultrasound investigation
GMP Guanosine monophosphate V/f Vision field
Hb Hemoglobin WBC White blood count
HbA1c Glycosylated hemoglobin X-ray Roentgenogram
Hct Hematocrit
HIV Human immunodeficiency virus
MINISTRY OF PUBLIC HEALTH OF UKRAINE

Department of human resources policy, education and science

Testing Board

Student ID Last name

Variant ________________

Test items for licensing examination

Krok 2
STOMATOLOGY
General Instruction
Every one of these numbered questions or unfinished statements in
this chapter corresponds to answers or statements endings. Choose the
answer (finished statements) that fits best and fill in the circle with the
corresponding Latin letter on the answer sheet.

ББК 54.1я73
УДК 61

Authors of items: Aksonova Ye.A., Amosova L.I., Babenko L.M., Babushkina N.S.,
Bahlyk T.V., Bas O.A., Beliaiev E.V., Bielikov O.B., Bik Ya.H., Bosa L.F., Chaikivsky R.V.,
Chernov D.V., Chumachenko V.A., Chyrkin V.I., Chyzhevsky I.V., Derkach L.Z., Dmitriyev M.O.,
Dorubets A.D., Dubrovina O.V., Dvornyk V.M., Dzetsiukh T.I., Eismund A.P., Fedorovych O.A.,
Flis P.S., Haiduk R.V., Hanchev K.S., Haranina T.S., Hembarovsky M.V., Herasym L.M.,
Hirchak H.V., Hladka O.M., Hodovanets O.I., Holik V.P., Holovko N.V., Holubieva I.M.,
Hordiychuk M.O., Hrad I.V., Hrechko N.B., Hrekuliak V.V., Huliuk A.H., Humetsky R.I.,
Hurzhiy O.V., Hrynkov Ye.I., Idashkina N.H., Ilenko N.M., Ilnytsky Ya.M., Ivchenko N.A.,
Karasiunok Ye.O., Karelina L.S., Kaskova L.F., Katurova H.F., Khalmatov B.D., Kharchenko O.I.,
Kharkov L.V., Kirsanova O.V., Klomin V.A., Konovalov M.F., Kopelian N.M., KosarievaL.I.,
Kotelevska N.V., Koval O.V., Kril A.Y., Kryzhanivska O.O., Kuz H.M., Kuchyrka L.I.,
Kyrychenko V.M., Larionov I.M., Lavrovska O.M., Levko V.P., Lokes K.P., Lunhu V.I.,
Lysiuk S.V., Lytovchenko Yu.O., Makarevych A.Yu., Malakhovska A.O., Mikhalova A.O.,
Moiseitseva L.O., Morozova M.M., Morozova N.P., Nemish T.Yu., Nesin O.F., Odzhubeiska O.D.,
Oktysiuk Yu.V., Onyshchenko S.I., Orlovsky V.O., Ozhohan Z.R., Parasochkina V.V.,
Pasechnyk A.M., Pavelko N.M., Petrushanko V.M., Polyshchuk L.F., Potiyko V.I.,
Prodanchuk A.I., Pryshko Z.R., Raida A.I., Romanenko I.H., Romankov I.O., Romashkina O.A.,
Rozumenko O.P., Ruzin H.P., Saiapina L.M., Samsonov O.V., Shakhnovsky I.V., Shcherbyna I.M.,
Shmat S.M., Shubladze H.K., Shuvalov S.M., Semenova O.O., Sidlak O.Ya., Skakun L.M.,
Skvortsova I.H., Smahliuk L.V., Stasiuk N.O., Svirchkov V.N., Sydorenko A.Yu., Sydorenko I.V.,
Sydorova A.I., Teslenko O.I., Tsilenko O.L.,Tyuhashkina Ye.H., Ushych A.H., Vasylenko V.M.,
Voliak M.N., Volkova O.S., Yermakova I.D., Yeroshenko A.V., Yevtushenko L.H., Zinchenko T.P.
and Committees of professional expertise.

Item reviewers. Bezvushko E.V., Bulbyk O.I., Chyzhevsky I.V., Dmytriieva A.A.,
Fastovets O.O., Gerelyuk V.I., Ilenko N.M., Kaskova L.F., Lungu V.I., Muntian L.M.,
Novikov V.M., Ostapko O.I., Smagliuk L.V., Solovey S.I., Tril S.I., Tsentylo V.G., Volynets V.M.,
Volyak M.N.

The book includes test items for use at licensing integrated examination “Krok 2. Stomatology” and
further use in teaching.
The book has been developed for students of stomatological faculties and academic staff of higher
medical educational establishments.

Approved by Ministry of Public Health of Ukraine as examination and teaching


publication based on expert conclusions (Orders of MPH of Ukraine of
14.08.1998 №251, of 27.12.1999 №303, of 16.10.2002 №374, of 29.05.2003 №233).

© Copyright Testing Board.


Krok 2 Stomatology (англомовний варiант, iноземнi студенти) 2018 рiк 1

1. A removable full denture for the lower A. Deep occlusion


jaw is being made for a 75-year-old man. B. Front teeth defects that cannot be corrected
Objectively the alveolar process is slightly with fillings
atrophied. Herbst tests are performed during C. Enamel hypoplasia with tooth deformation
fitting of an impression tray. When lips are and discoloration
stretched forwards the tray slips off. Where D. Tooth discoloration
should the tray edge be shortened in this E. Devitalized teeth defects that cannot be
case? corrected with dental inlays
A. From canine to canine on the vestibular 5. A 32-year-old woman needs a denture. On
side objective examination the decision was made
B. From canine to canine on the lingual side in favor of porcelain-fused-to-metal crown.
C. From behind the mandibular tuberosity to What material should be used in this case to
the mylohyoid line obtain the impression?
D. Along the mylohyoid line
E. In the premolar area on the lingual side A. Stomaflex
B. Repin
2. A 18-year-old boy complains of bleeding C. Stomalgin
and pain in her gums. The disease onset was D. Stens
4 days ago. Objectively the skin is pale, body E. Orthocor
temperature is 38.5o C . Her submandibular
lymph nodes on the left are enlarged, painful, 6. Parents of a 6-month-old child complain of
non-fused with the surrounding tissues. The their child having a large amount of dental
gingival papillae and gingival margin in the deposit in the oral cavity. Objectively the oral
area of 33, 34, 35, 36, and 37 are hyperemic, mucosa is hyperemic, covered in white easily
ulcerated, and covered in necrotic deposit. removed deposit that resembles curdled mi-
Teeth present with soft dental plaque. Make lk. What is the causative agent of this disease?
the diagnosis: A. Candida fungi
A. Necrotizing ulcerative gingivitis B. Herpes simplex virus
B. Acute catarrhal gingivitis C. Klebs-Loeffler bacillus
C. Chronic hypertrophic gingivitis D. Coxsackie virus
D. Chronic catarrhal gingivitis E. Epstein-Barr virus
E. - 7. A 55-year-old man complains of heartburn
3. A 60-year-old man complains of stabbing and unpleasant sensations in his tongue. The
pain near the root of the tongue on the ri- patient has been suffering from gastritis with
ght, which develops during eating, especially low acidity for approximately 5 years. What
sour food. Objectively: there is a swelling in changes in the tongue are the most likely to
the right submandibular area. On palpati- be detected in this patient?
on the submandibular gland is dense and A. Atrophied and smoothed out lingual papi-
enlarged. Excretory duct orifice of the right llae
submandibular gland is dilated and produces B. Hypertrophic lingual papillae
mucopurulent secretion. What is the most li- C. Erosions on the lateral surfaces of the
kely diagnosis? tongue
A. Sialolithiasis of the submandibular gland D. Coated tongue
B. Calculous sialadenitis of the sublingual E. Fissured tongue
gland 8. A 10-year-old child is referred by the
C. Acute suppurative lymphadenitis orthodontist for extraction of tooth 53.
D. Acute sialodochitis Objectively the crown of 53 is retained, the
E. Adenophlegmon of the right submandi- tooth is immobile. X-ray of tooth 53 shows
bular area root resorption by less than 1/3. Choose the
4. A 19-year-old woman, an actress, complai- best instrument for extraction of tooth 53:
ns of discoloration of her left maxillary A. Straight crown forceps
central incisor. One year ago the pulp of B. Straight elevator
this tooth was removed and the tooth was C. Root bayonet forceps
filled. Gradually the tooth assumed grayish D. Crown forceps with S-shaped handles
color. Objectively the 11 is filled, discolored, E. Beak-shaped root forceps
firm, painless on percussion. Deep occlusi-
on is observed. What part of the clinical 9. A 24-year-old woman has Richmond crown
presentation contraindicates installation of being made to restore the crown of the central
an all-porcelain crown? maxillar incisor. The cap is completed. What
is the next step of prosthesis-making?
Krok 2 Stomatology (англомовний варiант, iноземнi студенти) 2018 рiк 2

A. To fit the cap on the tooth stump and place 14. During preventive examination a 40-year-
the post in the root canal old man presents with the following changes:
B. To solder the post with the cap marginal gingiva is enlarged, torus-shaped,
C. To fit the cap and the post to the tooth root cyanotic, slightly bleeding when touched wi-
D. Making of combination dental crown th a dental probe; there is no pain. Staining
E. Tooth fixation with cement the gums with Lugol’s iodine solution results
in light-brown coloring of mucosa. Make the
10. Removable full dentures are being made diagnosis:
for a 65-year-old man. Progenic occlusion is
determined. What are the specifics of teeth A. Chronic catarrhal gingivitis
placement in case of progenic occlusion of B. Acute catarrhal gingivitis
edentulous jaws? C. Exacerbation of chronic catarrhal gingivitis
D. Chronic hypertrophic gingivitis
A. Superior dental arch is shortened by two E. Generalized periodontitis
premolars
B. Anterior teeth are placed in direct occlusi- 15. A 4-year-old boy has been diagnosed wi-
on th acute purulent periostitis of the upper jaw
C. Inferior dental arch is shortened by two originating from the 64 tooth. Choose the
premolars optimal treatment tactics:
D. Anterior teeth are placed in orthognathic
occlusion A. The 64 tooth extraction, periosteotomy,
E. Short-bite anterior teeth pharmacotherapy
B. The 64 tooth extraction, anti-inflammatory
11. A 53-year-old patient complains of pain pharmacotherapy
and clicking in the left temporomandibular C. Endodontological treatment of the 64
joint. Objectively: the face is symmetrical, tooth, anti-inflammatory pharmacotherapy
palpation of the lateral pterygoid muscles is D. Endodontological treatment of the 64
painful on the left side. Mouth opening is tooth, periosteotomy
reduced. Tomography shows smooth bone E. Periosteotomy, anti-inflammatory
outline of joint surfaces. Which disease of pharmacotherapy
those listed below corresponds with this clini-
cal presentation? 16. An adolescent complains of reduced and
painful mouth opening, difficulties when eati-
A. Temporomandibular joint dysfunction ng, and swelling in the left mandibular angle
B. Rheumatic arthritis that developed after tooth 37 was extracted 3
C. Deforming arthrosis days ago. Objectively the face is asymmetric
D. Acute posttraumatic arthritis due to soft tissue swelling in the area of the
E. Joint ankylosis left mandibular angle. Mouth opening is pai-
nful and reduced to 2.0 cm. Disturbed occlusi-
12. A 47-year-old man complains of partial on is observed. Palpation of the left mandi-
loss of his upper teeth. The patient’s medi- bular angle is painful, the tissues are soft,
cal history states loss of teeth due to trauma bone crepitus is detected. ”Indirect load to
sustained 3 months ago. 11 and 12 are lost. 13, the chin” symptom is positive in the area
21, and 22 are destroyed by 2/3 and restored of the left mandibular angle. The socket of
with fillings. Occlusion is orthognathic. What the extracted tooth is packed with iodoform
denture construction would be optimal for gauze. What is the most likely diagnosis?
this patient, considering his occupation as a
lecturer? A. Left mandibular angle fracture
B. Mandibular alveolar fracture
A. Porcelain-fused-to-metal dental bridge C. Anterior mandibular fracture
B. Plastic dental bridge D. Mandibular periostitis on the left
C. Clasp-retained (bugel) removable partial E. Odontogenic mandibular osteomyelitis
denture with attachments
D. Removable partial laminar denture for the 17. A 32-year-old man has metallic inlay made
upper jaw for him. The denture is being made for tooth
E. Swaged-soldered metal dental bridge with 36 with Black’s class I carious cavity. What
faceted intermediate part surfaces of the inlay should be filed down and
polished before fixing the denture?
13. A 7-year-old boy is diagnosed with epi-
demic parotitis (mumps). Name the most li- A. Occlusal surface
kely complication of this disease: B. Lateral surfaces
C. All surfaces
A. Orchitis D. Inlay bottom
B. Colitis E. Lateral surfaces and inlay bottom
C. Dermatitis
D. Pneumonia 18. A 12-year-old boy complains of pai-
E. Cholecystitis nful and bleeding gums on his upper jaw.
Objectively the gingival margin in the area
Krok 2 Stomatology (англомовний варiант, iноземнi студенти) 2018 рiк 3

of the 13, 12, 11, 21, 22, and 23 teeth is A. Adenocarcinoma of the right parotid gland
swollen, hyperemic, deformed due to gingi- B. Chronic non-epidemic parotitis
val overgrowths. Gingival papillae cover the C. Chronic lymphadenitis of the right parotid
crowns by 1/3 of their height, bleed on touch. area
Upper front teeth are overcrowded. X-ray D. Mixed tumor of the right parotid gland
shows no pathological changes of the peri- E. Actinomycosis of the right parotid gland
odontium. What drugs should be admini-
stered for topical treatment in the first place? 22. Parents of a 3-year-old child report that
the child suffers from constant pain in the
A. Nonsteroidal anti-inflammatory drugs upper front teeth. Objectively: the coronal
B. Sclerosants part of the 61 tooth is gray and decayed.
C. Steroidal anti-inflammatory drugs Probing of the root canal orifice is painful and
D. Keratoplastic agents accompanied by bleeding. The tooth percussi-
E. Cytostatic agents on provokes acute pain. Mucosa is hyperemic,
edematic and painful. Palpation in the region
19. A 43-year-old woman complains of of the 61 and 62 teeth reveals a fistula. What
mobility and displacement of her upper is the provisional diagnosis?
front teeth. Objectively: dental formula is
17 16 15 14 13 12 11 21 22 23 24 25 26 27 A. Exacerbation of chronic periodontitis
47 46 45 44 43 42 41 31 32 33 34 35 36 37 . B. Acute suppurative periodontitis
Teeth 12 11 21 22 are slanted towards C. Acute diffuse pulpitis
the vestibular side, diastema and tremata D. Chronic granulating periodontitis
are observed, I-II degree teeth mobility is E. Exacerbation of chronic pulpitis
detected. Select the orthodontic appliance 23. Puncture sample taken from a 13-year-
for correction of teeth misalignment as a part old child contains giant Reed-Sternberg cells.
of complex treatment of periodontal disease: What diagnosis can be confirmed by the cell
A. Palatal plate with vestibular arch content of this puncture material?
B. Bynin appliance A. Lymphogranulomatosis
C. Schwartz appliance B. Tuberculous lymphadenitis
D. Katz crown C. Lymphocytic leukemia
E. Palatal plate with inclined plane D. Lymph node actinomycosis
20. A 45-year-old man complains of E. Infectious mononucleosis
toothache and mobility of his upper front 24. A 25-year-old man complains of genelal
teeth. Objectively his dental formula is as malaise, high body temperature, acute gi-
follows: ngival bleeding, and gingival enlargement.
17 16 15 14 13 12 11 21 22 23 24 25 26 27
47 46 45 44 43 42 41 31 32 33 34 35 36 37 . He has a history of nosebleeds. Objectively
the patient presents with systemic lymphoid
Dental cervices of 13 12 11 21 22 are hyperplasia, pallor of skin and mucosa, II-III
exposed and demonstrate mobility of the degree hyperplasia of the gingival mucosa,
III degree. Mobile teeth are to be extracted hemorrhages into the buccal mucosa, and
and immediate denture is to be made for the ulcers covered with gray deposit. What
patient. How soon after the teeth extraction examination method would be optimal for
should such dentures be inserted? diagnosis-making in this case?
A. On the day of teeth extraction A. Complete blood test panel
B. In 1-2 days B. Yasynsky test
C. In 3-4 days C. Bacterioscopy
D. In 5-6 days D. Immunoassay
E. In 6-7 days E. Blood glucose test
21. A 56-year-old man complains of swelli- 25. A 19-year-old young man complains of
ng and pain in his right parotid area. The constant pain in tooth 22, which intensifies on
swelling was noticed 5-6 months ago. Objecti- biting with this tooth, sensation of ”protrudi-
vely right-sided paresis of the facial muscles ng” tooth, and upper lip edema. The pati-
can be determined. Palpation reveals there ent has history of upper jaw trauma. Objecti-
a modrately painful tuberous tumor fused vely tooth 22 is intact. Vertical percussion
with surrounding tissues. In the center of is acutely painful. The upper lip is swollen,
the tumor there is an area of softening. mucogingival fold in the area of tooth 22 is
Submandibular and cervical lymph nodes on red and painful on palpation. What exami-
the right are enlarged and dense. The mouth nation method is necessary for diagnosis-
can be opened without restriction. There is making in this case?
no saliva outflow from the opening of the ri-
ght parotid gland. What provisional diagnosis
can be made?
Krok 2 Stomatology (англомовний варiант, iноземнi студенти) 2018 рiк 4

A. X-ray A. Erythema multiforme exudativum


B. Dental pulp test B. Pemphigus vulgaris
C. Rheodentography C. Acute herpetic stomatitis
D. Transillumination D. Nonacantholytic pemphigus
E. Thermometry E. Dermatitis herpetiformis (Duhring’s di-
sease)
26. A patient complains of carious cavity in
tooth 11. The filling was lost one week ago. 30. A 28-year-old woman complains of persi-
The tooth crown is dark, there is residual fi- sting pain in tooth 34, which intensifies on
lling material at the bottom of the carious biting. Four days ago arsenic paste was left
cavity. Vertical percussion is painless. X-ray in the 34. The patient missed her appoi-
shows an oval area of bone tissue resorpti- ntment with the dentist. Objective examinati-
on with clear margins, 0.4х0.3 cm in size. The on detected occlusive dressing on the distal
root canal is filled by 2/3 of its length. What is masticatory surface of tooth 34, percussion is
the most likely diagnosis? acutely painful. What treatment tactics would
be the most advisable in this case?
A. Chronic granulomatous periodontitis
B. Chronic fibrous periodontitis A. Arsenic antidote is placed into the root
C. Chronic granulating periodontitis canal under occlusive dressing
D. Radicular cyst B. Dentin dressing is removed, electrophoresis
E. Exacerbation of chronic periodontitis with antidote along the mucogingival fold is
prescribed
27. A 28-year-old man is referred for oral C. The root canal is lavaged with antidote, the
cavity sanation. On examination there is a tooth remains uncovered
filling on the masticatory surface of tooth D. The root canal is lavaged with antidote and
17, percussion is painless. Mucosa in the root filled
apex projection of 17 is cyanotic, vasoparesis E. -
symptom is positive. X-ray shows foci of bone
tissue destruction with fuzzy margins in the 31. A 35-year-old man came to the
area of root apices, root canals are not filled. prosthodontic clinic with complaints of teeth
What is the most likely diagnosis? mobility on his lower jaw. What type of
occlusion stabilization is recommended in
A. Chronic granulating periodontitis this case?
B. Chronic fibrous periodontitis
C. Chronic granulomatous periodontitis A. Arch
D. Radicular cyst B. Sagittal
E. Chronic fibrous pulpitis C. Frontal
D. Parasagittal
28. A 30-year-old woman complains of mi- E. Frontosagittal
ld burning sensation in her lower lip and
its dryness. She peels skin scales off with 32. A 57-year-old patient complains of tooth
her teeth. She has been presenting with this mobility and inability to eat. Objectively: the
condition for 10 years. On examination the lower 35, 36, 37, 38, 44, 45, 46 and 48 teeth are
skin scales are gray and located on the lip missing; the 31, 32, 33, 34, 41, 42, 43, 47 teeth
from the Klein’s line to the center of the exhibit grade II mobility, their clinical crowns
vermillion border from angle to angle of the are low, tooth equator is not pronounced.
mouth. The scales are firmly attached in the What is the optimal denture construction in
center and are loose on the periphery. Their this case?
forcible removal does not result in erosions.
What is the most likely diagnosis? A. Removable cast splint
B. Removable partial denture
A. Exfoliative cheilitis C. Kurlyandsky splint bar
B. Lupus erythematosus D. Bynin removable splint
C. Meteorological cheilitis E. Removable splint with vestibulo-oral clasp
D. Allergic contact cheilitis
E. Eczematous cheilitis 33. A 16-year-old young man complai-
ns of temperature increase up to 38.7o C ,
29. A 32-year-old patient presents with pain when eating and swallowing, foul
body temperature of 38.9o C , general fatigue, acrid smell from his mouth. Lymph nodes,
impaired speech, inability to eat. This condi- especially cervical ones, are enlarged, mobi-
tion has been recurring for the last 4 years in le, and painless. Objectively the patient
autumn and spring. There are vesicles and presents with generalized hyperemia of the
erosions with grayish fibrinous coating on oral mucosa, multiple petechiae, necrotic
the hyperemic and swollen labial and buccal spots, and profuse coating of the anteri-
mucosa. Nikolsky’s sign is negative. What is or pharynx. Blood test: increased ESR,
the most likely diagnosis? marked leukocytosis, monocytosis, atypical
mononuclear cells, thrombocytopenia. What
is the most likely diagnosis?
Krok 2 Stomatology (англомовний варiант, iноземнi студенти) 2018 рiк 5

painful along the whole floor of the carious


A. Infectious mononucleosis cavity. Electric pulp test - 50 microamperes.
B. Vincent stomatitis (acute necrotizing Thermodiagnosis is painful. Make the di-
ulcerative gingivitis) agnosis:
C. Herpetic angina
D. Monoblastic leukemia A. Chronic fibrous pulpitis
E. Acute herpetic stomatitis B. Chronic deep caries
C. Chronic fibrous periodontitis
34. A 57-year-old retired man complains of D. Acute deep caries
attacks of burning pain and rashes on the E. Chronic gangrenous pulpitis
skin of his face and oral mucosa on the right.
Anamnesis: a course of radiation therapy for 38. A 35-year-old man complains of thickeni-
treatment of gastric disease, past case of chi- ng of his maxillary alveolar process. Prelimi-
ckenpox. Objectively: along the third branch nary diagnosis of maxillary radicular cyst was
of the trigeminal nerve the skin of the face made. What substance will be obtained as the
presents with isolated erosions covered in fi- result of the puncture of the alveolar process
brinous coating. There are multiple vesicles in the area of buccal thickening?
on the hyperemic and swollen oral mucosa.
Right-sided lymphadenitis is observed. What A. Yellowish liquid
is the most likely diagnosis? B. Blood
C. Pus
A. Herpes zoster D. Epithelium
B. Neuralgia E. Turbid infiltration
C. Murrain
D. Acute recurrent herpes 39. Six months ago a 40-year-old man had his
E. Neuritis tooth 26 extracted; afterwards his oral cavity
and maxillary sinus became communicating
35. A 12-year-old girl has complaint of a cari- and the patient developed the first signs of
ous cavity in her tooth. Objectively: there is maxillary sinusitis. What surgical procedure
Black’s class 1 carious cavity in the 36 tooth; should be performed in this case?
it is localized in the parapulpar dentin; the
mouth of the cavity is wide. The dentin is A. Maxillary sinusotomy with simultaneous
dense and pigmented. It is sensitive to cold plastic surgery for repair of the fistula
stimulus, percussion is painless. What is the B. Caldwell-Luc surgery
most likely diagnosis? C. Fistula plication
D. Fistula packing with Iodoform gauze
A. Chronic deep caries E. Osteotomy of the alveolar process
B. Chronic median caries
C. Acute deep caries 40. A 46-year-old woman complains of
D. Acute median caries bleeding gums, suppuration, teeth mobili-
E. - ty. She has been presenting with these signs
for 10 years. On examination her upper and
36. A 35-year-old woman has complaints of lower gums are hyperemic, swollen, bleed on
cosmetic defects of the front upper teeth touch. In the area of 42, 41, 31, and 32 peri-
crowns. The defects have been aggravati- odontal pockets are up to 8 mm deep, contain
ng for the last 10 years. The patient suffers purulent discharge; these teeth demonstrate
from unpleasant sensations when brushi- mobility of the II degree, other teeth present
ng her teeth, and when chemical stimuli with mobility of the I degree. In the area of
are applied. Objective examination revealed 42, 41, 31, and 32 X-ray shows interalveolar
defects localized in the enamel of the front septa resorption by 1/2 of the root length and
upper teeth vestibular surface. The defects signs of osteoporosis. What is the most advi-
are oval, saucer-shaped, and have clear margi- sable method of surgical treatment in this
ns. Response to probing and cold stimuli was case?
positive. Make the diagnosis:
A. Osteoplasty
A. Enamel erosion B. Curettage
B. Enamel hypoplasia C. Gingivotomy
C. Cuneiform defect D. Gingivectomy
D. Chemical necrosis of the tooth E. Flap surgery
E. Hyperesthesia of tooth hard tissues
41. A 40-year-old man had his root canal of
37. A patient complains of dull ache in the the 34 tooth filled due to chronic fibrous peri-
16 tooth, which occurs during eating cold odontitis. Soon the treated place became pai-
food. Previously the tooth had been filled nful. On X-ray the root canal of the 34 tooth
due to deep caries, the filling was lost 1 year is filled to the root apex. What tactics should
ago. Objectively: a deep carious cavity that the dentist choose to manage the pain?
does not communicate with the tooth cavity
is present; percussion is painless, probing is
Krok 2 Stomatology (англомовний варiант, iноземнi студенти) 2018 рiк 6

A. To prescribe physiotherapeutic procedures her. What material would be optimal for this
B. To rinse with antiseptic mouthwash splint?
C. To make insicion along the mucogingival
fold A. Cobalt nickel chromium alloy
D. To provide conduction anesthesia B. ”EI-95” alloy
E. To provide infiltration anesthesia C. Stainless steel
D. ”PD-250” alloy (silver palladium alloy)
42. A 49-year-old man was diagnosed wi- E. Gold alloy of 900 millesimal fineness
th recurrence of lower lip cancer two years
after he had undergone radiation therapy. 47. A 4-year-old practically healthy child
Objectively in the area of his lower right came for oral cavity sanation. Objectively on
lip there is a neoplasm 1x2 cm in size with the masticatory surface of 75 there is a cari-
an ulcer in its center. In the right submandi- ous cavity within mantle dentin. The cavity
bular area there are 2 round, enlarged, dense, is filled with softened dentin. Dentinoenamel
painless lymph nodes. What approach to the junction is painful on probing. What material
treatment would be optimal in this case? would be optimal for permanent filling?
A. Combined treatment A. Glass ionomer cement
B. Wedge resection of the lower lip B. Phosphate cement
C. Rectangular resection of the lower lip C. Silicophosphate cement
D. Trapezial resection of the lower lip D. Composite material
E. Vanakh’s operation E. Silicate cement
43. During regular check-up a 6.5-year-old 48. A 7.5-year-old practically healthy child
child presents with carious cavity on the di- complains of crown fracture and pain in the
stal proximal surface of 65 within mantle upper right incisor. Objectively 2/3 of crown
dentin. Cavity walls and bottom are pi- of 11 is absent, the pulp is exposed and red;
gmented, dense, painless on probing; there on probing it is acutely painful and bleedi-
is no response to cold stimulus; percussion ng; tooth percussion is painful. The trauma
is painless. During tooth preparation there occurred 2 hours ago. What would be the
is tenderness at the level of dentinoenamel optimal treatment method in this case?
junction. What is the most likely diagnosis?
A. Vital amputation
A. Chronic median caries B. Devital amputation
B. Acute median caries C. Vital extirpation
C. Chronic deep caries D. Devital extirpation
D. Chronic fibrous pulpitis E. Biological approach
E. Chronic granulating periodontitis
49. Parents of a 2.5-year-old child complain of
44. After adenotonsillectomia it is necessary gradual destruction of the upper front teeth
to break the mouth breathing habit in a 4- of their child for the last several months.
year-old child. The orthodontist recommends Objectively there are carious cavities within
application of an oral vestibular shield mantle dentin on the contact and vestibular
(Kerbitz’ vestibular plate). Vestibular shield surfaces of 52, 51, 61, and 62. The cavities are
facilitates training of the following muscle: filled with softened pigmented dentin that
can be easily removed with dental excavator.
A. Orbicular muscle Make the provisional diagnosis:
B. Temporal muscle
C. Masseter muscle A. Acute median caries
D. Lateral pterygoid muscle B. Chronic deep caries
E. Medial pterygoid muscle C. Acute deep caries
D. Chronic median caries
45. A 30-year-old man presents with fresh E. Chronic superficial caries
median mandibular fracture without visible
displacement of the fragments. What will be 50. A 49-year-old man complains of progressi-
the function of the dental apparatus to be ng reduction of mouth opening, pain on
prescribed in this case? the left when swallowing, severe deteriorati-
on of his general well-being, temperature
A. Fixation increase up to 39.3o C . Destroyed tooth 38
B. Setting presents with acute pain. Objectively the face
C. Directing is symmetrical, the submandibular lymph
D. Replacement nodes on the left are enlarged and painful
E. Formation on palpation. Palpation under the left mandi-
bular angle and in the the left retromandi-
46. A 43-year-old woman complains of her bular area provokes sharp pain. Mouth
lower teeth mobility. Objectively the teeth opening and movement of the mandible to
mobility is of the I-II degree. It is planned to the left are significantly reduced. The left
make a full-cast removable occlusal splint for pterygomandibular fold is hyperemic and
Krok 2 Stomatology (англомовний варiант, iноземнi студенти) 2018 рiк 7

infiltrated. What is the most likely diagnosis? transport immobilization of the fracture?
A. Phlegmon of the pterygomandibular space A. Patient’s dentures
B. Phlegmon of the parapharyngeal space B. Weber splint
C. Phlegmon of the retromandibular area C. Vankevych splint
D. Phlegmon of the submandibular space D. Zbarzh apparatus
E. Abscess of the sublingual fossa E. Vasiliev splint
51. A 40-year-old man presents with 55. A 57-year-old woman came to a dentist for
pathologic teeth grinding caused by their extraction of the 34 tooth due to exacerbati-
functional overload due to the loss of many on of chronic periodontitis. What instrument
antagonist teeth. With direct occlusion, verti- would be optimal for tooth extraction in the
cal grinding of the front teeth resulted given case?
in protrusion of the patient’s lower jaw
forwards. Interalveolar space is diminished, A. Beak-shaped non-crushing forceps
the lower third of the face is shortened. What B. Beak-shaped crushing forceps
would be the most advisable treatment in this C. Beak-shaped curved forceps
case? D. Straight elevator
E. Curved elevators
A. Prosthetics that increase interalveolar
height 56. A cast clasp-retained (bugel) removable
B. Prothetic treatment partial denture is being made for a 58-year-
C. Teeth shortening old patient. Impressions are made, centric jaw
D. Prosthetic treatment relation is determined, plaster casts are obtai-
E. Instrumental surgical treatment ned. What is the next stage?
52. A 7-year-old child complains of pain and A. Examination of the working model with a
swelling in the left submandibular region. parallelometer
The swelling in this region developed 2 days B. Transfer of denture frame pattern to the
ago. Objectively: the child is in a satisfactory working model
condition, body temperature is of 37.3o C . C. Wax modelling of the denture frame
Face is asymmetrical due to the soft tissue D. Duplication of the working model
swelling in the left submandibular region. E. Marking the border seal
Palpation reveals a round formation 2x2 cm
in size. The formation is mobile, painful, non- 57. A 7-year-old boy underwent fissure seali-
fused with the skin. The 74 tooth is discolored, ng in teeth 36 and 46. Fissure sealing would
percussion is painful. What is the provisional be most effective:
diagnosis? A. Immediately after eruption of the
A. Acute serous odontogenic lymphadenitis permanent tooth
of the left submandibular region B. After permanent occlusion is formed
B. Acute serous nonodontogenic lymphadeni- C. If permanent teeth are affected with caries
tis of the left submandibular region D. In 1-2 years after tooth eruption
C. Acute suppurative odontogenic E. In 3-4 years after tooth eruption
lymphadenitis of the left submandibular 58. A 6-year-old girl took paracetamol to
region treat a case of URTI two days ago, whi-
D. Phlegmonous adenitis of the right ch resulted in the development of her
submandibular region present condition. The disease onset was
E. Lateral cervical cyst acute with temperature increase up to
53. A 48-year-old patient complains of the 39.8o C . Objectively there are cockade-shaped
lower jaw teeth mobility. Van Thiel dental maculopapular rashes on her face. The vermi-
splint is to be made for prosthodontic lion border is swollen, hyperemic, covered
treatment. What construction elements are in massive brown crusts, and presents with
supposed to fix it in place? bleeding cracks. Conjunctivitis is detected.
Swollen and hyperemic oral mucosa presents
A. Whole piece proximal grip clasps with numerous erosions covered with fibri-
B. Full metal crowns nous incrustations; the erosions are sharply
C. Wire clasps painful on palpation. What is the most likely
D. Parapulpar posts diagnosis?
E. Equator crowns
A. Stevens-Johnson syndrome
54. A 55-year-old man suffered a blow to the B. Erythema multiforme exudativum
frontal mandibular area. He is diagnosed with C. Acute herpetic stomatitis
mandibular fracture. Prior to trauma he was D. Chronic recurrent aphthous stomatitis
wearing removable dentures (partial laminar E. Pemphigus
denture for the lower law and full denture
for the upper jaw). What can be used for 59. How often should the dentures be
Krok 2 Stomatology (англомовний варiант, iноземнi студенти) 2018 рiк 8

replaced in children during the period of milk covered in scales of varying size. In the angles
occlusion according to Ilyina-Markosian? of the mouth there are fissures covered in
white coating, the skin is macerated. What
A. Every 6-8 months ointment should be prescribed for topical
B. Every 8-10 months treatment in the given case?
C. Every 10-12 months
D. Every 12-16 months A. Clotrimazol
E. Every 16 months B. Interferon
C. Prednisolone
60. On objective examination a 59-year-old D. Lanolin
man with the edentulous mandible presents E. Erythromycin
with bone protrusions and mobile areas of
the alveolar crest. To ensure proper fixati- 64. A 35-year-old woman complains of lips
on of the denture and even load distribution enlargement. The first incident occurred one
the following functional impression should be year ago, when she developed lip edema that
made: abated quickly, but the lips remained slightly
enlarged. Three days ago after overexposure
A. Differentiated to cold her lips enlarged again. Objectively:
B. Complete anatomical ptosis, upper and lower lips are markedly
C. Compression enlarged, more on the left, soft, elastic, and
D. Decompression painless on palpation; no impressions on the
E. Combined lip surface are left after pressing it with a fi-
nger. The tongue is swollen, with tuberous
61. A 28-year-old man complains of pain in surface and folds on its back. What is the
the infraorbital and parotid region on the most likely diagnosis?
left. On examination: hemorrhage occurs in
the lower eyelid and conjunctiva of the left A. Melkersson-Rosenthal syndrome
eye, there are signs of crepitation and step B. Miescher’s granulomatous cheilitis
deformity of the eyesocket lower edge. The C. Quincke’s edema
mouth opens by 1 cm. Make the diagnosis: D. Achard’s syndrome
E. Meige’s trophedema
A. Zygomatic bone fracture
B. Malar arch fracture 65. A 45-year-old man complains of dryness
C. Left articular process fracture and pain in the lower lip. On examination:
D. Traumatic arthritis of the temporo- mandi- the lower lip is swollen, dry, covered in small
bular joint scales and fissures. In the Klein area (wet-dry
E. Hematoma of the infraorbital region line) there are dilated openings of salivatory
62. A child is 8 years old. There are complai- glands observed as red dots producing clear
nts of congested upper incisors. Objectively: substance. The lower lip mucosa is lumpy.
the first molars closure is of Angle’s I class, What is the most likely diagnosis?
frontal overbite is orthognathic. The 12 and A. Glandular cheilitis
22 teeth erupt palatinally with space defici- B. Actinic cheilitis
ency of 2/3 of the tooth crown. The 11 and C. Meteorological cheilitis
21 teeth are 10 mm each in cross-section. The D. Eczematous cheilitis
child has inherited father’s facial type with E. Exfoliative cheilitis
prognathism and macrodontia of the central
incisors. Choose the preventive treatment, 66. A 23-year-old man complains of gum
considering this hereditary pathology: bleeding when he brushes his teeth or eats
solid food. Objectively: the gums of the front
A. Hotz serial extraction to reduce the dental lower jaw are hyperemic, swollen and bleedi-
arch ng when palpated. Oral and gingival mucosa
B. Jaw expansion to provide the space for the in other areas are not affected. The occlusi-
12 and 21 teeth on is deep. The teeth are firm, except for the
C. Massage of the 12 and 21 teeth area to 41 and 31 (degree 1 mobility). X-ray shows
stimulate their eruption resorption of the alveolar septum in the area
D. Extraction of the 12 and 21 teeth to reduce of the 41, 42, 32, and 31 teeth up to 1/3 of the
the dental arch root length. What is the most likely diagnosis?
E. Shave off the approximal surfaces of the 11
and 21 to provide the space for the 12 and 22 A. Localized periodontitis
teeth B. Generalized periodontitis, initial stage
C. Generalized periodontitis, stage I
63. A 38-year-old woman complains of burni- D. Catarrhal gingivitis
ng pain in her lips and angles of her mouth, E. Parodontosis, stage I
their dryness. Anamnesis states that she has
been suffering from diabetes mellitus for 67. A 40-year-old man, a chemical industry
the last 8 years. Objectively: the vermilli- worker, notes the sour sensation in his mouth,
on border is dry, congestively hyperemic, pain response to thermal and chemical sti-
Krok 2 Stomatology (англомовний варiант, iноземнi студенти) 2018 рiк 9

muli. On examination: on the vestibular A. Osteoclastoma


surface and cutting edge of the front teeth B. Adamantinoma
there are chalky enamel defects with uneven C. Osteoma
scalloped margins. Make the diagnosis: D. Follicular cyst
E. Odontoma
A. Acidic necrosis of enamel
B. Superficial caries 72. A 6.5-year-old child has closed non-
C. Enamel hypoplasia (erosive form) pigmented fissures in the first permanent
D. Fluorosis (erosive form) molar, which have been revealed duri-
E. Median caries ng preventive examination. Enamel
transparency is retained, its probing reveals
68. A 42-year-old woman complains of acute no coarseness. Choose the optimal method of
lip enlargement, itching, and bursting sensati- treatment in this case:
on. She ascribes her condition to introduction
of a new lipstick. On examination the lips are A. Non-invasive sealing
significantly enlarged and turgid, on palpati- B. Invasive sealing
on they are firm, elastic, and painless. Regi- C. Preventive filling
onal lymph nodes are without changes. What D. ART technique
is the most likely diagnosis? E. Regular medical check-ups

A. Allergic contact cheilitis 73. A man complains of gingival pain in


B. Meteorological cheilitis his upper left jaw and bleeding that occurs
C. Exfoliative cheilitis when he brushes his teeth or eats solid food.
D. Glandular cheilitis Objectively on the upper jaw he wears a
E. Actinic cheilitis swaged-soldered metal bridge with 14 and
16 as abutment teeth. The crown edge is
69. A 50-year-old patient, an employee of the pushed under the gingival margin by 0.3
print shop, complains of foul smell from his mm. Intermediate part is closely fitted to
mouth and excessive salivation. Objectively the gums. The mucosa is hyperemic, swollen,
against the background of hyperemic and interdental papillae are smoothed out;
slightly swollen gums there is a blue-black touching mucosa with a dental instrument
border along the gum margin of the lower provokes bleeding. What medical tactics
jaw and upper front teeth. There is a large should the dentist choose in the first place?
amount of dental deposit observed. Name
the type of stomatitis in this patient: A. Remove the dental bridge
B. Refer the patient for cosultation with the
A. Lead dental therapist
B. Mercury C. Refer the patient for X-ray
C. Bismuth D. Refer the patient for clinical blood test
D. Catarrhal E. Make metal-fused-to-porcelain dental
E. Necrotizing ulcerative bridge
70. A 25-year-old man complains of 74. When checking construction of the
incorrectly positioned maxillary left central soldered dental bridge with the 35 and 38
incisor due to trauma sustained 2 months ago. abutment teeth the following was detected:
Objectively tooth 21 is rotated around its axis pores in the place where abutment crowns
into palatal position. What would be the most and intermediate part are soldered together;
advisable treatment method for correction of masticatory cusps are sharply defined; there
this defect? is early contact with antagonist teeth; the
intermediate part makes tight contact with
A. Orthodontic treatment the alveolar process mucosa. How can those
B. Surgical treatment flaws be corrected?
C. Instrumental surgical treatment
D. - A. Dental bridge should be remade
E. Splinting followed by prosthetic treatment B. Intermediate part should be corrected
C. Masticatory surface should be corrected,
71. A 44-year-old woman complains of the and soldered places - polished
face swelling in the right lower jaw area and D. Masticatory surface should be corrected,
teeth mobility. Objectively: soft tissues are and the height of the flushing part is to be
without changes, the regional lymph nodes increased up to 2 mm
cannot be palpated. The alveolar process and E. Tooth-antagonists should be shaved off
the body of the lower jaw near the 46, 47,
and 48 teeth are thickened, painless when 75. A 50-year-old man was diagnosed with
palpated, and lumpy. The teeth in the thi- sialolithiasis with the salivary gland stone
ckened area are mobile. Puncture consists located deep within the salivary gland.
of brown fluid without cholesterol crystals. Choose the optimal treatment tactics:
What is the provisional diagnosis?
Krok 2 Stomatology (англомовний варiант, iноземнi студенти) 2018 рiк 10

A. Submandibular gland excision A. Plaster


B. Radiation therapy B. Dentafol
C. Sclerotherapy C. Stens
D. Removal of the sialolith while retaining the D. Stomaflex
gland E. Repin
E. Conservative pharmacotherapy
81. A 19-year-old young man complains
76. Due to lack of timely specialized of cosmetic defect of all his teeth, which
treatment a 44-year-old man presents wi- developed immediately after the teeth erupti-
th incorrectly healed displaced mandibular on. Objectively on the vestibular and masti-
fracture. Objectively the lower jaw narrows catory surfaces of all patient’s teeth there
sharply, vestibular cusps of the lower teeth are enamel defects, tooth crowns present
contact with oral cusps of the upper teeth. with dark brown discoloration. Percussion
The patient declined surgical treatment. and probing are painful. In this area fluori-
What treatment tactics should be chosen by de levels in water are 2.6 mg/L. Make the
the dentist in this case? provisional diagnosis:
A. Make a prosthesis with double dentition A. Fluorosis
B. Make a non-removable dental bridge with B. Systemic hypoplasia
movable joint C. Chronic initial caries
C. Correct the malocclusion by filing down the D. Enamel erosion
patient’s teeth E. Chronic superficial caries
D. Correct the malocclusion via instrumental
surgical method 82. A 14-year-old teenager complains of dry
E. Make a dentogingival laminar denture and chapped lips especially in autumn and
winter. Objetively the vermillion border is
77. A 57-year-old man presents with habi- dry, infiltrated, and covered in numerous
tual mandibular dislocation. To reduce mouth scales. Skin of the lips is dense, pigmented,
opening, Yadrova apparatus was made. How with pronounced pattern, peeling, and radial
long should the treatment last in this case? cracks. At the same time the skin of the face
is dry, lichenified, and excoriated. What is the
A. 3 months provisional diagnosis?
B. 6 months
C. 9 months A. Atopic cheilitis
D. 12 months B. Exfoliative cheilitis
E. 18 months C. Actinic cheilitis
D. Meteorological cheilitis
78. An HIV-infected patients needs a dental E. Allergic contact cheilitis
prosthesis. The dentist plans to make dental
bridges for this patient. How should the 83. A 56-year-old patient suffering from
instruments be processed after the appoi- exacerbation of schizophrenia has been
ntment? hospitalized in an oral in-patient department
with a diagnosis of the displaced mandibular
A. According to the special scheme fracture in the area of teeth 34 and 35. What
B. According to the usual scheme method of treatment should be prescribed?
C. In a hot air sterilizer
D. With lysoformin A. Osteosynthesis
E. With 3% chloramine solution B. One arch smooth dental braces
C. Full dental splint
79. A 30-year-old man complains of pain in D. Weber splint
his front lower teeth, which he attributes to E. Vankevych splint
a trauma to the mental region. Objectively:
continuous dentition, orthognathic occlusion. 84. During preventive examination a 5-year-
X-ray shows a median mandibular fracture. old child was determined to have insufficient
What dental splint would be optimal? physiological attrition of the cusps of the deci-
duous canines. What treatment tactics should
A. Flat occlusal splint the doctor choose?
B. Soldered splint on rings
C. Cap splint A. To shave off the retained canine cusps
D. Weber’s splint B. Medical examination once a month until
E. Plastic mouthguard the incisors are replaced
C. Medical examination every 6 months until
80. To make the external prosthesis for a 62- the incisors are replaced
year-old man it is necessary to obtain a Hi- D. Medical examination every 6 months until
ppocrates facial moulage of this patient. What the canines are replaced
impression material should be used? E. No medical intervention is necessary
85. A 60-year-old patient has been undergoi-
Krok 2 Stomatology (англомовний варiант, iноземнi студенти) 2018 рiк 11

ng the procedure of checking the complete A. Periosteotomy and pharmacotherapy


removable dentures construction and fixi- followed by treatment of the causative tooth
ng teeth on wax bases. The following flaws B. -
have been detected: fissure between the teeth C. Extraction of the causative tooth, physi-
on the frontal area and cusp-to-cusp contact otherapy
in the lateral area. What mistake had been D. Extraction of the causative tooth,
made? pharmacotherapy
E. Extraction of the causative tooth, peri-
A. Anterior occlusion had been determined osteotomy
instead of central one
B. Posterior occlusion had been determined 89. A 30-year-old patient complains of pain
instead of central one and swelling in the area of the left parotid
C. Lateral occlusion had been determined salivary gland, which occurred 7 days after
instead of central one he had undergone abdominal cavity surgery.
D. Models had been plastered in a wrong way Objectively: body temperature is 39o C ,
in an occluder reduced mouth opening, dry mouth; when the
E. Swabs had been crushed during determi- gland is massaged, there is purulent exudate
nation of central occlusion being secreted from its duct. The patient can
be diagnosed with the following disease:
86. A 35-year-old man complains of sour
sensation in his mouth and front teeth A. Acute non-epidemic parotitis
sensitivity to thermal and mechanical sti- B. Acute epidemic parotitis
muli. Objective examination revealed visi- C. Phlegmon of submasseteric space
ble changes in the enamel of 13, 12, 11, 21, D. Parenchymatous parotitis
22, and 23. The enamel is dull, rough, mi- E. Phlegmon of parotid-masticatory region
ssing on the cutting edge. Probing of the
vestibular surface of these teeth is painful, 90. A 45-year-old man presents with facial
response to thermal stimuli is positive. The asymmetry due to a dense isolated infiltrati-
patient’s medical record states his occupati- on in his right buccal area; the skin over the
on in industrial production of inorganic acids. infiltration is cyanotic, thinned out; in the
What is the most likely diagnosis? center of the infiltration there is a fistula. In
the oral cavity the crown of 46 is destroyed
A. Necrosis of dental hard tissues by 2/3, along the mucogingival fold the band
B. Fluorosis connecting the tooth with the fistula can be
C. Enamel erosion palpated. Make the diagnosis:
D. Pathologic teeth grinding
E. Enamel hypoplasia A. Migrating facial granuloma
B. Chronic mandibular osteomyelitis
87. A 55-year-old man was delivered into the C. Cheek furuncle
hospital with bilateral mandibular fracture D. Odontogenic lymphadenitis
within the dentition. Objectively teeth 34, 35, E. Actinomycosis
36, 45, and 46 are missing. Lower incisors are
mobile (I-II degree). Fragment displacement 91. A 53-year-old patient complains of an
is insignificant. What splint should be made ulcer on the lateral surface of the tongue. The
for this patient? ulcer appeared 6 months ago as the result of
a trauma caused by sharp tip of the 37 tooth
A. Weber metal crown. A dentist replaced the crown
B. Vankevych with the one of better quality and prescribed
C. Tigerstedt keratoplastic drugs. Despite these measures
D. Flat occlusal splint the ulcer continues to grow. Lately there has
E. Gunning-Port been pain observed during talking, chewi-
ng, swallowing; sometimes the pain irradi-
88. A 30-year-old patient is diagnosed with ates to the pharynx. Objectively on the lateral
acute suppurative odontogenic periostitis of surface of the tongue there is a painful ulcer
the upper left jaw originating from tooth 23. with uneven raised dense margins and lumpy
The crown of 23 on the left is destroyed with floor with grayish necrotic coating. What is
caries by 1/3. Teeth 22 and 24 are intact. Spot- the most likely diagnosis?
film X-ray shows widening of the periodontal
fissure of 23. What treatment would be the A. Cancer of the lateral surface of the tongue
most advisable in this case? B. Trophic ulcer
C. Traumatic ulcer
D. Vincent’s necrotizing ulcerative stomatitis
E. Tuberculous ulcer
92. A 13-year-old child complains of peri-
odical gingival bleeding during teeth brushi-
ng, which has been observed for half a year.
Objectively the gingival mucosa in the frontal
Krok 2 Stomatology (англомовний варiант, iноземнi студенти) 2018 рiк 12

mandibular area presents with congestive A. Calcium hydroxide paste


hyperemia and edema. Decay-missing-filled B. Zinc phosphate cement
(DMF) index equals 4. Oral cavity hygiene is C. Resorcinol-formalin paste
unsatisfactory. In this case it is necessary to D. Glass ionomer cement
recommend the patient the toothpastes with: E. -

A. Herbal extracts 97. The medical station of a regiment received


B. Zinc citrate a patient with signs of bilateral mandibular
C. Calcium glycerophosphate fracture. What is the main task of first aid in
D. Amine fluorides this case?
E. Salt additives
A. To control shock, bleeding, and asphyxia
93. A 70-year-old man has edentulous maxi- and to provide transport immobilization
lla. Objectively maxillary tuberosity and B. To check and correct previously applied
alveolar processes are completely atrophi- bandages
ed; palatine vault is flat, its mucosal layer is C. To administer analgesics and cardiac medi-
moderately pliant. In this case the patient’s cations
atrophic edentulous maxilla can be classified D. To clean oral cavity from blood clots, tooth
as: shards, and bone fragments
E. To provide symptomatic therapy and care
A. Schroeder class III
B. Keller class III 98. A 35-year-old man has been hospitali-
C. Schroeder class II zed into a dentofacial unit with complaints of
D. Keller class II mobility of the 38, 37, and 36 teeth and a fi-
E. - stulous tract in the socket of the extracted 35
tooth. The condition has been persisting for 3
94. Teeth 71 and 81 erupted in a 6-year-old months. Insertion of a grooved probe into the
child, the lower jaw is retrogenic, the palate is fistulous tract palpated a bared coarse bone
flat with pronounced cross-folds. Determine fragment that easily moved under pressure.
the condition of the oral cavity: X-ray of the lower jaw demonstrates a focus
of bone tissue destruction, with a spot of
A. Physiologic dense bone tissue 0.5х0.3 cm in size. Make
B. Pathologic the diagnosis:
C. Abnormal
D. Subcompensated A. Chronic osteomyelitis
E. Decompensated B. Acute osteomyelitis
C. Exacerbation of chronic osteomyelitis
95. A 12-year-old girl complains of pain in her D. Chronic periostitis
mouth that occurs during eating. Accordi- E. Actinomycosis
ng to her medical history these symptoms
reemerge once or twice per year. Objectively 99. A 10-year-old child complains of persi-
on the mucogingival fold there are 3 aphthae sting throbbing pain in tooth 36, whi-
5-7 mm in size, they have yellowish coati- ch appeared one day ago. Hot stimulus
ng and inflamed red border and are acutely aggravates the pain, while cold mitigates it
painful on touch. Name the most likely di- slightly. Objectively on the masticatory and
agnosis: medial surfaces of tooth 36 there is a deep
carious cavity non-communicating with the
A. Chronic recurrent aphthous stomatitis dental cavity. Probing of the cavity bottom
B. Acute herpetic stomatitis and percussion are painful. X-ray shows no
C. Chronic recurrent herpetic stomatitis pathologic changes of the periodontium.
D. Erythema multiforme exudativum What treatment method shold be chosen in
E. Toxic-allergic drug-induced stomatitis this case?
96. An 8-year-old girl complains of tooth A. Vital extirpation
21 discoloration and pain response to hot B. Devital extirpation
stimulus. Several months ago tooth 21 was C. Vital amputation
treated for acute diffuse pulpitis by means of D. Devital amputation
vital amputation. Objectively tooth 21 is fi- E. Conservative treatment
lled, percussion is painless. X-ray shows the
rooth to be formed by 2/3, cortical plate of 100. A 11-year-old child complains of pain in
the tooth socket remains intact in the area of the lower right lateral tooth, which occurs
root radix. What material should be used for when eating, especially hot food. On the
root canal filling in this case? masticatory surface of the 46 tooth there is a
large carious cavity filled with softened light-
brown dentin. The cavity is located within
parapulpar dentin. In the projection of the
medial buccal pulp horn the carious cavity
communicates with the pulp chamber. Deep
Krok 2 Stomatology (англомовний варiант, iноземнi студенти) 2018 рiк 13

probing is painful. Electric pulp test - 60 mi- A. Make a temporary removable denture
croamperes. Make the diagnosis: B. Make a clasp-retained (bugel) removable
partial denture
A. Chronic gangrenous pulpitis C. Make a dental bridge with 12 and 21 as
B. Chronic hypertrophic pulpitis abutment teeth
C. Acute diffuse pulpitis D. Temporarily refrain from making a denture
D. Chronic fibrous pulpitis E. Perform implantation
E. Acute focal pulpitis
105. A 23-year-old man complains of acute
101. A 7-year-old practically healthy child was gingival bleeding and unpleasant smell from
undergoing the carious cavity preparation of the mouth that appeared 5 days ago. Objecti-
tooth 46 due to acute median caries. During vely gingival papillae and marginal gingi-
this procedure the mesio-buccal pulp horn va are friable, bright red, swollen, painful,
was accidentally exposed. What treatment and bleed profusely on palpation. Gingival
would be optimal in this case? pockets are 3 mm deep. X-ray shows marked
osteoporosis of the interalveolar septa, peri-
A. Biological approach odontal fissure in the apical areas of the
B. Devital amputation interalveolar septa is enlarged. Cortical plate
C. Devital extirpation is intact. Make the diagnosis:
D. Vital amputation
E. Vital extirpation A. Acute catarrhal gingivitis
B. Acute leukemia
102. A woman complains of pain in her C. Acute necrotizing ulcerative gingivitis
gums, unpleasant smell from her mouth, D. Generalized periodontitis, stage II,
difficult eating, general weakness, low- exacerbated development
grade fever. Objectively her gums are E. Hypovitaminosis C
hyperemic, with areas of ulceration, covered
in necrotic deposit. Microscopy revealed 106. A 27-year-old patient has been referred
fusospirochetosis. Choose the medication for by a prosthodontist for endodontic treatment
etiotropic treatment: of the 45 tooth. Objectively: the 45 tooth
crown is destroyed; the lateral surface of
A. Metronidazole the tongue and the buccal mucosa have
B. Keratoline patches of grayish macerated epithelium sli-
C. Galascorbin ghtly protruding above the mucosa surface at
D. Chlorhexidine the points of direct contact with the 45 tooth.
E. Chymotrypsin The uvula and palatal bars are stagnant-
red in colour; hard palate has papulae
103. A man complains of gingival bleeding surrounded with red margin and covered
that has been persisting for the last 2 years. in grayish epithelium. The submandibular,
Objectively he presents with chronic diffuse cervical, supraclavicular, and subclavicular
catarrhal gingivitis, teeth mobility is of the lymph nodes are enlarged and painless. What
I degree, periodontal pockets are 2-3 mm is the provisional diagnosis?
deep with small amount of serous exudate,
occlusion is markedly traumatic. X-ray shows A. Secondary syphilis
damaged cortical plate, enlarged periodontal B. Chronic recurrent aphthous stomatitis
fissure in the apical areas of the interalveolar C. Lupus erythematosus, patch stage
septa, osteoporosis, and interalveolar septa D. Soft leukoplakia (leucoplakia mollis)
resorption by 1/3 of their height. Make the E. Lichen ruber planus
diagnosis:
107. A 6-year-old boy with congenital heart
A. Generalized periodontitis, stage I, chronic disease (pulmonary artery stenosis) presents
development with suppurative periostitis of the maxilla.
B. Chronic catarrhal gingivitis The child needs surgical treatment. What unit
C. Parodontosis, stage I should he be referred to?
D. Generalized periodontitis, stage I,
exacerbated development A. The pediatric maxillofacial unit
E. Generalized periodontitis, early stage, B. The out-patient unit, no precautions are
chronic development necessary
C. The out-patient unit after preliminary
104. A 55-year-old man came to the cardiological treatment
prosthodontic clinic to have a denture made D. The cardiology unit
for him. Tooth 11 is missing in the pati- E. The out-patient or in-patient unit at the
ent. Two days ago he was released from the discretion of the child’s parents
in-patient unit after a case of myocardial
infarction. What tactics should the dentist 108. Parents of a 3-year-old child complain of
choose? food periodically getting into the child’s nasal
cavity during feeding. Objectively there is a
fissure in the area of the soft palate. Make the
Krok 2 Stomatology (англомовний варiант, iноземнi студенти) 2018 рiк 14

diagnosis: A. Temporary molars


B. Temporary central incisors
A. Isolated partial nonunion of the soft palate C. Temporary lateral incisors
B. Congenital hidden cleft palate D. Temporary canines
C. Isolated nonunion of the hard and soft E. Front teeth
palate
D. Cleft hard palate 113. Due to trauma of the area of teeth 44
E. Combined cleft palate and 45, a 12-year-old boy suffers from the
pathologic displacement of the mandibular
109. A 9-year-old girl complains of persisting alveolar process and rupture of the alveolar
pain in tooth 11 that one month ago sustained process mucosa. What additional examinati-
a trauma resulting in broken crown. The tooth on is necessary to specify the diagnosis?
received no treatment. Objectively the 1/4 of
the tooth crown of 11 is broken off, the remai- A. X-ray of the mandible in frontal and lateral
ning crown is grayish, the dental cavity is not projections
exposed. Percussion is acutely painful. The B. Skull X-ray in axillary projection
mucogingival fold is hyperemic and acutely C. X-ray of the mandible in frontal and Parma
painful on palpation. Make the diagnosis: projection
D. Computed tomography of the mandible
A. Acute traumatic periodontitis E. -
B. Acute diffuse suppurative pulpitis
C. Acute suppurative periodontitis 114. A 12-year-old child presents with
D. Exacerbation of chronic periodontitis temperature 38o C , chills, nausea, vomiti-
E. Chronic granulating periodontitis ng, delirium, and weakness. On the middle
third of the face there is hyperemia in a
110. A patient complains of fever up to 38o C , butterfly-shaped pattern. Regional lymph
headache, pain in the joints anf muscles, vesi- nodes are enlarged and mildly painful. In
cles in the oral cavity, mainly in the frontal blood: leukocytes - 12 · 109 /L, lymphocytes -
part. Eating is sharply painful. For the last 8.0 · 109 /L, ESR- 26 mm/hour. What diagnosis
several years the disease has been recurring should be made?
during wet and windy weather. The patient
often suffers from cases of URTI. Objecti- A. Erysipelatous inflammation
vely: on the buccal, lingual, labial mucosa B. Facial vein thrombophlebitis
there are confluent erosions against the C. Cutaneous actinomycosis
erythematous background, with gray-white D. Streptoderma, submandibular
coating. There are bloody scabs on the vermi- lymphadenopathy
llion border and in the angles of the mouth. E. Acute non-odontogenic maxillary sinusitis
Make the diagnosis:
115. X-ray of the patient shows a focus
A. Erythema multiforme exudativum of bone destruction 3x4 cm in size in the
B. Syphilis mandibular body. The focus is structured
C. Acute aphthous stomatitis as numerous small cavities different in size
D. Acute necrotizing ulcerative stomatitis and shape and separated by septa. Tumor
E. Chronic recurrent aphthous stomatitis puncture yielded brown liquid. What is the
most likely diagnosis?
111. A patient with complaints of toothache
in the left upper jaw has made an appoi- A. Giant cell tumor of the mandible
ntment with a dental clinic. He was diagnosed B. Radicular cyst of the mandible
with chronic periodontitis of the 24 tooth. C. Carcinoma of the mandible
What kind of anesthesia is necessary for pai- D. Soft odontoma of the mandible
nless extraction of the 24 tooth? E. Mandibular ameloblastoma
A. Infraorbital and palatinal anesthesia 116. An orthodontist has been addressed by
B. Tuberal and palatinal anesthesia parents of a 5-year-old child. The child has the
C. Infraorbital and incisor anesthesia 54 tooth extracted, all the other deciduous
D. Tuberal and incisor anesthesia teeth are present. The doctor made a thin-
E. Surface and tuberal anesthesia wall crown for the 55 tooth with interdental
wedge to the 53 tooth. What is the purpose of
112. A 2.5-year-old child is registered for such treatment?
regular check-ups with the orthodontist. The
I stage of physiogical occlusion development A. Prevention of dentition malformation
corresponds with eruption of the following B. Aesthetic restoration
group of temporary teeth: C. Restoration of masticatory efficiency
D. Acceleration of permanent tooth eruption
E. Deceleration of permanent tooth eruption
117. Removable partial dentures for upper
and lower teeth are being made for a 45-year-
Krok 2 Stomatology (англомовний варiант, iноземнi студенти) 2018 рiк 15

old man. Complete anatomical impressions should the dental surgeon give to the patient
were made using ”Ypeen” alginate material. for the procedure of tooth extraction?
What should be used for disinfection of obtai-
ned impressions? A. Mandibular and buccal anesthesia
B. Intraoral infraorbital nerve block
A. 2.5% glutaraldehyde with рH- 7.0 - 8.7 C. Tuberal anesthesia
B. 0.1% desoxone solution D. Mandibular anesthesia
C. - E. Mental nerve block
D. Phenol solution in proportion 1:20
E. 6% hydrogen peroxide solution 122. A 34-year-old man complains of soft ti-
ssues edema in his lower left jaw and fistulae
118. A 48-year-old man complains of gingival in the submandibular area. Teeth 36 and 37
overgrowth (”gums cover the teeth”). The are destroyed. Alveolar mucosa is swollen
patient suffers from epilepsy and takes anti- and hyperemic at the level of 36 and 37. X-ray
convulsant agents. Objectively gingival papi- detected sequestra in the mandibular body
llae are of normal color, dense, with lumpy on the left. What treatment method should
surface; they do not bleed on probing and be chosen in this case?
cover the lower front teeth up to their incisal
surfaces. What is the most likely diagnosis? A. Extraction of teeth 36 and 37 and mandi-
bular sequestrectomy
A. Hypertrophic gingivitis, fibrous form, B. Extraction of teeth 36 and 37
degree III C. Mandibular sequestrectomy
B. Hypertrophic gingivitis, edematous form, D. Puncture of the inflamed area
degree III E. Antibacterial treatment
C. Gingival fibromatosis
D. Hypertrophic gingivitis, fibrous form, 123. A 42-year-old man was delivered to the
degree II hospital in the severe condition: inert, body
E. Hypertrophic gingivitis, edematous form, temperature is 39.1o C , there is acutely painful
degree II infiltration of the mouth floor and submandi-
bular area on the right. The skin over the
119. A 21-year-old man came to the denti- infiltration is turgid and cyanotic. Palpati-
st complaining of general weakness, muscle on detects crepitus under the skin. What di-
pain, body temperature up to 38.3o C , indi- agnosis can be made in this case?
gestion, excessive salivation, and rashes in
the oral and nasal cavities, urethra, on the wi- A. Ludwig’s angina (suppurative-necrotic
ngs of the nose, and in the interdigital folds. phlegmon of the mouth floor)
These symptoms appeared after ingestion of B. Adenophlegmon of the mouth floor
milk during the patient’s stay in the village. C. Malignant tumor of the mouth floor
What is the most likely diagnosis? D. Actinomycosis of the mouth floor
E. Odontogenic phlegmon of the mouth floor
A. Murrain
B. Herpetic stomatitis 124. After a blow to the temporomandibular
C. Herpes zoster joint the patient developed facial hematoma,
D. Behcet’s disease the joint is difficult to move, mandibular
E. Infectious mononucleosis mobility is reduced. Attempts to open the
mouth wide are painful. What examination
120. A 28-year-old man complains of pai- should be performed to make the diagnosis?
nless sore in his mouth that persists despi-
te the attempts at self-treatment. Objecti- A. Bilateral X-ray of the temporomandibular
vely the regional lymph nodes on the left joint with mouth open and closed
are enlarged and painless. Mucosa of the left B. Limit the joint mobility
cheek presents with round ulcer, 1 cm in di- C. X-ray and consultation with the neurologist
ameter, with raised margins and cartilage-like D. Rheoencephalography and consultation
infiltration in its basis. The surface of the ulcer with the neurologist
is colored meat red and painless on palpation. E. Panoramic dental X-ray
What is the most likely diagnosis?
125. A 38-year-old man after a domestic acci-
A. Primary syphilis dent complains of pain and mobility of his
B. Cancer upper teeth, problems with eating. Objecti-
C. Secondary syphilis vely: soft tissues edema. The 11 and 21 teeth
D. Lupus vulgaris are displaced towards the palate, mobile
E. Decubitus ulcer (II degree), painful on percussion. Mucosa
surrounding the affected teeth is hyperemic
121. A 25-year-old woman made an appoi- and swollen. X-ray demonstrates widened
ntment with the dental surgeon for oral cavi- periodontal fissure of the 11 and 21. Choose
ty sanation. Objectively the crown of tooth 37 the treatment method:
is destroyed by 2/3. Gingival mucosa around
tooth 37 is without changes. What anesthesia
Krok 2 Stomatology (англомовний варiант, iноземнi студенти) 2018 рiк 16

A. Setting of the teeth and their fixation with is NOT a part of typical procedure of tooth
a flat occlusal splint extraction with forceps?
B. Extraction of the 11 and 21 teeth
C. Reimplantation of the 11 and 21 teeth A. Applying tip of forceps jaw to the edge of
D. Immobilization or mouthguard alveolar process
E. Removal of tooth pulp in the 11 and 21 B. Applying forceps jaw to the tooth
teeth C. Pushing forceps jaw to the cementoenamel
junction
126. A 4-year-old child has developed acute D. Closure of forceps handles
spontaneous pain in the tooth on the lower E. Tooth dislocation and extraction from the
right jaw, which aggravates on biting. Objecti- socket
vely: in the 85 tooth there is a deep carious
cavity non-communicating with the dental 131. A patient needs the 36 tooth extracted.
cavity. Probing is sharply painful at all poi- After administering anesthesia the doctor
nts of the cavity floor. Painful reaction to cold started applying the elevator. However,
water stimulus and percussion is observed; immediately after that the patient suddenly
mucosa surrounding the 85 is hyperemic. paled, complained of dizziness, ear noise, and
Submandibular lymphadenitis is detected. blackout and slid down in the chair. What is
Make the provisional diagnosis: the most likely diagnosis?
A. Acute pulpitis complicated with peri- A. Unconsciousness
odontitis B. Anaphylactic shock
B. Acute serous periostitis C. Collapse
C. Acute serous periodontitis D. Shock
D. Acute suppurative pulpitis E. Hypoglycemic coma
E. Exacerbation of chronic periodontitis
132. A child is diagnosed with congeni-
127. During Eschler-Bittner test the profile tal cleft in the soft palate and posteri-
of a 12-year-old girl with posterior occlusion or part of the hard palate. What type of
has shown some improvement. Specify the anesthesia should be given to the patient for
condition that resulted in the development of uranostaphyloplasty?
posterior occlusion in this patient:
A. Intubation narcosis
A. Mandibular underdevelopment B. Intravenous narcosis
B. Maxillary overdevelopment C. Anesthesia mask
C. Mandibular underdevelopment and maxi- D. Infiltration anesthesia
llary overdevelopment E. Conduction anesthesia
D. Mandibular overdevelopment
E. Maxillary underdevelopment 133. A 6-year-old child complains of pain
and edema in the upper right jaw, body
128. Parents of an 8-year-old girl complain of temperature up to 37.9o C , and deterioration
their child having an aesthetic defect of her of general well-being. Symptom onset was 3
teeth. Objectively the patient’s lower face is days ago. Objectively the face is asymmetric
shortened. Her chin protrudes forwards and due to soft tissue edema of buccal and
her upper lip is sunken. During teeth closure infraorbital regions on the right. The crown
the deep underbite becomes apparent. of 54 is destroyed by 1/2, percussion is pai-
Mesio-occlusion is observed in the lateral nful; the tooth previously had been treated
areas. Choose the apparatus optimal for the for complicated caries. On the palatine side
treatment: of the affected tooth area there is a painful
infiltration with fluctuation in its center; the
A. Frankel functional regulator - 3 tissues over the infiltration are hyperemic.
B. Frankel functional regulator - 2 Make the provisional diagnosis:
C. Osadchy apparatus
D. Andresen-Haupl activator A. Acute suppurative periostitis of the maxilla
E. Frankel functional regulator - 1 originating from tooth 54
B. Acute serous periostitis of the maxilla
129. During or immediately after an injecti- originating from tooth 54
on, certain local complications can develop. C. Acute odontogenic osteomyelitis of the
What is NOT one of those complications? maxilla
D. Exacerbation of chronic periodontitis of 54
A. Mucosal necrosis E. Chronic odontogenic osteomyelitis of the
B. Dermal ischemia maxilla
C. Diplopia
D. Functional paralysis or paresis of facial 134. A 27-year-old man presents with missi-
muscles ng crown of 11. Objectively teeth 21 and 12
E. Damage to a blood vessel by the needle are intact; intraoral spot film X-ray shows the
root of 11 to be filled to the apex, no changes
130. What manipulation of those listed below in the periapical tissues, no pathologic mobi-
Krok 2 Stomatology (англомовний варiант, iноземнi студенти) 2018 рiк 17

lity. What construction of the denture should socket and in the area of zygomaticoalveolar
be recommended for this patient? crest. What is the most likely diagnosis?
A. Metal stump inlay with overlaying A. Displaced fracture of the temporal bone
porcelain-fused-to-metal crown B. Le Fort II maxillary fracture
B. Stump inlay with overlaying swaged crown C. Nondisplaced fracture of the temporal bone
C. Stump inlay with overlaying plastic crown D. Le Fort I maxillary fracture
D. Stump inlay with overlaying full cast metal E. Zygomatic arch fracture
crown
E. Remove the root of 11 and perform 139. A 14-year-old boy complains of rapid
implantation wearing-off of tooth crowns. Objectively:
tooth crowns are worn-off by 1/3. Enamel
135. A 54-year-old patient complains of easily chips off and is pale gray in color. Make
frequent crunching sound in the right the diagnosis:
temporomandibular joint, which developed
one month ago. In the morning the crunching A. Stainton-Capdepont syndrome
is more frequent and decreases towards the B. Dentinogenesis imperfecta
evening. Objectively: the face is symmetrical, C. Fluorosis
the skin over the joint is without changes, the D. Systemic hypoplasia
mouth opens by 2.9 mm. What is the most E. Focal hypoplasia
likely diagnosis in this case?
140. A 63-year-old man complains of pain
A. Arthrosis in the area of maxillary mucogingival fold
B. Acute arthritis caused by using a removable laminar denture.
C. Temporomandibular joint dislocation Objectively: in the area of the mucogingi-
D. Chronic arthritis val fold there is a trophic ulcer with swollen
E. Temporomandibular joint pain dysfunction margins and hemorrhaging floor. Make the
syndrome diagnosis:

136. On examination of a 27-year-old pati- A. Denture-related stomatitis


ent the tip of the dental probe caught on B. Toxic chemical stomatitis
the fissures of the 36, 37, and 38 teeth. C. Toxic infectious stomatitis
Margins of the enamel defect are dark, the D. Allergic contact stomatitis
surface is coarse. Teeth transillumination wi- E. Greenhouse effect
th photopolymer lamp revealed the defect to
be limited to the enamel. What is the most 141. Objective examination of a 10-year-old
likely diagnosis? child revealed slight hyperemia, infiltration,
and dryness of the whole surface of the vermi-
A. Chronic superficial caries llion border. Architectonics of the lips is di-
B. Chronic median caries sturbed. Dryness and contracted sensation
C. Acute superficial caries are observed in the lips, especially during cold
D. Chronic initial caries seasons. Make the provisional diagnosis:
E. Acute initial caries
A. Meteorological cheilitis
137. A 68-year-old patient addressed a surgi- B. Atopic cheilitis
cal department of a dental clinic for extracti- C. Allergic contact cheilitis
on of the 45 tooth. During procedure the D. Exfoliative cheilitis
patient developed burning retrosternal pain E. Cheilitis of microbial origin
attack irradiating to the left shoulder, scapula,
hand. The skin is pale, BP is 140/100 mm 142. A 24-year-old woman made an appoi-
Hg, heart rate is rapid. Skin hyperplasia can ntment with the dental surgeon for extraction
be observed in the Zakharin-Head’s zones. of tooth 38. What anesthesia should be gi-
What emergency condition did the patient ven to the patient for the procedure of tooth
develop? extraction?

A. Angina pectoris attack A. Torusal


B. Heart failure B. Mandibular
C. Bronchial asthma attack C. Tuberal
D. Hypertensic crisis D. Infiltration
E. - E. Plexus

138. After sustained trauma a man developed 143. During application of tuberal anesthesia
nose bleeding, reduced mouth opening, the patient developed rapidly increasing ti-
sensation of paresthesia in the right infraorbi- ssue edema and reduced mouth opening.
tal region and lower eyelid. Objectively the What resulted in such a condition?
face is asymmetric due to concave right
temporal region; step deformity symptom is
observed in the middle of the lower right eye
Krok 2 Stomatology (англомовний варiант, iноземнi студенти) 2018 рiк 18

A. Vascular trauma
B. Muscle trauma during anesthesia applicati- A. Collapse
on B. Anaphylactic shock
C. Nerve trunk trauma C. Allergic response to the anesthetic
D. Intolerance to the anesthetic D. Pain shock
E. Anaphylactic shock E. Vertigo
144. A woman came to the dental surgeon wi- 149. An ambulance has delivered an 8-
th complaints of teeth mobility. After objecti- year-old child to an admission room. An
ve examination and X-ray analysis she was oral surgeon has made the following di-
diagnosed with generalized periodontitis of agnosis: odontogenic phlegmon of the right
stages I and II. Which teeth of those affected submandibular area. What surgical approach
by periodontitis should be extracted? would be advicable for surgical treatment of
this phlegmon?
A. With degrees 2-3 of tooth mobility
B. Intact teeth A. Dissection in the submandibular area,
C. Teeth with painful percussion parallel to the mandible
D. With degree 1 of tooth mobility B. Dissection parallel to the torus mandi-
E. Carious teeth bularis
C. Dissection around the mandibular angle
145. After the inflammatory process in the D. Dissection along the lower neck fold
parotid area a woman developed frequent E. Dissection in the area of pterygomandi-
pain attacks resembling electric current in bular fold
her face on the right. The attacks last for 15-
20 minutes. The most likely diagnosis is: 150. A 42-year-old man came to the
prosthodontics clinic to have dental
A. Trigeminal neuralgia prosthesis made for him. Objectively teeth
B. Tympanic plexus neuralgia 34, 35, 36, 43, and 16 are absent in this pati-
C. Trigeminal neuritis ent. During examination alginate impressions
D. Exacerbation of chronic maxillary sinusitis for diagnostic models were obtained. What
E. Exacerbation of chronic osteomyelitis should be used for disinfection of obtained
impressions?
146. Mother and her 11-year-old daughter
came to the medical station. According to the A. 0.5% sodium hypochlorite solution
mother her daughter has fallen when playing B. 3% hydrogen peroxide solution
outdoors. The doctor determined the patient C. 10% hydrogen peroxide solution
to have an isolated mechanical injury of soft D. Sterillium
tissues in her cheek with damaged skin. Make E. 3% sodium hypochlorite solution
the diagnosis:
151. A 36-year-old woman needs a dental
A. Abrasion prosthesis. Objectively there is a carious cavi-
B. Contusion ty on the mesial masticatory surface of tooth
C. Bruice 46, interdental contact is disturbed. Dental
D. Hematoma inlay is to be made for this patient. According
E. Wound to Black’s classification of dental caries this
cavity is class:
147. The patient with shallow vestibule of
mouth and edentulous mandible underwent a A. 2
surgery: a mucoperiosteal flap was relocated B. 3
from the alveolar ridge to the body of the C. 4
mandible and fixed with denture acting as a D. 5
bandage. What surgical procedure was used E. 1
for vestibular deepening?
152. A 45-year-old man complains of pain
A. Rumpel and crepitation in the temporomandibular
B. Kazanjian joint during the movements of the lower
C. Thiersch jaw. Objectively: the face is symmetrical,
D. Trauner the mouth opens with slight displacement
E. Rhermann to the left. Dentition is intact. On occlusi-
ography there were detected centric and
148. A 7-year-old girl received conduction eccentric supracontacts. What treatment
anesthesia with 2% articaine solution for methods should be applied in the first place?
extraction of tooth 16. She has no history of
allergies. After receiving anesthesia the pati-
ent complained of weakness, she developed
skin pallor, cyanosis, and nausea. Her blood
pressure dropped significantly. The patient is
conscious. What is the provisional diagnosis?
Krok 2 Stomatology (англомовний варiант, iноземнi студенти) 2018 рiк 19

A. Selective teeth shaving A. Fixation of the lower jaw and tongue with
B. Mouthguard for muscle relaxation standard Entin’s head-chin strap
C. Appliances that limit mouth opening B. Fixation of the tongue to the patient’s
D. Mouthguards that increase the height of collar
central occlusion C. Tracheostomy and artificial pulmonary
E. Lower jaw immobilization ventilation
D. Removal of foreign bodies from the oral
153. A 45-year-old patient after administrati- cavity
on of local anesthesia in preparation for oral E. Excision of injured mucosal flaps
surgery has suddenly felt unwell, developed
increasing edema of laryngeal mucosa and 157. The hospital received a 19-year-old pati-
respiration disorder. The dentist stopped the ent injured in a landmine explosion wi-
manipulations in the oral cavity. What type of th an open displaced mandibular fracture
asphyxia developed in the patient? in the area of the left mandibular angle
and a contused lacerated wound of the
A. Stenotic left submandibular area. The patient is in
B. Dislocational moderately severe condition, pulse is 80/min.,
C. Valvular blood pressure is 110/80 mm Hg. What
D. Obturative anesthesia should be given to the patient for
E. Aspiration initial surgical d-bridement?
154. A 19-year-old young man, who was wai- A. Endotracheal anesthesia
ting for the appointed time at the denti- B. Intravenous anesthesia
st’s, suddenly developed an attack: his face C. Neuroleptanalgesia
became purple, bloody foam flowed from D. Conduction anesthesia with premedication
his mouth (bitten tongue), pupils were di- E. Conduction anesthesia
lated and unresponsive to light, the patient
developed first tonic then clonic convulsi- 158. A 5-year-old child developed a
ons that stopped spontaneously, after that he hemorrhage after pulp extirpation of 74 due
calmed down and fell asleep quickly. What to exacerbated chronic pulpitis. The child
happened with the patient? suffers from Von Willebrand disease. What
actions should be taken by the dental surgeon
A. Epileptic seizure to stop bleeding?
B. Sympathoadrenal crisis
C. Spasmophilia A. To hospitalize the child to the hematologi-
D. Morgagni-Adams-Stokes syndrome cal unit
E. Pulmonary embolism B. To hospitalize the child to the maxillofacial
unit
155. A victim of a traffic accident was deli- C. To place sutures on the mucosa
vered into the admission room. The patient is D. To plug the cavity with epsilon-
supine and unconscious. His skin is cyanotic, aminocaproic acid dressing
respiration is extremely labored, mucosa is E. To plug the cavity with hemostatic sponge
pale, blood clots are accumulated in the oral
cavity. The patient is diagnosed with displaced 159. Parents of a 9-year-old child came to the
bilateral mandibular fracture. How should dentist complaining that their child presents
this patient be transported? with enlarged cervical lymph nodes on the ri-
ght. During examination palpation revealed
A. In the prone position on a soft stretcher the lymph nodes in the right submandibular,
B. Positioned on the side on a rigid stretcher cervical, supraclavicular, and infraclavicular
C. In the sitting position with the head thrown areas to be enlarged up to 2-2.5 cm in di-
back ameter, painless, non-matted together, and
D. In the sitting position with air tube inserted non-fused to the skin (resemble ”potatoes
into the upper airways in a sack”). The parents note rapid fatigabi-
E. Positioned on the side on a soft stretcher lity and night sweats in their child. What
additional examinations should the child be
156. A victim of a traffic accident was deli- referred for?
vered into the admission room. The patient is
supine and unconscious. His skin is cyanotic, A. Puncture biopsy of the lymph nodes
respiration is extremely labored, mucosa is B. Pirquet and Mantoux tests
pale, blood clots are accumulated in the oral C. Wassermann test
cavity. The patient is diagnosed with displaced D. CT of the cervical spine
bilateral mandibular fracture. What measures E. Clinical blood and urine tests
should be taken to prevent complications in
this case? 160. A 45-year-old patient complains of inabi-
lity to properly masticate due to the loss of
lateral teeth. The 17, 16, 15, 25, 26, 27, 37, 36,
35, 44, 45, and 46 teeth are missing. The retai-
ned teeth exhibit the I-II degree of mobi-
Krok 2 Stomatology (англомовний варiант, iноземнi студенти) 2018 рiк 20

lity. The patient is diagnosed with generali- probing causes severe pain, percussion of the
zed periodontitis. Kennedy class I dentiti- 17 tooth is painful. X-ray: there is slight wi-
on defects are observed. What construction dening of the periodontal fissure near the root
would be optimal in the given case? apex. Electric pulp test - 70 microamperes.
What final diagnosis can be made?
A. Clasp-retained (bugel) removable partial
denture with splinting elements A. Chronic gangrenous pulpitis
B. Partial laminar denture B. Chronic fibrous pulpitis
C. Elbrecht’s dental splint C. Acute purulent pulpitis
D. Mamlok’s dental splint D. Chronic fibrous periodontitis
E. Cantilever dental bridges E. Exacerbation of chronic fibrous periodonti-
tis
161. A 70-year-old patient addressed a hospi-
tal with complaints of poorly stabilized 165. Medical committee registers the patients,
complete removable dentures of the upper who for a long time lived in an area polluted
and lower jaws. What method of artificial with radiation. The patients are advised on
teeth arrangement is preferable in making the diet that will quickly purge the body from
of a new complete removable denture? radionuclides. The portion of products rich
in pectine should be increased in their diet.
A. According to individual occlusal curves Name these products:
B. According to disocclusal planes
C. According to standard occlusal curves A. Fruits and vegetables
D. According to spherical occlusal curves B. Meat products
E. According to prothetic occlusal planes C. Pasta
D. Dairy products
162. A 2.5-year-old child has fever up to E. Baked goods
38.5o C , low appetite, rashes in the oral cavity.
The disease onset was 3 days ago. Objecti- 166. A 44-year-old man came to extract
vely: the skin of the perioral area is covered in destroyed tooth 24. Objectively his face is
scarce vesicles with clear content. Within the symmetrical, the crown of 24 is destroyed by
oral cavity on the buccal and lingual mucosa 2/3, percussion is painless. Gingival mucosa
there are sharply painful erosions, 2-3 mm surrounding the tooth is unchanged. X-ray
in size, with white coating and hyperemic shows enlarged periodontal fissure in the area
crown. The gums are swollen, hyperemic. The of the root apex. What is the most likely di-
submandibular lymph nodes are enlarged, agnosis?
painful on palpation. Make the diagnosis:
A. Chronic fibrous periodontitis of 24
A. Acute herpetic stomatitis B. Chronic granulating periodontitis of 24
B. Stevens-Johnson syndrome C. Chronic granulomatous periodontitis of 24
C. Erythema multiforme exudativum D. Exacerbation of chronic periodontitis of 24
D. Stomatitis with the background of infecti- E. Chronic gangrenous periodontitis of 24
ous mononucleosis
E. Stomatitis with the background of chi- 167. A 46-year-old man complains of constant
ckenpox losing of a filling in his lower right tooth.
Objectively: in the 16 tooth on the approxi-
163. A 25-year-old man complains of short- mal masticatory surface there is a defect of
term pain in the tooth on the lower ri- crown hard tissues at 1/3. The tooth has no
ght jaw during eating sweet, hot, and cold discoloration, percussion is painless. What
food. Objectively: in the 36 tooth on the construction should be prescribed?
distal surface there is a carious cavity non-
communicating with the dental cavity, dentin A. Inlay
is softened. Probing of the cavity floor is pai- B. Porcelain-fused-to-metal crown
nful, percussion is painless. Electric pulp test C. Semi-crown
is 16 microamperes. Make the final diagnosis: D. 3/4 crown
E. Plastic crown
A. Acute deep caries
B. Acute median caries 168. A 10-year-old boy complains of acute
C. Pulpal hyperemia pain attacks in the area of his upper left teeth.
D. Chronic gangrenous pulpitis The toothache persisted for a night. Objecti-
E. Chronic fibrous periodontitis ve examination revealed a carious cavity on
the masticatory surface of the 26 tooth within
164. A 30-year-old woman came to the dentist parapulpar dentin. Probing is sharply painful
with complaints of uncomfortable sensation at all points of the cavity floor. Markedly
of pressure in her upper right tooth, which positive reaction to cold water stimulus is
aggravates in response to hot stimulus, and observed. Select the most likely diagnosis:
foul smell from the mouth. Objectively: there
is a deep carious cavity in the 17 tooth, whi-
ch communicates with the tooth cavity. Deep
Krok 2 Stomatology (англомовний варiант, iноземнi студенти) 2018 рiк 21

A. Acute diffuse pulpitis patient?


B. Acute serous periodontitis
C. Acute suppurative pulpitis A. Biological approach
D. Acute suppurative periodontitis B. Vital amputation
E. Acute local pulpitis C. Vital extirpation
D. Devital amputation
169. In a state of inebriation a 36-year-old E. Devital extirpation
woman suffered a domestic accident 4 hours
ago. She was delivered to the maxillofacial 173. A 37-year-old woman came to the dentist
surgery unit. On examination she was provisi- with complaints of brief attacks of toothache
onally diagnosed with a deep perforating inci- caused by eating sweets. Objectively there is
sed wound of the lateral surface of her face a shallow carious cavity within enamel. On
with injured perioral area and parotid gland. probing cavity walls and bottom are coarse;
The woung edges are gaping. This gaping is there is no response to thermal stimuli. Make
caused by: the diagnosis:
A. Location of facial muscles A. Acute superficial caries
B. Wound communication with the oral cavity B. Endemic fluorosis
C. Specifics of facial innervation C. Enamel hypoplasia
D. Location of masticatory muscles D. Acute median caries
E. Depth of the wound E. Chronic median caries
170. The maxillofacial surgery unit recei- 174. A 24-year-old patient came to the dentist
ved a patient with complaints of inability complaining of chalky lesions on the front
to close his mouth. This condition occurred teeth. Objectively teeth 13, 12, 11, 21, 22,
when the patient was biting an apple. Objecti- and 23 present with chalky lesions separated
vely there is a frightened expression on the by areas of healthy unchanged enamel. Lesi-
patient’s face, the mouth is open wide, the on surface is coarse; there is no response to
chin is displaced to the left, salivation is thermal stimuli. Childhood years of the pati-
observed. Palpation through the external ent were spent in the area with fluorine level
acoustic meatus detected no movements of of 1.8 mg/L in drinking water. Make the di-
the right articular head. What is the most li- agnosis:
kely diagnosis?
A. Endemic fluorosis
A. Right temporomandibular joint dislocation B. Enamel hypoplasia
B. Fracture of the mandibular process C. Enamel hyperplasia
C. Acute temporomandibular arthritis D. Acute superficial caries
D. Temporomandibular joint pain dysfunction E. Chronic superficial caries
syndrome
E. Bilateral temporomandibular joint di- 175. A 48-year-old patient has addressed a
slocation hospital with complaints of defects in the
paragingival area and slight sensitivity to
171. A 19-year-old girl addressed an oncologi- thermal stimuli. Objectively there are hard
st with complaints of slowly growing tumor- tissue defects that resemble a wedge with
like mass on the tip of her tongue. The mass smooth polished walls on the precervical
was first noticed 5-6 years ago. The pati- vestibular surface of the 23 and 24 teeth.
ent requested no medical help. Objectively: Thermal test is slightly positive. What is the
there is a pale pink round growth with wi- most likely diagnosis?
de pedicle on the tongue apex; the growth
is painless, elastic; there are no changes of A. Cuneiform defect
mucosa surrounding the pedicle. Submandi- B. Enamel necrosis
bular lymph nodes cannot be palpated. What C. Acute deep caries
kind of tumor is it? D. Enamel erosion
E. Endemic fluorosis
A. Papilloma
B. Atheroma 176. Parents of a 7-year-old child addressed a
C. Fibroma hospital with complaints of their child havi-
D. Lipoma ng no permanent teeth in the front area of
E. Keratoma the mandible. Anamnesis states that the fi-
rst deciduous teeth erupted at the age of
172. During carious cavity preparation in a 11 months. Objective clinical examination
20-year-old man, the pulp-chamber floor was revealed the following: appearance is without
accidentally perforated and horn of the pulp changes; milk occlusion; there are physiologi-
was exposed. On the carious cavity floor there cal diastema and tremata; edge-to-edge inci-
is a point-like puncture surrounded with whi- sor contact. What preliminary diagnosis can
te predentin. Pink pulp can be seen through be made according to Kalvelis classification?
the perforation, pulp probing is acutely pai-
nful. What treatment should be given to the
Krok 2 Stomatology (англомовний варiант, iноземнi студенти) 2018 рiк 22

A. Retarded eruption
B. Supernumerary tooth A. Mamlok’s dental splint
C. Adentia B. Removable segmented splint for the front
D. Dystopia teeth
E. Hypoplasia C. Cap splint
D. Splint with embrasure clasps
177. A 5.5-year-old child is undergoi- E. Semicrown splint
ng preventive examination. There are no
complaints from the patient. Objectively: in 182. A woman complains of spontaneous
the precervical area of buccal surface of the attacks of acute pain, with practically no
75 tooth there is an enamel patch with loss intermissions and irradiation into the temple;
of natural glossiness. The enamel surface is cold water slightly mitigates the pain. In tooth
coarse, painless on probing. Tooth percussi- 26 examiantion revealed deep carious cavity
on is painless. Thermodiagnosis is negative. non-communicating with the dental cavity.
Damaged enamel stains with 2% water soluti- Probing of the cavity bottom is acutely pai-
on of methylene blue. Make the diagnosis: nful, the tooth is tender on vertical percussi-
on. Make the provisional diagnosis regarding
A. Acute initial caries tooth 26:
B. Enamel hypoplasia
C. Fluorosis A. Acute suppurative pulpitis
D. Acute superficial caries B. Acute diffuse pulpitis
E. Chronic initial caries C. Pulpal hyperemia
D. Chronic fibrous pulpitis
178. A 68-year-old man has removable parti- E. Chronic hypertrophic pulpitis
al dentures made for him. At the stage of
checking the denture construction, occlusal 183. A 78-year-old man complains of a painful
contact in the lateral areas is cusp-to-cusp sore in his mouth that has been persisting for
and frontal area has sagittal fissure. What mi- 2 months already. The patient is a smoker.
stake was made by the doctor? Objectively on the buccal mucosa on the ri-
ght there is a shallow ulcer up to 1.5 cm in
A. Anterior occlusion is fixed size with lumpy floor and uneven margins.
B. Lateral occlusion is fixed There are yellowish granules on its periphery.
C. Prothetic plane is designed incorrectly Palpation is painful, the lesion is soft. Regi-
D. Height of centric occlusion is understated onal lymph nodes are enlarged, painful, and
E. Height of centric occlusion is overstated matted together. What is the provisional di-
agnosis?
179. An 80-year-old man needs a removable
partial denture for the lower jaw. How should A. Tuberculous ulcer
the denture boder be located relative to the B. Cancerous ulcer
natural teeth? C. Decubitus ulcer
D. Trophic ulcer
A. Covers the crowns by 2/3 of their height E. Hard chancre
B. Covers the crowns by 1/3 of their height
C. Fully covers the crowns 184. A 37-year-old patient has symmetrical
D. Reaches dental cervices face; the mucosa in the area of the 12 tooth
E. Located below dental cervices root apex projection is pale pink; palpation is
painless; the tooth crown is destroyed by 1/3;
180. A 52-year-old woman needs a dental percussion is painless. X-ray: the root canal of
prosthesis. Upon objective examination a the 12 tooth is filled to the apex; granuloma 4
removable partial laminar denture is determi- mm in diameter is located near the root apex.
ned to be optimal denture construction in her Choose the method of surgical treatment:
case. What material should be applied to the
impression tray edges? A. Granuloma removal with root apex resecti-
on
A. Orthocor B. Root hemisection
B. Plaster C. Coronary radicular tooth separation
C. Wax D. Root amputation
D. Silast E. Tooth extraction
E. Stens
185. A 48-year-old man presents with verruci-
181. A 46-year-old man, a teacher, complai- form, dense, gray-white growths on the buccal
ns of lower teeth mobility that impedes the mucosa. The growths protrude above the
process of biting. Objectively the dentition is neighbouring tissues and are surrounded by
uninterrupted, front teeth demonstrate the II keratinized gray-white spots that cannot be
degree of mobility. X-ray shows straight and scraped off. Make the provisional diagnosis:
filled root canals of 32, 31, 41, and 42. What
appliance will stabilize front teeth while retai-
ning their aesthetic appearance?
Krok 2 Stomatology (англомовний варiант, iноземнi студенти) 2018 рiк 23

A. Verrucous leukoplakia response to thermal stimuli. Percussion of


B. Erosive leukoplakia tooth 24 is acutely painful. X-ray shows no
C. Bowen’s disease pathologic changes of periapical tissues in
D. Erythroplasia of Queyrat the area of root apices of 24. What is the most
E. Papillomatosis likely diagnosis?
186. During preventive examination a A. Acute serous periodontitis
patient was diagnosed with precancerous B. Acute suppurative periodontitis
hyperkeratosis of the lower lip vermillion C. Acute suppurative pulpitis
border. What treatment should be prescri- D. Acute diffuse pulpitis
bed? E. Exacerbation of chronic periodontitis
A. Surgical removal of the focus within 190. A 22-year-old man complains of tearing,
healthy tissues throbbing, constant, intensifying pain in the
B. Surgical removal of the focus within healthy tooth on the upper left jaw. The pain has been
tissues + close-focus roentgenotherapy persisting for 4 days. Objectively tooth 26 has
C. No treatment is necessary deep carious cavity non-communicating with
D. Surgical removal of the focus within healthy the dental cavity. Probing is painless. Percussi-
tissues + chemotherapy on is acutely painful. The tooth is mobile.
E. Palliative treatment Mucogingival fold in the area of tooth 26 is
painful on palpation. Make the diagnosis:
187. A 63-year-old man complains of fever
and multiple painful rashes in his oral cavi- A. Acute suppurative periodontitis
ty and on his face and torso. 3-4 days before B. Acute serous periodontitis
the rashes appeared he had noticed a burni- C. Acute suppurative pulpitis
ng sensation followed by sharp shooting D. Exacerbation of chronic periodontitis
pain resembling that which occurs during E. Acute local pulpitis
lumbago. The patient takes cytotoxic drugs
for leukemia treatment. On examination 191. A 22-year-old patient has suffered uni-
there are multiple aphthae located in a row lateral linear fracture in the area of the gonial
on the vermillion border and labial, lingual, angle. Immobilization was provided with full
and buccal mucosa on the right. The aphthae dental brace with loops and intermaxillary
have hyperemic borders, they are not fused elastic expansion. Recovery was uncompli-
together and are sharply painful on palpati- cated. The brace should be removed after:
on. The right side of the face presents with
erythematous spots, vesicles, and erosions. A. 3 weeks
Make the provisional diagnosis: B. 2 weeks
C. 1 week
A. Herpes zoster D. 10 days
B. Toxic allergic dermatostomatitis E. -
C. Erythema multiforme exudativum
D. Secondary syphilis 192. A 27-year-old patient was provisi-
E. Chronic recurrent herpes onally diagnosed with acute suppurative
odontogenic maxillary sinusitis. What radi-
188. A 34-year-old man presents with persisti- ology method would be the most informative
ng dull pain in his tooth, which aggravates on in this case?
biting. One week ago the tooth was treated
for deep caries. Objectively on the masti- A. Computed tomography
catory surface of tooth 36 there is a filling, B. X-ray
percussion is painful, there is a supracontact C. Panoramic radiography
observed in the area of 36. X-ray shows D. Spot-film radiography
unchanged periodontium. What mistake was E. -
made when tooth 36 was filled?
193. A 22-year-old man presents with swollen
A. High filling and hyperemic mucosa of the retromolar
B. Filling without insulation layer area; tooth 38 is covered with hood-shaped
C. Gingival attachment is disturbed gingival flap that discharges pus on palpati-
D. Insulation layer exceeds borders of the on; body temperature is 37.5o C . What urgent
dentinoenamel junction aid should be given to the patient in this case?
E. Filling without medicinal substance sealed
inside A. Gingival flap incision and antibacterial
treatment
189. A 35-year-old man complains of persi- B. Gingival flap excision
sting pain in tooth 24, which intensifies on C. Extraction of tooth 38
biting. Objectively on the distal masticatory D. Antibiotic treatment
surface of tooth 24 there is a deep carious E. Gingival flap incision
cavity filled with food debris. Percussion of
cavity bottom is painless, there is no pain 194. A 43-year-old man came to the maxi-
Krok 2 Stomatology (англомовний варiант, iноземнi студенти) 2018 рiк 24

llofacial surgeon with complaints of aesthetic neoplasm in the left parotid-masticatory regi-
defect. Examination revealed excessive on. Skin over the tumor is without discolorati-
accumulation of adipose tissue in the pati- on. The tumor is painless, but when the head
ent’s neck and upper torso, which resembles bends down the tumor increases in size and
collar with unclear magins; neck mobility assumes bluish coloring. What disease can be
is reduced. Family history shows the same suspected in the child?
symptoms to be present in the patient’s
father. Make the provisional diagnosis: A. Hemangioma
B. Fibroma
A. Madelung’s deformity C. Atheroma
B. Lipoma D. Lymphangioma
C. Neurofibromatosis E. Cyst of the parotid gland
D. Fibroma
E. Lymphangioma 198. An 11-year-old child presents with
protrusion of the upper front teeth, there
195. A 73-year-old man is registered for are tremata and diastema between the chi-
regular check-ups in an oncological clinic ld’s teeth. What apparatus can be used for
after completion of the combined treatment treatment of this pathology?
for oral mucosa cancer stage II (radiation
therapy and surgery). During one of the routi- A. Osadchy
ne check-ups an area of exposed mandibular B. Brukle
bone is detected. There are no inflammatory C. Frankel, type 3
changes of surrounding mucosa. A fistula D. Vasylenko
tract with soft granulation is detected. Mandi- E. Mershon
bular X-ray shows a sequestrum without clear 199. A 40-year-old patient complains of pain
margin between healthy and necrotic bone. in the tragus area, clicking during mouth
What is the most likely provisional diagnosis? opening, stuffed ears. Objectively the face
A. Mandibular osteoradionecrosis is symmetrical, mouth opening path is strai-
B. Acute purulent mandibular osteomyelitis ght. Dentition defect can be estimated as the
C. Posttraumatic mandibular osteomyelitis I class by Kennedy; the 18, 17, 16, 26, 27, 28
D. Relapse of oral mucosa cancer teeth are missing. In this case the load would
E. Chronic mandibular periostitis be the most traumatizing for the following
anatomical structure:
196. A 19-year-old patient came to a
dentofacial clinic with complaints of pain A. Interarticular disk
in the gonial angle on the right, impaired B. Articular capsule
mouth opening and painful chewing. The si- C. Articular head
gns had been persisting for 5 days, emerged D. Distal slope of the articular tubercle
spontaneously and had been aggravating E. Floor of the temporal bone socket
gradually. Mandibular contracture is of the 200. A 38-year-old man complains of sensati-
III degree. On examination of the oral cavity: on of a foreign body on his tongue and
hyperemia, edema of the retromolar space on development of gag reflex during talking.
the right, hood-shaped mucosa from under The signs appeared after the prolonged taki-
which pus is being discharged and 2 tooth ng of antibiotics. Objective examination
tubercles can be detected. X-ray shows obli- detected thickened and pigmented filiform
que medial tooth position. Make the di- papillae enlarged to 2-3 cm in size. Histologi-
agnosis: cal analysis detected papillar hyperplasia and
A. Acute suppurative pericoronitis of the 48 marked keratinization without alteration of
tooth the surrounding tissues. What is the most li-
B. Acute suppurative periostitis from the 48 kely diagnosis?
tooth A. Black hairy tongue
C. Chronic local mandibular osteomyelitis B. Median rhomboid glossitis
D. Mandibular angle fracture C. Fissured tongue
E. Acute submandibular sialadenitis D. Glossitis areata exfoliativa
197. Parents of a 6-year-old child complain E. Geographic tongue
of their child having a gradually enlarging
INSTRUCTIONAL BOOK
Ministry of public health of Ukraine (MPH of Ukraine)
Department of human recources policy, education and science
Testing Board

TEST ITEMS FOR LICENSING EXAMINATION: KROK 2. STOMATOLOGY.

Kyiv. Testing Board.


(English language).

Approved to print 10.04./№32. Paper size 60х84 1/8


Offset paper. Typeface. Times New Roman Cyr. Offset print.
Conditional print pages 24. Accounting publishing pages 28.
Issue. 306 copies.
List of abbreviations

A/G Albumin/globulin ratio HR Heart rate


A-ANON Alcoholics anonymous IDDM Insulin dependent diabetes mellitus
ACT Abdominal computed tomography IFA Immunofluorescence assay
ADP Adenosine diphosphate IHD Ischemic heart disease
ALT Alanin aminotranspherase IU International unit
AMP Adenosine monophosphate LDH Lactate dehydrogenase
AP Action potential MSEC Medical and sanitary expert committee
ARF Acute renal failure NAD Nicotine amide adenine dinucleotide
AST Aspartat aminotranspherase NADPH Nicotine amide adenine dinucleotide
phosphate restored
ATP Adenosine triphosphate NIDDM Non-Insulin dependent diabetes
mellitus
BP Blood pressure PAC Polyunsaturated aromatic
carbohydrates
bpm Beats per minute PAS Periodic acid & shiff reaction
C.I. Color Index pCO2 CO2 partial pressure
CBC Complete blood count pO2 CO2 partial pressure
CHF Chronic heart failure pm Per minute
CT Computer tomography Ps Pulse rate
DIC Disseminated intravascular coagualtion r Roentgen
DCC Doctoral controlling committee RBC Red blood count
DM-2 Non-Insulin dependent diabetes mellitus RDHA Reverse direct hemagglutination assay
DTP Anti diphtheria-tetanus vaccine Rh Rhesus
ECG Electrocardiogram (R)CFT Reiter's complement fixation test
ESR Erythrocyte sedimentation rate RIHA Reverse indirect hemagglutination
assay
FC Function class RNA Ribonucleic acid
FAD Flavin adenine dinucleotide RR Respiratory rate
FADH2 Flavin adenine dinucleotide restored S1 Heart sound 1
FEGDS Fibro-esphago-gastro-duodenoscopy S2 Heart sound 2
FMNH2 Flavin mononucleotide restored TU Tuberculin unit
GIT Gastrointestinal tract U Unit
Gy Gray USI Ultrasound investigation
GMP Guanosine monophosphate V/f Vision field
Hb Hemoglobin WBC White blood count
HbA1c Glycosylated hemoglobin X-ray Roentgenogram
Hct Hematocrit
HIV Human immunodeficiency virus
Sample test questions

Krok 2
Stomatology
()
Терапевтична стоматологiя 2

1. Biopsy material was obtained temperature of 38.9o C , general fatigue,


from the focus of a lesion in the impaired speech, inability to eat. This
retromolar space. Pathohistological findings: condition has been recurring for the last
cellular polymorphism in the stratum 4 years in autumn and spring. There are
spinosum, increased mitotic number, giant vesicles and erosions covered in grayish
multinucleate cells, acanthosis, some cells fibrinous coating on the hyperemic and
in the stratum spinosum exhibit signs of swollen labial and buccal mucosa. Nikolsky’s
parakeratosis, keratin pearls are detected. sign is negative. What is the most likely
These pathohistological findings indicate the diagnosis?
following disease:
A. Erythema multiforme exudativum
A. Bowen disease B. Pemphigus vulgaris
B. Lupus erythematosus C. Acute herpetic stomatitis
C. Leukoplakia D. Nonacantholytic pemphigus
D. Lichen ruber planus, verrucous form E. -
E. Keratoacanthoma
6. A 36-year-old woman complains of dryness
2. During preventive examination a 40-year- and peeling of her lower lip vermillion border.
old man presents with the following changes: Both dryness and peeling have been observed
marginal gingiva is enlarged, torus-shaped, for a month. Application of indifferent
cyanotic, slightly bleeding when touched with ointments was ineffective. Objectively the
a dental probe; there is no pain. Staining the vermillion border of the lower lip is bright
gums with Lugol’s iodine solution results in red, moderately infiltrated, and densely
light-brown coloring of mucosa. Make the covered in white-gray scales, that cannot be
diagnosis: removed without pain and bleeding. The
lesion focus contains concave areas, while
A. Chronic catarrhal gingivitis on the periphery there are patches of dull
B. Acute catarrhal gingivitis epithelium that look like irregular white
C. Exacerbation of chronic catarrhal gingivitis stripes. What is the most likely diagnosis?
D. Chronic hypertrophic gingivitis
E. Generalized periodontitis A. Lupus erythematosus
B. Candidal cheilitis
3. A patient complains of carious cavity in C. Lichen ruber planus
tooth 11. The filling was lost one week ago. D. Leukoplakia
The tooth crown is dark, there is residual E. Exfoliative cheilitis
filling material at the bottom of the carious
cavity. Vertical percussion is painless. X-ray 7. A 35-year-old woman has complaints of
shows an oval area of bone tissue resorption cosmetic defects of the front upper teeth
with clear margins, 0.4х0.3 cm in size. The root crowns. The defects have been aggravating
canal is filled by 2/3 of its length. What is the for the last 10 years. The patient suffers
most likely diagnosis? from unpleasant sensations when brushing
her teeth, and when chemical stimuli are
A. Chronic granulomatous periodontitis applied. Objective examination revealed
B. Chronic fibrous periodontitis defects localized in the enamel of the front
C. Chronic granulating periodontitis upper teeth vestibular surface. The defects are
D. Radicular cyst oval, saucer-shaped, and have clear margins.
E. Exacerbation of chronic periodontitis Response to probing and cold stimuli was
positive. Make the diagnosis:
4. A 30-year-old woman complains of mild
burning sensation in her lower lip and its A. Enamel erosion
dryness. She peels skin scales off with her B. Enamel hypoplasia
teeth. She has been presenting with this C. Cuneiform defect
condition for 10 years. On examination the D. Chemical necrosis of the tooth
skin scales are gray and located on the lip from E. Hyperesthesia of tooth hard tissues
the Klein’s line to the center of the vermillion
border from angle to angle of the mouth. The 8. A 40-year-old man had his root canal of
scales are firmly attached in the center and are the 34 tooth filled due to chronic fibrous
loose on the periphery. Their forcible removal periodontitis. Soon the treated place became
does not result in erosions. What is the most painful. On X-ray the root canal of the 34
likely diagnosis? tooth is filled to the root apex. What tactics
should the dentist choose to manage the pain?
A. Exfoliative cheilitis
B. Lupus erythematosus A. To prescribe physiotherapeutic procedures
C. Meteorological cheilitis B. To rinse with antiseptic mouthwash
D. Allergic contact cheilitis C. To make insicion along the mucogingival
E. Eczematous cheilitis fold
D. To provide conduction anesthesia
5. A 32-year-old patient presents with body E. To provide infiltration anesthesia
Терапевтична стоматологiя 3

9. A 20-year-old man complains of the area of the 41, 42, 32, and 31 teeth up to
spontaneous pain attacks (5-7 minutes) in the 1/3 of the root length. What is the most likely
area of 36 that occur for the last 24 hours. diagnosis?
Objectively in 36 there is a deep carious cavity
on the masticatory surface. Probing is painful A. Localized periodontitis
at one point, response to cold stimulus is B. Generalized periodontitis, initial stage
painful, with an aftereffect that lasts for 5 C. Generalized periodontitis, stage I
minutes. Percussion is painless. X-ray shows D. Catarrhal gingivitis
root canals to be slightly curved, canal lumen E. Parodontosis, stage I
is clearly visible. The tooth is planned to be
used as an abutment for a dental bridge. What 13. A 45-year-old man complains of persisting
is the optimal treatment method in this case? dull pain in 46 that occurs in response to
thermal and chemical stimuli. Examination
A. Vital extirpation of 46 detected in the precervical area of its
B. Biological approach vestibular surace a deep carious cavity filled
C. Vital amputation with soft pigmented dentin. Probing is sharply
D. Devital amputation painful at one point. Cold water stimulus
E. Filling of the carious cavity induces a dull pain that gradually intensifies.
Make the diagnosis:
10. A 34-year-old man complains of acute
spasmodic pain in the region of his upper jaw A. Chronic fibrous pulpitis
on the left that is aggravating when affected B. Acute deep caries
by cold stimuli. Toothache irradiates to the C. Chronic deep caries
ear and temple. He had acute toothache in D. Chronic gangrenous pulpitis
the 27 tooth one year ago, but he did not E. Chronic fibrous periodontitis
consult a dentist. Pain recurred three days ago.
Objectively: the 27 tooth has a carious cavity 14. A 38-year-old man came to the dentist.
communicating with the dental cavity. Probing After history-taking and examination he
of the opened carious cavity is extremely was found to have exacerbated generalized
painful. X-ray picture shows widening of periodontitis of the II degree with periodontal
periodontal fissure at the root apex of the 27 pockets 3-5 mm deep. After the local
tooth. What is the most likely diagnosis? factors were removed and anti-inflammatory
treatment was provided to the patient, it is
A. Exacerbation of chronic fibrous pulpitis necessary to remove the periodontal pockets.
B. Exacerbation of chronic granulating What method should be applied in this case?
periodontitis
C. Exacerbation of chronic fibrous A. Curettage
periodontitis B. Gingivectomy
D. Acute diffuse pulpitis C. Gingivotomy
E. Exacerbation of chronic gangrenous pulpitis D. Flap surgery
E. Vestibuloplasty
11. A 38-year-old woman complains of
burning pain in her lips and angles of her 15. A 38-year-old man, an arc welder, is a
mouth, their dryness. She has an 8-year- chain smoker. He came to the dentist to have
long history of diabetes mellitus. Objectively: a denture made for him. Howerver, in the
the vermillion border is dry, congestively middle of his lower lip against the background
hyperemic, covered in scales of variable of unchanged vermillion border there is a
size. In the angles of the mouth there are gray-white polygonal lesion 6x4 mm with clear
fissures covered in white coating, the skin margins. The lesion is covered with tightly
is macerated. What ointment should be attached small scales and is level with the
prescribed for topical treatment in the given vermillion border (neither protruding nor
case? retracted). Palpation of the lesion focus is
painless, the lesion has no thickened base;
A. Clotrimazol lateral palpation detects thickened surface
B. Interferon of the lesion. Regional lymph nodes are
C. Prednisolone unchanged. Make the provisional diagnosis:
D. Lanolin
E. Erythromycin A. Localized precancerous hyperkeratosis
B. Verrucous precancer
12. A 23-year-old man complains of gum C. Lupus erythematosus
bleeding when he brushes his teeth or eats D. Cancer of the lower lip
solid food. Objectively: the gums of the E. Lichen ruber planus, hyperkeratotic form
front lower jaw are hyperemic, swollen and
bleeding when palpated. Oral and gingival 16. A 53-year-old patient complains of an
mucosa in other areas are not affected. The ulcer on the lateral surface of the tongue. The
occlusion is deep. The teeth are firm, except ulcer appeared 6 months ago in the result
for the 41 and 31 (degree 1 mobility). X-ray of a trauma caused by sharp tip of the 37
shows resorption of the alveolar septum in tooth metal crown. A dentist replaced the
Терапевтична стоматологiя 4

crown with the one of better quality and 20. A 28-year-old patient complains of pain
prescribed keratoplastic drugs. Despite these and bleeding of gums in the frontal part of the
measures the ulcer continues to grow. Lately upper jaw on the left. Two years ago, the 22
there has been pain during talking, chewing, tooth was covered with a porcelain-fused-to-
swallowing; sometimes the pain irradiates metal crown. Objectively: interdental papilla
to the pharynx. Objectively: on the lateral between the 21 and 22 tooth is hypertrophied,
surface of the tongue there is a painful ulcer markedly hyperemic, overlaps the crown of
with uneven raised dense margins and lumpy the 22 by 1/3 of its height, bleeds when
floor covered with grayish necrotic coating. touched. Periodontal pocket between the 21
What diagnosis is most likely? and 22 tooth is 4 mm deep. Artificial crown is
placed on the gingival margin. X-ray reveals
A. Cancer of the tongue lateral surface resorption of the interalveolar septa between
B. Trophic ulcer the 21 and 22 tooth by 1/3 of their height.
C. Traumatic ulcer Specify a priority action in the treatment of
D. Vincent’s necrotizing ulcerative stomatitis this patient:
E. Tuberculous ulcer
A. Removal of the artificial crown
17. A 20-year-old young woman complains B. Gingivectomy
of pain and marked bleeding in her gums C. Anti-inflammatory therapy
when she brushes her teeth and eats even D. Curettage of the periodontal pocket
soft food. Objectively her gingival mucosa is E. Sclerotherapy
swollen, hyperemic and bleeds even on the
slightest touch. The teeth are covered with 21. A 45-year-old patient complains of
moderate amount of soft dental plaque. What bleeding gums and bad breath. After
toothpaste should be recommended to this examination the patient was provisionally
patient as a part of complex treatment of her diagnosed with chronic generalized
condition? periodontitis of the II degree. To make the
final diagnosis, it is necessary to:
A. Herbal toothpaste
B. Mineral-rich toothpaste A. Perform an X-ray
C. Fluoride toothpaste B. Measure the depth of the periodontal
D. Antifungal toothpaste pockets
E. Gel toothpaste with microelements C. Determine teeth mobility
D. Perform Kulazhenko test (capillary
18. A 50-year-old man complains of gum resistance test)
discoloration, pain when eating spicy food, E. Perform Kotzshke test (periodontal pocket
weakness, and irritability. He notes that
periodically spots and vesicles appear on his suppuration test)
skin. For a long time he has been working 22. A 67-year-old woman complains of an
in the galvanic workshop of a factory. On ulcer on her oral mucosa and pain when
examination the patient is pale, he has a eating. She has a history of decompensated
gray-black border on his gums, and on his valvular heart diseases. Examination detected
oral mucosa there are single erosions that are in the posterior portion of her left buccal
painful on palpation. What is the most likely region an irregularly shaped ulcer with white-
provisional diagnosis? gray coating. Surrounding mucosa is not
A. Lead salt poisoning inflamed. Make the provisional diagnosis:
B. Necrotizing ulcerative stomatitis (Vincent A. Trophic ulcer
stomatitis) B. Necrotizing ulcerative stomatitis
C. Mercury salt poisoning C. Cancerous ulcer
D. Acute leukemia D. Decubitus ulcer
E. Erythema multiforme exudativum E. Tuberculous ulcer
19. A 32-year-old man was diagnosed with 23. A 53-year-old man complains of
ulcerative gingivitis. After processing the increased teeth sensitivity to chemical stimuli.
patient’s oral cavity with an antiseptic, the Objectively: the gums are pale pink, roots are
doctor applied anesthesia and removed dental exposed by 1/3 of their length. Small amount
plaque and necrotic coating. Final part of the of dental deposit is observed. The 15, 14, and
medical appointment is the application of 24 present with cuneiform defects. Probing of
gingival dressing. What etiotropic drug should the exposed cervices and defects is painful.
this dressing contain? What is the most likely diagnosis?
A. Metronidazole A. Periodontosis, I degree
B. Contrykal (Aprotinin) B. Catarrhal gingivitis
C. Methyluracil C. Periodontitis, II degree
D. Pimafucin (Natamycin) D. Periodontitis, I degree
E. Terrilytin E. Ulcerative gingivitis
Терапевтична стоматологiя 5

24. An 18-year-old young man complains of


a cavity in 27 and pain when eating solid A. Biological approach
food. He has a history of severe pain in 27 B. Vital extirpation
one year ago; he requested no medical help C. Devital amputation
at the time. Objectively on the masticatory D. Devital extirpation
surface of 27 there is a deep carious cavity E. Insulating inlay and permanent filling
that communicates with the pulp chamber.
The cavity is nearly completely filled with 29. A 30-year-old woman addressed a dental
overgrown granulation tissue. Probing of surgeon with complaints of uncomfortable
the granulations leads to mild pain and sensation of pressure in her upper right tooth,
bleeding. Percussion is painless. EPT is 45 which aggravates due to hot stimulus, and foul
microamperes. Make the diagnosis: smell from the mouth. Objectively: there is
a deep carious cavity in the 17 tooth, which
A. Chronic hypertrophic pulpitis penetrates into the tooth cavity. Deep probing
B. Chronic fibrous pulpitis causes severe pain, percussion of the 17 tooth
C. Chronic hypertrophic gingivitis is painful. X-ray: there is slight widening of
D. Chronic fibrous periodontitis the periodontal fissure near the root apex.
E. Chronic granulating periodontitis Electric pulp test - 70 microamperes. What
final diagnosis can be made?
25. A 25-year-old man complains of a cavity
in his lower left tooth, pain after eating sweet, A. Chronic gangrenous pulpitis
sour, or solid food. Objectively in tooth 16 B. Chronic fibrous pulpitis
there is a deep carious cavity filled with light- C. Acute purulent pulpitis
colored softened dentin. Probing of the cavity D. Chronic fibrous periodontitis
floor is painful, pain stimulus is painful, but E. Exacerbation of chronic fibrous
cold abates immediately after the stimulus is periodontitis
removed. What is the most likely diagnosis?
30. A woman complains of pain response
A. Acute deep caries to thermal stimuli in her lower right jaw.
B. Chronic deep caries The pain quickly abates after the stimulus is
C. Acute focal pulpitis withdrawn. Objectively on the masticatory
D. Chronic fibrous pulpitis surface of the 26 there is a carious cavity with
E. Acute median caries narrow ”entrance” located within the mantle
dentin. The dentin is soft and light-colored.
26. A 50-year-old man complains of painful Percussion of the 26 is painless. Make the
defect that manifests as a crack in his lower diagnosis:
lip. The defect appeared one month ago.
Objectively the lower lip is dry, intactness A. Acute median caries
of the vermillion border of the lower lip is B. Chronic superficial caries
linearly disturbed by a lesion that is located in C. Acute deep caries
the middle of the lip, is covered in dried blood, D. Chronic median caries
and extends to the labial mucosa. What is the E. Chronic deep caries
most likely diagnosis?
31. A 24-year-old patient came to the dentist
A. Chronic lip fissure complaining of chalky lesions on the front
B. Atopic cheilitis teeth. Objectively teeth 11, 21, 31, 41 present
C. Chronic lymphedema of lips with chalky lesions separated by areas of
D. Miescher cheilitis granulomatosa healthy unchanged enamel. Lesion surface is
E. Melkersson-Rosenthal syndrome coarse; there is no response to thermal stimuli.
Childhood years of the patient were spent
27. A group of experts conducts epidemiologic in the area with fluorine level of 1.8 mg/L in
investigation of certain age groups by severity drinking water. Make the diagnosis:
of signs of periodontal damage and their need
for treatment. What index allows to study A. Endemic fluorosis
these criteria? B. Enamel hypoplasia
C. Enamel hyperplasia
A. CPITN (WHO) D. Acute superficial caries
B. PMA (Parma) E. Chronic superficial caries
C. PDI (Ramfjord)
32. A 48-year-old patient has come to
D. OHI-S (Green-Vermillion) the hospital with complaints of defects in
E. PI (Russel) the paragingival area and slight sensitivity
28. During the treatment of acute deep caries to thermal stimuli. Objectively: there are
in 16 in an 18-year-old girl, the pulp chamber hard tissue defects that resemble a wedge
was accidentally perforated. Perforation is with smooth polished walls on precervical
punctate and located in the area of a pulp vestibular surface of the 23 and 24 teeth.
horn. What treatment tactics would be Thermal test is slightly positive. What is the
advisable in this case? most likely diagnosis?
Терапевтична стоматологiя 6

A. 3
A. Cuneiform defect B. 4
B. Enamel necrosis C. 2
C. Acute deep caries D. 1
D. Enamel erosion E. 5
E. Endemic fluorosis
37. A 35-year-old patient undergoes treatment
33. A 34-year-old man presents with persisting of chronic fibrous pulpitis of tooth 25.
dull pain in his tooth, which aggravates on The canal is to be filled using the warm
biting. One week ago the tooth was treated for gutta-percha vertical condensation technique.
deep caries. Objectively on the masticatory What instrument is needed for gutta-percha
surface of tooth 36 there is a filling, percussion condensation?
is painful, there is a supracontact observed
in the area of 36. X-ray shows unchanged A. Plugger
periodontium. What mistake was made when B. Spreader
tooth 36 was being filled? C. Endodontic probe
D. Root needle
A. High filling E. K-file
B. Filling without insulation layer
C. Gingival attachment is disturbed 38. A 27-year-old woman complains of an
D. Insulation layer exceeds borders of the aesthetical defect in the area of her upper
dentinoenamel junction central incisors. She was diagnosed with
E. Filling without medicinal substance sealed chronic deep caries, Black class IV. Aesthetic
inside restoration of teeth 11 and 21 is planned.
What material has optimal combination of
34. A 35-year-old man complains of persisting strength and aesthetical properties?
pain in tooth 24, which intensifies on biting.
Objectively on the distal masticatory surface A. Hybrid composites
of tooth 24 there is a deep carious cavity B. Macrofilled composites
filled with food debris. Percussion of cavity C. Microfilled composites
bottom is painless, there is no pain response D. Compomers
to thermal stimuli. Percussion of tooth 24 is E. Liquid composites
acutely painful. X-ray shows no pathologic
changes of periapical tissues in the area of 39. A 30-year-old man complains of lost
root apices of 24. What is the most likely dental filling on his upper right jaw.
diagnosis? Objectively in tooth 16 there is a deep carious
cavity filled with dense pigmented dentin.
A. Acute serous periodontitis What dentin layer forms as the result of tooth
B. Acute suppurative periodontitis irritation in the course of caries process?
C. Acute suppurative pulpitis
D. Acute diffuse pulpitis A. Tertiary
E. Exacerbation of chronic periodontitis B. Secondary
C. Primary
35. A 38-year-old man complains of sensation D. Hybrid
of a foreign body on his tongue and E. Predentin
development of gag reflex during talking.
The signs appeared after the prolonged 40. A 32-year-old man was diagnosed with
taking of antibiotics. Objective examination chronic fibrous pulpitis of tooth 27. Vital
detected thickened and pigmented filiform extirpation is chosen as the treatment method.
papillae enlarged to 2-3 cm in size. Amide anesthetic is to be used for tuberal and
Histologically papillar hyperplasia and palatal anesthesia. What anesthetic solution
marked keratinization without changes in the needs to be administered in this case?
surrounding tissues were detected. What is A. 3% mepivastesin (mepivacaine)
the most likely diagnosis?
B. 5% anaesthesin (benzocaine)
A. Black hairy tongue C. 10% lidocaine
B. Median rhomboid glossitis D. 2% novocaine (procaine)
C. Fissured tongue E. 2% dicain (tetracaine)
D. Glossitis areata exfoliativa
E. Geographic tongue 41. During preventive examination, a 20-year-
old man was found to have enamel defects
36. A new dental polyclinic is being opened that look like irregularly shaped coarse white
in the city. There are to be 38 doctors spots in the vestibular precervical area of
in the dental surgery department. How teeth 11 and 12. The spots stain with 2%
many department heads should supervise methylene blue solution. Make the diagnosis:
this number of personnel according to the
normatives?
Терапевтична стоматологiя 7

A. Acute initial caries of teeth 11 and 12, Black would be optimal in this case?
class V
B. Acute superficial caries of teeth 11 and 12, A. Professional teeth cleaning and
Black class V remineralization therapy
C. Acute initial caries of teeth 11 and 12, Black B. Professional teeth cleaning and filling of
class III teeth 12, 11, 21, and 22
D. Acute superficial caries of teeth 11 and 12, C. Postpone the treatment until 30 weeks of
Black class III gestation
E. Focal enamel hypoplasia of teeth 11 and 12 D. Remineralization therapy and filling of
teeth 12, 11, 21, and 22
42. An 18-year-old young man complains of E. Professional teeth cleaning,
tooth sensitivity in his lower left jaw when remineralization therapy, and filling of teeth
eating sweet food. Examination shows enamel 12, 11, 21, and 22
defects in the fissures on the masticatory
surface of tooth 37. The enamel there is dull 45. After extraction of tooth 46, a 41-year-
white and fragile when (chips off) probed. old man noticed a dark defect on the distal
Make the diagnosis: contact surface of tooth 45. The defect is
limited to the enamel, has blurred margins
A. Acute superficial caries of tooth 37 and dark brown color. The affected tissue
B. Chronic superficial caries of tooth 37 crumbles when probed. Make the diagnosis:
C. Acute initial caries of tooth 37
D. Chronic initial caries of tooth 37 A. Chronic superficial caries, Black class II
E. Chronic median caries of tooth 37 B. Chronic initial caries, Black class IV
C. Chronic median caries, Black class IV
43. A 28-year-old woman came to the dentist D. Chronic initial caries, Black class II
for oral cavity sanation. She was diagnosed E. Chronic superficial caries, Black class IV
with acute deep caries of tooth 24. She has
a history of seasonal allergic rhinitis and 46. A 25-year-old man was diagnosed with
drug allergy that manifests as a skin rash acute deep caries of tooth 13, Black class V.
and Quincke edema. She does not remember Photopolymer composite material is planned
whether she has been given local anesthesia to be used for aesthetical restoration of tooth
at the dentist’s office previously. What tactics 13. What material should be used as a lining
should the dentist choose regarding the for the floor of the carious cavity in this case?
anesthesia in this case?
A. Calcium-containing paste
A. Referral to the allergologist for consultation B. Dentin paste
and anesthetic allergy testing C. Devitalizing paste
B. Perform a cutaneous anesthetic allergy test D. Resorcin-formalin paste
in the dental chair immediately before giving E. Iodoform paste
anesthesia
C. Perform an anesthetic scratch test in 47. A patient needs endodontic treatment of
the dental chair immediately before giving tooth 21. The canal is being processed with
anesthesia manually operated endodontic instruments.
D. Use amide local anesthetic Name one such instrument made by means
E. Use ether anesthetic of conical spiral threading (turning) of a steel
wire with round cross-section (milling):
44. A 20-year-old pregnant woman with
the term of 22 weeks complains of thermal A. H-file
sensitivity of her upper front teeth. The B. K-file
sensitivity developed one month ago. She was C. K-reamer
diagnosed with acute initial caries of teeth 12, D. Spreader
11, 21, 22, Black class V. Fedorov-Volodkina E. Plugger
hygiene index is 1.8. What treatment tactics
Хiрургiчна стоматологiя 8

1. A 66-year-old woman complains of dull 21; periodontal fissure is visible. Make the
pain in her left parotid-masseteric region provisional diagnosis:
and increasing facial assymetry that she first
noticed 2-3 months ago. Objectively the A. Nasopalatine duct cyst
patient is undernourished, pale, and suffers B. Follicular cyst of the maxilla
from the left-sided paresis of the mimic C. Radicular cyst of the maxilla
muscles. In front of her earlobe, there is D. Maxillary ameloblastoma
a lumpy infiltration without clear margins. E. Giant-cell tumor of the maxilla
The infiltration is 4x5 cm in size. It is fused
to the surrounding tissues and moderately 5. A 28-year-old man complains of pain in the
painful. No saliva could be produced from the infraorbital and parotid region on the left. On
duct of the left parotid gland. Submandibular examination: hemorrhage occurs in the lower
and cervical lymph nodes on the left are eyelid and conjunctiva of the left eye, there
enlarged. Survey X-ray of the mandible shows are signs of crepitation and step deformity of
no changes in the structure of the bone tissue. the eyesocket lower edge. The mouth opens
What is the most likely diagnosis? by 1 cm. Make the diagnosis:

A. Adenocarcinoma A. Zygomatic bone fracture


B. Actinomycosis of the left parotid-masseteric B. Malar arch fracture
region C. Left articular process fracture
C. Herzenberg pseudoparotitis D. Traumatic arthritis of the
D. Mixed tumor of the left parotid gland temporomandibular joint
E. Exacerbated chronic parotitis E. Hematoma of the infraorbital region

2. After extraction of tooth 27 the dental 6. A patient received a chemical burn to the
surgeon suspected that the maxillary face. The burn was caused by sulfuric acid.
sinus was perforated. To clarify this What substance should be used in the wound
provisional diagnosis the doctor desided to processing to neutralize this acid?
conduct an oronasal communication test A. Alkali
(Valsalva maneuver). What manipulations are B. Flowing water
necessary for this test? C. Antiseptic
A. Pinch the nostrils together to occlude the D. Acid
patient’s nose and ask the patient to blow E. Alcohol
through the nose, while observing the socket 7. A 50-year-old man was diagnosed with
of 27 recurrent sialolithiasis with the sialolith
B. Pinch the nostrils together to occlude the located deep within the submandibular
patient’s nose and ask the patient to blow salivary gland. Choose the optimal treatment
through the mouth, while observing the socket tactics:
of 27
C. Ask the patient to blow through the nose, A. Submandibular gland excision
while observing the socket of 27 B. Radiation therapy
D. Ask the patient to inhale through the nose, C. Sclerotherapy
pinch the nostrils together, and exhale through D. Removal of the sialolith while retaining the
the mouth gland
E. Ask the patient to blow through the mouth, E. Conservative pharmacotherapy
while observing the socket of 27
8. An 18-year-old girl needs consultation
3. The 15 tooth must be extracted. The tooth of the dentist regarding a neoplasm that
crown is retained. What instrument should be appeared in the frontal area of her face.
used in this case? On the frontal area there is a pink-red
semicircular spot on her skin 3-4 cm in size,
A. Forceps with S-shaped handles with clear margins. When pressed, the skin in
B. Straight forceps the affected area discolors and becomes pale.
C. Bayonet forceps As soon as the pressing stops, the skin again
D. Left-sided forceps with S-shaped handles assumes its pink-red color. When the patient
E. Right-sided forceps with S-shaped handles bows her head, the spot darkens and enlarges
4. A 48-year-old man complains of a in volume; when the patient lifts her head, the
tumor-like formation on the hard palate. spot gradually returns to its initial pink-red
Examination detected a semi-spherical color. Make the diagnosis:
protrusion with clear margins in the anterior A. Capillary hemangioma
portion of the palate. Teeth 11 and 21 are B. Cavernous hemangioma
intact. X-ray shows a homogeneous focus
of bone tissue rarefaction. The focus is C. Birthmark (nevus)
3.5x2.5 cm and has clear margins. Against D. Lymphangioma
E. Neurofibromatosis
the background of rarefied bone tissue
there are projections of intact teeth 11 and 9. A 30-year-old patient is diagnosed with
Хiрургiчна стоматологiя 9

acute suppurative odontogenic periostitis of A. Chronic osteomyelitis


the upper left jaw originating from tooth 23. B. Acute osteomyelitis
The crown of 23 on the left is destroyed with C. Exacerbation of chronic osteomyelitis
caries by 1/3. Teeth 22 and 24 are intact. Spot- D. Chronic periostitis
film X-ray shows widening of the periodontal E. Actinomycosis
fissure of 23. What treatment would be the
most advisable in this case? 13. A 29-year-old man came to the surgery
department of a dental hospital. He
A. Periosteotomy and pharmacotherapy complains of painful facial swelling on the
followed by treatment of the causative tooth right that appeared 1.5-2 months ago and
B. Extraction of the causative tooth, has been gradually increasing since then.
periosteotomy Objectively mouth opening is unrestricted,
C. Extraction of the causative tooth, the face is asymmetrical due to a swelling in
physiotherapy the right parotid-masseteric region. The skin
D. Extraction of the causative tooth, over the edema is hyperemic and swollen;
pharmacotherapy palpation detects a painful infiltration 3 cm
E. - in size. Massage in this region results in
discharge of clear saliva from the excretory
10. A 45-year-old man presents with facial duct of the right parotid salivary gland. What
asymmetry due to a dense isolated infiltration disease can be suspected in this case?
in his right buccal area; the skin over the
infiltration is cyanotic, thinned out; in the A. Chronic suppurative lymphadenitis of the
center of the infiltration there is a fistula. In right parotid-masseteric region
the oral cavity the crown of 46 is destroyed B. Acute suppurative parotitis on the right
by 2/3, along the mucogingival fold the band C. Furuncle of the right parotid-masseteric
connecting the tooth with the fistula can be region
palpated. Make the diagnosis: D. Chronic interstitial parotitis on the right
E. Submasseteric abscess on the right
A. Migrating facial granuloma
B. Chronic mandibular osteomyelitis 14. A 25-year-old woman made an
C. Cheek furuncle appointment with the dental surgeon for
D. Odontogenic lymphadenitis oral cavity sanation. Objectively the crown
E. Actinomycosis of tooth 37 is destroyed by 2/3. Gingival
mucosa around tooth 37 is without changes.
11. A 28-year-old man has received a hit What anesthesia should the dental surgeon
to the chin. He complains of pain in the give to the patient for the procedure of tooth
right side of his face. Examination shows extraction?
asymmetrical face due to tissue swelling in
the upper right parotid area, mouth opening A. Mandibular and buccal anesthesia
is painful, limited, the jaw is displaced to the B. Intraoral infraorbital nerve block
right, right-sided crossbite is observed. Make C. Tuberal anesthesia
the provisional diagnosis: D. Mandibular anesthesia
E. Mental nerve block
A. Displaced traumatic fracture of the cervix
of the mandibular articular process 15. A 28-year-old man came to the
B. Displaced traumatic unilateral fracture of maxillofacial surgery department. He
the body of the mandible complains of a cutaneous neoplasm in his
C. Displaced traumatic double fracture of the right submandibular region. The neoplasm
body of the mandible appeared approximately 2 years ago. In
D. Displaced traumatic mental fracture of the the process of shaving, the neoplasm
mandible was frequently injured. He was diagnosed
E. Displaced traumatic central fracture of the with papilloma. In the neoplastic area,
mandible examination shows an ulcer against the
background of hyperemic skin. What tactics
12. A 35-year-old man has been hospitalized should the dental surgeon choose?
into a maxillofacial unit with complaints of
mobility of the 38, 37, and 36 teeth and a A. Prevention of malignant transformation of
fistulous tract in the socket of the extracted 35 the papilloma. Removal of the papilloma
tooth. The condition has been persisting for B. Antibacterial and anti-inflammatory
over 3 months. Insertion of a grooved probe therapy, observation
into the fistulous tract palpated exposed C. Removal of the papilloma and postoperative
coarse bone fragment that easily moved under radiation therapy
pressure. X-ray of the lower jaw demonstrates D. Anti-inflammatory therapy followed by
a focus of bone tissue destruction, with a spot removal of the papilloma
of dense bone tissue 0.5х0.3 cm in size. Make E. Anti-inflammatory therapy followed by
the diagnosis: observation
16. The dental surgery department received a
Хiрургiчна стоматологiя 10

women with complaints of swollen right side 20. A man complains of destroyed crown of
of the fase and indisposition. According to the 16 tooth. Objectively the crown portion
her medical history, one week ago she sensed of the 16 tooth is completely destroyed. The
itching in the right side of her face and noticed patient is diagnosed with chronic granulating
there a small dense nodule. She started periodontitis of the 16 tooth and this tooth
to apply various ointments. Swelling was needs to be extracted. What type of anesthesia
gradually increasing, the woman developed should be used for this procedure?
pain and fever. Objectively her condition
is of moderate severity, body temperature A. Tuberal and palatal anesthesia
is 39o C . Her right buccal region is swollen, B. Infraorbital anesthesia
with hyperemic skin; in the center there is an C. Tuberal anesthesia
oval edema elevated above the surrounding D. Infraorbital and tuberal anesthesia
tissues with a black scab visible on its apex. E. Tuberal and incisor anesthesia
Palpation detects a painful infiltration, 3.5
cm in diameter, and enlarged and painful 21. During application of tuberal anaesthesia
submandibular lymph nodes. Mouth opening the patient developed rapidly increasing
is slightly impaired, oral cavity is clean. Make tissue edema and reduced mouth opening.
the diagnosis: What resulted in such a condition?

A. Furuncle A. Vascular trauma


B. Carbuncle B. Muscle trauma during anaesthesia
C. Phlebitis of the facial vein application
D. Acute lymphadenitis C. Nerve trunk trauma
E. Acute lymphangitis D. Intolerance to the anaesthetic
E. Anaphylactic shock
17. A 38-year-old man after a domestic
accident complains of pain and mobility 22. A 38-year-old man needs to have his
of his upper teeth, problems with eating. 12 tooth extracted. What type of anesthesia
Objectively: soft tissues edema. The 11 and 21 should be used in this case?
teeth are displaced towards the palate, mobile A. Infraorbital and incisor
(II degree), painful on percussion. Mucosa B. Infraorbital and palatal
surrounding the affected teeth is hyperemic C. Incisor
and swollen. X-ray demonstrates widened D. Infraorbital, palatal and incisor
periodontal fissure of the 11 and 21. Choose E. Infraorbital
the treatment method:
23. A woman came to the dental surgeon
A. Setting of the teeth and their fixation with a with complaints of teeth mobility. After
flat occlusal splint objective examination and X-ray analysis she
B. Extraction of the 11 and 21 teeth was diagnosed with generalized periodontitis
C. Removal of tooth pulp in the 11 and 21 of the 1-2 degrees of severity. Which teeth
teeth of those affected by periodontitis should be
D. Immobilization with mouthguard extracted?
E. -
A. With degrees 2-3 of tooth mobility
18. A patient was diagnosed with cancer of the B. Intact teeth
right parotid gland, T2N2M0. What treatment C. Teeth with painful percussion
method should be chosen in this case? D. With degree 1 of tooth mobility
A. Combined therapy E. Carious teeth
B. Radiation therapy 24. After the inflammatory process in the
C. Chemotherapy parotid area a woman developed frequent
D. Surgical removal of the neoplasm pain attacks resembling electric current in her
E. Surgical removal of the lymph node face on the right. The attacks last for 15-20
19. A 27-year-old woman complains of general minutes. The most likely diagnosis is:
weakness and pain in her lower jaw during A. Trigeminal neuralgia
swallowing. Two days ago she developed pain B. Tympanic plexus neuralgia
in tooth 38. Mouth opening is impossible. C. Trigeminal neuritis
What anesthesia should be given to the D. Exacerbation of chronic maxillary sinusitis
patient before examination of the oral cavity E. Exacerbation of chronic osteomyelitis
and extraction of 38?
25. A woman complains of a neoplasm in her
A. Berchet-Dubov sublingual area. The neoplasm makes eating
B. Mental and speaking difficult. In the sublingual area
C. Verlotsky examination detects a soft-elastic neoplasm
D. Tuberal 1cm in diameter with smooth surface and
E. Infiltration clear margins. The sign of fluctuation is
noticeable on palpation. Oral mucosa is
Хiрургiчна стоматологiя 11

stretched tightly over the neoplasm and is 29. A 2-year-old child received a dental
blue-tinged and semi-transparent. What is the trauma. Objectively the crowns of 51 and
most likely diagnosis? 61 are shorter than the crowns of adjacent
teeth by 1/3. Mucosa in the area of 51 and
A. Ranula of the sublingual gland 61 is hyperemic and swollen. X-ray shows no
B. Adenolymphoma periodontal fissure in the apical area of the
C. Mixed tumor of the submandibular gland roots of 51 and 61. What treatment tactics
D. Hemangioma of the submandibular region would be optimal in this case?
E. Calculous sialoadenitis
A. Dispensary observation
26. An 18-year-old girl in her childhood B. Extraction of teeth 51, 61
underwent a surgery for complete bilateral C. Reposition of teeth 51, 61
cleft upper lip. Examination detects multiple D. Ligature splinting
scars on the markedly flat upper lip. There E. Reimplantation
is no visible asymmetry, but the upper lip is
slightly deformed. Wide bases of the wings 30. A 48-year-old woman complains of
of her nose are symmetrically displaced low-grade fever and a gradually enlarging
laterally and posteriorly. Her nasal septum ulcer on her gingival mucosa near the
is shortened. The tip of the nose is bifurcated molars; the teeth in the affected area are
and drawn to the upper lip. Her nasal dorsum mobile. Objectively on the gingival mucosa
is arcuate. What changes occurred in the between the lower left molars there are
patient’s jaws? two superficial sharply painful ulcers with
undermining margins. Floor of the ulcers
A. The upper jaw is underdeveloped, the is granulated and covered in yellow-gray
intermaxilla is frontally displaced due to coating. Small tubercles surround the ulcers.
interrupted labial muscle layer Tooth cervices are exposed, pathologic tooth
B. The lower jaw is underdeveloped due to mobility is observed. Regional lymph nodes
reduced masticatory load, the upper jaw is are enlarged, painful, and matted together
flattened into dense clusters. Make the provisional
C. The upper jaw is underdeveloped in its diagnosis:
frontal area, while its lateral areas remain
without pathologic changes A. Tuberculosis
D. Both upper and lower jaws are B. Syphilis
underdeveloped due to reduced masticatory C. Acute aphthous stomatitis
load and flattened in their frontal areas D. Cancerous ulcer
E. Both upper and lower jaws are flattened E. Trophic ulcer
in their lateral areas due to disturbed nasal
breathig 31. A 36-year-old man complains of an ulcer
on his lower left lip. Three weeks ago a smal
27. A man complains of pain in his lower right round red spot appeared on his lip. Eventually
jaw and impaired mouth opening. Objectively it became more dense and noticeable and 2-
on visual examination his face is symmetrical, 3 days ago developed an ulcer in its center,
mouth opening is slightly impaired, trismus while under the lower jaw appeared multiple
of the 1 degree is observed. Mucosa behind painless nodules. Objectively on the mucosa
tooth 47 is swollen, hyperemic, and painful on of the lower left lip there is an oval ulcer,
palpation. Medial cusps of tooth 48 are visible, 1.2 cm in diameter, with smooth clear margins
while the tooth itself has not fully erupted. and bright-red glossy floor. Ulcer edges form a
What provisional diagnosis can be made? cushion that smoothly descends to the bottom
of the ulcer. In the base of the ulcer there is
A. Pericoronitis a dense painless infiltration. Submandibular
B. Exacerbation of chronic granulating lymph nodes are enlarged and painless, the
periodontitis skin over them remains unchanged. This
C. Acute odontogenic osteomyelitis clinical presentation corresponds with the
D. Acute odontogenic periostitis following disease:
E. Abscess of the pterygomandibular space
A. Syphilis (initial period)
28. A 37-year-old woman came to the dentist B. Ulcerative cheilitis
to have her 25 tooth extracted. What type of C. Lip cancer
anesthesia should be used in this case? D. Trophic ulcer
E. Lupus erythematosus
A. Unilateral tuberal, infraorbital, and palatal
anesthesia 32. The maxillofacial surgery unit received a
B. Unilateral infraorbital and palatal patient with complaints of inability to close
anesthesia his mouth. This condition occurred when
C. Unilateral tuberal and palatal anesthesia the patient was biting an apple. Objectively
D. Unilateral infraorbital and incisor there is a frightened expression on the
anesthesia patient’s face, the mouth is open wide, the
E. Unilateral central anesthesia chin is displaced to the left, salivation is
Хiрургiчна стоматологiя 12

observed. Palpation through the external wound was initially bleeding, but by the
acoustic meatus detected no movements of time of examination the bleeding has already
the right articular head. What is the most stopped. Objectively in the area of the right
likely diagnosis? cheek there was a wound 4x1 cm with even
margins that does not penetrate into the oral
A. Right temporomandibular joint dislocation cavity and is filled with clotted blood. What
B. Fracture of the mandibular process sutures should be applied to the wound in this
C. Acute temporomandibular arthritis case?
D. Temporomandibular joint pain dysfunction
syndrome A. Primary blind suture
E. Bilateral temporomandibular joint B. Primary apposition suture
dislocation C. Primary approximation suture
D. Early secondary suture
33. A 37-year-old patient has symmetrical E. Late secondary suture
face; the mucosa in the area of the 12 tooth
root apex projection is pale pink; palpation is 38. A 22-year-old patient has suffered
painless; the tooth crown is destroyed by 1/3; unilateral linear fracture in the area of the
percussion is painless. X-ray: the root canal of gonial angle. Immobilization was provided
the 12 tooth is filled to the apex; granuloma with full dental brace with loops and
4 mm in diameter surrounds the root apex. intermaxillary elastic expansion. Recovery
Choose the method of surgical treatment: was uncomplicated. The brace should be
removed after:
A. Granuloma removal with root apex
resection A. 3 weeks
B. Root hemisection B. 2 weeks
C. Coronary radicular tooth separation C. 1 week
D. Root amputation D. 10 days
E. Tooth extraction E. -
34. A patient is diagnosed with mandibular 39. A 25-year-old man was brought to the
ameloblastoma. What type of surgery is hospital for specialized medical care 48 hours
recommended for this patient? after he received a shrapnel wound of the
maxillofacial area. Examination shows a
A. Mandibular resection at the distance of 1.5 large gaping wound of irregular shape in the
cm from the lesion focus soft facial tissues. Wound edges are infected
B. Tumor curettage within healthy tissue and swollen. What type of surgical wound
C. Tumor cryodestruction treatment should be conducted at this stage?
D. Cystectomy
E. Only conservative treatment A. Late primary
B. Early primary
35. A 48-year-old man presents with C. Secondary
verruciform, dense, gray-white growths on the D. Delayed primary
buccal mucosa. The growths protrude above E. -
the neighbouring tissues and are surrounded
by keratinized gray-white spots that cannot be 40. An 18-year-old girl complains of pain in
scraped off. Make the provisional diagnosis: her parotid regions and general indisposition.
Examination shows bilateral swelling of the
A. Verrucous leukoplakia parotid regions, painful mouth opening. In
B. Erosive leukoplakia the oral cavity, mucosa near the openings
C. Bowen’s disease of parotid salivary glands is hyperemic
D. Erythroplasia of Queyrat and swollen. There are signs of general
E. Papillomatosis intoxication: body temperature of 38o C and
myalgia. Make the provisional diagnosis:
36. During preventive examination a
patient was diagnosed with precancerous A. Acute epidemic parotitis
hyperkeratosis of the lower lip vermillion B. Acute non-epidemic parotitis
border. What treatment should be prescribed? C. Phlegmon of the parotid-masseteric region
D. Acute suppurative lymphadenitis
A. Surgical removal of the focus within healthy E. Herzenberg pseudoparotitis
tissues
B. Surgical removal of the focus within healthy 41. A 29-year-old man is diagnosed with
tissues + close-focus roentgenotherapy medial maxillary fracture. The line of the
C. No treatment is required fracture is symmetrical on both sides. In this
D. Surgical removal of the focus within healthy case sensory perception in the teeth and
tissues + chemotherapy mucosa is likely to be disturbed within the
E. Palliative treatment following interval:
37. A 21-year-old man 3 hours ago received
a knife wound to the right cheek; the
Хiрургiчна стоматологiя 13

A. From tooth 15 to tooth 25 A. Diseases of periodontal tissues


B. From tooth 11 to tooth 21 B. Delayed growth of the jaw
C. From tooth 13 to tooth 23 C. Overcrowding of the front teeth
D. From tooth 18 to tooth 28 D. Parafunction of mimic muscles
E. Sensitivity of the teeth and mucosa will E. Deep occlusion
remain undisturbed
44. To extract tooth 27, the patient was given
42. A 28-year-old woman complains of tuberal and palatal anesthesia. When the
dull pain when biting on tooth 16. Upon needle was being pulled out from the soft
examination she was diagnosed with tissues after the palatal anesthesia, the patient
exacerbation of chronic periodontitis. She is developed bleeding. What should be done to
prescribed extaction of tooth 16. What forceps stop the bleeding in this case?
should be used for tooth extraction in this
case? A. Press the place of injection with a tampon
B. Place sutures
A. S-shaped right forceps C. Intravenous administration of 10% calcium
B. S-shaped left forceps chloride solution
C. Straight forceps D. Intramuscular administration of vicasol
D. Beak-shaped non-crushing forceps (menadione)
E. Beak-shaped curved forceps E. Ligate the external carotid artery
43. A patient is prescribed mandibular 45. How often should a dentist’s office be
vestibuloplasty. What pathology is likely to thoroughly cleaned?
occur if the vestibule of the mouth is too
shallow? A. No less than once a week
B. No less than once a month
C. No less than once a year
D. No less than once every 2 weeks
E. No less than twice a week
Ортопедична стоматологiя 14

1. A removable full denture for the lower jaw teeth misalignment as a part of complex treatment
is being made for a 75-year-old man. Objectively of periodontal disease:
the alveolar process is slightly atrophied. Herbst
tests are performed during fitting of an impression A. Palatal plate with vestibular arch
tray. When lips are stretched forwards the tray B. Bynin appliance
slips off. Where should the tray edge be shortened C. Schwartz appliance
in this case? D. Katz crown
E. Palatal plate with inclined plane
A. From canine to canine on the vestibular side
B. From canine to canine on the lingual side 7. A 39-year-old woman complains of teeth
C. From behind the mandibular tuberosity to the mobility in her lower jaw. Objectively her dental
mylohyoid line formula is as follows: 17 16 15 14 13 12 11 / 21
D. Along the mylohyoid line 22 23 24 25 26 27, 47 46 45 44 43 42 41/31 32
E. In the premolar area on the lingual side 33 34 35 36 37. The teeth are intact, crowns are
tall. Gingival pockets and mobility of the I and
2. A 40-year-old man presents with a medial II degrees are observed in teeth 42 41/31 32. To
defect of the hard palate 2x3 cm in size. Dentition immobilize the mobile teeth, a cap splint with
is intact. What type of obturator would be optimal fixed crowns was made for 43/33. What type of
in this case? stabilization provides this splint?
A. Palatal plate A. Frontal
B. Plate prosthesis with obturator B. Parasagittal
C. Pomerantseva-Urbanska obturator C. Frontal-sagittal
D. Floating obturator D. Sagittal
E. Ilina-Markosian obturator E. Circumferential
3. After a clasp-retained (bugel) maxillary 8. A 35-year-old man suffers from localized
denture is made, it is necessary to assess the periodontitis of the front teeth on his lower
quality of the newly-made construction. Width jaw. Objectively on the upper jaw his dentition
of the denture arch should be: is uninterrupted, while on the lower jaw teeth
48, 47, 46, 45, 35, 36, 37, 38 are missing, other
A. 5-8 mm teeth present with mobility of the II degree,
B. 1.5-2 mm tremata, and diastema. What tactics should a
C. 3-5 mm prosthodontist choose in this case?
D. 10-12 mm
E. 12-15 mm A. Orthodontic treatment followed by splinting
and prosthesis-making
4. A 32-year-old woman needs a denture. After B. Restore the height of the occlusion with a
objective examination the decision was made in partial removable laminar denture
favor of porcelain-fused-to-metal crown. What C. Restore the height of the occlusion with a
material should be used in this case to obtain the clasp-retained (bugel) denture
impression? D. Splinting of the mobile teeth followed by
prosthesis-making
A. Stomaflex E. Temporary splinting of the teeth on the lower
B. Repin jaw
C. Stomalgin
D. Stens 9. What medical establishment provides dental
E. Orthocor and prosthodontic (making of splints, appliances,
5. A 60-year-old patient came to the maxillofacial and dentures) treatment for those who received
inpatient department. He complains of pain, maxillofacial injuries on the battlefield?
bleeding, and chewing problems. He has a history A. Specialized surgical field hospital
of mandibular trauma at the level of the central B. Separate medical battalion
incisors. Objectively the traumatized place is C. Separate medical platoon
swollen, mouth opening is unrestricted. He was D. Evacuation hospital
diagnosed with medial mandibular fracture. E. Civilian medical establishment
Both jaws are edentulous. What splint would be
optimal in this case? 10. One of the stages of making a removable
full denture includes fitting of an impression tray
A. Port gingival splint and obtaining the functional impression. What
B. Limberg gingival splint material is applied to the edges of the impression
C. Rudko appliance tray?
D. Gunning gingival splint
E. Vankevych dentogingival splint A. Orthocor
B. Plaster
6. A 43-year-old woman complains of C. Kromopan
mobility and displacement of her upper D. Thiodent
front teeth. Objectively: dental formula is E. Stomalgin
17 16 15 14 13 12 11 21 22 23 24 25 26 27
47 46 45 44 43 42 41 31 32 33 34 35 36 37 . 11. The 40-year-old woman complains of inability
Teeth 12 11 21 22 are slanted towards to properly masticate due to the loss of the
the vestibular side, diastema and tremata are following lateral teeth: 18, 16, 15, 25, 26, 28,
observed, I-II degree teeth mobility is detected. 38, 35, 36, 44-46, and 48. The rest of her
Select the orthodontic appliance for correction of teeth present with the I-II degree of mobility.
Ортопедична стоматологiя 15

Generalized periodontitis is observed. What What type of denture would be optimal for this
denture construction would be optimal in this patient?
case?
A. Clasp-retained (bugel) dentures with splinting
A. Removable dental splint elements
B. Metal-based denture B. Metal-based dentures
C. Removable laminar denture C. Non-removable full cast dental bridges
D. Fixed dental bridge D. Removable partial laminar denture
E. - E. -
12. A 55-year-old patient requires a denture. 17. An 18-year-old patient with complaint of
Objectively: Kennedy’s I class dentition defect; large diastem has made an appointment with
the 16, 17, 18, 26, 27, and 28 teeth are missing. prosthodontics specialist. Objectively: there is
The patient presents with fixed occlusion. The full lateral displacement of central incisors due
15 and 25 teeth have low crowns with poor to absence of the 12th and 22nd teeth. What
anatomic contours, intact. Clasp-retained (bugel) instrument is the most advisable for moving the
removable partial denture is being made for the central incisors closer together?
patient. What fixation system would be optimal in
this case? A. Korkhaus appliance
B. Vasylenko appliance
A. Telescopic fixation C. Simple cotton ligature
B. Attachments D. Kalvelis appliance
C. Roach clasp (clammer) E. Begg appliance
D. Aker-Roach combined clasp (clammer)
E. Continuous clasp (clammer) 18. A 25-year-old man complains of incorrectly
positioned maxillary left central incisor due to
13. A 54-year-old patient is prescribed full cast trauma sustained 2 months ago. Objectively tooth
porcelain-fused-to-metal dental bridges for the 21 is rotated around its axis into palatal position.
upper and lower jaws. What type of impression What would be the most advisable treatment
material should be chosen in this case? method for correction of this defect?

A. Silicone A. Orthodontic treatment


B. Plaster B. Surgical treatment
C. Thermoplastic C. Instrumental surgical treatment
D. Zinc oxide eugenol D. Splinting followed by prosthetic treatment
E. Alginate E. -

14. A 28-year-old patient complains of aesthetical 19. A 46-year-old man needs a classic 2-stage
defect. Objectively the crowns of 12, 11, 21, 22, surgical installation of implants into the upper
and 23 are destroyed by caries by over 2/3 of their jaw. How long should be the interval between
heights. The patient has orthognathic occlusion. stages 1 and 2?
X-ray shows the root canals of these teeth to be
completely filled; no pathologic changes detected A. 3-6 months
in the periapical tissues. What approach to teeth B. 1 month
restoration would be optimal in this case? C. 1.5 months
D. 2 months
A. Stump inlays and porcelain-fused-to-metal E. 0.5 months
crowns
B. Plastic-faced stump inlays 20. A 47-year-old man has lost his lower left
C. Crown restoration with photopolymer fillings premolars. After that the teeth at the edges of
D. Combined swaged crowns, made in Borodiuk the defect have been gradually sliding inside
technique the defect. At the same time the antagonist
E. Immediate denture with posts teeth started to protrude toward the edentulous
segment of the alveolar bone. In the medical
15. On objective examination a 59-year-old man literature, such clinical presentation is called:
with the edentulous mandible presents with bone
protrusions and mobile areas of the alveolar crest. A. Popov-Godon phenomenon
To ensure proper fixation of the denture and B. Castaigne syndrome
even load distribution the following functional C. Papillon-Lefevre syndrome
impression should be made: D. Kourliandski phenomenon
E. Phenomenon of relative tooth-alveolar
A. Differentiated lengthening
B. Complete anatomical
C. Compression 21. A 57-year-old man presents with habitual
D. Decompression mandibular dislocation. To reduce mouth
E. Combined opening, Yadrova apparatus was made. How long
should the treatment last in this case?
16. A 42-year-old man came to the prosthodontic
clinic. He complains of inability to chew his food A. 3 months
due to partial loss of teeth. Objectively in the B. 6 months
lateral regions teeth 18, 16, 15, 25, 26, 28, 38, 35, C. 9 months
36, 44, 46, 48 are missing. Other teeth present with D. 12 months
I-II degrees of mobility. The patient is diagnosed E. 18 months
with generalized periodontitis of the II degree. 22. A 27-year-old woman complains of recurrent
Ортопедична стоматологiя 16

loss of a tooth filling in the lower right jaw. patient is conscious. What first aid should be
Objectively: in the 46 tooth on the masticatory provided for this man?
approximal surface there is a defect of hard tooth
tissues affecting 1/3 of the tooth crown, no tooth A. Stop the external bleeding, pack the
discoloration; positive, quickly abating reaction wound, apply sterile bandage, provide transport
to cold stimulus is observed. What denture immobilization
construction would be optimal in this case? B. Use a syrette from the personal medical
kit to provide anesthesia with 2% promedol
A. Dental inlay (trimeperidine) solution
B. Combined crown C. Provide primary surgical processing of the
C. Porcelain-fused-to-metal crown wound, place the sutures
D. Plastic crown D. Position the flaps correctly and place the U-
E. Partial crown shaped mattress sutures
E. Thoroughly ligate the vessel in the wound,
23. To make the external prosthesis for a 62-year- position the flaps correctly, place the U-shaped
old man it is necessary to obtain a Hippocrates mattress sutures, apply sterile bandage
facial moulage of this patient. What impression
material should be used? 28. The medical station of a regiment received a
patient with signs of bilateral mandibular fracture.
A. Plaster What is the main task of first aid in this case?
B. Dentafol
C. Stens A. To control shock, bleeding, and asphyxia and to
D. Stomaflex provide transport immobilization
E. Repin B. To check and correct previously applied
bandages
24. A patient needs a removable full laminar C. To administer analgesics and cardiac
denture for the upper jaw. Objectively on the medications
mucosa of the denture bed there are numerous D. To clean the oral cavity from blood clots, tooth
dense papillomas of varying size. What tactics shards, and bone fragments
should the dentist choose? E. To provide symptomatic therapy and care
A. Remove papillomas and make a denture with a 29. A removable partial laminar denture for the
double layered base lined with elastic material upper jaw is being made for the patient. The
B. Reduce the area of the denture base central occlusion is determined and fixed. What
C. Make a 3D-model of the denture base stage is next?
D. Removable dentures are contraindicated in this
case A. Fitting the wax model of the denture in the oral
E. Make a denture base from metal cavity
B. Arrangement of artificial teeth
25. One week ago a patient received removable C. Replacing wax with plastic
dentures for the upper and lower jaws. During a D. Determining the edges of the base
follow-up visit to the dentist the patient complains E. Measuring the interalveolar height
of accidental cheek-biting and pain. Objectively
he presents with hyperemia, edema, ulceration of 30. A porcelain-fused-to-metal crown for tooth 11
the buccal mucosa along the line of teeth closure is being made for the patient. The tooth will be
in the area of the artificial molars. What is the left vital. What measures should be taken during
likely cause of this condition? treatment to prevent pulpitis in this tooth?
A. Cusps of the molars on the upper and lower jaw A. Pharmaceutical crown
are closing in one vertical plane B. Fluorine preparations
B. Due to diabetes mellitus, mucosa is more C. Calcium preparations
susceptible to trauma D. Anti-inflammatory preparations
C. Toxic-allergic reaction to the plastic components E. Physical therapy
of denture base
D. Fixed anterior occlusion 31. A 55-year-old man came to the prosthodontic
E. Incorrectly measured height of the occlusion clinic to have a denture made for him. Tooth 11
is missing in the patient. Two days ago he was
26. A 70-year-old man has edentulous maxilla. released from the inpatient unit after a case of
Objectively maxillary tuberosity and alveolar myocardial infarction. What tactics should the
processes are completely atrophied; palatine vault dentist choose?
is flat, its mucosal layer is moderately pliant. In
this case the patient’s atrophic edentulous maxilla A. Make a temporary removable denture
can be classified as: B. Make a clasp-retained (bugel) removable partial
denture
A. Schroeder class III C. Make a dental bridge with 12 and 21 as
B. Keller class III abutment teeth
C. Schroeder class II D. Temporarily refrain from making a denture
D. Keller class II E. Perform implantation
E. -
32. During his shift a dentist of the prosthodontics
27. In the epicenter of a natural disaster, a man unit has consulted and examined the patient,
with a lacerated wound of soft facial tissues was checked the crowns, installed dental bridges,
found. The wound cuts open the patient’s lower measured centric jaw relation, and made
lip and chin; external bleeding is observed. The necessary corrections to the removable dentures.
Ортопедична стоматологiя 17

What is the normal workload of a dentist per


shift and per month measured in arbitrary units A. Gold or platinum foil
of labor intensity (AU) in a five-day work week B. Silicone materials
and according to the normatives for provision of C. Thermoplastic materials
healthcare to the adults? D. Wax
E. Solid crystal materials
A. 6 AU - 126 AU
B. 5 AU - 105 AU 38. A 35-year-old man needs an adhesive dental
C. 7 AU - 147 AU bridge. Objectively tooth 15 is missing. Abutment
D. 8 AU - 168 AU teeth 14 and 16 are firm. Teeth crowns are of
E. 9 AU - 189 AU medium height. Dental equator is markedly
visible. The patient has orthognathic occlusion.
33. A non-removable porcelain-fused-to-metal What will most likely be an abutment part of this
dental bridge is being made for a 35-year-old adhesive denture?
woman. Two-layer impressions were obtained
using ”Silafex” silicone material. The impression A. Whole or perforated cast overlays
were sent for disinfection. What antiseptic should B. Full cast crowns
be used to process the impressions? C. Combined crowns
D. Stump inlays
A. 0.5% hypochlorite sodium solution E. Equator crowns
B. 3% chloramine solution
C. 70% alcohol solution 39. A 45-year-old man complains of pain
D. 1.5% sodium bicarbonate solution and crepitation in the temporomandibular
E. 0.5% hydrogen peroxide solution joint during the movements of the lower jaw.
Objectively: the face is symmetrical, the mouth
34. A 29-year-old man needs to replace a number opens with slight displacement to the left.
of dentition defects on his upper jaw. The Dentition is intact. On occlusiography there were
defects can be replaced with dental bridges with detected centric and eccentric supracontacts.
aesthetical coating. ”Sinma-M” plastic coating is What treatment methods should be applied in
securely attached to the frame of the full cast the first place?
metal-plastic dental bridges with:
A. Selective teeth shaving
A. Retention spheres B. Mouthguard for muscle relaxation
B. ”Conalor” dental resin C. Appliances that limit mouth opening
C. EDA varnish D. Mouthguards that increase the height of central
D. Sandblasting occlusion
E. Thorough polishing of the frame E. Lower jaw immobilization
35. A 60-year-old man needs a removable partial 40. A 42-year-old woman after examination
denture for the upper jaw. Arrangement of the was diagnosed with closed medial mandibular
artificial teeth on the wax base was checked in the fracture in the area of 31, 41, non-displaced.
oral cavity of the patient. What is the next stage Crown portions of her other teeth are intact.
in the making of a removable partial denture? Additionally she was diagnosed with chronic
generalized parodontitis of moderate severity.
A. Fitting and fixation of the removable partial What type of splint would be the most advisable
dentures in the oral cavity for fracture treatment in this patient?
B. Measurement and fixation of the centric jaw
relation A. Weber dentogingival splint
C. Placing the plaster casts in an articulator B. Tigerstedt splint
D. Final modelling of the denture base C. Tigerstedt splint with anchor hooks
E. Final polishing of the denture D. Vankevych dentogingival splint
E. Port gingival splint
36. A 64-year-old man undergoes fixation of a
removable partial denture for the lower jaw. The 41. A 37-year-old patient complains of an
dentist examined the denture base and noticed aesthetic defect. Objectively: the 13 tooth is
there a clearly visible ”marble pattern”. This net destroyed by 2/3. The tooth is pulpless, the root
of white lines runs through the whole plastic base. canal is filled. How deep should the root canal be
What is the most likely cause of this defect? opened for pivot crown in this patient?
A. Disturbed process of acrylic resin A. 2/3 of the root canal
polymerization B. 1/3 of the root canal
B. Uneven moistening of acrylic resin with a C. 3/4 of the root canal
monomer D. 1/2 of the root canal
C. Disturbed process of denture base polishing E. Full length of the root canal
D. Foreign admixtures in the denture base
E. Fractures in the denture base caused by 42. A 46-year-old man, a teacher, complains of
mechanical damage lower teeth mobility that impedes the process of
biting. Objectively the dentition is uninterrupted,
37. A 26-year-old woman needs dental prosthetics. front teeth demonstrate the II degree of mobility.
Objectively the crown of 16 is destroyed by 1/3. Its X-ray shows straight and filled root canals of 32,
anatomical shape will be restored with a porcelain 31, 41, and 42. What appliance will stabilize front
inlay. Direct method is chosen for inlay-making. teeth while retaining their aesthetic appearance?
What should be used to obtain the impression of
the formed inlay cavity?
Ортопедична стоматологiя 18

A. Mamlok’s dental splint aesthetical defects. Objectively teeth 14, 15, and
B. Removable segmented splint for the front teeth 22 are missing. According to Agapov, the loss of
C. Cap splint masticatory efficiency in this case is:
D. Splint with embrasure clasps
E. Semicrown splint A. 18%
B. 16%
43. A 40-year-old patient complains of pain in C. 9%
the tragus area, clicking during mouth opening, D. 20%
stuffed ears. Objectively: the face is symmetrical, E. 12%
mouth opening path is straight. Dentition defect
can be estimated as the I class by Kennedy; the 46. A 21-year-old man complains of difficult
18, 17, 16, 26, 27, 28 teeth are absent. In this case mouth opening and visible facial deformation
the load would be the most traumatizing for the throughout the last 2 years. He has a history
following anatomical structure: of facial trauma several years ago. Examination
shows the right side of the face to be flattened,
A. Interarticular disk the chin is displaced to the right. Palpation detects
B. Articular capsule diminished excursion of the articular head in the
C. Articular head left lower jaw. Mouth opening is reduced to 1 cm.
D. Distal slope of the articular tubercle Make the preliminary diagnosis:
E. Socket floor of the temporal bone
A. Ankylosis of the left temporomandibular joint
44. A 57-year-old man complains of pain and B. Exacerbated chronic arthritis of the left
creaking in his right temporomandibular joint temporomandibular joint
when eating. In the evening the signs diminish. C. Acute arthritis of the left temporomandibular
The pain is observed for the last 2-3 years. joint
Objectively the face is symmetrical, mouth D. Anterior dislocation of the lower jaw
opening is reduced to 2.5 cm. Molars are missing E. Arthrosis of the left temporomandibular joint
on both jaws. X-ray shows deformed articular
surfaces of the right temporomandibular joint. 47. A 50-year-old woman complains of pain
What is the most likely diagnosis? and creaking in her temporomandibular joint.
Objectively there are multiple premature occlusal
A. Arthrosis of the right temporomandibular joint contacts and localized pathologic tooth wear.
B. Ankylosis of the right temporomandibular joint The teeth are stable, with exposed cervices; the
C. Acute arthritis of the right temporomandibular gingival margin is hyperemic. Selective teeth
joint shaving is planned. How many visits to the dentist
D. Pain dysfunction syndrome of the right are necessary for the full procedure?
temporomandibular joint
E. Contracture of the right temporomandibular A. 3-4 visits at one-week intervals
joint B. 2-3 visits at one-month intervals
C. 1-2 visits within a year
45. A 44-year-old man came to the dental D. A single visit
polyclinic to have a denture made for him. E. 2-3 visits at one-day intervals
He complains of problematic chewing and
Дитяча терапевтична стоматологiя 19

1. A 18-year-old young man complains surfaces of 52, 51, 61, and 62. The cavities
of bleeding and pain in his gums. The are filled with softened pigmented dentin that
disease onset was 4 days ago. Objectively can be easily removed with dental excavator.
the skin is pale, body temperature is 38.5o C . Make the provisional diagnosis:
Submandibular lymph nodes on the left
are enlarged, painful, non-fused with the A. Acute median caries
surrounding tissues. The gingival papillae and B. Chronic deep caries
gingival margin in the area of 33, 34, 35, 36, C. Acute deep caries
and 37 are hyperemic, ulcerated, and covered D. Chronic median caries
in necrotic deposit. Teeth are covered with E. Chronic superficial caries
soft dental plaque. Make the diagnosis:
6. A 6-month-old child was diagnosed with
A. Necrotizing ulcerative gingivitis bilateral bronchopneumonia and prescribed
B. Acute catarrhal gingivitis broad-spectrum antibiotics. The child is
C. Chronic hypertrophic gingivitis formula-fed and presents with maldigestion
D. Chronic catarrhal gingivitis (dyspepsia). What pathologic changes can
E. - occur in the hard dental tissues in this case?
2. A 13.5-year-old girl complains of swollen A. Systemic enamel hypoplasia
gums that bleed when she brushes her teeth. B. Hutchinson teeth
This condition has been observed for the C. Local hypoplasia
last half a year. Objectively gingival mucosa D. Stainton-Capdepont syndrome
surrounding the front teeth of the upper jaw E. Fournier teeth
is swollen and cyanotic. Interdental papillae
are round, dense, and enlarged, they cover 7. A 6-year-old girl took paracetamol to treat
the crowns by 1/3 of their height. Teeth 13 a case of URTI two days ago, which resulted
and 23 are positioned vestibularly. Make the in the development of her present condition.
provisional diagnosis: The disease onset was acute with temperature
increase up to 39.8o C . Objectively there
A. Chronic hypertrophic gingivitis are cockade-shaped maculopapular rashes
B. Chronic catarrhal gingivitis on her face. The vermillion border is
C. Acute catarrhal gingivitis swollen, hyperemic, covered in massive
D. Generalized periodontitis, degree I brown crusts, and presents with bleeding
E. Localized periodontitis, degree I cracks. Conjunctivitis is detected. Swollen
and hyperemic oral mucosa presents with
3. Parents of a 5-year-old girl brought her to numerous erosions covered with fibrinous
the pediatric dentist for oral cavity sanation. incrustations; the erosions are sharply painful
After clinical examination she was diagnosed on palpation. What is the most likely
with chronic deep caries in 54. What filling diagnosis?
material would be optimal for the treatment
of tooth 54 in this case? A. Stevens-Johnson syndrome
B. Erythema multiforme exudativum
A. Glass ionomer cement C. Acute herpetic stomatitis
B. Zinc phosphate cement D. Chronic recurrent aphthous stomatitis
C. Silicate cement E. Pemphigus
D. Composite
E. Silicophosphate cement 8. Decay-missing-filled index of a 6.5-year-
old child is DMF+df=7. Fedorov-Volodkina
4. A 7.5-year-old practically healthy child hygiene index is 1.5. Fissures in the permanent
complains of crown fracture and pain in molars are intact, deep, and open. What
the upper right incisor. Objectively 2/3 of method of caries prevention would be
crown of 11 is absent, the pulp is exposed advisable in this case?
and red; on probing it is acutely painful and
bleeding; tooth percussion is painful. The A. Noninvasive fissure sealing
trauma occurred 2 hours ago. What would B. Invasive fissure sealing
be the optimal treatment method in this case? C. Application of remineralization solution
D. Application of fluoride varnish
A. Vital amputation E. Electrophoresis-aided application of
B. Devital amputation remineralization solution
C. Vital extirpation
D. Devital extirpation 9. A 7-year-old boy was brought to the
E. Biological approach dentist. Objectively his gums are hyperemic
and bleeding, the teeth are mobile and
5. Parents of a 2.5-year-old child complain of covered in plaque, their roots are exposed,
gradual destruction of the upper front teeth periodontal pockets are pathologic and filled
of their child for the last several months. with granulations. The child is registered
Objectively there are carious cavities within for regular check-ups with the pediatrician.
mantle dentin on the contact and vestibular According to the mother, lately the child’s
Дитяча терапевтична стоматологiя 20

condition has been deteriorating. The A. Amine fluoride


boy presents with diabetes insipidus and B. Enzyme
exophthalmos. What provisional diagnosis C. Antiseptic
can be made? D. Salt supplement
E. Herbal extract
A. Hand-Schuller-Christian disease
B. Taratinov disease (eosinophilic granuloma 14. A 6-year-old child for a 3rd day presents
of bone) with body temperature up to 38-39o C and
C. Letterer-Siwe disease painful swallowing. Objectively on the
D. Gaucher disease mucosa of the palatine arches, tonsils and
E. Papillon-Lefevre syndrome a part of the soft palate there are several
erosions up to 2 mm in diameter. The erosions
10. Parents of a 5-year-old child came to the are situated on the hyperemic mucosa and
dentist for sanation of the child’s oral cavity. covered in whitish plaque. Submandibular
Objectively on the masticatory surfaces of lymph nodes are enlarged and painless on
teeth 54, 64, 74, 75, and 85 there are carious palpation. What is the most likely diagnosis?
cavities within the softened mantle dentin.
The child is anxious and misbehaves. What A. Herpetic angina
treatment tactics would be optimal in this B. Diphtheria
case? C. Acute herpetic stomatitis
D. Infectious mononucleosis
A. ART technique E. Erythema multiforme exudativum
B. Impregnation treatment
C. Deep fluoridation 15. An 8-year-old boy complains of a dull pain
D. Preventive filling in his lower right tooth. The pain appears in
E. Invasive sealing response to thermal stimuli and slowly abates
after the stimuli are removed. Objectively on
11. A 10-year-old child complains of pain, the approximal surface of tooth 46 there is a
dryness, enlargement, and peeling of the deep carious cavity. During necrectomy with
lips. Exaination detected hyperemic labial a dental excavator, communication between
mucosa and enlarged lips; when the lips are the carious and dental cavities is detected.
pulled taut, secretion resembling ”dewdrops” Probing of the communication area results
is being produced from minor salivary glands. in sharp pain and bleeding. What method
What is the most likely provisional diagnosis? of pulpitis treatment is recommended in this
case?
A. Cheilitis glandularis
B. Meteorological cheilitis A. Vital amputation
C. Actinic cheilitis B. Vital extirpation
D. Exfoliative cheilitis C. Devital amputation
E. Allergic contact cheilitis D. Devital extirpation
E. Biological approach
12. An 8-year-old girl complains of bleeding
gums. The child has diabetes mellitus. 16. A 12-year-old boy is in the dental
Objectively the interdental papillae between office to complete the treatment of chronic
the teeth of the upper and lower jaw are periodontitis exacerbation in the 36 tooth.
hyperemic, swollen, and cover the crowns by There are no complaints. Objectively:
1/3 of their height. The teeth exhibit mobility occlusive dressing of the 36 tooth is intact;
of the I degree. What investigation technique tooth percussion is painless; there are no
is necessary for diagnosis-making in this case? pathologies of the mucosa surrounding the
36; palpation is painless. What material is the
A. X-ray most advisable for root canal filling in this
B. Rheoparodontography case?
C. Vacuum test
D. PMA index A. Sealer with gutta-percha post
E. Stomatoscopy B. Zinc oxide eugenol paste
C. Resorcinol-formalin paste
13. A 12.5-year-old child came to the dentist D. Calcium-containing paste
for oral cavity sanitation. Objectively caries E. Phosphate cement
progression in this child is severe: decay-
missing-filled index is DMF=2+0+4=6. Green- 17. During planned sanation of a 3-year-old
Vermillion index is 1.7. Gingival mucosa is pale child, a deep carious cavity filled with food
pink. What toothpastes should be prescribed debris and softened dentin is detected in
in this case for oral hygiene? tooth 54. After tooth preparation the pulp of
54 is bleeding, probing is sharply painful. To
treat pulpitis by means of devital amputation,
during the first visit it is necessary to use:
Дитяча терапевтична стоматологiя 21

A. Paraformaldehyde paste the skin of the perioral area is covered in


B. Zinc-eugenol paste scarce vesicles with clear content. Within the
C. Dentin paste oral cavity on the buccal and lingual mucosa
D. Calcium-containing paste there are sharply painful erosions, 2-3 mm
E. Iodoform paste in size, with white coating and hyperemic
crown. The gums are swollen, hyperemic. The
18. A 12-year-old girl complains of submandibular lymph nodes are enlarged,
spontaneous pain in tooth 16 with short painful on palpation. Make the diagnosis:
periods of remission (10-20 minutes) that
developed 4 days ago. The pain increases A. Acute herpetic stomatitis
at night and irradiates to the upper right B. Stevens-Johnson syndrome
jaw. Examination revealed a deep carious C. Erythema multiforme exudativum
cavity in 16, the dentin is soft, the probing D. Stomatitis with the background of infectious
is painful at all points of the cavity floor, mononucleosis
vertical percussion is slightly tender, the pain E. Stomatitis with the background of
increases in response to thermal stimuli. EPT chickenpox
is 25 microamperes. Make the diagnosis:
23. A 10-year-old boy complains of acute pain
A. Acute diffuse pulpitis attacks in the area of his upper left teeth.
B. Chronic concrementous pulpitis The toothache persisted for a night. Objective
C. Acute apical periodontitis examination revealed a carious cavity on the
D. Acute local pulpitis masticatory surface of the 26 tooth within
E. Chronic fibrous pulpitis parapulpar dentin. Probing is sharply painful
at all points of the cavity floor. Markedly
19. During preventive examination a 10-year- positive reaction to cold water stimulus is
old child was found to have matt white spots observed. Select the most likely diagnosis:
on the vestibular surface of the 11 and 21
teeth. The spots are located in the area of A. Acute diffuse pulpitis
tooth cervix. Enamel surface in the affected B. Acute serous periodontitis
area is undamaged and can be stained with C. Acute suppurative pulpitis
methylene blue. Subjectively the child has D. Acute suppurative periodontitis
no complaints. What is the optimal treatment E. -
method in this case?
24. A 15-year-old girl complains of brief pain
A. Remineralization therapy attacks in her teeth due to chemical stimuli.
B. Preparation and filling Objectively: on the contact surfaces of the
C. Impregnation method 11, 21, 22 teeth there are enamel areas matt
D. ART technique white in color, with lost shine, covered in large
E. Teeth sealing amount of dental deposit. Enamel is softened
and can be easily chipped off with excavator.
20. A 14-year-old boy complains of rapid Probing of lesions is painless. Percussion is
wearing-off of tooth crowns. Objectively: painless. No reaction to cold stimuli. Make
tooth crowns are worn-off by 1/3. Enamel the diagnosis:
easily chips off and is pale gray in color. Make
the diagnosis: A. Acute superficial caries
B. Acute median caries
A. Stainton-Capdepont syndrome C. Acute initial caries
B. Dentinogenesis imperfecta D. Chronic initial caries
C. Fluorosis E. Chronic superficial caries
D. Systemic hypoplasia
E. Focal hypoplasia 25. A 12-year-old boy complains of constant
pain in the upper tooth. The pain aggravates
21. Objective examination of a 10-year-old on biting. Objectively in 26 there is a deep
child revealed slight hyperemia, infiltration, carious cavity non-communicating with the
and dryness of the whole surface of the dental cavity. Thermal stimuli and probing
vermillion border. Architectonics of the lips is of the cavity floor are painless. Percussion is
disturbed. Dryness and contracted sensation sharply painful. Mucosa in the area of tooth
are observed in the lips, especially during cold 26 exhibits mild signs of inflammation. X-
seasons. Make the provisional diagnosis: ray of tooth 26 shows no changes. Make the
diagnosis:
A. Meteorological cheilitis
B. Atopic cheilitis A. Acute serous periodontitis
C. Allergic contact cheilitis B. Acute suppurative periodontitis
D. Exfoliative cheilitis C. Exacerbation of chronic periodontitis
E. Cheilitis of microbial origin D. Acute odontogenic periostitis
E. Acute suppurative pulpitis
22. A 2.5-year-old child has fever up to
38.5o C , low appetite, rashes in the oral cavity. 26. A 16-year-old girl complains of constant
The disease onset was 3 days ago. Objectively:
Дитяча терапевтична стоматологiя 22

dull pain in her lower tooth. The pain


aggravates on biting. Objectively in 46 there A. Exacerbation of chronic granulating
is a deep carious cavity communicating with periodontitis
the dental cavity. Comparative percussion is B. Acute serous periodontitis
sharply painful. Thermal stimuli and probing C. Acute suppurative periodontitis
are painless. Mucosa in the area of tooth 46 D. Exacerbation of chronic granulomatous
is markedly hyperemic, swollen, painful on periodontitis
palpation. X-ray shows an irregularly shaped E. Exacerbation of chronic fibrous
focus of bone tissue destruction with blurred periodontitis
margins. Make the diagnosis:
Дитяча хiрургiчна стоматологiя 23

1. A 10-year-old child is referred by tissue swelling in the left submandibular


the orthodontist for extraction of tooth 53. region. Palpation reveals a round formation
Objectively the crown of 53 is retained, the 2x2 cm in size. The formation is mobile,
tooth is immobile. X-ray of tooth 53 shows painful, unattached to the skin. The 74 tooth is
root resorption by less than 1/3. Choose the discolored, percussion is painful. What is the
best instrument for extraction of tooth 53: provisional diagnosis?
A. Straight crown forceps A. Acute serous odontogenic lymphadenitis of
B. Straight elevator the left submandibular region
C. Root bayonet forceps B. Acute serous nonodontogenic
D. Crown forceps with S-shaped handles lymphadenitis of the left submandibular
E. Beak-shaped root forceps region
C. Acute suppurative odontogenic
2. An adolescent complains of reduced and lymphadenitis of the left submandibular
painful mouth opening, difficulties when region
eating, and swelling in the left mandibular D. Phlegmonous adenitis of the right
angle that developed after tooth 37 was submandibular region
extracted 3 days ago. Objectively the face E. Lateral cervical cyst
is asymmetric due to soft tissue swelling in
the area of the left mandibular angle. Mouth 6. A hit to the face has forced the upper
opening is painful and reduced to 2.0 cm. central incisors of a 15-year-old boy to sink
Disturbed occlusion is observed. Palpation of into the jaw to the half of their crown height.
the left mandibular angle is painful, the tissues What treatment tactics should the dentist
are soft, bone crepitus is detected. ”Indirect choose?
load to the chin” symptom is positive in the
area of the left mandibular angle. The socket A. Surgical repositioning of 11 and 21, teeth
of the extracted tooth is packed with iodoform immobilization, endodontic treatment if
gauze. What is the most likely diagnosis? necessary
B. Extraction of 11 and 21, their replacement
A. Left mandibular angle fracture with dentures
B. Mandibular alveolar fracture C. Dynamic observation, endodontic treatment
C. Anterior mandibular fracture of 11 and 21 if necessary
D. Mandibular periostitis on the left D. Transplantation of 11 and 21 into the
E. Odontogenic mandibular osteomyelitis dentition, endodontic treatment
E. Replantation of 11 and 21 into the dentition,
3. A 6-year-old child was referred for ligation
extraction of the temporary lower central
incisors due to changes in the occlusion. The 7. A 7-year-old girl complains of a neoplasm
teeth exhibit the III degree of mobility. What on her lower lip. The neoplasm appeared
type of anesthesia would be optimal in this 3 months ago and has been slowly growing
case? since then. Objectively on the mucosa of the
lower right lip there is a round neoplasm
A. Application anesthesia 0.5-1 cm in diameter. It protrudes from the
B. Conduction anesthesia mucosa, has smooth surface, and its clear
C. Inhalation narcosis bluish content is visible through its walls. On
D. Infiltration anesthesia palpation it is painless and elastic. Make the
E. Intraosseous anesthesia clinical diagnosis:
4. An 8-year-old child was diagnosed with A. Retention cyst of the lower lip
acute odontogenic mandibular periostitis on B. Lymphangioma of the lower lip
the right originating from tooth 74. What C. Fibroma of the lower lip
treatment method would be optimal in this D. Hemangioma of the lower lip
case? E. Papilloma of the lower lip
A. Extraction of 74, periosteotomy, 8. An 11-year-old girl was diagnosed with
pharmacotherapy chronic parenchymatous parotitis. What X-
B. Endodontic treatment of 74, ray sign indicates this disease?
pharmacotherapy
C. Endodontic treatment of 74, periosteotomy A. ”Bunch of grapes” sign
D. Periosteotomy, pharmacotherapy B. ”Leafless tree” sign
E. - C. ”String of pearls” sign
D. Fluctuation sign
5. A 7-year-old child complains of pain and E. Filling defect
swelling in the left submandibular region.
The swelling in this region developed 2 days 9. A 12-year-old boy, due to trauma of the
ago. Objectively: the child is in a satisfactory 44th and 45th teeth area, suffers from the
condition, body temperature is of 37.3o C . following: pathologic displacement of the
The face is asymmetrical due to the soft mandibular alveolar process; rupture of the
Дитяча хiрургiчна стоматологiя 24

alveolar process mucosa. What additional 12. A 6-year-old child complains of pain
examination is necessary to specify the and edema in the upper right jaw, body
diagnosis? temperature up to 37.9oC , and deterioration
of general well-being. Symptom onset was 3
A. X-ray radiography of mandibula in frontal days ago. Objectively the face is asymmetric
and lateral projections due to soft tissue edema of buccal and
B. X-ray radiography of cranium in axillary infraorbital regions on the right. The crown of
projection 54 is destroyed by 1/2, percussion is painful;
C. X-ray radiography of mandibula in frontal the tooth previously had been treated for
and Parma projection complicated caries. On the palatine side of
D. Tomography of mandibula the affected tooth area there is a painful
E. - infiltration with fluctuation in its center; the
tissues over the infiltration are hyperemic.
10. During examination the child presents Make the provisional diagnosis:
with micrognathia and open bite. X-ray shows
no joint space, the right branch of the lower A. Acute suppurative periostitis of the maxilla
jaw immediately continues as the temporal originating from tooth 54
bone. Make the diagnosis: B. Acute serous periostitis of the maxilla
originating from tooth 54
A. Right-sided bony ankylosis of the C. Acute odontogenic osteomyelitis of the
temporomandibular joint maxilla
B. First and second branchial arch syndrome D. Exacerbation of chronic periodontitis of 54
C. Right-sided sclerosing arthrosis of the E. Chronic odontogenic osteomyelitis of the
temporomandibular joint maxilla
D. Right-sided fibrous ankylosis of the
temporomandibular joint 13. A 21-year-old man with facial furuncle
E. Chronic right-sided arthritis of the was brought to the maxillofacial department.
temporomandibular joint What facial localization of furuncles
and carbuncles is often complicated by
11. During examination the child presents trombophlebitis of the angular vein?
with retracted mucosa on the soft palate
and uvula. The child was diagnosed with A. Upper lip and infraorbital area
congenital submucous cleft soft palate. What B. Lower lip and jaw angle
surgical operation is necessary in this case? C. Lower lip and chin
D. Cheek and parotid-masseteric region
A. Veloplasty E. Nose and external canthus of the eye
B. Uranoplasty
C. Cheiloplasty
D. Uranostaphyloplasty
E. Rhinocheiloplasty
Ортодонтiя 25

1. A 10-year-old girl complains of an A. Hotz serial extraction to reduce the dental


aesthetic flaw. The anamnesis states, that she arch
had been sucking her right thumb up to the B. Jaw expansion to provide the space for the
age of 7. Objectively: the face lower third 12 and 21 teeth
is somewhat reduced. The sagittal fissure C. Massage of the 12 and 21 teeth area to
between the upper and lower incisors, is 9 stimulate their eruption
mm wide, class 2 according to the Angle D. Extraction of the 12 and 21 teeth to reduce
classification. Eshler-Bittner test leads to the dental arch
initial temporary improvement of the girl’s E. Filing down of the 11 and 21 approximal
face, followed by renewed deterioration. surfaces to provide the space for the 12 and 22
What clinical malocclusion is the most likely teeth
in this case?
5. What shape does the upper dental arch
A. Maxillary macrognathia and mandibular have in the permanent dentition?
micrognathia
B. Maxillary macrognathia A. Semi-ellipse
C. Mandibular micrognathia B. Semicircle
D. Maxillary prognathism with lateral C. Parabola
compression D. Saddle-shaped
E. Mandibular retrognathia E. Trapezoid

2. During examination of schoolchildren, 6. What prosthodontic appliance has


the orthodontist noticed that some of them mechanical type of action?
present with strained orbicularis oris muscle. A. Expansion plate with Coffin spring
What dental appliance can be used for B. Schonherr vestibular plate
exercising of the orbicularis oris muscle? C. Case obturator
A. Dass activator appliance D. Frankel functional regulator
B. Frankel functional regulator III E. Bynin appliance
C. Andresen-Haupl activator appliance 7. What period of bite formation in a child can
D. Angle appliance be normally characterized by physiological
E. Bruckl appliance tremata and diastemata?
3. After adenotonsillectomia it is necessary A. Preparation for the change of dentition
to break the mouth breathing habit in a 4-
(4.5-6 years)
year-old child. The orthodontist recommends
application of an oral vestibular shield B. Formation of milk occlusion (6 months - 3
(Kerbitz’ vestibular plate). Vestibular shield years)
facilitates training of the following muscle: C. Changing occlusion
D. Permanent occlusion
A. Orbicular muscle E. It is always a sign of pathology
B. Temporal muscle
C. Masseter muscle 8. During preventive examination a 5-year-
D. Lateral pterygoid muscle old child was found to have insufficient
E. Medial pterygoid muscle physiological attrition of cusps of the
deciduous canines. What treatment tactics
4. A child is 8 years old. There are complaints should a doctor choose?
of overcrowded upper incisors. Objectively:
the first molars closure is of Angle’s I class, A. To file down the retained canine cusps
frontal overbite is orthognathic. The 12 and 22 B. Medical examination once a month until
teeth erupt palatinally with space deficiency the incisors are replaced
of 2/3 of the tooth crown. The 11 and 21 C. Medical examination every 6 months until
teeth are 10 mm each in cross-section. The the incisors are replaced
child has inherited father’s facial type with D. Medical examination every 6 months until
prognathism and macrodontia of the central the canines are replaced
incisors. Choose the preventive treatment, E. No medical intervention is necessary
considering this hereditary pathology: 9. The parents of a 3-year-old child came
to the orthodontist. They complain of an
aesthetical defect in the teeth of their child.
The child has a history of tongue sucking
habit. In the front region there is a vertical
fissure 5 mm. What orthodontic appliance is
indicated for the treatment of this pathology?
Ортодонтiя 26

A. Kraus plate 11. To clarify the diagnosis, the orthodontist


B. Friel plate performed Eschler-Bittner clinical diagnostic
C. Schonherr plate test. As the result the facial signs became
D. Kerbitz plate more marked. What jaw abnormality is the
E. Dass activator appliance most likely in this case?
10. A 7-year-old boy during primary A. Maxillary macrognathia
examination was found to have a habit of B. Mandibular micrognathia
sleeping with his fist under the cheek. What C. Mandibular macrognathia
bite anomaly can develop due to this habit? D. Maxillary micrognathia
E. Combined pathology of both jaws
A. Crossbite
B. Deep bite
C. Mesial bite
D. Prognathism
E. Open bite
Ортодонтiя 27

1. A 56-year-old patient suffering from gag reflex in this patient?


exacerbation of schizophrenia has been
hospitalised in an oral in-patient department A. 10% lidocaine solution
with a diagnosis of the lower jaw displaced B. 10% glucose solution
fracture in the area of the 34-35 teeth. What C. 0.1% atropin solution
method of treatment should be prescribed? D. 1% dimedrol (diphenhydramine) solution
E. 0.2% noradrenaline solution
A. Osteosynthesis
B. One arch flat occlusal splint 6. During administration of ultracain solution
C. Dual splint for infiltration anesthesia the condition of
D. Weber’s dental splint a 22-year-old patient sharply deteriorated.
E. Vankevych dental splint The patient became inert, silent, developed
cough attacks, expiratory dyspnea, wheezing.
2. A 43-year-old man came to the admission Viscous slimy sputum is being expectorated.
room. He complains of facial edema and What urgent condition did the patient
enlarged upper lip and tongue. This condition develop?
developed approximately 40 minutes ago,
when he took aspirin. Objectively the patient A. Bronchial asthma attack
is anxious, his skin is of normal color, B. Anaphylactic shock
respiration is partially disturbed. The upper C. Hypertensic crisis
lip, eyelids, cheeks, tongue, and soft palate D. Asphyxia
are swollen. What is the most likely diagnosis? E. Collapse
A. Quincke edema 7. A 45-year-old patient after administration
B. Melkersson-Rosenthal syndrome of local anaesthesia in preparation for oral
C. Lymphedema of the lips surgery has suddenly felt unwell, developed
D. Allergic contact cheilitis increasing edema of laryngeal mucosa and
E. Cheilitis glandularis respiration disorder. The dentist stopped the
manipulations in the oral cavity. What type of
3. A 45-year-old man came to the asphyxia developed in the patient?
prosthodontic department to have a denture
made for him. Before the preparation of the A. Stenotic
hard tissues of the intact teeth, the patient B. Dislocational
was given an infiltration anesthesia with 0.5% C. Valvular
lidocaine solution. Several minutes later he D. Obturative
developed depressed level of consciousness, E. Aspiration
his blood pressure dropped, convulsions
started, and involuntary urination occurred. 8. A victim of a traffic accident was delivered
What emergency condition did the patient into the admission room. The patient is
develop? supine and unconscious. His skin is cyanotic,
respiration is extremely labored, mucosa is
A. Anaphylactic shock pale, blood clots are accumulated in the oral
B. Syncope cavity. The patient is diagnosed with displaced
C. Pain shock bilateral mandibular fracture. What measures
D. Heart failure should be taken to prevent complications in
E. Collapse this case?
4. During intraoral examination the dentist A. Fixation of the lower jaw and tongue with
suspected that the patient has syphilis. What standard Entin’s head-chin strap
should the dentist do in this case? B. Fixation of the tongue to the patient’s collar
C. Tracheostomy and artificial pulmonary
A. Complete the examination and refer the ventilation
patient for necessary tests D. Removal of foreign bodies from the oral
B. Continue the examination and start the cavity
teeth preparation E. Excision of injured mucosal flaps
C. Inform the patient of the suspected
diagnosis and end the visit 9. In the maxillofacial department a 6-
D. Decline to provide dental services for this month-old boy was hospitalized due to
patient bleeding from a wound in the frenulum of
E. Obtain the impressions to study the his tongue. He has a medical history of
diagnostic models frenulotomy the day before. The bleeding
started 6 hours after the surgery. Suturing the
5. A 65-year-old man with fully edentulous wound was ineffective against the bleeding.
upper jaw needs removable full laminar The child was brought to the hematology
denture. To make the denture, anatomical department, where the bleeding was stopped
impressions of the upper and lower jaws are by intravenous administration of coagulation
necessary. The patient exhibits heightened gag factor IX concentrate. Make the diagnosis:
reflex. What drug can be used to suppress the
Ортодонтiя 28

A. Hemophilia B 11. A 22-year-old man is referred for


B. Suppurated tongue wound extraction of tooth 36. In the office of the
C. Willebrand disease dental surgeon before the anestehesia was
D. Iron-deficiency anemia given to him he felt weakness and developed
E. Idiopathic thrombocytopenic purpura tinnitus and visual blackout. Objectively
the face is pale and covered in cold sweat,
10. After a tooth extraction, the patient blood pressure is 110/70 mm Hg. For several
developed a hemorrhage from the socket of seconds the patient was unconscious. What
the extracted tooth. What tactics should the complication occurred during the visit to the
dentist choose in this case? dentist?
A. Tightly pack the tooth socket with a strip of A. Fainting (Syncope)
iodoform gauze B. Epileptic seizure
B. Place sutures into the soft tissues C. Anaphylactic shock
C. Insert a gauze pack soaked in 3% hydrogen D. Angina pectoris
peroxide solution E. Collapse
D. Administer vicasol (menadione) solution
intramuscularly
E. Administer 5-10 mL of 10% calcium
chloride solution intravenously
2021 krok 2 dentistry https://krokology.com

2021 – Krok 2 – Dentistry


Revision 7
Important notes ✍️: This is revision 7 of this booklet. Please click here
to know if revision 7 is the last revision.

Frequently asked
• Q1: All answers here correct?
• A1: There is still no official answer for 2021 full booklet. However, we found
answers for following question in the official website: 21 40 57 61 83 91 102
110 114. Kindly, before rejecting answers, study the question carefully maybe
there is a reason behind the answer, or you might be right, and I will be happy
to correct it , that’s how any content get to the best results; continuous
revision .
• Q2: How can I know the official answers from krok center?
• A3: In case krok center published official answers to any questions, we will
update them immediately even if we don’t agree with them, so, make sure to
practice on Krokology website to get the recent updates.
• Q3: Is there any discussion group for krok 2 dentistry?
• A3: Yes, you are welcome to join the telegram group to ask and share your
useful information.
• Q4: What are the new questions in 2021?
• A4: Questions with relatively new ideas are: 8, 11, 21, 38, 32, 40, 41, 50, 55,
57, 61, 67, 72, 83, 91, 102, 110, 114,130, 132, 133, 142, 148, 149. Please note
that other questions might seem new, but the idea has been present before, just
in a different context. The idea of some new questions is tricky or not clear.
So, it’s okay to have a different opinion of them.

Thank you for understanding and keep practicing


And big thanks to doctor: Ilyas, Alan, Omayma and Mustafa for the revision
2021 krok 2 dentistry https://krokology.com

1. A 6-year-old child was A. V


referred for extraction of the B. I
temporary lower central incisors due C. II
to changes in the dentition. The teeth D. III
have the III degree of mobility. What E. IV
type of anesthesia would be optimal
in this case? 4. A 15-year-old girl complains
A. Conduction anesthesia of a carious cavity in her tooth and
B. Infiltration anesthesia spontaneous pain that quickly passes.
C. Inhalation narcosis The child is somatically healthy.
D. Intraosseous anesthesia Objectively, in tooth 36 there is a
E. Application anesthesia carious cavity within the parapulpar
dentin, the response to a cold
2. A 27-year-old woman stimulus is painful, the pain abates in
complains of an aesthetical defect in 1 2 minutes. Probing of the cavity
the area of her upper central incisors. floor is painful in one spot.
She was diagnosed with chronic deep Percussion is painless. Make the
caries, Black class IV. Aesthetic provisional diagnosis:
restoration of teeth 11 and 21 is A. Acute local pulpitis
planned. What material has optimal B. Acute deep caries
combination of strength and C. Chronic deep caries
aesthetical properties? D. Pulp hyperemia
A. Compomers E. Acute diffuse pulpitis
B. Hybrid composites
C. Microfilled composites 5. Preventive examination of a 7-
D. Macrofilled composites year old child revealed a carious
E. Liquid composites cavity with overhanging enamel
within the limited area of a fissure in
3. A 36-year-old woman needs a tooth 36. The probe catches in the
denture. Objectively, there is a softened dentin. In the other areas,
carious cavity on the mesial and the enamel on the edges of the
masticatory surfaces of tooth 46, the fissures is glossy and dense. A dentist
interdental contact is disturbed. A plans to fill the carious lesion of tooth
dental inlay is to be made for this 36. What tactics should be chosen
woman. According to Black's regarding the intact areas of the
classification of dental caries, this fissure?
cavity is class: A. Invasive sealing
2021 krok 2 dentistry https://krokology.com

B. Processing the tooth with an 8. A 33-year-old man undergoes


antiseptic solution treatment of tooth 15 for acute
C. Non-invasive sealing median caries. What type of
D. Application of fluoride varnishes anesthesia will be the most advisable
E. Application of calcium- in this case?
containing gels A. Application anesthesia
B. Non-inhalation anesthesia
6. A 47-year-old man complains C. Inhalation anesthesia
of frequent recurrent jaw D. Infiltration anesthesia
dislocations. Objectively, the teeth E. Conduction anesthesia
are retained and the molars are
covered with artificial crowns. What 9. A 38-year-old man complains
treatment tactics should be chosen in of a neoplasm 1 cm in size on the
this case? gums in the area of tooth 14. The
A. Physiotherapy neoplasm is bright red, soft, round,
B. Myogymnastics, physiotherapy and bleeds when touched. This
C. Restriction of the mouth opening, clinical presentation corresponds
using the Petrosov appliance with the following disease:
D. Restriction of the mouth opening, A. Angiomatous epulis
using the Schroeder appliance B. Fibrous epulis
E. Immobilization of the jaw with C. Eosinophilic granuloma
teeth ligation D. Fibroma
E. Migratory granuloma
7. A 2-year-old child is being
treated for severe acute herpetic 10. A 56-year-old man has an oval
stomatitis. Currently, it is the period erosion with a deep red smooth
of disease decline. What medicines surface on the vermillion border of
for topical application to the oral the lower lip between the angle of the
mucosa should be added specifically mouth and the midline. The surface
at this stage to the complex of of the erosion has bloody scabs that
medicines being taken? are difficult to remove. Their
A. Keratoplasties removal causes a slight bleeding.
B. Antiseptics Slight trauma of the erosion in a place
C. Anesthetics without scabs causes no bleeding.
D. Proteolytic enzymes What type of lower lip precancer is
E. Antiviral agents it?
A. Lupus erythematosus
2021 krok 2 dentistry https://krokology.com

B. Manganotti's abrasive A. Chronic lymphadenitis of the


precancerous cheilitis right parotid region
C. Circumscribed precancerous B. Mixed tumor of the right parotid
hyperkeratosis of the vermillion gland
border C. Chronic non-epidemic parotitis
D. Bowen's disease D. Adenocarcinoma of the right
E. Actinic cheilitis parotid gland
E. Actinomycosis of the right parotid
11. Preparation of the vital tooth gland
34 for the installation of an all-
ceramic crown is planned for a 38- 13. A 4-year-old child was
year-old man. The man suffers from diagnosed with exacerbation of
ischemic heart disease. What chronic periodontitis of tooth 85. X-
anesthesia is advisable in this case? ray shows destruction of the lamina
A. Torusal dura of the dental follicle of tooth 45,
B. Infiltration a shadow in the bifurcation area of
C. Tuberal tooth 85, and pathologic resorption of
D. Mental 2/3 of the medial root. What
E. Intraligamentary treatment tactics would you choose?
A. To fill the root canals in one visit
12. A 56-year-old man complains and perform periostotomy
of a painful swelling in his right B. To perform periostotomy and
parotid area. The swelling was hemisection of the resorbed root
noticed 5-6 months ago. Objectively, C. To extract tooth 85
he has a right-sided paresis of the D. To stop pain and inflammation,
facial muscles. Palpation reveals a conduct impregnation treatment
moderately painful tuberous tumor of root canals, and keep the tooth
fused with the surrounding tissues. In open until the natural replacement
the center of the tumor, there is an of the primary dentition occurs
area of softening. The submandibular E. To conduct endodontic and
and cervical lymph nodes on the right pharmacological treatment of the
are enlarged and dense. The mouth root canals of tooth 85 and apply
can be opened without restriction. No occlusive dressing
saliva flows out from the duct of the
right parotid gland. What provisional 14. A 14-year-old girl came to a
diagnosis can be made? dentist with complaints of enlarged
interdental papillae in the frontal
2021 krok 2 dentistry https://krokology.com

regions of her upper and lower jaw. D. Conduction (tuberal and palatal)
Examination shows that the E. Application
interdental papillae are hyperemic,
edematous, cover the crowns to 1/2 17. The parents of a 14-year-old
of their height, and bleed when girl brought her to a dentist,
touched. What system in this case has complaining that she has some kind
a pathology that plays the main role of growth on her tongue. It appeared
in the etiology of this disease? approximately 1.5 years ago and has
A. Central nervous system been slowly growing since then.
B. Endocrine system Objectively, on the lateral surface of
C. Cardiovascular system her tongue there is a neoplasm on the
D. Immune system left. It is pink, mobile, painless, hard,
E. Digestive system and spherical. It has clear margins,
wide base, and smooth surface. Make
15. A 71-year-old man came to a the provisional diagnosis:
dental orthopedist to have a denture B. Atheromax
made for him. During examination, C. Hemangiomax
the dentist noticed that the mucosa D. Lymphangiomax
that covers the palate and the alveolar E. Papilloma
processes was worn thin and barcly F. Fibroma
pliant. What Supple class is it?
A. Fourth 18. A man came to a dental
B. Third therapy clinic with complaints of
C. Second isolated transversal furrows on the
D. First crowns of his front and lateral teeth.
The furrows were detected
16. A porcelain-fused-to-metal immediately after the teeth eruption.
dental bridge is being made for a 31- In his childhood, in the age of under
year-old woman. Vital teeth 33, 36, a year, he had frequent acute
and 37 will function as the abutment respiratory viral infections.
teeth. The teeth preparation is Objectively, the crowns of teeth
planned to be done under anesthesia. 16,11, 21, and 26 have a single
What type of anesthesia will be the indentation (a band) in their middle
most effective in this case? within the enamel. The furrows do
A. Infiltration not stain with methylene blue. What
B. Intraligamentary is the most likely diagnosis in this
C. Conduction (torusal) case?
2021 krok 2 dentistry https://krokology.com

A. Erosion of hard dental tissues Objectively, on the vermillion border


B. Fluorosisx of the lower lip there is a red spot
C. Enamel hyperplasia with velvety surface. The lesion is
D. Systemic enamel hypoplasia slightly concave, when compared to
E. Amelogenesis imperfecta the surrounding tissues. It does not
change its color when pressed. Make
19. A 40-year-old man came to a the provisional diagnosis:
dental clinic with complaints of A. Bowen's disease
bleeding,edematous, and hyperemic B. Verrucous precancer
gums and missing lateral teeth on his C. Simple leukoplakia
lower jaw. A complex treatment is D. Hemangioma
planned for this man, along with E. Exfoliative cheilitis
making removable dentures. What
method of functional diagnostics. is 22. A 34-year-old woman
used to assess the condition of the complains of a pain in her lower left
periodontal vessels? jaw. Two months ago, her teeth 35,
A. Rheoparodontography 36, and 37 were extracted.
B. Masticatiography Objectively, her lower left jaw has a
C. Galvanometry cuff like thickening. X-ray shows a
D. D.X-ray sequestrum. What is the most likely
E. Myography diagnosis in this case?
A. Acute serous alveolitis
20. A 40-year-old man has a fresh B. Chronic odontogenic
mental mandibular fracture without osteomyelitis
visible displacement of the C. Mandibular sarcoma
fragments. What dental orthopedic D. Acute purulent periostitis
appliance is recommended in this E. Ameloblastoma
case?
A. Reponating 23. A 28-year-old man has
B. Forming. received a trauma of the mental
C. Fixing region. X-ray shows a bilateral
D. Replacing mental fracture of the lower jaw. In
E. Directing what direction will the displacement
of the lesser fragment occur?
21. A 58-year-old woman A. No displacement occurs
complains of a red spot on her lower B. -
lip that first appeared 4 years ago. C. Upwards and backwards
2021 krok 2 dentistry https://krokology.com

D. Downwards and backwards E. Acute catarrhal gingivitis


E. Downwards and forwards
26. A 37-year-old man complains
24. A 26-year-old man came to a of an unpleasant sensation in his
dental clinic. He was diagnosed with gums on the lower left jaw that
a non-displaced fracture of the developed after he underwent
maxillary alveolar process in the area treatment for dental caries. several
of teeth 14 and 15. The dentition months ago. Examination reveals
remains uninterrupted. All the teeth permanent fillings on the contact
are stable. What splint must be surfaces of teeth 36 and 37. The
applied in this case? edges of the fillings are hanging over
A. Tigerstedt splint with a spreader the interdental papilla. The gum in
bar the area of teeth 36 and 37 is
B. Tigerstedt splint with an inclined hyperemic and edematous. The
plane periodontal pocket is up to 3 mm
C. Vasiliev splint deep. What treatment will be
D. Smooth splint-bracket etiological in this case?
E. Tigerstedt splint with wire loops A. High-quality filling
B. Pharmacological treatment
25. A regular check-up has C. Professional hygiene
detected crowded teeth on the front D. Thorough personal hygiene
lower jaw of a 15-year-old boy. The E. Closed curettage
gums in this area are slightly
edematous and hyperemic with a 27. A 48-year-old man came to a
cyanotic tint. The gingival margin dental surgeon for extraction of the
has a torusal thickening. Dental roots of tooth 37. What instrument
calculus is observed on the teeth. The must be used for this procedure?
Schiller Pisarev test is positive. X-ray A. Beak-shaped crushing forceps
shows that in the front lower jaw the B. Bayonet forceps
contours of the cortical plate are C. Beak-shaped non-crushing
fuzzy on the apices of the interdental forceps
septa, though the cortical plate itself D. Beak-shaped forceps, curved in a
is intact. Make the diagnosis: planc
A. Acute ulcerative gingivitis E. S-shaped forceps
B. Desquamative gingivitis
C. Chronic hypertrophic gingivitis 28. After a trauma, a man has
D. Chronic catarrhal gingivitis developed complaints of displaced
2021 krok 2 dentistry https://krokology.com

tooth 32 and pain that intensifies C. Calcium-containing preparations


when he bites food. Examination D. Temporary crown
shows edema of the lower lip and E. Anti-inflammatory preparations
mucosal hematoma in the area of
tooth 32. Tooth 32 has the I degree of 31. A 22-year-old man came to a
mobility and a slight sagittal dentist with complaints of increased
displacement.X-ray shows a salivation and a sharp pain in the oral
widening of the periodontal fissure. cavity during eating and talking. He
Make the diagnosis: notes the acute onset of the disease
A. Complete dislocation of tooth 32 that occurred after an overexposure
B. Contusion of tooth 32 to cold, high body temperature (39.5
C. Incomplete dislocation of tooth 32 °C), general weakness, and
D. Root fracture of tooth 32 headache. He has a history of chronic
E. Impacted dislocation of tooth 32 tonsillitis and is allergic to certain
medicines. On examination his skin
29. An 11-year-old child is pale. On the dorsal surface of his
undergoes orthodontic treatment. hands there is a bluish-pink rash with
The child is prescribed a dental a hemorrhagic scab in the center. The
appliance with combined action vermilion border of the lips is
(functional direction and mechanical edematous and has hemorrhagic
action). What structural elements are scabs. The regional lymph nodes are
likely to be present in this dental enlarged, painful, and mobile. Make
appliance? the provisional diagnosis:
A. Occlusive onlays, buccal shields A. Drug-induced stomatitis
B. Omega loop, screw B. Erythema multiforme exudativum
C. Occlusive onlays, screw C. Acantholytic pemphigus
D. Inclined plane, tongue shield D. Stevens-Johnson syndrome
E. Occlusive onlays, labial bandages E. Acute herpetic stomatitis

30. A porcelain-fused-to-metal 32. A man presents with clinical


crown for tooth 11 is being made for signs that resemble acute herpetic
the patient. The tooth will be left stomatitis. This diagnosis will be
vital. What measures should be taken confirmed, if cytology detects the
during the treatment to prevent presence of the following cells:
pulpitis in this tooth? A. Multinucleated giant cells
A. Fluorine-containing preparations B. Nicmann-Pick cells
B. Physical therapy C. Atypical mononuclear cells
2021 krok 2 dentistry https://krokology.com

D. Tzanck cells D. 5%anaesthesin(benzocaine)


E. Gaucher cells E. 3%mepivastesin(mepivacaine)

33. A 45-year-old woman 35. A 54-year-old woman


complains of bleeding gums. The complains of a neoplasm on her
bleeding occurs when she brushes her lower lip that appeared 6 month ago.
teeth or eats hard food. Objectively, Twenty days ago, it suddenly started
in the frontal area of her lower jaw growing rapidly. Objectively, on the
the gums are hyperemic, edematous, vermillion border of the lower lip
and bleed when palpated. In the other there is a round red-gray node that
areas, her oral mucosa is normal. The rises 0.5 cm above the underlaying
woman has a deep bite. Her teeth are tissues. It is distinguished from the
stable, except for teeth 41 and 31 (the surrounding tissues and has a
I degree of mobility). X-ray shows funnelform depression in its center,
resorption of the interalveolar septa filled with keratinized masses. On
in the area of teeth 41, 42, 31, and 32 palpation, the nodule is dense,
to 1/3 of the root length. What is the mobile, and painless. What is the
most likely diagnosis? most likely diagnosis?
A. Generalized periodontitis, initial A. Keratoacanthoma
stage B. Verrucous precancer
B. Generalized periodontitis, I C. Verrucous leukoplakia
degree D. Papilloma
C. Periodontosis, I degree E. Manganotti's cheilitis
D. Catarrhal gingivitis
E. Localized periodontitis 36. A 20-year-old pregnant
woman with the term of 22 weeks
34. A 32-year-old man was complains of thermal sensitivity of
diagnosed with chronic fibrous her upper front teeth. The sensitivity
pulpitis of tooth 27. Vital extirpation developed one month ago. She was
is chosen as the treatment method. diagnosed with acute initial caries of
An amide anesthetic is to be used for teeth 12, 11, 21, 22, Black class V.
tuberal and palatal anesthesia. What The Fedorov-Volodkina hygiene
anesthetic solution needs to be index is 1.8. What treatment tactics
administered in this case? would be optimal in this case?
A. 10% lidocaine A. Postpone the treatment until 30
B. 2%dicain(tetracaine) weeks of gestation
C. 2% novocaine (procaine)
2021 krok 2 dentistry https://krokology.com

B. Remineralization therapy and A. Proteolytic enzymes


filling of teeth 12, 11, 21, and 22 B. Antifungal agents
C. Professional teeth cleaning and C. Antibioticsx
filling of teeth 12, 11, 21, and 22 D. Corticosteroids
D. Professional teeth cleaning and E. Non-steroidal anti-inflammatory
remineralization therapy drugs
E. Professional teeth cleaning,
remineralization therapy, and 39. A 62-year-old woman
filling of teeth 12, 11, 21, and 22 complains of intense pain attacks in
the area of her left lower jaw that last
37. A 5-year-old child has all 1-3 minutes. The pain occurs during
temporary teeth in the oral cavity talking, washing her face, and
with tremas between them. The touching the skin. Mandibular X-ray
incisors are in direct contact and their shows no destruction of bone tissue.
cutting edges are worn off. The distal Make the provisional diagnosis:
surfaces of two temporary molars A. Neuralgia of the 3rd branch of the
form a mesio-sagittal step the trigeminal nerve
Zielinsky sign. What period of bitc B. Neuralgia of the 2nd branch of the
formation is it? trigeminal nerve
A. Aging of the temporary bite C. Facial nerve neuritis
B. Early transitional dentition D. Neuritis of the 3rd branch of the
C. Stabilization of the temporary bite trigeminal nerve
D. Late transitional dentition E. Pterygopalatine ganglionitis
E. Formation of the temporary bite
40. A man with hemophilia came
38. A 52-year-old man complains to a dental clinic to have his tooth 36
of unpleasant sensations during extracted. His previous tooth
swallowing and erosions on the oral extraction resulted in a prolonged
mucosa. The disease onset was over bleeding. What tactics should the
a month ago. Objectively, on the doctor choose?
normal mucosa of the soft palate, A. Proper preoperative preparation
cheeks, and gums there are bright red must be provided in the clinic
erosions with the remains of erupted B. To extract the tooth in the clinic
vesicles. The mucosa easily sloughs under the supervision of a
off. The Nikolsky's sign is positive. hematologist
What medicines must be prescribed C. -
for the treatment first?
2021 krok 2 dentistry https://krokology.com

D. To extract the tooth in the of her neck. The neoplasm has a


outpatient department and smooth surface and is approximately
provide the patient with follow-up 3 cm in size. On palpation it is soft
recommendations and dough-like, mobile, and painless.
E. To extract the tooth in the What is the most likely diagnosis in
inpatient department with proper this case?
pre and postoperative preparation A. Lateral cyst of the neck
B. Adenomax
41. A practically healthy 7-year- C. Fibromax
old child complains of a broken D. Lymphomax
crown of the upper right front tooth E. Lipoma
and pain in this tooth. Objectively,
1/3 of the crown of tooth 11 is 44. A 40-year-old man came to a
missing, the pulp is exposed in one dental surgeon for extraction of tooth
spot, acutely painful, red, and bleeds 16. The crown of tooth 16 is retained.
when probed. Percussion of the tooth What forceps should be used to
is slightly painful. The trauma was 2 extract tooth 16 in this case?
hours ago. What is the optimal A. S-shaped forceps
treatment method in this case? B. Bayonet crown forceps
A. Vital extirpation C. S-shaped left-sided forceps
B. Devital extirpation D. Straight forceps
C. Devital amputation E. S-shaped right-sided forceps
D. Biological method
E. Vital amputation 45. The parents of a 7-year-old
Answer here might be controversial. child complain that their child for the
last 2 days had body temperature of
42. What prosthodontic appliance 38.8 ° C and sore throat. Objectively,
has the mechanical type of action? catarrhal stomatitis is observed on the
A. Case's obturator gums, cheeks, lips, and on the floor
B. Expansion plate with Coffin's of the oral cavity. The submandibular
spring lymph nodes are enlarged and
C. Bynin's appliance painful. Bright-red catarrhal
D. Frankel's functional regulator inflammation is present on the soft
E. Schoenherr's vestibular plate palate, tonsils, and palatine arches.
The tongue is edematous, red, and
43. A 63-year-old woman has an dry. The skin of the face is
oval neoplasm on the lateral surface hyperemic, except for the white
2021 krok 2 dentistry https://krokology.com

nasolabial triangle. Make the C. Quincke's edema


provisional diagnosis: D. Abscess
A. Measles E. Emphysema
B. Pertussis
C. Chickenpox 48. A non-removable porcelain-
D. Infectious mononucleosis fused-to metal dental bridge is being
E. Scarlet fever made for a 35 year-old woman. Two-
layer impressions were obtained
46. On the second day after the using the «Silaflex» > silicone
fixation of partial laminar dentures, a material. The impression were sent
man developed complaints of a for disinfection. What antiseptic
burning sensation and an unpleasant should be used to process the
taste in his mouth,when using the impressions?
dentures. Examination revealed A. 1.5% sodium bicarbonate solution
hyperemic mucosa and B. 0.5% hypochlorite sodium
hypersalivation. Prior to making new solution
dentures, he was using the old ones C. 3% chloramine solution
for 5 years and had no such D. 0.5% hydrogen peroxide solution
complaints. What is the likely cause E. 70% alcohol solution
of this phenomenon?
A. Allergic reaction to the 49. A child is 5 years old. There
components ofthe base plastic are complaints of an aesthetical
B. Incorrect usage of the dentures defect. An extraoral examination
C. The presence of a residual shows that the face is symmetrical,
monomer in the denture base but disproportionate because of
D. Poor quality of the base plastic reduced lower third of the face. In the
E. Improper oral hygiene oral cavity, teeth 55, 54, 64, and 65
are missing. What dental device is
47. When receiving necessary for this patient?
tuberalanesthesia intraorally, a man A. Dental bridge
developed a rapidly progressing B. Andresen-Haupl appliance
edema of the left cheek that soon C. Removable partial denture for the
spread to the temporal region. What upper jaw
complication has likely occurred in D. Clasp-retained (bugel) denture
this casc? E. Frankel appliance
A. Ischemia
B. Hematoma
2021 krok 2 dentistry https://krokology.com

50. A 65-year-old man was when palpated. What is the most


diagnosed with a bilateral likely diagnosis in this case?
uncomplicated mandibular fracture A. Acute serous periostitis
in the mental region. The patient uses B. Pericoronitis
removable complete dentures. C. Acute odontogenic osteomyelitis
Choose the long-term splint for this D. Acute serous alveolitis
patient: E. Chronic destructive osteomyelitis
A. Port splint
B. Vasiliev splint 53. A patient is undergoing
C. Weber splint treatment for chronic granulating
D. The patient's own dentures periodontitis of tooth 26. The root
E. Vankevich splint canals were irrigated with an
Answer here is controversial between antiseptic that can penetrate into the
A and D. structures of biofilms, destroy them,
and dissolve organic tissues. What
51. A 37-year-old woman came to antiseptic was used by the doctor?
a dentist to have her tooth 25 A. Hydrogen peroxide solution
extracted. What type of anesthesia B. Chlorhexidine solution
should be used in this case? C. Physiological salinex
A. Unilateral infraorbital and incisor D. D Furacilin(nitrofurazone)
anesthesia solution
B. Unilateral infraorbital and palatal E. Hypochlorite sodium solution
anesthesia
C. Bilateral central anesthesia 54. A 35-year-old woman came to
D. Unilateral tuberal and palatal the dental office complaining of teeth
anesthesia mobility. Objectively, her teeth 42,
E. Unilateral tuberal, infraorbital, 41, 31, 32 have the II degree mobility
and palatal anesthesia and teeth 43, 33. have the I degree
mobility. The tooth crowns are intact.
52. A 44-year-old man complains The woman presents with a deep bite.
of a constant dull pain in the area of X-ray shows resorption of the
his tooth 37 that was extracted two interdental septa by 1/3 of the root
days ago. The pain intensifies during height. What splint construction is
eating. Objectively, the socket of the advisable in this case for temporary
extracted tooth has a gray coating. use?
Mucosa around the socket is A. Cap splint
hyperemic,edematous, and painful B. Made of armored composite
2021 krok 2 dentistry https://krokology.com

C. Consisting of equator crowns D. Making of wax bases with bite


D. Clasp-retained (bugel) denture blocks
E. Consisting of semi-crowns E. Measuring the centric relation
between the jaws
55. A 35-year-old man complains
of missing teeth on his lower jaw. 57. How to separate teeth for
Objectively, teeth 36, 37,and, 38 are orthodontic crowns?
missing. Teeth 34 and 35 are intact. A. With separating discs
What type of dentition restoration B. With a diamond-tipped bore
would be optimal in this case? C. No separation is needed
A. Porcelain-fused-to-metal D. With elastics
cantilever denture with abutment E. With volcanic discs
crowns on teeth 34% and 35
B. Porcelain-fused-to-metal crowns 58. A 25-year-old man complains
with abutment implants in place of a dull pain in the area of his lower
of teeth 36 and 37 right second molar. This tooth was
C. Removable partial laminar treated for pulpitis one day ago. The
denture for / the lower jaw carious cavity has a temporary
D. Adhesive dental bridge filling. Objectively, X ray of tooth 47
E. Clasp (bugel) denture with a shows a perforation in the area of root
continuous clasp (clammer) for bifurcation within the bone tissue.
the lower jaw The root canals are filled and there
are no pathologic changes in the
56. Removable complete dentures periapical tissues. The depth of
are being made for the both jaws of a dentogingival probing in the
68 year-old woman. Objectively, the bifurcation area is normal. What
alveolar processes are slightly treatment method would be optimal
atrophied, the mucosa is moderately in this case?
pliant. Anatomical impressions are A. Crown lengthening
obtained. What is the next stage of B. Tooth extraction
denture-making in this case? C. Perforation closure with MTA
A. Making of individual impression D. Perforation closure with glass-
trays ionomer cement
B. Checking the denture E. Tooth replantation
construction
C. Obtaining the functional 59. A 48-year-old man complains
impressions of a dull pain in the area of his tooth
2021 krok 2 dentistry https://krokology.com

48 that intensifies when biting, 61. During preparation of a 38-


worsening of his general condition, year-old man for denture-making, a
and high body temperature of 38 ° C. perforation of the pulp chamber floor
Objectively, the retromolar mucosa is was detected in the bifurcation area
edematous and hyperemic. The distal of tooth 46. Objectively, tooth 46 is
surface of tooth 48 is covered with a immobile and has no periodontal
mucosal flap, palpation of which pockets. X-ray shows completely
provokes a sharp pain and discharge filledroot canals of tooth 46 without
of purulent exudate. What is the most pathologic changes in their periapical
likely diagnosis in this case? area. In the bifurcation area of tooth
A. Acute purulent periodontitis 46, the apex of the interradicular
B. Phlegmon of the submandibular septum is destroyed. What type of
region procedure is recommended in this
C. Acute purulent pericoronitis case to retain the tooth?
D. - A. Hemisection
E. Acute purulent periostitis B. Replantation
C. Amputation
60. A 32-year-old man has been D. Resection
brought into the maxillofacial E. Bicuspidization (bisection)
department of a clinic. He was
diagnosed with an open displaced 62. A child born with cleft palate
bilateral fracture of the lower jaw in needs an obturator. What is the
the area of the gonial angle. X-ray optimal age for installing a palatal
shows a large diastasis between the obturator in this case?
fragments and muscle interposition. A. 3-4 years
What main treatment technique B. The first days after birth
should be chosen for this patient? C. 1.5 years
A. Bilateral osteosynthesis of the D. 5-6 years
lower jaw E. 1 year
B. Make a Vankevich splint for the
fixation of the lower jaw 63. During intraoral examination
C. Apply a smooth splint-bracket the dentist suspected that the patient
D. Apply a splint with wire loops to has syphilis. What should the dentist
the lower jaw do in this case?
E. Make a Port splint for the fixation A. Continue the examination and
of the lower jaw start the teeth preparation
2021 krok 2 dentistry https://krokology.com

B. Decline to provide dental services branches of the trigeminal nerve and


for this patient alternates with brief painless
C. Inform the patient of the intervals. Objectively, on the medial
suspected diagnosis and end the surface of tooth 25 there is a deep
visit carious cavity within the parapulpar
D. Complete the examination and dentin. Probing of the cavity floor
refer the patient for necessary and percussion are painful. Thermal
tests stimulation of tooth 25 provokes a
E. Obtain the impressions to study pain attack. What is the most likely
the diagnostic dental casts diagnosis?
A. Acute purulent pulpitis.
64. A 48-year-old man came to a B. Trigeminal neurolagia
clinic for oral cavity sanation. He has C. Acute diffuse pulpitis
a history of mild diabetes mellitus. D. Acute purulent periodontitis
Objective examination reveals E. Acute deep caries
whitish papulae against the
background of normal buccal 66. An 8-year-old boy complains
mucosa. The papulae are slightly of an acute pain in his lower left jaw,
raised above the mucosa. They are general weakness, high body
located symmetrically, temperature of 38.6° C, and problems
predominantly in the olar space, and with opening the mouth and
form a lace-like pattern. On the upper swallowing. Objectively, the face is
jaw there are two soldered dental asymmetrical because of edema in
bridges, tooth 37 has an amalgam the submandibular region and in the
filling, tooth 36 is covered with a cast lower third of the left cheek. Mucosa
metal crown. What is the most likely in the area of teeth 73, 74, and 75 is
diagnosis? edematous on the both sides of the
A. Chronic atrophic candidiasis alveolar process. The Pulsief teeth
B. Secondary syphilis are mobile, percussion is positive, the
C. Lichen ruber planus Vincent's sign is positive. What is the
D. Leukoplakia most likely diagnosis in this case?
E. Lupus erythematosus A. Acute odontogenic osteomyelitis
B. Inflammatory infiltration of the
65. For 2 days a man has been left cheek
suffering from spontaneous C. Exacerbation of chronic
nocturnal pain attacks in his tooth 25. periodontitis
The pain irradiates along the
2021 krok 2 dentistry https://krokology.com

D. Odontogenic phlegmon of the muscles are strained. What tactics


submandibular region shoud be chosen by a dental
E. Non-odontogenic acute purulent orthopedist in this case?
lymphadenitis A. Make new dentures
B. Correct the occlusal surface of the
67. A 16-year-old girl came to a lower denture
dentist complaining of darkened C. Correct the occlusal surfaces of
crowns of her teeth 11 and 12. the both dentures
According to the girl, about one year D. Correct the occlusal surface of the
ago she had a sports trauma. upper denture
Objectively, the crowns of teeth 11 E. Make a new denture for the upper
and 12 are dark gray, intact, painless jaw x
on percussion. The mucosa of the
alveolar process is unchanged. X-ray 69. A 9-year-old boy complains of
shows round areas of bone tissue a spontaneous pulsing and irradiating
destruction in the region of the root pain in his lower right teeth that
apices of teeth 11 and 12. These bone slightly abates when cold is applied.
lesions are 0.3-0.4 cm in diameter Objectively, tooth 46 has a deep
and have clear margins. What tactics carious cavity with soft floor, located
would you choose? within the parapulpar dentin. Probing
A. Conservative treatment of teeth of the cavity floor is acutely painful.
11 and 12 Percussion of the tooth is painful.
B. Conservative surgical treatment The boy was diagnosed with acute
of teeth 11 and 12 purulent pulpitis of tooth 46. What
C. Removal of teeth 11 and 12 treatment method should be chosen
D. No treatment is needed in this case?
E. Monitoring the condition of teeth A. A Vital pulp amputation
11 and 12 for six months B. Biological treatment
C. Devital pulp extirpation
68. A 60-year-old woman D. Devital pulp amputation
complains of pain in her ears and in E. Vital pulp extirpation
the area of her temporomandibular
joint and her teeth clacking when she 70. A dentist treats chronic
talks. Her removable complete gangrenous pulpitis of tooth 47 in a
dentures were made two weeks 28-year-old man. What results of
ago.Objectively, her nasolabial folds electric pulp testing confirmed the
are smoothed out, her masticatory
2021 krok 2 dentistry https://krokology.com

diagnosis of chronic gangrenous A. Conical shape, convergence angle


pulpitis? of 15-20 degrees, single-plane
A. Electric pulp testing-110 preparation, circular ledge
microamperes B. Conical shape, convergence angle
B. Electric pulp testing-2-6 of 10-15 degrees, single-plane
microamperes preparation, circular ledge
C. Electric pulp testing -20 C. Cylindrical shape, two-plane
microamperes preparation, circular ledge
D. Electric pulp testing-60 D. Conical shape, convergence angle
microamperes of 5-7 degrees, two-plane
E. Electric pulp testing-10-12 preparation, circular ledge
microamperes E. Cylindrical shape, single-plane
preparation, no ledge
71. A 45-year-old man complains
of painful and loose front teeth on his 73. A clasp (bugel) denture with
lower jaw. Objectively, teeth 41, 42, clammer fixation is being made for a
31, and 32 have the III degree of partially man. edentulous (Kennedy
mobility. Plans are made to remove class I) Impressions were obtained,
these teeth and make a direct denture casts were made, and the centric
for this man. When should a direct relation of the jaws was determined
denture be installed after the teeth and fixed. What laboratory stage of
extraction? the denture-making is the next one?
A. 7 days after the teeth extraction A. Marking the denture frame
B. On the day of the teeth extraction B. Duplication of the cast
C. 1-2 days after the teeth extraction C. Obtaining a refractory cast
D. 14 days after the teeth extraction D. Installation of the gating system
E. 3-4 days after the teeth extraction E. Studying the working cast in a
parallelometer
72. A 40-year-old man complains
of constant decementation of the 74. Removable partial dentures for
porcelain fused-to-metal crown on both jaws are being made for a 60-
tooth 11. The crown was made for year-old man. The patient has a
him one year ago. How must the history of chronic angular stomatitis.
stump of the prepared tooth look like When making his dentures, special
for proper functioning of this type of attention should be paid to the:
dental crown? A. Choice of impression material
2021 krok 2 dentistry https://krokology.com

B. Restoration of the interalveolar from the excretory duct. Make the


height provisional diagnosis:
C. Final processing of the denture A. Herzenberg parotitis
base B. Acute non-epidemic parotitis
D. Choice of material for the denture C. Phlegmon of the parotid-
base masseteric region
E. Oral cavity sanation D. Acute mumps
E. Phlegmon of the submassseteric
75. A 37-year-old woman region
complains of a tumor on her lower lip
on the surface that faces the oral 77. During or immediately after an
cavity. The tumor appeared injection, certain local complications
approximately 2 weeks ago, can develop. What is not one of those
gradually enlarges, and makes lip complications?
movement problematic. Objectively, A. Dermal ischemia
within the lower lip there is a B. Diplopia
circumscribed round elastic painless C. Damage to a blood vessel caused
induration fused to the mucosa. The by the needle
mucosa over the induration is A. Mucosal necrosis
transparent, while around the D. Functional paralysis or paresis of
induratuion it remains unchanged. facial muscles
The regional lymph nodes are not
enlarged. What is the most likely 78. A regular check-up of a 7-
diagnosis in this case? year-old girl detected smooth chalk-
A. Retention cyst of the lip like spots on the vestibular surface of
B. Fibroma of the lip her upper central incisors, closer to
C. Lipoma of the lip their cutting edge. The surface of the
D. Lip abscess spots does not stain with methylene
E. Papilloma of the lip blue. At the age of 4, the child had a
trauma of teeth 51 and 61. What is the
76. On the 7 day after an most likely diagnosis in this case?
abdominal surgery, a 69-year-old A. Local hypoplasia
man has developed pain and swelling B. Systemic hypoplasia
in the area of his left parotid gland. C. Acute superficial caries
His body temperature is 39 ° C, his D. Hyperplasia
mouth is dry. Massage of the salivary E. Caries at the spot stage
gland produces purulent exudate
2021 krok 2 dentistry https://krokology.com

79. After the complex E. Chronic osteomyelitis,


examination of a 44 year-old man, destructive productive form
the dentist made a diagnosis of
generalized periodontitis, II degree, 81. During obtaining a full
exacerbated progression. Lately the anatomical impression of the upper
patient has been noticing a jaw with alginate mass, a 56-year-old
deterioration of his general condition, man developed acute dyspnea, pale
thirst, dry mouth, and itching skin. skin, increased sweating, cyanosis,
What additional examination needs and wheezing respirations. What
to be performed first in this case? urgent condition developed in this
A. Vitamin C levels in blood case?
B. Complete comprehensive blood A. Aspiration asphyxia
count B. Bronchial asthma attack
C. Blood sugar levels C. Pneumothorax
D. Blood iron levels D. Acute bronchitis
E. Immunologic blood test E. Overactive gag reflex

80. An 11-year-old child has a 82. A 26-year-old woman, who


thickening in the frontal region of the works with chemical substances at a
lower jaw that appeared after the factory, came to a dentist with
child fell down. The trauma occurred complaints of teeth discoloration and
one month ago. Objectively, on the extreme sensitivity to cold, sweet,
lower jaw, in the area of the and sour foods in her upper front
mucogingival junction, there is a teeth. Objectively, on the vestibular
circumscribed dense spot with surface of teeth 13, 12, 11, 21, 22,
smooth surface that is slightly painful and 23, at their equator, there are
on palpation. The teeth in the white spots that spread to the cutting
pathologic focus are intact and vital. edges. Probing detects rough and
X-ray detects an osteoporotic focus fragile enamel in the affected areas.
with thickened periosteum in the Vital staining is positive. Make the
frontal region of the lower jaw. What clinical diagnosis:
is the most likely diagnosis in this A. Acute initial caries
case? B. Systemic enamel hypoplasia
A. Chronic granulomatous C. Erosion of hard dental tissues
B. Chronic osteomyelitis, D. Hyperesthesia of hard dental
C. Rarefying periostitis tissues
D. Osteitis deformans E. Necrosis of hard dental tissues
2021 krok 2 dentistry https://krokology.com

83. A 35-year-old woman came to 85. A 42-year-old man came to a


a dental surgeon complaining of a dental surgeon for oral sanation.
slight deformity of the mandibular After application of anesthesia, the
body on the right. patient suddenly developed nausea,
Orthopantomogram shows a round weakness, constricting sensation
homogeneous shadow with clear behind the sternum, and cardiac pain.
margins in the mandibular body. The The patient's skin is pale and covered
density of the shadow is close to that in cold clammy sweat. The patient
of dental tissues. On its periphery, the started vomiting, lost his
neoplasm is surrounded by a consciousness, and developed
transparent zone 1 mm wide. What is seizures. The patient's pupils are
the most likely diagnosis in this case? unresponsive to the light. The pulse
A. Osteoma is thready, blood pressure cannot be
B. Odontogenic fibroma measured. Make the diagnosis:
C. Odontoma A. Anaphylactic shock
D. Ameloblastoma B. Epileptic seizure
E. Mixoma C. Traumatic shock
D. Collapse
84. A 42-year-old man complains E. Unconsciousness
of problems with chewing.
Objectively, his tooth 26 is missing. 86. A 37-year-old man was
On the medial surface of tooth 27 and prescribed an extraction of tooth 25
distal surface of tooth 25 there are under a local anesthesia. Where
class II carious cavities according to should the doctor be positioned in
Black's classification.It relation to the patient during the
isrecommended to make a full-cast extraction of tooth 25?
dental bridge with 27 and 25 as the A. Directly in front of the patient
abutment teeth. What material is used B. Behind and to the right of the
to assess whether the dental bridge patient
framework tightly adheres to the A. In front and to the right of the
abutment teeth? patient
A. - C. Behind and to the left of the
B. Corrective silicone mass patient
C. Articulating paper D. In front and to the left of the
D. Basic silicone mass patient
E. Model wax acute
2021 krok 2 dentistry https://krokology.com

87. A 44-year-old man came to a surfaces, the joint space is normal.


dental polyclinic to have a denture What is the most likely diagnosis in
made for him. He complains of this case?
problematic chewing and an A. Neuromuscular joint syndrome
aesthetical defect. Objectively, his B. Acute post-traumatic arthritis
teeth 14,15, and 22 are missing. C. Fibrous ankylosis of the joint
According to Agape, the loss of D. Deforming arthrosis
masticatory efficiency in this case is: E. Rheumatoid arthritis
A. 9%
B. 20% 90. A 60-year-old man came to a
C. 16% dentist two days after the fixation of
D. 12% a removable partial denture. He
E. 18% complains of an ulcer that appeared
on his oral mucosa. What has caused
88. Removable complete dentures the mucosal lesion near the edge of
are being made for a 70-year-old man the denture base?
with completely edentulous jaws and A. Mistakes when fitting the
marked progenia. What artificial artificial teeth
teeth are not placed in such dentures? B. Poorly fixed denture
A. Second molars on the upper jaw C. Mechanical trauma of the mucosa
B. Second premolars on the upper with the edge of the denture base
jaw D. Eating coarse food
C. Second molars on the lower jaw E. Incorrectly placed clasps
D. First premolars on the lower jaw
E. Second premolars on the lower 91. The anatomo-physiological
jaw method was applied to determine the
interalveolar height of a 65-year-old
89. A 52-year-old man complains man with completely edentulous
of pain and clicking in the area of his jaws. How does the distance between
left temporomandibular joint. the control points on the chin and at
Objectively, the face is symmetrical, the nasal base differ in this case from
palpation of the medial and lateral the physiological resting state of the
pterygoid muscles is painful on the lower jaw?
left. The mouth opening is limited to A. It is 2-3 mm longer
1.5 cm between the cutting edges of B. It is 5-6 mm shorter
the central incisors. X-ray shows C. It is 5-6 mm longer
smooth contours of the articular D. It is 2-3 mm shorter
2021 krok 2 dentistry https://krokology.com

E. The distance is the same opening. The cavity is located within


the parapulpar dentin and filled with
92. An 8-year-old girl complains dense pigmented dentin. The carious
of dry lips and a painful burning cavity does not communicate with
sensation in them. She has a history the dental cavity. Probing of the
of food allergies. Objectively, there is carious cavity walls and floor, cold
a slight edema and hyperemia of the stimulus, and percussion cause no
skin of her lips. The vermillion pain. Electric pulp testing-12
border is infiltrated and has fissures, microamperes. What is the most
scabs, and scales. The surface of the likely diagnosis?
lips is dry. In the angles of the mouth A. Chronic fibrous periodontitis
there are infiltrations, maceration, B. Chronic deep caries
and fissures. What is the most likely C. Chronic median caries
diagnosis in this case? D. Chronic gangrenous pulpitis
A. Eczematous cheilitis E. Chronic fibrous pulpitis
B. Meteorological cheilitis
C. Glandular cheilitis 95. An 18-year-old young man
D. Actinic cheilitis complains of a cavity in his tooth 37
E. Exfoliative cheilitis and pain that occurs when food gets
into this cavity. The tooth was painful
93. A 47-year-old man came to a one year ago but he did not make an
dentist to have a denture made for appointment with the doctor back
him. Objectively, his teeth 43, 44, then. Objectively, on the masticatory
and 45 are missing. What class of surface of tooth 37 there is a deep
mandibular dentition defect is it carious cavity that communicates
according to Kennedy? with the pulp chamber. Overgrown
A. - granulation tissue protrudes from the
B. 1 perforation into the pulp chamber and
C. 2 fills practically the whole carious
D. 3 cavity. Probing of the granulations
E. 4 provokes bleeding and slight pain.
Percussion is painless. Mucosa in the
94. A 28-year-old woman came to projection of the roots of tooth 37 is
a dentist with complaints of a cavity pale pink and has no pathologic
in her upper right tooth. Objectively, changes. Electric pulp testing-60
on the masticatory surface of tooth 16 microamperes. What is the most
there is a carious cavity with a wide likely diagnosis?
2021 krok 2 dentistry https://krokology.com

A. Hypertrophic papillitis B. Lymphadenitis of the


B. Chronic granulating periodontitis submandibular region
C. Chronic hypertrophic pulpitis C. Sialolithiasis
D. Granulations from the area of root D. Ranula of the sublingual salivary
bifurcation gland
E. Chronic fibrous pulpitis E. Mandibular osteoma

96. A 42-year-old man complains 98. Epidemiological examination


of periodic pain in his tooth 48. of certain age groups to study the
Examination shows that tooth 48 is prevalence of the signs that indicate
partially erupted with its medial periodontal diseases and to assess the
cusps, its mucosa is pale pink and need for treatment uses the following
painless on palpation, the mouth index:
opening is fully unrestricted. X-ray A. IR (Ramfjord)
shows tooth 48 to be located medially B. CPITN (WHO)
to tooth 47. Choose the surgical C. OHI-S (Green-Vermillion)
treatment: D. PI (Russel)
A. Operculum removal E. PMA (Parma)
B. Typical extraction of tooth 48
C. Bicuspidization (bisection) 99. A patient needs a removable
D. Atypical extraction of tooth 48 full laminar denture for the upper
E. Operculum dissection jaw. Objectively, on the mucosa of
the denture bed there are numerous
97. A 47-year-old man complains dense papillomas of varying size.
of pain and swelling in his left What tactics should the dentist
submandibular region and pain choose?
during eating. Objectively, the mouth A. Make a 3D-model of the denture
opening is unrestricted, mucosa of base
the left sublingual fold is hyperemic, B. Make the denture base from metal
the excretory duct of the C. Remove papillomas and make a
submandibular salivary gland denture with a double-layered
produces a small amount of turbid base lined with elastic material
saliva. X-ray of the mouth floor D. Removable dentures are
shows an oval shadow 1 cm in size. contraindicated in this case
Make the diagnosis: E. Reduce the area of the denture
A. Lipoma of the submandibular base
region
2021 krok 2 dentistry https://krokology.com

100. A 35-year-old man has missing, while teeth 14 and 16 are


developed edema and hyperemia that intact. What denture is preferable in
first appeared in the area of his wings this case?
of the nose and eventually spread to A. Adhesive dental bridge
the upper lip and checks. B. Porcelain-fused-to-metal dental
Examination detected a fissure in the bridge
area of the lower external edge of the C. Clasp (bugel) denture
left nostril. The skin there is taut, D. Swaged dental bridge
dense,purple-blue, with smoothed E. Partial removable laminar denture
out pattern. The local temperature is
elevated. Palpation is painful. The 103. A 55-year-old woman needs
affected area has clear irregularly- removable implant-retained dentures
shaped margins. Make the for both jaws. It is necessary to
provisional diagnosis: choose the type, the number, and the
A. Allergic dermatitis placement of implants. What
B. Lymphangitis additional examination method will
C. Odontogenic subcutaneous be the most efficient in making this
granuloma decision?
D. Eczema A. Computed tomography
E. Erysipelas B. Study of diagnostic dental casts
C. Ultrasound densitometry
101. A 39-year-old woman D. Spot X-ray
provisionally diagnosed with E. Occlusography
generalized periodontitis, I degree,
chronic progression. X-ray shows 104. A was 43-year-old woman
uneven resorption of interdental provisionally diagnosed with
septa by 1/3–1/4 of root length. What generalized periodontitis. What
surgical method must be used in this examination will be the most
case as a part of complex treatment? informative for the assessment of the
A. Gingivotomy severity of her condition?
B. Closed curettage A. Measuring the depth of the
C. Flap surgery periodontal pockets
D. Gingivo-osteoplasty B. Measuring the degree of tooth
E. Open curettage mobility
C. Determining the periodontal
102. A 27-year-old woman needs a index
denture. Objectively, her tooth 15 is D. Panoramic X-ray
2021 krok 2 dentistry https://krokology.com

E. Schiller-Pisarev test and misbehaves. What treatment


tactics would be optimal in this case?
105. A 7-year-old boy is diagnosed A. Preventive filling
with epidemic parotitis (mumps). B. ART technique
Name the most likely complication of C. Impregnation treatment
this disease: D. Deep fluoridation
A. Orchitis E. Invasive sealing
B. Dermatitis
C. Pneumonia 108. A 48-year-old man, who for
D. Cholecystitis several years already has been
E. Colitis suffering from enterocolitis,
complains of painful lesionsthat from
106. The parents of a 5-year-old time to time appear in different
child explain that lately their child places of his oral mucosa. Objective
has started chewing only on one side examination of the mucosa reveals
and refusing hard foods. Objectively, isolated areas of epithelial damage.
tooth 74 has a carious cavity and They are under 5 mm in diameter,
there are dental. deposits on teeth 73, have a regular oval shape, hyperemic
74, and 75. Probing of the cavity border, and yellow white coating.
floor in tooth 74 is painful and Their palpation is painful. What is the
provokes a slight bleeding in the area most likely diagnosis?
where the cavity communicates with A. Allergic stomatitis
the pulp chamber. Make the B. Secondary syphilis
diagnosis: C. Chronic recurrent herpes
A. Chronic hypertrophic pulpitis D. Chronic recurrent aphthous
B. Chronic fibrous pulpitis stomatitis
C. Acute median caries E. Erythema multiforme exudativum
D. Chronic gangrenous pulpitis
E. Acute deep caries 109. A 62-year-old man came to a
dentist to have his tooth 44 extracted.
107. Parents of a 5-year-old child During examination he suddenly fell
came to the dentist for sanation of the unconscious. His breathing and pulse
child's oral cavity. Objectively, on are slow, his head is bowed, his arms
the masticatory surfaces of teeth are flexed in the elbows and
54,64, 74, 75, and 85 there are wrists,his legs are stretched out, and
carious cavities within the softened his jaws are clenched tight. The
mantle dentin. The child is anxious patient developed clonic convulsion
2021 krok 2 dentistry https://krokology.com

and started foaming at the mouth.His displaced to the left. Palpation


pupils are dilated and unresponsive to detects diminished excursion of the
light. 1.5 minutes later, the patient's articular head in the left lower jaw.
muscles relaxed. The patient is The mouth opening is reduced to 1
mentally confused. What is the most cm. Make the provisional diagnosis:
likely diagnosis in this case? A. Exacerbated chronic arthritis of
A. Epileptic seizure the left temporomandibular joint
B. Paroxysmal tachycardia attack B. Arthrosis of the left
C. Hypertensive crisis temporomandibular joint
D. Syncope C. Anterior dislocation of the lower
E. Acute cerebrovascular accident jaw
D. Acute arthritis of the left
110. A removable complete denture temporomandibular joint
is being made for the lower jaw of a E. Ankylosis of the left
50 year-old man. During the fitting of temporomandibular joint
an individual impression tray, the
border in the area of the mylohyoid 112. Examination of the oral cavity
line needs to be determined. What of a 12-year-old child revealed a
Herbst test should be performed for carious cavity on the medial contact
this purpose? surface of tooth 46. The cavity is
A. Alternately touching the right and located within the mantle dentin, its
left cheeks with the tongue walls and floor are pigmented and
B. Stretching the tongue towards the dense. Thermal stimuli provoke no
tip of the nose response. Preparation of tooth 46 is
C. Swallowing saliva painful in the area of dentinoenamel
D. Licking the upper lip junction. The decay-missing-filled
E. Wide mouth opening index is DMF + df = 2. What filling
Answer here is controversial between material will be optimal in this case?
C and D. A. Silicophosphate cement
B. Glass-ionomer cement
111. A 21-year-old man complains C. Zinc phosphate cementy
of difficult mouth opening and D. Amalgam
visible facial deformation throughout E. Photopolymer composite
the last 2 years. He has a history of
facial trauma several years ago. 113. In the dentist's office, after a
Examination shows the right side of treatment of tooth 26 a 46-year-old
the face to be flattened, the chin is woman suddenly developed a pulsing
2021 krok 2 dentistry https://krokology.com

headache, dizziness, ringing in her


ears, tachycardia, and blood pressure 116. A 25-year-old man complains
of 220/160 mm Hg. Make the of brief pain attacks in response to
provisional diagnosis: sweet foods in one of his lower teeth
A. Hypertensive crisis and food retention in a carious cavity.
B. Anaphylactic shock Objectively, on the masticatory
C. Hypoglycemic coma surface of tooth 36 there is a carious
D. Syncope cavity within the parapulpar
E. Acute cerebrovascular accident dentin.The dentin is soft. The carious
cavity does not communicate with
114. A regular check-up of a 7- the dental cavity, probing of its floor
year-old girl detected chalk-like is painful. Percussion of tooth 36 and
spots on the vestibular surface of her palpation of gingival mucosa in the
teeth 11 and 21. The surface of the projection of root apex are painless.
spots is matt, smooth when probed, Thermal stimuli cause pain that
and can be stained with methylene quickly stops after the simulus is
blue. What treatment method would removed. What is the most likely
be rational in this case? diagnosis?
A. Impregnation therapy of teeth 11 A. Pulp hyperemia
and 21 B. Chronic fibrous pulpitis
B. Filling of teeth 11 and 21 C. Acute median caries
C. Filing down the defects of teeth D. Chronic deep caries
11 and 21 E. Acute deep carics
D. Regular check-ups to monitor the
child's condition 117. A 6-year-old child complains
E. Deep fluoridation of teeth 11 and of food retention in a tooth on the
21 lower jaw. Objectively, in tooth 75
on its masticatory surface within
115. A 7-year-old child needs mantle dentin there is a carious cavity
mandibular vestibuloplasty. What with a wide opening. The dentin of
pathology is most often caused by a the cavity walls and floor is dense
shallow vestibule of the mouth? and pigmented. Probing of the
A. Diseases of periodontal tissues carious cavity and percussion of the
B. Crowded front teeth tooth are painless. Thermal stimuli
C. Deep bite provoke no response. Mucosa in the
D. Mandibular growth retardation area of tooth 75 is without pathologic
E. Parafunction of the facial muscles
2021 krok 2 dentistry https://krokology.com

changes. The child belongs to health periodontal pockets. X-ray shows


group I. Make the diagnosis: osteosclerosis of the alveolar process
A. Acute deep caries and disturbed intactness of the lamina
B. Chronic median caries dura on the apices of the interdental
C. Chronic deep caries septa. The patient was diagnosed
D. Chronic fibrous pulpitis with initial periodontosis. What
E. Chronic superficial caries should be prescribed in this case for
local treatment of the systemic dental
118. A 12-year-old child complains hyperesthesia?
of high body temperature of 39.7 ° C, A. Keratoplastics
enlarged lymph nodes, sore throat, B. Antiseptics
runny nose, and cough. Objectively, C. Desensitizers
acute catarrhal stomatitis is observed D. Keratolytics
in the oral cavity, some of the E. Anesthetics
palatine tonsils have a yellow gray
coating that crumbles and can be 120. A 2-year-old girl has a bright
easily removed. The cervical, red neoplasm 1x1.5 cm in size on her
occipital, and submandibular lymph upper lip. The neoplasm does not
nodes are enlarged, mobile, and protrude above the mucosa and
slightly painful. Laboratory blood becomes paler, when pressed. The
testing shows leuko-, lympho-, and regional lymph nodes have no
monocytosis and atypical pathology. The results of blood and
mononuclear cells. What is the urine analysis are normal for her age.
causative agent of this disease? Make the provisional diagnosis:
A. Loeffler bacillus A. Systemic hemangiomatosis
B. Hemolytic streptococcus B. Cavernous hemangioma
C. Coxsackievirus C. Capillary lymphangioma
D. Herpes simplex virus D. Cavernous lymphangioma
E. Epstein-Barr virus E. Capillary hemangioma

119. A 47-year-old woman 121. A 14-year-old girl complains


complains of an itching sensation in of a pulsing pain in her upper left
her gums and increased teeth teeth that lasts for 3 days already and
sensitivity to thermal stimuli. sharply increases on biting.
Objectively, the roots of most of her Objectively, her tooth 26 has a deep
teeth are exposed to 1/3 of their carious cavity that does not
length, the gums are pale pink, no communicate with the dental cavity.
2021 krok 2 dentistry https://krokology.com

Probing of the cavity floor is 123. A 2-year-old child received a


painless. Vertical and horizontal dental trauma. Objectively, the
percussion is sharply painful. The crowns of teeth 51 and 61 are shorter
mucosa in the area of tooth 26 is than the crowns of the adjacent teeth
hyperemic. What is the provisional by 1/3. The mucosa in the area of
diagnosis in this case? teeth 51 and 61 is hyperemic and
A. Acute serous periodontitis edematous. X-ray shows no
B. Pulpitis complicated with periodontal fissure in the apical area
periodontitis of the roots of teeth 51 and 61. What
C. Acute purulent periodontitis treatment tactics would be optimal in
D. Acute purulent pulpitis this case?
E. Acute diffuse pulpitis A. Ligature splinting
B. Reimplantation
122. A man with an occupational C. Extraction of teeth 51, 61
face trauma on the right has been D. Reposition of teeth 51, 61
brought into the first-aid center. The E. Regular check-ups to monitor the
trauma was received approximately 3 child's condition
hours ago, the man is conscious.
Objectively, on his right cheek there 124. The parents of a 6-year-old
is an irregularly-shaped wound 5x3 child complain that their child for the
cm in size with uneven edges. Parting last three days had body temperature
of the wound edges exposes of 39 ° C, weakness, loss of appetite,
immobile teeth. In the oral cavity, and pain during swallowing.
there is a breach in the buccal Objectively, the mucosa of the
mucosa. The bite is not affected. tonsils, palatine arches, retromolar
What type of surgical debridement is space, and posterior and lateral
necessary for this patient? pharyngeal walls is swollen and has a
A. Early primary surgical dirty-gray coating that is tightly
debridement attached to the underlying
B. - tissues.The coating is very difficult to
C. Late primary surgical remove. When removed, it exposes a
debridement bleeding surface. The submandibular
D. Secondary surgical debridement lymph nodes are enlarged and
E. Delayed primary surgical painful. What is the most likely
debridement provisional diagnosis?
A. Scarlet fever
B. Measles
2021 krok 2 dentistry https://krokology.com

C. Infectious mononucleosis D. Chronic median caries


D. Acute pseudomembranous E. Acute deep caries
candidiasis Answer here is controversial
E. Diphtheria
127. During dental procedures, a 29
125. After a tooth extraction, the year-old man after receiving
patient developed a hemorrhage from anesthesia suddenly developed a
the socket of the extracted tooth. dense non-itching skin edema in the
What tactics should the dentist area of his face and neck. The edema
choose in this case? was accompanied by cough and
A. Insert a gauze pack soaked in 3% wheezing. What is the most likely
hydrogen peroxide solution diagnosis in this case?
B. Place sutures into the soft tissues A. Anaphylactic shock
C. Administer 5-10 ml of 10% B. Urticaria
calcium chloride solution C. Bronchial asthma attack
intravenously D. Contact dermatitis
D. Tightly pack the tooth socket with E. Quincke's edema
a strip of iodoform gauze
E. Administer vicasol (menadione) 128. A 35-year-old man undergoes
solution intramuscularly treatment for chronic fibrous pulpitis
of tooth 25. The canal is to be filled
126. A 21-year-old woman using the warm gutta-percha vertical
complains of short-term pain attacks condensation technique. What
in her lower left tooth that are instrument is needed for gutta-percha
provoked by thermal stimuli and condensation?
quickly stop after the stimulus is A. K-file
removed. On the masticatory surface B. Root needle
of tooth 47, examination reveals a C. Plugger
carious cavity with a narrow entrance D. Spreader
that is filled with light softened E. Endodontic probe
dentin. Probing of the cavity floor
and percussion of tooth 47 are 129. A 27-year-old woman came to
painless. Probing of the cavity walls the dentist for the purpose of oral
is painful. Make the diagnosis: cavity sanation. After anesthesia, she
A. Chronic deep caries started complaining of dizziness,
B. Acute median caries ringing in her ears, dimming vision,
C. Chronic superficial caries and general weakness. Objectively,
2021 krok 2 dentistry https://krokology.com

her skin is pale and covered in cold neoplasm is mobile, painless, and
sweat. Her pulse is 94/min., of low fused with the skin. Punctate skin
volume and pressure. Her blood depressions can be observed on its
pressure is 96/60 mm Hg. She had a surface. Make the provisional
brief episode of unconsciousness that diagnosis:
lasted 30 seconds. What urgent A. Frontal lipoma
condition developed in this woman? B. Exophytic form of skin cancer
A. Syncope C. Frontal atheroma
B. Hyperglycemic coma D. Frontal fibroma
C. Acute heart failure E. Frontal keratoacanthoma
D. Collapse
E. Hypertensive crisis 132. When fitting the metallic
frame of a porcelain-fused-to-metal
130. After examination, an 8-year- dental bridge, it was determined that
old child was diagnosed with in the central occlusion the frame is
torsiversion of the lateral maxillary in a contact with the antagonist teeth.
incisors and an arch length deficiency It reaches the ledges of the abutment
caused by macrodontia. To prevent teeth and is 0.3 mm thick. What
the vestibular position of the canines, tactics should a dentist choose?
a Hotz serial extraction was A. Send the frame to the next
performed. In what order were the laboratory stage of the denture-
teeth removed in this case? making
A. First temporary molars, first B. Determine the areas that prevent
premolars, temporary canines normal installation of the dental
B. First temporary molars, bridge.
temporarycanines, first premolars C. Obtain an impression with the
C. Temporary canines, first frame fitted into the oral cavity
temporary molars, first premolars D. File down the metallic frame in
D. Second incisors, temporary the areas of its contact with the
canines and first temporary molar antagonist teeth
E. First temporary molars, E. Complete the preparation of the
temporary canines, second abutment teeth and make a
premolars working impression
Answer here is controversial.
131. A patient complains of a
slowly growing neoplasm in the left 133. When pressing plastic dough,
frontal region of his head. The a dental technician several times
2021 krok 2 dentistry https://krokology.com

opened and closed the cuvette to palpation, the maxillary alveolar


check the quantity of the plastic mass process is mobile. Make the
in it. These actions may result in the provisional diagnosis:
development of: A. Le Fort maxillary fracture, type II
A. Residual stresses in the structure B. Le Fort maxillary fracture, type I
of a denture C. Maxillary contusion
B. Gas porosity D. Le Fort maxillary fracture, type
C. Compression porosity III
D. Granular porosity E. Zygomatic fracture
E. Cracks in the plastic
136. A man complains of muscle
134. A 26-year-old man came to a weakness, constipations, dry mouth,
dentist complaining of facial inability to swallow food, and vision
asymmetry that developed because of impairment. The day before, he was
a painful swelling in the left parotid eating homemade canned fish.
area that appeared 3 days ago. His Objectively, his skin and visible
body temperature is 37.2 ° C. Two mucosa are pale; he has diplopia and
weeks ago he had a case of acute bilateral ptosis. What disease can be
respiratory disease. Objectively, in suspected in this case?
the left parotid area there is a dense A. Botulism
round infiltration up to 2 cm in size B. Salmonellosis
that is slightly mobile and moderately C. Food poisoning
painful. The excretory duct of the D. Cholera
parotid gland produces clear E. Shigellosis
transparent saliva. Make the
provisional diagnosis: 137. During preventive
A. Herzenberg parotitis examination, a 20-year-old man
B. Acute purulent parotitis presents with enamel defects that
C. Mikulicz disease look like rough white spots with
D. Pleomorphic adenoma of the uneven margins in the vestibular
parotid gland precervical area of teeth 11 and 12.
E. Exacerbation of chronic parotitis The spots stain with 2% methylene
blue solution. Make the diagnosis:
135. A 45-year-old man complains A. Acute superficial caries of teeth
of a pain in his upper jaw. Two days 11 and 12, Black class V
ago he had a trauma. Objectively, he B. Acute superficial caries of teeth
has a disturbed occlusion. On 11 and 12, Black class III
2021 krok 2 dentistry https://krokology.com

C. Acute initial caries of teeth 11 and 140. A patient needs endodontic


12, Black class III T treatment of tooth 21. The canal is
D. Acute initial caries of teeth 11 and being processed with manually
12, Black class V operated endodontic instruments.
E. Focal enamel hypoplasia of teeth Name one such instrument made by
11 and 12 means of conical spiral threading
(turning) of a steel wire with a round
138. A man with distal bite cross section (milling):
underwent Eschler-Bittner test for A. K-reamer
the purpose of differential B. H-file
diagnostics. The shape and profile of C. Plugger
his face have notably improved. D. Spreader
What type of distal bite is it, E. K-file
according to test results?
A. Overdeveloped lower jaw 141. A 54-year-old man developed
B. Underdeveloped upper jaw a pustule on the skin of his chin. The
C. Overdeveloped upper jaw pustule rapidly progressed into a hard
D. Underdeveloped lower jaw and sharply painful infiltration 3x3.5
E. Maldevelopments and growth cm in size. The skin over the
abnormalities of the both jaws infiltration is bluish-red. In its center
there are three necrotic foci
139. A 35-year-old man has an surrounding the hair follicles.
open unilateral non-dispalced Palpation detects enlarged and
fracture of the mandibular body on painful submental lymph nodes. The
the left. The fracture line passes body temperature is 38.5 ° C. Make
between the missing teeth 33 and 34. the provisional diagnosis:
The fragments need to be A. Carbuncle
immobilized. What splint would be B. Allergic dermatitis
the most advisable for this purpose? C. Furuncle
A. Tigerstedt splint with a spreader D. Erysipelas
bar E. Festering atheroma
B. Port splint
C. Two-jaw splint with wire loops 142. A maxillofacial surgery
D. Splint with an inclined plane department has received a 34-year-
E. Smooth splint-bracket old man with a frostbite on his face.
Examination shows that the skin in
the affected area is edematous,
2021 krok 2 dentistry https://krokology.com

hyperemic, and cyanotic. No tissue C. Applying the tip of the forceps


necrosis. What degree of the frostbite jaws to the edge of alveolar
is it? process
A. II degree D. Pushing the forceps jaws to the
B. - cementoenamel junction
C. III degree E. Closure of the forceps handles
D. IV degree
E. I degree 145. A 30-year-old man complains
of a carious cavity in a lower left
143. A 50-year-old woman with tooth. Objectively, tooth 34 has a
compensated insulin-dependent deep carious cavity that at one point
diabetes mellitus needs dental communicates with the pulp
prosthetics. She has been in a clinic chamber. Probing of the cavity floor
for several hours already, during is painless, while percussion of the
which she was nervous and skipped tooth is slightly painful. Thermal
her meals. In the process of obtaining stimuli provoke no response. On the
her dental impressions, she suddenly gums, there is a fistula in the
became aggressive, paled, broke out projection of the root apex of tooth
in cold sweat, and fell unconscious. 34. Make the provisional diagnosis:
What substance is used for A. Chronic granulating periodontitis
emergency aid in such clinical B. Chronic fibrous periodontitis
situations? C. Chronic fibrous pulpitis
A. Ammonia solution D. Chronic hypertrophic pulpitis
B. Nitroglycerine E. Chronic deep caries
C. Glucose solution
D. Valocordin 146. A 48-year-old woman
E. Insulin complains of problems with chewing,
caused by missing teeth. Objectively,
144. What manipulation of those her teeth 31, 32, 33, and 41, 42, 43
listed below is not a part of typical have tall crowns and the I degree of
procedure of tooth extraction with mobility. The decision was reached
forceps? to make a clasp denture splint for her.
A. Tooth dislocation and extraction What type of clasps (clammers) will
from the socket ensure the splinting function of this
B. Applying the forceps jaws to the construction?
tooth A. Vestibular
B. Continuous
2021 krok 2 dentistry https://krokology.com

C. Retaining neoplasm yielded a yellowish liquid


D. Abutment with cholesterol crystals. Make the
E. Dentoalveolar provisional diagnosis:
A. Cementoma
147. A 28-year-old man came to a B. Residual cyst
dentist for the purpose of sanation. C. Paradental cyst
Objectively, on the masticatory D. Follicular cyst
surface of tooth 47 there is a deep E. Radicular cyst
carious cavity that communicates
with the dental cavity. Probing of the 149. The jaws of a newborn are in
cavity is painless, the tooth is non- direct relation to each other. The
responsive to thermal stimuli, its baby is full term, healthy, and
percussion is painless. Electric pulp breastfed. What bite can be
testing -108 microamperes. X-ray prognosed in this case?
shows traces of filling material in the A. Deep bite
root canals of tooth 47, the B. Direct bite
periodontal fissure is widened and C. Open bite
deformed in the apical region. Make D. Distal bite
the diagnosis: E. Mesial bite
A. Chronic fibrous periodontitis of
tooth 47 150. A 48-year-old woman
B. Chronic granulating periodontitis complains of itching gums and
of tooth 47 increased teeth sensitivity to thermal
C. Chronic gangrenous pulpitis of and chemical stimuli. Objectively,
tooth 47 the dental cervices are exposed. The
D. Chronic fibrous pulpitis of tooth gums are firm and pale. There is a
47 small amount of supragingival dental
E. Chronic granulomatous plaque. In the area of teeth 13, 14, 24,
periodontitis of tooth 47 and 25 there are cuneiform defects,
probing of which is painful. The
148. X-ray of the lower jaw of a 14- woman was provisionally diagnosed
year old boy shows a round with periodontosis. What X-ray
homogeneous lucency with clear presentation is characteristic of this
margins. This neoplasm is located disease?
within the body of the mandible in A. Horizontal resorption
the area of tooth 36 that was extracted B. Bone pocket formation
7 months ago. A puncture of the C. Vertical resorption
2021 krok 2 dentistry https://krokology.com

D. Uneven resorption of the alveolar


bone
E. Diffuse osteoporosis
2021 krok 2 dentistry
Practice form

Please note ✍

Answers for this booklet is randomized for practicing purpose. To


check the answer please visit Krokology.com
2021 krok 2 dentistry https://krokology.com

2021 – krok 2 – dentistry


Krokology.com

1. A 6-year-old child was woman. According to Black's


referred for extraction of the classification of dental caries, this
temporary lower central incisors due cavity is class:
to changes in the dentition. The teeth A. V
have the III degree of mobility. What B. I
type of anesthesia would be optimal C. II
in this case? D. III
A. Conduction anesthesia E. IV
B. Infiltration anesthesia
C. Inhalation narcosis 4. A 15-year-old girl complains
D. Intraosseous anesthesia of a carious cavity in her tooth and
E. Application anesthesia spontaneous pain that quickly passes.
The child is somatically healthy.
2. A 27-year-old woman Objectively, in tooth 36 there is a
complains of an aesthetical defect in carious cavity within the parapulpar
the area of her upper central incisors. dentin, the response to a cold
She was diagnosed with chronic deep stimulus is painful, the pain abates in
caries, Black class IV. Aesthetic 1 2 miuntes. Probing of the cavity
restoration of teeth 11 and 21 is floor is painful in one spot.
planned. What material has optimal Percussion is painless. Make the
combination of strength and provisional diagnosis:
aesthetical properties? A. Acute local pulpitis
A. Compomers B. Acute deep caries
B. Hybrid composites C. Chronic deep caries
C. Microfilled composites D. Pulp hyperemia
D. Macrofilled composites E. Acute diffuse pulpitis
E. Liquid composites
5. Preventive examination of a 7-
3. A 36-year-old woman needs a year old child revealed a carious
denture. Objectively, there is a cavity with overhanging enamel
carious cavity on the mesial and within the limited area of a fissure in
masticatory surfaces of tooth 46, the tooth 36. The probe catches in the
interdental contact is disturbed. A softened dentin. In the other areas,
dental inlay is to be made for this the enamel on the edges of the
2021 krok 2 dentistry https://krokology.com

fissures is glossy and dense. A dentist A. Keratoplasties


plans to fill the carious lesion of tooth B. Antiseptics
36. What tactics should be chosen C. Anesthetics
regarding the intact areas of the D. Proteolytic enzymes
fissure? E. Antiviral agents
A. Invasive sealing
B. Processing the tooth with an 8. A 33-year-old man undergoes
antiseptic solution treatment of tooth 15 for acute
C. Non-invasive sealing median caries. What type of
D. Application of fluoride varnishes anesthesia will be the most advisable
E. Application of calcium- in this case?
containing gels A. Application anesthesia
B. Non-inhalation anesthesia
6. A 47-year-old man complains C. Inhalation anesthesia
of frequent recurrent jaw D. Infiltration anesthesia
dislocations. Objectively, the teeth E. Conduction anesthesia
are retained and the molars are
covered with artificial crowns. What 9. A 38-year-old man complains
treatment tactics should be chosen in of a neoplasm 1 cm in size on the
this case? gums in the area of tooth 14. The
A. Physiotherapy neoplasm is bright red, soft, round,
B. Myogymnastics, physiotherapy and bleeds when touched. This
C. Restriction of the mouth opening, clinical presentation corresponds
using the Petrosov appliance with the following disease:
D. Restriction of the mouth opening, A. Angiomatous epulis
using the Schroeder appliance B. Fibrous epulis
E. Immobilization of the jaw with C. Eosinophilic granuloma
teeth ligation D. Fibroma
E. Migratory granuloma
7. A 2-year-old child is being
treated for severe acute herpetic 10. A 56-year-old man has an oval
stomatitis. Currently, it is the period erosion with a deep red smooth
of disease decline. What medicines surface on the vermillion border of
for topical application to the oral the lower lip between the angle of the
mucosa should be added specifically mouth and the midline. The surface
at this stage to the complex of of the erosion has bloody scabs that
medicines being taken? are difficult to remove. Their
2021 krok 2 dentistry https://krokology.com

removal causes a slight bleeding. are enlarged and dense. The mouth
Slight trauma of the erosion in a place can be opened without restriction. No
without scabs causes no bleeding. saliva flows out from the duct of the
What type of lower lip precancer is right parotid gland. What provisional
it? diagnosis can be made?
A. Lupus erythematosus A. Chronic lymphadenitis of the
B. Manganotti's abrasive right parotid region
precancerous cheilitis B. Mixed tumor of the right parotid
C. Circumscribed precancerous gland
hyperkeratosis of the vermillion C. Chronic non-epidemic parotitis
border D. Adenocarcinoma of the right
D. Bowen's disease parotid gland
E. Actinic cheilitis E. Actinomycosis of the right parotid
gland
11. Preparation of the vital tooth
34 for the installation of an all- 13. A 4-year-old child was
ceramic crown is planned for a 38- diagnosed with exacerbation of
year-old man. The man suffers from chronic periodontitis of tooth 85. X-
ischemic heart disease. What ray shows destruction of the lamina
anesthesia is advisable in this case? dura of the dental follicle of tooth 45,
A. Torusal a shadow in the bifurcation area of
B. Infiltration tooth 85, and pathologic resorption of
C. Tuberal 2/3 of the medial root. What
D. Mental treatment tactics would you choose?
E. Intraligamentary A. To fill the root canals in one visit
and perform periostotomy
12. A 56-year-old man complains B. To perform periostotomy and
of a painful swelling in his right hemisection of the resorbed root
parotid area. The swelling was C. To extract tooth 85
noticed 5-6 months ago. Objectively, D. To stop pain and inflammation,
he has a right-sided paresis of the conduct impregnation treatment
facial muscles. Palpation reveals a of root canals, and keep the tooth
moderately painful tuberous tumor open until the natural replacement
fused with the surrounding tissues. In of the primary dentition occurs
the center of the tumor, there is an E. To conduct endodontic and
area of softening. The submandibular pharmacological treatment of the
and cervical lymph nodes on the right
2021 krok 2 dentistry https://krokology.com

root canals of tooth 85 and apply planned to be done under anesthesia.


occlusive dressing What type of anesthesia will be the
most effective in this case?
14. A 14-year-old girl came to a A. Infiltration
dentist with complaints of enlarged B. Intraligamentary
interdental papillae in the frontal C. Conduction (torusal)
regions of her upper and lower jaw. D. Conduction (tuberal and palatal)
Examination shows that the E. Application
interdental papillae are hyperemic,
edematous, cover the crowns to 1/2 17. The parents of a 14-year-old
of their height, and bleed when girl brought her to a dentist,
touched. What system in this case has complaining that she has some kind
a pathology that plays the main role of growth on her tongue. It appeared
in the etiology of this disease? approximately 1.5 years ago and has
A. Central nervous system been slowly growing since then.
B. Endocrine system Objectively, on the lateral surface of
C. Cardiovascular system her tongue there is a neoplasm on the
D. Immune system left. It is pink, mobile, painless, hard,
E. Digestive system and spherical. It has clear margins,
wide base, and smooth surface. Make
15. A 71-year-old man came to a the provisional diagnosis:
dental orthopedist to have a denture A. Atheromax
made for him. During examination, B. Hemangiomax
the dentist noticed that the mucosa C. Lymphangiomax
that covers the palate and the alveolar D. Papilloma
processes was worn thin and barcly E. Fibroma
pliant. What Supple class is it?
A. Fourth 18. A man came to a dental
B. Third therapy clinic with complaints of
C. Second isolated transversal furrows on the
D. First crowns of his front and lateral teeth.
The furrows were detected
16. A porcelain-fused-to-metal immediately after the teeth eruption.
dental bridge is being made for a 31- In his childhood, in the age of under
year-old woman. Vital teeth 33, 36, a year, he had frequent acute
and 37 will function as the abutment respiratory viral infections.
teeth. The teeth preparation is Objectively, the crowns of teeth 16,
2021 krok 2 dentistry https://krokology.com

11, 21, and 26 have a single D. Replacing


indentation (a band) in their middle E. Directing
within the enamel. The furrows do
not stain with methylene blue. What 21. A 58-year-old woman
is the most likely diagnosis in this complains of a red spot on her lower
case? lip that first appeared 4 years ago.
A. Erosion of hard dental tissues Objectively, on the vermillion border
B. Fluorosisx of the lower lip there is a red spot
C. Enamel hyperplasia with velvety surface. The lesion is
D. Systemic enamel hypoplasia slightly concave, when compared to
E. Amelogenesis imperfecta the surrounding tissues. It does not
change its color when pressed. Make
19. A 40-year-old man came to a the provisional diagnosis:
dental clinic with complaints of A. Bowen's disease
bleeding, edematous, and hyperemic B. Verrucous precancer
gums and missing lateral teeth on his C. Simple leukoplakia
lower jaw. A complex treatment is D. Hemangioma
planned for this man, along with E. Exfoliative cheilitis
making removable dentures. What
method of functional diagnostics. is 22. A 34-year-old woman
used to assess the condition of the complains of a pain in her lower left
periodontal vessels? jaw. Two months ago, her teeth 35,
A. Rheoparodontography 36, and 37 were extracted.
B. Masticatiography Objectively, her lower left jaw has a
C. Galvanometry cuff like thickening. X-ray shows a
D. D.X-ray sequestrum. What is the most likely
E. Myography diagnosis in this case?
A. Acute serous alveolitis
20. A 40-year-old man has a fresh B. Chronic odontogenic
mental mandibular fracture without osteomyelitis
visible displacement of the C. Mandibular sarcoma
fragments. What dental orthopedic D. Acute purulent periostitis
appliance is recommended in this E. Ameloblastoma
case?
A. Reponating 23. A 28-year-old man has
B. Forming. received a trauma of the mental
C. Fixing region. X-ray shows a bilateral
2021 krok 2 dentistry https://krokology.com

mental fracture of the lower jaw. In septa, though the cortical plate itself
what direction will the displacement is intact. Make the diagnosis:
of the lesser fragment occur? A. Acute ulcerative gingivitis
A. No displacement occurs B. Desquamative gingivitis
B. - C. Chronic hypertrophic gingivitis
C. Upwards and backwards D. Chronic catarrhal gingivitis
D. Downwards and backwards E. Acute catarrhal gingivitis
E. Downwards and forwards
26. A 37-year-old man complains
24. A 26-year-old man came to a of an unpleasant sensation in his
dental clinic. He was diagnosed with gums on the lower left jaw that
a non-displaced fracture of the developed after he underwent
maxillary alveolar process in the area treatment for dental caries. several
of teeth 14 and 15. The dentition months ago. Examination reveals
remains uninterrupted. All the teeth permanent fillings on the contact
are stable. What splint must be surfaces of teeth 36 and 37. The
applied in this case? edges of the fillings are hanging over
A. Tigerstedt splint with a spreader the interdental papilla. The gum in
bar the area of teeth 36 and 37 is
B. Tigerstedt splint with an inclined hyperemic and edematous. The
plane periodontal pocket is up to 3 mm
C. Vasiliev splint deep. What treatment will be
D. Smooth splint-bracket etiological in this case?
E. Tigerstedt splint with wire loops A. High-quality filling
B. Pharmacological treatment
25. A regular check-up has C. Professional hygiene
detected crowded teeth on the front D. Thorough personal hygiene
lower jaw of a 15-year-old boy. The E. Closed curettage
gums in this area are slightly
edematous and hyperemic with a 27. A 48-year-old man came to a
cyanotic tint. The gingival margin dental surgeon for extraction of the
has a torusal thickening. Dental roots of tooth 37. What instrument
calculus is observed on the teeth. The must be used for this procedure?
Schiller Pisarev test is positive. X-ray A. Beak-shaped crushing forceps
shows that in the front lower jaw the B. Bayonet forceps
contours of the cortical plate are C. Beak-shaped non-crushing
fuzzy on the apices of the interdental forceps
2021 krok 2 dentistry https://krokology.com

D. Beak-shaped forceps, curved in a the patient. The tooth will be left


planc vital. What measures should be taken
E. S-shaped forceps during the treatment to prevent
pulpitis in this tooth?
28. After a trauma, a man has A. Fluorine-containing preparations
developed complaints of displaced B. Physical therapy
tooth 32 and pain that intensifies C. Calcium-containing preparations
when he bites food. Examination D. Temporary crown
shows edema of the lower lip and E. Anti-inflammatory preparations
mucosal hematoma in the area of
tooth 32. Tooth 32 has the I degree of 31. A 22-year-old man came to a
mobility and a slight sagittal dentist with complaints of increased
displacement. X-ray shows a salivation and a sharp pain in the oral
widening of the periodontal fissure. cavity during eating and talking. He
Make the diagnosis: notes the acute onset of the disease
A. Complete dislocation of tooth 32 that occurred after an overexposure
B. Contusion of tooth 32 to cold, high body temperature (39.5
C. Incomplete dislocation of tooth 32 ° C), general weakness, and
D. Root fracture of tooth 32 headache. He has a history of chronic
E. Impacted dislocation of tooth 32 tonsillitis and is allergic to certain
medicines. On examination his skin
29. An 11-year-old child is pale. On the dorsal surface of his
undergoes orthodontic treatment. hands there is a bluish-pink rash with
The child is prescribed a dental a hemorrhagic scab in the center. The
appliance with combined action vermilion border of the lips is
(functional direction and mechanical edematous and has hemorrhagic
action). What structural elements are scabs. The regional lymph nodes are
likely to be present in this dental enlarged, painful, and mobile. Make
appliance? the provisional diagnosis:
A. Occlusive onlays, buccal shields A. Drug-induced stomatitis
B. Omega loop, screw B. Erythema multiforme exudativum
C. Occlusive onlays, screw C. Acantholytic pemphigus
D. Inclined plane, tongue shield D. Stevens-Johnson syndrome
E. Occlusive onlays, labial bandages E. Acute herpetic stomatitis

30. A porcelain-fused-to-metal 32. A man presents with clinical


crown for tooth 11 is being made for signs that resemble acute herpetic
2021 krok 2 dentistry https://krokology.com

stomatitis. This diagnosis will be tuberal and palatal anesthesia. What


confirmed, if cytology detects the anesthetic solution needs to be
presence of the following cells: administered in this case?
A. Multinucleated giant cells A. 10% lidocaine
B. Nicmann-Pick cells B. 2% dicain (tetracaine)
C. Atypical mononuclear cells C. 2% novocaine (procaine)
D. Tzanck cells D. 5% anaesthesin (benzocaine)
E. Gaucher cells E. 3% mepivastesin (mepivacaine)

33. A 45-year-old woman 35. A 54-year-old woman


complains of bleeding gums. The complains of a neoplasm on her
bleeding occurs when she brushes her lower lip that appeared 6 month ago.
teeth or eats hard food. Objectively, Twenty days ago, it suddenly started
in the frontal area of her lower jaw growing rapidly. Objectively, on the
the gums are hyperemic, edematous, vermillion border of the lower lip
and bleed when palpated. In the other there is a round red-gray node that
areas, her oral mucosa is normal. The rises 0.5 cm above the underlaying
woman has a deep bite. Her teeth are tissues. It is distinguished from the
stable, except for teeth 41 and 31 (the surrounding tissues and has a
I degree of mobility). X-ray shows funnelform depression in its center,
resorption of the interalveolar septa filled with keratinized masses. On
in the area of teeth 41, 42, 31, and 32 palpation, the nodule is dense,
to 1/3 of the root length. What is the mobile, and painless. What is the
most likely diagnosis? most likely diagnosis?
A. Generalized periodontitis, initial A. Keratoacanthoma
stage B. Verrucous precancer
B. Generalized periodontitis, I C. Verrucous leukoplakia
degree D. Papilloma
C. Periodontosis, I degree E. Manganotti's cheilitis
D. Catarrhal gingivitis
E. Localized periodontitis 36. A 20-year-old pregnant
woman with the term of 22 weeks
34. A 32-year-old man was complains of thermal sensitivity of
diagnosed with chronic fibrous her upper front teeth. The sensitivity
pulpitis of tooth 27. Vital extirpation developed one month ago. She was
is chosen as the treatment method. diagnosed with acute initial caries of
An amide anesthetic is to be used for teeth 12, 11, 21, 22, Black class V.
2021 krok 2 dentistry https://krokology.com

The Fedorov-Volodkina hygiene erosions with the remains of erupted


index is 1.8. What treatment tactics vesicles. The mucosa easily sloughs
would be optimal in this case? off. The Nikolsky's sign is positive.
A. Postpone the treatment until 30 What medicines must be prescribed
weeks of gestation for the treatment first?
B. Remineralization therapy and A. Proteolytic enzymes
filling of teeth 12, 11, 21, and 22 B. Antifungal agents
C. Professional teeth cleaning and C. Antibioticsx
filling of teeth 12, 11, 21, and 22 D. Corticosteroids
D. Professional teeth cleaning and E. Non-steroidal anti-inflammatory
remineralization therapy drugs
E. Professional teeth cleaning,
remineralization therapy, and 39. A 62-year-old woman
filling of teeth 12, 11, 21, and 22 complains of intense pain attacks in
the area of her left lower jaw that last
37. A 5-year-old child has all 1-3 minutes. The pain occurs during
temporary teeth in the oral cavity talking, washing her face, and
with tremas between them. The touching the skin. Mandibular X-ray
incisors are in direct contact and their shows no destruction of bone tissue.
cutting edges are worn off. The distal Make the provisional diagnosis:
surfaces of two temporary molars A. Neuralgia of the 3rd branch of the
form a mesio-sagittal step the trigeminal nerve
Zielinsky sign. What period of bitc B. Neuralgia of the 2nd branch of the
formation is it? trigeminal nerve
A. Aging of the temporary bite C. Facial nerve neuritis
B. Early transitional dentition D. Neuritis of the 3rd branch of the
C. Stabilization of the temporary bite trigeminal nerve
D. Late transitional dentition E. Pterygopalatine ganglionitis
E. Formation of the temporary bite
40. A man with hemophilia came
38. A 52-year-old man complains to a dental clinic to have his tooth 36
of unpleasant sensations during extracted. His previous tooth
swallowing and erosions on the oral extraction resulted in a prolonged
mucosa. The disease onset was over bleeding. What tactics should the
a month ago. Objectively, on the doctor choose?
normal mucosa of the soft palate, A. Proper preoperative preparation
cheeks, and gums there are bright red must be provided in the clinic
2021 krok 2 dentistry https://krokology.com

B. To extract the tooth in the clinic E. Schoenherr's vestibular plate


under the supervision of a
hematologist 43. A 63-year-old woman has an
C. - oval neoplasm on the lateral surface
D. To extract the tooth in the of her neck. The neoplasm has a
outpatient department and smooth surface and is approximately
provide the patient with follow-up 3 cm in size. On palpation it is soft
recommendations and dough-like, mobile, and painless.
E. To extract the tooth in the What is the most likely diagnosis in
inpatient department with proper this case?
pre- and postoperative A. Lateral cyst of the neck
preparation B. Adenomax
C. Fibromax
41. A practically healthy Z-year- D. Lymphomax
old child complains of a broken E. Lipoma
crown of the upper right front tooth
and pain in this tooth. Objectively, 44. A 40-year-old man came to a
1/3 of the crown of tooth 11 is dental surgeon for extraction of tooth
missing, the pulp is exposed in one 16. The crown of tooth 16 is retained.
spot, acutely painful, red, and bleeds What forceps should be used to
when probed. Percussion of the tooth extract tooth 16 in this case?
is slightly painful. The trauma was 2 A. S-shaped forceps
hours ago. What is the optimal B. Bayonet crown forceps
treatment method in this case? C. S-shaped left-sided forceps
A. Vital extirpation D. Straight forceps
B. Devital extirpation E. E.S-shaped right-sided forceps
C. Devital amputation
D. Biological method 45. The parents of a 7-year-old
E. Vital amputation child complain that their child for the
last 2 days had body temperature of
42. What prosthodontic appliance 38.8 ° C and sore throat. Objectively,
has the mechanical type of action? catarrhal stomatitis is observed on the
A. Case's obturator gums, cheeks, lips, and on the floor
B. Expansion plate with Coffin's of the oral cavity. The submandibular
spring lymph nodes are enlarged and
C. Bynin's appliance painful. Bright-red catarrhal
D. Frankel's functional regulator inflammation is present on the soft
2021 krok 2 dentistry https://krokology.com

palate, tonsils, and palatine arches. complication has likely occurred in


The tongue is edematous, red, and this casc?
dry. The skin of the face is A. Ischemia
hyperemic, except for the white B. Hematoma
nasolabial triangle. Make the C. Quincke's edema
provisional diagnosis: D. Abscess
A. Measles E. Emphysema
B. Pertussis
C. Chickenpox 48. A non-removable porcelain-
D. Infectious mononucleosis fused-to metal dental bridge is being
E. Scarlet fever made for a 35 year-old woman. Two-
layer impressions were obtained
46. On the second day after the using the «Silaflex» > silicone
fixation of partial laminar dentures, a material. The impression were sent
man developed complaints of a for disinfection. What antiseptic
burning sensation and an unpleasant should be used to process the
taste in his mouth, when using the impressions?
dentures. Examination revealed A. 1.5% sodium bicarbonate solution
hyperemic mucosa and B. B.0.5% hypochlorite sodium
hypersalivation. Prior to making new solution
dentures, he was using the old ones C. 3% chloramine solution
for 5 years and had no such D. 0.5% hydrogen peroxide solution
complaints. What is the likely cause E. 70% alcohol solution
of this phenomenon?
A. Allergic reaction to the 49. A child is 5 years old. There
components of the base plastic are complaints of an aesthetical
B. Incorrect usage of the dentures defect. An extraoral examination
C. The presence of a residual shows that the face is symmetrical,
monomer in the denture base but disproportionate because of
D. Poor quality of the base plastic reduced lower third of the face. In the
E. Improper oral hygiene oral cavity, teeth 55, 54, 64, and 65
are missing. What dental device is
47. When receiving tuberal necessary for this patient?
anesthesia intraorally, a man A. Dental bridge
developed a rapidly progressing B. Andresen-Haupl appliance
edema of the left cheek that soon C. Removable partial denture for the
spread to the temporal region. What upper jaw
2021 krok 2 dentistry https://krokology.com

D. Clasp-retained (bugel) denture hyperemic, edematous, and painful


E. Frankel appliance when palpated. What is the most
likely diagnosis in this case?
50. A 65-year-old man was A. Acute serous periostitis
diagnosed with a bilateral B. Pericoronitis
uncomplicated mandibular fracture C. Acute odontogenic osteomyelitis
in the mental region. The patient uses D. Acute serous alveolitis
removable complete dentures. E. Chronic destructive osteomyelitis
Choose the long-term splint for this
patient: 53. A patient is undergoing
A. Port splint treatment for chronic granulating
B. Vasiliev splint periodontitis of tooth 26. The root
C. Weber splint canals were irrigated with an
D. The patient's own dentures antiseptic that can penetrate into the
E. Vankevich splint structures of biofilms, destroy them,
and dissolve organic tissues. What
51. A 37-year-old woman came to antiseptic was used by the doctor?
a dentist to have her tooth 25 A. Hydrogen peroxide solution
extracted. What type of anesthesia B. Chlorhexidine solution
should be used in this case? C. Physiological salinex
A. Unilateral infraorbital and incisor D. D Furacilin (nitrofurazone)
anesthesia solution
B. Unilateral infraorbital and palatal E. Hypochlorite sodium solution
anesthesia
C. Bilateral central anesthesia 54. A 35-year-old woman came to
D. Unilateral tuberal and palatal the dental office complaining of teeth
anesthesia mobility. Objectively, her teeth 42,
E. Unilateral tuberal, infraorbital, 41, 31, 32 have the II degree mobility
and palatal anesthesia and teeth 43, 33. have the I degree
mobility. The tooth crowns are intact.
52. A 44-year-old man complains The woman presents with a deep bite.
of a constant dull pain in the area of X-ray shows resorption of the
his tooth 37 that was extracted two interdental septa by 1/3 of the root
days ago. The pain intensifies during height. What splint construction is
eating. Objectively, the socket of the advisable in this case for temporary
extracted tooth has a gray coating. use?
Mucosa around the socket is A. Cap splint
2021 krok 2 dentistry https://krokology.com

B. Made of armored composite C. Obtaining the functional


C. Consisting of equator crowns impressions
D. Clasp-retained (bugel) denture D. Making of wax bases with bite
E. Consisting of semi-crowns blocks
E. Measuring the centric relation
55. A 35-year-old man complains between the jaws
of missing teeth on his lower jaw.
Objectively, teeth 36, 37, and, 38 are 57. How to separate teeth for
missing. Teeth 34 and 35 are intact. orthodontic crowns?
What type of dentition restoration A. A With separating discs
would be optimal in this case? B. With a diamond-tipped bore
A. Porcelain-fused-to-metal C. No separation is needed
cantilever denture with abutment D. With elastics
crowns on teeth 34% and 35 E. With volcanic discs
B. Porcelain-fused-to-metal crowns
with abutment implants in place 58. A 25-year-old man complains
of teeth 36 and 37 of a dull pain in the area of his lower
C. Removable partial laminar right second molar. This tooth was
denture for / the lower jaw treated for pulpitis one day ago. The
D. Adhesive dental bridge carious cavity has a temporary
E. Clasp (bugel) denture with a filling. Objectively, X ray of tooth 47
continuous clasp (clammer) for shows a perforation in the area of root
the lower jaw bifurcation within the bone tissue.
The root canals are filled and there
56. Removable complete dentures are no pathologic changes in the
are being made for the both jaws of a periapical tissues. The depth of
68 year-old woman. Objectively, the dentogingival probing in the
alveolar processes are slightly bifurcation area is normal. What
atrophied, the mucosa is moderately treatment method would be optimal
pliant. Anatomical impressions are in this case?
obtained. What is the next stage of A. Crown lengthening
denture-making in this case? B. Tooth extraction
A. Making of individual impression C. Perforation closure with MTA
trays D. Perforation closure with glass-
B. Checking the denture ionomer cement
construction E. Tooth replantation
2021 krok 2 dentistry https://krokology.com

59. A 48-year-old man complains E. Make a Port splint for the fixation
of a dull pain in the area of his tooth of the lower jaw
48 that intensifies when biting,
worsening of his general condition, 61. During preparation of a 38-
and high body temperature of 38 ° C. year-old man for denture-making, a
Objectively, the retromolar mucosa is perforation of the pulp chamber floor
edematous and hyperemic. The distal was detected in the bifurcation area
surface of tooth 48 is covered with a of tooth 46. Objectively, tooth 46 is
mucosal flap, palpation of which immobile and has no periodontal
provokes a sharp pain and discharge pockets. X-ray shows completely
of purulent exudate. What is the most filled root canals of tooth 46 without
likely diagnosis in this case? pathologic changes in their periapical
A. Acute purulent periodontitis area. In the bifurcation area of tooth
B. Phlegmon of the submandibular 46, the apex of the interradicular
region septum is destroyed. What type of
C. Acute purulent pericoronitis procedure is recommended in this
D. - case to retain the tooth?
E. Acute purulent periostitis A. Hemisection
B. Replantation
60. A 32-year-old man has been C. Amputation
brought into the maxillofacial D. Resection
department of a clinic. He was E. Bicuspidization (bisection)
diagnosed with an open displaced
bilateral fracture of the lower jaw in 62. A child born with cleft palate
the area of the gonial angle. X-ray needs an obturator. What is the
shows a large diastasis between the optimal age for installing a palatal
fragments and muscle interposition. obturator in this case?
What main treatment technique A. 3-4 years
should be chosen for this patient? B. The first days after birth
A. Bilateral osteosynthesis of the C. 1.5 years
lower jaw D. 5-6 years
B. B. Make a Vankevich splint for E. 1 year
the fixation of the lower jaw
C. Apply a smooth splint-bracket 63. During intraoral examination
D. Apply a splint with wire loops to the dentist suspected that the patient
the lower jaw has syphilis. What should the dentist
do in this case?
2021 krok 2 dentistry https://krokology.com

A. Continue the examination and nocturnal pain attacks in his tooth 25.
start the teeth preparation The pain irradiates along the
B. Decline to provide dental services branches of the trigeminal nerve and
for this patient alternates with brief painless
C. Inform the patient of the intervals. Objectively, on the medial
suspected diagnosis and end the surface of tooth 25 there is a deep
visit carious cavity within the parapulpar
D. Complete the examination and dentin. Probing of the cavity floor
refer the patient for necessary and percussion are painful. Thermal
tests stimulation of tooth 25 provokes a
E. Obtain the impressions to study pain attack. What is the most likely
the diagnostic dental casts diagnosis?
A. Acute purulent pulpitis.
64. A 48-year-old man came to a B. Trigeminal neurolagia
clinic for oral cavity sanation. He has C. Acute diffuse pulpitis
a history of mild diabetes mellitus. D. Acute purulent periodontitis
Objective examination reveals E. Acute deep caries
whitish papulae against the
background of normal buccal 66. An 8-year-old boy complains
mucosa. The papulae are slightly of an acute pain in his lower left jaw,
raised above the mucosa. They are general weakness, high body
located symmetrically, temperature of 38.6° C, and problems
predominantly in the olar space, and with opening the mouth and
form a lace-like pattern. On the upper swallowing. Objectively, the face is
jaw there are two soldered dental asymmetrical because of edema in
bridges, tooth 37 has an amalgam the submandibular region and in the
filling, tooth 36 is covered with a cast lower third of the left cheek. Mucosa
metal crown. What is the most likely in the area of teeth 73, 74, and 75 is
diagnosis? edematous on the both sides of the
A. Chronic atrophic candidiasis alveolar process. The Pulsief teeth
B. Secondary syphilis are mobile, percussion is positive, the
C. Lichen ruber planus Vincent's sign is positive. What is the
D. Leukoplakia most likely diagnosis in this case?
E. Lupus erythematosus A. Acute odontogenic osteomyelitis
B. Inflammatory infiltration of the
65. For 2 days a man has been left cheek
suffering from spontaneous
2021 krok 2 dentistry https://krokology.com

C. Exacerbation of chronic Objectively, her nasolabial folds are


periodontitis smoothed out, her masticatory
D. Odontogenic phlegmon of the muscles are strained. What tactics
submandibular region shoud be chosen by a dental
E. Non-odontogenic acute purulent orthopedist in this case?
lymphadenitis A. Make new dentures
B. Correct the occlusal surface of the
67. A 16-year-old girl came to a lower denture
dentist complaining of darkened C. Correct the occlusal surfaces of
crowns of her teeth 11 and 12. the both dentures
According to the girl, about one year D. Correct the occlusal surface of the
ago she had a sports trauma. upper denture
Objectively, the crowns of teeth 11 E. Make a new denture for the upper
and 12 are dark gray, intact, painless jaw x
on percussion. The mucosa of the
alveolar process is unchanged. X-ray 69. A 9-year-old boy complains of
shows round areas of bone tissue a spontaneous pulsing and irradiating
destruction in the region of the root pain in his lower right teeth that
apices of teeth 11 and 12. These bone slightly abates when cold is applied.
lesions are 0.3-0.4 cm in diameter Objectively, tooth 46 has a deep
and have clear margins. What tactics carious cavity with soft floor, located
would you choose? within the parapulpar dentin. Probing
A. Conservative treatment of teeth of the cavity floor is acutely painful.
11 and 12 Percussion of the tooth is painful.
B. Conservative surgical treatment The boy was diagnosed with acute
of teeth 11 and 12 purulent pulpitis of tooth 46. What
C. Removal of teeth 11 and 12 treatment method should be chosen
D. No treatment is needed in this case?
E. Monitoring the condition of teeth A. A Vital pulp amputation
11 and 12 for six months B. Biological treatment
C. Devital pulp extirpation
68. A 60-year-old woman D. Devital pulp amputation
complains of pain in her ears and in E. Vital pulp extirpation
the area of her temporomandibular
joint and her teeth clacking when she 70. A dentist treats chronic
talks. Her removable complete gangrenous pulpitis of tooth 47 in a
dentures were made two weeks ago. 28-year-old man. What results of
2021 krok 2 dentistry https://krokology.com

electric pulp testing confirmed the for proper functioning of this type of
diagnosis of chronic gangrenous dental crown?
pulpitis? A. Conical shape, convergence angle
of 15-20 degrees, single-plane
A. Electric pulp testing-110 preparation, circular ledge
microamperes B Conical shape, convergence angle
B. Electric pulp testing-2-6 of 10-15 degrees, single-plane
microamperes preparation, circular ledge
C. Electric pulp testing -20 C. Cylindrical shape, two-plane
microamperes preparation, circular ledge
D. Electric pulp testing-60 D. Conical shape, convergence angle
microamperes of 5-7 degrees, two-plane
E. Electric pulp testing-10-12 preparation, circular ledge
microamperes E. Cylindrical shape, single-plane
preparation, no ledge
71. A 45-year-old man complains
of painful and loose front teeth on his 73. A clasp (bugel) denture with
lower jaw. Objectively, teeth 41, 42, clammer fixation is being made for a
31, and 32 have the III degree of partially man. edentulous (Kennedy
mobility. Plans are made to remove class I) Impressions were obtained,
these teeth and make a direct denture casts were made, and the centric
for this man. When should a direct relation of the jaws was determined
denture be installed after the teeth and fixed. What laboratory stage of
extraction? the denture-making is the next one?
A. 7 days after the teeth extraction A. Marking the denture frame
B. On the day of the teeth extraction B. Duplication of the cast
C. 1-2 days after the teeth extraction C. Obtaining a refractory cast
D. 14 days after the teeth extraction D. Installation of the gating system
E. 3-4 days after the teeth extraction E. Studying the working cast in a
parallelometer
72. A 40-year-old man complains
of constant decementation of the 74. Removable partial dentures for
porcelain fused-to-metal crown on both jaws are being made for a 60-
tooth 11. The crown was made for year-old man. The patient has a
him one year ago. How must the history of chronic angular stomatitis.
stump of the prepared tooth look like When making his dentures, special
attention should be paid to the:
2021 krok 2 dentistry https://krokology.com

A. Choice of impression material gland produces purulent exudate


B. Restoration of the interalveolar from the excretory duct. Make the
height provisional diagnosis:
C. Final processing of the denture A. Herzenberg parotitis
base B. Acute non-epidemic parotitis
D. Choice of material for the denture C. Phlegmon of the parotid-
base masseteric region
E. Oral cavity sanation D. Acute mumps
E. Phlegmon of the submassseteric
75. A 37-year-old woman region
complains of a tumor on her lower lip
on the surface that faces the oral 77. During or immediately after an
cavity. The tumor appeared injection, certain local complications
approximately 2 weeks ago, can develop. What is not one of those
gradually enlarges, and makes lip complications?
movement problematic. Objectively, A. Dermal ischemia
within the lower lip there is a B. Diplopia
circumscribed round elastic painless C. Damage to a blood vessel caused
induration fused to the mucosa. The by the needle
mucosa over the induration is A. Mucosal necrosis
transparent, while around the D. Functional paralysis or paresis of
induratuion it remains unchanged. facial muscles
The regional lymph nodes are not
enlarged. What is the most likely 78. A regular check-up of a 7-
diagnosis in this case? year-old girl detected smooth chalk-
A. Retention cyst of the lip like spots on the vestibular surface of
B. Fibroma of the lip her upper central incisors, closer to
C. Lipoma of the lip their cutting edge. The surface of the
D. Lip abscess spots does not stain with methylene
E. Papilloma of the lip blue. At the age of 4, the child had a
trauma of teeth 51 and 61. What is the
76. On the 7 day after an most likely diagnosis in this case?
abdominal surgery, a 69-year-old A. Local hypoplasia
man has developed pain and swelling B. Systemic hypoplasia
in the area of his left parotid gland. C. Acute superficial caries
His body temperature is 39 ° C, his D. Hyperplasia
mouth is dry. Massage of the salivary E. Caries at the spot stage
2021 krok 2 dentistry https://krokology.com

D. Osteitis deformans
79. After the complex E. Chronic osteomyelitis,
examination of a 44 year-old man, destructive productive form
the dentist made a diagnosis of
generalized periodontitis, II degree, 81. During obtaining a full
exacerbated progression. Lately the anatomical impression of the upper
patient has been noticing a jaw with alginate mass, a 56-year-old
deterioration of his general condition, man developed acute dyspnea, pale
thirst, dry mouth, and itching skin. skin, increased sweating, cyanosis,
What additional examination needs and wheezing respirations. What
to be performed first in this case? urgent condition developed in this
A. Vitamin C levels in blood case?
B. Complete comprehensive blood A. Aspiration asphyxia
count B. Bronchial asthma attack
C. Blood sugar levels C. Pneumothorax
D. Blood iron levels D. Acute bronchitis
E. Immunologic blood test E. Overactive gag reflex

80. An 11-year-old child has a 82. A 26-year-old woman, who


thickening in the frontal region of the works with chemical substances at a
lower jaw that appeared after the factory, came to a dentist with
child fell down. The trauma occurred complaints of teeth discoloration and
one month ago. Objectively, on the extreme sensitivity to cold, sweet,
lower jaw, in the area of the and sour foods in her upper front
mucogingival junction, there is a teeth. Objectively, on the vestibular
circumscribed dense spot with surface of teeth 13, 12, 11, 21, 22,
smooth surface that is slightly painful and 23, at their equator, there are
on palpation. The teeth in the white spots that spread to the cutting
pathologic focus are intact and vital. edges. Probing detects rough and
X-ray detects an osteoporotic focus fragile enamel in the affected areas.
with thickened periosteum in the Vital staining is positive. Make the
frontal region of the lower jaw. What clinical diagnosis:
is the most likely diagnosis in this A. Acute initial caries
case? B. Systemic enamel hypoplasia
A. Chronic granulomatous C. Erosion of hard dental tissues
B. Chronic osteomyelitis, D. Hyperesthesia of hard dental
C. Rarefying periostitis tissues
2021 krok 2 dentistry https://krokology.com

E. Necrosis of hard dental tissues E. Model wax acute

83. A 35-year-old woman came to 85. A 42-year-old man came to a


a dental surgeon complaining of a dental surgeon for oral sanation.
slight deformity of the mandibular After application of anesthesia, the
body on the right. patient suddenly developed nausea,
Orthopantomogram shows a round weakness, constricting sensation
homogeneous shadow with clear behind the sternum, and cardiac pain.
margins in the mandibular body. The The patient's skin is pale and covered
density of the shadow is close to that in cold clammy sweat. The patient
of dental tissues. On its periphery, the started vomiting, lost his
neoplasm is surrounded by a consciousness, and developed
transparent zone 1 mm wide. What is seizures. The patient's pupils are
the most likely diagnosis in this case? unresponsive to the light. The pulse
A. Osteoma is thready, blood pressure cannot be
B. Odontogenic fibroma measured. Make the diagnosis:
C. Odontoma A. Anaphylactic shock
D. Ameloblastoma B. Epileptic seizure
E. Mixoma C. Traumatic shock
D. Collapse
84. A 42-year-old man complains E. Unconsciousness
of problems with chewing.
Objectively, his tooth 26 is missing. 86. A 37-year-old man was
On the medial surface of tooth 27 and prescribed an extraction of tooth 25
distal surface of tooth 25 there are under a local anesthesia. Where
class II carious cavities according to should the doctor be positioned in
Black's classification.It is relation to the patient during the
recommended to make a full-cast extraction of tooth 25?
dental bridge with 27 and 25 as the A. Directly in front of the patient
abutment teeth. What material is used B. Behind and to the right of the
to assess whether the dental bridge patient
framework tightly adheres to the A. In front and to the right of the
abutment teeth? patient
A. - C. Behind and to the left of the
B. Corrective silicone mass patient
C. Articulating paper D. In front and to the left of the
D. Basic silicone mass patient
2021 krok 2 dentistry https://krokology.com

smooth contours of the articular


87. A 44-year-old man came to a surfaces, the joint space is normal.
dental polyclinic to have a denture What is the most likely diagnosis in
made for him. He complains of this case?
problematic chewing and an A. Neuromuscular joint syndromet
aesthetical defect. Objectively, his B. Acute post-traumatic arthritis
teeth 14, 15, and 22 are missing. C. Fibrous ankylosis of the joint
According to Agape, the loss of D. Deforming arthrosis
masticatory efficiency in this case is: E. Rheumatoid arthritis
A. 9%
B. 20% 90. A 60-year-old man came to a
C. 16% dentist two days after the fixation of
D. 12% a removable partial denture. He
E. 18% complains of an ulcer that appeared
on his oral mucosa. What has caused
88. Removable complete dentures the mucosal lesion near the edge of
are being made for a 70-year-old man the denture base?
with completely edentulous jaws and A. Mistakes when fitting the
marked progenia. What artificial artificial teeth
teeth are not placed in such dentures? B. Poorly fixed denture
A. Second molars on the upper jaw C. Mechanical trauma of the mucosa
B. Second premolars on the upper with the edge of the denture base
jaw D. Eating coarse food
C. Second molars on the lower jaw E. Incorrectly placed clasps
D. First premolars on the lower jaw
E. Second premolars on the lower 91. The anatomo-physiological
jaw method was applied to determine the
interalveolar height of a 65-year-old
89. A 52-year-old man complains man with completely edentulous
of pain and clicking in the area of his jaws. How does the distance between
left temporomandibular joint. the control points on the chin and at
Objectively, the face is symmetrical, the nasal base differ in this case from
palpation of the medial and lateral the physiological resting state of the
pterygoid muscles is painful on the lower jaw?
left. The mouth opening is limited to A. It is 2-3 mm longer
1.5 cm between the cutting edges of B. It is 5-6 mm shorter
the central incisors. X-ray shows C. It is 5-6 mm longer
2021 krok 2 dentistry https://krokology.com

D. It is 2-3 mm shorter there is a carious cavity with a wide


E. The distance is the same opening. The cavity is located within
the parapulpar dentin and filled with
92. An 8-year-old girl complains dense pigmented dentin. The carious
of dry lips and a painful burning cavity does not communicate with
sensation in them. She has a history the dental cavity. Probing of the
of food allergies. Objectively, there is carious cavity walls and floor, cold
a slight edema and hyperemia of the stimulus, and percussion cause no
skin of her lips. The vermillion pain. Electric pulp testing-12
border is infiltrated and has fissures, microamperes. What is the most
scabs, and scales. The surface of the likely diagnosis?
lips is dry. In the angles of the mouth A. Chronic fibrous periodontitis
there are infiltrations, maceration, B. Chronic deep caries
and fissures. What is the most likely C. Chronic median caries
diagnosis in this case? D. Chronic gangrenous pulpitis
A. Eczematous cheilitis E. Chronic fibrous pulpitis
B. Meteorological cheilitis
C. Glandular cheilitis 95. An 18-year-old young man
D. Actinic cheilitis complains of a cavity in his tooth 37
E. Exfoliative cheilitis and pain that occurs when food gets
into this cavity. The tooth was painful
93. A 47-year-old man came to a one year ago but he did not make an
dentist to have a denture made for appointment with the doctor back
him. Objectively, his teeth 43, 44, then. Objectively, on the masticatory
and 45 are missing. What class of surface of tooth 37 there is a deep
mandibular dentition defect is it carious cavity that communicates
according to Kennedy? with the pulp chamber. Overgrown
A. - granulation tissue protrudes from the
B. 1 perforation into the pulp chamber and
C. 2 fills practically the whole carious
D. 3 cavity. Probing of the granulations
E. 4 provokes bleeding and slight pain.
Percussion is painless. Mucosa in the
94. A 28-year-old woman came to projection of the roots of tooth 37 is
a dentist with complaints of a cavity pale pink and has no pathologic
in her upper right tooth. Objectively, changes. Electric pulp testing-60
on the masticatory surface of tooth 16
2021 krok 2 dentistry https://krokology.com

microamperes. What is the most A. Lipoma of the submandibular


likely diagnosis? region
A. Hypertrophic papillitis B. Lymphadenitis of the
B. Chronic granulating periodontitis submandibular region
C. Chronic hypertrophic pulpitis C. Sialolithiasis
D. Granulations from the area of root D. Ranula of the sublingual salivary
bifurcation gland
E. Chronic fibrous pulpitis E. Mandibular osteoma

96. A 42-year-old man complains 98. Epidemiological examination


of periodic pain in his tooth 48. of certain age groups to study the
Examination shows that tooth 48 is prevalence of the signs that indicate
partially erupted with its medial periodontal diseases and to assess the
cusps, its mucosa is pale pink and need for treatment uses the following
painless on palpation, the mouth index:
opening is fully unrestricted. X-ray A. IR (Ramfjord)
shows tooth 48 to be located medially B. CPITN (WHO)
to tooth 47. Choose the surgical C. OHI-S (Green-Vermillion)
treatment: D. PI (Russel)
A. Operculum removal E. PMA (Parma)
B. Typical extraction of tooth 48
C. Bicuspidization (bisection) 99. A patient needs a removable
D. Atypical extraction of tooth 48 full laminar denture for the upper
E. Operculum dissection jaw. Objectively, on the mucosa of
the denture bed there are numerous
97. A 47-year-old man complains dense papillomas of varying size.
of pain and swelling in his left What tactics should the dentist
submandibular region and pain choose?
during eating. Objectively, the mouth A. Make a 3D-model of the denture
opening is unrestricted, mucosa of base
the left sublingual fold is hyperemic, B. Make the denture base from metal
the excretory duct of the C. Remove papillomas and make a
submandibular salivary gland denture with a double-layered
produces a small amount of turbid base lined with elastic material
saliva. X-ray of the mouth floor D. Removable dentures are
shows an oval shadow 1 cm in size. contraindicated in this case
Make the diagnosis:
2021 krok 2 dentistry https://krokology.com

E. Reduce the area of the denture


base 102. A 27-year-old woman needs a
denture. Objectively, her tooth 15 is
100. A 35-year-old man has missing, while teeth 14 and 16 are
developed edema and hyperemia that intact. What denture is preferable in
first appeared in the area of his wings this case?
of the nose and eventually spread to A. Adhesive dental bridge
the upper lip and checks. B. Porcelain-fused-to-metal dental
Examination detected a fissure in the bridge
area of the lower external edge of the C. Clasp (bugel) denture
left nostril. The skin there is taut, D. Swaged dental bridge
dense, purple-blue, with smoothed E. Partial removable laminar denture
out pattern. The local temperature is
elevated. Palpation is painful. The 103. A 55-year-old woman needs
affected area has clear irregularly- removable implant-retained dentures
shaped margins. Make the for both jaws. It is necessary to
provisional diagnosis: choose the type, the number, and the
A. Allergic dermatitis placement of implants. What
B. Lymphangitis additional examination method will
C. Odontogenic subcutaneous be the most efficient in making this
granuloma decision?
D. Eczema A. Computed tomography
E. Erysipelas B. Study of diagnostic dental casts
C. Ultrasound densitometry
101. A 39-year-old woman D. Spot X-ray
provisionally diagnosed with E. Occlusography
generalized periodontitis, I degree,
chronic progression. X-ray shows 104. A was 43-year-old woman
uneven resorption of interdental provisionally diagnosed with
septa by 1/3–1/4 of root length. What generalized periodontitis. What
surgical method must be used in this examination will be the most
case as a part of complex treatment? informative for the assessment of the
A. Gingivotomy severity of her condition?
B. Closed curettage A. Measuring the depth of the
C. Flap surgery periodontal pockets
D. Gingivo-osteoplasty B. Measuring the degree of tooth
E. Open curettage mobility
2021 krok 2 dentistry https://krokology.com

C. Determining the periodontal 64, 74, 75, and 85 there are carious
index cavities within the softened mantle
D. Panoramic X-ray dentin. The child is anxious and
E. Schiller-Pisarev test misbehaves. What treatment tactics
would be optimal in this case?
105. A 7-year-old boy is diagnosed A. Preventive filling
with epidemic parotitis (mumps). B. ART technique
Name the most likely complication of C. Impregnation treatment
this disease: D. Deep fluoridation
A. Orchitis E. Invasive sealing
B. Dermatitis
C. Pneumonia 108. A 48-year-old man, who for
D. Cholecystitis several years already has been
E. Colitis suffering from enterocolitis,
complains of painful lesions that
106. The parents of a 5-year-old from time to time appear in different
child explain that lately their child places of his oral mucosa. Objective
has started chewing only on one side examination of the mucosa reveals
and refusing hard foods. Objectively, isolated areas of epithelial damage.
tooth 74 has a carious cavity and They are under 5 mm in diameter,
there are dental. deposits on teeth 73, have a regular oval shape, hyperemic
74, and 75. Probing of the cavity border, and yellow white coating.
floor in tooth 74 is painful and Their palpation is painful. What is the
provokes a slight bleeding in the area most likely diagnosis?
where the cavity communicates with A. Allergic stomatitis
the pulp chamber. Make the B. Secondary syphilis
diagnosis: C. Chronic recurrent herpes
A. Chronic hypertrophic pulpitis D. Chronic recurrent aphthous
B. Chronic fibrous pulpitis stomatitis
C. Acute median caries E. Erythema multiforme exudativum
D. Chronic gangrenous pulpitis
E. Acute deep caries 109. A 62-year-old man came to a
dentist to have his tooth 44 extracted.
107. Parents of a 5-year-old child During examination he suddenly fell
came to the dentist for sanation of the unconscious. His breathing and pulse
child's oral cavity. Objectively, on are slow, his head is bowed, his arms
the masticatory surfaces of teeth 54, are flexed in the elbows and wrists,
2021 krok 2 dentistry https://krokology.com

his legs are stretched out, and his the face to be flattened, the chin is
jaws are clenched tight. The patient displaced to the left. Palpation
developed clonic convulsion and detects diminished excursion of the
started foaming at the mouth. His articular head in the left lower jaw.
pupils are dilated and unresponsive to The mouth opening is reduced to 1
light. 1.5 minutes later, the patient's cm. Make the provisional diagnosis:
muscles relaxed. The patient is A. Exacerbated chronic arthritis of
mentally confused. What is the most the left temporomandibular joint
likely diagnosis in this case? B. Arthrosis of the left
A. Epileptic seizure temporomandibular joint
B. Paroxysmal tachycardia attack C. Anterior dislocation of the lower
C. Hypertensive crisis jaw
D. Syncope D. Acute arthritis of the left
E. Acute cerebrovascular accident temporomandibular joint
E. Ankylosis of the left
110. A removable complete denture temporomandibular joint
is being made for the lower jaw of a
50 year-old man. During the fitting of 112. Examination of the oral cavity
an individual impression tray, the of a 12-year-old child revealed a
border in the area of the mylohyoid carious cavity on the medial contact
line needs to be determined. What surface of tooth 46. The cavity is
Herbst test should be performed for located within the mantle dentin, its
this purpose? walls and floor are pigmented and
A. Alternately touching the right and dense. Thermal stimuli provoke no
left cheeks with the tongue response. Preparation of tooth 46 is
B. Stretching the tongue towards the painful in the area of dentinoenamel
tip of the nose junction. The decay-missing-filled
C. Swallowing saliva index is DMF + df = 2. What filling
D. Licking the upper lip material will be optimal in this case?
E. Wide mouth opening A. Silicophosphate cement
B. Glass-ionomer cement
111. A 21-year-old man complains C. Zinc phosphate cementy
of difficult mouth opening and D. Amalgam
visible facial deformation throughout E. Photopolymer composite
the last 2 years. He has a history of
facial trauma several years ago. 113. In the dentist's office, after a
Examination shows the right side of treatment of tooth 26 a 46-year-old
2021 krok 2 dentistry https://krokology.com

woman suddenly developed a pulsing E. Parafunction of the facial muscles


headache, dizziness, ringing in her
ears, tachycardia, and blood pressure 116. A 25-year-old man complains
of 220/160 mm Hg. Make the of brief pain attacks in response to
provisional diagnosis: sweet foods in one of his lower teeth
A. Hypertensive crisis and food retention in a carious cavity.
B. Anaphylactic shock Objectively, on the masticatory
C. Hypoglycemic coma surface of tooth 36 there is a carious
D. Syncope cavity within the parapulpar dentin.
E. Acute cerebrovascular accident The dentin is soft. The carious cavity
does not communicate with the
114. A regular check-up of a 7- dental cavity, probing of its floor is
year-old girl detected chalk-like painful. Percussion of tooth 36 and
spots on the vestibular surface of her palpation of gingival mucosa in the
teeth 11 and 21. The surface of the projection of root apex are painless.
spots is matt, smooth when probed, Thermal stimuli cause pain that
and can be stained with methylene quickly stops after the simulus is
blue. What treatment method would removed. What is the most likely
be rational in this case? diagnosis?
A. Impregnation therapy of teeth 11 A. Pulp hyperemia
and 21 B. Chronic fibrous pulpitis
B. Filling of teeth 11 and 21 C. Acute median caries
C. Filing down the defects of teeth D. Chronic deep caries
11 and 21 E. Acute deep carics
D. Regular check-ups to monitor the
child's condition 117. A 6-year-old child complains
E. Deep fluoridation of teeth 11 and of food retention in a tooth on the
21 lower jaw. Objectively, in tooth 75
on its masticatory surface within
115. A 7-year-old child needs mantle dentin there is a carious cavity
mandibular vestibuloplasty. What with a wide opening. The dentin of
pathology is most often caused by a the cavity walls and floor is dense
shallow vestibule of the mouth? and pigmented. Probing of the
A. Diseases of periodontal tissues carious cavity and percussion of the
B. Crowded front teeth tooth are painless. Thermal stimuli
C. Deep bite provoke no response. Mucosa in the
D. Mandibular growth retardation area of tooth 75 is without pathologic
2021 krok 2 dentistry https://krokology.com

changes. The child belongs to health periodontal pockets. X-ray shows


group I. Make the diagnosis: osteosclerosis of the alveolar process
A. Acute deep caries and disturbed intactness of the lamina
B. Chronic median caries dura on the apices of the interdental
C. Chronic deep caries septa. The patient was diagnosed
D. Chronic fibrous pulpitis with initial periodontosis. What
E. Chronic superficial caries should be prescribed in this case for
local treatment of the systemic dental
118. A 12-year-old child complains hyperesthesia?
of high body temperature of 39.7 ° C, A. Keratoplastics
enlarged lymph nodes, sore throat, B. Antiseptics
runny nose, and cough. Objectively, C. Desensitizers
acute catarrhal stomatitis is observed D. Keratolytics
in the oral cavity, some of the E. Anesthetics
palatine tonsils have a yellow gray
coating that crumbles and can be 120. A 2-year-old girl has a bright
easily removed. The cervical, red neoplasm 1x1.5 cm in size on her
occipital, and submandibular lymph upper lip. The neoplasm does not
nodes are enlarged, mobile, and protrude above the mucosa and
slightly painful. Laboratory blood becomes paler, when pressed. The
testing shows leuko-, lympho-, and regional lymph nodes have no
monocytosis and atypical pathology. The results of blood and
mononuclear cells. What is the urine analysis are normal for her age.
causative agent of this disease? Make the provisional diagnosis:
A. Loeffler bacillus A. Systemic hemangiomatosis
B. Hemolytic streptococcus B. Cavernous hemangioma
C. Coxsackievirus C. Capillary lymphangioma
D. Herpes simplex virus D. Cavernous lymphangioma
E. Epstein-Barr virus E. Capillary hemangioma

119. A 47-year-old woman 121. A 14-year-old girl complains


complains of an itching sensation in of a pulsing pain in her upper left
her gums and increased teeth teeth that lasts for 3 days already and
sensitivity to thermal stimuli. sharply increases on biting.
Objectively, the roots of most of her Objectively, her tooth 26 has a deep
teeth are exposed to 1/3 of their carious cavity that does not
length, the gums are pale pink, no communicate with the dental cavity.
2021 krok 2 dentistry https://krokology.com

Probing of the cavity floor is 123. A 2-year-old child received a


painless. Vertical and horizontal dental trauma. Objectively, the
percussion is sharply painful. The crowns of teeth 51 and 61 are shorter
mucosa in the area of tooth 26 is than the crowns of the adjacent teeth
hyperemic. What is the provisional by 1/3. The mucosa in the area of
diagnosis in this case? teeth 51 and 61 is hyperemic and
A. Acute serous periodontitis edematous. X-ray shows no
B. Pulpitis complicated with periodontal fissure in the apical area
periodontitis of the roots of teeth 51 and 61. What
C. Acute purulent periodontitis treatment tactics would be optimal in
D. Acute purulent pulpitis this case?
E. Acute diffuse pulpitis A. Ligature splinting
B. Reimplantation
122. A man with an occupational C. Extraction of teeth 51, 61
face trauma on the right has been D. Reposition of teeth 51, 61
brought into the first-aid center. The E. Regular check-ups to monitor the
trauma was received approximately 3 child's condition
hours ago, the man is conscious.
Objectively, on his right cheek there 124. The parents of a 6-year-old
is an irregularly-shaped wound 5x3 child complain that their child for the
cm in size with uneven edges. Parting last three days had body temperature
of the wound edges exposes of 39 ° C, weakness, loss of appetite,
immobile teeth. In the oral cavity, and pain during swallowing.
there is a breach in the buccal Objectively, the mucosa of the
mucosa. The bite is not affected. tonsils, palatine arches, retromolar
What type of surgical debridement is space, and posterior and lateral
necessary for this patient? pharyngeal walls is swollen and has a
A. Early primary surgical dirty-gray coating that is tightly
debridement attached to the underlying tissues.
B. - The coating is very difficult to
C. Late primary surgical remove. When removed, it exposes a
debridement bleeding surface. The submandibular
D. Secondary surgical debridement lymph nodes are enlarged and
E. Delayed primary surgical painful. What is the most likely
debridement provisional diagnosis?
A. Scarlet fever
B. Measles
2021 krok 2 dentistry https://krokology.com

C. Infectious mononucleosis D. Chronic median caries


D. Acute pseudomembranous E. Acute deep caries
candidiasis
E. Diphtheria 127. During dental procedures, a 29
year-old man after receiving
125. After a tooth extraction, the anesthesia suddenly developed a
patient developed a hemorrhage from dense non-itching skin edema in the
the socket of the extracted tooth. area of his face and neck. The edema
What tactics should the dentist was accompanied by cough and
choose in this case? wheezing. What is the most likely
A. Insert a gauze pack soaked in 3% diagnosis in this case?
hydrogen peroxide solution A. Anaphylactic shock
B. Place sutures into the soft tissues B. Urticaria
C. Administer 5-10 ml of 10% C. Bronchial asthma attack
calcium chloride solution D. Contact dermatitis
intravenously E. Quincke's edema
D. Tightly pack the tooth socket with
a strip of iodoform gauze 128. A 35-year-old man undergoes
E. Administer vicasol (menadione) treatment for chronic fibrous pulpitis
solution intramuscularly of tooth 25. The canal is to be filled
using the warm gutta-percha vertical
126. A 21-year-old woman condensation technique. What
complains of short-term pain attacks instrument is needed for gutta-percha
in her lower left tooth that are condensation?
provoked by thermal stimuli and A. K-file
quickly stop after the stimulus is B. Root needle
removed. On the masticatory surface C. Plugger
of tooth 47, examination reveals a D. Spreader
carious cavity with a narrow entrance E. Endodontic probe
that is filled with light softened
dentin. Probing of the cavity floor 129. A 27-year-old woman came to
and percussion of tooth 47 are the dentist for the purpose of oral
painless. Probing of the cavity walls cavity sanation. After anesthesia, she
is painful. Make the diagnosis: started complaining of dizziness,
A. Chronic deep caries ringing in her ears, dimming vision,
B. Acute median caries and general weakness. Objectively,
C. Chronic superficial caries her skin is pale and covered in cold
2021 krok 2 dentistry https://krokology.com

sweat. Her pulse is 94/min., of low fused with the skin. Punctate skin
volume and pressure. Her blood depressions can be observed on its
pressure is 96/60 mm Hg. She had a surface. Make the provisional
brief episode of unconsciousness that diagnosis:
lasted 30 seconds. What urgent A. Frontal lipoma
condition developed in this woman? B. Exophytic form of skin cancer
A. Syncope C. Frontal atheroma
B. Hyperglycemic coma D. Frontal fibroma
C. Acute heart failure E. Frontal keratoacanthoma
D. Collapse
E. Hypertensive crisis 132. When fitting the metallic
frame of a porcelain-fused-to-metal
130. After examination, an 8-year- dental bridge, it was determined that
old child was diagnosed with in the central occlusion the frame is
torsiversion of the lateral maxillary in a contact with the antagonist teeth.
incisors and an arch length deficiency It reaches the ledges of the abutment
caused by macrodontia. To prevent teeth and is 0.3 mm thick. What
the vestibular position of the canines, tactics should a dentist choose?
a Hotz serial extraction was A. Send the frame to the next
performed. In what order were the laboratory stage of the denture-
teeth removed in this case? making
A. First temporary molars, first B. Determine the areas that prevent
premolars, temporary canines normal installation of the dental
B. First temporary molars, bridge.
temporary canines, first premolars C. Obtain an impression with the
C. Temporary canines, first frame fitted into the oral cavity
temporary molars, first premolars D. File down the metallic frame in
D. Second incisors, temporary the areas of its contact with the
canines and first temporary molar antagonist teeth
E. First temporary molars, E. Complete the preparation of the
temporary canines, second abutment teeth and make a
premolars working impression

131. A patient complains of a 133. When pressing plastic dough,


slowly growing neoplasm in the left a dental technician several times
frontal region of his head. The opened and closed the cuvette to
neoplasm is mobile, painless, and check the quantity of the plastic mass
2021 krok 2 dentistry https://krokology.com

in it. These actions may result in the process is mobile. Make the
development of: provisional diagnosis:
A. Residual stresses in the structure A. Le Fort maxillary fracture, type II
of a denture B. Le Fort maxillary fracture, type I
B. Gas porosity C. Maxillary contusion
C. Compression porosity D. Le Fort maxillary fracture, type
D. Granular porosity III
E. Cracks in the plastic E. Zygomatic fracture

134. A 26-year-old man came to a 136. A man complains of muscle


dentist complaining of facial weakness, constipations, dry mouth,
asymmetry that developed because of inability to swallow food, and vision
a painful swelling in the left parotid impairment. The day before, he was
area that appeared 3 days ago. His eating homemade canned fish.
body temperature is 37.2 ° C. Two Objectively, his skin and visible
weeks ago he had a case of acute mucosa are pale; he has diplopia and
respiratory disease. Objectively, in bilateral ptosis. What disease can be
the left parotid area there is a dense suspected in this case?
round infiltration up to 2 cm in size A. Botulism
that is slightly mobile and moderately B. Salmonellosis
painful. The excretory duct of the C. Food poisoning
parotid gland produces clear D. Cholera
transparent saliva. Make the E. Shigellosis
provisional diagnosis:
A. Herzenberg parotitis 137. During preventive
B. Acute purulent parotitis examination, a 20-year-old man
C. Mikulicz disease presents with enamel defects that
D. Pleomorphic adenoma of the look like rough white spots with
parotid gland uneven margins in the vestibular
E. Exacerbation of chronic parotitis precervical area of teeth 11 and 12.
The spots stain with 2% methylene
135. A 45-year-old man complains blue solution. Make the diagnosis:
of a pain in his upper jaw. Two days A. Acute superficial caries of teeth
ago he had a trauma. Objectively, he 11 and 12, Black class V
has a disturbed occlusion. On B. Acute superficial caries of teeth
palpation, the maxillary alveolar 11 and 12, Black class III
2021 krok 2 dentistry https://krokology.com

C. Acute initial caries of teeth 11 and 140. A patient needs endodontic


12, Black class III T treatment of tooth 21. The canal is
D. Acute initial caries of teeth 11 and being processed with manually
12, Black class V operated endodontic instruments.
E. Focal enamel hypoplasia of teeth Name one such instrument made by
11 and 12 means of conical spiral threading
(turning) of a steel wire with a round
138. A man with distal bite cross section (milling):
underwent Eschler-Bittner test for A. K-reamer
the purpose of differential B. H-file
diagnostics. The shape and profile of C. Plugger
his face have notably improved. D. Spreader
What type of distal bite is it, E. K-file
according to test results?
A. Overdeveloped lower jaw 141. A 54-year-old man developed
B. Underdeveloped upper jaw a pustule on the skin of his chin. The
C. Overdeveloped upper jaw pustule rapidly progressed into a hard
D. Underdeveloped lower jaw and sharply painful infiltration 3x3.5
E. Maldevelopments and growth cm in size. The skin over the
abnormalities of the both jaws infiltration is bluish-red. In its center
there are three necrotic foci
139. A 35-year-old man has an surrounding the hair follicles.
open unilateral non-dispalced Palpation detects enlarged and
fracture of the mandibular body on painful submental lymph nodes. The
the left. The fracture line passes body temperature is 38.5 ° C. Make
between the missing teeth 33 and 34. the provisional diagnosis:
The fragments need to be A. Carbuncle
immobilized. What splint would be B. Allergic dermatitis
the most advisable for this purpose? C. Furuncle
A. Tigerstedt splint with a spreader D. Erysipelas
bar E. Festering atheroma
B. Port splint
C. Two-jaw splint with wire loops 142. A maxillofacial surgery
D. Splint with an inclined plane department has received a 34-year-
E. Smooth splint-bracket old man with a frostbite on his face.
Examination shows that the skin in
the affected area is edematous,
2021 krok 2 dentistry https://krokology.com

hyperemic, and cyanotic. No tissue C. Applying the tip of the forceps


necrosis. What degree of the frostbite jaws to the edge of alveolar
is it? process
A. II degree D. Pushing the forceps jaws to the
B. - cementoenamel junction
C. III degree E. Closure of the forceps handles
D. IV degree
E. I degree 145. A 30-year-old man complains
of a carious cavity in a lower left
143. A 50-year-old woman with tooth. Objectively, tooth 34 has a
compensated insulin-dependent deep carious cavity that at one point
diabetes mellitus needs dental communicates with the pulp
prosthetics. She has been in a clinic chamber. Probing of the cavity floor
for several hours already, during is painless, while percussion of the
which she was nervous and skipped tooth is slightly painful. Thermal
her meals. In the process of obtaining stimuli provoke no response. On the
her dental impressions, she suddenly gums, there is a fistula in the
became aggressive, paled, broke out projection of the root apex of tooth
in cold sweat, and fell unconscious. 34. Make the provisional diagnosis:
What substance is used for A. Chronic granulating periodontitis
emergency aid in such clinical B. Chronic fibrous periodontitis
situations? C. Chronic fibrous pulpitis
A. Ammonia solution D. Chronic hypertrophic pulpitis
B. Nitroglycerine E. Chronic deep caries
C. Glucose solution
D. Valocordin 146. A 48-year-old woman
E. Insulin complains of problems with chewing,
caused by missing teeth. Objectively,
144. What manipulation of those her teeth 31, 32, 33, and 41, 42, 43
listed below is not a part of typical have tall crowns and the I degree of
procedure of tooth extraction with mobility. The decision was reached
forceps? to make a clasp denture splint for her.
A. Tooth dislocation and extraction What type of clasps (clammers) will
from the socket ensure the splinting function of this
B. Applying the forceps jaws to the construction?
tooth A. Vestibular
B. Continuous
2021 krok 2 dentistry https://krokology.com

C. Retaining neoplasm yielded a yellowish liquid


D. Abutment with cholesterol crystals. Make the
E. Dentoalveolar provisional diagnosis:
A. Cementoma
147. A 28-year-old man came to a B. Residual cyst
dentist for the purpose of sanation. C. Paradental cyst
Objectively, on the masticatory D. Follicular cyst
surface of tooth 47 there is a deep E. Radicular cyst
carious cavity that communicates
with the dental cavity. Probing of the 149. The jaws of a newborn are in
cavity is painless, the tooth is non- direct relation to each other. The
responsive to thermal stimuli, its baby is full term, healthy, and
percussion is painless. Electric pulp breastfed. What bite can be
testing -108 microamperes. X-ray prognosed in this case?
shows traces of filling material in the A. Deep bite
root canals of tooth 47, the B. Direct bite
periodontal fissure is widened and C. Open bite
deformed in the apical region. Make D. Distal bite
the diagnosis: E. Mesial bite
A. Chronic fibrous periodontitis of
tooth 47 150. A 48-year-old woman
B. Chronic granulating periodontitis complains of itching gums and
of tooth 47 increased teeth sensitivity to thermal
C. Chronic gangrenous pulpitis of and chemical stimuli. Objectively,
tooth 47 the dental cervices are exposed. The
D. Chronic fibrous pulpitis of tooth gums are firm and pale. There is a
47 small amount of supragingival dental
E. Chronic granulomatous plaque. In the area of teeth 13, 14, 24,
periodontitis of tooth 47 and 25 there are cuneiform defects,
probing of which is painful. The
148. X-ray of the lower jaw of a 14- woman was provisionally diagnosed
year old boy shows a round with periodontosis. What X-ray
homogeneous lucency with clear presentation is characteristic of this
margins. This neoplasm is located disease?
within the body of the mandible in A. Horizontal resorption
the area of tooth 36 that was extracted B. Bone pocket formation
7 months ago. A puncture of the C. Vertical resorption
2021 krok 2 dentistry https://krokology.com

D. Uneven resorption of the alveolar


bone
E. Diffuse osteoporosis
Krok 2 Stomatology 2009 1

1. A 45-year-old female patient A. Forming a flat floor


complains about pain caused by thermal B. Forming divergent walls
srtimuli and spontaneous pain in the 26 C. Forming convergent walls
tooth. A week ago this tooth was treated D. Forming a bevel
for pulpitis. Objectively: the 26 tooth is E. Absence of an additional cavity
filled, percussion is painful, thermal sti-
mulus causes slowly intensifying long- 5. Parents of a 9-year-old boy complain
lasting pain. X-ray picture shows that the about permanently open mouth of the
palatine canal is filled by 2/3, buccal canals child. External examination revealed
contain no filling material. What is the elongation of the lower face part, non-
most probable cause of this complication? closure of lips. Examination of the oral
cavity revealed early mixed dentiti-
A. Incomplete pulp extirpation on. Relationship of the first permanent
B. Inadequate canal filling molars is neutral, vertical space is 5 mm.
C. Infection What is the most likely diagnosis?
D. Inflammation in the periodontium
E. Trauma of periodontium tissue A. Open bite
B. Distal occlusion
2. Preventive examination of a 7-year- C. Mesial occlusion
old schoolboy revealed unremovable D. Deep overbite
grey-and-white layerings on the mucous E. Cross bite
membrane of cheek along the line of
teeth joining. Mucous membrane is sli- 6. After removal of dental deposit an 18-
ghtly hyperaemic, painless on palpation. year-old patient underwent preventive
The boy is emotionally unbalanced, bi- examination. It revealed painless chalky
tes his cheeks. What is the most likely di- spots in the precervical area on the vesti-
agnosis? bular surface of the 22 and 41 teeth.
Result of enamel resistance test is 7. What
A. Mild leukoplakia morphological changes are typical for this
B. Chronic recurrernt aphthous stomatitis disease?
C. Chronic candidous stomatitis
D. Lichen ruber planus A. Subsurface enamel demineralization
E. Multiform exudative erythema B. Changes in the mantle dentine
C. Damage of dentinoenamel junction
3. A 48-year-old patient complains about D. Superficial enamel demineralization
itching gums. Objectively: gums are E. Degeneratic changes of odontoblasts
dense, of normal colour; cold water
causes short-term toothache. X-ray pi- 7. A 32-year-old patient complains about
cture shows osteosclerosis of interalveolar gingival haemorrhages during eating and
septa (fine granular bone texture), hei- tooth brushing. It is known from the
ght of interalveolar septa and integrity anamnesis that 2 weeks ago the patient
of compact substance on their tops is got a cast metal crown. Objectively: the 27
unchanged. Which diagnosis is the most tooth has a cast metal crown going beyond
likely? the gingival edge by 1-1,5 mm, touching
causes haemorrhage. Percussion is slightly
A. Initial parodontosis painful. What tactics is the most appropri-
B. Atrophic gingivitis ate in this case?
C. Initial periodontitis
D. I degree periodontitis A. To remove the crown and fabricate a
E. I degree parodontosis splint
B. To administer gargling with Rotocan
4. A 26-year-old patient needs an inlay for C. To open the crown through the masti-
a class V cavity of the 35 tooth. During catory surface
the cavity preparation a prosthodontist D. To give the patient time to get used to
has accidentally opened the pulp. What the prosthesis
might be the most probable cause of this E. To remove the crown and administer
complication? treatment for marginal periodontitis
8. A 50-year-old patient complains about
missing of the 37, 36, 45, 46, 47 teeth.
The 35, 38, 44, 48 teeth are intact and
stable, their crowns exhibit anatomic
form, sufficient height, relative paralleli-
Krok 2 Stomatology 2009 2

sm. Fulcrum line typically goes through A. Method of transfer molding


the 35 and 44 abutment teeth. What fixing B. Direct method of compression molding
element should be used in a clasp denture C. Inverse method of compression molding
supported by the 35, 44 teeth? D. Combined method of compression
molding
A. Acker clasp E. In water under pressure
B. Roach clasp
C. Jackson clasp 12. A 20-year-old patient complains about
D. Telescopic fixation feeling spontaneous undurable localised
E. Bent wire clasp pain in the 26 tooth for one day. Objecti-
vely: the 26 tooth has a deep Black’s class
9. A 25-year-old patient complains about I carious cavity. Dentine of its walls and
a cosmetic defect in the frontal part of floor is unpigmented, softened, carious
his upper jaw on the left. Objectively: the cavity is not communicating with the tooth
23 tooth is pulpless. The filling restores cavity. Probing causes acute pain focused
vestibular-approximal surface of the tooth at a single point, cold test induces long-
crown by 1/3, it is stable, closely fits wi- lasting pain. What method of treatment is
thin cavity walls, its colour doesn’t match the most appropriate in this case?
the colour of the tooth. The tooth itself
has changed its colour, its percussion is A. Biological method
painless. What is the optimal prosthetic B. Vital amputation
construction for this patient? C. Vital extirpation
D. Devital extirpation
A. Metal-ceramic crown E. Devital amputation
B. Elastic crown
C. Metal swaged crown 13. A 42-year-old patient complains
D. Portion crown about acute pain in the region of the
E. Inlay left temporomandibular joint (TMJ) that
irradiates to the ear; headache, general
10. A 12-year-old patient complains about indisposition, impossible mastication
gingival haemorrhage and tooth mobility. and limited mouth opening. Objecti-
He has been suffering from this since the vely: the patient’s face is asymmetric
age of 4. Objectively: gums around all the due to the edema in the region of the
teeth are hyperemic and edematic, bleed left temporomandibular joint. The skin
during instrumental examination. Tooth in this region is hyperemic. The pain is
roots are exposed by 1/3 and covered with made worse by the smallest movements
whitish deposit. II degree tooth mobility of mandible. Palpation of the joint causes
is present. Dentogingival pouches are 4-5 acute pain. Mouth opening is limited
mm deep. External examination revealed down to 15-20 mm. What is the most li-
dryness and thickening of superficial skin kely diagnosis?
layer on the hands and feet, there are also
some cracks. What is the most likely di- A. Acute arthritis of the left TMJ
agnosis? B. Acute purulent parotitis
C. Mandible subluxation
A. Papillon-Lefevre syndrome D. Deforming arthrosis of the left TMJ
B. Hand-Schuller-Christian disease E. Myogenous osteoarthrosis
C. Generalized periodontitis
D. Letterer-Siwe disease 14. A 42-year-old patient complains about
E. Localized periodontitis pain in the submandibular and sublingual
region that is getting worse during eati-
11. A 39-year-old patient needs a clasp ng; body temperature rise up to 37, 6o C.
denture with porcelain teeth for the lower The patient has been suffering from this
jaw. What method should be applied whi- for 2 months. Objectively: along the ri-
le substituting wax with plastic in order ght sublingual plica there is infiltration,
to avoid mold flash that may cause bite hyperaemia, edema of soft tisues, acute
opening? pain on palpation. The duct of the right
submandibular salivary gland discharges
turbid saliva with pus admixtures. What is
the most likely diagnosis?
Krok 2 Stomatology 2009 3

A. Exacerbation of urolithiasis A. Occlusiography


B. Acute purulent lymphadenitis of B. Masticatiography
submandibular region C. Chewing test
C. Adenophlegmon of submandibular D. Roentgenography
region E. Gnathodynamometry
D. Abscess of alveololingual groove
E. Retention cyst of sublingual salivary 18. During extraction of the 47 tooth its
gland distal root was broken halfway along its
length. What tool should be chosen for
15. A 4-year-old girl presents with body extraction of the residual root fragments?
temperature rise, aggravation of general
condition. The symptoms has been A. Left angled elevator
observed for 3 days. Objectively: general B. Broad-beaked forceps
condition is grave, body temperature is C. Close-beaked forceps
38, 6oC, the girl is anxious and pale. She D. Right angled elevator
presents also with halitosis, hyperaemia E. Straight elevator
and edema of gingival mucous membrane
in the region of the 83, 84, 85 teeth on 19. A 31-year-old patient consulted a
both sides from the alveolar process. dentist about teeth sanitation. Objecti-
The mentioned teeth are mobile, their vely: in the precervical region of the 13
percussion causes acute pain; the 84 tooth and 23 teeth there are irregularly-shaped
is filled. What is the most likely diagnosis? defects of hard tissues within deep layers
of enamel. The floor of the defects is
A. Acute odontogenous mandibular rough, their edges are chalky. What tactics
osteomyelitis beginning from the 84 tooth should be chosen as to these defects?
B. Acute sialoadenitis of submandibular
salivary gland A. Preparation and filling
C. Exacerbation of chronic periodontitis of B. Filling with composite material without
the 84 tooth preparation
D. Suppuration of the radiculodental C. Remineralizing therapy
mandibular cyst beginning from the 84 D. Grinding of the defects
tooth E. Silvering of the defects
E. Acute odontogenous mandibular peri- 20. A 51-year-old female patient complai-
ostitis beginning from the 84 tooth ns about food sticking in a right inferi-
16. A 38-year-old female patient came to or tooth. Objectively: distal masticatory
a hospital and complained about a wound surface of the 45 tooth has a deep cari-
on her left cheek. She got this trauma 16- ous cavity filled with dense pigmented
18 hours ago, didn’t loose consciousness. dentin that doesn’t communicate with the
Objectively: a 3 cm long injury of skin, tooth cavity. The patient was diagnosed
subcutaneous fat and muscle. The wound with chronic deep caries. What method of
is bleeding. What kind of initial surgical examination allowed the dentist to elimi-
d-bridement should be performed? nate chronic periodontitis?

A. Early surgical d-bridement A. Electro-odontometry


B. Delayed surgical d-bridement B. Probing
C. Secondary surgical d-bridement C. Palpation of projection of root apex
D. Initial surgical d-bridement along with D. Percussion
plasty E. Cold test
E. - 21. An 18-year-old patient complains
17. A 47-year-old patient consulted a about body temperature rise, weakness,
dentist about dental cervix exposure on pain induced by eating and deglutition.
both jaws. Objectively: the dentitions are Objectively: mucous membrane of the
intact, the dental cervixes are exposed. oral cavity is erythematic with multi-
Untimely teeth contacts are present. It ple petechia. Pharynx is hyperaemic.
is planned to perform selective grindi- Regional lymph nodes are enlarged,
ng. What controlling method should be mobile, painless. In blood: leukocytosis,
chosen? monocytosis, atypic mononuclears, ESR
is 30 mm/h. What is the leading factor of
disease development?
Krok 2 Stomatology 2009 4

A. Viral infection Objectively: cheek is swollen in the regi-


B. Bacterial infection on of mandible on the left, palpation
C. Autoimmune disorders revealed induration of soft tissues. In
D. Immediate allergy projection of the 35, 36 teeth there is a
E. Delayed allergy fistula containing pus and granulations.
X-ray picture shows destruction of bone,
22. A female patient consulted a sequestrum. What is the most likely di-
stomatologist about inability to close her agnosis?
mouth, speech difficulty. Objectively: the
oral cavity is half-open, there is drooling, A. Chronic osteomyelitis
central line deviates to the right. There B. Actinomycosis
is a depression in front of the tragus, C. Syphilis
and above the malar arch the mandibular D. Sarcoma
condyle bulges inward the infratemporal E. Osteoma
fossa. What is the most likely diagnosis?
26. A 23-year-old patient complains about
A. Unilateral anterior dislocation of a small ulcer on the red border of her
mandible on the left lower lip that has been irresponsive to
B. Unilateral anterior dislocation of self-treatment for two weeks. Objectively:
mandible on the right unchanged red border of lower lip has a
C. Unilateral posterior dislocation of circular ulcer of 2 mm in diameter with
mandible on the left raised regular edges, its floor is of meat-
D. Unilateral posterior dislocation of like colour, dense, shiny, with "stearic fi-
mandible on the right lm", with cartilaginoid infiltration, pai-
E. Bilateral anterior dislocation nless on palpation. Regional lymph node
is enlarged, of tight elastic consistency,
23. Analysis of a 10-year-old boy’s jaw painless, mobile. What is the most likely
models revealed that occlusal plane of diagnosis?
the frontal maxillary teeth was of concave
form, its lateral parts were convex. Form A. Primary syphilis
of the alveolar process also represents B. Decubital ulcer
deformation of dental arches. The upper C. Cancerous ulcer
jaw is of saddle-like form with abrupt D. Lupus erythematosus
narrowing in the region of premolar teeth. E. Tuberculous ulcer
What type of bite is it?
27. X-ray picture depicts a circular well-
A. Open defined area of bone tissue destruction
B. Distal 0,7х0,7 cm large in the projection of root
C. Deep apex. What is the most likely diagnosis?
D. Mesial
E. Cross A. Cystogranuloma
B. Cyst
24. A 50-year-old patient complains about C. Granuloma
increased sensibility of the exposed tooth D. Odontoma
cervixes, displacement of teeth, gum itch, E. Osteoma
pain in the region of the 43, 42, 41,
31, 32, 33 teeth caused by chemical and 28. A 29-year-old patient complains about
thermal stimuli. Objectively: the gums are acute attack-like pain in the region of
dense and anemic. X-ray picture shows his upper jaw on the left, as well as in
atrophy of the alveolar bone reaching 2/3 the region of his left maxillary sinus, eye
of interalveolar septa height. What is the and temple. The pain is long-lasting (2-3
most likely diagnosis? hours), it is getting worse at night. The
patient has a history of recent acute respi-
A. III degree parodontosis ratory disease. Objectively: the 26 tooth
B. II degree parodontosis has a carious cavity, floor probing is pai-
C. Chronic generalized III degree nful, thermal stimuli cause long-lasting
parodontosis pain, percussion causes slight pain. What
D. Chronic generalized II degree is the most likely diagnosis?
parodontosis
E. Atrophic gingivitis
25. A 32-year-old patient complains of
a fistula in the submandibular region.
Krok 2 Stomatology 2009 5

A. Acute diffuse pulpitis old child revealed untimely missing of


B. Acute focal pulpitis all the upper molars. The lower incisors
C. Acute apical periodontitis contact with mucous membrane of palate.
D. Inflammation of maxillary sinus What is the tactics of choice?
E. Exacerbation of chronic periodontitis
A. Fabrication of a removable lamellar
29. A 45-year-old patient complains about prosthesis
pain in his mandible that arose after B. Fabrication of an orthodontic appliance
extraction of the 36 tooth. Objectively: for deep overbite correction
alveolar socket is covered with bloody C. Annual examination till cutting of the
clot. X-ray picture shows unextracted root permanent teeth
of the 36 tooth. What tools are necessary D. Half-yearly examination till cutting of
for extraction of this root? the permanent teeth
E. Medical intervention is not required
A. Angled elevator curved right
B. Angled elevator curved left 33. A 20-year-old patient complains about
C. Straight elevator a carious cavity in an upper right tooth.
D. S-shaped forceps Objectively: the 16 tooth has a deep cari-
E. Bayonet-shaped forceps ous cavity communicating with the tooth
cavity, probing at the opening point is pai-
30. A patient complains of pain and swelli- nless, percussion of the 16 causes mild
ng in the right submandibular area. She pain. There is a fistula on the gingiva in
has been treating the 45 tooth for a week. the region of root apex projection of the
Objectively: body temperature is 38oC. 16 tooth. What is the most probable di-
There is a painful tense infiltration in agnosis?
the right submandibular region. The skin
doesn’t make a fold, its hyperemic and A. Chronic granulating periodontitis
glossy. The mouth can be opened by 3 cm. B. Chronic fibrous periodontitis
Deglutition is painless. These clinical fi- C. Chronic granulomatous periodontitis
ndings correspond with the following di- D. Chronic hypertrophic pulpitis
sease: E. Chronic gangrenous pulpitis
A. Odontogenous phlegmon of the right 34. A 16-year-old patient complains about
submandibular region a cosmetic defect in form of white spots in
B. Abscess of the right alveololingual the region of the upper frontal teeth. The
groove defect was revealed long ago and doesn’t
C. Adenophlegmon of the right submandi- change with time. Objectively: white spots
bular region on the vestibular surfaces of the 11, 12, 21,
D. Acute odontogenous sialoadenitis 22 teeth close to the cutting edge and on
E. Phlegmon of pterygomandibular space the vestibular surfaces of the 16, 26, 36, 46
teeth close to the masticatory surface. On
31. A 5-year-old boy presents with body probing the spot surface was smooth, pai-
temperature rise up to 39, 2oC, sore nless; cold stimulus produced no pain. The
throat, headache, nausea. Objectively: spots couldn’t be stained with 2% solution
mucous membrane of soft palate and of methylene blue. What is the most likely
palatine arches is evidently hyperemic, diagnosis?
there is distinct border between it and
surrounding tissues. The child’s tongue is A. Systemic enamel hypoplasia
dry, edematic, its lateral surfaces are red B. Local enamel hypoplasia
and free of fur, fungiform papillae are evi- C. Acute initial caries
dently enlarged. Face skin is covered wi- D. Fluorosis in form of spots
th spotty rash, hyperemic except for pale E. Erosion of dental solid tissues
nasolabial triangle. Submandibular lymph
nodes are painful on palpation. What is 35. A 35-year-old patient complains about
the causative agent of this disease? a cavity in a lower jaw tooth on the left,
and the pain caused by sweet, sour and
A. Haemolytic streptococcus solid food. Examination of the 36 tooth
B. Coxsackie virus revealed a deep carious cavity filled with
C. Herpes virus light softened dentine. Probing of the cari-
D. Bordet-Gengou bacillus ous cavity floor is painful, reaction to the
E. Loffler’s bacillus cold stimulus is painful, undurable. What
is the most likely diagnosis?
32. Preventive examination of a 4,5-year-
Krok 2 Stomatology 2009 6

A. Acute deep caries 47 teeth are missing; there is postoperati-


B. Acute focal pulpitis ve midline defect of hard palate. It was
C. Chronic fibrous pulpitis decided to make a clasp dental prosthesis
D. Acute median caries with obturating part. The obturating part
E. Chronic deep caries should be placed on the following element
of the clasp dental prosthesis:
36. A 31-year-old male patient complai-
ns of dryness and burning of tongue A. On the arch
back that appeared for about a week B. On the saddle
ago and get worse when he eats irritati- C. On the artificial teeth
ng food. The patient has a history of D. On the base
recent pneumonia. He had been treated E. On the clasps
in the in-patient hospital for 2 weeks, the
treatment program included antibiotics. 40. A 10-year-old boy consulted a denti-
Now he doesn’t take any drugs. Objecti- st about pain in the palate during eating.
vely: mucous membrane of the oral cavity Objectively: the lower third of his face is
is hyperemic, dry, glossy. Tongue back and shortened, mouth opening is not limited.
palate have greyish-white plicae that can By joining the teeth the cutting edge of
be easily removed. Threads of saliva trail inferior incisors contacts with the mucous
behind the spatula. What is the most likely membrane of palate. Mucous membrane
diagnosis? in the contact point is hyperemic, slightly
edematic. Lateral teeth exhibit Angle’s
A. Acute pseudomembranous candidiasis class I malocclusion. What is the most
B. Chronic hyperplastic candidiasis appropriate plan of treatment of the lower
C. Acute atrophic candidiasis jaw?
D. Medicamental stomatitis
E. Chronic atrophic candidiasis A. To impact frontal part
B. To impact lateral parts
37. A 53-year-old patient consulted a C. To widen the lower jaw
prosthodontist about lateral teeth mobili- D. To protract frontal part
ty, frequent falling out of fillings. Objecti- E. To protract lateral parts
vely: all the molars and premolars of
the lower jaw exhibit I degree mobility. 41. A patient consulted an oral surgeon
Approximal masticatory surfaces have fi- about pain in the region of his left upper
llings. What splint construction should be jaw that appeared 3 days ago. After
applied in this case? examination the patient was diagnosed
with exacerbation of chronic periodontitis
A. Inlay splint of the 17 tooth. It is indicated to extract
B. Fixed crown splint the 17 tooth. What nerves should be
C. Crown cap splint blocked for painless extraction of the 17
D. Equator crown splint tooth?
E. Intradental splint
A. Posterior alveolar nerves and greater
38. A 28-year-old patient consulted a palatine nerve
prosthodontist about dental prostheti- B. Greater palatine nerve
cs. Objectively: crown of the 13 tooth is C. Anterior alveolar nerves and incisor
decayed by 2/3 of its height, the stump sti- nerve
cks out above the gum line by 2 mm. X-ray D. Median alveolar nerves and greater
picture shows that the root canal is filled palatine nerve
up to the top. What construction should E. Nasopalatine nerve
be fabricated for the patient?
42. A 40-year-old patient complains about
A. Pivot tooth frequent falling out of a filling. Objecti-
B. Crown vely: the 46 tooth has a carious (Black’s
C. Inlay class II) cavity. It is planned to restore the
D. Equator crown anatomic form of the tooth by means of
E. The crown should be restored with a metal inlay. What is the peculiarity of
photopolymer the cavity preparation for the inlay in this
case?
39. A 65-year-old patient complains about
partially missing teeth on his upper jaw,
difficult mastication, rhinolalia. Objecti-
vely: the 18, 16, 15, 11, 23, 28, 35, 38, 48,
Krok 2 Stomatology 2009 7

A. Forming a bevel
B. Forming an additional shoulder A. Exacerbation of chronic granulating
C. Cavity floor widening periodontitis
D. Preparation for parapulpar posts B. Acute purulent pulpitis complicated by
E. Cavity deepening periodontitis
C. Exacarbation of chronic pulpitis
43. A 10-year-old child complains about D. Exacerbation of chronic granulomatous
acute spontaneous spasmodic pain in an periodontitis
upper jaw tooth on the left. Objectively: E. Acute serous periodontitis
distal contact surface of the 26 tooth
exhibits a carious cavity filled with li- 47. A patient undergoes orthopaedic
ght softened dentine and localized within treatment of bounded edentulous spaces
parapulpar dentine. Probing of the cavity on the upper jaw. He needs fixed full-
floor causes acute pain, percussion is pai- cast dentures. During his second visit it
nless. Cold stimuli cause a long-standing is required to check whether the internal
pain attack. The child has a history of li- surface of the metal framework of the
docaine allergy. Choose an optimal paste future metal-ceramic denture matches the
to be used during the first visit: surfaces of the prepared teeth. In what
way could this be done?
A. Paraformaldehyde
B. Iodoform A. In the oral cavity by means of silicone
C. Thymol materials
D. Formocresol B. Visually by means of models in the
E. Zinc oxide eugenol articulator
C. In the oral cavity by means of tracing
44. A 35-year-old patient complains about paper
itch, burning and edema of lips. He has D. In the oral cavity by means of a wax
been suffering from this for a week. plate
Objectively: reddening of red border and E. In the oral cavity by means of
skin, especially in the region of mouth stomatoscopic method
corners, there are also vesicles, crusts,
small cracks along with erythematous 48. An 18-year-old female patient
affection of red border. What is the most consulted a dentist about dental prostheti-
likely diagnosis? cs. Objectively: the 21 tooth is pulpless,
of dark-grey colour. The patient has
A. Acute eczematous cheilitis orthognathic occlusion. It is planned to
B. Multiform exudative erythema crown the tooth with a plastic crown.
C. Acute herpetic cheilitis Choose the plastic mass for its fabrication:
D. Allergic contact cheilitis
E. Exudative form of exfoliative cheilitis A. Sinma
B. Ethacryl
45. Preventive examination of an 8-year- C. Acrel
old boy revealed some lusterless chalk- D. Ftorax
like spots on the vestibular surface of the E. Bakril
11 and 21 teeth, which are localised in the
precervical region. Subjective complaints 49. A 47-year-old patient complains about
are absent. What is the most likely di- permanent pain in the 27 tooth that
agnosis? is getting worse when biting down on
food. Objectively: the patient’s face is
A. Acute initial caries symmetric, skin is of normal colouring,
B. White-spotted fluorosis the mouth can be fully opened, mucous
C. Local enamel hypoplasia membrane of the alveolar process is
D. Acute superficial caries edematic and hyperemic at a level of the
E. Chronic initial caries 27 tooth. The 27 tooth has a deep carious
cavity interconnecting with pulp chamber.
46. A 27- year-old patient complains Percussion of the 27 tooth causes acute
about acute pain in the region of the pain. What is the most likely diagnosis?
34 tooth that is getting worse when bi-
ting down on food. Roentgenographical
survey revealed an ill-defined zone of
bone tissue destruction in the periapical
region of root of the 34 tooth. What is the
most likely diagnosis?
Krok 2 Stomatology 2009 8

A. Exacerbated chronic periodontitis of A. Chronic lymphadenitis


the 27 tooth B. Chronic sialoadenitis
B. Chronic periodontitis of the 27 tooth C. Sialolithiasis
C. Acute generalized purulent pulpitis of D. Adenoma of salivary gland
the 27 tooth E. Atheroma
D. Acute purulent periostitis of the upper
jaw beginning from the 27 tooth 53. A 38-year-old patient ordered a
E. Chronic left-sided odontogenous hi- clasp upper jaw prosthesis. Its fabricati-
ghmoritis on involves forming of a fireproof model.
What material should be applied for
50. A 64-year-old female patient was wai- duplication?
ting for her turn at the dentist’s. Suddenly
she fell down, her respiration became A. Gelin
hoarse, she got convulsive twitching in B. Stomatoplast
her upper and lower limbs, face and neck C. Stomalgin
turned cyanotic, eye pupils became mydri- D. Orthocor
atic, reaction of eye to light was absent. E. Dentafol
Arterial pressure and pulse couldn’t be
measured. Heart sounds couldn’t be 54. A 35-year-old patient needs a metal
auscultated. Involuntary urination was inlay for the 37th tooth. Objectively:
noted. What condition is characterized by masticatory surface of the 37 tooth has
such symptoms? a carious cavity. What is the peculiarity of
tooth preparation?
A. Clinical death
B. Epilepsy A. Forming a bevel
C. Shock B. Forming an additional shoulder
D. Coma C. Widening of cavity floor
E. Collapse D. Forming flat floor
E. Forming additional cavity
51. A 3-year-old child complains of a cavi-
ty in a lower jaw tooth. Objectively: masti- 55. A 64-year-old patient complains about
catory surface of the 84 tooth exhibits a slightly painful ulcer on the lateral
a carious cavity within mantle dentine. surface of his tongue that appeared over 2
Dentine is softened, probing of the cavi- months ago. Objectively: lateral surface of
ty walls is painful, percussion is painless. tongue has an ulcer with raised edges, ti-
What is the most likely diagnosis? ssues around it are infiltrated. Submandi-
bular lymph nodes are enlarged, adheri-
A. Acute median caries ng to each other and surrounding tissues.
B. Chronic superficial caries What is your provisional diagnosis?
C. Chronic fibrous pulpitis
D. Acute deep caries A. Lingual cancer
E. Chronic deep caries B. Syphilis (hard chancre)
C. Tuberculous ulcer
52. A 43-year-old patient complains D. Decubital ulcer
about a neoplasm in the right submandi- E. Ulcero-necrotic lesion associated with
bular region that appeared a month ago blood disorders
after angina. Body temperature is up to
37, 0 − 37, 2oC. The patient underwent 56. A 56-year-old patient complains about
anti-inflammatory therapy but it led to no missing lateral teeth on both sides of
reduction of the neoplasm. Objectively: in mandible. Objectively: the 48, 47, 46, 45,
the right submandibular region palpati- 35, 36, 37, 38 teeth are missing. Make a di-
on reveals a slightly painful, spherical, agnosis:
freely movable, well-defined neoplasm of A. Kennedy I denttion defect
dense and elastic consistency. The duct of B. Kennedy IV denttion defect
submandibular salivary gland discharges C. Kennedy II denttion defect
transparent saliva. The sublingual plica is D. Kennedy III denttion defect
unchanged. What is the most likely di- E. Betel’man II denttion defect
agnosis?
57. A 4-year-old child got a face trauma 2
hours ago. A stomatologist on duty made
a diagnosis: intrusive luxation of the 61
tooth. What is the tactics of choice?
Krok 2 Stomatology 2009 9

A. Chronic deep caries


A. Extraction of the 61 tooth B. Chronic fibrous periodontitis
B. Observation C. Chronic fibrous pulpitis
C. Reposition of the 61 tooth D. Chronic median caries
D. Splinting of the 61 tooth E. Chronic granulomatous periodontitis
E. Removal of pulp of the 61 tooth
61. A 7-year-old child complains about
58. An 18-year-old student complains spontaneous pain in the upper right
of roughness and dryness of mucous molars. Both medial contact and masti-
membrane of the oral cavity. He always bi- catory surfaces of the 55 tooth have a
tes uncontrolledly off mucous membrane carious cavitiy filled with clear softened
along the line of teeth joining. The pati- dentin and localized within parapulpar
ent suffers from chronic cholecystitis, he dentin. Floor probing causes acute pain,
is smoker. Hygiene index is 2,3. Objecti- tooth percussion is slightly painful.
vely: mucous membrane along the line of Mucous membrane of the alveolar process
teeth joining is quaggy, edematic, whitish. in the root projection of the 55 is intact,
What is the leading factor of this disease examination of this region revealed no
development? roentgenological changes. What is the
most likely diagnosis?
A. Habitual biting of mucous membrane
of the oral cavity A. Acute diffuse pulpitis
B. Unsatisfactory level of oral cavity B. Chronic fibrous pulpitis
hygiene C. Chronic gangrenous pulpitis
C. Somatic pathology D. Exacerbation of chronic periodontitis
D. Xerostomia E. Deep acute caries
E. Smoking
62. A 55-year-old patient has a repeated
59. A 57-year-old patient complains about appointment with a dentist. He has
dryness and burning of the lateral surface ordered complete removable lamellar
of her tongue. These sensations disappear dentures for both upper and lower jaw.
during eating. She noted such sensations During his previous visit the dentist
three months ago. She has a history of took complete anatomic impressions of
gastritis with reduced secretory function. both jaws and sent them to the dental
Objectively: mucous membrane of tongue mechanic. What manipulations should the
and oral cavity has no peculiarities. The dentist perform at the next clinical stage?
back of tongue has thin white coating.
Regional lymph nodes are unpalpable. A. Adjust individual trays
Oral cavity is sanitized. What is the most B. Locate and fix central occlusion
likely diagnosis? C. Check the prosthesis construction
within the oral cavity
A. Glossodynia D. Correct the prosthesis and give the
B. Lingual nerve neuritis patient necessary instructions
C. Candidiasis E. Fabricate occlusal rims
D. Desquamative glossitis
E. Hunter-Moeller glossitis 63. A 33-year-old patient complai-
ns about pain, gingival haemorrhage,
60. A 23-year-old patient complains halitosis, body temperature rise up
about a carious cavity in the 15 tooth to 37, 8oC. Objectively: gums are
that he noted a year ago. Examinati- apparently hyperaemic, edematic, bleed
on revealed that approximal surface had easily, parodontal pouches are 3-
a deep carious cavity not communi- 4 mm deep, and contain purulent
cating with the tooth cavity and fi- exudate. Orthopantomogram shows di-
lled with dense pigmented dentine. ffuse osteoporosis of alveolar process,
Probing, cold test, percussion cause no resorption of interdental septa down to
pain. Electroodontodiagnosis is 10 mi- 1/3 of their height. What is the most likely
croampere. What is the most likely di- diagnosis?
agnosis?
Krok 2 Stomatology 2009 10

A. Exacerbation of chronic generalized I 67. A 69-year-old patient needs extraction


degree periodontitis of the 12, 11 teeth. He is diagnosed with
B. Chronic generalized I degree peri- generalized periodontitis, the 12 and 11
odontitis teeth exhibit II degree mobility. Choose a
C. Exacerbation of chronic generalized II proper instrument for extraction:
degree periodontitis
D. Chronic generalized II degree peri- A. Straight forceps
odontitis B. S-shaped forceps
E. Exacerbation of chronic catarrhal gingi- C. S-shaped forceps curved right
vitis D. Root bayonet-shaped forceps
E. Crown bayonet-shaped forceps
64. An 8-year-old boy complains about
acute short-term pain in the 11 tooth 68. An 8-year-old child presents with an
caused by cold and sweet stimuli. Objecti- edema of the submandibular region, the
vely: medial contact surface of the 11 mouth can be opened by 1,5 cm, further
tooth has a carious cavity within denti- opening is difficult, body temperature is
noenamel junction filled with softened 37, 6oC, mucogingival fold is vestibularly
moist dentine that can be easily removed. flattened, hyperaemic and swollen. The 84
Enamel edges are white, undermined, and 85 teeth have fillings, their percussi-
friable. Superficial probing is painless, on is painless. The 84 tooth exhibits I
thermal test is positive, percussion is degree mobility. What is the most likely
negative. What is the filling material of diagnosis?
choice?
A. Acute odontogenic periostitis
A. Glass ionomer cement B. Acute odontogenic osteomyelitis
B. Silicophosphate cement C. Chronic odontogenic periostitis
C. Composite material D. Chronic odontogenic osteomyelitis
D. Silver amalgam E. Exacerbation of chronic periodontitis
E. Silicate cement
69. A 25-year-old patient has a median
65. A 40-year-old patient complains about fracture of mandible without evident di-
body temperature rise up to 38o C, and splacement of fragments. All the teeth are
of a roundish infiltration on his upper lip. intact. What kind of wire splint is the most
Objectively: there is a roundish infiltration appropriate in this case?
on the upper lip on the left, the skin above
it is wine red, the infiltration adheres to A. Smooth splint
the surrounding tissues and has a seapus B. Smooth splint with a spreading curve
necroticus in the middle. The upper lip C. Splint with hooks
is hyperaemic and edematic. What is the D. Splint with guide plane
most likely diagnosis? E. Pomerantseva-Urbanskaya’s splint

A. Upper lip furuncle 70. A 19-year-old patient complains about


B. Acute periostitis of the upper lip pain in the submental part of mandible.
C. Retention cyst The day before he got a trauma. Objecti-
D. Acute lymphadenitis vely: there is a slight swelling of tissues
E. Upper lip carbuncle in the mental area. The mouth can be
opened widely enough. All the teeth are
66. A female patient complains about pain intact. Mucous membrane is edematic in
in the 11 tooth caused by sour and sweet the region of central incisors, it bleeds sli-
food. Objectively: enamel changes in form ghtly. In this region mobility of mandible
of chalky appearance, a defect with light fragments is present. Occlusion is undi-
bottom within dentinoenamel junction on sturbed. What splint should be chosen by
the vestibular surface in the precervical the dentist?
area of the 11 tooth. Probing was painless,
percussion and cold stimulus caused no A. Smooth brace
pain. What is the most likely diagnosis? B. Splint with spreading curve
C. Splint with guide plane
A. Acute superficial caries D. Anchor splint with intermandibular
B. Acute initial caries fixation
C. Acute median caries E. Ivy loops for 31 32, 42 41
D. Enamel hypoplasia
E. Fluorosis 71. A 25-year-old man complains of itchi-
ng and reddening of skin in the buccal
Krok 2 Stomatology 2009 11

area, general weakness, inertness. He 75. A 13-year-old girl complains about


associates origin of the disease with a frequent falling out of a filling in the 21
skin injury he got during shaving. Objecti- tooth. It is known from the anamnesis that
vely: body temperature is 39, 0o C. A well- 2 years ago she underwent treatment on
defined section of skin in the buccal area account of a dental trauma. Objectively:
is hyperemic, it slightly protrudes above a transverse defect of 1/3 of the crown in
the surface. Hyperemic surface has some the 21 tooth. Percussion is painless. Tooth
vesicles containing serous fluid. What is colour is unchanged. X-ray picture shows
the most likely diagnosis? that root canal is filled by 1 mm from the
apex, filling material closely fits the walls
A. Erysipelatous inflammation of skin of root canal. What stomatological tactics
B. Phlegmon of buccal area should be chosen?
C. Furuncle
D. Anthrax A. The defect should be restored with
E. Streptococcal impetigo photopolymer
B. The root canal should be refilled
72. A 30-year-old saxophonist complai- C. The root apex should be resected
ns about wearing of the occlusal surface D. The tooth should be extracted
of his frontal teeth along with separati- E. The tooth should be crowned with an
on between the upper and lower teeth artificial crown
(apertognathia). Height of the lower third
of the patient’s face remains unchanged. 76. A 35-year-old patient was diagnosed
What method should be applied for elimi- with chronic median caries of the 36 tooth.
nation of this deformity? There is a Black’s class II cavity affecting
masticatory surface. What material should
A. Protetic be chosen for the tooth filling?
B. Increase of the interalveolar height
C. Shortening of teeth A. Light-cure microhybride composite
D. Orthodontic B. Glass ionomer cement
E. Instrumental-and-surgical C. Silicophosphate cement
D. Light-cure fluid composite
73. A 32-year-old patient complains about E. Light-cure microfilled composite
mouth soreness, body temperature rise
up to 38, 5oC, indisposition. Such condi- 77. A 79-year-old female patient
tion has been occurring periodically for consulted a prosthodontist about denture
several years after the patient had had a replacement. The patient has a history
cold. Objectively: lips are covered with of a stroke. Objectively: acute irregular
haemorrhagic crusts, hyperaemic mucous atrophy of the alveolar processes of both
membrane of lips and cheeks has erosions jaws is present; mucous membrane of
covered with fibrinous films. Hypersali- the oral cavity is dry and nonmobile.
vation is present. What is the most likely The previous dentures cannot be fixed.
diagnosis? What is the most appropriate prosthetic
construction?
A. Multiform exudative erythema
B. Pemphigus vulgaris A. Dentures with elastic lining
C. Herpes recidivicus B. Dentures with extended borders
D. Herpetiform Duhring’s dermatitis C. Dentures with metal bases
E. Stevens-Johnson syndrome D. Dentures with shortened borders
E. Implant-supported dentures
74. Examination of a patient revealed a
roundish neoplasm in the submental area. 78. A 48-year-old teacher complained
Skin above it is unchanged, makes a fold about considerable mobility of the 42, 41,
easily. Puncture sample contains strow- 31, 32 teeth. Objectively: cervixes of the
yellow liquid with cholesterol admixture. 42, 41, 31, 32 are exposed by 2/3, there is
What is the most likely diagnosis? pathological III degree mobility. It is indi-
cated to extract the 42, 41, 31, 32 teeth.
A. Inborn median cyst What type of denture should be used for
B. Lipoma the direct prosthetics?
C. Lymphadenitis
D. Retention cyst of sublingual salivary
gland
E. Dermoid cyst
Krok 2 Stomatology 2009 12

A. Removable partial lamellar denture the central position is measured by an


B. Clasp denture angle of 100 − 110o . What is characterized
C. Adhesive by such a value of incisor point deviation?
D. Metal-ceramic bridge-like denture
E. Swaged-and-soldered bridge-like A. Lateral incisor path
denture B. Lateral joint path
C. Sagittal incisor path
79. Examination of a 13-year-old patient D. Sagittal joint path
allowed to make a final diagnosis: vesti- E. Bennett’s angle
bular position of the 13 and 23 teeth with
the total space deficit, narrowing of maxi- 83. Parents of a 1,5-year-old chi-
llary dental arch, torsion of the 12 and 22 ld complain about the child’s tongue
tooth. To eliminate this pathology it was enlargement and ingestion disorder. The
suggested to widen the dental arch and child has been suffering from this since bi-
to extract some teeth. What teeth have rthday. Objectively: general condition has
orthodontic indication for their extracti- no peculiarities. The tongue is enlarged
on? (macroglossia). Its mucous membrane
exhibits granular vesicular outgrowths.
A. First premolars The tongue is dense, painless on palpati-
B. Canines on. What is the most likely diagnosis?
C. Second incisors
D. Second premolars A. Lymphangioma of tongue
E. First molars B. Hemangioma of tongue
C. Fibroma of tongue
80. Parents of a 2-year-old girl complain D. Cyst of tongue
about fistulas with purulent discharge in E. Cancer of tongue
the region of the upper frontal teeth.
Objectively: crowns of the 51, 52, 61, 62 84. A 52-year-old patient complains of
teeth are significantly decayed, probing significant tooth wear on both jaws.
of root canal orifices is slightly painful, it Objectively: in the lateral parts of mandi-
causes significant haemorrhage. Percussi- ble teeth are worn down to the gums,
on is painless. Mucous membrane of the in the frontal parts teeth are worn by
alveolar process is pastose and cyantotic, 1/3. Treatment includes two stages. What
there are cicatrices and fistulas in this regi- prosthetic construction should be appli-
on. What is the most likely diagnosis? ed for bite disjoining and myotatic reflex
change?
A. Exacerbation of chronic granulating
periodontitis A. Periodontal splint for the lateral parts
B. Exacerbation of chronic granulomatous B. Plastic splint for the whole dentition
periodontitis C. Plastic splint for the frontal part
C. Exacerbation of chronic fibrous peri- D. Crown splint for the lateral parts
odontitis E. Elastic plastic splint
D. Chronic granulomatous periodontitis
E. Chronic granulating periodontitis 85. A girl is 18 months old. Vestibular
surfaces of the 52, 51, 61, 62 teeth have wi-
81. A month after cementing the metal- de carious cavities within enamel. Probing
ceramic crown on the 23 tooth a patient is slightly painful, percussion of the 52, 51,
complained of its decementing. Exami- 61, 62 is painless. What is the treatment of
nation revealed that the tooth stump was choice?
of sufficient height, stump walls were
convergent to the vertical tooth axis at an A. Silver impregnation
angle of approximately 30 degrees. What B. Filling with amalgam
angle of convergence had to be formed? C. Remineralizing therapy
D. Fluorine laquer coating
A. Up to 8 degrees E. Filling with phosphate cement
B. 12-15 degrees
C. 15-18 degrees 86. A 53-year-old patient consulted a
D. 22-25 degrees prosthodontist about dental prosthetics.
E. 10-12 degrees Objectively: the 13 and 24 teeth remain
as well as all the teeth on the lower jaw.
82. During lateral movements of mandi- The lower third of the patient’s face is
ble the frontal teeth are displaced si- shortened, crowns of the 13, 24 teeth
dewards. Deviation of incisor point from are worn off by 2/3. What constructional
Krok 2 Stomatology 2009 13

element will provide optimal fixation of on and right zygomatic region, skin
the partial removable prosthesis on the numbness in the area of the right half of
upper jaw? his upper lip; nasal haemorrhage. These
symptoms turned up after a trauma. What
A. Telescopic crowns disease should be suspected?
B. Compound clasps
C. Retaining clasps A. Fracture of zygomatic bone
D. Bars B. Le Fort I maxillary fracture
E. Dentoalveolar clasps C. Le Fort II maxillary fracture
D. Le Fort III maxillary fracture
87. Parents of a 6-year-old child applied to E. Fracture of nose bones
a pedodontist for preventive examinati-
on of their child. The oral cavity is sani- 91. A 56-year-old patient has an oval,
tized. According to the parents, the child smooth, bright-red erosion on the red
has recently cut the 36 and the 46 tooth. border of her lower lip. Erosion is covered
What method of caries prevention should with haemorrhagic crusts that can be
be applied within 1,5-2 years after cutting hardly removed. Crust removal induces
of the mentioned teeth? slight haemorrhage. Light traumatization
of crust-free surface of erosion induces no
A. Fissure hermetization haemorrhage. Specify the type of lower
B. Fissure silvering lip precancer:
C. Coating the teeth with fluorine lacquer
Ftorlak A. Abrasive precancerous Manganotti’s
D. Remodentum solution applications cheilitis
E. Gargling with sodium fluoride B. Verrucous precancer of red border
C. Localized precancerous hyperkeratosis
88. Examination of a 6-year-old girl of red border
revealed a deep carious cavity in the 85 D. Bowen’s disease
tooth. Percussion and probing are pai- E. Erythroplasia
nless. After removal of the softened denti-
ne communication with the tooth cavity 92. A 12-year-old child complains about
showed up. Deep probing is painless. X- bleeding from the tooth socket during
ray picture of the 85 tooth shows the focus eating and tooth brushing. The tooth
of destruction of bone tissue in the region has hurt him before. Objectively: the 36
of bifurcation; cortical plate of the 35 has tooth has a deep cavity communicating
no pathological changes. It is most expedi- with the tooth cavity and filled with red
ent to use the following material for the excrescences. Probing causes pain and sli-
root filling: ght haemorrhage; percussion is painless,
thermal stimuli cause mild pain. What is
A. Zinc oxide eugenol cement your provisional diagnosis?
B. Resorcin-formalin paste
C. Glass ionomer cement A. Chronic hypertrophic pulpitis
D. Phosphate cement B. Chronic granulating pulpitis
E. Calcium-containing paste C. Chronic papillitis
D. Gingival polyp
89. A 48-year-old patient got a pustule E. Chronic simple pulpitis
on his chin that quickly developed into a
dense and acutely painful infiltration 3x5 93. A 23-year-old patient complains
cm large. The skin above it is of blue-red about gingival haemorrhage during tooth
colour. In the centre one can see three brushing and eating solid food. Objecti-
zones of necrosis around the hair follicles. vely: gingiva of the frontal part of mandi-
Lymph nodes of chin are enlarged and ble is hyperaemic, edematic, it bleeds on
painful. What is the most likely diagnosis? palpation. Mucous membrane of the oral
cavity as well as gingiva in other regions
A. Chin carbuncle present no changes. The patient has deep
B. Erysipelatous inflammation of chin overbite. Teeth are stable except for the
C. Dermal actinomycosis of chin 41 and 31 (I degree of mobility). X-ray
D. Suppurated atheroma picture shows resorption of interalveolar
E. Chin furuncle septa by 1/3 of root length in the region of
the 42, 41, 32, 31 teeth. What is the most
90. A patient applied to the oral surgery likely diagnosis?
department and complained about pain
and edema in the right infraorbital regi-
Krok 2 Stomatology 2009 14

A. Localised periodontitis the lower dental arch. It is planned to


B. Initial generalized periodontitis make an implant-supported bridge for its
C. Generalized I degree periodontitis restoration. X-ray picture shows that the
D. Catarrhal gingivitis height of the bone mass from projection of
E. I degree parodontosis mandibular canal up to the top of alveolar
crest is 2 cm. What type of implant should
94. A 13-year-old patient complains be applied?
about gingival haemorrhage during tooth
brushing. Objectively: gums around all the A. Threaded
teeth are hyperemic and edematic, PMA B. Endodontic-endoosseous
index (papillary marginal alveolary index) C. Plate-form
is 46%, Greene-Vermillion hygiene index D. Subperiosteal
is 2,5. Provisional diagnosis: exacerbation E. Conical
of chronic generalized catarrhal gingivitis.
This patient should be recommended to 98. A 10-year-old child undergoes sanitati-
use a toothpaste with the following active on of the oral cavity. The girl was found
component: to have chalky spots on the vestibular
surfaces in the precervical region of the
A. Chlorhexidine 21 and 12 teeth. Enamel surface is dull,
B. Calcium glycerophosphate smooth. Pain reaction to the temperature
C. Monofluorophosphate stimuli is absent. What additional method
D. Vitamins A, D, E of examination is expected to confirm the
E. Microelement complex diagnosis?
95. A 30-year-old patient has got an A. Vital staining
edema and hyperemia of nose wing skin B. Orthopantomography
spreading to the skin of his upper lip and C. Intraoral roentgenography
cheek. Examination revealed a fissure D. Electroodontodiagnostics
along the infra-external edge of the left E. Ultraviolet stomatoscopy
nostril. The skin is tense, dense, red-and-
cyanotic, skin line pattern is smoothed; 99. A 20-year-old patient complains about
rise of local temperature is present. The pain and haemorrhages in the region
zone of affection is well-defined and of the 36 tooth occuring during eating
irregularly shaped. What is the most li- solid food. Objectively: medial masti-
kely diagnosis? catory surface of the 36 tooth has a large
carious cavity occupied by a carneous
A. Erysipelatous inflammation tumour-like formation, probing induces
B. Dermal actinomycosis haemorrhage and pain in the region
C. Dermal tuberculosis of connection of the carious cavity wi-
D. Dermal cancer th the pulp chamber. Percussion is pai-
E. Thrombophlebitis of the facial vein nless. Electroodontodiagnosis is 40 mi-
croampere. Roentgenological changes are
96. On the second day after tooth extracti- absent. What is the most likely diagnosis?
on a 35-year-old woman applied to a facial
surgeon and complained about pain in the A. Chronic hypertrophic pulpitis
parotid-masticatory region and sensation B. Epulis
of lacking contact between the upper and C. Hypertrophic papillitis
lower jaw teeth on the right. Examination D. Chronic gangrenous pulpitis
revealed chin deviation to the left, half- E. Chronic fibrous pulpitis
open mouth, closed lips, disturbed occlusi-
on. There is also limitation of lateral 100. An infant was born full-term with
mandible movements. In front of the right body weight at a rate of 3200 g and body
ear tragus soft tissues sink down. What length at a rate of 53 cm. It was the first
examination is expected to be the most physiological delivery. What position of
informative in this case? child’s mandible is usually observed after
birth?
A. TMJ roentgenography
B. TMJ ultrasound A. Physiological retrogenia
C. Mandible roentgenography B. Physiological progenia
D. Bimanual TMJ investigation C. Direct relation
E. - D. Deep overbite
E. Open bite
97. A 50-year-old patient has a defect of
Krok 2 Stomatology 2009 15

101. A 14-year-old child has orthodontic A. Acute purulent periodontitis


indication for extraction of the 14 tooth. B. Acute purulent pulpitis
What forceps should be applied for C. Acute serous periodontitis
extraction of the 14 tooth? D. Exacerbation of chronic periodontitis
E. Acute diffuse pulpitis
A. S-shaped forceps
B. Straight forceps 105. A patient complains of burning,
C. Bayonet-shaped forceps itch and lower lip enlargement. He has
D. Beak-shaped forceps been suffering from this for a long ti-
E. Curved on flat forceps me. Objectively: the patient’s face is
asymmetric due to the flattening of
102. A 14-year-old child was undergoing nasolabial fold. His lower lip is edematic,
extraction of the 16 tooth on account of of normal colour, painless on palpation.
chronic periodontitis. During the tooth The patient has plicated tongue. What is
extraction it came to perforation of maxi- your provisional diagnosis?
llary sinus along with penetration of the
distal buccal root into the maxillary sinus. A. Melkersson-Rosenthal syndrome
What is the further dentist’s tactics? B. Quincke’s edema
C. Lymphangioma
A. The patient should be directed to the D. Hemangioma
hospital for a surgical procedure E. Granulomatous Miescher’s cheilitis
B. The dentist himself should try to extract
the root 106. A 30-year-old patient consulted a
C. The dentist should form a clottage dentist about a cosmetic defect and slight
without informing the patient pain in the 44 tooth during tooth brushi-
D. The dentist should close the perforation ng. Objectively: there is a Black’s class
with mucoperiosteal graft V carious cavity within mantle dentine.
E. The dentist should perform maxillary Probing of the cavity walls causes slight
sinusotomy in the outpatient setting pain, thermal probe is positive. What filli-
ng material is to be chosen for restoration
103. A 2-month-old child is anxious, sleeps of this cavity?
badly, refuses food, has subfebrile body
temperature. Objectively: hyperaemic A. Compomer restorative material
mucous membrane of the child’s tongue, B. Chemical-cure composite
lips, cheeks and palate has a caseous coati- C. Amalgam
ng that can be easily removed with a D. Polycarboxylate cement
tampon. Regional lymph nodes are sli- E. Metal ceramics
ghtly enlarged and painful on palpation.
What disease are these symptoms typical 107. An 18-year-old patient complains of
for? short-term pain in the 37 tooth caused
by sweet and cold stimuli. Objectively:
A. Acute pseudomembranous candidous masticatory surface of the 37 tooth has
stomatitis a carious cavity within mantle dentine.
B. Chronic atrophic candidous stomatitis The cavity is filled with softened dentine.
C. Acute herpetic stomatitis Probing of the cavity walls induces pain
D. Diphtheria in the region of dentinoenamel junction,
E. Measles-associated stomatitis electroodontodiagnosis is 6 microampere.
What is the most likely diagnosis?
104. A 35-year-old patient complains
about progressing throbbing pain in the A. Acute median caries
26 tooth. Objectively: the 26 tooth has a B. Acute superficial caries
carious cavity filled with softened denti- C. Acute deep caries
ne, tooth cavity is closed, probing of the D. Chronic fibrous pulpitis
cavity floor is painless, percussion causes E. Pulp hyperaemia
acute pain. There is I degree tooth mobi-
lity. Roentgenological changes are absent. 108. A 24-year-old soldier was injured by
What is the most likely diagnosis? a shell splinter. He was diagnosed with a
mandibular fracture with a bone defect in
the mental region over 2 cm long. What
method of fixation of mandible fragments
is indicated?
Krok 2 Stomatology 2009 16

A. Fragments fixation by means of Rudko’s impressions of both jaws. What is his next
apparatus step?
B. Tigerstedt’s splints
C. Intermandibular Ivy ligature A. To send the impressions for disinfection
D. Gunning-Port’s splint B. To let the impressions dry out in the
E. Direct osteosynthesis open air
C. To invite a dental mechanic for joint
109. A 35-year-old female patient analysis of the impressions
consulted a dentist about a painless, D. To send the impressions immediately to
slowly growing neoplasm in the area of the laboratory
the 11 and 12 teeth. Examination revealed E. To put the impressions into the microten
that the tumour was light-pink, flattened, bag for 90 minutes
adjacent to the teeth, had a pedicle. The
tumour was up to 1,5 cm large, with 113. A 12-year-old patient complains
smooth surface and dense consistency. It about an aesthetic defect. Objectively: the
was diagnosed as an epulis in the regiob of lower third of face is shortened, upper
the 11 and 12 teeth. What form of epulis frontal teeth overbite the lower teeth
are these clinical findings typical for? by 3/3 of height, exhibit oral inclinati-
on, lateral parts all along exhibit cusp-to-
A. Fibrous cusp relationship between the antagoni-
B. Angiomatous sts; Angle’s class II malocclusion (joining
C. Giant-cell of the upper permanent molars) is also
D. Pregnancy epulis present. Malocclusion is observed in the
E. - following planes:
110. A 49-year-old patient applied to the A. In sagittal and vertical
oral surgery department and complained B. In transversal
about permanent intense dull pain in the C. In transversal and vertical
region of the right upper jaw. It is known D. In vertical
from the anamnesis that the 17 tooth has E. In sagittal
been repeatedly treated for exacerbation
of chronic periodontitis but the treatment 114. A 65-year-old patient consulted a
appeared to be ineffective. What kind of prosthodontist about fabrication of an
anesthesia should be applied for extracti- external prosthesis of orbit that was lost
on of the 17 tooth? as a result of a trauma. What is the fixing
element of the orbit prosthesis?
A. Tuberal and palatinal
B. Tuberal and incisor A. Spectacle frame
C. Tuberal, incisor and palatinal B. Watch spring
D. Incisor and paltinal C. Swivel devices
E. Torus D. Clamps
E. Magnets
111. Stomatological examination of a chi-
ld revealed abnormal form of the central 115. A 56-year-old patient needs a partial
incisors: they are barrel-shaped, there removable lamellar denture. Objectively:
is a semilunar groove on the cutting the 17, 16, 15, 14, 25, 26, 27, 28 teeth on the
edge. It is known from the anamnesis upper jaw are missing. In order to provi-
that the child’s mother had syphilis de transversal line of clasps the clasp arms
during pregnancy. Besides the dental should be placed upon the following teeth:
abnormality the child presents also with
deafness and parenchymatous keratitis. A. 13 and 24
This abnormality of tooth development is B. 13 and 18
called: C. 24 and 18
D. 24, 13 and 18
A. Hutchinson’s teeth E. -
B. Pfluger teeth
C. Wedge-shaped defect 116. A 56-year-old patient consulted a
D. Fluorosis (destructive form) prosthodontist about pain underneath the
E. Erosion of hard tissues bar of her clasp prosthesis. It is known
from the anamnesis that the clasp was
112. A patient ordered partial removable placed in another city a week ago. Exami-
lamellar dentures for the upper and lower nation revealed a mechanic injury of
jaw. An orthodontist made elastic alginate mucous membrane of the palatine vault
Krok 2 Stomatology 2009 17

caused by the bar of the clasp prosthesis. of crown height by the 12, 13, 14 teeth.
What distance should exist between the Formalin test is painless. What is the most
palate and the bar of the clasp prosthesis likely diagnosis?
in order to prevent this complication?
A. Hypertrophic gingivitis
A. 0,5 mm B. Generalized II degree periodontitis,
B. 2-3 mm chronic course
C. 0,2-0,3 mm C. Catarrhal gingivitis
D. 1,5-2,0 mm D. Ulcero-necrotic gingivitis
E. 5-6 mm E. Exacerbation of generalized I degree
periodontitis
117. A 47-year-old patient complains
about limited mobility of her lower jaw 120. A 34-year-old patient got a trauma
in the morning; periodical dull pain in (fall) that resulted in mobility of alveolar
the right temporomandibular joint (TMJ) process and all the upper jaw teeth,
and general joint stiffness. According occlusion was also changed. X-ray pi-
to the patient, the stiffness disappears cture depicts the fracture line that runs in
throughout the day after joint "exerci- both directions from the piriform opening
sing". Objectively: the patient’s face is along the floor of maxillary sinus. What is
symmetric, mouth opening is limited the most likely diagnosis?
down to 2,5 cm, there is also joint clicking.
Median line deviates to the right by 3-4 A. Le Fort I maxillary fracture
mm, palpation of the right articular head B. Partial fracture of the alveolar process
is painless. What is your provisional di- C. Le Fort II maxillary fracture
agnosis? D. Le Fort III maxillary fracture
E. Unilateral maxillary fracture
A. Arthrosis of the right TMJ
B. Acute serous arthritis of the right TMJ 121. A boy is 1 month old. At the medi-
C. Chronic arthritis of the right TMJ al edge of the inferior eyelid on the ri-
D. Fracure of the right condyle of mandi- ght there is a wound with purulent di-
ble scharge. The boy fell ill suddenly, body
E. Right-sided anterior dislocation of temperature rose up to 40o C. The general
mandible condition is grave. On the second day
of disease there appeared an infiltrati-
118. A 50-year-old patient complains on at the internal edge of eye socket
about problems with mastication, tooth and right cheek. The skin above it is
mobility, halitosis, gingival haemorrhages. hyperemic, fluctuation cannot be determi-
Objectively: gums are hyperemic wi- ned. Palpebral fissure is narrowed. The
th cyanotic colouring, there is dental right nasal meatus discharges pus. There
calculus. Parodontal pouches of the is an infiltration on the vestibular surface
superior molars are 8 mm deep, the of alveolar process and on the right palate.
pouches of other teeth are 6 mm deep. Mucous membrane above it is hyperemic
X-ray picture shows resorption of bone ti- along the mucogingival fold, fluctuation
ssue by 2/3-1/2 of root length. What is the can be determined. What is the most li-
most likely diagnosis? kely diagnosis?
A. Chronic generalized periodontitis of III A. Acute hematogenous osteomyelitis
degree B. Acute dacryocystitis
B. Chronic generalized periodontitis of II C. Phlegmon of the right eye socket
degree D. Acute right-sided highmoritis
C. Chronic generalized periodontitis of I E. Acute serous periostitis
degree
D. Acute generalized periodontitis of III 122. A 20-year-old patient got an injury.
degree Objectively: the patient’s chin and lower
E. Acute generalized periodontitis of II jaw up to the 34 and 45 teeth are missing.
degree The 45, 46, 47, 48, 34, 35, 36, 37 teeth are
stable. At what stage of medical evacuati-
119. An 18-year-old patient complains on the patient will get special medical aid?
about gingival enlargement, pain and
haemorrhage when eating solid food.
Objectively: hyperaemia, gingival edema,
hypertrophy of gingival edge up to 1/2
Krok 2 Stomatology 2009 18

A. Specialized army surgical hospital remaining 18, 17, 13, 12, 11, 21, 22, 23,
B. Battalion aid station 24 teeth are stable. What fixing elements
C. Regimental aid station should be used in a clasp denture for
D. Separate medical detachment cosmetic purposes?
E. Separate medical battalion
A. Attachments and bar system
123. A 45-year-old patient complains B. Telescopic crowns
about a rapidly growing formation on his C. Ney clasps
lower lip. Examination of the red border D. Jackson clasps
of lips revealed a greyish-red nodule wi- E. Dentoalveolar clasps
th a hollow in the centre which is filled
with corneous masses that can be easily 127. A 34-year-old patient consulted a
removed. The nodule is painless, mobile. prosthodontist about pain and clicking in
What is your provisional diagnosis? the final stage of mouth opening. The pati-
ent has a history of a dislocation. Ampli-
A. Keratoacanthoma tude of maximal mouth opening is 58 mm.
B. Papilloma Mouth opening should be limited down to
C. Nodulous verrucous precancer of red the following amplitude:
border
D. Basal cell carcinoma A. Up to 40-50 mm
E. Localized precancerous hyperkeratosis B. Up to 25-30 mm
of red border C. Up to 50-60 mm
D. Up to 10-15 mm
124. A 60-year-old patient underwents E. Up to 90-100 mm
sanitation of the oral cavity before an
operation on account of cataract. After 128. A 12-year-old patient presents wi-
examination the patient was diagnosed th abnormal position of the upper jaw
with chronic median caries of the 22 tooth canine. The 13 tooth is in the vestibular
(Black’s class V). What filling material position, above the occlusal plane. Space
should not be used in this patient? between the 14 and the 12 tooth is 6,5 mm.
Choose a rational treatment method:
A. Light-cure microhybrid material
B. Chemical-cure microhybrid material A. Instrumental
C. Chemical-cure glass ionomer cement B. Surgical and instrumental
D. Silicate cement C. Surgical and physiotherapeutic
E. Chemical-cure macrofilled composite D. Instrumental and myogymnastics
E. Surgical and myogymnastics
125. A 23-year-old patient complains
about periodical pain in the region of 129. Unused stomatological instruments
the 11 tooth, protrusion of the alveolar were left on a sterile table at the end of
process. The patient got a trauma 4 years the working day. What measures should
ago. Objectively: crown of the 11 tooth be taken in order to provide sterility of
is dark, percussion is painless. X-ray pi- these instruments?
cture shows roundish well-defined area of
bone tissue rarefication by the root apex A. Sterilization without preliminary
of the 11 tooth. The area is 2,0 cm in di- processing
ameter. Puncture results: yellow fluid with B. Disinfection, sterilization
cholesterol crystals. What is the most li- C. Disinfection, presterilization treatment,
kely diagnosis? sterilization
D. Presterilization treatment, sterilization
A. Maxillary radicular cyst E. Disinfection only
B. Chronic maxillary osteomyelitis
C. Maxillary ameloblastoma 130. A 42-year-old patient complains
D. Soft maxillary odontoma about gingival pain, progressing gingi-
E. Maxillary osteoclastoma val haemorrhage, increasing tooth mobi-
lity, halitosis. Objectively: gums are evi-
126. A 45-year-old female patient dently hyperaemic, extremely edematic,
consulted a prosthodontist about dental they bleed easily on palpation. Tooth roots
prosthetics. She works as TV announcer. are exposed, parodontal pouches are 4-6
Objectively: the lower jaw dentition is wi- mm deep, and contain purulent exudate,
thout spaces, the upper jaw has a free- there is also supragingival and subgingival
end edentulous space and a boundary dental calculus. II-III grade tooth mobi-
edentulous space in the lateral parts. The lity is present. Orthopantomogram shows
Krok 2 Stomatology 2009 19

resorption of interdental septa down to ble movements. Objectively: skin over


1/2 of their height. What is the most likely the left temporomandibular joint (TMJ)
diagnosis? is reddened, the surrounding tissues
are edematic. Disfunction of the left
A. Exacerbation of generalized II degree temporomandibular joint is present. What
periodontitis is the most likely diagnosis?
B. Exacerbation of generalized I degree
periodontitis A. Acute purulent left-sided arthritis of
C. Exacerbation of generalized III degree TMJ
periodontitis B. Acute serous left-sided arthritis of TMJ
D. Chronic generalized II degree peri- C. Arthrosis of the left TMJ
odontitis D. Deforming arthrosis of the left TMJ
E. Chronic generalized III degree peri- E. Ankylosis of the left TMJ
odontitis
135. A patient complains about li-
131. A 60-year-old patient complains mited mouth opening. She has a hi-
about difficult mastication due to the story of intra-articular disorders in
mandible displacement that resulted from the left temporomandibular joint.
an untreated fracture. Objectively: the Roentgenological examination revealed
35, 36, 38, 45, 46 teeth are missing. The subchondral sclerosis of articular plates,
remaining teeth are intact. The 43, 44, 47, regular narrowing of articular cavity, limi-
48 teeth have no contact with the upper ted excursion of condylar process of the
lateral teeth, and exhibit oral deviation up left temporomandibular joint. What is the
to 1 cm. What is the optimal prosthetic most likely diagnosis?
construction for the manible?
A. Arthrosis of the left temporomandi-
A. Prosthesis with double dentition bular joint
B. Whole-piece bridge B. Deforming arthrosis of the left
C. Arch bar prosthesis temporomandibular joint
D. Adhesive prosthesis C. Ankylosis of the left temporomandi-
E. Vankevich splint bular joint
D. Acute purulent left-sided arthritis of
132. A patient complains about a cosmetic the temporomandibular joint
defect of the 23 tooth. Objectively: the E. Acute serous left-sided arthritis of the
crown of the 23 tooth is decayed by 80%, temporomandibular joint
the root is stable, the canal is filled up to
the top. After examination it was decided 136. A 46-year-old patient complains
to restore the decayed tooth with a cast of difficult opening of her mouth, body
stump inlay. The root canal of the 23 tooth temperature rise, tissue edema around
should be broadened by: both temporomandibular joints (TMJ).
It is known from the anamnesis that the
A. 2/3 of canal length patient had short-term non-intense bi-
B. 1/2 of canal length lateral pain in the parotid-masticatory
C. 1/3 of canal length region, limited mouth opening, tension
D. 1/4 of canal length and discomfort in the region of both
E. By the total canal length temporomandibular joints that had been
133. Regimental Aid Station (RAS) admi- observed for a couple of days. What is the
tted a soldier with an injury in the most likely diagnosis?
maxillofacial region. Dosimetric control A. Rheumatic arthritis
revealed radiation affection in this pati- B. Infectional arthritis
ent. The injured should be referred to the C. Arthrosis
following functional unit of RAS: D. Fibrous ankylosis
A. Decontamination station E. Deforming atrhrosis
B. Dressing pavilion 137. A boy is 10 years old. His face is
C. Evacuation pavilion symmetric and proportional. He presents
D. Isolator with mouth breath. Examination of the
E. Admission and sorting pavilion oral cavity revealed saddle-like form of
134. A 46-year-old patient complai- dental arches and high arched palate.
ns about pain in the left parotid- Upper first molar relationship (Angle’s
masticatory region, disorder of mandi- key to occlusion) remains intact. What is
Krok 2 Stomatology 2009 20

the most likely diagnosis? A. 500 lux


B. 400 lux
A. Narrowing of dental arches C. 300 lux
B. Distal occlusion D. 200 lux
C. Mesial occlusion E. 100 lux
D. Widening of dental arches
E. Elongation of dental arches 142. A patient complains about a cosmetic
defect. Examination revealed that the 21
138. A 56-year-old patient complains tooth was missing, the crowns of the 11
about a painless neoplasm on the alveolar and the 22 tooth are intact, high, relati-
crest of the right upper jaw. Objectively: vely parallel. The patient refused teeth
there is a bright-red wide-based tumour preparation as well as implantation or
in the region of premolars. It is locali- fabrication of a removable denture. What
zed on both sides from the alveolar crest. orthopedic construction should be fabri-
The tumour is dense and elastic, painless. cated?
What is the most likely diagnosis?
A. Adhesive denture
A. Peripheral osteoclastoma B. Clasp denture
B. Central osteoclastoma C. Lamellar denture
C. Gingival fibromatosis D. Immediate denture
D. Fibrous epulis E. Polypropylene denture
E. Papilloma
143. A 47-year-old patient consulted a
139. A 37-year-old patient was admitted to doctor about an itching tumour in the
the oral surgery department. The woman right buccal area that has significantly
complains of pain in her cheek that is enlarged lately. Examination revealed an
made worse by touch. It is known from the irregularly shaped tumour up to 1,5 cm
anamnesis that five days ago the patient large, the tumour is black and glossy. What
fell down from the stairs and "hurt"her is your provisional diagnosis?
cheek. Objectively: in the depth of cheek
examination revealed a circumscribed A. Dermal melanoma
infiltration, cheek skin is hyperaemic and B. Pigmented nevus
doesn’t make a fold, fluctuation symptom C. Dermal basalioma
is present. Mucous membrane is edematic D. Dermal cancer
and has teeth indentations. What is the E. Cutaneous horn
most likely diagnosis?
144. A 38-year-old patient consulted an
A. Suppurated cheek haematoma orthopedist about metal taste, dry mouth
B. Traumatic osteomyelitis of mandible and tongue burning. Objectively: defects
C. Cheek haematoma of the lower dental arch were replaced wi-
D. Cheek phlegmon th soldered stainless steel bridges. What
E. Acute lymphadenitis examination method would be the most
appropriate in this case?
140. A 40-year-old patient suffers
from chronic generalized periodontitis. A. Galvanometry
Objectively: both dentitions are intact, B. Masticatiography
the 42, 41, 31, 32 teeth exhibit I degree C. Occlusiography
mobility. What splint would be aestheti- D. Myography
cally acceptable for this group of teeth? E. Electroodontometry
A. Glass Span and photopolymer splinting 145. A 5-year-old child complains about
B. Guard splint spontaneous pain in an upper jaw tooth on
C. Splint of soldered combined crowns the right that is getting worse at night and
D. Cap splint during eating cold food. Objectively: the
E. Ring splint 65 tooth has a deep cavity communicating
with the tooth cavity. Probing is painful,
141. Renovation of a dental room involves percussion is painless. Cold water causes
installation of luminous tube lamps. Speci- long-standing pain. What is your provisi-
fy the required illuminance level in lux: onal diagnosis?
Krok 2 Stomatology 2009 21

A. Exacerbation of chronic pulpitis lasting. Electroodontodiagnosis is 60 mi-


B. Acute periodontitis croampere. X-ray picture shows slight wi-
C. Exacerbation of chronic periodontitis dening of periodontal fissure at the root
D. Acute serous pulpitis apex of the 25 tooth. What is the most
E. Acute purulent pulpitis probable diagnosis?
146. A 28-year-old patient complains A. Exacerbation of chronic pulpitis
about a cosmetic defect in the frontal part B. Acute generalized pulpitis
of his upper jaw. Objectively: the crown C. Acute purulent pulpitis
part of the 11 tooth is decayed below the D. Acute purulent periodontitis
gum level. The root is stable, percussion E. Exacerbation of chronic periodontitis
is painless. It is planned to restore the
tooth with a stump inlay and cover it with 149. A 23-year-old patient complains
a metal-ceramic crown. What additional about gingival haemorrhage during tooth
method of diagnostics should be applied brushing, intensive formation of dental
in this clinical situation? plaque despite thorough dental care.
Objectively: gingival papillae are slightly
A. X-ray diagnostics edematic, congestively hyperemic, bleed
B. Electromyography when touched. Hygiene index according
C. Masticatiography to Fedorov and Volodkina is 3,5. What
D. Gnathodynamometry toothpaste would you recommend this
E. Electroodontodiagnostics patient as a part of complex therapy?
147. Parents of a 6-year-old child complain A. Salt-containing toothpaste
about pain in the child’s submandibular B. Toothpaste containing mineralizing
region on the left, body temperature ri- components
se up to 37, 5o C. Objectively: the child’s C. Fluorine-containing toothpaste
face is asymmetric due to the infiltration D. Gel toothpaste with microelements
of the submandibular region on the left. E. Toothpaste with antifungal agents
The infiltration is soft and elastic, mobile,
2х2,5 cm large; its palpation is slightly pai- 150. A pregnant 24-year-old woman
nful, the skin is unchanged. The teeth are complains about emergence of several
intact. Pharynx is hyperaemic. What is the new carious cavities, falling out of old
most likely diagnosis? fillings. Objective examination revealed:
index of decayed, missing and filled teeth
A. Acute serous nonodontogenic (DMF) = 16, Feodorov-Volodkina hygi-
submandibular lymphadenitis ene index is 3,3. Choose the optimal
B. Acute serous odontogenic submandi- material for carious cavity filling in this
bular lymphadenitis case:
C. Acute purulent nonodontogenic
submandibular lymphadenitis A. Glass ionomer cement
D. Acute purulent odontogenic submandi- B. Silver amalgam
bular lymphadenitis C. Chemical-cure composite
E. Submandibular adenophlegmon D. Light-cure composite
E. Silicophosphate cement
148. A patient complains about
paroxysmal upper jaw toothache on 151. A group of specialists conducts an
the left that is getting worse at night. epidemiological survey of certain age
Toothache intensifies also under stimulati- groups of population aimed at evaluati-
on and irradiates to the left eye and on of periodontal disease prevalence and
temple. Similar attacks were noted three treatment needs. These rates are studied
months ago, the patient didn’t undergo by means of the following index:
any treatment. Objectively: the 25 tooth
has a deep carious cavity communicati- A. CPITN (WHO index)
ng with the tooth cavity. Probing causes B. OHI-S (Green-Vermillion index)
acute pain at the point of communicati- C. PDI (Ramfjord index)
on, vertical percussion is slightly pai- D. PI (Russel index)
nful, horizontal one is painless. Mucous E. PMA (Parma)
membrane in the projection of root apex
of the 25 tooth is unchanged, its palpati- 152. After the unproblematic extraction of
on is painless. Thermal probe causes the 37 tooth a 60-year-old patient presents
acute pain, the pain attack is long- with profuse haemorrhage from the tooth
Krok 2 Stomatology 2009 22

socket. The patient has a 6-year history 156. A 68-year-old patient complains
of essential hypertension. Now his AP about pain in the palate that occurs duri-
is 180/110 mm Hg. What emergency aid ng wearing a complete removable denture
should be rendered? the patient got 3 months ago. Objectively:
palatine torus is strongly marked, mucous
A. Injection of hypotensive drugs and tight membrane around it is hyperaemic and
pack of the tooth socket edematic. What is the most probable
B. Pack of the tooth socket with cause of this complication?
haemostatic sponge
C. Pack of the tooth socket with iodoform A. Torus wasn’t isolated
tampon B. Previous impression was made in plaster
D. Suture ligature of the tooth socket C. Functional impression was taken by
E. Injection of haemostatic drugs means of a rigid individual tray
D. It was made anatomic teeth
153. A 48-year-old patient complains arrangement
about permanent pain in the region of E. Prosthesis base is made of acrylic plastic
the 38 tooth. She has been suffering from
this for 3 days. Crown of the 28 tooth is 157. A 27-year-old patient complains of a
completely decayed. What forceps should swelling in the region of her lower jaw on
be applied for extraction of roots of the 28 the right. Objectively: the patient’s face is
tooth? slightly asymmetric due to thickening of
mandibular body on the right. Adjacent
A. Bayonet-shaped forceps soft tissues are of unchanged colour and
B. S-shaped forceps curved left normal consistency. Mouth opening is not
C. Straight root forceps limited. Mucous membrane of oral cavi-
D. S-shaped close-beak forceps ty presents no changes. In the right lower
E. S-shaped broad-beak forceps dentition the 45 tooth is missing. X-ray pi-
cture of the right half of the mandible in
154. During military operations the head its lateral view shows an oval well-defined
of the Army Medical Department gave 2x3 cm large radiolucency in the bone ti-
the order to deploy a stomatological aid ssue. The coronal portion of the retinated
unit on the territory of the Army Hospi- horizontally positioned 45 tooth is turned
tal Base. A patient with missile wound of inward the radiolucent area. What is the
face was admitted to the unit. What kind most probable diagnosis?
of aid can be rendered in the stomatologi-
cal unit? A. Follicular cyst of mandible
B. Adamantinoma of mandible
A. Specialized (secondary) aid C. Sarcoma of mandible
B. Qualified aid D. Odontoma of mandible
C. Consultation aid E. Osteoclastoma of mandible
D. Stomatological aid
E. Prosthodontic aid 158. A 52-year-old patient complains
about a nonhealing ulcer on his lower
155. A 62-year-old patient has a median lip. The patient is smoker. He hasn’t ever
fracture of mandible along with formati- consulted a doctor about it. In the region
on of a false joint. Objectively: dental of red border of the lower lip a roundi-
formula is 33, 34, 35, 36, 27, 47, 46, 45, 44, sh ulcer is present. It is up to 2,0 cm in
43. The teeth are intact, stable, with high diameter. The ulcer edges are thickened
crowns. Fragment mobility is insignificant, and a little bit raised in form of a whitish
there is no displacement. X-ray picture swelling. In the left submandibular regi-
shows a bone defect 0,8 cm large. What on palpation revealed enlarged, painless,
prosthesis is indicated? dense lymph nodes with limited mobility.
A. Bridge-like prosthesis with a pivot point What is the most likely diagnosis?
B. Lamellar prosthesis without a pivot A. Cancer of the lower lip
point B. Erosive verrucous leukoplakia
C. Lamellar prosthesis with Gavrilow’s C. Keratoacanthoma
pivot point D. Fibroma of the lower lip
D. Lamellar prosthesis with Oxman’s pivot E. Syphilitic ulcer
point
E. Lamellar prosthesis with Weinstein’s 159. A 25-year-old patient complains
pivot point about acute pain in the mouth, headache,
articular pain, body temperature rise up to
Krok 2 Stomatology 2009 23

38, 6oC. Red border of lips is covered with appeared several months ago. Objecti-
haemorrhagic crusts, mucous membrane vely: the patient has a lot of carious
of the oral cavity has big erosions and and completely decayed teeth, lateral
ulcers coated with greyish incrustation. surface of tongue exhibits a painless whiti-
Hand skin exhibits erythematous spots 1- sh formation 10x5 mm large with irregular
1,5 cm in diameter with a vesicle in the surface in form of verrucae. Histologi-
middle. What is the most likely diagnosis? cal examination revealed thickening of
corneal epithelial layer of intermittent
A. Stevens-Johnson syndrome keratinization type. What is the most li-
B. Behcet’s syndrome kely diagnosis?
C. Lyell’s syndrome
D. Multiform exudative erythema A. Verrucous form of leukoplakia
E. Medicamentous stomatitis B. Verrucous precancer
C. Hyperplastic form of candidiasis
160. A 14-year-old child complains about D. Hyperkeratotic form of lichen ruber
acute spontaneous spasmodic pain in an planus
upper jaw tooth on the right. The pain E. Keratoacanthoma
has been lasting for 3 days, it is throbbing,
irradiating to the temple, getting worse at 164. A 35-year-old man has a medi-
night. Objectively: surface of the 15 tooth um deep carious cavity in the 37 tooth
exhibits a carious cavity within parapulpar (Black’s class II). For its filling a denti-
dentine. Dentine is softened, of greyish st chose technique of layer-by-layer
colour. Probing of the whole cavity floor is restoration. What composite should be
painful, percussion of the 15 tooth is pai- coating the floor and walls of the carious
nless. What is the most likely diagnosis? cavity in order to form superadaptive ini-
tial layer?
A. Acute purulent pulpitis
B. Acute diffuse pulpitis A. Flowable
C. Acute focal pulpitis B. Condensable
D. Acute periodontitis C. Macrofilled
E. Exacerbation of chronic periodontitis D. Microhybrid
E. Microfilled
161. A 3-month-old child has an inborn
tissue defect in the oral cavity. Objecti- 165. A 9-year-old boy presents with
vely: the lip is intact, the oral cavity exhi- face asymmetry due to the chin devi-
bits a cleft defect of soft palate and medi- ation to the left. When the third Il’ina-
an part of hard palate. The child was di- Marcosian diagnostic test is performed
agnosed with complete schistasis of soft face asymmetry disappears. What is the
palate and partial schistasis of hard palate. most likely clinical form of this occlusal
Specify the dispensary group according to anomaly?
A.A.Kolesov:
A. Habitual deviation of mandible
A. First B. Ankylosis of the temporomandibular
B. Second joint
C. Third C. Unilateral hypoplasia of mandible
D. Fourth D. Bilateral narrowing of the maxillary
E. Fith dental arch
E. Unilateral narrowing of the maxillary
162. A dentist treats a 22-year-old patient dental arch
on account of acute deep caries of the 26
tooth. As lining material the dentist chose 166. A completely edentulous 70-year-old
calcium salicylate cement "Life". What patient has ordered complete removable
material should be chosen for insulation dentures. Artificial teeth are placed upon
of the lining? the spherical surface. Close teeth contact
by movements of the lower jaw will be
A. Glass ionomer cement guaranteed by the following average radi-
B. Insulating varnish us of the spherical surface:
C. Adhesive composite system
D. Zinc phosphate cement
E. Zinc oxide eugenol cement
163. A 62-year-old patient complains of
a painless formation on his tongue that
Krok 2 Stomatology 2009 24

A. 9 cm A. The dentist should break the procedure


B. 5 cm off and give the patient sugar in any form
C. 7 cm B. The dentist should lay the patient down
D. 12 cm on a flat surface, lower cephalic pole, open
E. 18 cm the windows
C. The dentist should give the patient
167. A 40-year-old patient consulted a nitroglycerine in form of spray or subli-
prosthodontist about upper jaw teeth ngually
mobility that turned up after therapeutic D. The dentist should make intramuscular
treatment. Objectively: the 13, 12, 11, injection of adrenaline (0,5-1,0 mg)
21, 22 teeth are pulpless, exhibit II E. The dentist should make intravenous
degree mobility, unchanged colour. What injection of corticosteroids
prosthetic construction should be appli-
ed? 171. Tooth replacement with complete
removable dentures involves adjustment
A. Mamlock splint of occlusal relations by different
B. Crown soldered splint movements of the lower jaw. Transversal
C. Elastic crown splint movements of the lower jaw are initiated
D. Cast bar splint by the following muscle:
E. Wire-band splint
A. External (lateral) pterygoid muscle
168. A 5-year-old child was bitten by a dog B. Internal (medial) pterygoid muscle
2 days ago. The child is diagnosed with a C. Temporal muscle
bite wound of cheek. The parent’s didn’t D. Mastication muscle
appeal for medical aid in proper time. The E. Digastric muscle
wound exhibits pyoinflammatory process.
What kind of surgical d-bridement should 172. A 50-year-old patient complains
be performed? about pain in the region of the left
temporomandibular joint (TMJ) during
A. Secondary mouth opening. Dental formula is 14, 13,
B. Initial early 12, 11, 21, 22, 23, 24, 33, 32, 31, 41, 42, 43,
C. Initial delayed 44. While opening the mouth the lower
D. Initial late jaw moves in a zigzag manner towards the
E. Surgical procedure is not required affected joint. What pathological conditi-
169. A 16-year-old teenager complains on are these symptoms typical for?
about halitosis, general weakness, body A. Musculoarticular dysfunction
temperature rise up to 37, 6o C. These B. Habitual TMJ dislocation
symptoms turned up 2 days ago, the boy C. Sclerosing arthrosis of TMJ
has a history of recent angina. Objecti- D. Chronic arthritis of TMJ
vely: oral cavity hygiene is unsatisfactory, E. Deforming arthrosis of TMJ
teeth are covered with soft white deposit.
Gums are hyperaemic, gingival papillae 173. A 35-year-old patient is at a denti-
are covered with greyish coating. What is st. He is anxious, complains of being
the most likely diagnosis? afraid of the dentist, is unwilling to let
the dentist examine him. Drugs of which
A. Ulcero-necrotic gingivitis group should be given the patient before
B. Acute catarrhal gingivitis stomatological manipulations?
C. Hypertrohic gingivitis
D. Chronic catarrhal gingivitis A. Sedative
E. Desquamative gingivitis B. Analgetics
C. Hypotensive
170. A patient being at a dentist presented D. Cardiac glycosides
suddenly with tachycardia, sweating, E. Desensitizing
trembling, nausea, hunger. The patient is
conscious. He suffers from diabetes melli- 174. A 76-year-old patient has a roundish
tus. What aid should be rendered by the ulcer in the area of the right nasolabial
dentist? sulcus. The ulcer has dense, rough floor
covered with yellowish-grey crusts, and
resembles of a cone. The ulcer edges are
dense, crater-like; the ulcer grows both
sidewards and deepwards. What is your
provisional diagnosis?
Krok 2 Stomatology 2009 25

about the astringent sensation in the


A. Basalioma upper jaw incisors, which appeared 1,5
B. Tuberculous ulcer years ago. Objectively: the most convex
C. Actinomycotic ulcer part of the vestibular surfaces of the 12,
D. Syphilitic ulcer 11, 21, 22 crowns has roundish defects wi-
E. Squamous cell carcinoma of skin th smooth, shiny, dense surface, that reach
dentine in depth. The depth of defects is
175. A 40-year-old patient complains gradually decreasing from the centre to
about a long-existing fissure in the central the periphery. What is the most likely di-
part of red border of his lower lip. He has agnosis?
been smoking from the age of 16. Objecti-
vely: there is a deep 1,0 cm long fissure A. Enamel erosion
along the median line of red border of B. Wedge-shaped defect
the lower lip, the fissure edges are bulgi- C. Systemic hypoplasia
ngl. Characteristic quality of the fissure D. Chronic median caries
is its spontaneous healing, but after epi- E. Destructive fluorosis
thelization the fissure recurs. The pati-
ent was diagnosed with chronic fissure 179. Examination of a 6-year-old boy
of the lower lip. Conservative treatment revealed enlarged lymph nodes in both
proved to be ineffective. Choose the most submandibular and cervical regions.
appropriate treatment method in this si- Objectively: the 75, 84 and 85 teeth
tuation: are decayed, there are presentations of
commissural cheilitis. According to the
A. Excision within evidently healthy ti- boy’s mother, he has been quickly getti-
ssues ng tired, sweating from the least physi-
B. Cryodestruction cal exercise, complaining about weakness
C. Diathermo-coagulation throughout the last 2-3 months. He also
D. Excision 0,5 cm off the neoplasm limits gave up training in a sports class. What
E. Close-focus roentgenotherapy plan of additional examination should be
adopted?
176. It is planned to lance an abscess
of hard palate (it is localized close to A. Complete blood count, haematologist
the 23, 24, 25 teeth) and to extract the consultation
causative 24 tooth that had caused abscess B. Biochemical blood test, endocrinologist
development. What type of anaesthesia is consultation
indicated for this operation? C. Puncture biopsy, oncologist consultation
D. Magnetic resonance tomography,
A. Infraorbital, palatinal and incisor immunologist consultation
B. Tuberal, infraorbital and incisor E. Pulmonary roentgenography,
C. Tuberal, infraorbital and palatinal pulmonologist consultation
D. Tuberal and incisor
E. Tuberal and palatinal 180. A 32-year-old patient complains
about pain and swelling in the region of
177. A 36-year-old patient consulted a his mandible and left cheek. He has been
dentist about permanent acute pain in the suffering from this for 2 days. Objecti-
upper jaw teeth, body temperature rise. vely: his general condition is satisfactory.
The dentist found out that the 26 tooth Body temperature is 37, 5o C. There is an
had been repeatedly treated, the rest of edema of soft tissues in the left buccal
teeth were intact. After roentgenological region. Submandibular lymph nodes are
examination the patient was diagnosed bilaterally enlarged, painful on palpati-
with acute highmoritis. What is the most on. Crown of the 37 tooth is partially
effective way of treatment? decayed, the tooth is immobile, percussi-
on is slightly painful. Mucogingival fold
A. Extraction of the causative tooth and in the region of the 37 tooth is edematic,
medicamentous therapy hyperaemic, flattened, its palpation causes
B. Caldwell-Luc maxillary sinusotomy acute pain. What is the most likely di-
C. Puncture of the maxillary sinus and agnosis?
medicamentous therapy
D. Medicamentous therapy
E. Medicamentous and physiotherapeutic
treatment
178. A 44-year-old patient complains
Krok 2 Stomatology 2009 26

A. Acute purulent periostitis of mandible permanent intense throbbing toothache


beginning from the 37 tooth that is getting worse when biting down
B. Acute odontogenous osteomyelitis of on food. Objectively: the patient’s face
mandible is asymmetric because of a collateral
C. Chronic productive periostitis of mandi- edema of soft tissues, submandibular
ble beginning from the 37 tooth lymph nodes are enlarged and painful on
D. Chronic odontogenous osteomyelitis of palpation. The 26 tooth has a deep cavity
mandible not communicating with the tooth cavity.
E. Abscess of alveololingual groove on the Thermal test is positive, probing is pai-
left nless, percussion causes acute pain, the
tooth is mobile. X-ray picture of the 26
181. A 12-year-old girl complains about tooth shows no changes in the periodonti-
swelling, reddening and itching of lips. She um. What is the most likely diagnosis?
has a history of bronchial asthma. Such
condition has been observed in the child A. Acute purulent periodontitis
for 3 years in winter period. Objectively: B. Acute serous periodontitis
red border of lips and adjacent skin exhi- C. Exacerbation of chronic periodontitis
bit a well-defined erythema and edema, D. Acute purulent pulpitis
small cracks. There are signs of lip li- E. Pulpitis complicated by periodontitis
chenification. There are also marks of
scratches on the skin of lips, cheeks and 185. A 6-year-old girl was brought to the
forehead in form of abrasions and crusts. dentist for completing the treatment of
Mucous membrane exhibits no pathologi- the 75 tooth for chronic granulating peri-
cal changes. What is the most likely di- odontitis. The patient has no complaints.
agnosis? Objectively: occlusive dressing on the 75
tooth remains intact, percussion is pai-
A. Atopic cheilitis nless, mucous membrane in the region
B. Contact allergic cheilitis of the 75 tooth exhibits no pathological
C. Meteorological cheilitis changes, is painless on palpation. What
D. Exfoliative cheilitis material is the most appropriate for the
E. Microbal cheilitis root canal filling in this case?
182. Parents of a 5-year-old child noted A. Zinc oxide eugenol paste
irregular colour of the child’s back of B. Sealer with gutta-percha point
tongue and consulted a pedodontist C. Calcium-containing paste
about this. Objectively: the oral cavity D. Glass ionomer cement
is sanitized, DF index = 4, dorsal and E. Iodoform paste
lateral surfaces of tongue have area of
desquamating epithelium bordering upon 186. A 4-year-old child presents with
areas of hypertrophy and increased corni- headache, nausea, pain when swallowi-
fication of filiform papillae. The child ng. Body temperature is 39o C. Examinati-
has a history of being allergic to some on of the oral cavity revealed extremely
foodstuffs. What is the clinical diagnosis? hyperaemic mucous membrane and tonsi-
ls. On the first day of disease the child’s
A. Desquamative glossitis tongue was furred with greyish coating,
B. Tuberculous glossitis on the second day the tongue was found
C. Candidous glossitis to be self-purified. On the third day the
D. Herpetic tongue lesion tongue was smooth, of crimson colour, wi-
E. Acute catarrhal glossitis th hyperaemic fungiform papillae. What is
presumptive diagnosis?
183. A 4-year-old girl was bitten by a dog
in her upper lip. Which of the following A. Scarlet fever
solutions should be primarily used for the B. Varicella
wound d-bridement? C. Measles
D. Diphtheria
A. 10% solution of laundry soap E. Herpangina
B. 0,002% solution of chlorhexidine
C. 1% solution of hydrogen oxide 187. A 15-year-old child presents with
D. 1 : 5000 solution of potassium puffiness in the region of the mandi-
permanganate ble branch; enlarged, dense and painless
E. 3% soda solution lymph nodes adhering to the surrounding
tissues. X-ray picture of mandible branch
184. A 12-year-old boy complains about shows a well-defined bone resorption
Krok 2 Stomatology 2009 27

area containing small sequestra. After 191. A 22-year-old patient is afraid of pain
Mantoux test a 12 mm papule was noted. from conduction anaesthesia. A dentist
What is the most likely diagnosis? decided that this anaesthesia should be
preceded by applicational anaesthesia of
A. Tuberculosis of mandible branch mucous membrane on the spot of injecti-
B. Mandibular actinomycosis on. What drug should be used for this
C. Chronic osteomyelitis of mandible purpose?
branch
D. Acute mandibular osteomyelitis A. 5% lidocaine ointment
E. Ewing’s sarcoma B. 1% synthomycin ointment
C. 5% oxacillin ointment
188. An 11-year-old child complains about D. 3% sinaflan ointment
missing crown of the 12 tooth as a result E. 3% doxycyclin ointment
of a trauma. The tooth root is well treated.
What prosthetic construction is indicated 192. A 12-year-old girl complains about
for elimination of this defect? intense pain in the region of the 46 tooth
socket that was extracted 3 days ago. The
A. Il’ina-Marcosian’s pivot tooth pain is irradiating along the branches of
B. Cantilever prosthesis supported by the trigeminus. Objectively: lymph nodes are
11 tooth enlarged and painful on palpation, tissues
C. Cantilever prosthesis supported by the around the tooth socket are edematic
13 tooth and hyperaemic. The socket walls are
D. Bridge-like prosthesis supported by the covered with grey-and-green deposition
13 and 11 teeth with putrid smell. What is the most likely
E. Partial removable replacing prosthesis diagnosis?
189. A 34-year-old patient has indicati- A. Alveolitis
on for extraction of the 38 tooth because B. Pericoronaritis
of chronic fibrous periodontitis. Oral C. Ostitis
surgeon performed torus anaesthesia. D. Periostitis
This anaesthesia blocked transmission E. Osteomyelitis
of pain information from the following
nerves: 193. A 42-year-old patient consulted
a dentist about intense lancinating
A. Infraalveolar, lingual, buccal paroxysmal pain accompanied by a
B. Lingual, buccal sensation of current passage in the regi-
C. Infraalveolar, buccal on of her upper lip on the right. Pain
D. Infraalveolar, lingual attacks occur spontaneously and last 3-
E. Lingual, buccal, mental 5 minutes. The patient usually has 2-3
attacks a day. The patient is unable to
190. A patient got a trauma that establish the cause of this disease. Exami-
resulted in limited mouth opening, nasal nation of her oral cavity revealed no
haemorrhage, numbness of inferior eyelid pathological changes. What is the most
as well as skin in the infraorbital area. likely diagnosis?
Objectively: there is face deformation
caused by depression of soft tissues in the A. Peripheral neuralgia of the II branch of
malar region on the left; there is also a trigeminus
step-off deformity in the median part of B. Pterygopalatine ganglionitis
the infraorbital rim and in the region of C. Neuritis of the II branch of trigeminus
zygomatico-alveolar crest. What method D. Central neuralgia of the II branch of
of examination will be the most effective trigeminus
for the diagnostics? E. Right-sided upper jaw pulpitis
A. X-ray study of facial skull in the axial 194. A 23-year-old patient was deli-
projection vered to a traumatology centre with
B. X-ray study of facial skull in the frontal a dirty cut wound of her right foot.
projection A doctor performed initial surgical d-
C. X-ray study of facial skull in the lateral bridement and made an injection of anti-
projection tetanus serum. Some time later the pati-
D. X-ray study of facial skull and paranasal ent’s condition got abruptly worse: she
sinuses developed extreme weakness, dizziness,
E. Orthopantomogram palpitation. Objectively: the skin is pale,
the patient has cold sweat, frequent pulse
Krok 2 Stomatology 2009 28

of poor volume at a rate of 100 bpm, AP A. Start cardiopulmonary resuscitation


is 90/40 mm Hg. What is the cause of such B. Deliver the patient to the hospital as
aggravation? soon as possible
C. Make an intracardiac injection of
A. Anaphylactic shock adrenaline
B. Haemorrhagic shock D. Facilitate oxygen inhalation
C. Pain shock E. Make an intravenous injection of
D. Infectious-toxic shock polyglucinum
E. Drug disease (seroreaction)
198. An 18-year-old woman has been
195. A man was found in a street suffering from diabetes mellitus for 5
in unconscious state and delivered to years. She receives 36 units of insulin per
the sanitary inspection room. At small day. During pneumonia her condition got
intervals the man presents with recurrent abruptly worse: the patient presented with
attacks of tonic and clonic convulsions of increased thirst, abdominal pain, nausea,
the upper and lower limbs. Objectively: vomiting and sleepiness. In the evening
the patient is unconscious, his pupils are the patient refused to eat and didn’t get
mydriatic, don’t react to light. The tongue her regular insulin dose; in the morning
has teeth indentations. Involuntary uri- she lost consciousness. Objectively: the
nation was noted. Examination revealed patient is unconscious, the skin is dry,
no symptoms of focal cerebral lesion. AP turgor is decreased. The tongue is also
is 140/90 mm Hg. The patients head can dry. Respiration is deep and noisy, there is
be freely brought to the chest. Specify the sharp smell of acetone from the mouth.
most probable pathology: Body temperature is 36, 6o C, Ps is 100
bpm, small; AP is 90/50 mm Hg. Urine
A. Epilepsy, status epilepticus exhibits extremely positive reaction to
B. Parenchymatous hemorrhage acetone. Blood glucose is 33 millimole/l.
C. Cerebral infraction What is your
D. Tetanus
E. Acute meningoencephalitis A. Ketoacidotic coma
B. Hyperosmolar coma
196. A patient was found unconscious C. Lactacidemic coma
in her flat heated with firewood. The D. Hepatic coma
stove pipe damper was closed, it smelt of E. Cerebral coma
smoke in the room. Objectively: the pati-
ent is unconscious, her skin and mucous 199. A 28-year-old man attended a
membranes are of cherry red colour. Pupi- glasshouse in a botanic garden. After he
ls exhibit no reaction to light, corneal and had smelt at an orchid he turned pale and
tendinous reflexes are absent, muscle tone lost consciousness. Objectively: heart rate
is decreased. Ps is 96 bpm, of poor volume. is 115/min, arterial pressure is 50/0 mm
Heart sounds are muffled, there are soli- Hg. What drug should be injected to the
tary extrasystoles, AP is 90/60 mm Hg. patient in the first place?
In the lower lungs solitary small bubbli-
ng rales are present, respiratory rate is A. Prednisolone
28/min. What is the most likely diagnosis? B. Cordiamin
C. Strophanthine
A. Acute intoxication with carbon oxide D. Mesaton
B. Acute chlorine intoxication E. Dimedrol
C. Acute intoxication with sulfur dioxide
gas 200. A 68-year-old patient has been taking
D. Acute methane intoxication diclofenac because of pain in the knee joi-
E. Acute benzol intoxication nts for about a year. In the evening after
substantial meal the patient presented wi-
197. During transportation of a 60-year- th liquid black stool, drastic weakness,
old patient an emergency doctor noted single vomiting after which the patient
aggravation of his general condition, lost consciousness. Emergency team deli-
pulselessness, mydriatic pupils, absence of vered him to the hospital. Objectively:
respiratory movements. It will take 5 mi- the patient is conscious, with slightly
nutes to get to the admission ward. What deferred reaction, pale. Heart rate is 20
actions should the doctor take in the first breaths/min, Ps is 102 bpm, AP is 110/70
place? mm Hg. Abdomen is soft, painful in the
epigastrium. What is the most likely di-
Krok 2 Stomatology 2009 29

agnosis? A. Gastrointestinal haemorrhage


B. Food poisoning
C. Myocardium infarction
D. Apoplectic attack
E. Thrombosis of mesenterial arteries
2023- May-k2-Dentistry Krokology.com

Important notes ✍️🙋‍♂️


Revision 3
• This is revision 2 of May-2023. Make sure you have the latest revision by
clicking here.
• Color indication:
- Red color: confirmed by the official site or repeated before.
- Green color: not confirmed but that’s the likely answer.
- Purple: can’t find a valid explanation for it
• If any answer is spotted in official site, we’ll update it. Make sure to practice
on Krokology to get the recent update.
• Thanks to Dr. Elias and Rabee for doing the first revision.

1. A 53-year-old patient has been C. Chemical adhesion to dental hard


diagnosed with a non-displaced tissues
mandibular fracture in the frontal D. Polymerization shrinkage
region. Objectively, there are fixed E. Release of fluorine ions
orthopedic appliances on the upper
jaw, the lateral group of teeth is 3. A 2-year-old girl developed a
missing on the both sides of the lower fever of 38°C that lasts for three days
jaw, the teeth located from canine to already. She is fussy and refuses to
canine ate intact. What splint should eat. Examination detects numerous
be chosen for the treatment in this small crosions covered in a fibrinous
clinical case? plaque on the brightly. hyperemic
A. Ivy ligature binding mucosa of the lips, checks and soft
B. Port splint palate. Submandibular lymph nodes
C. Limberg splint are painful and slightly enlarged. The
D. Vankevich splint child has been diagnosed with acute
E. Vasilyev splint catarrhal gingivitis. What medicine
should be prescribed for etiotropic
2. A 28-year-old woman came to treatment in this case?
a dentist with complaints of a carious A. Metrogyl (Metronidazole)
cavity in her lower right tooth and B. Clotrimazole
food getting stuck in the affected C. Solcoseryl
area. The following diagnosis was D. Acyclovir
made: chronic median caries of tooth E. Hydrocortisone
47 Black's class II. What properties
of glass ionomer cements makes 4. A 12-year-old girl complains
them an unsuitable material for of periodic bleeding in the area of her
filling in this case? lower right tooth. Objectively, the
A. Insufficient aesthetic masticatory surface. of tooth 46 has a
characteristics carious cavity filled with a soft red
B. Fragility tissue. Superficial probing is mildly

Dentistry Telegram
2023- May-k2-Dentistry Krokology.com

painful, while deeper probing causes made for him. Before the preparation
pain and bleeding. The tooth is of the hard tissues of the intact teeth,
almost unresponsive to thermal the patient was given an infiltration
stimuli, Percussion. of tooth 46 is anesthesia with 0.5 % lidocaine
painless. X-ray detects no solution. Several minutes later, he
pathological changes in the developed depressed level of
periodontal region of the tooth. What consciousness, his blood pressure
diagnosis can be made in this case? dropped, convulsions started, and
A. Chronic hypertrophic pulpitis involuntary urination occurred. What
B. Chronic localized hypertrophic emergency condition did the patient
gingivitis develop?
C. Chronic granulating periodontitis A. Unconsciousness
D. Chronic librous pulpitis B. Pain shock
E. Chronic gangrenous pulpitis C. Heart failure
D. Anaphylactic shock
5. A patient complains of burning E. Collapse
in the area of the hard palate mucosa
when using a partial removable 7. A bugel (clasp) denture is to be
laminar denture made. one week ago. made for the upper jaw of a 54-year-
The following diagnosis has been old man. The working impression has
made allergic contact stomatitis been obtained with. a silicone
Determine the treatment tactics, if it impression material, while the
is t known that there are no auxiliary impression has been made
conditions for making an arch with an alginate material. How many
denture and the patient is using a casts are necessary to make one bugel
removable denture for the first time. denture?
A. Make a new removable denture A. Two working casts and two
with a metal base. auxiliary casts
B. Apply the previously made B. One working cast and one
denture again with frequent auxiliary cast
rinsing with water and taking C. One working cast
hyposensitizing agents D. Two working casts and one
C. Boil the previously made denture auxiliary cast
in distilled water for 10 minutes E. One working cast and two
D. Make a new denture, adding a auxiliary casts
smaller amount of polymer to the
plastic dough 8. A dental orthodontist has
E. Make a new denture, adding a diagnosed an 11-year-old child with
smaller amount of monomer to the microdontia of the upper incisors.
plastic dough What diagnostic method was used in
this case?
6. A 45-year-old man came to an A. Gerlach
orthopedic dentist to have a denture B. Korkhauz

Dentistry Telegram
2023- May-k2-Dentistry Krokology.com

C. Howes would be optimal for fixing an


D. Tonn artificial crown in this case?
E. Pont A. Latelux
B. Filtek Supreme
9. A 6-year-old child is being C. Unifas
treated for acute diffuse pulpitis of D. Evicrol
tooth 85 using the technique of pulp E. Fuji I
extirpation. What type of material
would be optimal in this case for 12. A 73-year-old woman came to
permanent obturation of root canals? a dentist with complaints of angular
A. Calcium hydroxide-based pastes cheilitis. She has been using
B. Epoxy resin-based pastes with complete removable laminar
gutta percha pins dentures for the upper and lower jaw
C. Zinc phosphate cements for 5 years. Objectively, the lower
D. Zinc eugenol pastes third of her face is shortened, the
E. Glass ionomer cements mouth angles are downturned and the
skin there is macerated. What is the
10. A 45-year-old woman most likely cause of this
complains of bleeding gums. The complication?
bleeding occurs when she brushes her A. Age of the patient.
teeth or eats hard food. Objectively, B. Reduction of the interalveolar
in the frontal area of her lower jaw height
the gums are hyperemic, edematous, C. Deformation of the denture bases
and bleed when palpated. In the other D. Poor oral hygiene
areas, her oral mucosa is normal. The E. Permanent usage of the dentures
woman has a deep bite. Her teeth are
stable, except for teeth 41 and 31 that 13. During examination by a
have the I degree of mobility X-ray dental orthodontist, a 10-year-old
shows resorption of the interalveolar child presents with a chin that
septa in the area of teeth 42, 41, 32, protrudes forwards, a sunken upper
and 31 to 1/3 of the root length. What lip, and a concave facial profile.
diagnosis can be made in this case? Intraoral examination detects a
A. Generalized periodontitis, initial mesial shift of the crowns of teeth 36
stage and 46, the upper front teeth are
B. Generalized periodontitis, I overlapped by the lower front teeth,
degree the upper front teeth I have an oral
C. Localized periodontitis inclination. What appliance should
D. Catarrhal gingivitis be used to treat this pathology?
E. Periodontosis, I degree A. Frankel functional regulator, type
1
11. An orthopedic dentist fixes an B. Khurgina appliance
artificial porcelain-fused-to-metal C. Oral vestibular plate
crown to vital tooth 47 What material D. Klammt activator

Dentistry Telegram
2023- May-k2-Dentistry Krokology.com

E. Bruckl appliance pulse is of poor volume, the blood


pressure is 90/60 mm Hg. What is the
14. The parents of an 8-year-old most likely diagnosis in this case?
boy came to a dentist complaining of A. Epilepsy attack
a broken. off crown of a child's front B. Unconsciousness
tooth. The coronal tooth fracture C. Hyperglycemic coma
occurred 2 hours ago as a result of an D. Anaphylactic shock
injury sustained during a sports E. Collapse
competition. Objectively, one third
of the crown of tooth 11 is missing, 17. A 42-year-old man complains
the pulp is exposed at one point. of periodic pain in the area of his
bright red, and sharply painful to tooth 48. Objectively, tooth 48 has
probing Tooth percussion is tender. partially erupted with its mesial
What treatment method would be cusps, mucosa is pale pink and
optimal for tooth 11 in this case? painless to palpation, the mouth
A. Devital extirpation opening is unrestricted. X-ray shows
B. Conservative treatment tooth 48 to be located medially to
C. Tooth extraction tooth 47 Select the surgical treatment
D. Vital amputation method that would be optimal in this
E. Vital extirpation case.
A. Typical extraction of tooth 48
15. Removable complete dentures B. Operculum remova.
are being made for a 70-year-old man C. Bicuspidization
with completely edentulous jaws and D. Operculum dissection.
marked progenia. What artificial E. Atypical extraction of tooth 48
teeth are not placed in such dentures?
A. Second molars on the lower jaw 18. Removable dentures were
B. Second premolars on the lower made for a 5-year-old child due to the
jaw early loss of deciduous second
C. Second molars on the upper jaw molars. When will the dentures need
D. Second premolars on the upper replacing in this case?
jaw A. In 1.5 years.
E. First premolars on the lower jaw B. In 2.5 years
C. In 2 years
16. During the frenuloplasty of the D. In 0.5 years
superior labial frenulum, a 9-year-old E. In 1 year
girl, who received infiltration
anesthesia with 1.0 mL of Sol. 19. A patient complains of a
Ultracain DS, complained of neoplasm in the left half of the frontal
dizziness, nausea, weakness, and region. The neoplasm appeared 2
ringing in her ears. Objectively, her months ago and is slowly increasing
skin is pale and covered in cold in size. It is dense, elastic, round,
sweat, her respiration is rapid, the mobile, and painless. The skin color

Dentistry Telegram
2023- May-k2-Dentistry Krokology.com

over the neoplasm is unchanged. It 22. A 62-year-old woman


has a punctate indentation on its complains of intense pain attacks in
surface, from which whitish foul- the area of her left. lower jaw that last
smelling masses are discharged. 1-3 minutes. The pain occurs during
Make the provisional diagnosis. talking washing her face, and
A. Atheroma touching the skin. Mandibular X-ray
B. Keratoacanthoma detects no pathological changes in
C. Exophytic form of skin cancer tissue. Make the provisional
D. Lipoma diagnosis.
E. Fibroma A. Pterygopalatine ganglionitis
B. Neuralgia of the second branch of
20. A 47-year-old man complains the trigeminal nerve
of an ulcer on the back of his tongue. C. Neuritis of the third branch of the
Hist history states that before the trigeminal nerve
ulcer, there. was a tuberculous D. Neuralgia of the third branch of
infiltrate for a long time. The the trigeminal nerve
infiltrate was gradually enlarging and E. Facial nerve neuritis
then an ulcer formed. Objectively.
there is a shallow elongated ulcer on 23. A 48-year-old man complains
the dorsum of the patient's tongue. of a burning sensation in his mouth
The ulcer is painful, has overhanging that occurs. when he wears a
edges and is surrounded with removable partial laminar denture for
tubercles. Make the provisional the lower jaw that was made for him
diagnosis. 2 weeks ago Objectively, he has
A. Actinomycosis of the tongue hypersalivation, his oral mucosa is
B. Tuberculous ulcer hyperemic and edematous.
C. Tongue abscess Previously, the patient had been
D. Primary syphilis using a removable partial laminar
E. Decubitus ulcer denture, made for him 4 years ago,
and he had no such complaints. What
21. A dentist made a porcelain- is the most likely cause of the
fused-to metal crown for tooth 11 of complication in this case?
a 34-year-old patient. The tooth was A. Incorrect usage of the denture
prepared in compliance with all the B. Poor quality of the denture base
requirements. At what stage of plastic
crown-making was gum retraction C. Allergy to the components of the
performed? denture. base plastic
A. After obtaining the impression D. Poor oral hygiene
B. Before fitting the frame E. Residual monomer remaining in
C. Before obtaining the impression the denture base
D. After fitting the frame
E. After the crown fixation 24. During examination, an 8-
year-old child was diagnosed with

Dentistry Telegram
2023- May-k2-Dentistry Krokology.com

torsiversion of the lateral maxillary In this case, they are doing the procedure
incisors and an arch length deficiency from 0, nothing have been placed yet so it’ll
take the whole 6 months.
caused by macrodontia. To prevent
the vestibular position of the canines, 27. A 56-year-old woman
a Hotz serial extraction undergoes a preventive examination
was performed. What is the correct by a dentist. She has an oval erosion
order of teeth extraction in this case? on the vermilion border of her lower
A. Deciduous canines, first lip. The erosion is deep red, its
deciduous molars, first premolars surface is smooth. Bloody scabs that
B. Second incisors, deciduous are difficult to remove are observed
canines and first deciduous on the erosion surface. Removal of
molars
the scabs causes mild bleeding. Slight
C. First deciduous canines, second injury of the erosion in the places
premolars molars, deciduous
where there are no scabs causes no
D. First deciduous molars, deciduous bleeding. What type of lower lip
canines, first premolars precancer is it?
E. First deciduous molars, first A. Limited precancerous
premolars, deciduous canines hyperkeratosis of the vermilion
25. What is used to transfer to the border of the lip
B. Bowen's disease
articulator the data about the position
C. Lupus erythematosus
of the upper jaw in relation to the
hinge axis of the temporomandibular D. Verrucous precancer of the
vermilion border of the lip
joint?
A. Facebow E. Manganotti's abrasive
precancerous cheilitis
B. Bite blocks
C. Occluder 28. In a 3-year-old child, a dentist
D. Parallelometer has detected a carious cavity on the
E. Auxiliary impressions masticatory surface of tooth 74
26. A newborn child has been within the mantle dentin. The
overhanging edges of the enamel are
diagnosed with congenital unilateral
complete non-union of the upper lip matte white. The dentin of the carious
cavity is light-colored, soft, and can
with deformation of the cutaneo-
be removed in layers. Probing of the
cartilaginous portion of the nose. At
what age should the surgical dentinocnamel junction is tender.
Cold stimulus provokes pain that
treatment be performed in this case?
A. 6 months quickly stops after the stimulus is
removed. What material should be
B. 1.5 years
used for the filling of the carious
C. 12 months
D. 3 years cavity in this case?
A. Composite filling material
E. 10 days
B. Amalgam
C. ICON

Dentistry Telegram
2023- May-k2-Dentistry Krokology.com

D. Zinc phosphate cement made for a patient. The central


E. Glass ionomer cement occlusion was determined and fixed.
What clinical stage is next?
29. An 8-year-old child was A. Replacing wax with plastic
hospitalized into the maxillofacial B. Fixation of the finished denture in
department. Objectively, the child the oral cavity
has fever of 39 ° C and facial C. Arrangement of artificial teeth
asymmetry on the left due to a painful D. Wax try-in of the denture in the
swelling in the submandibular oral cavity
region. The skin in this region is E. Determining the edges of the
hyperemic, tense, and cannot be denture base
pinched. The sign of fluctuation is
positive. The mucogingival junction 32. A 32-year-old person was
in the area of teeth 74, 75, and 36 is hospitalized to the emergency
smoothed-out and hyperemic. Tooth hospital from the site of a car
75 is destroyed by 2/3. Make the accident. X-ray allowed diagnosing
provisional diagnosis. the patient with a displaced unilateral
A. Migratory granuloma open mandibular fracture on the left
B. Odontogenic submandibular at the level of the mental foramen.
phlegmon on the left The patient has all teeth. The fracture
C. Odontogenic osteomyelitis on the line passes between teeth 34 and 35.
left What splint should be used in this
D. Odontogenic submandibular clinical case?
lymphadenitis on the left A. Double-jaw splint with loops
E. Non-odontogenic B. Vankevich splint
adenophlegmon of the C. Port splint
submandibular space on the left D. Double-jaw splint with a spreader
bar
30. A 67-year-old man came to the E. Smooth splint-bracket
orthopedic dentistry clinic to have
dentures made for him. Examination 33. During a regular check-up of a
detects a marked atrophy of his 7-year -old girl, a dentist detected
maxillary alveolar process. What chalk-like spots on the vestibular
type of atrophic edentulous. maxilla surface of her teeth 11 and 21 in their
is observed in this patient? paracervical region. The spots have a
A. Schroeder type 2 matt surface, they are smooth when
B. Schroeder type 1 probed and can be stained with
C. Schroeder type 3 methylene blue. Choose the
D. Oxman type 1 treatment method that would be
E. Oxman type 2 optimal in this case.
A. Regular check-ups
31. A removable partial laminar B. Filling the defects
denture for the upper jaw is being C. Impregnation therapy

Dentistry Telegram
2023- May-k2-Dentistry Krokology.com

D. Deep fluoridation D. Limberg appliance


E. Filing down the defects E. Petrosov appliance

34. Three days ago a complete 36. A clasp (bugel) denture with
removable laminar denture was fixed clammer fixation is being made for a
to the upper jaw of a 57-year-old partially edentulous patient
patient. The patient. complains of a (Kennedy class 1). Impressions were
sharp pain in the mucogingival obtained, casts were made, and the
junction on the left. The pain centric relation of the jaws was
intensifies during masticatory determined and fixed. What
movements. Objectively, in the area laboratory stage of the denture-
of missing teeth 25, 26, and 27 the making is the next one?
mucosa of the mucogingival junction A. Studying the working cast in a
is hyperemic and painful when parallelometer
touched. What will be the doctor's B. Obtaining a refractory cast
tactics in this case? C. Duplication of the cast
A. Articulation correction of the D. Installation of the gating system
masticatory. surface of artificial E. Marking the denture frame
teeth
B. Make a new partial removable 37. A 67-year-old patient
laminar denture complains of being unable to use
C. Rebasing of the removable complete removable laminar
denture using an indirect method dentures for the upper and lower jaw.
D. Correction of the denture base The dentures were made 2 months
edge in the hyperemic area ago. Objectively, the patient presents
E. Rebasing of the removable with smoothed-out nasolabial folds
denture using a direct method and mental crease, teeth chattering
Officially confirmed by the test can be heard during a conversation
center. test, there is no gap between the teeth
in the state of physiological rest. At
what stage of denture-making did a
35. A 32-year-old woman mistake occur?
complains of periodic dislocations of A. Obtaining functional impressions
her lower jaw. Objectively, she has a B. Determining the central occlusion
direct occlusion, the continuity of her C. Wax try-in of the dentures
dentition is intact. When she opens D. Installing artificial teeth into the
her mouth, a clicking can be felt in dentures
the temporomandibular joint. What E. Installing the dentures
appliance should be used in this case
to prevent a habitual dislocation? 38. A 57-year-old man came to a
A. Katz appliance dentist with complaints of a dull
B. Oxman appliance aching pain in the area of his right
C. Darcissac appliance temporomandibular joint and

Dentistry Telegram
2023- May-k2-Dentistry Krokology.com

morning stiffness of the lower jaw. D. Acute pseudomembranous


His condition has a wave-like course. candidiasis.
throughout the last 3 years. E. Verrucous leukoplakia
Objectively, the face is symmetrical,
the mouth opening reduced to 3 cm. 40. A woman came to a dentist
During the movements of the lower with complaints of periodic pain in
jaw, an articular noise and clicking the area of her tooth 48. Extraction of
(step-like displacement of the lower tooth 48 is indicated. What forceps
jaw) occur in the area of the right must be used for the tooth extraction
TMJ. The skin over the joint is in this case?
unchanged. Palpation detects no A. Bayonet-like forceps
changes in the muscle tone. What is B. Beak-shaped forceps, bent along
the most likely diagnosis in this case? the plane
A. Acute arthritis of the right. C. Beak-shaped forceps, bent along
temporomandibular joint the rib
B. Neuromuscular syndrome D. S-shaped right-sided forceps
C. Fibrous ankylosis of the right E. Beak-shaped forceps with
temporomandibular joint converging beaks
D. Subluxation of the lower jaw
E. Arthrosis of the right 41. A dentist performs endodontic
temporomandibular joint treatment of tooth 46 for chronic
fibrous pulpitis. The dentist chose the
39. A 55-year-old woman came to step-back technique to scout and
a dentist with complaints of burning widen the root canals. At the first
in her oral cavity when eating irritant stage, the root canal was scouted and
foods, a sensation of roughness, and its working length was determined.
an unusual appearance of the buccal What is the next step according to
mucosa. The patient has a history of this technique?
chronic cholecystitis. Objectively, A. Exposing the apical opening
she has artificial crowns made of B. Formation of the apical stop
various metals in her oral cavity C. Smoothing the walls of the root
Against the background of canal
unchanged buccal mucosa, she has D. Formation of the middle third of
patches of small whitish papules in the root canal
the retromolar space. They are E. Formation of the upper third of the
keratinized, slightly raised, form a root canal
ring-shaped pattern, and cannot be Officially confirmed by the test
removed by scraping. Make the center.
provisional diagnosis.
A. Lupus erythematosus 42. After receiving tuberal
B. Secondary syphilis anesthesia, a 45-year-old patient
C. Lichen ruber planus suddenly developed face and neck
edema with unclear borders,

Dentistry Telegram
2023- May-k2-Dentistry Krokology.com

significant edema of the lips, skin


pallor, difficulty breathing, and 45. A 34-year-old man came to a
hoarseness of the voice. What dentist with complaints of a constant
emergency condition has developed dull pain in his upper left tooth. The
in this case? pain intensifies when cold stimuli are
A. Obstructive asphyxia applied and radiates to the ear and
B. Bronchial asthma attack temple. One year ago, the patient had
C. Epilepsy attack an intense pain in this tooth, but did
D. Angioedema not consult a dentist. The pain
E. Pulmonary edema recurred three days ago. Objectively,
tooth 27 has a deep carious cavity
43. A 32-year-old man came to a that communicates with the pulp
dentist with complaints of pain in the chamber. Probing of the exposed spot
lower jaw on the right and problems is extremely painful. X-ray of tooth
with mouth opening. Objectively, the 27 shows widening of the periodontal
face is symmetrical, mouth opening fissure in the area of its root apices.
is somewhat difficult (first degree), What is the most likely diagnosis in
the mucosa behind tooth 47 is this case?
swollen, hyperemic, and painful to A. Acute diffuse pulpitis
palpation. The mesial cusps of tooth B. Exacerbation of chronic fibrous
48 that has not fully erupted are pulpitis
visible. What diagnosis can be made C. Exacerbation of chronic
in this case? gangrenous pulpitis
A. Acute odontogenic periostitis D. Exacerbation of chronic fibrous
B. Acute odontogenic osteomyelitis periodontitis
C. Exacerbation of chronic E. Exacerbation of chronic
granulating periodontitis granulomatous periodontitis
D. Pericoronitis
E. Abscess of the pterygomandibular 46. A 42-year-old man came to a
space. dentist with complaints of bleeding
gums and bad breath. After
44. When receiving tuberal examination the following
anesthesia intraorally, a patient provisional diagnosis was made:
developed a rapidly progressing chronic generalized parodontitis, II
edema of the left check that soon degree. examination technique would
spread to the temporal region. What be most informative for establishing
complication has likely occurred in the final diagnosis in this case?
this clinical case? A. Kotzshke test
A. Ischemia B. X-ray
B. Quincke's edema. C. Depth of periodontal pockets
C. Hematoma D. Teeth mobility
D. Emphysema E. Kulazhenko test
E. Abscess

Dentistry Telegram
2023- May-k2-Dentistry Krokology.com

47. A 39-year-old woman E. Chronic fibrous periodontitis


complains of general malaise,
headache, fever of 38° C, and pain in 49. An 18-year-old girl complains
the oral cavity. Similar condition of general weakness, a fever of
occurs periodically, more often in 38.2°C, and moderately sore throat.
autumn and spring. Objectively, her Examination shows hyperemic
sharply hyperemic and swollen mucosa of the posterior pharyngeal
mucosa of the cheeks, lips, and soft wall with a cyanotic tint. The tonsils
palate has erythematous spots, burst are enlarged and partially covered in
blisters, and erosions covered with islets of white gray dense filmy
fibrinous plaque. Nikolsky's sign is deposits. The deposits are difficult to
negative. Hypersalivation is remove and expose a bleeding
observed. Submandibular lymph surface when removed. Palpation
nodes are enlarged, mobile, and detects moderately painful regional
somewhat painful to palpation. What lymph nodes 15 cm in size. Make the
is the most likely diagnosis in this provisional diagnosis.
case? A. Measles
A. Chronic recurrent herpes B. Diphtheria
B. Chronic recurrent aphthous C. Infectious mononucleosis
stomatitis D. Herpetic tonsillitis
C. Pemphigus vulgaris E. Scarlet fever
D. Lichen ruber planus, erosive-
ulcerative form 50. An 18-year-old young man
E. Erythema multiforme exudativum came to a dentist with complaints of
teeth sensitivity on his lower left jaw,
48. A 30-year-old woman came to provoked by cold stimuli.
a dentist with complaints of Examination detects a hard tissue.
discoloration of her t upper front defect within the enamel on the
tooth and food getting stuck between vestibular surface of tooth 37 in its
the teeth. Objectively, a carious paracervical region. The enamel
cavity is observed on the mesial there is matte white, not glossy, and
contact surface of tooth 12 within the fragile when probed. What diagnosis
mantle dentin. The walls and floor of can be made in this case?
the carious cavity are dense and A. Local enamel hypoplasia
pigmented. The dentinoenamel B. Acute median caries
border is tender to probing. Thermal C. Cuneiform defect
stimulus provokes no response. D. Acute superficial caries
Percussion results are negative. Make E. Acute initial caries
the diagnosis.
A. Chronic deep caries 51. A 43-year-old man developed
B. Chronic fibrous pulpitis a sharp headache, dizziness, nausea,
C. Chronic superficial caries and visual snow during tooth
D. Chronic median caries extraction. The patient is excited,

Dentistry Telegram
2023- May-k2-Dentistry Krokology.com

feels heat and trembling in the whole crowns of teeth 51 and 61 are shorter
body. Red spots appeared on the skin than the crowns of the adjacent teeth
of his neck and face. He presents with by 1/3. The mucosa in the area of
tachycardia and blood pressure of teeth 51 and 61 is hyperemic and
190/110 mm Hg. What emergency edematous X-ray shows no
condition has developed in the periodontal fissure in the apical part
patient? of the roots of teeth 51 and 61. What
A. Myocardial infarction treatment tactics would be optimal in
B. Anaphylactic shock this clinical case?
C. Stroke A. Reposition of teeth 51 and 61
D. Angina pectoris attack B. Ligature splinting
E. Hypertensive crisis C. Heplantation of teeth 51 and 61
D. Regular check-ups
52. A 5-year-old child has all the E. Removal of teeth 51 and 61
temporary teeth in the oral cavity
with tremata between them. The 55. A 28-year-old woman came to
incisors are in direct contact and their a dentist with complaints of acute
cutting edges are worn off the distal paroxysmal spontaneous pain in her
surfaces of two temporary molars upper left tooth. The pain attacks last
form a mesiosagittal step-the 5-10 minutes. Objective examination
Tsilinsky sign. What period of bite detects a carious cavity in tooth 26.
formation is it? The carious cavity has a narrow
A. Formation of the temporary bite entrance, is located within the
B. Stabilization of the temporary bite softened non-pigmented peripulpal
C. Aging of the temporary bite dentin, and does not communicate
D. Early transitional dentition with the pulp. chamber. Probing of
E. Late transitional dentition the cavity floor is painful in the
projection of the mesiobuccal pulp
53. The vestibular surfaces of the horn. The response to cold is painful
upper incisors of a 25-year-old man and prolonged. Percussion is
are affected by fluorosis that painless. Electric pulp testing i-15
manifests as spots with marble-like microamperes. What diagnosis can
appearance. What aesthetic design be made in this case?
should be proposed for this patient? A. Acute median caries
A. Dental vencers. B. Acute diffuse pulpitis
B. Porcelain crowns C. Exacerbation of chronic fibrous
C. Porcelain crowns attached to pulpitis
implants D. Acute limited pulpitis
D. Porcelain-fused-to-metal crowns E. Acute deep caries
E. Plastic crowns
56. A 34-year-old woman came to
54. A 2-year-old child has suffered a dentist with complaints of short-
a dental trauma. Objectively, the term pain caused by thermal stimuli

Dentistry Telegram
2023- May-k2-Dentistry Krokology.com

in her teeth 34 and 35. Objectively, around the lips, there are small
the visible surfaces of teeth 34 and 35 blisters with a serous content. In
are intact, the contact surfaces cannot some areas, the blisters merge
be examined due to the dense together. Maceration and weeping of
arrangement of the teeth. What the skin are observed, especially in
examination technique would be the corners of the mouth. What is the
most informative in this case, if a most likely diagnosis in this case?
carious cavity is suspected on the A. Exfoliative cheilitis.
contact surfaces of the teeth? B. Actinic cheilitis
A. X-ray C. Eczematous cheilitis
B. Percussion D. Meteorological cheilitis
C. Vital staining E. Atopic cheilitis
D. Electric pulp testing
E. Probing 59. A patient is to undergo
endodontic treatment of tooth 21 due
57. A 56-year-old man complains to exacerbation of chronic pulpitis.
of pain and swelling in the left buccal The dentist chooses tools for the
region and progressive facial treatment. What is the function of a
asymmetry. He noticed these spreader?
symptoms two months ago. A. Pulp removal from root canals
Objectively, he has paresis of mimic B. Widening of root canals.
muscles on the left and a lumpy C. Root canal length measuring
infiltrate without clear boundaries in D. Filling of root canals
the left parotid-masticatory region. E. Assessment root canal patency
The skin over the infiltrate is thinned-
out and bluish. Enlarged lymph nodes 60. A 16-year-old girl complains
can be palpated on the left. The of burning. painful, and bleeding
mouth opening is reduced to 2.5 cm. gums. Objectively, the gums of her
No saliva is secreted from the duct of upper and lower jaw are edematous,
the left parotid gland. Make the brightly hyperemic, and bleed when
provisional diagnosis. probed. Periodontal pockets are up to
A. Chronic sialadenitis 4 mm deep and contain seropurulent
B. Adenocarcinoma exudate. The teeth have pathological
C. Chronic lymphadenitis. mobility of the first degree. X-ray
D. Acute non-epidemic parotitis shows uneven resorption of the
E. Cyst of the parotid gland interalveolar septa up to 1/2 of their
height with diffuser osteoporosis of
58. A 9-year-old child complains the spongy (cancellous) bone tissue.
of painful rashes that appeared on the Make the diagnosis.
lips. Objectively, the vermilion A. Generalized parodontitis, I
border of the lips is edematous, degree, chronic course
hyperemic, and covered in cracks and B. Generalized parodontitis, III
numerous bloody scabs. On the skin degree. chronic course

Dentistry Telegram
2023- May-k2-Dentistry Krokology.com

C. C Generalized parodontitis, II- A. In front of the patient and to the


degree, chronic course left
D. Generalized parodontitis, I B. Behind the patient and to the left
degree. exacerbated course C. In front of the patient
E. Generalized parodontitis, II D. Behind the patient
degree, exacerbated course E. Behind the patient and to the right

61. A person has been hospitalized 64. A 44-year-old man has been
into the department of maxillofacial diagnosed with generalized
surgery with the following diagnosis: parodontitis, II-degree, chronic
a displaced traumatic open right- progression. What must be done first
sided mental fracture of the during the management of this
mandible. What is the typical patient?
direction of displacement of the A. Extraction of mobile teeth.
larger fragment? B. Professional oral hygiene
A. Only towards the fracture C. Elimination of traumatic
B. Upwards and outwards occlusion
C. Downwards and inwards D. Adhesive splinting of mobile
D. Upwards and inwards teeth
E. Downwards and outwards E. Curettage of periodontal pockets

62. A 58-year-old woman 65. A 33-year-old woman came to


complains of a red spot on her lower a dentist complaining of an aesthetic
lip that first appeared 4 years ago. defect in her teeth 11 and 21 and a
Objectively, on the vermilion border short-term pain that occurs when she
of the lower lip there is a red spot eats sour or sweet foods. Objectively,
with velvety surface. The mucosal the vestibular surface of teeth 11 and
lesion is slightly concave, when 21 have defects of hard tissues within
compared to the surrounding tissues. the dentinoenamel junction. The
It does not change its color when defects have gently sloping walls and
pressed. Make the provisional a dense floor and are painful to
diagnosis. probing. The skin of the face, the
A. Verrucous precancer vermilion border of the lips, and oral
B. Exfoliative cheilitis mucosa have no visible pathological
C. Bowen's disease changes. The woman additionally
D. Simple leukoplakia complains of sweating. tachycardia,
E. Hemangioma and increased appetite. She is thin
and has exophthalmos. What has
63. Extraction of tooth 46 under caused the development of this
local anesthesia is indicated for a 42- pathology?
year-old patient. Where should the A. Cariogenic microflora
dentist be in relation to the patient B. Gastrointestinal tract disease
during the extraction of this tooth? C. Endocrine system disease

Dentistry Telegram
2023- May-k2-Dentistry Krokology.com

D. Hereditary disease C. Exacerbation of chronic


E. Cardiovascular system disease periodontitis
D. Acute purulent periodontitis
66. A 23-year-old young man E. Acute purulent pulpitis
complains of acute bleeding from the
guys and bad breath that appeared 5 68. When does the upper lip
days ago. Objectively, the interdental become fully formed during the
papillae and the gingival margin are gestation of a human fetus?
bright red, edematous painful, and A. Third month
bleed intensely when palpated. B. Second month
Gingival pockets are 2-3 mm deep. X C. First month
ray shows marked osteoporosis of the D. Fifth month
interalveolar septa and widening of E. Fourth month
the periodontal fissure around the
apices of the interalveolar septa. The 69. During the treatment of
lamina dura is intact. What is the ulcerative gingivitis, a 38-year-old
most likely diagnosis in this case? man was locally prescribed a drug
A. Acute leukemia that belongs to the hydrolase
B. Acute necrotizing ulcerative enzymes and is a thermostable
gingivitis protein contained in tears, saliva, and
C. Chronic hypertrophic gingivitis breast milk as an innate resistance
D. Generalized parodontitis, II factor of human body Name this
degree, exacerbated course drug.
E. Acute catarrhal gingivitis A. Interleukin
B. Lysozyme
67. A 35-year-old man complains C. Penicillin
of constant pain in the area of his D. Interferon
upper left lateral tooth. The pain is E. E Imaninum
observed throughout the last 24 hours
and intensifies during biting on the 70. At the clinical stage of trying-
affected tooth. Objectively, a carious in the frame of the bugel (clasp)
cavity is observed on the mesial denture in the oral cavity, an uneven
surface of tooth 25. The carious gap that reaches 0.8cm was detected
cavity does not communicate with between its arch and the mucosa of
the pulp chamber. Thermal stimulus the hard palate and alveolar process.
provokes no response. Percussion of What method can be used to correct
tooth 25 is sharply painful. X-ray this defect?
detects no pathological changes in A. This defect does not require
the bone tissue of the periapical correction
region. What diagnosis can be made B. Shape the frame with a hammer
in this case? and anvil
A. Acute serous periodontitis C. Shape the frame with crampon
B. Acute diffuse pulpitis forceps

Dentistry Telegram
2023- May-k2-Dentistry Krokology.com

D. Heat the metal again and shape it stopped. Objectively, in the area of
by pressing it to the model the right check there is a wound 4x1
E. Make a new frame for the bugel cm 1 with even margins not penetrate
(clasp) denture into the oral cavity and is filled with
a blood clot. What suture should be
71. During a visit to the dentist, a applied to the wound in this case?
patient complained of feeling unwell. A. Early secondary suture
Objectively. the patient presents with B. Late secondary suture
sharp pallor of the skin, cold sweat, C. C Delayed primary suture
sunken peripheral veins, mental D. -
confusion. The pulse is 130 / min. E. Primary blind suture.
thready, arrhythmic, of poor volume.
The blood pressure is 80/40 mm Hg. 74. A 28-year-old man came to the
The respiration is rapid and shallow. maxillofacial surgery department He
Make the diagnosis. complains of a cutaneous neoplasm
A. Myocardial infarction in his right submandibular region.
B. Hypertensive crisis The neoplasm appeared 2 years ago.
C. Collapse In the process of shaving, the
D. Anaphylactic shock neoplasm is frequenily injured. The
E. Unconsciousness following provisional diagnosis has
been made: papilloma. In the
72. During preparation of the teeth neoplastic area, examination shows
for installment of an orthopedic an ulcer against the background of
appliance, a 65-year-old woman hyperemic skin. What tactics should
complained of a sharp headache, a the dental surgeon choose?
sensation of heat, nausea, numbness A. Removal of the papilloma
of the limbs, and impaired vision. B. Anti-inflammatory therapy
Objectively, asymmetric due to the followed by removal of the
smoothed-out nasolabial fold on one papilloma and
side, she has difficulty speaking. Her C. Removal of the papilloma
blood pressure is 150/90 mm Hg. postoperative radiation therapy
Make the provisional diagnosis. D. Prevention of malignant
A. Angina pectoris transformation. of the papilloma
B. Acute cerebrovascular accident followed by removal of the
C. Unconsciousness papilloma.
D. Myocardial infarction E. Anti-inflammatory therapy
E. Hypertensive crisis followed by observation
73. A 21-year-old man 3 hours ago 75. A 45-year-old man has been
received a knife wound to the right diagnosed with a non-displaced
check. The wound was initially median mandibular fracture. For the
bleeding, but by the time of the treatment of the fracture, a smooth
examination the bleeding has already splint-bracket is indicated. To what

Dentistry Telegram
2023- May-k2-Dentistry Krokology.com

group of maxillofacial appliances 78. For 2 days a man has been


does it belong? suffering from spontaneous nocturnal
A. Combined appliances pain attacks in the area of his tooth
B. Fixation appliances 25. The pain radiates along the
C. Formation appliances branches of the trigeminal nerve and
D. Substitution appliances alternates with brief painless
E. Reponation appliances intervals. Objectively, the mesial
surface of tooth 25 has a deep carious
76. A 21-year-old woman cavity within the parapulpar dentin.
complains of short-term pain attacks Probing of the cavity floor and
in her lower left provoked by thermal percussion are painful. Thermal
stimuli and quickly stop after the stimulation of tooth 25 provokes a
stimulus removed. On the pain attack. What diagnosis can be
masticatory surface of tooth 47 made in this case?
examination reveals a carious-cavity A. Acute purulent periodontitis
with a narrow entrance that is filled B. Acute diffuse pulpitis
with light softened dentin. Probing of C. Trigeminal neuralgia
the cavity floor and percussion of D. B. Acute deep caries
tooth 47 are painless. Probing of the E. Acute purulent pulpitis
cavity walls is painful. What
diagnosis can be made in this case? 79. A 28-year-old man has
A. Acute median caries received a hit to the chin. He
B. Acute deep caries complains of pain in the right lateral
C. Chronic superficial caries part of his face. Examination shows
D. Chronic median caries facial asymmetry due to tissue
E. Chronic deep caries swelling in the upper right parotid
area, mouth opening is painful and
77. A porcelain-fused-to-metal restricted, the jaw is displaced to the
dental bridge is being made for a 51- right, right-sided crossbite is
year-old woman. Vital teeth 33, 36, observed. Make the provisional
and 37 will function as the abutment diagnosis.
teeth. The teeth preparation is A. Displaced traumatic fracture of
planned to be done under anesthesia. the neck of the mandibular
What type of anesthesia would be articular process
most effective in this case? B. Displaced traumatic unilateral
A. Conduction anesthesia (tuberal fracture of the body of the
and palatal) mandible
B. Infiltration anesthesia C. Displaced traumatic double
C. Intraligamentary anesthesia fracture of the body of the
D. Conduction anesthesia (torusal) mandible.
E. Topical anesthesia D. Displaced traumatic mental
fracture of the mandible

Dentistry Telegram
2023- May-k2-Dentistry Krokology.com

E. Displaced traumatic central edematous. What diagnosis can be


fracture of the mandible made in this case?
A. Chronic recurrent herpes
80. A 27-year-old woman needs a B. Erythema multiforme exudativum
denture. Objectively, her tooth 15 is C. Chronic recurrent aphthous
missing, while teeth 14 and 16 are stomatitis
intact. What denture would be D. Primary syphilis
optimal in this case? E. Trophic ulcer
A. Partial removable laminar denture
B. Adhesive dental bridge 83. A 50-year-old man came to a
C. Porcelain-fused-to-metal dental dentist to have dentures made for
bridge. him. During preparation of teeth, the
D. Clasp (bugel) denture patient complained of a headache and
E. Swaged dental bridge dizziness, he became irritable and
anxious and suddenly fainted.
81. A 13-year-old girl complains Objectively, the pupils were dilated
of swollen gums that bleed when she and unresponsive to light, tonic
brushes her teeth. This condition has convulsions appeared and became
been observed for the last half a year. clonic, salivation increased. In a few
Objectively, her gingival mucosa in minutes, the convulsions stopped.
the area of the upper front teeth is The patient experiences mental
edematous and cyanotic Interdental confusion. What emergency
papillae are round, doughy, enlarged, condition most likely occurred in this
and cover the crowns by 1/3 of their case?
height. Teeth 13 and 23 are A. Hyperglycemic coma
positioned vestibularly Make the B. Unconsciousness
provisional diagnosis. C. Hypoglycemic coma
A. Localized parodontitis, I degree D. Epilepsy attack
B. Acute catarrhal gingivitis E. Anaphylactic shock
C. Generalized parodontitis, I degree
D. Chronic catarrhal gingivitis 84. When pressing plastic dough,
E. Chronic hypertrophic gingivitis the dental technician opened and
closed the cuvette several times to
82. A 48-year-old woman came to control the amount of the added
a dentist with complaints of a painful mass. What can develop as a result of
ulcer on her tongue. She had a similar such actions?
ulcer one year ago Objectively, on A. Porosity caused by compression
the lateral surface of the tongue there B. Cracks in plastic
is an oval erosion up to 5 mm in size, C. Residual stresses within the
covered with a fibrinous plaque, denture
surrounded by a hyperemic margin, D. Granular porosity
soft and sharply painful to palpation. E. Gas porosity
Oral mucosa in other areas is pale and

Dentistry Telegram
2023- May-k2-Dentistry Krokology.com

85. A 14-year-old boy complains mobility. Objectively, all his teeth are
of a fever of 38-39 C that has been intact, the front lower teeth have the I
observed for the last 5 days, runny degree of mobility. The man was
nose, and sore throat. Examination diagnosed with chronic generalized
detects signs of catarrhal stomatitis, parodontitis, II degree. As a part of
tonsillitis, and pharyngitis and the treatment, the doctor splinted the
enlarged lymph nodes along the front group of the patient's teeth with
sternocleidomastoid muscle that a removable splint. In this case, the
resemble a chain and are mobile and splint should reach the following
mildly painful. The palatine tonsils teeth:
are sharply hyperemic and covered in A. Second molars
a plaque that can be easily removed. B. Canines
Make the provisional diagnosis. C. Premolars
A. Diphtheria D. First molars
B. Herpetic tonsillitis E. Lateral incisors
C. Infectious mononucleosis
D. Scarlet fever 88. What concentration of
E. Acute candidal stomatitis articaine solution is used for
infraorbital anesthesia?
86. A 48-year-old man complains A. 5 %
of a tumor-like formation on his hard B. 10 %
palate. Examination detects a semi- C. 4 %
spherical protrusion with clear D. 1 %
margins in the anterior part of the E. 2 %
palate. X-ray shows. a homogeneous
focus of bone tissue. rarefaction. The 89. A 13-year-old boy undergoes
focus is 1.5x2 cm in size and has the treatment of acute deep caries of
clear margins. Against the tooth 25. At the preparation stage, the
background of bone tissue dentist accidentally perforated the
rarefaction, the roots intact teeth 11 pulp chamber in the projection of the
and 21 are projected; their pulp horn. What will be the dentist's
periodontal fissure can be traced tactics in this case?
along its whole length. What A. Apply a devitalizing paste
diagnosis can be made in this case? B. Perform vital extirpation of the
A. Giant-cell tumor of the maxilla pulp
B. Follicular cyst of the maxilla C. Apply an MTA group material
C. Radicular cyst of the maxilla D. Perform diathermocoagulation of
D. Nasopalatine duct cyst the pulp
E. Maxillary ameloblastoma E. Fill the carious cavity using a
glass ionomer cement

87. A 45-year-old man came to a 90. A 15-year-old girl complains


dentist with complaints of teeth of throbbing pain and a swelling

Dentistry Telegram
2023- May-k2-Dentistry Krokology.com

above her upper lip, fever of 38.0°C, upper left tooth. She was diagnosed
and deterioration of her general with acute diffuse pulpitis of tooth
condition. Two days ago, she tried to What treatment method would be
pop a pimple in this area on her own. optimal in this case?
Objectively, she has a dense painful A. Devital amputation
infiltration on the skin of her upper B. Vital amputation
lip. The infiltration is rounded, cone- C. Vital extirpation
shaped. up to 2.5 cm in diameter. The D. Biological method
skin over the infiltration is sharply E. Devital extirpation.
hyperemic, with a necrotic rod in the
center. Make the diagnosis. 93. A 35-year-old man complains
A. Carbuncle on the upper lip of missing teeth on his lower jaw.
B. Hematoma of the upper lip at the Objectively, teeth 36, 37 and 38 are
stage of suppuration missing. Other teeth on the lower jaw
C. Atheroma of the upper lip at the are intact. What type of dentition
stage of suppuration restoration would be optimal in this
D. Furuncle with abscess on the case?
upper lip A. Porcelain-fused-to-metal
E. Odontogenic abscess of the upper cantilever denture with abutment
lip crowns on teeth 34 and 35
B. Removable partial laminar
91. A 52-year-old man complains denture for the lower jaw
of unpleasant sensations during C. Clasp (bugel) denture with a
swallowing and erosions appearing continuous clasp (clammer) for
on his oral mucosa. The disease onset the lower jaw
was over a month ago. Objectively, D. Adhesive dental bridge
on the normal mucosa of the soft E. Porcelain-fused-to-metal crowns
palate, checks, and gums there are with abutment implants in place
bright red erosions with the remains of teeth 36 and 37
of erupted vesicles. The mucosa
easily sloughs off. The Nikolsky's 94. Dentures are being made for a
sign is positive. What medicines must 50-year-old woman with
be prescribed for the treatment first? compensated insulin dependent
A. Proteolytic enzymes diabetes mellitus. She has been in a
B. Antifungal agents clinic for several hours already,
C. Non-steroidal anti-inflammatory during which she was nervous and
drugs skipped her meals. In the process of
D. Antibiotics. obtaining her dental impressions, she
E. Corticosteroids suddenly became aggressive, paled,
broke out in cold sweat, and fell
92. A 14-year-old girl came to a unconscious. What should be used
dentist with complaints of a severe for emergency aid in this clinical
long-term pain in the area of her case?

Dentistry Telegram
2023- May-k2-Dentistry Krokology.com

A. Valocordin 97. A 38-year-old woman came to


B. Nitroglycerine a dentist to have a denture made for
C. Insulin her. It is planned to make a porcelain-
D. Ammonia solution fused-to-metal crown for her tooth
E. Glucose solution 24. To what thickness should its
occlusal surface be filed down?
95. A 49-year-old patient A. 0.25-0.3 mm
complains of teeth mobility on the B. 1.5-2 mm
right lower jaw and purulent C. 1-14 mm
discharge from the socket of tooth 46 D. 0.5-0.9 mm
that was removed 1.5 months ago E. 2.1-2.5 mm
Objectively, there is slight swelling
of soft tissues in the submandibular 98. The parents of an 8-year-old
area on the right, right submandibular girl have brought their child to a
lymph nodes are enlarged and mildly dentist for a preventive examination.
painful to palpation. The alveolar After the examination, the dentist
process of the lower jaw is thickened determined that the decay-missing-
on the right. Teeth 45 and 47 have the filled index in this child is df + DFM
first-degree mobility. A fistula with = 12. What concentration of fluoride
protruding granulations was detected ions in toothpaste should be
on the mucogingival junction in this recommended for personal hygiene
area. Granulations fill the socket of of the children with high levels of
the extracted tooth 46. What caries intensity?
diagnosis can be made in this case? A. 1000 ppm
A. Acute alveolitis B. 900 ppm
B. Chronic osteomyelitis C. 500 ppm
C. Malignant tumor D. 1100 ppm
D. Acute osteomyelitis E. 1500 ppm
E. Eosinophilic granuloma
99. A 14-year-old patient was
96. A 45-year-old man came to a referred. to a dental orthodontist for
dentist for tooth extraction. The consultation. Objectively, the upper
dentist chose S shaped left-sided canines have erupted on the
forceps for this purpose. What teeth vestibular side, the upper and lower
are most often extracted with this dentition is narrowed. What auxiliary
type of forceps ? method will allow diagnosing the
A. 24, 25 narrowing of the upper dentition?
B. 14, 15 A. Gerlach
C. 36, 37 B. Nance
D. 26, 27 C. Tonn
E. 16, 17 D. Pont
E. X-ray

Dentistry Telegram
2023- May-k2-Dentistry Krokology.com

100. A 40-year-old woman observed on the surface: of individual


complains of pain in her right papules. Submandibular and cervical
temporomandibular joint. lymph nodes are moderately
Comprehensive examination detects enlarged, painless, mobile. Make the
premature contacts between her provisional diagnosis.
teeth. Selective filing down of her A. Pemphigus vulgaris
teeth is planned. What method must B. Sutton's stomatitis
be used to control this procedure? C. Erythema multiforme exudativum
A. Electric pulp testing. D. Lichen ruber planus
B. Parallelometry E. Secondary syphilis
C. Computed tomography
D. Orthopantomography 103. The parents of an 11-year-old
E. Occlusiography boy came to a dentist with complaints
that their child's gums are painful and
101. A 13-year-old girl complains bleeding during eating, the child has
of a sharp pain when biting on her bad breath and a fever of 37.4°C. The
tooth 21. The pain. developed after an boy has a recent history of an acute
injury. Objectively, the crown of respiratory viral infection.
tooth 21 has two fracture lines, the Objectively, the gingival mucosa is
fragments are slightly displaced, brightly hyperemic and bleeds easily
blood is oozing from the pulp when touched. The apices of the
chamber, percussion is painful. X-ray interdental papillae are covered with
shows a diagonal fracture of the a grayish necrotic plaque. The
crown and root of tooth 21. The depth removal of the plaque exposes an
of the subgingival edge of the ulcerated. surface. There is a large
fracture line is over 4 mm. What amount of dental plaque. What is the
should be the dentist's tactics causative agent of this disease?
regarding tooth 21? A. Staphylococci
A. Extraction B. Streptococci
B. Splinting C. Anaerobic microfloral
C. Filling D. Herpes virus
D. Replantation E. Yeast-like fungi
E. Monitoring for the next month
104. A 32-year-old man came to a
102. During preventive dentist with complaints of skin
examination by a dentist, a 27-year- redness and a swelling in the area of
old woman presents. with a painless his left check. Two months ago, he
rash on the mucosa of her checks, noticed a dense spot on his left cheek
hard palate, and lower lip. The rash that increased in size over time. The
manifests as isolated round copper- skin over the spot has become bluish
red papules 1-1.5 cm in diameter, red and a softening has formed in its
with a narrow border of hyperemia. center. Objectively, a pathological
Erosions or whitish coating can be focus 2.3x1.4 cm in size with clear

Dentistry Telegram
2023- May-k2-Dentistry Krokology.com

borders is observed on the skin of the A. Tooth opening with prescription


left cheek. The skin there is sharply of lavages
thinned-out, bluish, shiny, and forms B. Tooth extraction
folds and irregularities. Intraorally a C. Endodontic treatment with
palpable cord is detected that leads application of temporary
from tooth 46 to the lesion on the therapeutic dressing
check. Tooth 46 is filled. Make the D. Application of a devitalizing paste
provisional diagnosis. E. Endodontic treatment with
A. Furuncle permanent obturation of root
B. Carbuncle canals
C. Chronic lymphadenitis
D. Phlegmon of the buccal region 107. A 44-year-old man came to a
E. Migratory subcutaneous dental surgeon for installation of
granuloma dental implants. It is planned to carry
out a two-stage implantation in the
105. An 8-year-old child has a area of missing teeth 15. 16, and 17
neutral ratio of first permanent What is the minimum period for
molars, a vertical gap 4 mm wide complete osseointegration of the
between the front teeth, and a bad implants into the upper jaw?
habit of sucking the tongue. What A. 12 months
orthodontic appliance should be used B. 6 months
for correction of this condition? C. 9 months
A. Removable orthodontic appliance D. 3 months
for the upper jaw with an inclined E. 1 month
plane and a vestibular arch
B. Frankel functional regulator, type 108. Examination of a 12-year-old
2 girl bu a dentist revealed a carious
C. Bruckl appliance cavity on the mesial contact surface
D. Frankel functional regulator, type of tooth 46. The cavity is located
3 within the mantle dentin, its walls
E. Removable orthodontic appliance and floor are pigmented and dense.
for the upper jaw with occlusal Thermal stimuli provoke no
overlays and Rudolph loops response. Preparation of tooth 46 is
painful in the area of the
106. A 9-year-old girl complains of dentinoenamel junction. The decay-
a sharp throbbing pain in her lower missing-filled index is DFM + df = 2.
tooth. The pain intensifies during What filling material would be
biting on this tooth. She was optimal in this case?
diagnosed with acute purulent. A. Silicophosphate cement
periodontitis of tooth 75. What B. Zinc phosphate cement
treatment tactics would be optimal in C. Amalgam
this case? D. Glass-ionomer cement
E. Photopolymer composite

Dentistry Telegram
2023- May-k2-Dentistry Krokology.com

109. A 75-year-old man complains 112. A 26-year-old man came to a


of poor fixation of his full removable dentist complaining of facial
denture for the lower jaw and food asymmetry that developed because of
particles accumulating under the a painful swelling in the left parotid
denture base. The lower third of his area that appeared 3 days ago. His
face is shortened. The alveolar body temperature is 37.2°C. Two
processes are markedly atrophied. weeks ago he had a case of acute
What anatomical structure of the respiratory disease. Objectively, in
lower jaw can be used for the left parotid area there is a dense
improvement of full removable round infiltration up to 2 cm in size
denture fixation? that is slightly mobile and moderately
A. Internal oblique line painful. The excretory duct of the
B. Mylohyoid ridge parotid gland produces clear saliva.
C. Retroalveolar space Make the provisional diagnosis.
D. Vestibule of the oral cavity A. Herzenberg pseudoparotitis
E. Mucogingival fold B. Acute purulent parotitis
C. Pleomorphic adenoma of the
110. Tooth 46 is being parotid gland
endodontically treated for chronic D. Mikulicz disease
fibrous periodontitis. Its root canals E. Exacerbation of chronic parotitis
are narrow and sclerosed. What tool
should be used to widen the root 113. A 14-year-old girl came to a
canals in this case? dentist. complaining of a growth on
A. Chlorhexidine bigluconate her tongue. It appeared
B. Sodium hypochlorite approximately 1.5 years ago and has
C. Orthophosphoric acid been slowly growing since then.
D. Ethylenediaminetetraacetic acid Objectively, on the left lateral surface
E. Aminocaproic acid of her tongue there is a pink
formation. This formation is mobile,
111. A 25-year-old woman came to painless, dense, and spherical. It has
a dental surgeon for a planned tooth a clear margin, wide base, and
extraction. Objectively, the crown of smooth surface. Make the provisional
her tooth 37 is destroyed by 2/3. The diagnosis.
gingival mucosa in the area of tooth A. Papilloma
37 is without changes. What type of B. Atheroma
anesthesia should the doctor use to C. Hemangioma
remove tooth 37? D. Fibroma
A. Mental E. Lymphangioma
B. Tuberal and palatal
C. Intraoral infraorbital 114. A 51-year-old man complains
D. Infiltration of mobility of his porcelain-fused-to-
E. Mandibular and buccal metal dental bridge with 43 and 47 as

Dentistry Telegram
2023- May-k2-Dentistry Krokology.com

abutment teeth. He has been using


this denture for 9 months. X-ray 116. After a clinical examination of
shows atrophy of the alveolar process a 28-year-old patient, a dentist made
in the area of tooth 43 to 1/2 of its the following diagnosis: acute
root length and in the area of tooth 47 median caries of tooth 25, Black's
to 2/3 of its root length. Examination. class 1. What dental should be used
of the oral cavity detects mobility of in this case to open the carious cavity
the abutment teeth and periodontal during the treatment?
pockets in this area, as well as A. Spherical, hard-alloy
symptomatic gingivitis. What is the B. Spherical, diamond
cause of the pathological mobility of C. Cylindrical, hard-alloy
the abutment teeth? D. Wheel-shaped, diamond
A. Functional overload of the E. Inverted cone, diamond.
abutment teeth
B. Injury of the circular tooth 117. A 52-year-old man came to a
ligament by the edges of the dentist with complaints of mobility of
crowns his lower front teeth. Objectively, his
C. Modeling of a lingual bar in the lower incisors are intact and have the
paracervical area first-degree mobility, the canines and
D. Depulpation of the abutment teeth lateral teeth are stable.
E. Massive filing down of hard Orthopantomography shows
tissues of the abutment teeth resorption of the interalveolar septa
in the area of the incisors to 1/4 of
115. A 28-year-old woman came to their height. Between the front teeth,
a dentist with complaints of a the patient has a diastema and tremata
periodically appearing fistula on her up to 1 mm in width. Temporary
gums in the area of tooth 111 splinting is planned. What splint is
projection and unpleasant sensations indicated in this case?
in this tooth. Previously, this tooth A. Splint made of semi-crowns.
was treated for chronic periodontitis. B. Fiberglass splint
The root canal of this tooth was C. Intradental splint
treated twice, with no positive result. D. Splint made of plastic crowns
Target X-ray of tooth 11 shows the E. Cap splint
root canal that is scaled along its
entire length and a focus of X-ray 118. A patient with a gunshot
lucency up to 0.8 cm in size in the wound to the face and laceration of
area of its apex. Determine the the palatal and pharyngeal tissues
treatment method in this case. develops increasing respiratory
A. Repeated endodontic treatment failure. The flap that hangs from the
B. Replantation wound sometimes partially and
C. Monitoring throughout a year sometimes completely closes the
D. Resection of the root apex entrance. to the larynx during
E. Extraction

Dentistry Telegram
2023- May-k2-Dentistry Krokology.com

breathing. What type of asphyxia can 121. A 30-year-old woman


be expected to develop in this case? complains of problems with mimics
A. Stenotic asphyxia on the right side of her face and a
B. Dislocation asphyxia fever of 37.9°C. The signs developed
C. Obstructive asphyxia after an overexposure to cold.
D. Aspiration asphyxia Objectively, the patient cannot
E. Valve asphyxia wrinkle her forehead on the right or
close her right eye, supraorbital
119. A porcelain-fused-to-metal reflex on the right is absent, the right
crown for tooth 11 is being made for nasolabial fold is smoothed-out.
the patient. The tooth will be left What pathology is it?
vital. What measures. should be A. Neuritis of the facial nerve
taken during the treatment to prevent B. Trigeminal neuralgia
development of pulpitis in this tooth? C. Progressive facial hemiatrophy
A. Prescribe calcium-containing D. -
preparations E. Hemifacial spasm
B. Make a temporary crown
C. Physical therapy 122. During the fitting of the frame
D. Prescribe anti-inflammatory of a porcelain-fused-to-metal dental
preparations bridge, the frame is in contact with
E. Prescribe fluorine-containing the antagonist teeth in the central
preparations occlusion, reaches the ledges on the
abutment teeth, and is 0.3 mm thick.
120. A 52-year-old man complains What would be the dentist's tactics in
of pain and clicking in the area of his this case?
left temporomandibular joint. A. Obtain an impression with the
Objectively, the face is symmetrical, frame installed
palpation of the medial and lateral B. Finish preparation of the
pterygoid muscles is painful on the abutment teeth, obtain the
left. The mouth opening is limited to working impression
1.5 cm between the cutting edges of C. Pass on the frame to the next
the central incisors. X-ray shows laboratory stage
smooth contours of the articular D. File down the metal frame in the
surfaces, the joint space is normal. areas of contact with antagonist
What is the most likely diagnosis in teeth
this case? E. Identify the areas that cause
A. Fibrous ankylosis of the joint problems during the installation
B. Acute post-traumatic arthritis of the dental bridge
C. Deforming arthrosis
D. Rheumatoid arthritis 123. The parents of a 6-year-old girl
E. Neuromuscular joint syndrome complain of a sharp increase in the
child's body temperature,
deterioration of her general

Dentistry Telegram
2023- May-k2-Dentistry Krokology.com

condition, disturbed sleep, pain and of her tooth 38. Her term of
edema in the right parotid region. pregnancy is 22 weeks. She has no
Objectively, mucosa around the history. of allergies. Objectively, the
opening of the excretory duct of the crown of tooth 38 is destroyed by 2/3,
right parotid salivary gland is percussion is sharply painful. Tooth
hyperemic, the girl feels pain during extraction is indicated. What
palpation on the right side in front of anesthetic is the drug of choice for
the tragus and in the area of the gonial pregnant patients?
angle. Massage of the salivary gland A. Bupivacaine
produces a very small amount of B. Mepivacaine
viscous transparent saliva from the C. Novocaine (Procaine)
duct. Make the provisional diagnosis. D. Lidocaine
A. Acute bacterial parotitis E. Articaine.
B. Epidemic parotitis
C. Herzenberg's pseudoparotitis 126. A 28-year-old man came to a
D. Calculous sialadenitis dentist with complaints of pain in his
E. Chronic parenchymal sialadenitis lower left tooth. The pain is provoked
by sweet foods and stops
124. A 45-year-old woman needs immediately after the stimulus is
dentures. for her missing teeth 24 and removed. Objectively, the distal
25. X-ray shows a small patch of surface of tooth 34 has a deep carious
bone tissue destruction, with clear cavity filled with soft pigmented
contours in the apical area of the root dentin. Probing the of the cavity floor
of tooth 23. The root canal of tooth 23 is painful. No communication can be
is sealed with a non-homogeneous detected between the cavity and the
filling material to the 2/3 of its length. pulp chamber. Thermal stimulation
There is a permanent composite results are positive, short-term. What
filling without defects. Percussion of diagnosis can be made in this case?
the tooth is painless, the alveolar A. Chronic deep caries
mucosa in this area is of normal B. Acute deep caries
color. What would be the dentist's C. Chronic gangrenous pulpitis
tactics regarding tooth 23? D. Acute limited pulpitis
A. Conduct a course of E. Chronic fibrous periodontitis
physiotherapeutic treatment
B. Perform a resection of the root 127. A 50-year-old man complains
apex of a formation 4 cm in size in the
C. Repeat endodontic treatment submental. region. On palpation, the
D. Extract the tooth neoplasm is homogeneous, soft,
E. Repeat X-ray in 6 months elastic, and doughy. It is not fused
with the skin or surrounding tissues.
125. A 24-year-old pregnant Its slow growth has been observed for
woman came to a dentist with 3 years already What disease can be
complaints of acute pain in the area characterized by such clinical signs?

Dentistry Telegram
2023- May-k2-Dentistry Krokology.com

A. Migrating granuloma Officially confirmed by the test


B. Fibroma center.
C. Median cyst
D. Fibromyoma 130. At a clinic of orthopedic
E. Lipoma dentistry, at the stage of try-in of the
metal frame of a porcelain-fused-to-
128. A 50-year-old woman metal dental bridge. a dentist uses a
suddenly complained of shortness of micrometer to determine the
breath and a feeling of stiffness in her thickness of the metal cap made of
chest during installation of dentures. cobalt-chromium alloy What should
The patient has cough and wheezing be the minimum thickness of the
respiration with a long exhale. It is cobalt-chromium metal cap to make
known from her history that such an abutment porcelain fused-to-metal
symptoms occur after a contact with crown?
allergens or airborne irritants. What A. A.0.4
emergency condition did the patient B. 0,5
develop? C. 0,3
A. Bronchial asthma attack D. 0,6
B. Acute respiratory failure. E. 0,1
C. Exacerbation of chronic
obstructive pulmonary disease 131. A dentist prepares to receive a
D. Acute heart failure patient. The dental unit has no
E. Allergic rhinitis modern equipment for handpiece
disinfection. The design of the
129. A man with hemophilia came handpiece does not allow for its
to a dentist with complaints of pain in autoclaving. Specify the algorithm
his lower left tooth. A root fracture for the handpiece disinfection in this
was diagnosed in tooth 36. Tooth case.
extraction is planned. What will be A. One treatment with 70 % alcohol
the dentist's tactics in this case? or Bacillol AF
A. Perform the tooth extraction in the B. Two treatments with 70 % alcohol
maxillofacial surgery department or Bacillol AF with an interval of
B. Refer to the hematology 15 minutes between them
department for the tooth C. Two treatments with 6 %
extraction hydrogen peroxide and 70 %
C. Perform the tooth extraction in the alcohol with an interval of 15
vascular surgery department minutes between them
D. Perform the tooth extraction in a D. Two treatments with 70 % alcohol
dental polyclinic with pre- and or Bacillol AF with an interval of
post-operative care 5 minutes between them
E. Perform the tooth extraction in a E. Two treatments with 6 %
dental polyclinic with pre- hydrogen peroxide with an
operative care

Dentistry Telegram
2023- May-k2-Dentistry Krokology.com

interval of 15 minutes between induration of a part of the duct.


them Ultrasound reveals a hyperechogenic
zone with an " absorption path " What
132. A 50-year-old man came to a disease can be characterized by such
dentist with complaints of the clinical presentation?
mobility of his lower teeth. A. Interstitial parotitis
Objectively, the dentition is intact. B. Sialodochitis
the necks of the teeth are exposed, the C. Calculous submaxillitis
crowns are tall, the teeth have D. Sclerosing submaxillitis
mobility of the first degree. What E. Parenchymatous parotitis
splint would be optimal in this
clinical case? 135. A 7-year-old child is sick for 3
A. Mamlok splint days. already. The child has fever of
B. Splint made of equator crowns 38°C. weakness, and facial
C. Elbrecht splint asymmetry due to edema of the upper
D. Splint made of full crowns lip and left infraorbital area. The skin
E. Cap splint color is unchanged. The
mucogingival junction in the
133. A 15-year-old girl has been projection of the roots of teeth 63 and
provisionally diagnosed with chronic 64 is smoothed-out and hyperemic.
parenchymal parotitis of the right Palpation detects a positive sign of
parotid salivary gland. What changes fluctuation. Tooth 63 is partially
will be observed in the sialogram in destroyed, mobile, its percussion is
this case? slightly painful. Make the provisional
A. Widening of the excretory duct diagnosis.
B. Duct interruption followed by its A. Acute purulent odontogenic
widening periostitis of the upper jaw on the
C. Filling defect left
D. Uniform narrowing of excretory B. Phlegmon of the infraorbital
ducts of the gland space on the left
E. Round cavities in place of ducts of C. Acute purulent odontogenic
the third and fourth order lymphadenitis
D. Acute serous periodontitis of
134. A 12-year-old girl complains tooth 63
of a painful neoplasm under her E. Acute odontogenic osteomyelitis
tongue on the left. It appears during of the upper jaw on the left
eating (especially salty or sour foods)
and then slowly diminishes. Mouth 136. A 40-year-old man came to a
opening is unrestricted. The duct dentist with complaints of a
orifice of the left sublingual salivary neoplasm on the mucosa of the lower
gland is dilated and hyperemic Saliva lip. The neoplasm appeared
is clear and contains pus admixtures. approximately one month ago and is
Bimanual palpation detects an slowly increasing in size.

Dentistry Telegram
2023- May-k2-Dentistry Krokology.com

Objectively, under the lower lip technique should be chosen for this
mucosa there is a formation 0.7 cm in patient?
size. The formation is circumscribed, A. Perform bilateral osteosynthesis
elastic, painless, round, and of the lower jaw
translucent. The mucosa over the B. Apply a smooth splint-bracket
formation is thinned-out, while C. Make a Port splint for the fixation
surrounding mucosal remains of the lower jaw
unchanged. What diagnosis can be D. Make a Vankevich splint for the
made in this case? fixation of the lower jaw
A. Abscess of a minor salivary gland E. ply a splint with loops for the
B. Papilloma of the lower lip lower jaw
C. Cyst of a minor salivary gland
D. Fibroma of the lower lip 139. A dentist performs a
E. Lipoma of the lower lip preventive examination of a 12-year-
old girl. Objectively, her face is pale,
137. A porcelain-fused-to-metal the vermilion border of her lips is dry
crown is being made for tooth 24 of a and covered in small scales, she has
32-year-old patient. At the stage of cracks in the corners of her mouth.
fitting the crown, its placement on the Her oral mucosa is pale and
tooth stump turned out to be difficult edematous, while the tongue is bright
due to an excess of porcelain and red and smooth, with atrophied
metal mass on the surfaces that are in filiform papillae. What general
contact with the adjacent teeth. What somatic disorder is typically
can be used to detect the areas with accompanied by such signs?
an excess of the porcelain mass? A. Acute leukemia
A. – B. Werlhof disease
B. Chemical pencil C. Hemophilia
C. Basic wax D. Von Willebrand disease
D. Corrective impression material E. Iron deficiency anemia
E. Copy paper
Officially confirmed by the test 140. A 16-year-old girl came to a
center. dentist complaining of darkened
crowns of her teeth 11 and 12.
138. A 32-year-old man has been According to the girl, approximately
hospitalized maxillofacial one year ago she had a sports trauma.
department of a clinic. The following Objectively, the crowns of teeth 11
diagnosis has been made: an open and 12 are dark gray, intact. painless
displaced bilateral fracture of the to percussion. The mucosa of the
lower jaw in the area of the gonial alveolar process is unchanged. X-ray
angle. X-ray shows a large diastasis shows round areas of bone tissue
between the fragments and muscle destruction in the region of the root
interposition. What main treatment apices of teeth 11 and 12. These bone
lesions are 0.3-0.4 cm in diameter

Dentistry Telegram
2023- May-k2-Dentistry Krokology.com

and have clear margins. What tactics C. Mobility of the causative tooth
should the doctor choose in this case? and its adjacent teeth
A. Extraction of teeth 11 and 12 D. Presence of an infiltrate on the
B. Conservative and surgical both sides of the alveolar process
treatment of teeth 11 and 12 E. Painful percussion of the
C. Aesthetic restoration of teeth 11 causative and its adjacent teeth
and 21
D. Conservative treatment of teeth 143. In a 16-year-old girl, a dentist
11 and 12 detected patches of white enamel that
E. Monitoring the condition of teeth has lost its natural luster in the
11 and 12 for six months paracervical regions of her teeth 11
and 21. The enamel surface is smooth
141. A 44-year-old man complains and dense to probing. Thermal
of indisposition, fever of 38°C, and a stimulation results are negative. The
swelling under the lower jaw on the lesions can be stained with a 2 %.
left. The patient's history states that aqueous solution of methylene blue.
throughout the last week he felt pain In this case, the infiltration technique
in the area of his tooth 36. was chosen for the treatment of initial
Objectively, in the left caries. What material is used in this
submandibular area there is an technique?
enlarged painful nodule. The skin A. ICON
above the nodule is hyperemic, B. Calcium hydroxide-containing
strained, and cannot be pinched. paste
Tooth 36 is partially destroyed in the C. Glass ionomer cement
oral cavity. What diagnosis can be D. Silver nitrate solution
made in this case? E. Dental compomer
A. Abscess of the submandibular
region 144. A 30-year-old woman came to
B. Acute purulent lymphadenitis a dentist with complaints of an
C. Acute scrous lymphadenitis unpleasant bursting sensation in her
D. Chronic lymphadenitis upper right tooth. Heat makes this
E. Furuncle sensation worse. Objectively, the
masticatory surface of tooth 17 has a
142. A 37-year-old patient has been deep carious cavity that
diagnosed with acute osteomyelitis communicates with the pulp
of the mandible on the right in the chamber. Docp probing is painful.
area of teeth 45, 46, and 47 Percussion of tooth 17 is mildly
Examination detects Vincent's sign. painful. X ray shows slight widening
Name the characteristics of this sign. of the periodontal fissure at the root
A. Intense pain in the area of the apices Electric pulp testing-70
causative tooth microamperes. What diagnosis can
B. Numbness of the soft tissues the be made in this case?
lower lip and chin A. Chronic gangrenoux pulpitis

Dentistry Telegram
2023- May-k2-Dentistry Krokology.com

B. Acute purulent pulpitis E. Frankel functional regulator, type


C. Chronic fibrous pulpitis 3
D. Chronic fibrous periodontitis
E. Exacerbation of chronic 147. A 43-year-old man complains
granulating periodontitis of a sharp pulsing pain in the area of
his right upper jaw. The pain
145. A 54-year-old woman appeared three days ago. Objectively,
complains of a neoplasm on her an inflamed round infiltration can be
lower lip that appeared 6 months ago. palpated on the vestibular surface of
During the last 20 days, it started the gingival mucosa in the area of
sharply increasing in size. tooth 16, closer to the gingival
Objectively, on the vermilion border margin. The tooth is intact, with the
of the lower lip there is a round gray- II-degree mobility Its horizontal and
red node that protrudes by 0.5 cm vertical percussion is painful. The
above the underlying tissues, is periodontal pocket is 4-5 mm deep.
demarcated, and has a funnel-shaped What diagnosis can be made in this
indent in its center, filled with case?
keratinized masses. The node is A. Periodontal abscess
dense, mobile, and painless to B. Acute serous periodontitis
palpation. What is the most likely C. Exacerbation of chronic
diagnosis in this case? periodontitis
A. Keratoacanthoma D. Maxillary periostitis
B. Verrucous leukoplakia E. Acute suppurative periodontitis
C. Manganotti's chcilitis
D. Verrucous precancer 148. A 38-year-old man has been
E. Papilloma diagnosed with chronic generalized
parodontitis, initial stage.
146. The parents of an 8-year-old Professional oral hygiene was
girl came. to a dental orthodontist performed. What oral care product
complaining of an aesthetic defect in should be recommended to this
their child. Objectively, the lower patient for removal of food debris and
part of the face is shortened, the chin massage of the gums?
is pushed forward, the upper lip A. Toothpicks
sinks. When teeth are closed, reverse B. Irrigator
deep incisal overlap is revealed. C. Hygienic toothbrush
Mesioocclusion is observed laterally. D. Interdental stimulator.
Select the appliance for the treatment. E. Dental floss
A. Frankel functional regulator, type
2 149. A 30-year-old woman
B. Andresen-Haupl activator complains of a painless swelling in
C. Frankel functional regulator, type the area of her right lower jaw. The
1 swelling appeared 6 months ago and
D. Osadchy appliance is slowly increasing in size.

Dentistry Telegram
2023- May-k2-Dentistry Krokology.com

Objectively, on the body of the lower


jaw on the right in the area of teeth 45
and 46. there is a dense neoplasm
2.5x1.5 cm in size, painless to
palpation. The skin and mucosa over
the neoplasm are mobile and without
discoloration. The oral cavity is
sanated. The teeth on the lower jaw
are intact. The sensitivity of the lower
lip is unchanged. Regional lymph
nodes are not palpable. X ray detects
a polycystic formation with clear
boundaries in the area of the body of
the mandible on the right. Make the
provisional diagnosis.
A. Ameloblastoma
B. Odontoma
C. Odontogenic fibroma
D. Follicular cyst
E. Osteosarcoma

150. A 52-year-old woman


complains of cosmetic defects in her
teeth that she noticed several years
ago. Objectively, on the vestibular
surfaces in the paracervical areas of
her premolars and incisors, there are
fissure-like defects within the mantle
dentine, formed by smooth glossy
surfaces. The necks of the teeth are
exposed. The gums are firm and pale
pink. The defects are painless when
probed. Make the diagnosis.
A. Cuneiform defect
B. Chronic median caries
C. Pathological wear of teeth
D. Necrosis of dental hard tissues
E. Erosion of dental hard tissues

Dentistry Telegram

You might also like